rapid review of clinical medicine for mrcp part 2

433

Upload: jewel-billah

Post on 01-Nov-2014

426 views

Category:

Health & Medicine


17 download

DESCRIPTION

SONJOY SHARMA

TRANSCRIPT

Page 1: Rapid review of clinical medicine for mrcp part 2
Page 2: Rapid review of clinical medicine for mrcp part 2

Rapid Review of

ClinicalMedicinefor MRCP Part 2

Second Edition

Sanjay SharmaBSc (Hons) MD FRCP (UK) FESC

Professor of Clinical CardiologyConsultant Cardiologist and Physician

St George’s University of LondonSt George’s Hospital NHS TrustUniversity Hospital Lewisham

London, UK

Rashmi KaushalBSc (Hons) FRCP (UK)

Consultant Physician and EndocrinologistWest Middlesex Hospital

Kingston, UK

MANSONPUBLISHING

Page 3: Rapid review of clinical medicine for mrcp part 2

Fourth impression 2010Third impression 2009Second impression 2007

Copyright © 2006 Manson Publishing Ltd

ISBN: 978-1-84076-070-5

All rights reserved. No part of this publication may be reproduced, stored in a retrievalsystem, or transmitted in any form or by any means without the written permission ofthe copyright holder or in accordance with the provisions of the Copyright Act 1956(as amended), or under the terms of any licence permitting limited copying issued bythe Copyright Licensing Agency, 33–34 Alfred Place, London WC1E 7DP.

Any person who does any unauthorized act in relation to this publication may be liableto criminal prosecution and civil claims for damages.

A CIP catalogue record for this book is available from the British Library.

For full details of all Manson Publishing titles, please write to:Manson Publishing Ltd73 Corringham RoadLondon NW11 7DL, UKTel: +44 (0)20 8905 5150Fax: +44 (0)20 8201 9233Website: www.mansonpublishing.com

Printed in Spain

DedicationFor Ravi, Ashna, Anushka, Ishan, Shivani and Milan

Dr L Wilkinson, Ms S Gowrinath, Ms H Derry, Mr P Radomskij, Dr J Waktare, Ms A O’Donoghue, Dr S Rosen,Dr A Mehta, Dr L Shapiro, Professor M E Hodson, Dr G Rai, Dr A Ghuran, Professor C Oakley, Ms F Goulder,Dr J Axford, Dr S Jain, Dr M Stodell, Dr B Harold, Dr D Seigler, Dr C Travill, Dr G Barrison, Dr D Hackett,Dr J Bayliss, Dr R Lancaster, Dr R Foale, Dr W Davies, Professor D Sheridan, Professor W McKenna,Professor G MacGregor, Dr A Belli, Dr Adams, Dr J Joseph, Dr M Impallomeni, Dr D Banerjee, Dr N Essex,Dr S Nussey, Dr S Hyer, Dr A Rodin, Dr M Prentice, Dr N Mir, Mrs K Patel and Dr J Jacomb-Hood.

We are also grateful for the assistance of the Audiovisual Departments at Luton and Dunstable Hospital, St Mary’s(Paddington) Hospital and St George’s Hospital Medical School and the ECG, Echocardiography and RadiologyDepartment at St George’s Hospital Medical School and University Hospital Lewisham.

AcknowledgementsWe are grateful for the help of several colleagues who helped provide slides for the book:

Page 4: Rapid review of clinical medicine for mrcp part 2

ContentsAcknowledgements 2Preface 3Classification of Cases 4Abbreviations 5Clinical Cases 7Data Interpretations Tutorials 415

Calcium Biochemistry 415Genetics 415Audiograms 416Guidelines for the Interpretation of Cardiac Catheter Data 418Respiratory Function Tests 419Interpretation of Respiratory Flow Loop Curves 420Echocardiography 421Acid–base Disturbance 426

Normal Ranges 427Index 429

3

Passing specialist examinations in internal medicine is adifficult milestone for many doctors, but is a mandatoryrequirement for career progression. Pass rates in theseexaminations are generally low due to ‘high standards’and ‘stiff competition’. Thorough preparation is essentialand requires a broad knowledge of internal medicine.The pressures of a busy clinical job and nights ‘on call’make it almost impossible for doctors to wade throughheaps of large text books to acquire all the knowledgethat is required to pass the examinations.

The aim of this book is to provide the busy doctor witha comprehensive review of questions featured mostfrequently in the MRCP (II) examination in internalmedicine. The MRCP (II) examination has a best of 5/nfrom many answer format. The vast majority of thequestions in the book follow the same pattern; however,we have chosen to include several scenarios with openended questions to stimulate the medical thought process.The level of difficulty of each question is of the samestandard as MRCP (II) examination. However, some casesare deliberately more difficult for teaching purposes.

A broad range of subjects is covered in over 400questions ranging from metabolic medicine to infectiousdiseases. Precise answers and detailed discussion followeach question. Where appropriate, important differential

diagnoses, diagnostic algorithms and up-to-date medicallists are presented. Many questions comprise illustratedmaterial in the form of radiographic material, electro -cardiograms, echocardiograms, blood films, audiograms,respiratory flow loops, histological material, and slides inophthalmology, dermatology and infectious diseases.Over 200 commonly examined illustrations are included.

Tutorials are included at the end of the book to aidthe interpretation of illustrated material as well as impor -tant, and sometimes difficult, clinical data, such as respir -atory function tests, cardiac catheter data and dynamicendocrine tests.

The book will prove invaluable to all those studyingfor higher examinations in internal medicine, and to theirinstructors.

Sanjay Sharma Professor of Clinical Cardiology

Consultant Cardiologist and Physician Lecturer for Medibyte Intensive Courses

for the MRCP Part 2

Rashmi KaushalConsultant Physician and Endocrinologist

Preface

Page 5: Rapid review of clinical medicine for mrcp part 2

Cardiology1, 10, 11, 13, 22, 25, 32, 40, 52, 53, 54, 62, 63, 66, 68,74, 78, 80, 94, 95, 100, 121, 123, 125, 130–132, 138,144, 150, 160, 167, 178, 180, 184, 193, 197, 199, 202,203, 207, 208, 223, 226, 229, 232, 235, 237, 243, 246,259, 266, 270, 285, 287, 291, 296, 301, 305, 307, 309,318, 323, 324, 327, 331, 332, 335, 342, 350, 353, 362,368, 377, 387, 389, 391

Dermatology116, 154, 173, 316

Endocrinology and diabetes5, 9, 23, 39, 46, 76, 82, 89, 92, 101, 106, 107, 127,134, 146, 159, 164, 168, 173, 181, 199, 218, 220, 238,242, 254, 260, 261, 273, 281, 328, 334, 372, 373, 379,397, 401

Environmental medicine140

Gastroenterology3, 6, 19, 24, 33, 64, 72, 75, 104, 127, 133, 143, 148,162, 169, 182, 188, 201, 231, 276, 293, 306, 338, 339,347, 367, 369, 371, 383, 393, 394, 400

Genetics47, 85, 151, 170, 194, 195, 269, 315, 361

Haematology12, 38, 49, 69, 70, 73, 86, 87, 102, 114, 115, 117, 120,122, 142, 156, 163, 175, 191, 204, 211, 216, 219, 233,258, 263, 265, 295, 297, 299, 308, 313, 336, 346, 351,352, 358, 376, 385, 392, 394

Immunology15, 155, 374

Infectious diseases16, 18, 26, 41, 51, 83, 88, 93, 110, 128, 142, 143, 149,152, 154, 158, 166, 176, 212, 221, 225, 234, 262, 267,277, 280, 319, 322, 325, 337, 345, 351, 383, 386, 388

Metabolic medicine9, 29, 34, 38, 50, 71, 74, 81, 82, 84, 90, 129, 134, 136,147, 153, 161, 179, 189, 214, 215, 230, 248, 257, 271,275, 283, 310, 321, 326, 329, 333, 334, 398

Nephrology4, 17, 24, 29, 44, 53, 59, 60, 85, 92, 118, 119, 126, 135,137, 141, 152, 185, 198, 228, 244, 245, 249, 250, 251,278, 289, 294, 303, 304, 317, 328, 344, 354, 381, 382

Neurology30, 65, 67, 93, 98, 103, 105, 108, 112, 128, 139, 145,190, 192, 200, 239–241, 247, 253, 255, 256, 268, 274,288, 290, 292, 307, 314, 330, 345, 365, 390, 395, 399

Obstetric medicine130–132, 190, 193, 348

Oncology117, 216, 258, 358, 359

Ophthalmology282, 345

Radiology2, 18, 64, 88, 97, 99, 124, 183, 187, 222, 227, 252,280, 300, 302, 311, 343, 349, 355, 357, 360, 363

Respiratory medicine8, 14, 21, 35, 36, 37, 43, 45, 55, 56, 58, 61, 72, 79, 91,99, 111, 113, 157, 164, 196, 217, 225, 272, 279, 298,304, 327, 341, 349, 356, 370, 380, 384, 396

Rheumatology4, 15, 17, 31, 42, 71, 77, 87, 96, 109, 141, 171, 174,177, 196, 198, 200, 210, 236, 264, 320, 324, 340, 364,375, 401, 402

Therapeutics/toxicology7, 8, 20, 27, 28, 36, 48, 57, 68, 77, 116, 118, 119, 165,172, 175, 186, 205, 206, 209, 213, 224, 251, 284, 286,312, 316, 317, 332, 339, 366, 378

4

Classification of Cases

Page 6: Rapid review of clinical medicine for mrcp part 2

5

5-HIAA 5'-hydroxyindole aceticacid

AIIRB angiotensin II receptorblocker

AAFB acid–alcohol fast bacilliACE angiotensin-converting

enzymeACTH adrenocorticotrophic

hormoneADH antidiuretic hormoneAF atrial fibrillationAIDS acquired immune-

deficiency syndromeAIN acute interstitial nephritisAIP acute intermittent

porphyriaALA aminolaevulinic acid ALT alanine transaminase

(SGPT)AML acute myeloid leukaemiaAMP adenosine 5'-

monophosphateANA antinuclear antibodyANCA antineutrophil cytoplasmic

antibodiesANF antinuclear factorAPCKD adult polycystic kidney

diseaseAPTT activated partial

thromboplastin timeAR aortic regurgitationARDS adult respiratory distress

syndromeARVC arrhythmogenic right

ventricular cardiomyopathyAS aortic stenosisASD atrial septal defectASO antistreptolysin AST aspartate transaminase

(SGOT)ATN acute tubular necrosisAZT zidovudineBCG bacille Calmette–GuérinBIH benign intracranial

hypertensionBP blood pressureBT bleeding timeBTS British Thoracic SocietyCAH chronic active hepatitisCAP community acquired

pneumoniaCCF congestive cardiac failureCFTR cystic fibrosis

transmembrane regulator(protein)

CML chronic myeloid leukaemiaCMV cytomegalovirusCOPD chronic obstructive

pulmonary diseaseCPAP continuous positive airway

pressureCREST calcinosis, Raynaud’s

syndrome, oesophagealproblems, scleroderma,telangiectasia

CRF chronic renal failureCRP C-reactive proteinCSF cerebrospinal fluidCSS Churg–Strauss syndromeCT computed tomographyCVA cerebrovascular accident CVP central venous pressureCXR chest X-rayDBP diastolic blood pressureDC direct currentDHCC dihydroxy-cholecalciferolDIC disseminated intravascular

coagulationDIDMOAD diabetes insipidus,

diabetes mellitus, opticatrophy and deafness

DM diabetes mellitusDT delerium tremensDVT deep-vein thrombosisEAA extrinsic allergic alveolitisEBV Epstein–Barr virusECG electrocardiogramEEG electroencephalogramELISA enzyme-linked

immunosorbent assayEMF endomyocardial fibrosisEMG electromyogramENT ear, nose and throatEPO erythropoietinERCP endoscopic retrograde

cholangiopancreatogramESR erythrocyte sedimentation

rateFBC full blood countFDP fibrinogen degradation

productFES fat embolism syndromeFEV1 fixed expiration volume in

1 secondFFP fresh-frozen plasmaFNA fine-needle aspirationFSH follicle stimulating

hormoneFTA fluorescent treponemal

antibody

FVC forced vital capacityGBM glomerular basement

membraneGCT giant cell tumourGFR glomerular filtration rateGH growth hormoneGHRH growth hormone releasing

hormoneGI gastrointestinalGP general practitionerGPI glucophosphatidylinositolGT glutamyltransferaseGTN glyceryl trinitrateHb haemoglobinHbSS sickle cell anaemiaHC Hereditary Copro

porphyriaHCC hydroxy-cholecalciferolHCM hypertrophic

cardiomyopathyHCV hepatitis C virusHCG human chorionic

gonadotrophinHELLP haemolysis, elevated liver

enzymes and low plateletsHHT hereditary haemorrhagic

telangiectasiaHIT heparin-induced

thrombocytopeniaHIV human immunodeficiency

virusHONK hypersimilar non-ketotic

diabetic comaHR heart rateHRT hormone replacement

therapyHS hereditary spherocytosisHSMN hereditary sensorimotor

neuropathyHUS haemolytic uraemic

syndromeICD implantable cardioverter

defibrillatorICP intracranial pressureINR International Normalized

RatioIPF idiopathic pulmonary

fibrosisIVP intravenous pyelogramIVU intravenous urogramJVP jugular venous pressureKCO corrected carbon monoxide

transfer factorLBBB left bundle branch blockLDH lactate dehydrogenase

Abbreviations

Page 7: Rapid review of clinical medicine for mrcp part 2

6

LFT liver function testsLH luteinizing hormoneLHON Leber’s hereditary optic

neuropathyLHRH luteinizing hormone

releasing hormoneLMWH low-molecular weight

heparinLQTS long QT-syndromeLVEDP left ventricular end-diastolic

pressureLVH left ventricular hypertrophyMAHA microangiopathic

haemolytic anaemiaMAOI monoamine oxidase

inhibitorMCH mean cell haemoglobinMCHC mean cell haemoglobin

contentMCV mean cell volumeMELAS mitochondrial

encephalopathy, lacticacidosis, stroke-likesyndrome

MEN multiple endocrineneoplasia

MERRF myoclonic epilepsy and redragged fibres

MGUS monoclonal gammopathyof undeterminedsignificance

MPO myeloperoxidaseMR mitral regurgitationMRA magnetic resonance

angiographyMRCP magnetic resonance

cholangiopancreatogramMRI magnetic resonance

imagingMRSA methicillin resistant

Staphylococcus aureusMRV magnetic resonance

venographyMSH melanocyte stimulating

hormoneNADPH nicotinamide adenine

dinucleotide phosphate(reduced)

NAPQI N-acetyl-p-benzoquinoneimine

NARP neuropathy, ataxia, retinitispigmentosa

NASH non-alcoholicsteatohepatitis

NIPPV non-invasive positivepressure ventilation

NSAID non-steroidal anti-inflammatory drug

NSTEMI non-ST elevationmyocardial infarction

NYHA New York HeartAssociation

OSA obstructive sleep apnoeaPAN polyarteritis nodosaPAS periodic acid-SchiffPBC primary biliary cirrhosisPBG porphobilinogenPCOS polycystic ovary syndromePCR polymerase chain reactionPCT porphyria cutanea tardaPCV packed cell volumePCWP pulmonary capillary wedge

pressurePE pulmonary embolismPEFR peak expiratory flow ratePFO patent foramen ovalePKD polycystic kidney diseasePMLE progressive multifocal

leucoencephalopathyPMR polymyalgia rheumaticaPNH paroxysmal nocturnal

haemoglobinuriaPRL prolactinPRV polycythaemia rubra veraPSC primary sclerosing

cholangitisPT prothrombin timePTH parathormone or

parathyroid hormonePVE prosthetic valve

endocarditisRA rheumatoid arthritisRBBB right bundle branch blockREM rapid eye movementRMAT rapid macroagglutination

testRTA renal tubular acidosisRV residual volumeSADS sudden adult death

syndromeSAM systolic anterior motion of

the mitral valveSAP serum amyloid proteinSIADH syndrome of inappropriate

antidiuretic hormoneSLE systemic lupus

erythematosusSMA smooth muscle antibodySPECT single photon emission

computed tomographySROS Steele–Richardson–

Olszewski syndromeSTEMI ST elevation myocardial

infarctionSVT supraventricular tachycardiaTB tuberculosis

TCAD tricyclic antidepressantoverdose

TIA transient ischaemic attackTIBC total iron-binding capacityTIPSS transjugular intrahepatic

portosystemic shuntTLC total lung capacityTLCO total lung carbon

monoxide transfer factorTOE transoesophageal

echocardiographyTPA tissue plasminogen

activatorTPHA treponema pallidum

haemagglutination testTRH thyrotrophin releasing

hormoneTSAT transferrin saturationTSH thyroid stimulating

hormoneTT thrombin timeTTP thrombotic

thrombocytopenic purpuraU&E urea and electrolytesURTI upper respiratory tract

infectionUS ultrasoundUTI urinary tract infectionVDRL Venereal Diseases Research

Laboratory testVF ventricular fibrillationVIP vasointestinal polypeptideVMA vanilyl mandelic acidVP variegate porphyriaVR ventricular rateVSD ventricular septal defectVT ventricular tachycardiaWCC white cell countWPW Wolff–Parkinson–White

(syndrome)

Page 8: Rapid review of clinical medicine for mrcp part 2

A 49-year-old male presented to the Accident andEmergency Department with a one-hour history of severecentral chest pain. He smoked 30 cigarettes per day.Physical examination was normal. The 12-lead ECGrevealed ST segment elevation in leads V1–V4. Therewere no contraindications to thrombolysis.

Clinical Cases 7

Question 1

What is the best treatment to improve coronary perfusion?a. IV Streptokinase.b. IV Tenectoplase.c. IV Alteplase.d. Half-dose tenectoplase and half-dose abciximab.e. Primary coronary angioplasty.

Question 2

A 68-year-old woman presented with pain and tingling inthe left arm when she raised her hands for prolongedperiods. On examination both pulses were palpable in theupper limbs. The chest X-ray was abnormal. Aortographywas performed with the arms down (2a) and with thearms up (2b).

What was the abnormality on the chest X-ray?a. Left-sided bronchogenic carcinoma.b. Left cervical rib.c. Retrosternal thyroid.d. Notching of the ribs.e. Widened mediastinum.

A 28-year-old male presented with a six-month history ofweight loss of 8 kg, generalized abdominal discomfortand diarrhoea. On examination he was pale and slim, butthere were no other significant abnormalities.

Investigations are shown.

Question 3

Hb 9 g/dlWCC 4.6 � 109/lPlatelets 200 � 109/lMCV 76 flESR 38 mm/hSodium 141 mmol/lPotassium 4 mmol/lUrea 3 mmol/lCreatinine 68 �mol/lCorrected calcium 2.02 mmol/lphosphate 0.8 mmol/lAlkaline phosphatase 190 iu/lAlbumin 38 g/lIgA <0.1 g/l (NR 0.8–4.0 g/l)IgG 9.0 g/l (NR 7.0–18.0 g/l)IgM 0.6 g/l (NR 0.4–2.5 g/l)IgA anti-endomyosial Absentantibody

What is the diagnosis?a. Crohn’s disease.b. Intestinal lymphangiectasia.c. Coeliac disease.d. Small bowel lymphoma.e. Hypogammaglobulinaemia.

22aa 22bb

Page 9: Rapid review of clinical medicine for mrcp part 2

8

Answer 1

Coronary reperfusion may be achieved with thrombolyticagents (which promote fibrinolysis) or by coronaryangioplasty. In the UK patients with ST elevationmyocardial infarction are conventionally treated withthrombolytic agents. Early treatment is crucial to salvagemyocardium and reduce the risk of sudden death andsevere left ventricular dysfunction. Current goals for thespeed of treating with a thrombolytic agent include adoor-to-needle time of 20 minutes or a call-to-needletime of 60 minutes.

Thrombolytic agents used commonly includestreptokinase, alteplase, tenectoplase and reteplase.Streptokinase is less favoured compared with the otherthrombolytic agents because it is less effective at restoringcoronary perfusion and is associated with slightly worseoutcomes. The GUSTO I study compared front-loadedalteplase therapy with streptokinase in patients with STEMI. Alteplase was superior to streptokinase in reducingmortality (1% absolute reduction in mortality at 30 dayswith alteplase) and was associated with greater coronarypatency rates. In the GUSTO trial the benefit wasgreatest in patients aged under 75 years and those withanterior myocardial infarction. However, streptokinase isstill used extensively in developing countries and in manyhospitals in the UK. Alteplase, tenectoplase and reteplaseappear to be equally effective. Tenectoplase and reteplaseare easier to administer (as a single bolus).

There have been trials evaluating the role of combinedhalf-dose thrombolytic therapy and half-dose plateletglycoprotein IIb/IIIa receptor blockers, e.g. tenectoplaseplus abciximab (ASSENT 3) and reteplase plus abciximab(GUSTO IV). These trials suggest that the combinationmay be associated with slightly higher coronary patencyrates and fewer ischaemic events but they have notdemonstrated a mortality benefit. These trials have alsodemonstrated higher rates of intracranial bleeding in theelderly, hence combination therapy is not recommendedat present.

Although thrombolytic treatment is associated with asignificant reduction in mortality from myocardialinfarction, it does have important limitations. Firstly,greatest benefit from thrombolysis is achieved in patientstreated within 4 hours of the onset of symptoms. Even withthrombolysis normalization of blood flow is seen in only50–60% of cases. Recurrent ischaemia occurs in 30% ofcases and frank thrombotic coronary occlusion in 5–15%.Re-infarction occurs in up to 5% of cases while in hospital.Also major bleeding is recognized in 2–3% of cases. Forthese reasons several trials were set up comparing primaryangioplasty with thrombolysis in STEMI.

Primary angioplasty is superior to thrombolysis. It isassociated with lower mortality and lower re-infarctionrates. The likelihood of a pre-discharge positive exercisetest is also reduced by primary angioplasty. In hospitalswhere facilities for primary angioplasty are available,primary angioplasty should be considered overthrombolysis. Best results occur when the door-to-balloon time is less than 2 hours.

e. Primary coronary angioplasty.

There is mechanical occlusion of the left subclavian arteryon raising the left arm due to a left cervical rib. Cervical

ribs are common in the normal population and areusually asymptomatic. In rare circumstances a cervical ribmay cause pressure on the subclavian vessels and thebrachial plexus causing transient vascular insufficiency orparaesthesiae in the upper limb.

Answer 2

b. Left cervical rib.

Diarrhoea, weight loss, abdominal discomfort andisolated IgA deficiency are highly suggestive of coeliacdisease. Anti-endomyosial antibodies are highly sensitiveand specific for the diagnosis of coeliac disease. Anti-

endomyosial antibodies are IgA antibodies, thereforethey will not be detected in patients with low IgAantibody levels. Since coeliac disease is also associatedwith IgA deficiency it is important to be aware of serumIgA levels before interpreting anti-endomyosialantibodies in patients with malabsorption. (See Question276.)

Answer 3

c. Coeliac disease.

msm7225
Strike-Out
Page 10: Rapid review of clinical medicine for mrcp part 2

Clinical Cases 9

Question 4

A 53-year-old male was admitted to hospital with a two-week history of coughing and breathlessness. Apart froma longstanding history of mild asthma he had beenrelatively well with respect to the respiratory tract. Hehad been on a skiing trip six weeks previously, withoutany respiratory problems.

He had a past history of depression, for which he tooklithium five years ago, and suffered from occasionaltension headaches, for which he took simple analgesia.

On examination he appeared pale and unwell. Hisheart rate was 90 beats/min and regular. His bloodpressure measured 160/94 mmHg. The JVP was notraised. Both heart sounds were normal and the chest wasclear. Abdominal examination did not reveal anyabnormality. Urinalysis demonstrated blood ++ andprotein ++.

Investigations performed in hospital are shown.

Hb 7 g/dlWCC 11 � 109/1

(neutrophils 8 � 109/l,lymphocytes 2 � 109/l,eosinophils 1 � 109/l)

ESR 38 mm/hSodium 134 mmol/lPotassium 4.6 mmol/lUrea 48 mmol/lCreatinine 798 mmol /lRenal ultrasoundBoth kidneys measured 12 cm: there was noevidence of ureteric obstruction.

Question 5

A 52-year-old male presented with impotence. He had afour-year history of insulin-dependent diabetes mellitus.There was no history of headaches or vomiting. Thepatient was a non-smoker and did not consume alcohol.Apart from insulin he took simple analgesia for jointpains.

Investigations are shown.

FBC NormalSodium 135 mmol/lPotassium 4 mmol/lUrea 6 mmol/lCreatinine 100 mmol/l Bilirubin 12 mmol/l AST 200 iu/lALT 220 iu/lAlkaline phosphatase 128 iu/lAlbumin 8 g/lThyroxine 100 nmol/lTSH 2.6 mu/lTestosterone 7 nmol/l (NR 10–35 nmol/l)

LH 1.5 iu/l (NR 1–10 iu/l)FSH 1 iu/l NR 1–7 iu/l)LHRH test: 20 min: 60 min:LH 3 iu/l 2 iu/lFSH 2 iu/l 2 iu/l

What is the most likely diagnosis?a. Rapidly progressive glomerulonephritis.b. Analgesic nephropathy.c. Renal amyloidosis.d. Churg–Strauss syndrome.e. IgA nephritis.

What test would you perform to confirm thediagnosis?

a. MRI scan of the brain.b. Serum prolactin level.c. Serum ferritin.d. Dynamic pituitary function tests.e. Liver ultrasound.

Page 11: Rapid review of clinical medicine for mrcp part 2

The clinical features and the data are consistent with thediagnosis of idiopathic haemochromatosis. The insulin-dependent diabetes mellitus suggests pancreatic

involvement, and abnormal liver function is consistentwith hepatic infiltration.

The patient has a low testosterone level with aninappropriately low gonadotrophin response indicatingsecondary hypogonadism due to excessive iron depositionin the pituitary. Secondary hypogonadism is the most

The patient has a past history of asthma, eosinophilia andrapidly progressive glomerulonephritis. The most probablediagnosis is Churg–Strauss syndrome. The assumption thathe probably has rapidly progressive glomerulonephritis isbased on the fact that he was well enough to ski six weeksago, which would be highly unlikely in a patient with end-stage renal disease. The identification of normal-sizedkidneys during renal ultrasonography supports acute ratherthan chronic renal failure (Table A).

Churg–Strauss syndrome is a small-vessel multi-systemvasculitis characterized by cutaneous vasculitic lesions,eosinophilia (usually <2.0 � 109/l), asthma (usuallymild), mononeuritis or polyneuropathy and rarelyglomerulonephritis (10% of cases). Gastrointestinal andcardiac involvement is recognized.

Pulmonary findings dominate the clinical presentationwith paroxysmal asthma attacks and presence of fleetingpulmonary infiltrates. Asthma is the cardinal feature and maybe present for years before overt features of a multi-systemvasculitis become apparent. Skin lesions, which includepurpura and cutaneous and subcutaneous nodules, occur inup to 70% of patients. Gastrointestinal complications includemesenteric ischaemia or gastrointestinal haemorrhage.Cardiac involvement is characterized by myo-pericarditis.

The diagnosis is usually clinical and supported by thepresence of a necrotizing granulomatous vasculitis withextravascular eosinophilic infiltration on lung, renal or suralbiopsy. The American College or Rheumatology criteriafor the diagnosis of Churg–Strauss syndrome are tabulated(Table B). Serum ANCA (MPO subset) are elevated butthis finding is also present in microscopic polyangitis.

The prognosis of untreated CSS is poor, with areported five-year survival rate of only 25%. Corticosteroidtherapy has been reported to increase the five-yearsurvival rate to more than 50%. In patients with acutevasculitis the combination of cyclophosphamide andprednisone is superior to prednisolone alone.

Although rapidly progressive glomerulonephritis alsofeatures in the answer options section, the presence ofasthma and eosinophilia make Churg–Strauss syndromethe best answer. It is worth noting however, that rapidlyprogressive glomerulonephritis may also rarely be

associated with eosinophilia. Causes of renal failure andeosinophilia are tabulated (Table C).

The history of analgesia for headaches raises thepossibility of analgesic nephropathy as the cause of hispresentation; however, analgesic nephropathy is usuallyinsidious and many patients present for the first time withrenal failure. The majority have abnormalities on renalultrasound scans. Analgesic nephropathy alone does notexplain asthma or eosinophilia.

d. Churg–Strauss syndrome.

10

Table A Phases of Churg–Strauss syndrome:

1. The prodromal phase, which may be present for years and comprises of rhinitis, nasal polyposis and frequently asthma.

2. The eosinophilic phase, which can remit and recur for years. It is characterized by the onset of peripheral blood and tissue eosinophilia, resembling Loeffler’s syndrome, chronic eosinophilic pneumonia or eosinophilic gastroenteritis.

3. The vasculitic phase, which usually occurs in the third or fourth decades of life and is characterized by a life-threatening systemic vasculitis of small and occasionally medium-sized vessels. This phase is associated with constitutional symptoms and signs, fever and weight loss.

Table B American College of Rheumatology 1990criteria for Churg–Strauss syndrome

The presence of four or more of the manifestationsbelow is highly indicative of Churg–Strauss syndrome:• Asthma • Eosinophilia (10% on WCC differential) • Mononeuropathy or polyneuropathy • Migratory or transient pulmonary infiltrates • Systemic vasculitis (cardiac, renal, hepatic) • Extravascular eosinophils on a biopsy including

artery, arteriole or venule

Table C Causes of renal failure and eosinophilia

• Rapidly progressive glomerulonephritis• Churg–Strauss syndrome• Acute tubulo-interstitial nephritis• Cholesterol micro-emboli

Answer 4

Answer 5

c. Serum ferritin.

Page 12: Rapid review of clinical medicine for mrcp part 2

common endocrine deficiency in hereditary haemo -chromatosis. Primary hypogonadism due to testiculariron deposition may occur with this disorder but is muchless common than secondary hypogonadism.

In the context of the question, a serum ferritin level>500 mg/l would be diagnostic of primary haemo -chromatosis. Alcohol-related liver disease, chronic viralhepatitis, non-alcoholic steatohepatitis and porphyriacutanea tarda also cause liver disease and increased serum

ferritin con centrations even in the absence of iron overload. Hepatic iron overload in haemochromatosis is associated

with an increased risk of hepatocellular carcinoma. Patientswith haemochromatosis are also at increased risk ofhypothyroidism and are susceptible to certain infectionsfrom siderophoric (iron-loving) organisms such as Listeriaspp., Yersinia enterocolitica and Vibrio vulnificus, which arepicked up from eating uncooked seafood.

Clinical Cases 11

A 38-year-old English male was investigated after he wasfound to have an abnormal liver function test during ahealth insurance medical check. He worked in aninformation technology firm. Apart from occasionalfatigue he was well. He consumed less than 20 units ofalcohol per week. The patient had only travelled out ofEurope twice and on both occasions he had been toNorth America. He took very infrequent paracetamol foraches and pains in his ankles and knees. There was nohistory of hepatitis or transfusion or blood products. Hehad been married for 5 years. Systemic enquiry revealedinfrequent episodes of loose stool for almost 4 years.

On examination he appeared well. There were nostigmata of chronic liver disease. Abdominal examinationrevealed a palpable liver edge 3 cm below the costalmargin. There were no other masses. Examination of thecentral nervous system was normal.

Investigations were as shown.

Hb 12.6 g/dlWCC 8 � 109/lPlatelets 210 � 109/lMCV 90 flSodium 136 mmol/lPotassium 4.1 mmol/lUrea 6 mmol/lCreatinine 100 mmol/lAST 60 iu/l (NR 10–40 iu/l)ALT 78 iu/l (NR 5–30 iu/l)Alkaline phosphatase 350 iu/l (NR 25–100 iu/l)Bilirubin 22 mmol/l (NR 2–17 μmol/l)Albumin 38 g/l (NR 34–48 g/l)Total cholesterol 5.2 mmol/lTriglyceride 3.1 mmol/lBlood glucose 6 mmol/lFerritin 256 mg/l (NR 15–250 mg/l) Serum Fe 28 mmol/l

(NR 14–32 mmol/l)TIBC 50 mmol/l

(NR 40–80 mmol/l)Serum Slightly reduced

caeruloplasmin24-hr urine copper Slightly elevatedIgG 19 g/l (NR 7–18 g/l)IgA 4.2 g/l (NR 0.8–4.0 g/l)IgM 5.0 g/l (NR 0.4–2.5 g/l)Anti-nuclear Positive 1/32

antibodiesSmooth muscle Not detected

antibodiesAntimitochondrial Not detected

antibodiesHep B sAg Not detectedHep C virus Not detected

antibodiesAbdominal ultrasound Normal

What is the most probable diagnosis?a. Autoimmune hepatitis.b. Primary sclerosing cholangitis.c. Primary biliary cirrhosis.d. Haemochromatosis.e. Wilson’s disease.

Question 6

Question 7

A 17-year-old girl presented with jaundice three daysafter having taken a paracetamol and alcohol overdoseduring an argument with her boyfriend.

What is the best marker of prognosis?a. Serum aspartase transaminase.b. Serum alkaline phosphatase.c. Serum bilirubin.d. Prothrombin time.e. Paracetamol level.

Page 13: Rapid review of clinical medicine for mrcp part 2

This is a relatively difficult question. The history of loosestool is crucial in making the diagnosis in this particularcase in the absence of data from the ERCP. Diarrhoeaand biochemical evidence of cholestasis (alkalinephosphatase greater than transaminases) should lead tothe clinical suspicion of primary sclerosing cholangitis(PSC). The aetiology of PSC is unknown butimmunological destruction of intra- and extra-hepaticbile ducts is the main pathological feature. 90% of PSC isassociated with inflammatory bowel disease, particularlyulcerative colitis, and hence the importance of theintermittent diarrhoea. Ulcerative colitis is the mostfrequent association with primary sclerosing cholangitis.A raised alkaline phosphatase level in a patient withulcerative colitis (in the absence of bone disease) shouldraise the possibility of PSC. The frequency of PSC isinversely proportional to the severity of ulcerative colitis.Other associations of PSC include coeliac disease.

Patients with PSC may be asymptomatic at pre -sentation but can present with advanced liver disease.Fatigue and pruritus are common complaints as with theother cholestatic disorders. Approximately one-fifth ofthe patients also complain of right upper quadrant pain.

The diagnosis is confirmed with ERCP that showsstrictures within biliary ducts. Complications are those ofchronic cholestasis, notably statorrhoea, fat-solublevitamin malabsorption, large biliary strictures, cholangitis,cholangiocarcinoma and colonic carcinoma. There are noeffective pharmacological agents that greatly retard the

progression of the disorder. Patients are treated withcholestyramine to reduce pruritus. Fat-soluble vitaminsupplementation is necessary owing to steatorrhoea.Antibiotic prophylaxis during instrumentation of thebiliary tree is mandatory to reduce the risk of bacterialcholangitis. Ciprofloxacin is the prophylactic antibioticdrug of choice prior to ERCP. Biliary stenting mayimprove biochemistry and symptoms; however, thedefinitive treatment for PSC is hepatic transplantation.

Although a cholestatic picture is also recognized inprimary biliary cirrhosis, alcohol abuse and viral hepatitisthere is nothing in the history or investigations to indicatethese conditions as the cause of his illness. Primary biliarycirrhosis affects mainly females in the fifth decadeonwards. Furthermore, the absence of anti-mitochondrialantibodies is against the diagnosis. The ferritin is modestlyraised but not high enough to suggest hereditaryhaemochromatosis. High ferritin levels are also a featureof chronic viral hepatitis, alcohol-related hepatitis andnon-alcoholic steato-hepatitis. Hyper gammaglobulinaemiaand raised autoantibody titres are features of primarysclerosing cholangitis but also occur in otherimmunological liver disorders such as chronic active viralhepatitis, auto-immune hepatitis and biliary cirrhosis.

Patients with cholestasis also have lowish caerulo -plasmin levels and increased blood and urine copperlevels. The abnormal copper metabolism in this caseshould not lead to the candidate diagnosing Wilson’sdisease, since there are many features above to indicatePSC. Furthermore, patients with Wilson’s disease usuallyhave a hepatitic biochemistry picture and often have co-existing neuro-psychiatric disease.

b. Primary sclerosing cholangitis.

Answer 6

12

Important risk markers for severe hepatic injury afterparacetamol overdose include a PT >20 seconds 24 hafter ingestion, pH <7.3 and creatinine >300 �mol/l.(See Questions 27 and 206.)

Answer 7

d. Prothrombin time.

Page 14: Rapid review of clinical medicine for mrcp part 2

A 40-year-old woman with dilated cardiomyopathy isseen in the heart failure clinic complaining of a persistentdry cough. Her exercise capacity is 1 mile while walkingon the flat. She can climb two flights of stairs withoutdifficulty. Her medication consists of ramipril 10 mgdaily, aspirin 75 mg daily, carvedilol 6.25 mg twice dailyand frusemide 40 mg daily. On examination her heartrate is 70 beats/min and her blood pressure is100/60 mmHg. Both heart sounds are normal and thechest is clear.

A 16-year-old girl presented with an 18-month history ofprogressive breathlessness on exertion. On admission shewas breathless at rest. She had a past history of acutemyeloid leukaemia, for which she had been treated withsix courses of chemotherapy, followed by bone marrowtransplantation supplemented with radiotherapy andcyclophosphamide treatment five years ago. She wasregularly followed up in the haematology clinic. Lungfunction tests three years ago revealed an FEV1/FVCratio of 80%. On examination she was breathless at rest,and cyanosed. There was no evidence of clubbing.Auscultation of the lung fields revealed fine inspiratory

crackles in the mid and lower zones. Repeat lungfunction tests revealed an FEV1/FVC ratio of 86% and atransfer factor of 60% predicted.

Clinical Cases 13

Question 8

What is the cause of her symptoms?a. Previous radiotherapy.b. CMV pneumonitis.c. Pneumocystis carinii pneumonia.d. Cyclophosphamide-induced lung fibrosis.e. Severe anaemia.

A 21-year-old man was admitted to the intensive careunit after a road traffic accident during which he suffered

a severe head injury. He required ventilation. Investigations are shown.

Sodium 128 mmol/l Potassium 3.6 mmol/l Creatinine 81 mmol/lUrea 4 mmol/lThyroxine 30 nmol/l TSH 2 mu/lSerum cortisol 1000 nmol/l

(NR 170–700 nmol/l)

Question 9

Question 10

How would you alter her treatment?a. Add spironolactone.b. Substitute ramipril with losartan.c. Reduce carvedilol to 3.125 mg twice daily.d. Double the dose of furosemide.e. Add digoxin.

What is the cause of the hyponatraemia? a. Hypopituitarism.b. Addison’s disease.c. Syndrome of inappropriate ADH secretion.d. Hypothyroidism.e. Cushing’s syndrome.

Page 15: Rapid review of clinical medicine for mrcp part 2

The patient has a low sodium concentration in thecontext of a head injury. The thyroid function testssuggest the possibility of a secondary hypothyroidism, i.e.a low TSH and a low thyroxine concentration, and hencethe possibility of damage to the pituitary. However, thevery high cortisol level indicates that pituitary function isprobably normal (high ACTH production secondary tostress) and therefore the abnormal thyroid function testsrepresent sick euthyroid syndrome. Low T4, T3 and TSHlevels are recognized in critically ill patients with non-thyroid illnesses. Originally such patients were thought to

be euthyroid, therefore the term sick euthyroid syndromewas used to describe these biochemical abnormalities.There is evidence now that these abnormalities representgenuine acquired transient central hypothyroidism.Treatment with thyroxine in these situations is nothelpful and may be harmful. It is thought that thesechanges in thyroid function during severe illness may beprotective by preventing excessive tissue catabolism.Thyroid function tests should be repeated after at least sixweeks following recovery.

Critical illness may also reduce T4 by reducing thyroidbinding globulin levels, and T3 is rapidly reduced owingto inhibition of peripheral de-iodination of T4.

The patient presents with progressive symptomsassociated with a restrictive lung defect and a low transfer factor. The findings are most consistent withcyclophosphamide-induced pulmonary fibrosis.

Cyclophosphamide-induced lung fibrosis is rare and ismost likely to occur in patients who have had concomitantpulmonary radiation therapy or have taken other drugsassociated with pulmonary toxicity. The disorder usuallyoccurs in patients who have been taking low doses forrelatively prolonged periods (over six months) andpresents several years after cessation of the drug andhence the deterioration of symptoms with time. Thedisorder has a relentless progression and inevitably resultsin terminal respiratory failure. It is minimally responsiveto corticosteroids. Fine end-inspiratory crackles andclubbing do not usually form part of the clinicalspectrum.

The diagnosis is clinical. Chest X-ray reveals reticulo-nodular shadowing of the upper zones. Lung functiontests demonstrate a restrictive lung defect. Lung biopsy isnot helpful.

Cyclophosphamide per se is not toxic to the lungs;however, it is metabolized in the liver to toxicmetabolites such as hydroxycyclophosphamide, acroleinand phosphoramide mustard, which are responsible forpulmonary damage. Genetic factors may play a role indetermining which individuals develop pulmonaryfibrosis after exposure to the drug.

Cyclophosphamide therapy can also result in an acutepneumonitis during treatment with the drug that causescough, dyspnoea, hypoxia and bilateral nodular opacitiesin the upper zones of the lung. Acute cyclophosphamide-induced pneumonitis responds to cessation of the drugand corticosteroid therapy.

The differential diagnosis in this case is radiation-induced fibrosis. Radiotherapy to the pulmonary areausually causes a pneumonitis that presents with cough,dyspnoea, a restrictive lung defect and low transfer factor.It is more common in patients also taking cyclo -phosphamide or bleomycin. Unlike cyclophosphamide-induced pulmonary fibrosis the condition is notassociated with an inexorable decline. Indeed manypatients show improvement in symptoms and objectivepulmonary function testing within 18 months ofstopping radiotherapy.

14

Answer 8

d. Cyclophosphamide-induced lung fibrosis.

Causes of drug-induced pulmonary fibrosis

• Cyclophosphamide• Busulphan• Methysergide• Methotrexate• Amiodarone• Nitrofurantoin• Minocycline• Ethambutol• Penicillamine

Answer 9

c. Syndrome of inappropriate ADH secretion.

Page 16: Rapid review of clinical medicine for mrcp part 2

A 60-year-old male was admitted to the coronary care unitwith central chest pain. Physical examination was normal.The blood pressure measured 110/68 mmHg. The 12-lead ECG was normal and the troponin T level was notraised. The blood sugar was normal. The cholesterol levelon admission was 6.3 mmol/l. The patient underwent anexercise stress test that was positive. A subsequentcoronary angiogram revealed an 80% stenosis in theproximal aspect of the left anterior descending artery thatwas successfully treated with a coronary artery stent.Echocardiography revealed a normal-sized left ventriclewith good systolic function. The patient was dischargedhome on aspirin 75 mg daily, clopidogrel 75 mg daily andsimvastatin 40 mg daily. He had been completely pain free

after the procedure, and an exercise stress test performedfour weeks after the procedure was negative formyocardial ischaemia for 10 minutes.

The patient is in NYHA functional class II with respect toher symptoms. She is on the correct dose of ramipril andis appropriately being treated with a beta-blocker. Thedry cough that the patient is experiencing is almostcertainly the side-effect of ramipril. Angiotensin-converting enzyme inhibitors are associated with a drycough in 15–20% of patients owing to increases incirculating bradykinin levels. In such patients the ACEinhibitor should be stopped and substituted with anangiotensin receptor blocker such as losartan. Theefficacy of losartan compared with an ACE inhibitor(captopril) was fully evaluated in the ELITE II study.

The study revealed similar mortality rates and similarrates of progression of heart failure when comparingpatients on losartan 50 mg daily with those prescribedcaptopril 50 mg three times daily. The study suggeststhat losartan is as effective as ACE inhibitors in themanagement of heart failure. However, the use oflosartan in heart failure is still currently reserved forpatients who develop side-effects to ACE inhibitors. Arecent study evaluating the role of angiotensin receptorblockers (CHARM study; evaluated candesartan) inpatients with heart failure showed reduced hospitalizationrates and mortality in heart failure patients who were oncandesartan instead of an ACE inhibitor, or candesartanas additional therapy to an ACE inhibitor.

Clinical Cases 15

Answer 10

b. Substitute ramipril with losartan.

Question 11

What other medication should the patient receive toimprove his cardiovascular prognosis?

a. Atenolol.b. Ramipril.c. Candesartan.d. No further treatment required.e. Isosorbide dinitrate.

A 62-year-old obese male with a known medical historyof hypertension presented with generalized headachesand lethargy. He was taking bendroflumethiazide,2.5 mg once daily for hypertension. The only other pastmedical history included a left-sided deep veinthrombosis six months previously. There was no historyof alcohol abuse or smoking.

On examination he was obese. His chest was clear andexamination of the abdomen did not reveal anyabnormality.

Investigations are shown.

Hb 20 g/dlMCV 88 flWCC 15 � 109/lPlatelets 500 � 109/lPCV 0.66 l/lSodium 141 mmol/lPotassium 4.2 mmol/lUrea 8 mmol/lCreatinine 110 �mol/lUrate 0.44 mmol/l

What is the cause of his symptoms?a. Obstructive sleep apnoea.b. Gaissbock’s syndrome.c. Polycythaemia rubra vera.d. Renal cell carcinoma.e. Chronic hypoxaemia.

Question 12

Page 17: Rapid review of clinical medicine for mrcp part 2

The Heart Outcomes Prevention Evaluation Study(HOPE) evaluated the role of angiotensin-convertingenzyme inhibitors (ramipril) in populations at high risk ofcardiovascular events without any evidence of leftventricular dysfunction. The study assessed 9297 high-risk patients, defined as (1) aged >55 years; (2) history ofcoronary artery disease, stroke or peripheral vasculardisease; or (3) diabetes mellitus and at least one riskfactor for coronary artery disease including hypertension,increased total cholesterol, smoking and micro-albuminuria. The patients were randomized to ramipril10 mg daily or placebo. The primary outcome was acombined endpoint of myocardial infarction, stroke orcardiovascular death. The mean follow up was five years.

Patients treated with ramipril had a 14% event rate ofthe combined morbidity and mortality endpoint whereasplacebo-treated patients had a 17.8% event rate. The 21%decrease in events was seen in all pre-specified groups,indicating that ACE inhibitor therapy with ramiprilsignificantly reduces morbidity and mortality in a high-

risk population with normal left ventricular function.Based on this study all patients with coronary arterydisease, cerebrovascular disease, peripheral vasculardisease and diabetes mellitus plus one other risk factor forcoronary artery disease should be prescribed an ACEinhibitor, specifically ramipril.

The patient should remain on aspirin for life and takeclopidogrel for a year following deployment of a stent.The CURE study showed that aspirin and clopidogreltogether were associated with a lower incidence ofmyocardial infarction and death in patients with unstableangina and non-ST elevation myocardial infarctioncompared with aspirin alone for up to a year.

The patient no longer has subjective or objectiveevidence of myocardial ischaemia, and in the absence ofhypertension or left ventricular dysfunction there is noindication for a beta-blocker.

Nitrates do not alter prognosis in coronary arterydisease. There is no evidence as yet that angiotensinreceptor blockers improve cardiovascular prognosis inpatients with coronary artery disease in the absence ofhypertension or left ventricular dysfunction.

16

Answer 11

b. Ramipril.

The high Hb is suggestive of polycythaemia. There isnothing in the history to indicate a secondary cause, e.g.hypoxia, renal carcinoma, adrenal tumour. Although hewas obese, there was nothing else in the history to allowthe diagnosis of obstructive sleep apnoea.

The high white cell count and platelet count favourprimary polycythaemia (polycythaemia rubra vera).Headache and lethargy are common symptoms ofpolycythaemia rubra vera. Polycythaemia rubra veracauses lethargy due to hyperviscosity and raisedinterleukin-6 levels. Other classic features include visualdisturbance, abdominal pain and pruritus.

Many patients with polycythaemia rubra vera havesplenomegaly; however, a palpable spleen is absent inapproximately one third of patients.

Answer 12

c. Polycythaemia rubra vera.

Criteria for the diagnosis of polycythaemia rubravera

Raised red cell mass and normal pO2 with eithersplenomegaly or two of the following:• WCC >12 � 109/l• Platelets >400 � 109/l• Raised B12 binding protein • Low neutrophil alkaline phosphatase

concentration

(See Questions 39, 73 and 211.)

Page 18: Rapid review of clinical medicine for mrcp part 2

An 18-year-old male was admitted with sudden sharppain in the left infrascapular area. He was not breathlesson mild exertion. He was usually fit and well. He was anoccasional smoker. There was no history of respiratoryproblems. On examination there was reduced air entry atthe left lung base. The oxygen saturation on air was 96%.The CXR revealed a left-sided pneumothorax. There wasless than 2 cm rim of air between the edge of the lungand the ribs.

Clinical Cases 17

Question 14

What is the management?a. Admit and observe for 24 hours.b. Attempt aspiration of pneumothorax.c. Prescribe 100% oxygen for a few hours.d. Insert chest drain.e. Allow home and repeat CXR after a week.

The ECG below was taken from a young boy whoexperienced syncope. On examination he had a systolicmurmur.

Question 13

What is the most probable underlying diagnosis?a. Coarctation of the aorta.b. Dextrocardia.c. Pulmonary stenosis.d. Wolff–Parkinson–White syndrome.e. Hypertrophic cardiomyopathy.

1133

Page 19: Rapid review of clinical medicine for mrcp part 2

18

The question tests knowledge of the guidelines for themanagement of pneumothorax set by the BritishThoracic Society.

The patient has a relatively small pneumothorax (<2 cm rim of air between lung and ribs) with minimalsymptoms and can walk slowly without becomingbreathless. There is no history to suggest chronic lungdisease. In such a case no treatment is recommended andthe patient may be discharged. Patients are advised not toover-exert themselves and to return if they developbreathlessness. A repeat CXR is recommended after aweek to ensure that the pneumothorax has resolved.

If the patient has a pneumothorax >2 cm rim of airbetween the lung and the chest wall on the CXR, or haspain or dyspnoea at rest or on minimal exertion thenaspiration is recommended. If aspiration is successful thepatient is allowed home and reviewed with repeat CXR inone week. If aspiration is unsuccessful a second attempt ismade at aspiration. If the lung still remains deflated theninsertion of a chest drain is recommended.

In patients with chronic lung disease the followingcriteria should be used to decide whether aspiration orinsertion of a chest drain is the first procedure of choice.Patients aged <50 years, who are relatively asymptomaticand have a small pneumothorax, should be aspirated andobserved in hospital for 24 hours (assuming aspiration issuccessful). If aspiration is unsuccessful in this group ofpatients then insertion of a chest drain is advised. Inpatients aged >50 years, with symptoms and with largerpneumothoraces (>2 cm air between lung and chest wall)a chest drain is necessary.

Answer 14

e. Allow home and repeat CXR after a week.

The patient has a systolic murmur. The ECG showsright axis deviation, a dominant R wave in V1 andrelatively prominent S waves in V5 and V6. The sum ofthe R in V1 and in V6 is > 1.25 mV which indicates right

ventricular hypertrophy. The answer that would fit withall the information is pulmonary stenosis. Coarctation ofthe aorta and hypertophic cardiomyopathy are associatedwith left ventricular hypertrophy. The absence of a shortPR interval and delta waves are against the diagnosis ofWPW syndrome.

Answer 13

c. Pulmonary stenosis.

Spontaneouspneumothorax

< 2cm rim of air on CXRMinimal symptoms

Yes No

Allow homeRepeat CXR in

7–10 days

Aspirate

SuccessfulIf unsuccessful,

repeat aspiration.If still unsuccessful, insert chest drain

Management of pneumothorax

Page 20: Rapid review of clinical medicine for mrcp part 2

Clinical Cases 19

A 44-year-old was seen in the rheumatology clinic inDecember complaining of malaise, joint pains andtingling in the hands and feet. She had been diagnosed ashaving Raynaud’s phenomenon several years ago. Thepatient had consulted several doctors for intermittentmalaise and joint pains. There was no history of nightsweats, dyspnoea, or problems with swallowing. Thepatient took paracetamol on a PRN basis for joint pains.

On examination she had palpable purpura on thethighs and arms. There was no obvious evidence of jointswelling. Abdominal examination revealed hepatomegalypalpable 3 cm below the costal margin. Neurologicalexamination revealed decreased sensation in the handsand feet. The blood pressure was 110/80 mmHg.

Investigations are shown.

Hb 10 g/dlWCC 9 � 109/lPlatelets 490 � 109/lESR 90 mm/hSodium 139 mmol/lPotassium 4.2 mmol/lUrea 9 mol/lCreatinine 140 �mol/lBilirubin 15 mmol/lAST 90 iu/lAlkaline phosphatase 122 iu/lAlbumin 33 g/lRheumatoid factor IgM Positive (titre 1/640)C3 0.2 g/l (NR 0.55–1.2 g/l)C4 0.09 g/l (NR 0.2–0.5 g/l)Hep C Virus AB PositiveHep B sAg NegativeUrinalysis Blood +

Protein ++

Question 15

Question 16

A 30-year-old businessman developed sudden onset offever, sore throat, diarrhoea and myalgia. Over the nextthree days he noticed a widespread rash affecting his face,trunk, palms and soles. He was usually fit and well andhad only consulted his GP once in the past 10 years for atyphoid vaccine before travelling to India. Over the pastfour months he had established business links with acompany in Thailand and had visited the country onthree occasions. His last visit to Thailand was eight weekspreviously. He was married with two young children. Hewas not taking any medications and had no history ofdrug allergy.

On examination his temperature was 38.6°C. Therewas cervical lymphadenopathy. Inspection of the oralcavity revealed several painful ulcers affecting the tongue.The pharynx was oedematous and red with minimaltonsillar exudates. The chest was clear. Abdominalexamination was normal.

Investigations are shown.

What is the best management of the patient’s illness?a. Prednisolone.b. Cyclophosphamide.c. Chlorambucil.d. Pegylated interferon-� plus ribavarin.e. Plasmapharesis.

What is the diagnosis?a. Acute HIV infection.b. Secondary syphilis.c. Acute hepatitis infection.d. Infectious mononucleosis.e. Acute CMV infection.

Hb 13 g/dlWCC 11 � 109/l

(neutrophils 6 � 109/l,lymphocytes 4 � 109/l)

Platelets 130 � 109/lMonspot test NegativeSodium 135 mmol/lPotassium 3.8 mmol/lUrea 6 mmol/lCreatinine 80 �mol/lBilirubin 23 �mol/lALT 45 iu/lAST 49 iu/l

Page 21: Rapid review of clinical medicine for mrcp part 2

20

The main differential diagnosis is between infectiousmononucleosis, CMV infection and acute HIV infection.All three are associated with sore throat, rash, fever andatypical lymphocytes. Mouth ulcers are usually absent in

EBV and CMV infection. Furthermore the rash ininfectious mononucleosis is usually an idiosyncraticreaction to ampicillin whereas it is part of HIVseroconverson. The main clinical features differentiatinginfectious mononucleosis from acute HIV infection aretabulated below. The rash in CMV infection usually sparesthe palms and soles. (See Question 325.)

Answer 16

a. Acute HIV infection.

Differentiation between infectious mononucleosis and acute HIV infection

Parameter Infections mononucleosis HIV infectionOnset of symptoms Over a few days AbruptMouth ulcers Absent usually Often presentRash Usually secondary to ampicillin Part of HIV seroconversionDiarrhoea Unusual CommonTonsillar exudates Prominent MildWhite cell count May be elevated Elevated or suppressedAtypical lymphocytes Frequent (90%) and numerous Present in 50%Transaminitis Common CommonThrombocytopenia Common Common

This is a difficult question; however, the clue lies in thefact that the patient has evidence of current or previousinfection with hepatitis virus and has Raynaud’sphenomenon, palpable purpura (vasculitis), neuropathyand hypocomplementaemia. The diagnosis is consistent

with mixed essential cryoglobulinaemia. Cryoglobulinsare immunoglobulins that precipitate in the cold. Theyare associated with auto-immune haemolysis, Raynaud’sdisease (in severe cases they can cause acronecrosis),vasculitis, peripheral neuropathy, glomerulonephritis andhepatosplenomegaly. Complement is reduced. HCV isthought to play an aetiological role in the development intype II and type III cryoglobulinaemia.

Answer 15

d. Pegylated interferon-� plus ribavarin.

Types of cryoglobulinaemia

Type Immunoglobulins Associated condition(s)I Monoclonal immunoglobulin Multiple myeloma

Waldenstrom’s macroglobulinaemiaII Polyclonal IgG and monoclonal Hepatitis C and hepatitis B

rheumatoid factor IgMIII Mixed IgG and polyclonal Chronic inflammation

rheumatoid factor Hepatitis CLymphoproliferative disease

The diagnosis is based upon history, skin biopsy (ifpurpura present), hypocomplementaemia and presence ofcryoglobulins. Investigation for cryoglobulinaemiashould always include serology for hepatitis C infection. Treatment for acute cryoglobulinaemia causing severerenal impairment or acronecrosis is plasmapharesis,though in less acute situations prednisolone and

cyclophosphamide are effective. Chlorambucil has alsobeen used with success. When cryoglobulinaemia issecondary to HCV infection, the treatment of choiceincludes the combination of pegylated interferon-a andribavarin. Ribavarin should be used with caution inpatients with renal failure.

Page 22: Rapid review of clinical medicine for mrcp part 2

Clinical Cases 21

A 69-year-old woman with rheumatoid arthritispresented with swollen ankles. She was diagnosed ashaving rheumatoid arthritis over 18 years ago and hadbeen relatively well controlled on non-steroidal anti-inflammatory drugs until six months ago, when her jointpains and swelling required the addition of penicillamineto control her symptoms. The patient had a past historyof hypertension, for which she took bendroflumethiazide.

On examination she had symmetrical joint deformitiesconsistent with rheumatoid arthritis. The heart rate was90 beats/min and irregular. Her blood pressuremeasured 140/90 mmHg. The JVP was not raised. Both

heart sounds were normal and the chest was clear.Abdominal examination was normal. Inspection of thelower limbs revealed pitting oedema.

Investigations are shown.

Hb 11 g/dlWCC 5 � 109/lPlatelets 190 �109/lSodium 134 mmol/lPotassium 4.5 mmol/lUrea 6 mmol/lCreatinine 70 �mol/lBilirubin 11�mol/lAlkaline phosphatase 100 iu/lAlbumin 26 g/lUrinalysis Protein ���

Question 17

What is the management?a. Stop penicillamine.b. Start prednisolone.c. Start ACE inhibitor therapy.d. Arrange renal biopsy.e. Arrange IVU.

A 59-year-old female presented with weakness of bothlegs. An MRI scan of the spine is shown (18).

Question 18

What is the cause of her symptoms?a. Syringomyelia.b. Paravertebral abscess.c. Thoracic disc prolapse.d. Metastatic spinal cord compression.e. Extradural meningioma.

1188

Page 23: Rapid review of clinical medicine for mrcp part 2

22

The patient has heavy proteinuria and gives a relativelyrecent history of onset of swollen ankles shortly afterstarting penicillamine. The most likely diagnosis ispenicillamine-induced membranous nephropathy, whichusually occurs within 6–12 months of the initiation ofdrug therapy. Proteinuria resolves in virtually all casesafter stopping the drug but this may take several months.Other causes of heavy proteinuria secondary tomembranous nephropathy in rheumatoid arthritis includegold therapy.

Renal amyloidosis is a recognized cause of heavyproteinuria complicating chronic rheumatoid arthritis.While it is possible that the patient may have renalamyloidosis, the relationship of the proteinuria to the

initiation of penicillamine points to a drug-inducedmembranous nephropathy.

Other causes of renal disease in rheumatoid arthritisinclude analgesic nephropathy, focal segmentalglomerulonephritis and rheumatoid vasculitis. All arecharacterized by blood in the urine. Analgesicnephropathy is usually secondary to non-steroidal anti-inflammatory drugs and paracetamol. The proteinuria israrely severe enough to cause nephrotic syndrome. Focalsegmental glomerulonephritis is rare and is excluded bythe absence of red cells in the urine. Rheumatoidvasculitis has a predilection for skin and the peripheralnervous system but in very rare circumstances may affectthe kidneys. It is more likely in patients with severedisease, nodule formation, high titres of rheumatoidfactor and hypocomplementaemia. (See Question 320.)

Answer 17

a. Stop penicillamine.

Answer 18

This is a T2 weighted image that shows evidence of cordcompression from a collapsed thoracic vertebra. The

vertebra in question is infiltrated by tumour and appearswhite. The vertebra above are also infiltrated withtumour (appear white). The vertebra below the collapsedvertebra appear normal (black).

d. Metastatic spinal cord compression.

Interpretation of MRI Scans

Substance T1 weighted T2 weighted

Water/vitreous/CSF black light grey or whiteFat white light greyMuscle grey greyAir black blackFatty bone marrow white light greyBrain white matter light grey greyBrain grey matter grey very light grey

T1 Weighted Imaging

Provides anatomical information

Low signal – Black• Cortical bone• Air• Rapidly flowing blood• CSF

Intermediate signal – Grey• Grey matter is darker than white matter

High signal – White• Fat in bone, scalp and orbit

T2 Weighted Imaging

Provides pathological information

Low signal – Black• Cortical bone• Air• Rapidly flowing blood• Haemosiderin

Intermediate signal – Grey• White matter is darker than grey matter

High signal – White• CSF or water

Page 24: Rapid review of clinical medicine for mrcp part 2

Clinical Cases 23

A 16-year-old girl presented with intermittent episodes oflower colicky abdominal pain for six months. In theinterim she had lost almost 6.4 kg in weight. Herappetite was not impaired. There was no history ofdiarrhoea, although the patient had complained ofintermittent constipation and abdominal bloating. Thepatient was English in origin. She had no family historyof note. She had last travelled abroad to Barbados onholiday a year ago. The only other past medical historyincluded a short episode of painful ankles associated withcircular erythematous skin lesions.

On examination she was thin and mildly clubbed. Theheart rate was 90 beats/min and regular. The bloodpressure measured 100/55 mmHg. There was evidence

of a BCG scar on inspection of the left upper arm. Bothheart sounds were normal and the chest was clear.Abdominal examination revealed vague tendernessaffecting the hypogastrum and right iliac fossa.

Investigations are shown.

Question 19

Hb 10 g/dlWCC 11 � 109/lPlatelets 498 � 109/lESR 55 mm/hU&E NormalAST 20 iu/lALT 22 iu/lBilirubin 12 �mol/lAlbumin 33 g/lStool culture NegativeChest X-ray Minor calcification, a few

perihilar nodes

What is the diagnosis?a. Sarcoidosis.b. Intestinal lymphoma.c. Intestinal tuberculosis.d. Crohn’s disease.e. Irritable bowel syndrome.

A 24-year-old patient was admitted to hospital with acuteasthma for the fourth time in the past six years. Theasthma was usually precipitated by a coryzal illness orexposure to allergens. There was no other past medicalhistory of note. The patient usually inhaled ventolin asrequired, salmeterol inhaler twice daily, becotide inhalertwice daily and had recently been prescribedaminophylline 450 mg twice daily.

On admission she had a bilateral wheeze. The PEFRwas 200 l/min. The oxygen saturation on air was 86%and on 28% oxygen it was 94%. The chest X-ray revealedhyperinflated lungs. The patient was commenced onnebulized bronchodilators, prednisolone 30 mg daily andamoxycillin. The following day she developed a rashtherefore the amoxycill in was substituted witherythromycin.

The patient improved significantly over the next 48hours but then suffered three successive grand malseizures, which necessitated ventilation.

Question 20

What was the most likely cause of the epilepticseizures?

a. Hypoxia.b. Meningitis.c. Benign intracranial hypertension.d. Theophylline toxicity.e. Herpes encephalitis.

Page 25: Rapid review of clinical medicine for mrcp part 2

24

Abdominal cramps, weight loss, erythema nodosum(raised circular skin lesions) and raised inflammatorymarkers are highly suggestive of inflammatory boweldisease. Tenderness in the right iliac fossa points to thepossibility of terminal ileal disease and hence Crohn’sdisease, although this is a non-specific feature since manyconditions may cause right iliac fossa tenderness.Diarrhoea is not always a prominent feature in Crohn’sdisease.

Although ileo-caecal TB may present in a similar fashion,her race and the presence of a BCG scar is against thediagnosis. Sarcoidosis enteropathy has been reported butthis is very rare and usually in association with otherfeatures of this multi-system disorder. Small bowellymphoma may present in a similar fashion; however,diarrhoea is a prominent feature. Raised inflammatorymarkers are against the diagnosis of irritable boweldisease, which is a functional rather than inflammatorydisorder. (See Answers 31, 394.)

Answer 19

d. Crohn’s disease.

The question tests the candidate’s knowledge aboutdrugs interacting with aminophylline and inhibiting itsmetabolism. With respect to the treatment of lowerrespiratory tract infections, both quinolone andmacrolide antibiotics (e.g. ciprofloxacin, erythromycinrespectively) inhibit aminophylline metabolism.

Features of theophylline toxicity include nausea,vomiting, hypotension, cardiac arrhythmias and seizures.Other drugs that inhibit the metabolism of theophyllineinclude cimetidine, propranolol, allopurinol,thiobendazole and the contraceptive pill. In the context

of asthma, hypokalaemia (sometimes a consequence ofnebulized salbutamol) is also associated with theophyllinetoxicity.

Symptoms do not usually occur until plasmatheophylline concentrations exceed 20 mg/l. The mostadverse effects of theophylline toxicity, such as cardiacarrhythmias and seizures, generally occur at plasmatheophylline levels >40 mg/l.

The management of theophylline toxicity is usuallysupportive. In patients who have taken an overdose, theaim is to prevent absorption in the stomach. There arethree main strategies in the management of theophyllinetoxicity (shown below):

Answer 20

d. Theophylline toxicity.

Strategy 1 (if patient is stable)• Gastric lavage followed by oral activated charcoal administration is effective.

Strategy 2• Treat arrhythmias with beta-blockers; unfortunately many patients taking theophylline for therapeutic reasons

have contraindications to beta-blockers. In these patients lignocaine may be used for ventricular arrhythmiasand verapamil for supraventricular arrhythmias including atrial fibrillation.

• Treat seizures with diazepam or barbiturates; phenytoin is not very effective.

Strategy 3 (rarely required)• Haemodialysis is very effective in treating life-threatening toxicity, i.e. patients with a plasma theophylline

level of >100 mg/l who have profound hypotension, fatal cardiac arrhythmias and seizures. Age andconcomitant hepatic disease are important factors in relation to prognosis with theophylline toxicity. Patientsaged >60 years with liver disease may be dialysed at theophylline levels of around 60 mg/l.

Page 26: Rapid review of clinical medicine for mrcp part 2

Clinical Cases 25

A 64-year-old Asian man presented with a six-weekhistory of dyspnoea and wheeze. For two weeks he hadalso developed a cough productive of yellow sputum andfever. There was no history of night sweats. The patienthad not travelled abroad for over 20 years.

Investigations are shown.

Question 21

Hb 13 g/dlWCC 11 � 109/l

(neutrophils 8 � 109/l,lymphocytes 1 � 10 9/leosinophils 2 � 109/l)

Platelets 258 � 109/lESR 30 mm/hBiochemistry NormalChest X-ray Diffuse perihilar infiltratesSputum culture Negative

What is the most probable diagnosis?a. Churg–Strauss syndrome.b. Tuberculosis.c. Allergic bronchopulmonary aspergillosis.d. Tropical pulmonary eosinophilia.e. Asthma.

A 78-year-old patient presented with sudden onset ofsevere breathlessness. He had a history of ischaemic heartdisease and had suffered two myocardial infarctions in thepast three years. He had an 11-year history ofhypertension that had been well controlled. He was anon-smoker. His medication consisted of aspirin,ramipril, atenolol, bendroflumethiazide and simvastatin.

On examination he had a heart rate of 146 beats/min.The pulse was irregular. The blood pressure was100/68 mmHg. Both heart sounds were quiet.Auscultation of the lungs revealed widespread inspiratorycrackle and expiratory wheeze. The ECG showed atrial

fibrillation with a rapid ventricular rate and q waves in theanterior leads.

Question 22

What is the safest drug for restoring sinus rhythm?a. IV digoxin.b. IV amiodarone.c. IV flecanide.d. IV esmolol.e. IV dofetolide.

Question 23

An 81-year-old man with non-insulin-dependent diabetesmellitus was found unconscious by his carer. Blood testsperformed on admission to hospital are shown.

Sodium 153 mmol/lPotassium 5.4 mmol/lUrea 40 mmol/lCreatinine 310 �mol/lGlucose 60 mmol/l

What is the best combination of infusions in themanagement of the patient?

a. IV saline (0.45%), IV insulin and subcutaneous heparin.

b. IV saline (0.9%), IV insulin and subcutaneous heparin.

c. IV sodium bicarbonate, IV insulin and subcutaneous heparin.

d. IV dextrose saline, IV insulin and subcutaneous heparin.

e. IV dextrose (5%), IV insulin and subcutaneous heparin.

Page 27: Rapid review of clinical medicine for mrcp part 2

26

The history of cough sputum, eosinophilia and perihilarinfiltrates is most consistent with allergic broncho -pulmonary aspergillosis in the context of the historygiven. There is no drug history to indicate aneosinophilic pneumonitis, nor a history of travel to thetropics to suggest tropical pulmonary eosinophilia.Churg–Strauss syndrome is unlikely in the absence ofvasculitis, neuropathy or renal involvement. Asthma doesnot cause pulmonary infiltrates. Tuberculosis does notusually cause eosinophilia.

The diagnosis of allergic bronchopulmonaryeosinophilia is made in patients with asthma, proximal

bronchiectasis and parenchymal infiltrates in the perihilararea. The presence of high titres of IgE and IgGantibodies and a positive hypersensitivity skin test toAspergillus fumigatus testing confirm the diagnosis.Treatment is with a prolonged course of itraconazole.

Tropical pulmonary eosinophila is an immune reactionto infection with the human filarial parasites Wucheriabancrofti and Brugia malayi. It is characterized by a non-productive cough, wheeze, fever, weight loss,lymphadenopathy, eosinophilia and patchy infiltrates onthe chest X-ray. The condition occurs in patients infectedin the tropics. The worm is rarely identified but thecondition responds to diethycarbamazine, the drugnormally used to treat filariasis. (See Question 396.)

Answer 21

c. Allergic bronchopulmonary aspergillosis.

The patient has rapid atrial fibrillation in the context ofischaemic heart disease and has evidence of pulmonaryoedema. One has to assume that left ventricular functionis impaired to answer this question since it is highlyunlikely that a heart rate of 146 beats/min would causeleft ventricular failure in a patient with normal leftventricular function. Amiodarone, dofetolide andflecainide are capable of restoring sinus rhythm. Of theseamiodarone is the least negatively inotropic.

Flecainide is relatively contraindicated in patients withknown coronary artery disease. Dofetolide is a class III

anti-arrhythmic agent that is effective at restoring sinusrhythm in patients with persistent AF (up to seven days)or more permanent AF. Dofetolide is less negativelyinotropic than many other drugs that may be effective inrestoring sinus rhythm such as propafenone (class IC),quinidine, disopyramide (class IA) and sotalol (class III),but experience regarding its use in the UK is relativelylimited. Digoxin is effective in controlling ventricular ratein AF but does not restore sinus rhythm. Esmolol is avery short-acting beta-blocker (class II antiarrhythmicagent) that is not useful at restoring sinus rhythm.

Answer 22

b. IV amiodarone.

The patient has a hyperosmolar non-ketotic diabeticcoma (HONK). The fluid of choice is saline. Thestrength of saline used initially is always 0.9% since it iseffective at restoring volume and has a lower risk ofcausing large drops in plasma osmolality, a risk factor for

the development of cerebral oedema. If despite adequatehydration, the sodium remains >150 mmol/l someauthorities advocate switching to half-strength saline(0.45%). The patient has severe dehydration creating ahyperviscosity state that may predispose him to arterialand venous thromboses. Heparin therapy is mandatory toprevent such complications during the management ofHONK. (See Question 84.)

Answer 23

b. IV saline (0.9%), IV insulin and subcutaneous heparin.

Page 28: Rapid review of clinical medicine for mrcp part 2

Clinical Cases 27

A 13-year-old girl was admitted with a two-day history oflower abdominal pain and blood-stained diarrhoea. Threedays later, she developed pains in her ankles and rightelbow and felt nauseous. Positive findings onexamination were a purpuric rash affecting the arms andlegs, periorbital oedema and a blood pressure of150/95 mmHg.

Investigations are shown.

Question 24

Hb 10 g/dlWCC 12 � 109/lPlatelets 136 � 109/lMCV 70 flESR 35 mm/hPT 13 s (control 13 s)APTT 34 s (control 36 s)Sodium 138 mmol/lPotassium 5.9 mmol/lCreatinine 130 �mol/lUrinalysis Blood ++

Protein ++

2: List two investigations that would be most useful in confirming the diagnosis.a. Skin biopsy.b. Renal biopsy.c. Blood cultures.d. TT.e. Serum fibrinogen level.f. Serum IgA level.g. Serum ANF level.h. Serum ANCA.i. Blood film.j. 24-hour urine collection for protein.

Question 25

A 39-year-old African male was referred to the bloodpressure unit with persistent blood pressure readings of140–150/90–95 mmHg over the past six months. Hewas a non-smoker and consumed 4 units of alcohol perweek. The patient weighed 89 kg and measured 1.7 m.Physical examination was normal with the exception of ablood pressure reading of 150/92 mmHg.

Investigations are shown.

What is the best initial management for the raisedblood pressure?

a. Beta-blocker.b. Angiotensin-converting enzyme inhibitor.c. Low-salt diet, regular exercise.d. Calcium channel antagonist.e. Thiazide diuretic.

Sodium 136 mmol/lPotassium 4.2 mmol/lUrea 5 mmol/lGlucose 4.1 mmol/lTotal cholesterol 4.1 mmol/lTriglycerides 1.2 mmol/l12-lead ECG Sinus rhythm

Right axis deviationUrinalysis Protein 0

Blood 0Cells 0

1: What is the most probable diagnosis?a. Haemolytic–uraemic syndrome.b. IgA nephritis.c. Henoch–Schönlein purpura.d. Systemic lupus erythematosus.e. Polyarteritis nodosa.

Page 29: Rapid review of clinical medicine for mrcp part 2

28

The combination of lower abdominal pain, bloodydiarrhoea, purpuric rash and nephritis in a young girlare highly suggestive of Henoch–Schönlein purpura.The condition is a small-vessel vasculitis that occursmost commonly in children aged 4–15 years. It ischaracterized by gastrointestinal symptoms whichcomprise abdominal pain, diarrhoea, and rectalbleeding, flitting arthralgia affecting large joints, apurpuric rash characteristically affecting the lower limbsand buttocks and an acute nephritis. Complicationsinclude intestinal perforation, haemorrhage andintussusception and acute renal failure.

The diagnosis is usually clinical; however, tissuediagnosis is possible with skin or renal biopsy. Skinbiopsy demonstrates a leucoclastic vasculitis with IgAdeposit ion. Renal biopsy reveals mesangial IgAdeposition associated with a glomerulonephritis. Renalhistology is indistinguishable from IgA nephropathy. Inthis case, more marks are given to skin biopsy because itis safer and more practical than renal biopsy. Serum IgAlevels are depressed in approximately 50% of cases. Themanagement is usually supportive, although there maybe a role for methylprednisolone in cases of acutecrescentic nephritis.

The condition must not be confused with thehaemolytic uraemic syndrome (discussed in Answer152), which is also characterized by diarrhoea and renalfailure.

Answer 24

1: c. Henoch–Schönlein purpura.2: a. Skin biopsy.

b. Renal biopsy.

The patient is young and has mild hypertension onpresentation. He does not have any other risk factors forcardiovascular disease or evidence of secondary end-organ damage as a result of the raised blood pressure.In this particular case the initial management planshould include a low-salt diet, regular exercise andweight loss. The patient should be observed carefullyfor up to a year and should only be commenced onpharmacological therapy if the blood pressure remainsabove 140/85 mmHg.

If treatment is indicated after a year, the drugs ofchoice are thiazide diuretics or calcium channelblockers. Angiotensin-converting enzyme inhibitors andbeta-blockers are not part icularly effective asmonotherapy because both drugs act by suppressingrenin levels, which are already relatively low in Afro-

Caribbean patients. However, these patients mayrespond to ACE inhibitors and beta-blockers whenprescribed with drugs that activate the renin–angiotensin–aldosterone system, i.e. thiazide diureticsand calcium channel blockers.

Both lifestyle modification and pharmacologicaltherapy would be indicated if the patient had a bloodpressure ≥160/100 mmHg, or evidence of secondaryend-organ damage, or other risk factors for coronaryartery disease at presentation.

There is a high prevalence of hypertension inindividuals of Afro-Caribbean origin, with almost 50%of patients over the age of 40 years being affected. Thisparticular group of patients generally develop hyper -tension at a younger age and are at higher risk ofhypertensive complications such as stroke, heart failureand renal failure than Caucasian patients. Hypertensionin Afro-Caribbean patients is salt sensitive and respondswell to a low-salt diet.

Answer 25

c. Low-salt diet, regular exercise.

Page 30: Rapid review of clinical medicine for mrcp part 2

Clinical Cases 29

Question 26

A 41-year-old male was admitted to hospital with acuteconfusion. He had been generally unwell for two days. Aworried neighbour looked through his letter box when hefailed to answer the doorbell, and found him lying on thefloor. There was no other history of note.

On examination, he was confused. He had a wide -spread rash (26a). His left eye is shown (26b). The heartrate was 120 beats/min; the blood pressure wasunrecordable. There was no evidence of nuchal rigidity,and Kernig’s sign was negative. There was no focalneurological deficit. Examination of the cardiovascular,respiratory and gastrointestinal tract was normal.

Investigations are shown.Hb 11 g/dlWCC 24 � 109/lPlatelets 30 � 109/lSodium 135 mmol/lPotassium 5 mmol/lCreatinine 156 μmol/lPT 48 s (control 13 s) Fibrinogen degradation >25,000

products

A 36-year-old woman is seen in the Accident andEmergency Department after having taken 40 paracetamoltablets with a quarter-bottle of vodka six hours earlier,following an argument with her husband. She wasnauseous, but had not vomited. There was no past medicalhistory of note. A physical examination was normal.

Investigations are shown.

Sodium 138 mmol/lPotassium 3.9 mmol/lUrea 5.1 mmol/lBicarbonate 18 mmol/lPT 15 s (control 15 s)Serum paracetamol 200 mg/l

level1. List two immediate management steps from thefollowing list:

a. Gastric lavage.b. IV dextrose 5%.c. Oral activated charcoal.d. Fresh-frozen plasma.e. IV sodium bicarbonate.f. IV N-acetyl cysteine.g. IV flumazenil.h. IV vitamin K.i. IV saline (0.9%).j. IV prochlorperazine.

2. Which one of the following investigations wouldyou perform the next day to assess her prognosis?

a. Serum potassium.b. Serum magnesium.c. Arterial pH.d. Serum paracetamol concentration.e. Serum aspartate transaminase.

Question 27

Which single investigation would you perform next?a. Blood cultures.b. CT scan brain.c. CSF for microscopy and culture.d. Chest X-ray.e. Throat swab for culture.

2266aa 2266bb

Page 31: Rapid review of clinical medicine for mrcp part 2

30

The eye demonstrates a conjunctival haemorrhage. Therash is a necrotic purpuric rash, which is typical ofmeningococcal septicaemia. The patient has septic shockand requires immediate therapy. The recognition that hehas meningococcal septicaemia is important for the choiceof antibiotics that you will use. In medical emergencies, thereader must be familiar with the drugs that are used in thatparticular emergency, but not necessarily the dosage, asthis can be found in the British National Formulary or theequivalent. Although most Neisseria meningitidis strainsare sensitive to benzyl penicillin, it is prudent to cover thepatient with additional cephalosporin or aminoglycosideantibiotic therapy until the sensitivities of the organism areknown. The circulation must be restored to preventhypoperfusion of vital organs, particularly the kidneys. Thepresence of low platelets, high fibrinogen degradationproducts and abnormal clotting is suggestive of DIC,which should be treated with fresh-frozen plasma toprevent haemorrhage.

Neisseria meningitidis, the causal Gram-negativediplococcus, can be cultured from the CSF in over 80% ofcases with evidence of neurological involvement. However,

the presence of very low levels of platelets and DIC iscontraindicated because of the dangers of bleeding into thespinal canal, particularly because the yield is just as highfrom nasal swabs. It is also possible to isolate themeningococcal antigen from blood before blood cultureresults are available. This test is particularly useful ifantibiotics have been given before the patient is brought tohospital (negative blood cultures).

Meningococcal meningitis and septicaemia are causedby serogroups B and C. Septicaemia is associated withwidespread petechial haemorrhage. Conjunctivalhaemorrhage may be the first physical manifestation. Shockis common owing to the production of a circulatingendotoxin. DIC is a commonly recognized complicationthat may result in adrenal haemorrhage(Waterhouse–Friderichsen syndrome). Meningitis is oftencharacterized by a myalgia, headache, photophobia, neckstiffness, nausea and vomiting. In the absence of DIC, thediagnosis is made rapidly by performing a Gram stain onthe CSF. Blood cultures are positive in the majority ofpatients with meningitis. Focal neurological signs are lesscommon than in pneumococcal meningitis.

Note: individuals in contact with affected patients mustreceive rifampicin chemoprophylaxis.

Answer 26

The drug should be given within 8–10 hours of ingestionof the overdose, and continued while the liver function isabnormal. It is useful because it replenishes cellularglutathione stores and reduces oxidative damage causedby the toxic metabolite, NAPQI. An alternative to this ismethionine. Gastric lavage is useful if performed within 1hour of the overdose.

The patient has taken 20 g of paracetamol. Aningestion of 15 g is considered potentially serious in mostpatients. The toxicity of paracetamol is related to theproduction of a toxic metabolite of paracetamol. This isNAPQI, which usually is immediately conjugated withglutathione and excreted. In paracetamol overdose, thetoxic metabolite is produced in excess and depletes

cellular glutathione. The liver is unable to deactivateNAPQI, which is responsible for massive hepatic necrosisand hepatic failure. Patients may have nausea, anorexia orvomiting on the first day. After 72 hours, features of liverand renal failure may ensue.

The three most important prognostic markers inparacetamol overdose are serum creatinine concentration,arterial pH and prothrombin time. A rise in serumcreatinine level due to renal failure is a bad prognosticsign. A level of over 300 mmol/l is associated with over70% mortality. Systemic acidosis (due to the failure ofclearance of lactate by the liver) more than 24 hours afterthe overdose is associated with a poor prognosis. A pH ofbelow 7.3 is associated with only a 15% chance of survival.The PT is usually the first liver test to become abnormal.A PT of >20 s at 24 hours after overdose is suggestive ofsignificant hepatic damage, and a peak PT of >180 s isassociated with a 90% mortality.

1. c. Oral activated charcoal.f. IV N-acetyl cysteine.

2. c. Arterial pH.

Answer 27

a. Blood cultures.

Page 32: Rapid review of clinical medicine for mrcp part 2

Clinical Cases 31

A 52-year-old female was broughtinto the Accident and EmergencyDepartment after being foundcollapsed outside a public house.There was no one accompanyingher, and there was no informationregarding her next of kin.

On examination, she was verydrowsy and had a Glasgow comascore of 6 out of 15. Her pupilswere 10 mm each and reacted verysluggishly to light. On attempting toexamine her fundi, she was noted tohave coarse nystagmus, but a clearview of her fundi did notdemonstrate any abnormalities. The tone in all her limbswas increased and her reflexes were brisk. The plantarswere both upgoing. The heart rate was 135 beats/min,and regular. The blood pressure was 105/60 mmHg. Therespiratory rate was 20/min. Examination of theprecordium and lung fields was normal, but examination ofthe abdomen revealed a firm palpable mass 4 cm above thesymphysis pubis. The patient was catheterized and drainedof 2 litres of urine. Investigations are shown.

Shortly after the lumbar puncture, the patient had ageneralized seizure which lasted 30 s. The attending nurseraised concerns about an arrhythmia on the cardiacmonitor, and a 12-lead ECG was performed (28).

Question 28

Hb 14 g/dlWCC 12 � 109/lPlatelets 221 � 109/lMCV 88 flSodium 144 mmol/lPotassium 4.0 mmol/lUrea 13 mmol/lCreatinine 100 μmol/lBicarbonate 20 mmol/lChloride 108 mmol/lCalcium 2.4 mmol/lPhosphate 1.3 mmol/lBilirubin 12 μmol/lAST 33 iu/lGamma GT 28 iu/lAlkaline phosphatase 120 iu/lPlasma osmolality 333 mOsm/lUrine osmolality 120 mOsm/lBlood glucose 6 mmol/lChest X-ray Normal-sized heart and

clear lung fieldsSkull X-ray Normal; no fractures seenBrain CT scan NormalLumbar puncture:

CSF pressure 100 mmH2OCells 3/mm3

Protein 0.35 g/lGlucose 3 mmol/l

ECG Sinus tachycardia; right axis deviation

1. Calculate the plasma osmolality.2. Explain the discrepancy between the calculated

plasma osmolality and the measured plasma osmolality.

3. Give two possible explanations for the low urine osmolality.

4. What is shown on the ECG?5. What diagnosis best fits all the information given

above?6. What three investigations would you perform to

help in this patient’s management?

2288

Page 33: Rapid review of clinical medicine for mrcp part 2

32

Arrhythmias usually settle on correction of hypoxia andacidosis. Administration of class I antiarrhythmic agentsmay paradoxically worsen arrhythmias, with the exceptionof phenytoin. Status epilepticus should be corrected withintravenous diazepam.

Epileptic seizures and ventricular arrhythmias in apatient found collapsed should raise the suspicion oftricyclic antidepressant drug overdose. The low urineosmolality suggests that the patient has probably takenthe overdose together with alcohol, and it is possible thatshe may have also taken lithium. The normal CT scan ofthe brain and normal CSF are against pathology in thecentral nervous system. She has dilated pupils, which isagainst narcotic abuse. Indeed, the combination ofdilated pupils, tachycardia and urinary retention are allsuggestive of the anticholinergic side-effects of tricyclicantidepressants. Severe lithium toxicity is associated withseizures, coma and ventricular arrhythmias, but

anticholinergic effects are not a feature. In addition,lithium toxicity is associated with ataxia and dysarthria.Chronic lithium ingestion may cause hypothyroidism.Sodium-depleting drugs such as diuretics lead to excessabsorption of lithium by the kidney, and predispose totoxicity.

The arterial blood gases are an important investigationbecause they will identify hypoxia and acidosis, both ofwhich precipitate ventricular arrhythmias in patients withtricyclic antidepressant overdose. The serum lithium levelwill be useful to determine whether lithium has beeningested, and will help decide whether the patient shouldhave forced diuresis. In general, patients with a serumlithium of >3 mmol/l should have forced diuresis.Haemodialysis is recommended if serum lithium exceeds4 mmol/l.

The management of the patient is outlined below.

1. The plasma osmolality is calculated by the formula2 ([Na] + [K]) + [Urea] + [Glucose]. In this case,the calculated plasma osmolality is 315 mOsm/l.

2. The measured plasma osmolality is higher than thecalculated one, suggesting that the patient has ingested something which has not been measured, but has the effect of increasing the plasma osmolality. The most likely possibility in this case is alcohol ingestion. Although lithium contributes to plasma osmolality, it would be very unusual for the lithium concentration to be high enough to increase the plasma osmolality by 18 mOsm/l, considering that a serum lithium concentration of 2.5 mmol/l causes dangerous toxicity.

3. Nephrogenic diabetes insipidus from lithium therapy or inhibition of ADH secretion as a result of alcohol ingestion.

4. There is a broad-complex tachycardia with extreme axis deviation and concordance of the QRS complexes in the chest leads. These findings are suggestive of ventricular tachycardia.

5. Tricyclic antidepressant drug overdose with alcohol.

6. i. Arterial blood gases.ii. Serum lithium level.iii. Blood alcohol level.

Answer 28

The management of tricyclic antidepressant overdose

• Protect the airway, and give oxygen via a mask• Gastric lavage under anaesthetic supervision (within 12

hours of ingestion) followed by activated charcoal via anasogastric tube

• Monitor on a high-dependency unit• Correct hypoxia• Correct acidosis with IV sodium bicarbonate• Intravenous fluids to improve blood pressure• Epileptic seizures should be corrected with IV lorazepam

or diazepam. Phenytoin is contra-indicated • Ventricular arrhythmias respond to correction of acidosis

and hypoxia. IV sodium bicarbonate is the mainstay ofprevention and treatment of ventricular arrhythmias andshould be administered in all patients with ventriculartachycardia or acidosis or in patients with a QRS duration>110 msec

Page 34: Rapid review of clinical medicine for mrcp part 2

Clinical Cases 33

A 76-year-old male presented with a three-month historyof anorexia, weight loss and fever. Apart from sweatingexcessively at night and feeling very thirsty, he did nothave any other symptoms. He was a non-smoker and hadbeen a schoolteacher for 40 years before retiring.

On examination, he was thin. The finding oninspection of his hands is shown (29a). The heart ratewas 100 beats/min and the blood pressure 180/105mmHg. His temperature was 37.8°C (100°F).Examination of the cardiovascular system and therespiratory system was normal. Abdominal examinationrevealed minimal tenderness and some fullness in theright loin. Examination of the genitalia revealed someoedema of the scrotum. The lower limbs wereoedematous.

Investigations are shown.

Hb 18 g/dlWCC 10 � 109/lPlatelets 300 � 109/lESR 110 mm/hSodium 140 mmol/lPotassium 3.1 mmol/lCreatinine 120 μmol/lCalcium 2.6 mmol/lAlbumin 36 g/lChest X-ray (29b)CT scan of abdomen (29c)Urinalysis Blood +++

Protein +Bilirubin 0

1. Give two explanations for the serum calcium level.2. What is the most probable diagnosis?3. List three important tests you would perform to

help achieve a diagnosis.4. What is the management?

Question 29

2299aa 2299bb

2299cc

Page 35: Rapid review of clinical medicine for mrcp part 2

34

An 84-year-old female was referred to clinic with increasingforgetfulness. Her GP had commenced her on a small doseof haloperidol for agitation eight months ago. According tothe staff at the nursing home where she resided, she hadbecome increasingly confused over the past few monthsand more recently had developed odd movements affectingher face, arms and legs. Her GP had reviewed her twoweeks previously and stopped the haloperidol; however, sheremained confused and the movement disorder hadbecome much more pronounced. She was not taking anyother medication.

On examination, she had a mental test score of 4/10.Her vital parameters were normal. She exhibitedintermittent yawning motions of the mouth, withoccasional tongue protrusion. There were semi-purposeful

movements of her arms and legs. There was also clinicalevidence of increased tone and cogwheel rigidity onneurological examination of her limbs.

A CT scan of the brain revealed generalized cerebralatrophy and calcification of the basal ganglia.

This patient presents with anorexia, weight loss, and afever that may represent sepsis or malignancy. The rightloin tenderness and haematuria are suggestive of renalinvolvement. The chest X-ray reveals multiple opacities inboth lungs which represents a cannon-ball metastasesfrom the right kidney. CT scan of the abdomen reveals acarcinoma of the right kidney which is invading theinferior vena cava; hence the scrotal and lower-limboedema. Hypernephroma characteristically presents witha triad of haematuria, loin pain and swelling. Haematuriais present in 50% of cases, but pain and swelling are lessfrequent. Non-specific symptoms such as anorexia,weight loss and fatigue may be present for several

months before the diagnosis is made. The neoplastic cellsoften produce peptide hormones such as erythropoietin,renin, ADH and PTH-related peptide. This patient has arelative polycythaemia, hypercalcaemia, hypokalaemiaand hypertension, which reflect erythropoietin, PTH-related peptide and renin secretion, respectively. Fever ispresent in approximately 20% of patients and is probablysecondary to the secretion of a pyrogen by the tumour.Hypertension is present in approximately 30% of patients.Metastases usually occur via the bloodstream, althoughdirect invasion of the renal veins or the inferior vena cavais relatively common. Some 10% of hypernephromas arebilateral, so close attention is given to the contralateralkidney when reviewing the CT scan. Venography andarteriography allow assessment of invasion of the veinsand the vascularity of the tumour, respectively. Urinecytology may reveal malignant cells but the diagnosticyield is low. Removal of the hypernephroma (even whendistant metastases are present) improves survival andcauses regression of the metastases in many, but not all,patients. Radiotherapy and chemotherapy have been usedin the treatment of this tumour, but the results are notvery encouraging. The overall survival rate is 30–50%.

Answer 29

1. i. Bone metastases.ii. PTH-related peptide secretion from a right-sided

renal carcinoma.2. Right-sided renal carcinoma.3. i. Renal angiography.

ii. Renal venography and inferior vena cavogram.iii. Bone scan to detect bony metastases.

4. Surgical removal of the right kidney if his general health will allow.

What is the cause of her movement disorder?a. Multi-infarct dementia.b. Lewy body dementia.c. Extrapyramidal side-effects of haloperidol.d. Pseudohypoparathyroidism.e. Hypoparathyroidism.

Question 30

A 33-year-old Iranian male was investigated for a six-month history of general malaise, weight loss, fever, painin his knees, ankles and wrists and a sore mouth. Onsystematic enquiry, he gave a two-year history of arecurrent sore mouth that made it difficult for him to eat.Just before the onset of all his symptoms he hadexperienced an attack of abdominal pain and bloody

diarrhoea which resolved after a week. He was seen by agastroenterologist shortly afterwards, who diagnosed aninflammatory colitis, possibly secondary to infection. Arectal biopsy was performed by the gastroenterologist, andthis was reported as a non-specific colitis. The patient hadnever experienced any abdominal symptoms after this, buthad several episodes of soreness affecting the mouth. In

Question 31

Page 36: Rapid review of clinical medicine for mrcp part 2

Clinical Cases 35

addition, he developed painful eyes and pain onintercourse and on voiding urine. There was no history ofurethral discharge. Shortly afterwards he was admitted tohospital with a femoral vein thrombosis, which was treatedwith anticoagulants and thought to be secondary todehydration and immobility from his diarrhoeal illness.During the past six weeks his health had deteriorated. Hehad arthralgia and a fever. He had been married for fiveyears. He denied extramarital sex. His wife was well, andhad not experienced any similar symptoms.

On examination, the patient appeared unwell.

Examination of his oral cavity revealed an abnormality(31a). His eyes were sore (31b). He had submandibularlymphadenopathy. His ankle, wrist and knee joints weretender, and joint movements were restricted. In addition,he had painful lesions on his legs (31c). Examination ofhis genitalia and anal areas are shown (31d, e). He alsopointed out an erythematous lesion approximately 2 cm indiameter that had developed at the site of venepunctureduring a blood test performed by his GP two days earlier.All other aspects of the physical examination were normal.Investigations are shown.

3311aa 3311bb

3311cc 3311dd

3311ee

Hb 9.8 g/dlWCC 13 � 109/lPlatelets 450 � 109/lMCV 82 flCRP 200 g/lRheumatoid factor AbsentAntinuclear antibody Absent Radiology of painful joints Normal

What is the most probable diagnosis?a. Crohn’s disease.b. Ulcerative colitis.c. Reiter’s syndrome.d. Behçet’s syndrome.e. Gonococcal septicaemia.

Page 37: Rapid review of clinical medicine for mrcp part 2

36

The woman has clinical evidence of dyskinesia andparkinsonism. The most common cause of her neurologicalsigns is drug-induced extrapyramidal disease. Neurolepticdrugs which include haloperidol are extensively used intreating agitation in the elderly. By blocking dopaminereceptors in the basal ganglia, these drugs can offsetextrapyramidal side-effects which include tremor, dystonia,akathisia, parkinsonism and tardive dyskinesia. Acutedystonic reactions appear within the first few hours or days,and consist of oculogyric crises, torticollis or trismus.Fortunately, they are uncommon, and resolve as soon as thedrug is withdrawn. Chronic tardive dyskinesias are the mostserious complication and affect 20% of patients on chronicneuroleptic therapy. They usually occur after a patient hasbeen on treatment for at least three months, and can bemade worse in the first few weeks after stopping theoffending drug. In 60% of cases the dyskinesia resolves overthree years after drug withdrawal; however, in theremainder of patients the movement disorder persists and isvery difficult to treat. Characteristic features involve lipsmacking, tongue protrusion, orofacial mouthing, trunkrocking and distal chorea of the hands and feet. Acombination of any of these features may be present.

Cerebral calcification is an incidental finding in 0.5% ofCT scans in the elderly. About 20–30% of patients withwidespread calcification of the basal ganglia exhibit

neurological signs which include parkinsonism, chorea,epilepsy, ataxia and dementia. There is an associationbetween calcification of the basal ganglia andhypoparathyroidism or pseudohypoparathyroidism. Rarecauses of basal ganglia calcification (30, arrowed) arecerebral irradiation and mitochondrial diseases.

c. Extrapyramidal side-effects of haloperidol.

Answer 30

The patient has oral, genital and anal ulcers (31a, d and e,respectively), conjunctivitis (31b), arthritis, erythemanodosum (31c) and features of a systemic illness. There hasbeen a single episode of bloody diarrhoea and a previousfemoral vein thrombosis. The differential diagnoses includeCrohn’s disease, Reiter’s syndrome and Behçet’s syndrome(Table A). Reiter’s syndrome is classically a triad ofconjunctivitis, urethritis and arthritis 1–4 weeks after anepisode of bacterial dysentery or a sexually transmittedurethritis. Other features include plantar fasciitis, Achillestendinitis, keratoderma blennorrhagica, circinate balanitis,stomatitis, hepatitis, cardiac and neurological involvement,and occasionally amyloidosis. It is possible that thediarrhoeal illness may have been dysenteric and offset thereactive features of Reiter’s disease. There is no history ofpromiscuity or urethral discharge. Venous thrombosisaffects about 4% of patients with Reiter’s disease and occursearly in the disease. Mouth ulcers are common in Reiter’sdisease and are painless. Erythema nodosum is not a featureof Reiter’s syndrome. Arthritis is asymmetrical and usually

affects the knee and ankle joints. The most common jointto be affected in the upper limb is the wrist, as in this case;however, joint involvement occurs early, whereas in thiscase the disease has been present for two years.

Crohn’s disease is a chronic granulomatousinflammatory disease of the gastrointestinal tract ofunknown cause, and is a strong possibility in this case.Bloody diarrhoea is a recognized feature of Crohn’s colitis.Colonic disease is associated with perianal disease in justover 30% of patients. A seronegative reactive arthritis is arecognized complication of Crohn’s disease. Erythemanodosum occurs in some cases. Genital ulcers are rare, as isdeep-vein thrombosis. Urethral involvement and dysuriaonly occur when an inflammatory fistula develops betweenthe colon and the ureter. Recurrent urinary tract infectionsdue to faecaluria can cause urethral stricture.

Behçet’s syndrome is the most probable diagnosis.Behçet’s syndrome is a recurrent multifocal disorder thatpersists over many years. It is characterized by recurrentmouth and genital ulcers, ocular lesions, and skin, jointand neurological involvement. The incidence is high inJapan and in countries bordering the Mediterranean.Oral and genital ulcers are present in most patients.

d. Behçet’s syndrome.

Answer 31

3300

Page 38: Rapid review of clinical medicine for mrcp part 2

Clinical Cases 37

Ulcers can affect the pharynx and cause dysphagia.Genital ulcers can cause dysuria and dyspareunia. Ocularlesions are the most serious development. Recurrentuveitis and iridocyclitis, retinal vascular lesions and opticatrophy can lead to loss of vision in 50% of patients withocular involvement. Erythema nodosum is a recognizedfeature. Other skin manifestations include a diffusepustular rash affecting the face, erythema multiforme.The pathergy is a useful diagnostic sign. Pricking the skincan lead to erythema around the affected part within24–48 hours, which is a relevant feature in our patient.

A seronegative arthritis affects about 40% of patients andcommonly involves knees, ankles and wrists. Recurrentthrombophlebitis of the legs is a significant feature ofBehçet’s syndrome, leading to venous thrombosis. Lessoften, superior or inferior cava thrombosis may occur.Abdominal pain and bloody diarrhoea have also beendocumented. Asymptomatic proteinuria is a recognizedfeature, but on a few occasions may reflect renalamyloidosis. Neurological complications occur in 20% ofpatients. Organic confusional states, meningoencephalitis,transient or persistent brainstem syndromes, multiplesclerosis and parkinsonian-type disorders are all recognized.

Behçet’s syndrome is a clinical diagnosis. There is nospecific diagnostic test. HLA-B51, B12, DR2, DR7 andDrw52 are associated with the syndrome. Acute-phase

proteins are elevated and immune complexes are present.The pathergy test is a simple useful test. Genital ulcersand oral complications are treated with topical steroids.In severe cases, systemic steroids become necessary,together with azathioprine, which acts as a steroid-sparing agent. Colchicine, cyclosporin and levamisolehave also been used in the management of this condition.The causes of orogenital ulceration are given in Table B.

Behçet’s syndromeCrohn’s diseaseHerpes simplex virusUlcerative colitisReiter’s syndromeLichen planus

SyphilisGonococcal infectionHIVPemphigus pemphigoidStevens–Johnson

syndrome

Table A Causes of orogenital ulcers

A 16-year-old female was admitted with a six-monthhistory of myalgia, loss of weight and night sweats. Overthe past six weeks she had started to become breathless onexertion. On admission to hospital she had a temperatureof 38.1°C (100.6°F). On auscultation of the precordium,there was an early diastolic murmur at the left lowersternal edge. Examination of the chest, abdomen andcentral nervous system was normal, with the exception ofher fundi, one of which is shown (32a). An echo cardio-gram was performed to investigate the murmur (32b).

3322aa

3322bb

1. What is the diagnosis?a. Systemic lupus erythematosus.b. Polyarteritis nodosa.c. Infective endocarditis.d. Marantic endocarditis.e. Libmann–Sacks endocarditis.

2. What investigation would you perform to confirmthe diagnosis?a. Autoantibody screen.b. Serum ANCA.c. Serum complement.d. Blood cultures.e. Serology for atypical bacteria.

Question 32

Table B Causes of orogenital ulcers and venousthromboses

Behçet’s syndrome Ulcerative colitisCrohn’s disease Reiter’s syndrome

Page 39: Rapid review of clinical medicine for mrcp part 2

38

Fever, diastolic murmur, presence of echogenic mass onthe aortic valve and a Roth’s spot in the retina(haemorrhagic areas with a pale centre) are consistentwith the diagnosis of infective endocarditis. Clinicalfeatures of infective endocarditis are tabulated (Table A).Blood cultures are the single most importantinvestigation in the diagnosis of infective endocarditis,and are positive in 90% of cases. Serial blood culturesshould be performed, because a single set of bloodcultures may not necessarily reveal the culprit organism.Although transthoracic echocardiography is extremelyuseful in confirming the presence of vegetations, theabsence of vegetations does not exclude endocarditis, as

vegetations <3 mm will not be seen on a transthoracicechocardiogram. A transoesophageal echocardiogram hasa much higher sensitivity in identifying bacterialvegetations compared to transthoracic echocardiography.In approximately 10% of cases an organism is notidentified despite repeated blood cultures. The two mostcommon reasons for this are prior treatment withantibiotics before culturing the blood, or that theendocarditis is due to a fastidious organism with respectto conventional blood culture media. In patients who areimmunosuppressed, endocarditis may be due to fungithat may be difficult to culture, and in very rarecircumstances endocarditis may be a manifestation ofSLE (Libmann–Sacks endocarditis) or a manifestation ofmalignancy, when it is termed marantic endocarditis(Tables B and C).

Table A Causes of Roth spots and other stigmata of endocarditis

Causes of Roth spots Clinical features of endocarditisInfective endocarditis PyrexiaSLE Cardiac murmurPolyarteritis nodosa Splinter haemorrhagesSevere anaemia Osler’s nodes: small; painfulLeukaemia Janeway lesions: macular; painless

Roth spotsSplenomegalyMicroscopic haematuria/nephritis

Table B Causes of culture-negative endocarditis

Infective Non-infective• Brucella spp. • SLE (Libmann–Sacs)• Coxiella burnetti • Marantic endocarditis• Chlamydia spp.• Mycoplasma spp.• Legionella spp.• Histoplasmosis• Tropherema whipelli• Fungi

Table C Common causes of culture-positive endocarditis

• Streptococcus viridans• Staphyloccus aureus• Streptococcus faecalis (associated with

carcinoma of the colon)• Enterococci

1. c. Infective endocarditis.2. d. Blood cultures.

Answer 32

Page 40: Rapid review of clinical medicine for mrcp part 2

Clinical Cases 39

A 51-year-old accountant presented with a six-monthhistory of persistent dull right upper quadrant pain andfever. The pain did not radiate elsewhere, but wasexacerbated on lying on her right side. During this periodshe had intermittent pale bulky stool which was difficultto flush, and episodic dark urine. More recently, herappetite was reduced and she had lost approximately 1 kgin weight during the past month. Over the past week shehad difficulty sleeping due to itching all over her body,and her colleague at work commented on a yellowishpigmentation in her eyes. Six months before this, she hadbeen relatively well. She had a past history of acholecystectomy for cholesterol stones at the age of 32and subsequently had an ERCP and removal of sludgefrom the common bile duct six years ago. She consumed10 units of alcohol per week. She was married with twosons, aged 20 and 18. Three months ago she had beenon holiday in Scotland. She was not taking any regularmedication.

On examination, she was slightly icteric. Inspection ofher hands is shown (33).

There were spider naevi on her arms neck and face andscratch marks around her trunk and lower limbs. She hada temperature of 39°C (102.2°F). Her heart rate was120 beats/min and blood pressure 140/80 mmHg.There were a few inspiratory crackles on auscultation ofthe right lung base. Abdominal examination demon -strated firm, slightly tender hepatomegaly 4 cm belowthe costal margin, and a moderately enlarged spleen.There were no other abdominal masses, and there was noevidence of shifting dullness. Rectal examination wasnormal.

Investigations are shown.

AST 60 iu/lAlkaline phosphatase 360 iu/lBilirubin 82 μmol/lAlbumin 25 g/lTotal protein 93 g/lINR 1.49Chest X-ray Normal

A 75-year-old male was seen by his GP with a five-dayhistory of wheeze and ankle swelling. He was prescribedsome medication, but continued to deteriorate and wasadmitted to hospital.

Investigations are shown.

1. What investigation would you perform to ascertain the cause of her illness?a. Blood cultures.b. Liver biopsy.c. ERCP.d. Ultrasound of the biliary tract.e. Alpha-fetoprotein level.

2. What is the cause of her current presentation?a. Sclerosing cholangitis.b. Secondary biliary cirrhosis.c. Primary biliary cirrhosis.d. Cholangiocarcinoma.e. Recurrent cholangitis.

1. What is the acid-base disturbance?2. Suggest two possible causes for this metabolic

picture.

Arterial blood gases:pH 7.33PaO2 7 kPaPaCO2 6.5 kPaBicarbonate 20 mmol/l

Sodium 133 mmol/lPotassium 5 mmol/lUrea 28 mmol/lCreatinine 200 μmol/l

Question 33

Question 34

3333

Page 41: Rapid review of clinical medicine for mrcp part 2

40

The key in this question is a prior history of cholelithiasisaffecting the gallbladder and the biliary tree. Stones in thebile duct are a nidus for infections, particularly when therehas been instrumentation, as in this case. Cholangitis ischaracterized by the triad of right upper quadrant pain,jaundice and fever. This patient has had previous episodessuggestive of cholangitis, and the best answer for thepresenting complaint is recurrent cholangitis. She also hasevidence of cirrhosis, which is almost certainly secondary tochronic cholestasis. Although cholangiocarcinoma mayrarely complicate biliary cholestasis, this is unlikely giventhe mode of presentation. Cholangiocarcinomacharacteristically presents with intermittent jaundice, andcauses duct obstruction before the onset of biliary cirrhosis.Primary sclerosing cholangitis is very uncommon. Theaetiology is thought to be auto-immune, but the triggerfactor is unknown. More usually, it is associated with

ulcerative colitis, but may occur in patients with HIVeither as a direct insult from HIV or due to opportunisticCMV, cryptosporidial and Entamoeba infection. Althoughit may lead to secondary biliary cirrhosis, it is unlikely inthis case, given the history of chronic cholestasis. Primarybiliary cirrhosis (PBC) is usually insidious. It is much morecommon in females and usually presents with pruritus andjaundice. Stones are not a recognized feature of PBC.

The investigation of choice is blood cultures, whichhave high positive yield in cholangitis. The most commonorganism isolated is Escherichia coli. Hepaticultrasonography should be performed in all cases toexclude a secondary hepatic abscess, and to assess thecommon bile duct. ERCP and sphincterotomy may betherapeutic. There is no role for α-fetoprotein and liverbiopsy in this case. α-Fetoprotein is often modestlyelevated in cirrhosis of the liver, but grossly elevated inhepatocellular carcinoma. The presentation here is ofcholangitis. A liver biopsy should never be the first line ofinvestigation in a jaundiced patient with a fever.

The patient has a respiratory acidosis that is characterizedby a PaCO2 of 6.5 kPa or more, and a pH below 7.35, andis hypoxic. In the acute situation, respiratory acidosis is notcompensated by the kidney, but after 3–5 days the kidneysretain bicarbonate ions to compensate, which results innormalization of the pH at the expense of a relativemetabolic alkalosis. In this patient the bicarbonate isslightly low, suggesting a metabolic acidosis. This may be

due to coexistent renal disease that prevents adequatecompensation, or to another factor causing metabolicacidosis. The serum urea and creatinine levels are elevated,but there is a relatively larger increase in the serum urea,suggesting dehydration in this case. It is possible that hewas prescribed a diuretic for symptoms of chronicobstructive airways disease, which have precipitated renalfailure by causing dehydration.

An alternative suggestion is that he has developedsevere cardiac failure leading to pulmonary oedemacausing hypoxia and respiratory acidosis andhypoperfusion of the kidneys, resulting in metabolicacidosis from renal failure. Other causes of combinedrespiratory and metabolic disorders are shown (Table).

Other causes of combined respiratory and metabolic acidosis

• Aspirin poisoning• Severe pneumonia with renal failure due to septicaemia or interstitial nephritis (Legionnaire’s disease)• Septicaemia from any cause complicated by ARDS• Malaria complicated by pneumonia• Acute renal failure and fluid overload• Renal pulmonary syndromes: anti-GBM disease, Wegener’s granulomatosis, microscopic polyarteritis nodosa• Acute massive pulmonary embolism• Cardiac arrest (before ventilation)

1. a. Blood cultures.2. e. Recurrent cholangitis.

1. Combined respiratory and metabolic acidosis.2.i. Acute cardiac failure.ii. Severe exacerbation of obstructive airways disease

and pre-renal failure from diuretics.

Answer 33

Answer 34

Page 42: Rapid review of clinical medicine for mrcp part 2

Clinical Cases 41

Actual PredictedFEV1 (l) 2.0 4.5FVC (l) 4.0 5.6FEV1/FVC 50% 80%TLC (l) 4.9 7.0Residual volume (l) 1.7 2.2KCO (mmol/l/kPa) 5.0 10

1. What is demonstrated by the respiratory function tests?

2. In the context of the history, what is the diagnosis?3. How would you confirm the diagnosis?4. If these were the respiratory function tests of a

young male with recurrent respiratory tractinfections, what diagnosis would you consider?

1. Give one possible explanation for the breathlessnessand all the abnormalities on the blood tests.

2. What investigation would you perform to investigate the breathlessness further?

A 64-year-old patient who had been attending thecardiology clinic for 15 years presented with increasingbreathlessness on exertion and a reduced appetite.Examination of the face is shown (36a, b).

Investigations are shown.

3366aa

3366bb

Hb 13 g/dlWCC 7 � 109/lPlatelets 190 � 109/lMCV 80 flSodium 135 mmol/lPotassium 4.1 mmol/lUrea 8 mmol/lCreatinine 100 μmol/lBilirubin 48 μmol/lAST 160 iu/lAlkaline phosphatase 180 iu/lAlbumin 39 g/lThyroxine 190 nmol/l TSH <0.1 mu/lECG Atrial fibrillation

Ventricular rate 80/minEchocardiogram Enlarged left atrium

Normal-sized left ventricle with good function

Normal valves

A 14-year-old male presented with joint pains, polydipsiaand polyuria.

Respiratory function tests are shown.

Question 35

Question 36

Page 43: Rapid review of clinical medicine for mrcp part 2

42

Histiocytosis X is a benign disorder of unknown aetiology.It generally affects many systems, but in up to 25% ofcases only one system may be involved. The peak age ofpresentation is between 2 and 4 years, and the conditionaffects males more than females. The skin, bones, ears,lungs, eyes, and the reticuloendothelial and centralnervous systems are most often affected. A skin rash is

common. Individual histiocytomas are pinkish-brownpapules of 1–5 mm in diameter. Respiratory mani -festations include lung fibrosis, bullae and large airwayobstruction by histiocytomas. Therefore, both restrictiveand/or obstructive respiratory function patterns mayoccur. Bone involvement with marrow infiltration may beassociated with pancytopenia. Any bone may be involved,but there appears to be a special predilection to the skullbones, where it manifests as large radiolucent lesions.Otitis externa and a troublesome aural discharge occur insome patients. Ocular involvement is usually characterizedby a retro-orbital mass, resulting in proptosis. Hepato -splenomegaly and lymphadenopathy may be present.Occasionally, lymphadenopathy may be massive. One-third of patients develop diabetes insipidus due tohistiocytic infiltration of the pituitary stalk. Spinal cordcompression may occur in exceptional cases.

The diagnosis is based on the demonstration ofhistiocytes and small round cells in histological specimens ofaffected tissues. Management is with steroids, but in somecases cytotoxic drugs such as vincristine or etoposide maybe useful. In some cases spontaneous regression may occur.

The patient has biochemical evidence of hepatitis andhyperthyroidism. In a patient who has been attending thecardiology clinic for several years, and has underlying atrialfibrillation, it is highly likely that these abnormalities aredue to the toxicity of amiodarone which was prescribed tocontrol the AF. Amiodarone is a class III anti-arrhythmicagent which is very effective in the management ofventricular and supraventricular arrhythmias; however, it isassociated with several side-effects and complications(Table). Lung fibrosis is an important complication andalmost certainly accounts for the patient’s breathlessness.Early interstitial fibrosis may not be apparent on chest X-ray, but is suggested by a low KCO on respiratory functiontests. Both hyperthyroidism and hypothyroidism arerecognized due to the iodine content of the drug.Asymptomatic biochemical hepatitis may occur and somepatients may progress to cirrhosis of the liver. Patients onamiodarone should have an annual thyroid test, an LFT

and a KCO estimation. Photosensitivity is common, andlong-term use is associated with slate-grey skinpigmentation (36a, b). Corneal microdeposits areuniversal and may cause night glare; however, these resolveon withdrawal of the drug.

Side-effects and complications of amiodarone

Side-effects ComplicationsNausea Optic neuritisMetallic taste Peripheral neuropathy

in the mouth MyopathyNightmares Hepatitis/cirrhosisTremor AlveolitisHeadaches Hyper/hypothyroidismRashes including Epididymitis

phototoxicity Conduction tissue disturbances

1. A mixed obstructive and restrictive defect with a reduced KCO. The low FEV1/FVC ratio is in keeping with an obstructive defect; however, the low residual volume also suggests a restrictive pattern.

2. The combination of polydipsia and polyuria withrespiratory involvement is suggestive of diabetes insipidus. In a young patient with a mixed restrictive and obstructive defect the most likely diagnosis is histiocytosis X.

3. Transbronchial lung biopsy/liver biopsy/trephine aspirate may all show histiocytes and small round cells.

4. Cystic fibrosis.

1. Amiodarone toxicity.2. Formal lung function tests including a KCO

estimation.

Answer 35

Answer 36

Page 44: Rapid review of clinical medicine for mrcp part 2

Clinical Cases 43

A 66-year-old retired plumber was seen in the Accidentand Emergency Department with progressively increasingdyspnoea and left-sided chest discomfort, which wasworse on inspiration. He had been seen by his GP oneweek previously, and treated with antibiotics for a chestinfection. According to his wife, his appetite had beenreduced and he had lost at least 6.5 kg in weight over sixweeks. He stopped smoking five years previously, butbefore this was smoking 30 cigarettes per day for over 40years.

On examination, he appeared thin and tachypnoeic.There was no clinical evidence of cyanosis, but he had earlyclubbing. His voice was hoarse. Respiratory examinationrevealed reduced chest expansion of the left hemithoraxand dullness to percussion with reduced air entry affectingthe left anterior hemithorax. A chest radiograph taken isshown (37a).

The patient was given oral amoxycill in anderythromycin for one week, and asked to return forrepeat chest X-ray in two weeks. However, only four dayslater he was admitted as an emergency with increasingconfusion. According to his wife, he had been extremelythirsty and was waking several times each night to passlarge volumes of urine!

Investigations on this occasion are shown.

Hb 10.8 g/dlWCC 14.2 � 109/lPlatelets 80 � 109/lMCV 91 flSodium 133 mmol/lPotassium 3.9 mmol/lUrea 18 mmol/lCreatinine 179 μmol/lGlucose 8.1 mmol/lUrinalysis Glucose 0

Ketones +Blood film (37b)Flow loop curve (37c)

3377aa 3377bb

3377cc

1. What is the probable underlying diagnosis?2. What are the two abnormalities on the flow loop

curve?3. Give five possible reasons for his confusion.4. What are the most likely reasons for his polydipsia

and polyuria?5. Give two reasons why this man is incurable.

0 2 4 6Change in lung volume (l)

Expiration

Inspiration

Normal

Patient

10

8

6

4

2

0

2

4

6

8

10

Flow (l/s)

Question 37

Page 45: Rapid review of clinical medicine for mrcp part 2

44

A 76-year-old Polish male was found collapsed in the toiletby the home help. He had been generally unwell for thepast five months and was easily fatigued. His appetite wasreduced because he felt constantly nauseous. He was seenin the Accident and Emergency Department one monthago with abdominal pain, and was diagnosed as havingconstipation, for which he was prescribed senna. He hadnot been out of bed for four days. He had a past history ofhypertension and osteoarthritis. He lived alone and wasbecoming increasingly dependent on the social services ashis health deteriorated. He smoked ten cigarettes per day.

On examination, he was confused and agitated, andappeared pale. He was malnourished and clinicallydehydrated. There was no evidence of cyanosis orlymphadenopathy. His temperature was 39°C (102.2°F).The heart rate was 110 beats/min and blood pressure105/50 mmHg. The JVP was not raised. Examination ofhis tongue is shown (38a). On examination of thecardiovascular system, he had a fourth heart sound.Respiratory examination revealed a respiratory rate of30/min. Movement of the right hemithorax was reduced.Percussion note was dull from the right mid-zone to thebase. The abdomen was thin. A liver edge was palpable

3 cm below the costal margin. Examination of the centralnervous system was difficult owing to his confusion andagitation, but there was no evidence of paralysis. He wasclutching his right forearm and seemed to be in severe painon the slightest movement of his right arm. The knee andankle jerks were absent, but the plantar response wasnormal. Examination of the left fundus revealed anabnormality (38b). Inspection of his legs is shown (38c).

Investigations are shown.

The chest X-ray (37a) demonstrates a large mass arisingfrom the left hilum, which is complicated by left upperlobe collapse. The risks for developing bronchialcarcinoma are smoking and the possibility of asbestosexposure during his occupation as a plumber.

The excessive thirst and polyuria reflect diabetesinsipidus. Cerebral metastases may infiltrate thehypothalamus or posterior pituitary to cause cranialdiabetes insipidus by affecting the production and releaseof ADH, respectively. Hypercalcaemia may render thedistal renal tubules refractory to the action of ADH andproduce a nephrogenic diabetes insipidus. In this case,the hypercalcaemia is probably due to bony metastasesbecause the blood film demonstrates a leuco-erythroblastic picture suggesting marrow infiltration(37b); however, in squamous cell bronchial carcinomathe hypercalcaemia may also be due to ectopic secretionof a PTH-related peptide.

The main contraindications to surgery include localand distant metastases and poor respiratory reserve.

1. Bronchial carcinoma affecting the left lung. 2. i. Severely reduced expiratory flow consistent with

obstructive airways disease.ii. Reduced inspiratory flow limb consistent with

upper airways obstruction. In this case it is probably due to a laryngeal nerve palsy causingvocal cord paralysis. (See ‘Interpretation of Respiratory Flow Loop Curves’, page 420.)

3. i. Hypercalcaemia.ii. Cerebral metastases.iii. Hypoxia.iv. Concurrent respiratory tract infection.v. Dehydration.

4. i. Hypercalcaemia.ii. Cranial diabetes insipidus.

5. i. Local and distant metastases (recurrent laryngeal nerve, bone and brain).

ii. Poor respiratory reserve.

Answer 37

3388aa

Question 38

Page 46: Rapid review of clinical medicine for mrcp part 2

Clinical Cases 45

3388bb

3388cc

3388dd

3388ee

3388ff

Hb 8 g/dlWCC 25 � 109/lPlatelets 90 � 109/lBlood film (38d)ESR 148 mm/hSodium 130 mmol/lPotassium 4 mmol/lUrea 16 mmol/lTotal protein 50 g/lAlbumin 18 g/lAlkaline phosphatase 220 iu/lAST 26 iu/lBilirubin 13 μmol/lArterial blood gases (40% O2):

pH 7.4PaO2 9 kPaPaCO2 5 kPaBicarbonate 18 mmol/l

Chest X-ray (38e)X-ray, right arm (38f)Bone marrow aspirate Dry tapUrinalysis Protein +++

No organisms

1. What is the diagnosis?2. What is the cause of the abnormality in the left

fundus?3. Why does the patient have a painful right arm?4. Comment on two abnormalities seen on his legs

and give an explanation for each one.5. List five complications of the disease that are

evident in this patient.

Page 47: Rapid review of clinical medicine for mrcp part 2

46

Answer 38

Recognized causes of renal failure in myeloma

• Tubular obstruction by light chain deposits• Infiltration of plasma cells in the kidney (‘myeloma kidney’)• Hypercalcaemia• Hyperuricaemic nephropathy• Amyloidosis• Recurrent pyelonephritis secondary to immunoparesis• Renal vein thrombosis secondary to hyperviscosity• Renal failure secondary to analgesics, such as non-steroidal

anti-inflammatory drugs• Renal failure secondary to contrast agents, e.g. for an IVP

3388gg

1. Multiple myeloma.2. Branch retinal vein occlusion.3. He has a pathological fracture affecting his right

radius.4. i. Oedema due to nephrotic syndrome.

ii. Bruising secondary to thrombocytopenia oramyloid related capillary fragility.

5. i. Sepsis affecting the right lung as a result ofimmunoparesis. Bronchopneumonia and renalfailure are the most common causes of death inmultiple myeloma.

ii. Amyloidosis. The tongue is enlarged owing toamyloid infiltration. The patient is oedematousand hypoalbuminaemic and has markedproteinuria, suggesting nephrotic syndrome,which is a manifestation of renal amyloid.Amyloid often affects the heart in myeloma, andis an important cause of death. Skin deposits, aperipheral neuropathy and massive hepaticinfiltration are all recognized features ofamyloidosis associated with multiple myeloma.

iii. Bony involvement. There are numerous lyticlesions affecting the right humerus and apathological fracture. The blood picture (38d)reflects a leuco-erythroblastic picture which, inthis case, is probably due to marrow infiltrationof myeloma cells, although severe sepsis is also arecognized cause. The typical appearance ofplasma cells is shown in the marrow (38g). Thecell nuclei sit at the edge of the cytoplasm andare well defined.

iv. Hyperviscosity leading to venous thrombosis inthe retinal vein. Hyperviscosity occurs when theparaprotein level is very high. Venous and arterialthrombosis can occur almost anywhere.

v. Renal failure. The raised urea suggests renalfailure. There are several recognized causes ofrenal failure in myeloma, which are shown(Table). Myeloma is also a recognized cause ofproximal RTA; however, this does not causerenal failure per se.

Page 48: Rapid review of clinical medicine for mrcp part 2

Clinical Cases 47

A 60-year-old female presented with a four-month historyof headaches and general malaise. Over the past threeweeks she had developed nausea and epigastric pain aftermeals. She was restless at night and had noticed occasionalpalpitations, but attributed these symptoms to fear ofillness. She also had of episodic diarrhoea. On examination,she appeared anxious. She had an irregular swelling in herneck, which moved vertically with swallowing. Her heartrate was 120 beats/min, and regular; blood pressure was180/105 mmHg. Examination of all the major systemswas normal. Investigations are shown.

Hb 17 g/dlWCC 9 � 109/lPlatelets 300 � 109/lSodium 135 mmol/lPotassium 3.4 mmol/lUrea 10 mmol/lCreatinine 120 μmol/lCalcium 2.8 mmol/lPhosphate 0.6 mmol/lBlood glucose 10 mmol/lTSH 3 mu/l

M-mode echocardiograms of twowomen (A and B), who wereinvestigated for breathlessness in abusy cardiology clinic, are shown(40a, b).

4400aa

4400bb

1. What is the cause of this patient’s epigastric pain?2. Give two explanations for the diarrhoea.3. What is the most likely cause for the hypertension?4. How would you explain her haemoglobin level?5. What is the complete diagnosis?6. List two tests you would perform to investigate the

thyroid swelling.

Question 40

1. What is the diagnosis in female A?a. Mitral valve prolapse.b. Hypertrophic cardiomyopathy.c. Mitral valve endocarditis.d. Rheumatic mitral stenosis.e. Aortic regurgitation.

2. What is the diagnosis in female B?a. Hypertrophic cardiomyopathy.b. Atrial myxoma.c. Mitral valve endocarditis.d. Pericardial effusion.e. Rheumatic mitral stenosis.

Question 39

Page 49: Rapid review of clinical medicine for mrcp part 2

48

The patient has hypercalcaemia and hypertensionassociated with palpitations. She has an irregular swellingin the neck, which is consistent with a thyroid goitre. TheTSH is normal, indicating that she is euthyroid. The lowserum phosphate and high calcium levels indicate that primary hyperparathyroidism is the most likely cause of the raised calcium. Although primaryhyperparathyroidism by itself is a recognized cause ofhypertension, it does not account for the palpitations andrestlessness, which are best explained by a coexistingphaeochromocytoma. The combination of primaryhyperparathyroidism and phaeochromocytoma shouldlead one to consider the possibility of MEN 2. In thecontext of this question it is very probable that thethyroid goitre represents a medullary cell carcinoma.

MEN 2 consists of tumours arising from thyroid C cellcarcinoma, the parathyroid gland and the adrenal

medulla; the latter is the most common expression of thedisorder. It is rare below the age of 20, and becomesmore common from there onwards. One-third of casesoccur in patients aged over 70 years. The geneticabnormality is within the RET proto-oncogene onchromosome 10. The gene encodes a transmembraneglycoprotein receptor tyrosine kinase.

Medullary cell carcinoma develops in 80% of cases andis usually the first presentation; however, MEN 2accounts for only 10–20% of all medullary cellcarcinomas. Management is with surgery. Persistentlyelevated calcitonin levels after surgery suggest that thepatient is not cured. Phaeochromocytomas occur in 50%of cases and are usually bilateral and may beasymptomatic; therefore, annual screening with urinarycatecholamine measurements is advised. Hyperpara -thyroidism occurs in only 5–10% and is restricted tocertain families.

Screening in families with known disease is by genetictesting. Patients who are gene-positive should have aprophylactic thyroidectomy at a young age, and shouldthen be screened annually with serum PTH, and urinarycatecholamine estimation. In those families in whom agenetic abnormality has not been demonstrated, annualserum calcitonin should also be performed, and fine-needle aspiration of the thyroid is indicated if a goitreappears, or if the calcitonin becomes elevated.

Female A (40a): the mitral valve has a normal appearancein diastole. In systole, both the anterior and posteriorleaflets appose normally in the beginning; however, inmid systole there is posterior movement (prolapse) ofboth leaflets, but predominantly the posterior leaflet.This is characteristic of mitral valve prolapse and isresponsible for the mid-systolic clicks and/or the mid-systolic murmur of mitral regurgitation. Mitral valveprolapse is present in 2–5% of the general population. Itis generally benign but may be associated with symptomsof sharp inframammary chest pain, breathlessness evenwith normal ventricular function, palpitation and oddneurological symptoms. There is a risk of infectiveendocarditis in patients with consequent mitral

regurgitation, in whom prophylactic antibiotics duringdental work are recommended, and supraventriculararrhythmias. Mitral valve prolapse has been implicated asa cause of sudden cardiac death in highly trained athletes,but its significance as the only finding at post-mortem iscontroversial because the high prevalence of thecondition may mean the finding is coincidental, ratherthan causal. It is more likely that death may have beencaused by the association of mitral valve prolapse with theWolff–Parkinson–White syndrome causing atrialfibrillation which degenerates to ventricular fibrillation,or the LQTS, which causes polymorphic ventriculartachycardia.

Female B (40b): there is echogenic shadowing in themitral valve orifice that is typical of atrial myxoma. Thecondition is discussed in Answer 178. (See alsoEchocardiography, page 421.)

1. Hypercalcaemia (peptic ulcer disease, pancreatitis).2. i. Raised calcitonin levels.

ii. Phaeochromocytoma.3. Phaeochromocytoma.4. Secondary polycythaemia in association with

phaeochromocytoma.5. Multiple endocrine neoplasia type 2 (MEN 2).6. i. Fine-needle aspiration of the thyroid.

ii. Serum calcitonin.

1. a. Mitral valve prolapse.2. b. Atrial myxoma.

Answer 39

Answer 40

Page 50: Rapid review of clinical medicine for mrcp part 2

Clinical Cases 49

A 42-year-old woman with a past medical history ofsystemic lupus erythematosus that was well-controlled onmethotrexate, presented with a four-day history of rightlower chest pain that was sharp in character and worse on

inspiration, along with chest wall tenderness. There wasno history of cough, breathlessness, haemoptysis or fever.

Investigations are shown.

Question 42

Hb 11.2 g/dlWCC 6 � 109/lPlatelets 260 � 109/lESR 23 mm/hCRP 3 g/lSodium 134 mmol/lPotassium 4.1 mmol/lUrea 9 mmol/lCreatinine 130 �mol/lC3 0.35 g/l (NR 0.55–1.20 g/l)C4 0.26 g/l (NR 0.2–0.5 g/l)Chest X-ray NormalOxygen saturation on air 98%

What is the diagnosis?a. Costochondritis.b. Pulmonary embolism.c. Relapse of systemic lupus erythematosus.d. Right lower lobe pneumonia.e. Pneumocystis carinii infection.

A 45-year-old woman presented with a three-weekhistory of fever, persistent dry cough, headache andphotophobia. She lived with her son, who owned twogreyhounds and several racing pigeons. The patient hadnot travelled abroad.

On examination she had a temperature of 38.2°C(100.8°F) and was slightly confused. She had ageneralized macular rash but there was no obviousevidence of lymphadenopathy.

Examination of the respiratory system was normal,with the exception of a respiratory rate of 18 per min.Both heart sounds were normal; there were no addedheart sounds or murmurs. Abdominal examinationrevealed hepatomegaly palpable three finger-breadthsfrom the costal margin and a palpable spleen 2 cm belowthe costal margin.

Investigations are shown.

Question 41

Hb 11 g/dlWCC 12 � 109/lPlatelets 380 � 109/lSodium 130 mmol/lPotassium 3.7 mmol/lUrea 8 mmol/lCreatinine 70 �mol/lBilirubin 12 �mol/lAST 46 iu/lALT 52 iu/lAlkaline phosphatase 100 iu/lAlbumin 34 g/l

What is the first-line antibiotic of choice in hermanagement?

a. Ciprofloxacin.b. Tetracycline.c. Erythromycin.d. Benzyl penicillin.e. Rifampicin.

Page 51: Rapid review of clinical medicine for mrcp part 2

50

This is a difficult question since there is a very limitedamount of data provided. However, the combination of

serositis, myalgia, low ESR relative to CRP, andcomplement at the lower limits of normal in a patientwith SLE is suggestive of recurrence of lupus rather thanany of the other options provided. (See Question 264.)

Answer 42

c. Relapse of systemic lupus erythematosus.

A history of dry cough, fever, headache and photophobiain a patient who is in contact with racing pigeons isconsistent with the diagnosis of psittacosis, which iscaused by the Gram-negative bacterium Chlamydiapsittaci. Psittacine birds are the primary reservoir andman is an incidental host. The organism is transmitted toman via aerosol inhalation of faecal and urinary productson bird feathers. Chlamydia psittaci infection is anoccupational hazard in pet-shop employees, veterinarydoctors and nurses, zoo personnel and workers in poultryprocessing plants. Chlamydia psittaci infection is rare andaccounts for 1% of all atypical pneumonias.

Clinical presentation includes a dry cough and fever,which may be abrupt in onset. Headache is a prominentfeature and may be accompanied by photophobia;however, genuine meningitis is a rare complication of theinfection. Malaise, night sweats, anorexia, diarrhoea anddelirium are recognized. The respiratory features may bemild but fulminant respiratory failure may occur in a fewcases.

Extrapulmonary complications do occur but with amuch lower frequency than in Mycoplasma pneumoniainfection (Table A). Jaundice is rare.

Respiratory examination reveals evidence of pleurisy,consolidation or effusion. Splenomegaly and hepatomegalyare present in 10% of cases.

Rapid diagnosis is not possible. The diagnosticinvestigation of choice is the complement fixation test.First-line therapy is tetracycline or doxycycline.Erythromycin is a second-line drug.

Other chlamydial infections (Table B)Infection due to Chlamydia pneumoniae causespneumonia that is indistinguishable from pneumoniasecondary to Streptococcus pneumoniae infection. UnlikeMycoplasma infection, which affects young individuals,Chlamydia pneumoniae infection affects patients in theseventh and eighth decades. Hoarseness, pharyngitis andsinusitis are common. The white cell count is normal and

the chest X-ray usually shows patchy subsegmentalconsolidation.

Extrapulmonary complications as seen in Mycoplasmapneumonia and psittacosis are recognized but rare. Ofinterest is the fact that infection has been associated withatherosclerosis. Diagnosis and treatment are the same asfor Chlamydia psittaci infection.

Answer 41

b. Tetracycline.

Table A Extrapulmonary complications ofChlamydia psittaci infection (psittacosis)

Cardiac Endocarditis, pericarditis and myocarditis

Neurological Encephalitis, transverse myelitis, Guillain–Barré syndrome

GlomerulonephritisPancreatitisHaemolysisThyroiditisDermatological Erythema nodosum

Table B Infections causes by Chlamydia species

Chlamydiatrachomatis Males

Non-gonococcal urethritisLymphogranuloma venerumEpididymitisProstatitisProctitis (homosexuals)Reactive arthritis

(Reiter syndrome)Females

Non-gonococcal urethritisCervicitisPelvic inflammatory diseasePeri-hepatitis (Fitzhugh–

Curtis syndrome)Newborns (born to infected mothers)

ConjunctivitisPneumonia

Chlamydiapneumoniae Pneumonia

Chlamydia psittaci Psittacosis

Page 52: Rapid review of clinical medicine for mrcp part 2

Clinical Cases 51

A 72-year-old male with a known history of chronicobstructive airway disease had eight admissions tohospital with a cough and breathlessness over a 12-month period. He had stopped smoking two yearspreviously. His normal exercise capacity was limited to27.5 metres on the flat before stopping. He was takingsalmeterol, pulmicort and oxivent inhalers, and hadreceived several courses of steroids in the past year.

On examination he was cyanosed. His heart rate was100 beats/min and regular. His blood pressure measured110/70 mmHg. The JVP was raised 4 cm above thecostal margin at 45°. The trachea was central. Chestexpansion was generally reduced. Percussion note was

hyper-resonant and on auscultation of the lungs therewas generalized reduction of air entry throughout. Bothheart sounds were soft but there were no added heartsounds or murmurs.

Investigations are shown.

Question 43

Hb 19 g/dlPCV 0.56 l/lWCC 8 � 109/lPlatelets 340 � 109/lMCV 82 flChest X-ray Hyper-inflated lung fieldsArterial blood gases:

pH 7.35PaO2 7.5 kPaPaCO2 6.1 kPaBicarbonate 34 mmol/l

Echocardiography Normal left ventricle. Right ventricular

hypertrophy and dilatation Pulmonary artery

pressure of 42 mmHg

Which treatment is most useful for long-termsurvival?

a. Oral aminophylline.b. Diuretic therapy.c. Regular venesection.d. High-dose steroids.e. Long-term oxygen therapy.

A 35-year-old type 1 diabetic presented eight monthsfollowing a renal transplant with fever, night sweats andmalaise. He had experienced two episodes of transplantrejection, which were successfully reversed withcorticosteroids. He was currently maintained onprednisolone and ciclosporin. On examination he had atemperature of 39°C (102.2°F). The heart rate was120 beats/min and blood pressure was 140/60 mmHg.The respiratory rate was 40/min. Precordial examinationrevealed an early diastolic murmur at the left lowersternal edge. The chest was clear. The abdomenincluding the graft was non-tender.

Investigations are shown.

Question 44

Hb 10 g/dlWCC 10 � 109/lNeutrophils 7 � 109/lLymphocytes 2.5 � 109/lEosinophils 0.04 � 109/lBasophils 0.1 � 109/lMonocytes 0.4 � 109/lPlatelets 300 � 109/lESR 100Sodium 130 mmol/lPotassium 5.5 mmol/lChloride 87 mmol/lBicarbonate 22 mmol/lUrea 9 mmol/lCreatinine 130 �mol/lGlucose 6 mmol/lUrine output 45 ml/hrUrinary microscopy Red blood cells and no

white cell castsECG Sinus tachycardia

What is the most likely diagnosis?a. Acute renal transplant rejection.b. CMV infection.c. Infective endocarditis.d. Gram-negative sepsis.e. Ciclosporin nephrotoxicity.

Page 53: Rapid review of clinical medicine for mrcp part 2

52

Indications for long-term oxygen therapy are a PO2 of7.3–8 kPa once stable and either secondary polycy -thaemia, nocturnal hypoxaemia, peripheral oedema or

pulmonary hypertension. The patient in question fulfilsthe criteria. Other patients for whom long-term oxygentherapy may be considered include those who havenormal oxygen saturation at rest but desaturate onexercise, and those in whom exercise capacity is improvedwith oxygen therapy.

Answer 43

e. Long-term oxygen therapy.

Infection is the commonest life-threatening complicationof long-term immunosuppressive therapy (Table). Morethan 6 months post-transplant most patients have stableallografts and are maintained on minimalimmunosuppressive therapy. Infection in the majority ofthese patients is usually similar to that seen in the generalpopulation. The patient in question has a murmur ofaortic regurgitation and a high fever, raising thepossibility of infective endocarditis.

The overall incidence of bacterial endocarditis is muchgreater in renal transplant recipients than in the generalpopulation. Independent risk factors for bacterialendocarditis include a history of hospitalization forvalvular heart disease, graft loss and increased duration ofdialysis before transplantation. The mean time fromtransplantation to diagnosis of endocarditis is estimatedto be three and a half years (range two months to 15years), with an overall mortality of 50%.

The increased susceptibility of renal transplantrecipients to infective endocarditis can probably beexplained by a combination of necessary invasiveprocedures (e.g. intravascular line placement) during theimmediate post-transplant period as well asimmunosuppression for prevention of organ rejection.Pre-existing valvular abnormality in renal transplantpatients may also be conducive to endocarditis similar tothat in non-transplant patients. Immunosuppression playsan important role in the development of fungalendocarditis, but may also be important in the clearance

of streptococci from the blood in this patient population.In contrast to the frequent pre-existing valve abnormalityassociated with bacterial endocarditis, the great majorityof solid organ transplant recipients with Aspergillusendocarditis have no pre-existing valvular abnormalities.Myocardial abscesses have been commonly reported withAspergillus endocarditis.

The most common causes of endocarditis among renaltransplant recipients are fungi (particularly Aspergillusspecies) and S. aureus. Less common causes includeCorynebacterium species, viridans streptococci,vancomycin-resistant Enterococcus species, Brucellaspecies, Clostridium species, Nocardia species andErysipelothrix species.

Skin manifestations of endocarditis and/orsplenomegaly seem to be uncommon in transplantpatients, while septic emboli and mycotic aneurysms aremore frequent. The most common clinical signs andsymptoms at the time of presentation are fever andembolic phenomena. The mitral valve is thought to be themost frequently involved cardiac valve, followed by theintramural cardiac surface and the aortic valve. Even in theabsence of immunosuppression the leucocyte count iselevated in only 20–30% of cases and may be normal.

Most episodes of acute rejection occur in the first sixmonths after surgery. However, the absence of a rise inserum creatinine, fall in urine output and graftpain/tenderness make rejection unlikely.

Ciclosporin nephrotoxicty is a recognized compli -cation in transplant patients and is characterized by afall ing glomerular fi ltration rate, hyperkalaemia,hyperuricaemia and a hyperchloraemic acidosis.

Answer 44

c. Infective endocarditis.

Infection in immunosuppressive therapy

Time InfectionMonth 1 Usual post-surgical infections (as seen in immunocompetent patients)Months 2–6 CMV and other opportunistic infectionsMonth 6 onwards Community acquired infections (viral and bacterial) and recurrence of

inadequately treated opportunistic infections

Page 54: Rapid review of clinical medicine for mrcp part 2

Clinical Cases 53

A 28-year-old woman who is eight weeks pregnantattends a genetic counselling service with her husband,who is a lawyer. His brother is heterozygous for the Δ508mutation for cystic fibrosis. The husband agrees to bescreened and is also found to be heterozygous for themutation. The pregnant woman in question declines thescreening test.

Question 47

Assuming that the Δ508 mutation has a prevalence of1:20 in the British population, what is the chance ofher having a child with the condition?

a. 1 in 20.b. 1 in 80.c. 1 in 400.d. 1 in 160.e. 1 in 4.

A 19-year-old man was admitted for further investigationfor a six-month history of lethargy and depression. 11months previously he had collapsed while on holiday inSan Francisco when walking on a very hot day and was

treated for dehydration. The patient had a history ofepilepsy that was confined to the winter months! He wasone of three siblings. His sisters, aged 17 and 20, were fitand well.

On examination he was 1.52 m tall. He had a shortneck and a left-sided divergent squint. There was nopallor. His heart rate was 68 beats/min and his bloodpressure was 110/70 mmHg. There was a palpablethyroid goitre. Neurological examination was normal aswas examination of all other systems.

Blood tests performed by the GP are shown.

Question 46

Hb 13 g/dl WCC 4.5 � 109/lPlatelets 259 � 109/lSodium 137 mmol/l Potassium 3.9 mmol/l Urea 4 mmol/l Creatinine 70 �mol/lCalcium 1.8 mmol/l Phosphate 1.9 mmol/l Albumin 42 g/l Alkaline phosphatase 100 iu/l Bicarbonate 24 mmol/lTSH 18 iu/lThyroxine 40 iu/l 24-h urinary calcium excretion IncreasedChest X-ray Normal

What is the most probable unifying diagnosis?a. Hypoparathyroidism.b. Hypothyroidism.c. Pseudohypoparathyroidism.d. Polyglandular endocrine deficiency type II.e. Polyglandular endocrine deficiency type I.

A 52-year-old woman was referred to a chest physicianwith a six-month history of dyspnoea associated with anon-productive cough, intermittent fever and nightsweats. At the onset of the illness her symptoms wereintermittent, usually worse in the evenings, but over thepast few weeks they had become more persistent. Thepatient also complained of reduced appetite and loss ofweight. She had worked on a farm with her husband forthe past four years. She smoked ten cigarettes per day.There was no previous history of breathlessness.

On examination she was thin. There was evidence ofperipheral cyanosis but no clubbing. Auscultation of the

lungs was normal. The chest X-ray showed reticulo-nodular shadowing affecting the upper and mid zones.

Question 45

What is the most probable diagnosis?a. Cryptogenic fibrosing alveolitis.b. Chronic bronchitis.c. Tuberculosis.d. Extrinsic allergic alveolitis.e. Allergic bronchopulmonary aspergillosis.

Page 55: Rapid review of clinical medicine for mrcp part 2

54

The gene frequency of the mutation in question is 1/20,hence the chances of the pregnant woman carrying themutation is 1 in 20. Her husband has been screened andis a definite carrier. Therefore, the chances of them bothhaving a heterozygous genetic mutation is

1/20 � 1/1 = 1/20.

If both partners are heterozygous for an autosomalrecessive disorder such as cystic fibrosis, the chances of achild being affected is 1 in 4. Therefore the overallchances of the couple having a child with cystic fibrosis is1/20 � 1/4 = 1/80. (See Genetics page 415.)

Answer 47

b. 1 in 80.

The patient has low calcium and high phosphate levels inthe absence of abnormal renal function. The differentialdiagnosis is between hypoparathyroidism andpseudohypoparathyroidism. Pseudohyoparathyroidism isdue to the end-organ resistance to the effects of PTH,

which are mediated via adenylate cyclase. Themorphological features in this patient (short stature,squint) are more common in pseudohyoparathyroidism.Furthermore, the patient has biochemical hypo -thyroidism, which is a more common association withpseudohypoparathyroidism and is due to end-organresistance to the actions of TSH, which also mediates itseffects via adenylate cyclase. (See Question 159.)

Answer 46

c. Pseudohypoparathyroidism.

The patient is a farmer’s wife and her original symptomswere consistent with an allergic (hypersensitivity)pneumonitis; however, more recently she has developedpersistent breathlessness, suggesting advanced paren -chymal lung disease. Repeated episodes of pneumonitis

progress to pulmonary fibrosis. The CXR in the acuteand subacute forms of the disorder may reveal fleetingmicronodular interstitial shadows affecting the upper,mid and lower zones. It is important to note that theCXR may be normal in a few cases. In more chronic casesthe CXR is similar to that seen in cryptogenic fibrosingalveolitis except that in EAA fibrosis is usually morepronounced in the upper zones.

Answer 45

d. Extrinsic allergic alveolitis.

Diagnostic features of EAA:

1. Evidence of exposure to a recognized antigen 2. Compatible clinical radiographic and physiological features (i.e. cough, wheeze, fever, micronodular shadows

in upper, mid or lower zones, restrictive lung defect)3. Bronchoalveolar lavage with lymphocytosis (with low CD4 to CD8 ratio)4. Positive inhalation challenge test5. Compatible histopathological changes

The diagnosis is possible without histological con -firmation on lung biopsy if criteria 1–3 are met.

Farm workers have a higher risk of developing chronicobstructive airways disease than the general population;however, the history and chest X-ray findings mean thatEAA is a better answer. Cryptogenic pulmonary fibrosisis unlikely in the presence of an obvious precipitating

factor such as Micropolyspora faenii in this case andshould only be considered once EAA has been excluded.Allergic bronchopulmonary aspergillosis is characterizedby wheeze, raised eosinophil count, and proximalbronchiectasis on the chest X-ray. (See Question 272.)

Page 56: Rapid review of clinical medicine for mrcp part 2

Clinical Cases 55

An alcoholic vagrant was admitted to the Accident andEmergency Department after being found collapsed onthe street. Four hours previously he had been spottedwandering outside the hospital and appeared well.

On examination he had a Glasgow coma scale of 5.His heart rate was 120 beats/min and regular. The blood

pressure measured 80/50 mmHg. The respiratory ratewas 24/min. The patient was apyrexial. The pupils weredilated. Neurological examination revealed blurred diskmargins and depressed reflexes.

Investigations are shown.

Question 48

What is the diagnosis?a. Euglycaemic ketoacidosis.b. Methanol toxicity.c. Ethylene glycol poisoning.d. Tricyclic antidepressant overdose.e. Hepatic encephalopathy.

Sodium 131 mmol/lPotassium 5.4 mmol/lUrea 5 mmol/lCreatinine 110 �mol/lBicarbonate 12 mmol/lChloride 96 mmol/lGlucose 6 mmo/lAmylase 120 iu/l

(NR <220 iu/l)Plasma osmolality 320 mOsm/lECG Sinus tachycardia, left axis deviationArterial blood gases:

pH 7.21 PaO2 9 kPaPaCO2 2.1 kPaBicarbonate 12 mmol/lBlood lactate 6 mmol/l

Urinalysis Protein 0Glucose 0Ketones +

A 29-year-old woman presented with sudden onset ofweakness affecting the left side of the body. There was nohistory of headaches or head injury. She had previouslybeen well with the exception of having mild Raynaud’sdisease. The patient had a history of two previousmiscarriages four years ago but had had a successfulpregnancy six months previously that was complicated bya below-knee deep vein thrombosis. She worked as aclerk in a large business firm prior to her pregnancy. Herappetite was good and her weight was stable.

On examination she had a left facial palsy and obviousleft-sided weakness. Her heart rate was 80 beats/min andregular. The blood pressure was 150/88 mmHg. Thetemperature was 36.8°C (98.2°F). Both heart soundswere normal; there was a soft systolic murmur in thepulmonary area. The chest was clear. Investigations areshown.

Question 49

What is the most probable cause of the stroke?a. Atrial septal defect complicated by paradoxical

embolus.b. Atrial myxoma.c. Saggital vein thrombosis complicating recent

pregnancy.d. Lupus anticoagulant syndrome.e. Cerebral abscess.

Hb 11 g/dlWCC 11 � 109/lPlatelets 152 � 109/lESR 11 mm/hCT scan brain Infarct in right middle cerebral

artery territory Trans-thoracicechocardiogram Normal.

Page 57: Rapid review of clinical medicine for mrcp part 2

56

The main differential diagnoses of severe metabolicacidosis and collapse specific to an individual who abusesalcohol includes methanol toxicity, ethylene glycolpoisoning, acute severe pancreatitis and acute hepaticfailure. The patient has a normal amylase therefore acutepancreatitis is unlikely. The actual plasma osmolality ishigher than the calculated plasma osmolality(approximately 284 mOsm/l) suggesting the presence ofa large concentration of an osmotically active substance.The most probable candidates include methanol orethylene glycol (anti-freeze). Both conditions presentwith drunkenness, acidosis and coma and in untreatedcases the mortality is high. Methanol and ethylene glycolare relatively toxic; however, once ingested they aremetabolized to formic acid (methanoic acid) and glycolicacid, respectively, by the enzyme alcohol dehydrogenase,both of which are very toxic.

Patients with methanol toxicity may complain ofheadache, nausea, fatigue or reduced visual acuity if not

comatosed. Reduced visual acuity is due to optic nervedamage resulting from increased concentrations of formicacid, the oxidized metabolite of methanol. Mydriasis,reduced visual reflexes to light and hyperaemia of theoptic disc are early features of methanol toxicity.Untreated patients may develop blindness. Patients withethylene glycol poisoning may present in a similarfashion; however, reduced visual acuity is not a classicalfeature whereas flank pain and renal failure due tocrystallization of oxalic acid in the renal tubules iscommon. Urinalysis under Wood’s light in patients withethylene glycol poisoning may reveal oxalate crystals inthe urine but an absence does not exclude the diagnosis.Patients with ethylene glycol poisoning also developsevere cardiac failure if untreated. The presence of visualsymptoms favour methanol intoxication in this case.

The management of methanol toxicity and ethyleneglycol toxicity is the same, as tabulated below. Tricyclicanti-depressant overdose can also present with acidosisand dilated pupils; however, one would have expectedbrisk reflexes and possible wide QRS complexes on the12-lead ECG. (See Question 28.)

Answer 48

b. Methanol toxicity.

The patient has a predisposition to arterial and venousthromboses. She has had miscarriages in the secondtrimester. She also has Raynaud’s phenomenon andthrombocytopenia, all of which are characteristic of lupusanticoagulant syndrome. The normal echocardiogram inthis context rules out an atrial septal defect and hence aparadoxical embolus as a cause of the stroke.Echocardiography may show evidence of verrucous(Libmann–Sack) endocarditis. The manifestations ofantiphospholipid syndrome are tabulated.

Answer 49

d. Lupus anticoagulant syndrome. Manifestations of antiphospholipid syndrome

• Predisposition to arterial and venous thromboses• Mid-trimester miscarriages in females• Migraines• Depression• Chorea• Epilepsy• Livedo reticularis• Thrombocytopenia• Haemolysis• Avascular necrosis of head of femur• Pulmonary hypertension• Libmann–Sack endocarditis

The management of methanol toxicity and ethylene glycol toxicity

Step 1 Gastric decontamination within the first 4 hours of ingestionStep 2 Intravenous bicarbonate in the presence of acidosis (100 ml of 8.4% sodium bicarbonate)Step 3 Intravenous fomepizole (specific alcohol dehydrogenase inhibitor) should be used early, starting with

a bolus of 15 mg/kg in dextrose 5% in 100 ml followed by an infusion of 10 mg/kg every 12 hoursfor the next 48 hours and then 15 mg/kg every 12 hours until the concentration of methanol orethylene glycol has fallen below 20 mg/dl

Step 4 Haemodialysis in patients with refractory acidosis despite bicarbonate and fomepizole therapy or asfirst line in patients with a methanol level of >15 mmol/l (50 mg/dl) or an ethylene glycol level of>20 mg/dl

Step 5 Folic acid in methanol toxicity promotes the metabolism of formic acid to carbon dioxide and water.Pyridoxine and thiamine in ethylene glycol poisoning promote metabolism of ethylene glycol to lesstoxic metabolites such as glycine

Page 58: Rapid review of clinical medicine for mrcp part 2

Clinical Cases 57

A 70-year-old male presented with a one-week history ofincreasing confusion. According to his wife his generalhealth had been deteriorating for about three months.His appetite was reduced. He complained of abdominal

discomfort and felt excessively thirsty. He had been aheavy smoker since the age of 16.

On examination he had reduced skin turgor. Theheart rate was 110 beats/min. Heart sounds werenormal. The chest was clear. Abdominal examinationrevealed a palpable colon with hard faeces.

Investigations are shown.

Question 50

What is the immediate step in his management?a. IV pamidronate.b. IV furosemide.c. IV saline (0.9%).d. IV hydrocortisone.e. IV calcitonin.

Hb 11 g/dlWCC 11 � 109/lPlatelets 100 � 109/lSodium 139 mmol/lPotassium 5 mmol/lUrea 20 mmol/lCreatinine 190 �mol/lCalcium 3.2 mmol/lPhosphate 1.4 mmol/lAlbumin 32 g/l

A 40-year-old woman presented at the Accident andEmergency De partment with a 48-hour history of headacheand neck stiffness. She had returned from a holiday in the

south of France just one week previously. On examinationshe had a temperature of 37.8°C (100.0°F). Her bloodpressure measured 120/70 mmHg. There was no rash.Neurological examination was normal, with the exceptionof mild nuchal rigidity. Investigations are shown.

Question 51

What is the diagnosist?a. Herpes simplex encephalitis.b. Listeria meningitis.c. Viral meningitis.d. Lyme disease.e. Bacterial meningitis.

CT scan of brain NormalCSF analysis: Appearance – clearWCC 20 per mm3

(80% lymphocytes)Protein 0.48 g/lGlucose 2.3 mmol/l (simultaneous

blood glucose 4.5 mmol/l)Gram stain No organism identified

5522aa

Question 52

Which five statements about this 12-lead ECG (52a)are correct?

a. Ventricular tachycardia.b. Left axis deviation.c. Voltage for left atrial enlargement.d. LBBB.e. Right axis deviation.f. Voltage criteria for right ventricular hypertrophy.g. RBBB.h. Voltage criteria for right atrial enlargement.i. Voltage criteria for left ventricular hypertrophy.j. Dissociated P-waves.k. Capture beats.l. Delta waves.

Page 59: Rapid review of clinical medicine for mrcp part 2

58

The patient has symptomatic hypercalcaemia. In generalpatients with hypercalcaemia rarely exhibit symptoms ifthe serum calcium is below 3 mmol/l. Treatment ofsymptomatic hypercalcaemia has to be instituted beforethe underlying cause is identified (Table). The mostimportant aspect of management is vigorous rehydration.Most patients are dehydrated owing to fluid loss from thekidneys resulting from nephrogenic diabetes insipidus.Fluid loss may also occur from the gastrointestinal tractowing to vomiting. The general recommendation is totreat with 4 litres of saline over 24 hours and reviewhydration status. Some guidelines recommend theconcomitant use of furosemide, as this promotes hyper -calcuria; however, in diuresis it may worsen dehydrationand is best reserved for patients who exhibit fluidoverload after rehydration.

All patients should receive intravenousbisphosphonates after rehydration. Most patients respondrapidly to bisphosphonate therapy. The drug can cause abone pain, transient pyrexia, flu-like illness, rashes oriritis. Patients who do not respond to bisphosphonatesmay benefit from intramuscular calcitonin but thehormone is usually poorly tolerated owing to flushing,nausea and diarrhoea. Some cases of resistanthypercalcaemia respond to high-dose corticosteroidtherapy. Following the acute treatment of hypercalcaemiathe aim is to identify and treat the underlying cause.

Answer 50

c. IV saline (0.9%).

Management of symptomatic hypercalcaemia

Step 1 Rehydration with intravenous salineStep 2 Intravenous bisophosphonate therapyStep 3 Identify and treat underlying cause if

possible

The patient has a lymphocytic meningitis with a normalCSF protein and a very slight reduction in CSF glucose(Table). The differential diagnosis is between viralmeningitis (and some cases of viral meningitis do cause avery slight reduction in glucose) or meningitis due tobrucellosis, lyme disease, listeria monocytogenes, andpartially treated bacterial meningitis. The patient has beento the south of France and one could infer that she mayhave indulged in soft cheeses and developed listerosis;however, listerosis usually affects immuno compromised

patients including neonates and pregnant women. Thepatient has none of the pointers for diagnoses other thanviral meningitis. (See Question 267.)

Answer 51

c. Viral meningitis.

Normal CSF values

Opening pressure 60–150 mmH2OProtein content 0.2–0.4 g/lGlucose <2/3 blood glucoseWhite cell count <5/mm3

Red cells 0IgG 15% of total CSF protein

The criteria for left and right atrial and ventricularhypertrophy are discussed in Answer 259. Bundle branchblock is characterized by a wider QRS complex (morethan 120 ms; three small squares on the ECG). In RBBBthe QRS complex in lead VI has an rsR pattern and inLBBB the QRS complex in lead VI has a qS pattern(52b)

Answer 52

b. Left axis deviation.c. Voltage for left atrial enlargement.f. Voltage criteria for right ventricular hypertrophy.g. RBBB.h. Voltage criteria for right atrial enlargement.

5522bbRBBB LBBB

Lead VIr

s

Rq

S

Page 60: Rapid review of clinical medicine for mrcp part 2

Clinical Cases 59

A 63-year-old male with a known history of myocardialinfarction, hypertension and peripheral vascular disease is

admitted with breathlessness. The blood pressuremeasured 158/96 mmHg. Blood results are shown.

He was treated with diuretics and then commenced onan ACE inhibitor but after 72 hours the urea was 20 mmol/l and creatinine was 250 mmol/l. Echocardio -graphy revealed mild impairment of left ventricularfunction.

Question 53

What is the most probable cause for his presentation?a. Myocardial infarction.b. Hypertensive cardiomyopathy.c. Renal artery stenosis.d. Cardiac tachycarrhythmia.e. Myocardial dysfunction secondary to acidosis.

Sodium 137 mmol/lPotassium 4.5 mmol/lUrea 11 mmol/lCreatinine 130 mmol/lTroponin T <0.1ng/l (NR <0.1 ng/l)Arterial gases (room air):

pH 7.31PaCO2 6.4 kPaPaO2 7.1 kPaBicarbonate 16 mmol/l

Chest X-ray Severe pulmonary oedemaECG LBBBUrinalysis Protein 0

Blood 0

A 74-year-old woman presented with a transient syncopalepisode. There was a previous history of paroxysmal atrialfibrillation and congestive cardiac failure. Medications onadmission included amiodarone 200 mg od, furosemide40 mg od, enalapril 20 mg od and warfarin. The ECG onadmission is shown (54a). Electrolytes were as follows:

While on the ward the patient complained of feelingunwell. A repeat ECG is shown (54b). The patient wasfully conscious. The blood pressure was 160/60 mmHg.

Question 54

Sodium 138 mmol/lPotassium 4.2 mmol/lUrea 11 mmol/lCreatinine 128 �mol/lMagnesium 0.8 mmol/l (NR)

What is the immediate treatment of the arrhythmiashown on 54b?

a. IV amiodarone 300 mg.b. IV metoprolol 5 mg. c. IV isoprenaline infusion.d. IV magnesium bolus (8 mmol/l).e. IV lignocaine 100 mg.

5544aa

5544bb

Page 61: Rapid review of clinical medicine for mrcp part 2

60

The patient presents with acute pulmonary oedema. Hehas a background history of ischaemic heart disease,peripheral vascular disease and hypertension. Thedifferential diagnosis is between heart failure secondary tohypertensive heart disease, myocardial infarction or flashpulmonary oedema secondary to renal artery stenosis.The possibility of renal artery stenosis should always beconsidered in a patient with peripheral vascular diseaseand impaired renal function. The serum cardiac troponinis not raised, excluding myocardial infarction. The leftventricular function is only mildly impaired and shouldnot cause such severe pulmonary oedema unless thepatient suffered an episode of myocardial stunning due to

severe ischaemia resulting from major atheromatousdisease in the proximal left coronary artery, or a veryrapid tachyarrhythmia. The diagnosis of renal arterystenosis is evident by the doubling of serum creatininewithin three days of starting an ACE inhibitor.

Flash pulmonary oedema is a recognized complication ofrenal artery stenosis due to diastolic dysfunction fromchronic hypertension. Presumably labile hypertension andsalt and water retention through activation of therenin–angiotensin–aldosterone system are responsible. Flashpulmonary oedema is usually a feature of bilateral renalrather than unilateral renal artery stenosis. The diagnosiscan be made using low-contrast CT scan of the kidneys ormagnetic resonance angiography of the renal arteries.

The causes of simultaneous cardiac and renal failureare tabulated below.

Answer 53

c. Renal artery stenosis.

Causes of simultaneous cardiac and renal failure

• Chronic hypertension• Diabetes mellitus• Generalized atherosclerosis (coronary and reno-vascular disease)• Polyarteritis nodosa and other vasculitides• Systemic sclerosis• Antiphospholipid syndrome• Infective endocarditis• Amyloidosis• Ethylene glycol poisoning

The initial ECG shows a nodal bradycardia with markedprolongation of the QT interval and U waves. The serumpotassium and magnesium are normal. The mostprobable cause of the bradyarrhythmia and theprolongation of the QT is amiodarone toxicity.Prolonged QT is usually associated with polymorphic VT(torsades de pointes) but monomorphic ventricular

tachycardia is also recognized. The most effective urgenttreatment to terminate the VT is IV magnesium even inthe presence of a normal serum magnesium level. Theamiodarone should be stopped immediately. The patientwill be at risk of further ventricular tachycardia while theQT remains prolonged. The QT interval may beshortened by an isoprenaline infusion or by temporarycardiac pacing at a ventricular rate >60/min. It may takeseveral days before the effects of amiodarone are reversed.

Answer 54

d. IV magnesium bolus (8 mmol/l).

Page 62: Rapid review of clinical medicine for mrcp part 2

Clinical Cases 61

A 69-year-old male consulted his GP with pleuritic chestpain and had reduced air entry at the left lung base. Hehad experienced a dry cough, fever and myalgia for 48hours prior to the onset of the chest pain. He was a non-smoker. On examination he was alert and orientated witha mini-mental score test of 10/10. The blood pressurewas 100/66 mmHg. The respiratory rate was 32/min.

Investigations are shown.

The patient received seven days of appropriate antibiotictherapy but remains unwell with intermittent pyrexia andanorexia. A repeat chest X-ray revealed findings consistentwith a pleural effusion. A diagnostic aspiration wasperformed and analysis of the effusion is shown below.

Questions 55 and 56

Question 55

What is the management of this patient?a. Treat in the community with amoxycillin.b. Treat in the community with amoxycillin and

clarythromycin.c. Admit to hospital and treat with oral amoxycillin

and clarythromycin.d. Admit to hospital and treat with IV cefuroxime

and oral clarythromycin.e. Admit to hospital and treat with ciprofloxacin.

Hb 14 g/dlWCC 9 � 109/lPlatelets 480 � 109/lCRP 60 g/lSodium 133 mmol/lPotassium 4.1 mmol/lUrea 7.5 mmol/lCreatinine 110 �mol/lChest X-ray Left lower lobe

consolidationOxygen saturation on air 94%

Question 56

What is the ongoing management of this patient?a. Continue with current antibiotic therapy for

another week and repeat CXR.b. Initiate anti-tuberculous therapy.c. Organize CT scan thorax.d. Perform pleural biopsy.e. Drain effusion using an intercostal drain and

continue with aggressive antibiotic therapy.

Aspiration:Appearance Purulent Gram stain NegativeCulture SterilepH 7.1Pleural fluid LDH 1000 iu/l

(Plasma LDH 200 iu/l) Pleural fluid protein 40 g/l

(Total plasma protein 72 g/l)

A 19-year-old girl was seen by her GP complaining of a24-hour history of nausea and vomiting. She wasprescribed some medication. Later that evening herboyfriend brought her into the Accident and EmergencyDepartment stating that she had been unable to speakand had a very stiff neck.

On examination she appeared well. She was alert andorientated without any difficulty with her speech. She wasunable to account for the episode encountered by herboyfriend but denied any previous similar ‘attacks’. Whilebeing examined she was seen to become rigid. Her jawwas fixed open and her eyes were deviated up and to the left.

Question 57

Which single step would you take in the managementof this patient?

a. Intravenous diazepam.b. Intravenous procyclidine.c. Intramuscular haloperidol.d. Reassure strongly.e. Guanidine hydrochloride.

Page 63: Rapid review of clinical medicine for mrcp part 2

62

The patient has a community-acquired pneumonia (CAP)as evidenced by symptoms and signs of a lower respiratorytract infection and consolidation on the chest X-ray.

Streptococcus pneumoniae is the most frequently isolatedorganism in CAP, followed by Haemophilus influenzae andthen Mycoplasma pneumoniae. Accurate assessment of theseverity of pneumonia is necessary to guide the need forhospitalization and/or intravenous antibiotic therapy. Inthe UK, the CURB-65 score has been adopted for assessingthe severity of pneumonia and stratifying patients intoprognostic groups and management pathways (algorithm).Patients with a CURB-65 score of 3 or more have severeCAP and require hospitalization and treatment with intra -venous antibiotics (Table). Other markers of poor prognosisin relation to pneumonia include a white cell count <4 or>20 �109/l and an albumin concentration <30 g/l.

Recommended antibiotic therapy for severe CAP is abeta-lactam based antibiotic in combination with amacrolide (Table). In this particular case, where theCURB-65 score is 3, the patient should be admitted tohospital and receive intravenous cefuroxime and oralclarythromycin.

Answers 55 and 56

Answer 55

d. Admit to hospital and treat with IV cefuroxime and oral clarythromycin.

Antibiotic recommendations for CAP

Preferred AlternativeHome Oral amoxycillin Oral erythromycintreated or

Oral clarythromycinHospital Oral amoxycillin Oral levofloxacintreated + or(not Oral erythromycin Oral moxifloxacinsevere) or

Oral clarythromycinHospital IV co-amoxiclav Oral levofloxacintreated or +(severe) IV cefuroxime IV benzylpenicillin

orIV cefotaxime

+Oral erythromycin

orOral clarythromycin

The pneumonia has been complicated by an empyema,which is characterized by frank pus in the pleural space.Organisms may be seen on Gram stain or isolated duringculture. Biochemical investigation of the purulent fluidwill reveal high protein and LDH concentrations and lowpH and glucose concentration. The treatment ofempyema involves drainage with an intercostal drainfollowed by a prolonged course of antibiotic therapy,including anaerobic cover.

Parapneumonic effusions complicate approximately50% of all pneumonias. Most effusions are sterile andresolve with conventional antibiotic therapy. A smallproportion become infected (complicated parapneumoniceffusion or empyema) and require drainage. Features ofcomplicated parapneumonic effusions include highprotein and LDH concentration (patients with empyemacharacteristically have an LDH concentration >1000 iu)and low pH (<7.3) The presence of frank pus, organismson pleural fluid Gram stain or culture and low pleuralfluid pH are indications for prompt drainage using anintercostal drain.

Answer 56

e. Drain effusion using an intercostal drain and continue with aggressive antibiotic therapy.

One point for any one of the followingConfusion (Mental test score ≤ 8/10)

Urea > 7 mmol/lRespiratory rate ≥ 30/min

Blood pressure (SBP < 90 mmHg or DBP≤60 mmHg)

Age ≥ 65 years

Consider hospitalsupervised

treatment eitherShort stay inhospital or

hospitalsupervised out-

patient

Manage inhospital assevere CAP

Assess forICU if

CURB is4 or 5

Hometreatment

CURB-65score

2

GROUP 3Mortality

high (22%)

CURB-65score0 or 1

GROUP 2Mortality

intermediate(9.2%)

CURB-65score

3 or more

GROUP 1Mortality low

(1.5%)

Page 64: Rapid review of clinical medicine for mrcp part 2

Clinical Cases 63

A 10-year-old boy presented with a three-week history ofswollen ankles. On examination his blood pressure

measured 100/60 mmHg. The JVP was not raised. Bothheart sounds were normal and his chest was clear.

Investigations are shown.

A 72-year-old male presented with a two-month historyof right-sided chest discomfort and weight loss. The

symptoms were present most of the day and relievedpartly by paracetamol. He was a heavy smoker and had asix-year history of chronic obstructive pulmonary diseaserequiring three hospital admissions. He had just finisheda two-week course of high-dose prednisolone two weeksbefore developing chest discomfort. He had been treatedfor pulmonary tuberculosis at the age of 34 years.

On examination he was afebrile. He did not havefinger clubbing. The percussion note at the base of theright lung was dull. Auscultation of the lungs revealedreduced air entry at the right lung base.

Investigations are shown.

Question 58

What investigation is required next?a. Bronchoscopy.b. Contrast enhanced thoracic CT scan.c. Ventilation/perfusion scan.d. Echocardiogram.e. Pleural biopsy.

Hb 11 g/dlWCC 12 � 109/lPlatelets 300 � 109/lSodium 131 mmol/lPotassium 3.6 mmol/lUrea 7 mmol/lGlucose 4.8 mmol/lProtein 60 g/lLDH 180 iu/lChest X-ray Hyperinflated left lung

Moderate pleural effusion right lung

Pleural fluid:Protein 29 g/lLDH 180 iu/lGlucose 3 mmol/lGram stain NegativeCytology Mesothelial cells only

Question 59

Hb 12 g/dlWCC 5 � 109/lPlatelets 200 � 109/lBlood film NormalSodium 134 mmol/lPotassium 3.7 mmol/lUrea 6 mmol/lCreatinine 90 �mol/lAlbumin 21g/l24-hour urine protein 3.6 g/l

What is the first-line management step?a. Oral high-dose prednisolone therapy.b. No therapy (mere observation).c. Intravenous furosemide infusion.d. Perform renal biopsy before instituting

treatment.e. Cyclophosphamide.

The description is typical of acute dystonia followingtreatment for vomiting. The culprit agent is a neurolepticagent used to treat vomiting, classically metoclopramideor prochlorperazine. Both drugs work by blocking

dopaminergic type 2 receptors in the mid-brain. Side-effects include acute dystonia. Chronic use is associatedwith tardive dyskinesia. Treatment is with intravenousprocyclidine or benzotropine, which are both anti -cholinergic agents, or intramuscular diphenhydramine,which is an anti-histamine.

Answer 57

b. Intravenous procyclidine.

Page 65: Rapid review of clinical medicine for mrcp part 2

64

The patient is a smoker and presents with right-sidedchest discomfort and has a pleural effusion. The pleuralfluid protein concentration is <30 g/l. Sole reliance onthe pleural fluid protein content would result in anerroneous misinterpretation of a transudative effusion.However, pleural fluid protein measurements should beinterpreted in the context of the serum proteinconcentration. In patients with abnormal serum proteinor pleural fluid protein concentrations close to 30 g/lmore stringent criteria are required to differentiate apleural transudate from an exudate (Table A).

According to the Light’s criteria, the effusion has thebiochemical features of an exudate. The pleural glucose

concentration is also low (Table B). The differentialdiagnosis is between malignant pleural involvement,complicated parapneumonic effusion, and tuberculouseffusion. There is no obvious evidence of an infection andin the context of a long history of smoking, the prospectof malignancy has to be considered. Indeed malignancy isthe commonest cause of an exudative pleural effusion inpatients aged >60 years. Cytological analysis revealsmalignant cells in <25% of cases with malignant pleuraleffusions. The patient should have a contrast enhancedCT scan followed by a CT guided or thoracoscopicpleural biopsy. Contrast enhanced CT scan shouldprecede pleural biopsy in most cases of undiagnosedeffusion. Bronchoscopy is not helpful unless there arespecific features to suggest an underlying bronchialmalignancy such as haemoptysis.

Answer 58

b. Contrast enhanced thoracic CT scan.

Table A Light’s criteria for pleural fluid exudates

• Pleural fluid protein to serum protein ratio >0.5• Pleural fluid LDH to serum LDH ratio >0.6• Pleural fluid LDH >2/3 the upper limit of

normal serum LDH

Table B Causes of low pH and low glucose effusions

• Complicated parapneumonic effusion orempyema

• Malignant pleural effusion• Rheumatoid pleuritis• Lupus pleuritis • Tuberculous pleural effusion• Oesophageal rupture (also has high amylase)

The patient has nephrotic syndrome. The commonestcause of nephrotic syndrome in children is minimal changeglomerulonephritis, accounting for 90% of all cases ofnephrotic syndrome in children aged below 10 years, and50% of children aged above 10 years. The causes ofnephrotic syndrome are tabulated below (Table A).

A trial of high-dose prednisolone (1 mg/kg per day) isthe first-line management step in children presentingwith pure heavy proteinuria and peripheral oedema. Mostchildren achieve remission within two weeks and almost

all children are in remission within eight weeks. Patientsrefractory to high-dose steroids are treated withcyclophosphamide, ciclosporin or chlorambucil.Furosemide infusion is rarely indicated unless the patienthas marked peripheral oedema.

Adults presenting with heavy proteinuria have minimalchange glomerulonpehritis in 20% of cases and generallyrespond more slowly to, and relapse more frequently on,steroid therapy. Therefore a renal biopsy is generallyadvocated to help elucidate the exact cause beforeinstituting therapy. In contrast renal biopsy in children isonly indicated in rare circumstances (Table B). (SeeQuestion 137.)

Answer 59

Table A Causes of minimal changeglomerulonephritis

• Immune• Non-steroidal anti-inflammatory drugs• Hodgkin's lumphoma• Food allergy (cow’s milk)• Bee-sting reaction• Lithium • Pamidronate

a. Oral high dose prednisolone therapy.

Table B Indications for renal biopsy inchildren with heavy proteinuria

• Presence of red cells/casts in the urine• Obvious evidence of a multi-system vasculitis• Patients with frequent relapses• Patients refractory to conventional therapy

Page 66: Rapid review of clinical medicine for mrcp part 2

Clinical Cases 65

A 50-year-old shipyard worker presented with a singleepisode of haemoptysis comprising approximately oneegg cup-full of fresh blood. He gave a two-year history ofprogressively increasing breathlessness and a persistentcough. He had never experienced any chest pain. Hestopped smoking two months previously after a chestinfection which took several weeks to resolve, but beforethat was smoking 20–30 cigarettes/day. He kept pigeonsin his teenage years, but had not had any direct contactwith birds since. The only other past medical history ofnote was a four-year history of mild hypertension, forwhich he was taking propranolol 80 mg three times daily.

On examination, he was centrally cyanosed and had earlyfinger clubbing. The respiratory rate was 23/min. Onexamination of the respiratory system the chest expansionwas symmetrical, but generally reduced. The trachea wascentral. On auscultation of the lung fields there were fineend-inspiratory crackles at both lung bases. Precordialexamination was normal, with the exception of a loudpulmonary component to the second heart sound.

Investigations are shown.

Question 61

Hb 14 g/dlWCC 9 � 109/lPlatelets 190 � 109/lMCV 91 flESR 64 mm/hSodium 132 mmol/lPotassium 3.7 mmol/lUrea 7 mmol/lCreatinine 91 �mol/lArterial blood gases:

pH 7.44PaO2 7.1PaCO2 3.4Bicarbonate 21O2 saturation 75%

Chest X-ray Irregular, linear shadowing at both lung bases

Sputum microscopy No abnormalitiesFlow loop (61)

1: What is the cause of the haemoptysis?a. Mesothelioma.b. Carcinoma of the bronchus.c. Extrinsic allergic alveolitis.d. TB.e. Pulmonary embolus.

2: What is the cause of his breathlessness?a. Chronic obstructive airways disease.b. Therapy with propranolol.c. Extrinsic allergic alveolitis.d. Cryptogenic fibrosing alveolitis/diffuse

pulmonary fibrosis.e. Asbestosis.

6611

A 68-year-old man presented with a cholestatic bloodpicture, and hepatobiliary ultrasound revealed dilatedextra-hepatic ducts. He underwent an ERCP which wascomplicated by E. coli septicaemia. One week followingthe procedure the renal function was as follows:

The urea and electrolytes prior to ERCP were normal.On examination he was clinically euvolaemic. His heart

rate was 80 beats/min and regular, and his bloodpressure measured 130/80 mmHg. A urinary catheterwas placed in the bladder and he was passing 40 ml ofurine per hour.

Question 60

Sodium 131 mmol/lPotassium 4.9 mmol/lUrea 28 mmol/lCreatinine 430 �mol/l

Which of the following agents are of proven benefitin improving renal function in a situation such as this?

a. High-dose steroids.b. Intravenous dopamine.c. High-dose furosemide infusion.d. Intravenous mannitol.e. None of the above.

0 1 2 3 4 5 6 7 8Change in lung volume (l)

10

8

6

4

2

0

2

4

6

8

10

Flow(l/sec) Normal

Patient

Page 67: Rapid review of clinical medicine for mrcp part 2

66

The main features of this case are a long-term smoker whopresents with haemoptysis. He has worked in the shipyard,an occupation which was historically associated withasbestos exposure. He has objective evidence of lungfibrosis; he is hypoxic, his chest X-ray shows linearshadowing in the lower zones and the flow loop isconsistent with shrunken lungs. In the context of this casethe most likely cause for lung fibrosis and breathlessness isasbestosis. The history of keeping pigeons may implicateextrinsic allergic alveolitis as a possible cause; however, ithas been a long time since he was exposed to avianantigens. Furthermore, EAA classically causes upper zonereticulo-nodular shadowing on the chest X-ray, whereasasbestosis is associated with shadowing in the lower zones.

While there are several causes of haemoptysis, themost probable cause in this case is bronchial carcinoma.Both bronchial carcinoma and mesotheliomamalignancies are associated with asbestos exposure.However, the chest X-ray does not mention obviouspleural thickening or pleural effusion, which are classicfeatures of mesothelioma; therefore an underlyingbronchial carcinoma would need to be excluded.Smoking and asbestos exposure together increase the riskof bronchial carcinoma several fold. With the exceptionof EAA, all the possibilities mentioned may also causehaemoptysis; however, the history is atypical forpulmonary embolus and tuberculosis would be lowerdown in the differential diagnosis than lung malignancy.

Answer 61

1: b. Carcinoma of the bronchus.2: e. Asbestosis.

The patient has probably developed renal failuresecondary to acute tubular necrosis arising from E. colisepticaemia. Acute tubular necrosis generally lastsbetween one and three weeks. Most patients make arecovery and return to their normal or near normal renalfunction. The management of acute renal failure due toacute tubular necrosis is supportive.

Nephrotoxic agents are stopped or avoided.Hypovolaemia is corrected with intravenous saline whilstmonitoring central venous pressure. Urine output ismonitored with a urinary catheter in situ. Once thepatient is euvolaemic, the quantitative daily fluidreplacement is equal to the volume of urine output plusan additional 500 ml to allow for insensible losses. Thepatient is carefully monitored for signs of fluid overload

or uraemic encephalopathy. The serum urea, electrolyteand arterial pH are monitored closely.

Acute haemodialysis is recommended in patients withrenal failure associated with fluid overload causingpulmonary oedema, uraemic encephalopathy, dangeroushyperkalaemia (≥7 mmol/l) or a low arterial pH.

There is no proven therapy to help expedite recovery.Although dopamine has been advocated in the past,prospective controlled studies have failed to show anyimprovement in renal function in acute renal failurepatients treated with dopamine. Furosemide has beenconventionally used to convert oliguric acute tubularnecrosis to non-oliguric tubular necrosis, however,studies suggest that the use of high-dose diuretics may beassociated with death and retard recovery of renalfunction. Steroids have no role in the management ofacute tubular necrosis but are helpful in the managementof acute glomerulonephritis.

Answer 60

e. None of the above.

Page 68: Rapid review of clinical medicine for mrcp part 2

Clinical Cases 67

A 62-year-old male presented with fever andbreathlessness four weeks after a prosthetic aortic valvereplacement for aortic stenosis. On examination heappeared pale and had a temperature of 38°C (100.4°F).Auscultation of the heart revealed a prosthetic secondheart sound and a long early diastolic murmur.Auscultation of the lungs revealed inspiratory crackles atboth lung bases.

Initial investigations are shown.

Questions 62 and 63

Hb 11 g/dlWCC 15 � 109/lPlatelets 400 � 109/lESR 70 mm/hSodium 140 mmol/lPotassium 4.2 mmol/lUrea 8 mmol/lCreatinine 130 �mol/l12-lead ECG Left bundle branch block (old)Chest X-ray Cardiomegaly and pulmonary

oedemaQuestion 62

Which two of the following investigations willprovide the most diagnostic information?

a. C-reactive protein.b. Renal ultrasound.c. ASO titres.d. Serial blood cultures.e. Transthoracic echocardiography.f. Urinalysis for blood.g. Complement fixation tests for Coxiella burnetii.h. Transoesophageal echocardiography.

Question 63

Which organism is most likely to be present in theblood culture?

a. Streptococcus viridans.b. Staphylococcus aureus.c. Staphylococcus epidermidis. d. Organism from the HACEK group.e. Enterococci.

A 38-year-old male was investigated for abnormal liverfunction tests following investigation of intermittentepisodes of diarrhoea. He was generally fit and well. Heconsumed no more than 10 units of alcohol per week.He did not take any medications. He had a past history

of jaundice after a visit to India 8 years previously, whichhad resolved spontaneously.

Investigations are shown.

Question 64

Hb 11 g/dlWCC 7 � 109/lPlatelets 190 � 109/lCRP 12 g/lBilirubin 19 μmol/lALT 32 iu/lAlkaline phosphatase 178 iu/lAlbumin 33 g/lThyroxine 102 nmol/lTSH 1.2 nmol/lHepatobiliary ultrasound NormalERCP (64)

What is the cause of the raised alkaline phosphatase?a. Primary sclerosing cholangitis.b. Primary biliary cirrhosis.c. Stones in the common bile duct.d. Amoebic liver abscess.e. Hepatic vein thrombosis.

6644

Page 69: Rapid review of clinical medicine for mrcp part 2

68

Indications for surgery in valve endocarditis

Prosthetic valve endocarditis• Early PVE in first 2 months or less after surgery• Murmurs suggestive of valve dysfunction• Moderate to severe heart failure• Annular or aortic root abscess or new cardiac

conduction abnormalities on ECG• Persistent fever for 10 or more days despite

appropriate antibiotic therapy • Staphylococcus aureus or fungi cultured from the

blood

Native valve endocarditis• Acute AR or MR with heart failure• Annular or aortic root abscess• Fungal endocarditis• Evidence of persistent infection despite

appropriate antibiotic therapy for 10 days (fever,leucocytosis and bacteraemia)

The patient has developed a fever and murmur of aorticregurgitation only four weeks after a prosthetic aorticvalve replacement, indicating early prosthetic valveendocarditis. In early PVE, micro-organisms usually reachthe prosthesis by direct contamination during the intra-operative period or via haematogenous spread severaldays or weeks after surgery. The consequences of earlyPVE are grave as the organisms have direct access to theprosthesis annulus-interface and to perivalvular tissuearound the sutures lines since the valve is notendothelialized. Patients with early PVE commonlydevelop valve dehiscence and annular abscesses. The riskof embolic phenomena is also greater with PVE. Theclinical features are similar to native valve endocarditis.

Diagnosis relies on culturing the organisms from theblood. Three sets of blood cultures should be taken atintervals of >1 hour within the first 24 hours if thediagnosis is highly likely in a sick patient. If the patient isnot acutely ill or when the diagnosis is not obvious, sixsets of blood cultures should be taken within the first 24to 48 hours. The commonest organisms cultured in earlyPVE are Staphylococcus epidermidis followed byStaphylococcus aureus and then fungi. In contrast,organisms cultured in late PVE are similar to thosecausing infection on native valves.

Transoesophageal echocardiography is theinvestigation of choice and is advocated as the first-lineechocardiographic modality in the investigation of PVE.TOE is much more sensitive than transthoracicechocardiography at identifying vegetations on theprosthetic valve. It is also superior at identifyingcomplications such as paravalvular abscesses and fistulaformation.

The treatment comprises high-dose intravenousantibiotics for six weeks. The need for surgical inter -vention is much higher than with native valveendocarditis. Indications for surgery in PVE and nativevalve endocarditis are tabulated (Table).

Answers 62 and 63

Answer 62

d. Serial blood cultures.h. Transoesophageal echocardiography.

Answer 63c. Staphylococcus epidermidis.

The ERCP shows multiple strictures and dilatations inthe intrahepatic ducts and in the left and right hepaticducts. The finding is characteristic of primary sclerosingcholangitis, which is a chronic progressive inflammatory

disorder of medium and small-sized bile ducts. Thehistory of intermittent loose stool in the patient isparticularly relevant in this question because over 90% ofcases are associated with ulcerative colitis. The mostuseful diagnostic test is an ERCP or a MRCP (magneticresonance cholangiopancreatogram). A liver biopsy israrely diagnostic. (See Questions 6 and 104.)

Answer 64

a. Primary sclerosing cholangitis.

Page 70: Rapid review of clinical medicine for mrcp part 2

Clinical Cases 69

A 52-year-old man was admitted to the intensive careunit with difficulty in breathing after a coryzal illness. Hehad a six-week history of rapidly progressive muscularfatigue which was worse at the end of the day. Morerecently he had had difficulty with speech and chewingfood while eating.

On examination he had bilateral ptosis and markedfacial muscle weakness. Power in all four limbs wasdiminished. The heart rate was 90 beats/min andregular. The respiratory rate was 30/min but respirationwas shallow. Investigations are shown.

His condition deteriorated and he was transferred tothe intensive care unit for invasive ventilation.

Question 65

Hb 15 g/dlWCC 9 � 109/lPlatelets 200 � 109/lElectrolytes NormalChest X-ray Elevation of both hemi-

diaphragmsLungs normal

Arterial blood gases:pH 7.1 PaCO2 9 kPaPaO2 7 kPa

FVC (baseline) 0.8 lFVC (after 2 mg 1.2 l

edrophonium)

What is the ideal therapeutic strategy in his immediatemanagement?

a. Emergency thymectomy.b. Intravenous neostigmine.c. Intravenous methylprednisolone.d. Plasmapheresis.e. Azathioprine.

What is the ECG diagnosis (66)?a. Right ventricular hypertrophy.b. Dextrocardia.c. Wolff–Parkinson–White syndrome.d. Left ventricular hypertrophy.e. Right bundle branch block.

Question 66

6666

Page 71: Rapid review of clinical medicine for mrcp part 2

70

The patient presents with acute generalized muscleweakness and respiratory failure. There is a six-weekhistory of muscle fatigue, which is a cardinal feature ofmyasthenia gravis. Response to edrophonium (Tensilon),a short-acting anti-cholinesterase agent, supports thediagnosis of myasthenia gravis, although false-positiveresults are common.

Patients with myasthenia gravis may present withmyasthenic crises, which comprise severe muscleweakness including respiratory and bulbar muscles. Such

crises may be precipitated by intercurrent illness as in thiscase, or they may be spontaneous events. The treatmentfor myasthenic crisis is shown in Table A.

Plasmapheresis directly removes anti-cholinergicreceptor antibodies from the circulation. Treatment isgiven for up to 2 weeks and is effective for 1–2 months.Plasmapheresis is reserved for acute situations orpreparation for thymectomy. Intravenous immuno -globulin is as effective as plasmapheresis in acutemyasthenic crises. The mode of action of intravenousimmunoglobulin is uncertain. Anti-cholinesterase drugtherapy may cause a cholinergic crisis that is alsocharacterized by muscle weakness. The differentiationbetween a cholinergic crisis and a myasthenic crisis isbased upon the presence of abdominal cramps, excessivesweating and bradycardia in the former. However, ahistory is not always available during a crisis, thereforeanti-cholinesterase drugs are withdrawn in patients withmyasthenia gravis who present with features of severegeneralized muscle weakness.

The management of patients with chronic myastheniagravis is summarized in Table B.

Answer 65

d. Plasmapheresis.

Table A Management of myasthenic crisis

Step 1 Elective ventilationStep 2 Withdrawal of anti-cholinesterase drugs

(if applicable)Step 3 Plasmapheresis or intravenous

immunoglobulin

Table B Management of chronic myasthenia gravis

Treatment regime Indications

Oral anti-cholinesterase therapy Mild symptomsOcular myasthenia

Immunosuppressive drugsCorticosteroids To attempt remission in patients with moderate or severe symptoms Azathioprine Failure to remit on steroid therapyCyclosporinThymectomy* Patients with thymoma

* Some neurologists recommend thymectomy even in the absence of thymoma in all patients between pubertyand 60 years of age since most patients have abnormalities of the thymus gland.

The patient has a dominant R wave in V1 which wouldbe consistent with all five choices provided in thequestion. There is a rightward QRS and P wave axis, adeep S wave in lead I and dominant R wave in aVr which

are highly unusual and should raise the suspicion ofeither erroneous upper limb lead transposition ordextrocardia. The progressive dimunition of the R waveson transition from lead V1 to V6 indicates dextrocardia.In pure upper limb lead transposition the R wavetransition in V1 to V6 is normal.

Answer 66

b. Dextrocardia.

Page 72: Rapid review of clinical medicine for mrcp part 2

Clinical Cases 71

An 18-year-old girl presented with a six-month history ofdaytime somnolence that was causing embarrassmentduring lecture theatres and while talking to friends. Shewas apprehensive about driving because she did not haveany recollection about how she had driven from onedestination to another and felt that she may have fallenasleep while driving on a few occasions. She had vividfrightening dreams just as she fell asleep, which wouldwake her frequently. On waking she was unable to movefor a few minutes. Three months previously the patient

underwent cardiac investigations for intermittentepisodes of sudden collapse that occurred when shelaughed out loud. The episodes were not associated withloss of consciousness.

The patient had a past history of insulin-dependentdiabetes mellitus since the age of 7 years that was verywell controlled. She had recently commenced the oralcontraceptive pill but was not on any other medication.There was no family history of note. Investigations areshown.

Question 67

What is the diagnosis?a. Acute intermittent porphyria.b. Episodic hypoglycaemia.c. Epilepsy.d. Narcolepsy.e. Long QT syndrome.

Hb 11 g/dlWCC 15 � 109/lPlatelets 400 � 109/lESR 70 mm/hSodium 140 mmol/lPotassium 4.2 mmol/lUrea 8 mmol/lCreatinine 130 �mol/l12-lead ECG Left bundle branch block (old)Chest X-ray Cardiomegaly and pulmonary

oedema

A 70-year-old man with a known history of ischaemicheart disease and congestive cardiac failure presentedwith nausea and abdominal discomfort. He was takingdigoxin 250 μg , frusemide 40 mg and lisinopril 20 mg.The patient had been recently commenced onamiodarone for atrial fibrillation. On examination theJVP was raised 4 cm above the sternal angle. There was athird heart sound. The liver edge was palpable 3 cmbelow the costal margin and there was mild ankleoedema. The digoxin level was 3 iu (NR 1–2 iu).

Question 68

What is the best action to improve his symptomswithout compromising cardiac function?

a. Stop digoxin.b. Stop amiodarone.c. Stop enalapril.d. Halve the dose of digoxin.e. Halve the dose of amiodarone.

Page 73: Rapid review of clinical medicine for mrcp part 2

72

The patient has abdominal discomfort and nausea. Thedifferential diagnosis for his symptoms includes hepaticcongestion secondary to heart failure, digoxin toxicity orgastro-intestinal side-effects of amiodarone (usually occurwithin a few days of starting the drug). In this particularsituation the plasma digoxin level is raised and hencedigoxin toxicity is the most likely cause (Tables A and B).Amiodarone may promote digoxin toxicity by competingfor digoxin binding sites hence increasing plasma digoxin

levels. The dose of digoxin should be halved whenamiodarone is initiated. The patient had clinical evidenceof cardiac decompensation; therefore, the bestmanagement step would be halve the dose of digoxinrather than stop it altogether. Digoxin is cleared fairlyquickly by the kidneys (one-third of the body contentper day) and the symptoms should settle. Remember thatdigoxin toxicity can cause fatal rhythm disturbances(both tachy- and bradyarrhythmias).

The patient has excessive daytime somnolence,hypnagogic hallucinations, cataplexy and sleep paralysis.This combination is consistent with the diagnosis ofnarcolepsy. Spontaneous recovery of the events isinconsistent with hypoglycaemic episodes. Torsades depointes due to long QT syndrome may present withepisodic collapse during intense emotion; however,consciousness is impaired in collapse associated with longQT syndrome. (See Question 266.)

Excessive daytime somnolence is a cardinal feature ofnarcolepsy. The symptoms are improved after a brief nap.

Hypnagogic hallucinations are vivid and oftenfrightening hallucinations just after falling asleep or justbefore waking. They are effectively a mixture of REMsleep and wakefulness.

Cataplexy is defined as sudden episodes of bilateralmuscle weakness leading to partial or complete collapse. Itis usually triggered by strong emotions such as anger,laughter or intense excitement. Weakness is present for 1–2minutes. Consciousness is not impaired. Almost two-thirdsof patients with narcolepsy go on to develop cataplexy.

Sleep paralysis is a complete inability to move 1 or 2minutes after waking and is often accompanied byhypnagogic hallucinations.

Narcolepsy is caused by the loss of the hypothalamicneuropeptide orexin. Orexins are released from synapticterminals during wakefulness and increase the activity ofbrain regions associated with wakefulness. Narcolepsy hasstrong association with the HLA antigens DR2 and DQ1(HLA DR2 [99%]).

The diagnosis is clinical but an overnightpolysomonogram and a multiple sleep latency test areperformed to exclude other causes of hypersomnolence.Overnight polysomonography may be normal in patientswith narcolepsy but may demonstrate spontaneouswakenings and REM sleep within 20 minutes of fallingasleep. Normal patients do not go into REM sleep for80–100 minutes after the onset of sleep since orexininhibits REM sleep. The following day a multiple sleeplatency test is performed in which a patient is allowed tonap every 2 hours on four or five occasions. Patients withnarcolepsy fall asleep within 5 minutes whereas mostnormal patients take 10–15 minutes to fall asleep. Thenaps in narcolepsy patients induce REM sleep, and thepresence of sleep-onset REM in two or more naps ishighly suggestive of narcolepsy.

Treatment is with modafinil, a non-amphetaminewake-inducing drug. The alternative is methylphenidate,an amphetamine drug.

Answer 67

d. Narcolepsy.

Answer 68

d. Halve the dose of digoxin.

Table A Features of digoxin toxicity

• Gastro-intestinal effectsAnorexia, nausea, vomiting, abdominal pain and diarrhoea

• Xanthopsia• Neuralgia• Cardiac rhythm disturbances• Headache• Dizziness• Epileptic seizures

Table B Causes of digoxin toxicity

• Electrolyte disturbancesHypokalaemiaHypomagnesaemiaHypercalcaemia

• Hypothyroidism• Renal impairment• Drugs

Amiodarone Increases plasma levels by displacing it from binding proteins

Calcium antagonistsCiclosporinQuinidineACE inhibitors May reduce renal clearance

Page 74: Rapid review of clinical medicine for mrcp part 2

Clinical Cases 73

A 15-year-old Nigerian female was admitted with rightupper quadrant abdominal pain. She had a history ofsickle cell anaemia, and had several admissions with handand foot crises. During the last two admissions shecomplained of abdominal pain. She was born in England.She was the only child of a middle-class family.

On examination, she was distressed with pain. Thetemperature was 37.6°C (99.7°F). She was mildlyjaundiced. The pulse was 105 beats/min, and bloodpressure 100/75 mmHg. On examination of therespiratory system there was dullness to percussion on theanterior aspect of the right lung. Abdominal examinationrevealed tenderness in the right upper quadrant. The liverwas palpable 3 cm below the costal margin.

Investigations are shown.

Hb 6 g/dlWCC 12 � 109/lPlatelets 200 � 109/lU&E NormalAST 50 iu/lAlkaline phosphatase 150 iu/lAlbumin 34 g/lUrine osmolality 120 mOsm/kgUrinalysis Urobilinogen +++

Blood 0Protein 0Bilirubin 0

Chest X-ray Opacification in the right upper zone

Hb 12 g/dlWCC 7 � 109/lPlatelets 140 � 109/lPT 13 s (control 12 s)APTT 59 s (control 40 s)Antinuclear Not detected

antibody factorDouble-stranded Not detectedDNA

1. What is the cause of the abdominal pain?2. What investigation would you request next?3. What is the cause of the girl’s respiratory and chest

X-ray findings?4. Explain the urine osmolality.

A 25-year-old female presented with a right calf deep-vein thrombosis.

Blood results are shown.

1. What is the diagnosis?2. List three tests you would perform to confirm the

diagnosis.3. Apart from thrombotic tendencies, list other

manifestations of this condition.

Question 69

Question 70

Page 75: Rapid review of clinical medicine for mrcp part 2

74

Sickle cell anaemia is discussed in detail in Answer 263.This question tests the reader’s knowledge ofcomplications secondary to sickle cell anaemia.

The story is typical of cholecystitis which, in this case, issecondary to pigment gallstones. Hepatic infarction due toa sickling crisis in the liver is unlikely because it would beassociated with a higher AST. Sequestration is unlikelybecause it is very rare in older patients with sickle cellanaemia. The shadowing on the chest X-ray is probablysecondary to streptococcal pneumonia. Patients with sicklecell anaemia are hyposplenic as a result of autosplenicinfarction. As a result, they are predisposed to bacterialinfections by capsulated organisms such as Streptococcuspneumoniae, Neisseria meningitidis and Haemophilusinfluenzae. Pulmonary infarction may also causepulmonary shadowing on the chest X-ray – but is less likelyin the absence of pleurisy – and severe breathlessness.

Sickle cell anaemia is inherited as an autosomal recessivecondition. A point mutation in the β-globin gene leads tothe substitution of glutamine by valine at position 6. Inthe deoxygenated state, this leads to sickling of cells.Chronic low-grade haemolysis is common and predisposesto pigment gallstones. Painful sickle cell crises are due toocclusion of small vessels, producing severe pain fromaffected organs. The bones are most commonly involved,particularly those of the hands and feet. Other crisesinclude haemolytic, aplastic and sequestration crises(Table). An aplastic crisis may complicate infection withparvovirus B17. Sequestration crises are fatal and usuallyoccur in young patients. Precipitating factors for sicklinginclude hypoxia, infection, dehydration and pain.

Sickle cell crises

• Thrombotic (hand foot syndrome, abdominal pain,pulmonary infarction and strokes)

• Haemolytic• Aplastic • Sequestration

1. Acute cholecystitis.2. Ultrasound examination of the liver.3. Right upper lobe pneumonia.4. Chronic tubulo-interstitial nephritis or renal papillary

necrosis leading to impaired concentrating ability.

Answer 69

Answer 70

The antiphospholipid syndrome usually affects youngwomen and is characterized by arterial and venousthromboses and recurrent abortions (Table). There maybe evidence of thrombocytopenia. It is caused bycirculating antiphospholipid antibodies, which are IgGand IgM immunoglobulins directed against phospholipidmolecules. They were first described in SLE, but may bepresent in a wide variety of autoimmune conditions andhave been associated with multiple sclerosis, acquiredimmunodeficiency syndrome (AIDS) and carcinomatosis.

Antiphospholipid antibodies can be present in patientstaking medications such as phenytoin, procainamide andhydralazine. The antibodies may occur in isolation – theantiphospholipid syndrome. Cross-reaction with otherantiphospholipid antibodies produces a false-positiveWassermann (VDRL) test. Some antibodies have ananticoagulant effect on coagulation tests in vitro,characterized by a prolonged APTT (with or without aminor prolongation of the PT time) which fails to correctafter addition of normal plasma, suggesting the presenceof a coagulation inhibitor. These antibodies are termedthe ‘lupus anticoagulant’. Despite the in vitro effect, thepresence of the lupus anticoagulant has the oppositeeffect in vivo, and predisposes affected individuals torecurrent thrombotic events. Recurrent abortions arethought to be due to placental infarction. Otherassociations of the antiphospholipid syndrome have beenmentioned (see Answer 49, 53). The diagnosis is made

1. Antiphospholipid syndrome.2 . i. Antiphospholipid antibodies (these may result in

a false positive VDRL test).ii. Failure of APTT to improve after addition of

normal plasma.iii. Lupus anticoagulant.

3. Any of the following:i. Recurrent abortions.ii. Livedo reticularis.iii. Pulmonary hypertension.iv. Transient ischaemic attacks.v. Chorea.vi. Migraines.vii. Valvular heart disease.viii. Thrombocytopenia.

Causes of arterial and venous thromboses

• Paroxysmal nocturnal haemoglobinuria• Paradoxical emboli• Myeloproliferative disorders• Homocystinuria• Sickle cell anaemia

Page 76: Rapid review of clinical medicine for mrcp part 2

Clinical Cases 75

by detection of antiphospholipid antibodies in the serum.Specific tests for the detection of the lupus anticoagulantantibodies are available. A negative VDRL and absence of

circulating lupus anticoagulant does not exclude thepresence of antiphospholipid antibodies. Management iswith prophylactic anticoagulant therapy.

Question 71

7711aa 7711bb

7711ccA 35-year-old male was seen in clinic with a 12-year history ofbackache. He was a keen football player, but for over 11 years hadnot participated in sport owing to the disabling pain. He had beenseeing an osteopath for almost 10 years. His back pain was originallycontrolled, but had now become unbearable. He had pain in hislower and upper back that was worsened by movement of the spine.Over the past two years he had noticed intermittent pain and swellingin both his knees, which had responded to ibuprofen. There was nohistory of diarrhoea, skin rashes or mouth ulcers. He experiencedtransient dysuria after a trip to Amsterdam at the age of 21, but thiscleared after a course of penicillin from his doctor. There was nofamily history of back pain.

On examination, he was in pain. He had lost his lumbar lordosis.Examination of his eyes and ear are shown (71a, b). His knee jointswere swollen, with bilateral effusions; examination of the rest of hisjoints was normal. Cardiovascular, respiratory, abdominal andneurological examinations were all normal.

Investigations are shown.

Hb 15 g/dlWCC 10 � 109/lPlatelets 230 � 109/lESR 30 mm/hCalcium 2.3 mmol/lPhosphate 0.8 mmol/lAlkaline phosphatase 90 iu/lAlbumin 40 g/lX-ray of spine (71c)Urinalysis Glucose ++

Protein 0

1. What is the diagnosis?2. How would you confirm the diagnosis?3. How would you account for the result of urinalysis?

Page 77: Rapid review of clinical medicine for mrcp part 2

76

Alkaptonuria is inherited as an autosomal recessive trait.There is a deficiency in the enzyme homogentisic acidoxidase, leading to accumulation of homogentisic acid inthe urine and increased pigmentation (ochronosis) incartilage and connective tissue. Urine becomes dark onstanding owing to oxidation and polymerization ofhomogentisic acid. Freshly voided urine appears normalin colour, so these patients do not often report darkurine. Abnormal pigmentation is found in the ear andsclerae (71a, b) as well as articular cartilage (at surgery).Premature arthritis predominantly affecting the back, and

later the large joints, is the most serious manifestation ofthe disorder. Intervertebral disc calcification ischaracteristic of alkaptonuria and is seen on the spinal X-ray (71c). The knees are commonly affected. The sacro-iliac joints are spared. Calcification of the aorta is oftenseen on the chest X-ray.

A low-protein diet is recommended to reduce theamount of homogentisic acid, which is a breakdownproduct of tyrosine; however, this does not prove veryeffective.

Causes of a false-positive Clinistix test

• Fructose• Pentose• Lactose• Salicylates• Ascorbic acid• Homogentisic acid

1. Alkaptonuria.2. Urinary homogentisic acid level. Urine becomes

dark on standing.3. Homogentisic acid is a reducing substance and

therefore gives a positive reaction to glucostix (Clinistix). Other substances that produce a false-positive glucostix test are shown (Table). Glucose oxidase (Clinitest) sticks do not give this false-positive reaction.

Answer 71

Question 72

An 18-year-old male was referred to the respiratory out-patient clinic with a three-year history of progressivelyincreasing breathlessness. Until the age of 15 years he wasable to participate in school sports, but since then he hadnoticed increasing fatigue and breathlessness on exertion.He was diagnosed as having exercise-induced asthma andwas prescribed salbutamol and beclomethasone inhalers,with initial relief of his symptoms until he was 16 years ofage, when his symptoms started to deteriorate. At thisstage he was seen by a hospital specialist, who felt that hehad resistant asthma and started him on a two-week trial oforal steroids, which proved to be successful. Subsequently,the dose of inhaled steroid was increased. Three monthsbefore coming to clinic his symptoms had deterioratedconsiderably. He was breathless on walking less than 200metres on the flat at a slow pace. His appetite was reduced,and he had noticed that his ankles were swollen towardsthe end of the day. He was a non-smoker, and had neverconsumed alcohol. He was currently at a collegeattempting to do his GCSE examinations, but was having

considerable difficulty because of his illness. He was anonly child. Both parents were well and had a sharedbusiness. They were non-smokers. The family had a petbudgerigar.

On examination, the patient was thin. He was belowthe 10th centile for weight, and on the 50th centile forheight. He appeared very slightly jaundiced, but was notcyanosed or clubbed. His heart rate was 120 beats/minand regular. The blood pressure was 110/65 mmHg. TheJVP was not elevated. The respiratory rate was 28/min.Chest expansion was symmetrical, but moderately reduced.The percussion note was resonant and auscultationrevealed reduced breath sounds in the lungs, particularly atboth bases. On examination of the abdomen, the liveredge was palpable 3 cm below the costal margin. Inaddition, the tip of the spleen was just palpable. There wasno clinical evidence of ascites; however, there was pittingankle oedema to the shins.

Investigations are shown.

Page 78: Rapid review of clinical medicine for mrcp part 2

Clinical Cases 77

7722

Hb 17 g/dlWCC 9 � 109/lPlatelets 130 � 109/lMCV 100 flSodium 135 mmol/lPotassium 4.1 mmol/lUrea 3 mmol/lCreatinine 60 μmol/lCalcium 2.1 mmol/lPhosphate 1.0 mmol/lTotal protein 66 g/lAlbumin 31 g/lBilirubin 58 μmol/lAST 49 iu/lAlkaline phosphatase 600 u/l

Chest X-ray (72)Lung function:

PEFR 200/min (predicted 550/min)FEV1 2.0 l (predicted 4.2 l)FVC 4.1 l (predicted 5.6 l)TLC 7.5 l (predicted 6.4 l)KCO 0.7 mmol/l/kPa

(predicted 1.6 mmol/l/kPa )Arterial blood gases:

pH 7.34PaO2 8.4 kPaPaCO2 6.6 kPaBicarbonate 34 mmol/lBase excess +10

What single test would you request to confirm the diagnosis?

a. Sweat sodium concentration.b. Liver biopsy.c. Transbronchial lung biopsy.d. Serum immunoglobulins.e. Measurement of serum α-1 antitrypsin and its

electrophoretic mobility.

A 48-year-old baker presented with a right deep-veinthrombosis affecting the right calf. Apart from a swollenright calf, physical examination did not demonstrate anyother abnormality.

Investigations are shown.

Hb 18 g/dlPCV 0.57 l/lMCV 89 flWCC 9 � 109/lNeutrophils 92%Lymphocytes 8%Monocytes 4%Platelets 388 � 109/lBlood film Normal

1. What is the haematological abnormality?a. Pseudopolycythaemia.b. Secondary polycythaemia.c. Primary polycythaemia.d. Myelofibrosis.e. Essential thrombocythaemia.

2. List at least three investigations you would perform to determine the underlying cause.

Question 73

Page 79: Rapid review of clinical medicine for mrcp part 2

78

The patient has hyperinflated lung fields, an obstructivelung defect with reduced transfer factor and evidence ofcirrhosis of the liver. The combination of chronic lungdisease and cirrhosis of the liver in a young patient shouldraise suspicion of two important diagnoses: α-1antitrypsin deficiency; and cystic fibrosis. In this case, thechest X-ray is against the diagnosis of cystic fibrosis (72).Patients with cystic fibrosis often have cysticbronchiectasis, patchy consolidation and fibrotic changes.Furthermore, the absence of cough productive of sputumis also against the diagnosis of cystic fibrosis.

α-1 antitrypsin deficiency is a rare autosomal recessivedisorder due to a genetic abnormality on chromosome14. The absence or reduced levels of this proteaseinhibitor leads to alveolar and liver damage. There areseveral different abnormal alleles for the gene. ‘M’ is thenormal allele, and ‘Z’ and ‘S’ are the most frequentlyencountered abnormal alleles. The expression of eachphenotype is suffixed by the letters ‘Pi’, which stand for

protease inhibitor. PiZZ and Pi null null are associatedwith a high risk of premature emphysema, but only thePiZZ genotype is associated with cirrhosis (Table).

The majority of patients with α-1 antitrypsin levels ofless than 20% present with emphysema in the fifthdecade. Cirrhosis usually occurs at a young age, andpatients with the PiZZ genotype may present withneonatal jaundice. The diagnosis is confirmed bymeasuring the levels of α-1 antitrypsin.

The respiratory manifestations are treated with somesuccess by using synthetic α-1 antitrypsin, but the hepaticmanifestation of this condition requires l ivertransplantation.

Genotype α-1 antitrypsin levels

PiMM NormalPiSS >50%PiZZ <10%PiSZ 20%Pi null null Nil

e. Measurement of the serum α-1 antitrypsin and its electrophoretic mobility.

Answer 72

Answer 73

The patient has an elevated haemoglobin and a raisedPCV. The differential diagnosis is betweenpseudopolycythaemia, primary polycythaemia(polycythaemia rubra vera) or a secondary polycythaemia.In pseudopolycythaemia, the Hb and the PCV are raisedowing to haemoconcentration resulting from any causeof reduced extracellular volume (dehydration, diuretics).Pseudopolycythaemia can be differentiated from truepolycythaemia by red cell mass estimation, which willreveal a normal red cell volume to extracellular fluidvolume ratio in pseudopolycythaemia, and a raised ratioin true polycythaemia. A popular cause ofpseudopolycythaemia in the MRCP and similarexaminations is Gaissbock’s syndrome, which occurs inmiddle-aged men who are usually obese and have asmoking history. In this case, the red cell volume isnormal, but the plasma volume is reduced. Truepolycythaemia is divided into primary polycythaemia andsecondary polycythaemia. The former is a result of a

myeloproliferative disorder causing erythroid hyperplasiaindependent of raised circulating erythropoietin, and isbetter known as polycythaemia rubra vera. In addition toa raised Hb and red cell count, the white cells are alsoincreased in 70% and the platelets in 50% of cases, andphysical examination may demonstrate a palpable spleen.In contrast, secondary polycythaemia is due to raisedlevels of circulating erythropoietin, either owing to aphysiological reason, for example, living at high altitude,chronic lung disease, or cyanotic heart disease or owingto inappropriate secretion of erythropoietin from varioussources (Table). In this condition, the white cells andplatelets are unaffected, and the spleen is not palpable.

When faced with a patient with raised Hb and PCV whoalso has raised white cells, platelets and a palpable spleen, thediagnosis is polycythaemia rubra vera. The investigationrequired to confirm the diagnosis is a bone marrow aspirate,which will show erythroid hyperplasia and increasedmegakaryocytes. The leucocyte alkaline phosphataseconcentration is also increased. When the white cell andplatelet counts are normal, and there is no evidence ofsplenomegaly, then secondary causes must be sought. Theinvestigation of secondary causes is shown (73).Important note: in polycythaemia rubra vera there is atendency to bleed. Many patients have occult bleedingfrom the gastrointestinal tract, and have normal Hb anda high red cell count, but microcytic red cell indices. Inthese patients, the clue that they may have polycythaemiarubra vera comes from the elevated white cell and plateletcounts (see Answer 211).

1. b. Secondary polycythaemia.2. i. Red cell mass estimation.

ii. Arterial blood gases.iii. Abdominal ultrasound scan with particular

reference to the kidneys and the liver.iv. α-fetoprotein.v. Erythropoietin level.

Page 80: Rapid review of clinical medicine for mrcp part 2

Clinical Cases 79

Causes of polycythaemia

Pseudopolycythaemia• Dehydration• Gaissbock’s syndrome

Primary• Polycythaemia rubra vera

Secondary1. Physiological increase in erythropoietin (due to chronic hypoxaemia):

• Chronic lung disease• Living at high altitudes• Cyanotic heart disease• Smoking (carboxyhaemoglobin)• Familial (HbM)

2. Inappropriate increase in erythropoietin:• Renal cell carcinoma, polycystic kidneys, transplant kidney• Adrenal tumours (phaeochromocytoma)• Hepatocellular carcinoma• Ovarian fibroma• Cerebellar haemangioblastoma (part of Von Hippel–Lindau)

73

PCV >0.55 l/l

Normal WCCNormal plateletsNo splenomegaly

Normal

Pseudopolycythaemia

Dehydration

Diuretics

Gaissbock’s syndrome

High

O2

saturation <90%

Chronic obstructive pulmonary diseaseCyanotic heart diseaseHigh altitudePickwickian

True erythrocytosis

Investigation for secondary polycythaemia

Important investigations

Red cell mass

O2 saturation >90%

Renal ultrasound

Hepatic ultrasound/α-fetoprotein

Arterial blood gases

HypernephromaAPCKD

Hepatocellular carcinoma

Page 81: Rapid review of clinical medicine for mrcp part 2

80

Question 74

A 62-year-old former insurance broker presented to thecardiology out-patient clinic with a five-week history ofincreasing breathlessness and ankle swelling that startedwhile on a three-week holiday in Spain. Over the past threedays he had become breathless at rest, and was having tosleep propped up on four pillows to help his symptoms.His GP had commenced him on a small dose of diuretic,which had not made any difference. There was no historyof chest pain and no coryzal illness over the past fewmonths. Eight months previously he was diagnosed ashaving diabetes mellitus, which was not well controlled onoral hypoglycaemic agents, and he had recently startedinsulin injections. The only other past medical history ofnote was intermittent pain and swelling in both knees thatwas helped by taking an NSAID. He was married and hadthree sons. He had never smoked, and consumed a veryoccasional glass of wine. There was a strong family historyof diabetes and ischaemic heart disease. His father had diedfrom ‘heart failure’.

On examination, he was slightly tanned. He wasbreathless on mild exertion. His heart rate was 110beats/min and regular. His blood pressure was100/60 mmHg. Examination of the neck is shown (74a).The apex was displaced in the 6th intercostal space andanterior axillary line. On auscultation of the precordiumthere was a loud third heart sound, and a soft systolicmurmur in the mitral area. Auscultation of the lung fieldsdemonstrated basal inspiratory crackles bilaterally. The

ankles were swollen. Abdominal examination revealed apalpable liver edge 4 cm below the costal margin, butthere was no other palpable organomegaly.

Initial investigations are shown below.The patient was treated with intravenous diuretics and

made a good recovery. He underwent invasiveinvestigation (74f, g), and was discharged on an ACEinhibitor, furosemide and digoxin. He was reviewed in thecardiology clinic three months later when he complainedof some right upper quadrant discomfort and that he hadexperienced three hypoglycaemic episodes followingdischarge. The examining cardiologist detected a palpableliver edge, but there was no obvious evidence of cardiacfailure. Plans were made for him to have a liver ultrasoundscan and to be reviewed again in three months. However,one week before his clinic appointment he was admittedwith increasing right upper quadrant pain, abdominaldistension and ankle oedema.

On examination he had a heart rate of 100 beats/minand a blood pressure of 100/65 mmHg. The JVP was notraised and the heart sounds were normal. The abdomenwas distended from ascites, and the ankles were swollen.Urinalysis was normal. The dose of his diuretics wasincreased and his ascites started to resolve. An abdominalultrasound scan was performed. The ultrasoundappearance, with the probe in the left lower costal margin,is shown (74h). A liver biopsy was also performed (74i).

Hb 14 g/dlWCC 10 � 109/lPlatelets 158 � 109/lMCV 90 flSodium 134 mmol/lPotassium 3.5 mmol/lUrea 11 mmol/lCreatinine 110 μmol/lAST 100 iu/lALT 110 iu/lAlkaline phosphatase 130 u/lAlbumin 38 g/lBlood glucose 6 mmol/lECG (74b)Chest X-ray (74c)Echocardiogram Parasternal long-axis view

(74d); four-chamber view (74e)

1. What was the original cause of this patient’s admission?

2. What is shown on the chest X-ray?3. What is the abnormality on the 12-lead ECG?4. What does the echocardiogram demonstrate?5. What invasive investigation was performed during

the initial hospital admission, and what did it demonstrate?

6. What is the most probable cause for the pain in his knees?

7. What was the cause of his second admission?8. What is demonstrated on the abdominal

ultrasound scan?9. What is the stain used on the liver biopsy, and

what is its significance?10. What is the unifying diagnosis?11. Suggest one therapeutic management step.12. Suggest one non-therapeutic management step.

Page 82: Rapid review of clinical medicine for mrcp part 2

Clinical Cases 81

7744aa 7744bb

7744cc 7744dd

7744ee 7744ff

7744hh 7744ii7744gg

Page 83: Rapid review of clinical medicine for mrcp part 2

82

In the absence of alcohol abuse, the combination of adilated cardiomyopathy with normal coronary arteries,diabetes mellitus, arthropathy, and cirrhosis of the liver ina pigmented individual can all be explained by idiopathichaemochromatosis, an autosomal recessive geneticdisorder characterized by increased iron absorption fromthe gastrointestinal tract with subsequent deposition inthe skin, liver, heart, pancreas, gonads and the pituitarygland. The genetic defect is a point mutation on thegene HLA FE, which is the short arm of chromosome 6.The mutation results in the substitution of the aminoacid cysteine by tyrosine at position 282. The actualfunction of the gene is unknown. There is also anassociation with HLA-A3, B7 and HLA-A3, B14. Themost common presentation in homozygotes is withlethargy and malaise, although joint pains, particularly inthe knee, symptoms of diabetes mellitus and skinpigmentation may also be the initial presentingcomplaints. Males are more susceptible than females whoare protected by menstruation. Alcohol abuse may cause the same clinical picture in heterozygotes. Cardiac involvement leads to dilated or restrictivecardiomyopathy and an abnormal ECG, buthaemochromatosis is a rare cause of cardiomyopathy.Deposition of excess iron in the liver leads to cirrhosis andan increased predisposition to hepatocellular carcinoma.Chondrocalcinosis is a recognized complication, most

commonly affecting the metacarpophalangeal and kneejoints. Hypogonadism results from direct infiltration intothe gonads and into the pituitary.

The diagnosis can be made genetically, but this iscurrently expensive and iron studies continue to remainthe ‘gold standard’ test. The serum iron is high, the totaliron-binding capacity is low, and the serum ferritin isusually in the thousands. The transferrin saturation and

Table A Causes of LBBB

• Ischaemic heart disease• Cardiomyopathy• Aortic stenosis• Hypertensive heart disease• Cardiac conduction tissue disease• Following insertion of a pacemaker

Table B Causes of chondrocalcinosis

• Calcium pyrophosphate arthropathy (pseudogout)

• Gout• Hyperparathyroidism• Haemochromatosis• Hypophosphatasia• Hypothyroidism• Wilson’s disease• Alkaptonuria• Lead poisoning

1. Congestive cardiac failure.2. Cardiomegaly and pulmonary oedema.3. LBBB (Table A).4. Dilated left ventricle. The upper limit of normal

size for the left ventricular end-diastolic dimension is 54 mm. Each white line on the scan represents 10 mm.

5. Coronary angiography, which demonstrates a normal left and right coronary artery. See diagrammatic representation of the coronary anatomy (74J).

6. Chondrocalcinosis (Table B).7. Cirrhosis of the liver complicated by portal

hypertension.8. Enlarged spleen which, in the presence of ascites,

would be consistent with the diagnosis of portal hypertension in the context of this history.

9. Perl’s stain, which is used to demonstrate the presence of iron. In the presence of iron, Perl’s stain produces a dark blue/black colour.

10. Idiopathic haemochromatosis.11. Weekly venesection until the serum ferritin is

normal.12. Screen his sons for the genetic disorder. This

could be done genetically, or with iron studies. Affected asymptomatic relatives can thus be treated before there is any tissue damage. LMS

RCAmid

RCAproximal

Key:CX Circumflex arteryLAD Left anterior descending arteryLMS Left main stemOM Obtuse marginalPD Posterior descending arteryRCA Right coronary artery

LAD proximal

LADmid

LADdistal

Inter-mediate

OM1

OM2

PD

CX

RCAdistal

7744jj

Answer 74

Page 84: Rapid review of clinical medicine for mrcp part 2

Clinical Cases 83

the iron-binding saturation index ([Fe]/[TIBC]) are verysensitive and are typically above 90% in patients withhomozygous haemochromatosis. A liver biopsydemonstrates iron overload in the presence or absence ofmicronodular cirrhosis. There is deposition of iron in theliver parenchyma and the bile duct canaliculi, but theKupffer cells are spared. A dry iron content of over180 mg is highly suggestive of the diagnosis.

In some cases, idiopathic haemochromatosis is difficult todifferentiate from alcoholic liver disease, which is alsoassociated with raised serum ferritin levels and increased irondeposition in the liver (Table C). The liver biopsy is veryhelpful in this respect because it demonstrates coexistingchanges of alcohol-related liver disease such as Mallory’shyaline and, in contrast to idiopathic haemochromatosis,iron deposition occurs in the Kupffer cells.

Table C Differential diagnosis of conditions causing simultaneous cardiac and liver disease

Liver HeartAlcohol abuse Cirrhosis Dilated cardiomyopathyHaemochromatosis Cirrhosis Dilated cardiomyopathyPericardial constriction Cirrhosis Cardiac failureChronic tricuspid regurgitation Hepatic congestion Cardiac failureCarcinoid tumour with hepatic metastases Metastases Pulmonary/tricuspid stenosisSarcoidosis Granulomas Restrictive cardiomyopathyHIV Hepatitis MyopericarditisNeoplasia Metastases Pericardial effusionStorage disorders Hepatic dysfunction Cardiomyopathy

A 55-year-old male was admitted with a three-monthhistory of abdominal pain that was dull in nature andoften radiated to his back. The pain was precipitated bymeals and started 2 hours after eating. The patient hadintermittent diarrhoea productive of foul, bulky stool,and had lost almost 3 kg in weight in three months. Hehad been diagnosed as having non-insulin-dependentdiabetes mellitus six years ago, and had an inferiormyocardial infarction two years later. He smoked 20cigarettes per day and had never consumed alcohol.There was a strong family history of ischaemic heart

1. What is the diagnosis?a. Cholelithiasis.b. Mesenteric ischaemia.c. Carcinoma of the head of pancreas.d. Chronic pancreatitis.e. Non-alcoholic steatohepatitis.

2. How would you confirm the diagnosis?a. Hepatobiliary ultrasonography.b. CT scan abdomen.c. ERCP.d. Mesenteric angiography.e. Liver biopsy.

Question 75

disease. A barium meal and ultrasound scan of thehepatobiliary system were normal.

On examination, he was thin. The external appearanceof his eyes is shown (75). There was no pallor, clubbingor lymphadenopathy. His heart rate was 98 beats/minand blood pressure 140/90 mmHg. The heart soundswere normal and his chest was clear. He had a soft leftcarotid bruit. On examination of his abdomen he wastender in the epigastrum and umbilical area. There wereno abdominal bruits. Neurological examination wasnormal.

7755

Page 85: Rapid review of clinical medicine for mrcp part 2

The patient has chronic abdominal pain after meals,steatorrhoea and weight loss. All three differentialdiagnoses could account for this picture. He has stigmataof hyperlipidaemia (the eyes reveal xanthelasma andcorneal arcus) and peripheral vascular disease (carotidbruit). He is diabetic, and a long-term smoker. All thesefactors favour atherosclerosis, making the diagnosis ofintestinal ischaemia more likely in the examinationsituation, although there is no reason why a patient withgeneralized arteriosclerosis should not develop carcinomaof the pancreas or chronic pancreatitis, which isidiopathic in almost 50% of cases. Severe atheromaaffecting the coeliac axis would account for thesymptoms. Increased intestinal motility after a mealexacerbates ischaemia, causing pain. The ischaemic smallbowel loses functional capacity and therefore a

malabsorption syndrome develops. The combination offear of pain from eating and malabsorption is responsiblefor weight loss. Zollinger–Ellison syndrome can presentwith abdominal pain following meals and steatorrhoea;however, in this case the barium meal is invariablyabnormal, revealing multiple gastric, duodenal or jejunalulcers. The possibility of cholelithiasis is excluded by thenormal hepatobiliary ultrasound scan.

Mesenteric ischaemia is essentially a diagnosis ofexclusion. In this situation the patient would have a CTscan of the abdomen and an ERCP to check forcarcinoma of the pancreas and chronic pancreatitis. Ifthese tests are negative, mesenteric angiography wouldhelp validate the clinical suspicion of mesentericischaemia. Some patients can have severe atherosclerosisof the mesenteric arteries and be completelyasymptomatic; therefore, this finding does not necessarilydiagnose mesenteric ischaemia as the cause of hissymptoms unless other potential causes have beenexcluded. Non-alcoholic steatohepatitis is not usuallyassociated with abdominal pain.

84

1. b. Mesenteric ischaemia. The differential diagnosis includes carcinoma of the pancreas or chronic pancreatitis.

2. d. Mesenteric angiography.

Answer 75

Question 76

A 34-year-old female presented with a six-week history ofepigastric pain that radiated to her back and that wasprecipitated by meals. There was no accompanyingnausea or vomiting. Her GP prescribed ranitidine andantacid solution, which relieved the discomfort to someextent. Just one month previously she had requested ablood sugar check because she had developed nocturia,but the test was normal. According to her husband, shehad started to become miserable and spoke very little.She attributed this to fear of having an abdominalmalignancy because her maternal aunt had just died fromcarcinoma of the stomach. The only other family historyof significance was that her father had undergone anoperation on his neck 20 years ago to treat kidney stones.She did not consume alcohol and was a non-smoker. Sheworked in a confectioner’s shop near the house. She hadtwo daughters, aged eight and six.

On examination, the patient had mild tenderness onpalpation of the epigastrum, but all other physicalexamination was normal.

Investigations are shown.The patient underwent surgery shortly afterwards,

which improved all her symptoms, and she remained wellfor six years; following this, the abdominal pain returned.The pain was associated with dyspepsia. She alsocomplained of intermittent episodes of foul-smellingdiarrhoea. She was prescribed ranitidine once again;however, on this occasion it proved to be ineffective. She

was advised to stay on ranitidine and referred to agastroenterologist, who performed upper gastrointestinalendoscopy and demonstrated many small ulcers in theantrum of the stomach and the first and second part ofthe duodenum. A serum gastrin level was also performedand it was 500 pg/ml (NR <100 pg/ml). Hecommenced the patient on 40 mg of omeprazole, after

Hb 12 g/dlWCC 7 � 109/lPlatelets 380 � 109/lMCV 88 flESR 11 mm/hSodium 135 mmol/lPotassium 4 mmol/lUrea 7.1 mmol/lCreatinine 89 μmol/lChloride 118 mmol/lBicarbonate 19 mmol/lCalcium 3.0 mmol/lPhosphate 0.7 mmol/lAlkaline phosphatase 345 iu/lBlood glucose 4.6 mmol/l Abdominal X-ray Faecal loading, but no

bowel obstructionUrinary calcium 20 mmol/l

Page 86: Rapid review of clinical medicine for mrcp part 2

Clinical Cases 85

which her symptoms improved. Her GP had tried toreduce the dose to 20 mg after six weeks, but she starteddeveloping abdominal pain and the dose was raised backup to 40 mg.

The patient remained well for a further two years, afterwhich she presented with increasing lethargy andheadaches. She had stopped menstruating, and hadreduced libido. Her appetite was reduced, but she hadnot lost any weight.

On examination, she was pale and had very scantyaxillary and pubic hair. Her skin was dry. Her pulse was62 beats/min and blood pressure 105/65 mmHg.Examination of her fundi showed slightly pale discs. Hervisual fields are shown (76a). Neurological examinationof her arms and legs and examination of the othersystems was normal.

Further investigations are shown.

76b

7766aa

1. What was the initial diagnosis?2. Which surgical procedure had her father

undergone?3. What was the cause of the pain six years later?4. Which investigation would you have performed to

confirm the diagnosis six years later?

5. What is the abnormality on the visual fields?6. What is demonstrated on the skull X-ray?7. What was the cause of her headaches?8. Which single diagnosis accounts for all her

presentations?9. Should anyone else be investigated?

Hb 10 g/dlWCC 6 � 109/lPlatelets 300 � 109/lMCV 81 flESR 25 mm/hSodium 129 mmol/lPotassium 4 mmol/lUrea 4 mmol/lCreatinine 89 μmol/lCalcium 2.1 mmol/lPhosphate 1.2 mmol/lAlkaline phosphatase 92 iu/lThyroxine 4 nmol/lTSH 0.1 mu/lFSH 0.3 iu/lLH 0.3 iu/lPRL 4,500 mu/lSkull X-ray (76b)

Page 87: Rapid review of clinical medicine for mrcp part 2

86

MEN 1 is characterized by tumours of the parathyroidgland (95%), islet cells (50%) and the pituitary gland(33%). Abnormality within a gene termed the menin geneon chromosome 11 is responsible, but the function of thegene is yet to be elucidated. Hypercalcaemia is the firstpresentation, usually from the third decade onwards. Islet

cell tumours (usually gastrinomas or insulinomas) occur inthe fourth or fifth decade, and pituitary tumours (usuallyprolactinomas) occur in the fifth or sixth decade.

Screening is with yearly calcium/PTH and PRL, and atwo-yearly pituitary MRI. Patients are made aware ofsymptoms of islet cell tumours so that they present early.

Causes of hyperprolactinaemia

• Prolactinoma• Hypothalamic or pituitary stalk lesions• Hypothyroidism• Coitus• Pregnancy• Nipple suckling• Stress• Oral contraceptive pill• Dopamine antagonist drugs, e.g. phenothiazines• PCOS• Chronic renal failure• Chest wall damage/trauma

1. Hyperparathyroidism.2. Parathyroidectomy.3. Peptic ulceration resulting from a gastrinoma.4. Assessment of serum gastrin after intravenous

secretin or measurement of gastric acid output afterthe administration of pentagastrin. In normalindividuals, secretin causes suppression of serumgastrin levels; however, in patients with gastrinomas,gastrin secretion is not suppressed and may evendemonstrate a paradoxical rise. Pentagastrin reducesgastric acid production in normal individuals, butnot in patients with gastrinomas.

5. The patient has a bitemporal hemianopia, suggestinga lesion compressing the optic chiasm. This could bea meningioma, an aneurysm, a pituitary tumour or acerebral metastasis.

6. There is enlargement of the pituitary fossa, a doublefloor to the sella turcica and erosion of the posteriorclinoid process. The findings are consistent with apituitary tumour.

7. Raised intracranial pressure resulting from a largeprolactinoma. A prolactinoma is the most commonpituitary tumour associated with multiple endocrineneoplasia (MEN) 1. Prolactinomas are associatedwith amenorrhoea, galactorrhoea, hirsutism andinfertility in females. In males prolactinomas causeimpotence, oligospermia, infertility andgalactorrhoea. Note: hyperprolactinaemia does notcause gynaecomastia. There are several causes ofraised PRL (Table), but a PRL of over 3,000 mu/l ishighly suggestive of prolactinoma. Treatment is withbromocriptine for small tumours and surgery forlarger ones.

8. Multiple endocrine neoplasia type 1.9. The condition is autosomal dominant, so there is a

50% chance of her offspring inheriting the condition.Therefore, both daughters should be screened forthe condition.

Answer 76

Page 88: Rapid review of clinical medicine for mrcp part 2

Clinical Cases 87

A 48-year-old male with rheumatoid arthritis presentedwith a cough, high fever and painless jaundice. He had aseven-year history of rheumatoid arthritis affecting hisshoulders, elbows, wrists, hands and feet, and which wasrefractory to therapy with NSAIDs. Over the past 16months his symptoms had progressed, and he had beenswitched to immunosuppressant drugs, which hadcontrolled his symptoms. He was a non-smoker andconsumed less than 2 units of alcohol per week.

On examination he was unwell, jaundiced and had atemperature of 39°C (102.2°F). Examination of the oralcavity revealed candidal infection and palatal petechiae.On auscultation of the lung fields there were coarsecrackles at the right lung base. On abdominalexamination there was evidence of a palpable spleen 3 cmbelow the costal margin.

Investigations are shown.

Hb 7.1 g/dlWCC 1.4 � 109/lPlatelets 33 � 109/lMCV 103 flReticulocyte count <1%INR 1.9ESR 70 mm/hCRP 190 g/lSodium 136 mmol/lPotassium 4 mmol/lUrea 6 mmol/lCreatinine 110 μmol/lBilirubin 180 μmol/lAST 180 iu/lAlkaline phosphatase 450 iu/lAlbumin 29 g/l

1. Give three possible causes for this patient’s presentation.

2. Which six investigations would you perform on admission?

3. Give three explanations for the raised MCV.4. List two immediate management steps.

Question 77

The following cardiac catheter data is from a 10-year-oldmale after a murmur was heard during a school medical.

Chamber Pressure (mmHg) Oxygen saturation(%)Right atrium 5 74Right ventricle 20/6 74Pulmonary artery 32/12 92PCWP 3Left ventricle 100/8 96Aorta 90/65 96

1. What are the two main abnormalities?2. What is the diagnosis?

Question 78

Page 89: Rapid review of clinical medicine for mrcp part 2

88

This patient with rheumatoid arthritis presents with arespiratory tract infection and high fever. The blood testsreveal a profound neutropenia, which has almost certainlypredisposed him to the respiratory and oral Candidainfection. In the context of rheumatoid arthritis, theneutropenia is either drug-induced or secondary tohypersplenism (part of Felty’s syndrome). Felty’s

syndrome alone cannot explain the abnormal bilirubinand alkaline phosphatase which, in the absence ofabdominal pain, suggest either a cholestatic hepatitis orcirrhosis of the liver. Both methotrexate andazathioprine, which are sometimes used asimmunosuppressants in rheumatoid arthritis, may alsocause hepatitis and cirrhosis. The mode of action of thesedrugs is to interfere with purine nucleotide synthesisrequired for nucleic acid generation. All cells in the bodywith a rapid turnover time are affected, particularly thecells in the marrow – hence the pancytopenia. The raisedMCV is probably due to a megaloblastic anaemia, butmay be due to chronic liver disease.

Methotrexate-induced liver cirrhosis only complicates1% of all patients taking the drug. Liver disease is morecommon in patients consuming over 10 units of alcoholper week, patients with diabetes and patients taking thedrug frequently. In the early stages the symptoms arenon-specific and comprise anorexia and nausea; however,late disease presents with portal hypertension, varices orhepatocellular failure. Azathioprine-induced hepatitis israre but may occur early in patients taking allopurinol, asthe latter drug potentiates the effects of azathioprine.

The immediate management of this patient is a septicscreen (blood cultures, urine culture, throat and nasalswabs) followed by broad-spectrum antibiotics. Be carefulwhen performing blood gases in a patient with lowplatelets and abnormal clotting. Oxygen saturation usinga pulse oximeter is the safest method of determiningarterial oxygenation in this case. An ultrasound scan ofthe biliary system should be requested to exclude biliaryobstruction.

Answer 78

1. i. Septicaemia secondary to respiratory infection precipitated by methotrexate-induced neutropenia.

ii. Septicaemia secondary to respiratory infection precipitated by azathioprine-induced neutropenia.

iii. Septicaemia secondary to respiratory tractinfection due to Felty’s syndrome.

2. i. Chest X-ray.ii. Oxygen saturation.iii. Blood cultures.iv. Urine culture.v. Throat swab for culture.vi. Hepatobiliary ultrasound scan.

3. i. Megaloblastic anaemia due to methotrexate or azathioprine.

ii. Megaloblastic anaemia due to folate deficiency (methotrexate inhibits dihydrofolate reductase).

iii. Liver cirrhosis.4. i. Stop immunosuppressant drug.

ii. Broad-spectrum intravenous antibiotics following a septic screen. Usually the combination of a penicillin and aminoglycoside or a third-line cephalosporin is used.

1. i. Elevated pulmonary artery pressure.ii. Step up in oxygen saturation in the pulmonary

artery.2. The findings are consistent with a patent ductus

arteriosus with a left-to-right shunt.

Answer 77

Page 90: Rapid review of clinical medicine for mrcp part 2

Clinical Cases 89

A 66-year-old female was investigated for a six-monthhistory of increasing breathlessness and palpitation. Shehad a long history of hypertension. A chest X-ray (80a),ECG (80b), M-mode and 2-D echocardiograms (80c, d,respectively) are shown.

A 25-year-old male presented to the Accident andEmergency Department with a three-day history ofpleuritic, left-sided chest pain and increasing breathlessness.The symptoms started suddenly while he was playingfootball. He denied any chest trauma. Until then he hadbeen fit and well, and there was no past medical history ofnote. His father died suddenly at the age of 32 years.

On examination he was thin and tall and distressedwith pain. There was no pallor or cyanosis. He had awide arm span. The heart rate was 100 beats/min andblood pressure 150/80 mmHg. The respiratory rate was32/min. The trachea was deviated to the right.Percussion appeared to be reduced on the right side andwas loud on the left side. The left lung base was dull topercussion. On auscultation of the lung fields breath

sounds were absent on the left side and normal on theright side. Auscultation of the precordium demonstratedquiet heart sounds and a soft mid-systolic murmur. Thefemoral pulses were easily palpable. The left calf wasbruised and slightly tender. He attributed this to aninjury sustained during the football match. The ECGrevealed normal sinus rhythm. The ECG complexes weresmall, but there were no other abnormalities.

8800aa 8800bb

8800cc

8800dd

1. Which test would you perform to reach a diagnosis?2. What is the cause of his respiratory symptoms?3. What is the management?4. What is the unifying diagnosis?

1. What does the chest X-ray reveal?2. What is the abnormality on the ECG?3. List two abnormalities on the 2-D echocardiogram.4. List four abnormalities on the M-mode

echocardiogram.5. Which three pharmacological agents would you

prescribe for this patient?

Question 80

Question 79

Page 91: Rapid review of clinical medicine for mrcp part 2

90

A large pneumothorax in a symptomatic patient receivesrelief with chest drain insertion rather than simple needleaspiration. Pleuritic chest pain and increasingbreathlessness occur in pleurisy with or without pleuraleffusion, pneumonia, pulmonary embolism and lungmalignancy. The physical signs are suggestive of a largeleft pneumothorax. The trachea is deviated to the right,and there are no breath sounds on auscultation of the leftlung. The right lung sounds comparatively dull topercussion when compared with the left lung, which ishyper-resonant. There also appears to be a small effusionat the base of the left lung, which is not uncommon withpneumothoraces and represents either blood or aninflammatory exudate.

Causes of pneumothorax include asthma, emphysema,stab wound to the lung or following a fractured rib,fibrotic lung disease, suppurative lung disease (rupture ofa lung abscess into a bronchus, staphylococcalpneumonia), pneumoconiosis, neurofibromatosis,

tuberose sclerosis, Marfan’s syndrome, Ehlers–Danlossyndrome and lymphangioleiomyomatosis. Most cases areidiopathic and occur in tall, thin men. A rare form,usually left-sided, occurs in women during menstruation(catamenial pneumothorax). It is possibly due topulmonary endometriosis.

The underlying diagnosis and the cause of thepnemothorax in this case is Marfan’s syndrome. This issuggested by his tall height, cardiac mumur consistentwith mitral valve prolapse and a possible family history ofthe condition, i.e. his father died prematurely. Thesyndrome is caused by a mutation in the gene encodingthe connective tissue protein fibrillin on chromosome 15.The condition is inherited as an autosomal dominanttrait. Affected individuals are tall, have a wide arm span,arachnodactyly and skeletal deformities which includekyphoscoliosis, high arched palate, and pectus excavatum.The joints are hyperextensile. Other associations includelens dislocation and retinal detachment. Mitral valveprolapse and aortic dilatation are recognised cardiacmanifestations. Aortic dissection or rupture are thecommonest cause of death. Patients with Marfan’ssyndrome have a higher incidence of pneumothorax thanthe general population.

Answer 80

1. Chest radiography.2. A large, left-sided pneumothorax.3. Insertion of a chest drain.4. Marfan’s syndrome.

1. An enlarged cardiothoracic ratio and evidence of an enlarged left atrium. Features of an enlarged left atrium include a prominent left heart border, a double shadow behind the heart and splaying of the carinae, with elevation of the left hilum.

2. Atrial fibrillation (for causes, see Table, Answer 138). There is a complete absence of P-waves and a beat-to-beat variation of the RR-interval. This patient also has voltage criteria for left ventricular hypertrophy, presumably secondary to the hypertension or coexisting mitral regurgitation.

3. i. Enlarged left atrium.ii. A thickened dome-shaped appearance of the

mitral valve in diastole, which is consistent with mitral stenosis.

4. i. Thickening of the anterior mitral valve leaflet, which is characterized by loss of its normal ‘M’-shape in diastole.

ii. Multiple echoes on the anterior mitral valve leaflet that suggest fibrosis.

iii. Reduced mitral valve excursion.iv. Anterior movement of the posterior leaflet in

diastole (in normal individuals the posterior leaflet moves backwards in diastole). (See Echocardiography, page 421.)

5. i. Digoxin to control ventricular rate.ii. Furosemide to help improve breathlessness from

pulmonary congestion.iii. Warfarin as prophylaxis to thromboembolism.

Patients with mitral stenosis and atrial fibrillation have a 35% chance of having an embolic CVA per annum.

Answer 79

Page 92: Rapid review of clinical medicine for mrcp part 2

Clinical Cases 91

A 47-year-old businessman was referred to thedermatology clinic with a rash over his hands, scalp andface. He had been generally well. He smoked 20 cigarettesper day and drank 8–10 units of alcohol on a daily basis.There was no other significant medical history of note. Thepatient had not taken any medication, with the exception

of occasional chlorpheniramine to help relieve hissymptoms. On examination he had vesicular lesionsaffecting his face, forearms, hands (81) and scalp. All otherphysical examination was essentially normal.

Investigations are shown.

A 13-year-old female was admitted under the surgeonswith acute abdominal pain. The blood pressure was100/60 mmHg.

Investigations are shown.

Hb 18.5 g/dlWCC 7 � 109/lPlatelets 149 � 109/lMCV 101 flSodium 135 mmol/lPotassium 3.9 mmol/lUrea 4 mmol/lCreatinine 80 μmol/lBlood glucose 10 mmol/lBilirubin 25 μmol/lAST 52 iu/lAlkaline phosphatase 450 iu/lAlbumin 37 g/lGlobulins 30 g/lGamma GT 52 iu/lSerum ferritin 550 μg/lSkin biopsy Subepidermal blisters with perivascular

deposition of PAS-staining material

Sodium 131 mmol/lPotassium 7.2 mmol/lUrea 13 mmol/lCreatinine 121 μmol/lBicarbonate 8 mmol/lChloride 96 mmol/lAbdominal X-ray NormalUrinalysis Glucose +++

1. What is the most probable diagnosis?2. State two tests to confirm the diagnosis.3. What two factors should this patient avoid?4. Apart from avoidance, give two other therapeutic measures.

1. What is the acid-base disturbance?2. What is the most likely cause for the abnormality?3. List three therapeutic management steps.

Question 82

Question 81

8811

Page 93: Rapid review of clinical medicine for mrcp part 2

92

The patient has a very low serum bicarbonate, which isindicative of a metabolic acidosis. The heavy glycosuria isthe clue to the cause of the acidosis. Diabetic ketoacidosisis the result of insulin deficiency. It may occur inpreviously undiagnosed insulin-dependent diabetesmellitus, established insulin-dependent diabetics who are

on suboptimal insulin doses or who have interruptedinsulin therapy, and insulin-dependent diabetics who haveintercurrent illness resulting in increases in hormoneswhich antagonize the effects of insulin. In the absence ofinsulin, hepatic gluconeogenesis is increased, butperipheral glucose uptake is decreased; the result is anosmotic diuresis and dehydration. Peripheral oxidation offree fatty acids is increased, which results in an increase inketones, leading to a metabolic acidosis. Both the acidosisand hypoperfusion of the kidneys due to hypovolaemiacontribute to the hyperkalaemia (reduced potassiumexcretion). Treatment involves rehydration and insulinreplacement. If the serum potassium is above 6.8 mmol/l,calcium gluconate is advised to prevent hyperkalaemia-induced cardiac arrest. Intravenous bicarbonate shouldonly be given if the arterial pH is <7. (See Acid–baseDisturbance, page 426.)

The blistering skin rash in a patient with a history ofexcess alcohol consumption and biochemical evidence ofliver disease is suggestive of the diagnosis of PCT.

PCT is a rare disorder of haem synthesis that isinherited as an autosomal dominant trait. It is one of themost commonly tested porphyrias in higher medicalexaminations, and is caused by a reduced activity orcomplete absence of the enzyme uroporphyrindecarboxylase, which catalyses the conversion ofuroporphyrin to coproporphyrin. This leads to an excessof uroporphyrins in the blood and urine. The result is apruritic photosensitive blistering rash that affects areasexposed to sunlight, and cirrhosis of the liver. Blisters onthe skin lesion contain PAS-positive material. The clinical

effect of this deficiency becomes most apparent ingenetically predisposed patients who consume excessalcohol or other drugs that are enzyme-inducers for haemsynthesis (most commonly anticonvulsants andoestrogens). Hepatic involvement is common and ispredominantly due to the effects of excess alcohol. Theliver biopsy demonstrates macronodular cirrhosis andMallory’s hyaline. A proportion of patients with liverdisease also have evidence of iron overload in the bloodand liver, resulting in a raised serum ferritin and a positivePerl’s stain on liver biopsy, respectively; in addition,approximately 20% develop diabetes mellitus. Thecondition may be confused with haemochromatosis (ironoverload, cirrhosis and diabetes); however,haemochromatosis is not associated with a blistering rash.A secondary polycythaemia may also occur and appears tocompound the condition.

Treatment is by abstaining from alcohol and avoidingsunlight. In patients with a high Hb, weekly venesection(not more than 1 unit) until the Hb is less than 12 g/dlhelps to alleviate the skin manifestations. Chloroquine,which binds uroporphyrin and helps urinary clearance, iscommonly used in the management of PCT.

1. PCT.2. i. Urinary uroporphyrin level.

ii. Uroporphyrin decarboxylase assay on red blood cells.

3. i. Sunlight.ii. Alcohol.

4. i. Venesection.ii. Chloroquine.

1. Metabolic acidosis with a high anion gap.

2. Diabetic ketoacidosis.

3. i. Intravenous calcium gluconate to prevent hyperkalaemic cardiac arrest.

ii. Intravenous insulin infusion to halt further ketoacidosis.

iii. Intravenous saline to rehydrate patient and correct acidosis.

Answer 82

Answer 81

Page 94: Rapid review of clinical medicine for mrcp part 2

Clinical Cases 93

A 14-year-old male was admitted with a head ache anddrowsiness. Three days previously he had complained of asore throat.

Investigations are shown.

CT scan of brain NormalCSF:

Opening pressure 160 mmH2OCells 200 neutrophils/mm3

20 red cells/mm3

Protein 1.2 g/lGlucose 2.4 mmol/lGram stain (83a)

A 64-year-old West Indian male was admitted after hisdaughter noted that he had become increasingly drowsyover the past 48 hours. He had a 10-year history ofdiabetes mellitus for which he was takingchlorpropamide. Before the deterioration, he hadcomplained of increasing urinary frequency and excessivethirst. He recently saw his GP, and was noted to have ablood pressure of 170/100 mmHg for which he hadbeen commenced on a thiazide diuretic.

On examination he was drowsy and apyrexial, butanswered questions appropriately. There was reducedskin turgor. The heart rate was 110 beats/min andregular, and the blood pressure was 115/60 mmHg. Theheart sounds were normal, and the chest was clear.Examination of the abdomen did not reveal anyabnormality, and there was no focal neurological deficit.

Investigations are shown.

Hb 15 g/dlWCC 13 � 109/lPlatelets 179 � 109/lSodium 151 mmol/lPotassium 5 mmol/Urea 17 mmol/lCreatinine 140 μmol/lUrinalysis Glucose +++

Protein +

Question 83

1. What is the diagnosis?a. Streptococcal meningitis.b. Meningococcal meningitis.c. Haemophilus influenzae meningitis.d. Tuberculous meningitis.e. Cryptococcal meningitis.

2. What is the management?

1. What is the diagnosis?2. List six investigations which should be performed

on this patient.3. List four steps in your management.

Question 84

8833aa

Page 95: Rapid review of clinical medicine for mrcp part 2

94

8833bb

8833cc

Hyperosmolar non-ketotic diabetic coma usuallycomplicates middle-aged or elderly patients with non-insulin-dependent diabetes mellitus and is characterizedby a marked increase in blood glucose (usually higher than50 mmol/l) and a consequent increase in plasmaosmolality. Ketonuria is absent and acidosis is mild orabsent. The actual pathogenesis of the condition isunclear, but possible triggers include consumption oflarge amounts of sugary drinks, infection, recentprescription of thiazide diuretics or steroids andmyocardial infarction. Patients present with thirst,polyuria, impaired level of consciousness and sometimesevidence of the precipitating disorder.

Investigations reveal a high blood glucose and plasmaosmolality. The plasma sodium is usually elevated owingto pure water loss from osmotic diuresis; however, it maybe falsely low (pseudohyponatraemia) owing to thepresence of excess lipids. The serum pH is usually normal,but the patient may have a mild acidosis. Otherinvestigations that should be performed are an infectionscreen and an ECG (beware of silent myocardial infarctionin a diabetic patient!).

Management includes rehydration with normal saline. Inpatients with persisting severe hypernatraemia despitetreatment with normal saline (sodium >150 mmol/l), half-strength saline is used. Insulin is infused carefully as HONKis more sensitive to insulin than diabetic ketoacidosis. The patient is given 4 units immediately followed by an infusion of 1–3 units/h. The aim is to reduce the plasma osmolality by approximately 3 mOsm/h. Arapid reduction in plasma osmolality is dangerous because itmay precipitate cerebral oedema. Bicarbonate infusion isnot required. Prophylactic anticoagulation with heparin isrecommended because the high glucose concentration anddehydration result in hyperviscosity. After treatment of theacute state the patient should be recommenced on an oralhypoglycaemic agent.

1. a. Streptococcal meningitis.2. Intravenous cefotaxime on admission. The

antibiotic can be switched to benzylpenicillin if theorganism is sensitive to penicillin.

1. Hyperosmolar non-ketotic diabetic coma (HONK).

2. i. Plasma osmolality.ii. Blood glucose.iii. Blood cultures.iv. Urine culture.v. Chest X-ray.vi. ECG.

3. i. Rehydration with 0.9% saline.ii. Insulin infusion of 1–3 units/h.iii. Anticoagulation.iv. Treat underlying cause.

Answer 83

The CSF neutrophil count is consistent with a bacterialmeningitis (see Answer 267). The three commonestbacterial meningitides worldwide include those due toStreptococcus pneumoniae, Neisseria meningitidis, andHaemophilus influenzae. The Gram stain reveals Gram-positive (stain blue–purple) cocci, therefore the mostlikely diagnosis is S. pneumoniae.

N. menigitidis is a Gram-negative (stains pink)diploccocus (83b). H. influenzae is a Gram-negativebacillus (pink rods in 83c).

All three present with fever, headache, neck stiffnessand photophobia. A necrotic purpuric rash is charac -teristic of menigitis due to N. meningitidis, but mayoccur with all three types.

Answer 84

Page 96: Rapid review of clinical medicine for mrcp part 2

Clinical Cases 95

The chart above (85a) is the pedigree of a family with a rare condition. Allmembers are affected by the same condition. Patient A is dead, but he had ahistory of progressive renal failure, and had a renal transplant which wasaffected by a separate renal disease causing haematuria and rapid deteriorationof renal function. An audiogram and a picture of his fundus have beenremoved from his case notes and are shown (85b, c). His sister (patient C),aged 70 years, was deaf and had intermittent haematuria, but her creatininewas 89 μmol/l. His brother (patient B), aged 65 years, had a renal transplant14 years previously and was also deaf. Patients D, F, G, H, I and J all had ahistory of intermittent haematuria, but serum creatinine was within thenormal range in each case. Patients D and H had premature cataracts, andpatient J was blind, but did not have cataracts. Patient E was aged 38 and wason the transplant list for renal failure. He had been fitted with a hearing aidaged 26 years, and had had intermittent haematuria since he was aged 7 years.

8855aa

8855cc

A 45-year-old male attended to become a blood donor.Blood results were as follows (0 = no agglutination, + = agglutination):

Patient’s cells against anti-A = 0Patient’s cells against anti-B = 0Patient’s cells against anti-A and anti-B = 0Patient’s serum against red cells with antigen A = 0Patient’s serum against red cells with antigen B = 0Patient’s cells against anti-D = +Immunoglobulins: IgM 0.2 g/l; IgG 1.5 g/l; IgA 0.1 g/l

1. What is the mode of transmission of the disorder?2. What is shown on the audiogram?3. Why may patient J have been blind?

4. What is the underlying condition in the affected members of this family?

5. Which renal disease did patient A develop in his transplanted kidney?

1. What is the patient’s blood group?2. Give at least two possible underlying diagnoses.

A C B

D E F G H

I J

Affected male Unaffected male*

Affected female Unaffected female*

* Does not exclude inheritance of an autosomalrecessive gene in either sex or an X-linked recessivedisease in a female

8855bb

Question 85

125 250 500 1000 2000 4000 8000 Frequency HZ

125 250 500 1000 2000 4000 8000 Frequency HZ

-100

102030405060708090

100110120130140

Hea

ring

leve

l (dB

)

-100

102030405060708090

100110120130140

Hea

ring

leve

l (dB

)

RightLeft

RightLeftUnmasked

AC

BC

Question 86

Page 97: Rapid review of clinical medicine for mrcp part 2

96

There is no male-to-male transmission which is charac -teristic of X-linked conditions. Affected males transmitthe disease to all female offspring in conditions inheritedby an X-linked dominant trait, whereas affected femalespass the disease to 50% of all their offspring.

Alports’s syndrome is an X-linked hereditary nephritischaracterized by progressive glomerular disease, ocularand auditory abnormalities. The renal manifestationsbegin early in childhood, comprising asymptomatichaematuria and proteinuria. Progressive renal deteriora -tion has a bimodal age distribution, presenting between

16–35 years and 45–60 years. Only males develop pro -gressive renal failure. Affected females develop asympto -matic haematuria, but progression to renal failure doesnot occur. Sensorineuronal loss occurs in 30–50% ofpatients, and many patients have ocular abnormalitieswhich include myopia, retinitis pigmentosa (85c) andcataracts. Renal biopsy reveals thinning of the GBM, butas the disease progresses the membranes become thin andstart to split. The genetic defect is in the α-5 chain of typeIV collagen which is located on the X chromosome, butcollagen defects in the α-3 and α-4 chains of type IVcollagen on chromosome 2 have also been described insome families. All three types of α chains combine to formthe collagen network in the GBM. Some patients withAlport’s syndrome develop anti-GBM antibodies afterrenal transplantation. The risk is greatest in patients with alarge deletion in the gene encoding the α-3 chain of typeIV collagen, in which case the Good pasture antigen is notexpressed and therefore the Good pasture antigen in thetransplanted organ is recognized as foreign. Some 5% ofpatients with transplants for Alport’s syndrome willdevelop anti-GBM disease.

The patient’s red cells fail to agglutinate with seracontaining anti-A and anti-B, suggesting that the patientis blood group O, i.e. the cells do not express the anti -gens A or B on the cell surface. His cells agglutinate withserum containing anti-D, indicating that his cells expressthe rhesus D antigen. Patients with blood group O haveanti-A and anti-B antibodies which are of the IgG, IgMand the IgA class. The serum from an O-positiveindividual should normally agglutinate blood cells

expressing antigens A and B, unless the antibodies arepresent in very low titre. Failure of the appropriateagglutination may arise in conditions predisposing tohypogammaglobulinaemia and with paraproteinaemias.In generalized hypogammaglobulinaemia the low levelsof IgG, IgM and IgA are responsible for the poorresponse. With paraproteinaemias, one of the immuno -globulin subset levels may be elevated (usually IgG) dueto autonomous production. These antibodies are mono -clonal and are not directed towards any specific antigen.They are generally ineffective. Autonomous productionof a monoclonal antibody also leads to paraparesis of theother immunoglobulin subtypes, causing furtherimmunosuppression.

This man has evidence of hypogammaglobulinaemia.He is 40 years of age and is unlikely to have an inheritedhypogammaglobulinaemia because one would associatethis with a previous history of infections. An acquiredhypogammaglobulinaemia is more likely, and examplesinclude chronic lymphatic leukaemia, lymphoma, mye -loma and thymoma.

1. The mode of inheritance is X-linked dominant.2. There is progressive and severe hearing loss at

higher frequencies which is typical of sensorineuronal deafness. (See Audiograms, page 416.)

3. It is possible that she had retinitis pigmentosa just like her great uncle (patient A).

4. The condition is Alport’s syndrome. 5. Anti-GBM disease.

1. Blood group O rhesus-positive (absent anti-A and anti-B).

2. Hypogammaglobulinaemia due to:i. Thymoma.ii. Chronic lymphatic leukaemia.iii. Myeloma.iv. Lymphoma.v. Drugs.vi. Common variable hypogammaglobulinaemia.vii. HIV infection.

Answer 85

Answer 86

Page 98: Rapid review of clinical medicine for mrcp part 2

Clinical Cases 97

A 35-year-old woman presented with an epileptic seizurefive days after having a baby by Caesarean section. Therewas no previous history of epilepsy. She had been wellthroughout her pregnancy. She had had two normalpregnancies without any complication.

On examination her heart rate was 90 beats/min andregular and her temperature was 37.9°C (100.2°F). Theblood pressure was 180/102 mmHg. She had a Glasgowcoma score of 13/15. There was no evidence of a focalneurological deficit. Investigations are shown.

Question 87

Hb 10 g/dlWCC 11 � 109/lPlatelets 45 � 109/lBlood film Normochromic normocytic anaemia

Fragmented red cells; microspherocytes

PT 14 s (control 14 s)APTT 45 s (control 44 s)Factor V level NormalFactor VII level Normal

Sodium 137 mmol/lPotassium 4.6 mmol/lUrea 10 mmol/lCreatinine 130 �mol/lBilirubin 11 �mol/lAST 34 iu/lLDH 1530 iu/l (NR 252–525 iu/l) Alkaline phosphatase 80 iu/lAlbumin 30 g/lUrinalysis Protein ++CT scan brain Normal

What is the immediate management? a. Intravenous high-dose steroids.b. Platelet transfusion.c. Vitamin K.d. Plasma exchange with fresh-frozen plasma.e. Warfarin.

A 68-year-old Sri Lankan woman presented with a three-week history of lower back pain and progressive weaknessof the lower limbs. She had a temperature of 37.9°C(100.2°F). Investigationsare shown.

Question 88

Hb 10 g/dlWCC 13 � 109/lPlatelets 460 � 109/lESR 120 mm/hCRP 200 g/lBlood cultures SterileLumbar spine X-ray (88a)MRI scan lumbar (88b)spine

What would be the nextmanagement step?

a. Start IV flucloxacillin and IVgentamicin.

b. Start conventional anti-tuberculous treatment.

c. Arrange bone scan.d. Organize CT-guided

aspiration/biopsy of the lowerlumbar spine.

e. Perform a Heaf test.

8888aa 8888bb

Page 99: Rapid review of clinical medicine for mrcp part 2

The patient is post-partum and presents with an epilepticseizure (Table A). She has a fever, thrombocytopenia,microangiopathic haemolytic anaemia and impaired renalfunction, raising the possibility of either haemolyticuraemic syndrome or thrombotic thrombo cytopenicpurpura. The two conditions are probably part of thesame disorder. Classically TTP is characterized byneurological manifestation and more subtle deteriorationin renal function compared with HUS, whereneurological dysfunction is relatively infrequent. In TTPneurological manifestations include headache, seizuresand coma. Objective localizing signs are rare. Diagnosisof TTP is based on the pentad below (Table B).

The differential diagnosis for TTP or HUS includesDIC, anti-phospholipid syndrome, vasculitides andmalignant hypertension. TTP or HUS can usually bedifferentiated from these conditions (Table C)

In some patients TTP is secondary to deficiency of theenzyme von Willebrand factor cleaving protease(ADAMTS13). Von Willebrand factor is synthesized inendothelial cells and assembled in larger multimers thatare present in normal plasma. The larger multimers,called unusually large von Willebrand factor (ULvWf),are rapidly degraded in the circulation into the normalsize range of vWf multimers by a specific von Willebrandfactor-cleaving protease (ADAMTS13).

ULvWf multimers (including unique forms arisingfrom proteolytic digestion) accumulate in patients withTTP, being found in the platelet thrombi and serum.These ULvWf multimers can attach to activated platelets,thereby promoting platelet aggregation. ULvWfmultimer accumulation in TTP is associated with absentor markedly diminished ADAMTS13 activity due to aninherited or acquired deficiency.

Treatment is with plasma exchange using largevolumes of fresh-frozen plasma. The mechanism bywhich this method is therapeutic is not well understood.One possibility is that plasma exchange allows repletionof the missing enzyme ADAMTS13.

98

Answer 87

d. Plasma exchange with fresh frozen plasma.

Table A Differential diagnosis in a pregnantwoman presenting with epilepsy

• Cerebral neoplasm• Cerebral haemorrhage• Eclampsia• Haemolysis, elevated liver enzymes and low

platelets• Thrombotic thrombocytopenic purpura• Cerebral vein thrombosis secondary to

hyperviscocity

Table B Pentad in TTP*

1. MAHA2. Thrombocytopenia3. Fever4. Renal involvement5. Neurological involvement

* Not all patients have the classic pentad. Usually1 and 2 are required to make the diagnosis.

Table C Differentiation of TTP and HUS from other conditions

Medical condition Differentiation from TTP or HUSDIC Abnormal clotting, low fibrinogen, high FDPs, low factors V and VIIIVasculitides Usually have history of rash, arthralgia with normal platelets and peripheral

nervous system involvement Antiphospholipid syndrome Can cause neurological problems but APPT is highMalignant hypertension Diastolic blood pressure usually >130 mmHg and evidence of haemorrhages,

exudates and papilloedema on retinal examination

Page 100: Rapid review of clinical medicine for mrcp part 2

Clinical Cases 99

A 23-year-old woman with insulin-dependent diabetesmellitus had experienced recurrent episodes of cystitis forwhich she was taking trimethoprim. She smoked 15–20cigarettes per day. Her blood pressure measured120/80 mmHg. Urinalysis did not reveal any evidence ofmicralbuminuria.

Question 89

What is the best method of preserving renal function?a. Cessation of smoking.b. Strict glycaemic control.c. ACE inhibitor therapy.d. Lifelong prophylactic antibiotics.e. Angiotensin II receptor blocker.

A 35-year-old woman presented with vomiting andepigastric discomfort after a weekend of binge drinking.On examination she was alert. She had several spidernaevi on the face and chest wall. The heart rate was100 beats/min and regular. The blood pressure was160/96 mmHg. The respiratory rate was 20/min. Bothheart sounds were normal. The abdomen was soft.Inspection of the fundi was normal.

Initial investigations are shown.

Question 90

Hb 12 g/dlWCC 12 � 109/lPlatelets 100 � 109/lMCV 102 flSodium 132 mmol/lPotassium 3.3 mmol/lUrea 7 mmol/lCreatinine 80 �mol/lBicarbonate 11 mmol/lChloride 98 mmol/lBilirubin 18 �mol/lAST 160 iu/lGamma GT 100 iu/lPlasma osmolality 290 mOsm/lBlood glucose 5 mmol/lUrinalysis Ketones ++++

No crystals

What is the diagnosis?a. Diabetic ketoacidosis.b. Methanol poisoning.c. Euglycaemic ketoacidosis.d. Renal tubular acidosis.e. Lactic acidosis.

The presentation of back pain associated with weaknessof the lower limbs is consistent with cord compression.The accompanying fever, night sweats and raisedinflammatory markers are highly suggestive of infectionof the lower spine or a involvement of bone withhaematological malignancy. The plain lumbar spine filmshows minor destruction of the third lumbar vertebra andreduced joint space between the third and fourth lumbarvertebrae to indicate a discitis. The MRI scan showsdestruction of the third and fourth lumbar vertebrae andmarked discitis (reduced joint space). There is acollection of pus around the anterior and posterioraspects of the third and fourth lumbar vertebrae. Theanterior collection is pressing on the spinal cord. Themost probable diagnosis is tuberculous osteomyelitis ofthe lumbar spine in a female of her ethnicity.

Spinal TB usually occurs in immunocompromisedadults and is due to reactivation of TB acquired from a

primary infection. The lag period between primary TBand spinal TB may be decades. Spinal TB may occur as animmediate complication of the primary infection andmost commonly affects the lower thoracic and lumbarspine. Patients present with back pain, night sweats,weight loss and signs of cord compression from epiduralabscess formation. The differential diagnosis isosteomyelitis secondary to Staphylococcus aureusinfection, in which blood cultures are positive in over90% of cases.

A chest X-ray is rarely helpful in the diagnosis since itis only abnormal in 50% of cases. Whereas a Heaf test ispositive in over 90% of immunocompetent patients withspinal TB, many patients with spinal TB areimmunocompromised and may have a negative test. Theinvestigation of choice following MRI of the spine is aCT-guided aspirate/biopsy of the affected area beforestarting treatment. An attempt to make a tissue diagnosisshould be made whenever possible before initiating anti-TB therapy. Treatment is with conventional anti-TBtreatment for 12–18 months.

Answer 88

d. Organize CT-guided aspiration/biopsy of the lower lumbar spine.

Page 101: Rapid review of clinical medicine for mrcp part 2

100

The patient does not have hypertension or micro -albuminuria, therefore the best treatment for preventingnephropathy is meticulous glycaemic control. Glycaemiccontrol and blood pressure are the main predictors for thedevelopment of microalbuminuria (the earliest marker ofdiabetic nephropathy) in both type 1 and type 2 diabetes.Data from the DCCT trial in type 1 diabetes and UKPDStrial in type 2 diabetes have shown that meticulousglycaemic control alone delays the onset of diabeticnephropathy in patients who do not have coexistenthypertension. In patients with diabetes and hypertensionprevention of diabetic nephropathy requires bothmeticulous glycaemic control and maintaining a normalblood pressure. Better glycaemic control reduces glomerularbasement membrane thickening and microalbuminuria.

Once microalbuminuria has developed there is littleevidence that improving glycaemic control alone delaysprogression of nephropathy. In these circumstances,

control of blood pressure and the intraglomerularpressure, particularly using ACE inhibitors or angiotensinII receptor blockers, can greatly retard the progression ofnephropathy (see Algorithm).

In young type 1 diabetics with microalbuminuria orproteinuria, an ACE inhibitor should be prescribedregardless of the blood pressure. The target bloodpressure (which may require multiple other anti-hypertensive agents in addition to ACE inhibitors) inpatients with microalbuminuria is <120/70 mmHg andin patients with proteinuria it is <130/75 mmHg.

In patients with type 2 diabetes there is morecompelling evidence from large trials that angiotensin IIreceptor blockers prevent development of proteinuria inpatients with microalbuminuria. Target blood pressure is<130/75 mmHg.

Smoking has also been identified as a risk factor for thedevelopment of microalbuminuria. However, there are noprospective studies showing the benefit of cessation ofsmoking on renal function in type 1 diabetics at least.

Answer 89

b. Strict glycaemic control.

Prevention and management of established diabetic nephropathy

Microalbuminuria* No microalbuminuria

ACE inhibitor (type 1 diabetes)Angiotensin receptor blocker (type 2 diabetes)

� other antihypersensitive agentsTarget BP <120/70 mmHg

*In patients with frank proteinuriatarget BP is <130/75 mmHg

Yes No

Meticulous glycaemiccontrol and aggressive

BP control Target BP

130/75 mmHg

Meticulous glycaemiccontrol Hblc ≤7%

The patient is admitted after an episode of alcoholbingeing. Apart from the history, her investigations raisethe suspicion of alcohol abuse (macrocytosis, lowplatelets and abnormal liver function tests). Patients whoabuse alcohol may present with ketoacidosis in theabsence of insulin deficiency. There are several factorsthat contribute to ketoacidosis. Firstly, alcohol inhibitsgluconeogenesis, which offsets glucagon-mediatedlipolysis and ketone generation. Secondly, alcohol itselfmay be metabolized to ketones. Thirdly, patients whoabuse alcohol generally have a low calorie intake and areeffectively starving on a chronic basis. Patients usually

have normal or low blood glucose, but a high glucose isalso recognized. There is no clear explanation for highglucose levels except that some patients may haveconcomitant diabetes mellitus.

The plasma osmolality may also be increased owing toalcohol itself, and the generation of endogenous acids butalso raises the differential diagnosis of methanol orethylene glycol poisoning. Therefore it is important totake a detailed alcohol history and to check the urine forethylene glycol crystals. Salicylate toxicity may also presentin a similar fashion and should be excluded by measuringplasma salicylate levels. The treatment is IV dextrose andsaline solutions as well as thiamine replacement. Thedextrose replaces the glucose required to inhibit glucagonproduction and the saline replenishes fluid loss.

Answer 90

c. Euglycaemic ketoacidosis.

Diabetes

Page 102: Rapid review of clinical medicine for mrcp part 2

Clinical Cases 101

A 27-year-old female artist was seen in the chest clinicwith a three-day history of worsening dyspnoea requiringthe use of an inhaled short-acting �2-agonist every 3–4hours. Over the past few months she had been using thebronchodilator with increasing frequency. She had alsoexperienced a non-productive cough particularly at nightand after exercise. She was diagnosed with asthma sincethe age of 14 years and was hospitalized on twooccasions more than 10 years ago for infectiveexacerbations of asthma. Her only other medication wasa low-dose inhaled steroid, which she takes twice daily.

On examination she was afebrile. She was 1.52 m tall.The peak expiratory flow rate was 365 l/min, with hernormal best at 470 l/min. She could complete sentencesin one breath. Her respiratory rate was 20/min, pulserate 80 beats/min and blood pressure 110/70 mmHg.Auscultation of the lungs revealed mild generalizedpolyphonic wheeze.

Question 91

Blood gases (air):

PaCO2 5 kPaPaO2 12 kPapH 7.34

Which two of the following would be possibleoptions in stepping up this patient’s asthmamanagement?

a. Continue current treatment and add regular oral prednisolone.

b. Continue short-acting β2-agonist as requiredand substitute high-dose inhaled steroid twicedaily.

c. Start regular short-acting inhaled β2-agonist and continue on low-dose inhaled steroid twicedaily.

d. Start regular short-acting β2-agonist and substitute high-dose inhaled steroid twice daily.

e. Continue short-acting β2-agonist as requiredand continue on low-dose inhaled steroid, add long-acting inhaled β2-agonist twice daily.

f. Continue short-acting β2-agonist as required,add sodium cromoglycate and stop low-doseinhaled steroid.

g. Continue short-acting β2-agonist as required,add modified release theophylline and stop low-dose inhaled steroid.

h. Continue current treatment and add inhaled ipratropium bromide twice daily.

i. Continue current treatment and add regular oral prednisolone and erythromycin 250 mg QDS.

j. Continue current treatment and add high-dose inhaled steroid twice daily.

A 52-year-old male with non-insulin-dependent diabetesmellitus has a blood pressure of 148/94 mmHg.Fundoscopy reveals evidence of background diabeticretinopathy.

Investigations are shown.

What is the best treatment for reducing the rate ofnephropathy?

a. Losartan.b. Ramipril.c. Insulin.d. Metformin.e. Atenolol.Hb 13.1 g/dl

WCC 5 � 109/lPlatelets 290 � 109/lSodium 138 mmol/lPotassium 4.1 mmol/lUrea 9 mmol/lCreatinine 138 μmol/lGlucose 8 mmol/lCholesterol 5.8 mmol/lTriglycerides 3.2 mmol/l12-lead ECG Normal24-hour urine protein 1g

Question 92

Page 103: Rapid review of clinical medicine for mrcp part 2

102

The British Thoracic Society (BTS) published guidelinesfor the management of asthma in 1990, which have sincebeen updated (1997), in response to concerns over theincreasing prevalence, morbidity, admissions and deathsfrom asthma. Supported by randomized controlled trials,steroids have since been established as the mainstay ofpreventative treatment in all but the mildest cases ofasthma. This question tests the candidate’s knowledge ofthe BTS guidelines of the stepwise approach to themanagement of chronic asthma with the introduction ofinhaled steroids early in the treatment (Table A).

The patient described had none of the features ofacute severe or life-threatening asthma and had a PEFR >75% of predicted. She was on step 2 of the managementladder of chronic asthma. She had symptoms indicatingloss of control of asthma and the need to step upmanagement (Table B).

Patients should start treatment at the step mostappropriate to the initial severity. A rescue course ofprednisolone may be needed at any time and at any step.Until growth is complete any child requiring inhaledhigh-dose steroids or oral steroids should be referred to apaediatrician with an interest in asthma. The aim is toachieve control and then to reduce treatment.

A peak-flow meter should be prescribed and theresponse to treatment should be reviewed every 3–6months. In patients with chronic asthma a 3–6 monthperiod of stability should be shown before slow stepwisereduction in the treatment is undertaken.

Answer 91

b. Continue short-acting �2-agonist as required and substitute high-dose inhaled steroid twice daily.

e. Continue short-acting �2-agonist as required and continue on low-dose inhaled steroid, add long-acting inhaled �2-agonist twice daily.

Table A Stepwise management of asthma

Step 1 Occasional use of inhaled short-acting �2-agonist as required for symptom relief.If needed more than once daily move tostep 2

Step 2 Occasional use of inhaled short-acting �2-agonist as required plus low-doseinhaled steroid twice dailyAlternatively use cromoglycate but startinhaled steroids if control not achieved.

Step 3 Occasional use of inhaled short-acting �2-agonist as required plus high-doseinhaled steroid daily or low-dose inhaledsteroid twice daily plus long-acting �2-agonist twice daily (especially ifexperiencing side-effects with high-doseinhaled steroids)Alternatively use MR theophylline orcromoglycate but start inhaled steroids ifcontrol not achieved

Step 4 Occasional use of inhaled short-acting �2-agonist as required plus high-doseinhaled steroid daily plus sequentialtherapeutic trial of one or more of thefollowing:• Long-acting �2-agonist twice daily• MR theophylline• Inhaled ipratropium bromide or

oxitropium• Long-acting �2-agonist tablets• Cromoglycate

Step 5 Occasional use of inhaled short-acting �2-agonist as required plus high-doseinhaled steroid daily plus one or more ofthe long-acting bronchodilators plusregular prednisolone tablets in a single dailydose

Table B Symptoms indicating inadequatecontrol of asthma

• The need for frequent relieving bronchodilatortherapy

• Chronic cough and nocturnal symptoms• Limitations on activity including exercise• PEFR <80%• Frequent exacerbations• Circadian variation in PEFR >20%• Adverse effects from medications

Page 104: Rapid review of clinical medicine for mrcp part 2

Clinical Cases 103

A 45-year-old man with HIV syndrome presented withweakness affecting the left upper and lower limbs. Onexamination he was confused and had a temperature of38°C (100.4°F). He had recently been treated for aPneumocystis carinii infection. The CD4 count was 150.CT scan of the brain with contrast revealed subcorticalatrophy and multiple contrast-enhancing ring lesions inthe cortex and subcortical areas.

Question 93

What is the most probable diagnosis?a. Primary CNS lymphoma.b. Tuberculous meningitis.c. CMV encephalitis.d. Cerebral toxoplasmosis.e. Progressive multifocal leucoencephalopathy.

A 60-year-old male was noted to have an irregular pulse.A subsequent 12-lead ECG showed atrial fibrillation witha ventricular rate of 80 beats/min. The patient wasasymptomatic. There was no past medical history of note.The patient consumed 10 units of alcohol per week. His

blood pressure was 110/80 mmHg and both heartsounds were normal.

Investigations are shown.

Question 94

Hb 12 g/dlBlood glucose 4.2 mmol/lTotal cholesterol 5.4 mmol/lTSH 1.3 mu/lTotal thyroxine 170 nmol/lChest X-ray Normal12-lead ECG Atrial fibrillation

Normal QRS complexesEchocardiography Left atrial diameter 40 mm

Normal left ventricle Normal systolic function Normal valves

Which additional treatment is required to reduce therisk of stroke?

a. Warfarin.b. Clopidogrel.c. Ramipril.d. Aspirin.e. Pravastatin.

The patient has moderate proteinuria and abnormal renalfunction. The question specifically relates to thetreatment of diabetic nephropathy in a patient with type2 diabetes mellitus. While ACE inhibitors have beenclearly shown to attenuate micro-albuminuria in patientswithout overt nephropathy and retard progression ofnephropathy in those patients with established diabeticnephropathy in type 1 diabetes, the management ofnephropathy in type 2 diabetes is still evolving.

It is clear that tight glycaemic control is effective inretarding nephropathy in both type 1 and type 2 diabetesmellitus and that control of hypertension is also necessaryin both conditions. In contrast with type 1 diabetesmellitus, there is much less information on the effects of

ACE inhibitors in treating diabetic nephropathy in type 2diabetes mellitus. However, two important studies haveshown that in patients with type 2 diabetes, angiotensinreceptor blockers are as effective in retarding nephropathyas ACE inhibitors are in type 1 diabetes mellitus.

For example, in the RENAAL trial, 1513 patients withtype 2 diabetes and nephropathy were randomly assignedto losartan (50 titrating up to 100 mg once daily) orplacebo, both in addition to conventional antihyper -tensive therapy (but not ACE inhibitors). Compared toplacebo, losartan reduced the incidence of a doubling ofthe plasma creatinine by 25% and end-stage renal diseaseby 28%; the mean follow-up was 3.4 years. These benefitswere not associated with differences in blood pressurelevels between the groups. Subsequent analysis foundthat the most significant risk factor for progressive kidneydisease was the initial degree of proteinuria.

Answer 92

a. Losartan.

Page 105: Rapid review of clinical medicine for mrcp part 2

104

The most probable diagnosis is cerebral toxoplasmosis.The CT scan is consistent with the diagnosis; the sub -cortical atrophy probably represents HIV encephalopathy.

The exact cause of CNS involvement in HIV infectionis largely influenced by the CD4 count. Patients withCD4 counts >500 have benign and malignant braintumours similar to those seen in immunocompetentpatients. Patients with a CD4 count of between 200 and500 often have cognitive disorders associated with HIVsuch as HIV dementia and progressive leucoence -phalopathy, which are not mass lesions. Patients with aCD4 count <200 have either opportunistic CNSinfections or HIV-related cerebral tumours.

Opportunistic infections include toxoplasmosis, TB,CMV and cryptococcal meningitis (see Question 277).Toxoplasmosis is by far the commonest cerebral masslesion seen in HIV patients and is usually associated withmultiple ring-enhancing lesions either in the cortico-

medullary junction or around the basal ganglia. Patientsoften have headache, confusion and fever. Focalneurological signs or seizure are common. The otheropportunistic infections rarely result in mass lesions in theabsence of disseminated infection.

Primary CNS lymphoma does display some degree ofcontrast enhancement but it is usually nodular or patchy;however, lesions may be multiple and difficult todifferentiate from toxoplasmosis. Thallium single photonemission computed tomography has been shown to beuseful in differentiating cerebral toxo plasmosis fromcerebral lymphoma. Primary CNS lymphoma is rarer thancerebral toxoplasmosis in HIV infection. Generallypatients with multiple ring-enhancing lesions on the CTscan with mass effect are treated empirically fortoxoplasma infection for two weeks with a combinationof pyrimethamine and sulphadiazine. Failure of aradiological response (on CT scan) is an indication forbrain biopsy to enable an alternative diagnosis such aslymphoma.

Answer 93

d. Cerebral toxoplasmosis.

The patient has atrial fibrillation which is associated witha five-fold increase in stroke secondary to systemicthromboembolism. The two main pharmacologicaltreatments available for preventing systemic thrombo -embolism in AF are antithrombotic agents and oralanticoagulant drugs. Aspirin is the only antithromboticagent recommended for preventing systemic thromobo -embolism in AF. Warfarin is the most commonly usedoral anticoagulant drug used in this particular situation.

Studies have shown that in high-risk patients (seebelow) warfarin is superior to aspirin in preventingsystemic thromboembolism. However, there is noconclusive evidence that warfarin is superior to aspirin inthis regard in low-risk patients. In patients at moderaterisk both aspirin and warfarin appear to be equallyeffective in reducing systemic thromboembolism.Patients' age above 75 years of age, rheumatic mitralvalve disease and prior embolic events are the greatest riskfactors for systemic thromboembolism in AF (Table).

The patient in question is at low risk of systemicthromboembolism, therefore he should be treated withaspirin instead of warfarin.

An alternative method of determining who will benefitfrom warfarin is to use the CHADS score (right): a score>2 is an indication of anticoagulation therapy; patientswith a score of 1–2 could be considered for aspirin orwarfarin; and patients with a score of 0 are deemed atrelatively low risk and suitable for aspirin (150–300 mg).

C = coronary artery disease 1 pointH = hypertension 1 pointA = age >75 1 pointD = diabetes mellitus 1 pointS = previous stroke 2 points

Answer 94

d. Aspirin.Treatment to prevent systemic

thromboembolism in atrial fibrillation

Factor Risk Treatment

Any ofAge ≥75 years High WarfarinPrevious stroke or TIAUncontrolled

hypertensionImpaired left ventricular

systolic functionRheumatic mitral

valve disease

Any ofAge 65–74 years Moderate WarfarinControlled or

hypertension aspirinDiabetes mellitusIschaemic heart diseaseThyrotoxicosis

All of Age <65 years Low AspirinNormal left

ventricular functionNo evidence of

rheumatic heart diseaseNo hypertension

Page 106: Rapid review of clinical medicine for mrcp part 2

An 84-year-old fully independent male was admitted withvery transient episodes of dizziness. The 12-lead ECGshowed first degree AV block and left bundle branchblock. A subsequent 24-hour ECG revealed episodicthird degree AV block.

Clinical Cases 105

Question 95

Which of the following treatments is most suitable forthe patient?

a. AAIR pacemaker.b. VVI pacemaker.c. DDDR pacemaker.d. VVIR pacemaker.e. None of the above.

A patient with long-standing rheumatoid arthritis isnoted to have splenomegaly. Results of haematologicalinvestigations are shown.

Question 96

Hb 10 g/dlWCC 3.1 � 109/l

(neutrophils, 1.5; lymphocytes, 0.7; eosinophils, 0.9)

Platelets 108 � 109/lMCV 80 flBone marrow: Erythroid hyperplasia,

myeloid hyperplasia but lack of mature formsMultiple megakaryocytes

What is the cause of the low white cell count?a. Folate deficiency.b. Drug-induced marrow aplasia.c. Myelodysplastic syndrome.d. Felty’s syndrome.e. Hodgkin’s lymphoma.

A 50-year-old woman presented with pain in her rightshoulder and weight loss of 4 kg over the past fourweeks. She had been taking non-steroidal anti-inflammatory drugs with only mild relief of her pain. Shesmoked 15–20 cigarettes per day.

Investigations are shown.

Question 97

Hb 11.2 g/dlWCC 8.2 � 109/lPlatelets 500 � 109/lESR 72 mm/hChest X-ray (97)

What is the diagnosis?a. Pancoast’s tumour.b. Multiple myeloma.c. Osteomyelitis affecting the right shoulder joint.d. Metastatic bone disease.e. Avascular necrosis of the head of femur.

9977

Page 107: Rapid review of clinical medicine for mrcp part 2

The patient has third degree AV block. The condition isassociated with an annual mortality exceeding 15% in the

absence of permanent cardiac pacing. The aim would beto maintain physiological rhythm and to pace both theatrium and the ventricle, i.e. he should have a dualchamber pacemaker (Table). He is fully independent andshould have a rate response facility (R).

106

Answer 95

c. DDDR Pacemaker.

Choice of pacemaker

Condition Pacemaker of choicePure sustained AF VVI or VVIRPure sinus node dysfunction without AF or AAIR (or DDDR)*

evidence of AV block at rapid heart rates (>150/min)

Second or third degree AVB or other bradyarrhythmias with visible P waves DDD or DDDR

*Most cardiologists in the UK implant DDD (DDDR) pacemakers in patients with sick sinus syndrome as manypatients go on to develop atrioventricular block. First letter, chamber(s) paced; second letter, chamber(s)sensed; third letter, mode of sensing (I = inhibition and D = triggering and inhibition); fourth letter (R), rateresponse facility.

In a patient with chronic rheumatoid arthritis,neutropaenia and splenomegaly, the most probablediagnosis is Felty’s syndrome. However, other conditionsassociated with a low white cell count in rheumatoidarthritis include drugs such as penicillamine, gold andmethotrexate, which are associated with bone marrowaplasia. Folate deficiency may also cause pancytopenia.The bone marrow does not reveal hypoplasia, thereforedrug-induced marrow aplasia is unlikely here. Theabsence of megaloblasts is against the diagnosis of folatedeficiency.

While Hodgkin’s lymphoma may explain thesplenomegaly and eosinophilia (NB: eosinophila is wellrecognized in rheumatoid arthritis, particularly inpatients with skin vasculitis, pulmonary fibrosis andsubcutaneous nodules), it is less likely than Felty’ssyndrome, particularly as the disease would have to befairly advanced to produce pancytopenia as in this case.Furthermore, at such an advanced state one wouldexpect the bone marrow to have a better yield for thediagnosis than it normally has in the very early stages ofHodgkin’s lymphoma. Myelodysplasia may cause ahypercellular marrow and a peripheral eosinophilia butsplenomegaly is very uncommon.

Answer 96

d. Felty’s syndrome.

There are small lucencies in the head of the humerus anda large lucency in the lateral border of the scapula. Thereis only one breast shadow present (on the right). The leftbreast shadow is absent, indicating that the patient has

had a mastectomy, presumably for treatment ofcarcinoma. In this case the lucencies have to be assumedto be secondary to disseminated breast carcinoma untilproven otherwise. In the exam always ensure both breastshadows are present on chest radiographs in femalepatients. Evidence of a mastectomy may be a major clueto the answer in a question – as in this case.

Answer 97

d. Metastatic bone disease.

Page 108: Rapid review of clinical medicine for mrcp part 2

Clinical Cases 107

A 22-year-old medical student developed suddendysphasia and right-sided weakness while on holiday inAustralia. She was afebrile. Apart from the neurologicalabnormality described, there were no other abnormalphysical signs.

Investigations are shown.

Question 98

Hb 15 g/dlWCC 10 � 109/lPlatelets 200 � 109/lESR 8 mm/hAutoantibody screen Normal12-lead ECG Normal2-D echocardiogram NormalCarotid Doppler study Normal

What is the most probable diagnosis?a. Carotid dissection.b. Encephalitis.c. Small intracerebral haemorrhage.d. Paradoxical cerebral embolus via patent foramen

ovale.e. Cardiac embolus from paroxysmal atrial

fibrillation.

A 50-year-old Jamaican male was admitted with discom -fort in the right shoulder. On examination he had weak -ness of abduction of the right upper limb.

Investigations are shown.

Question 99

What is the diagnosis?a. Sarcoidosis.b. Multiple myeloma.c. Paget’s disease.d. Bronchial carcinoma.e. Retrosternal goitre.

Hb 11 g/dlWCC 11.5 � 109/lPlatelets 200 � 109/lSodium 133 mmol/lPotassium 4.6 mmol/lUrea 7.4 mmol/lCalcium 2.76 mmol/lAlbumin 36 g/lChest X-ray (99a)

9999aa

Page 109: Rapid review of clinical medicine for mrcp part 2

108

A young patient presenting with stroke with a normalcardiac examination, 2-D echocardiogram, carotidDoppler study, autoantibody screen and inflammatorymarkers is highly consistent with the diagnosis ofparadoxical embolism via a patent foramen ovale.

A PFO occurs when the primum and secundum septafail to fuse completely leaving a small flap-likecommunication allowing the possibility of a shunt. Shehas been on a long flight from Australia and therefore itis possible that she had deep-vein thrombi, which havetraversed the PFO and passed into the systemiccirculation. PFOs are relatively common and may bepresent in up to 30% of the general population. PFOs

more than 4 mm in diameter and associated withintermittent right-to-left intracardiac shunting are morelikely to be associated with systemic emboli. PFOs maynot be identified on a 2-D echocardiogram. Atransoesophageal echocardiogram with contrast studies isthe diagnostic investigation of choice. Patients withsystemic embolism in association with a PFO with ademonstrable right-to-left shunt with following Valsalvamanoeuvre (provided there is no other obvious causesuch as atrial fibrillation or intramural thrombus) may betreated by closure using a mechanical devise (Amplatzerclosure device) via the right superficial femoral vein.

While cerebrovascular accidents are uncommon inyoung individuals, there are several potential causes dueto disorders affecting the heart and vasculature supplyingthe brain and primary central nervous system (Table).

Answer 98

d. Paradoxical embolus via patent foramen ovale.

Causes of stroke in young (≤35 years of age) individuals

Cause NotesCerebral haemorrhage Sudden onset severe headache and vomitingMeningoencephalitis Fever, confusion, meningism, seizuresNeoplasm Headache, focal neurology, seizuresCarotid or vertebral dissection Trauma/pain/unilateral lower cranial nerve palsiesMigraine Usually female; may be precipitated by typical auraInflammatory conditions (vasculitides, antiphospholipid Systemic features, raised inflammatory markers, syndrome, isolated cranial angitis, Takayasu’s disease) positive autoantibody screenStructural arterial disease Involves carotid artery; associated intracranial(Fibromuscular dysplasia) aneurysms; renal artery commonly affectedCerebral vein thrombosis Hyperviscocity, pregnancy, contraceptive pill, ear,

sinus or facial skin infection; headache, focal neurology, seizures

Thrombophilic states Previous arterial or venous thrombosisCardiac causes (severe hypertension, AF, Irregular pulse; murmurrheumatic heart disease, mechanical valves, cardiomyopathy, septal defects, PFOs)Haematological (polycythaemia, thrombocythaemia, paroxysmal nocturnal haemaglobinuria, sickle cell anaemia)Mitochondrial disease, e.g. MELAS (see Question 315)

There is opacification of the right upper lobe consistentwith a mass lesion. The first rib is eroded on the rightside indicating involvement of bone. The findings aretypical of a Pancoast tumour. The raised calcium may besecondary to bone metastases or secretion of PTH-related peptide by the carcinoma. Patients with Pancoasttumour present with symptoms and signs of pressure onthe brachial plexus, the sympathetic trunk (Horner’ssyndrome) and bone metastases (99b, arrows).

Answer 99

d. Bronchial carcinoma.9999bb

Page 110: Rapid review of clinical medicine for mrcp part 2

Clinical Cases 109

A 67-year-old male with dilated cardiomyopathy isadmitted for the third time in four months withincreasing breathlessness and swollen ankles. Heconsumed 2 units of alcohol per week. His medicationsconsisted of furosemide 80 mg daily, ramipril 10 mgdaily, candesartan 6 mg daily, spironolactone 25 mg

daily, bisoprolol 7.5 mg daily and isosorbide mononitrateSR 60 mg daily.

On examination his heart rate was 70 beats/min andregular. The blood pressure measured 80/40 mmHg. TheJVP was raised. Examination of the precordium revealed adisplaced apex in the sixth intercostal space and a forcefulright ventricular impulse. Auscultation of the heart revealeda systolic murmur in the mitral area and a third heartsound. On auscultation of the lungs there was evidence ofbilateral pleural effusions. Investigations are shown.

Hb 11 g/dlWCC 7 � 109/lPlatelets 200 � 109/lMCV 90 flSodium 129 mmol/lPotassium 5.9 mmol/lUrea 17 mmol/lECG Sinus rhythm; LBBBChest X-ray Enlarged heart; bilateral

pleural effusionsEchocardiography:

Dilated left and right ventricles. Ejection fraction 20%. Severe functional mitral regurgitation

What is the most effective management step toimprove his symptoms?

a. Add digoxin.b. Consider mitral valve repair.c. Increase dose of bisoprolol to 10 mg.d. Implant biventricular pacemaker.e. Increase dose of diuretics.

Question 100

A 21-year-old woman presented with a painless swellingin her neck. There was no history of a recent viral illness.The patient did not have symptoms of hyper- orhypothyroidism. She was not taking any medications. On

examination she had a 2 cm painless mass thyroid lumpassociated with cervical lymphadenopathy.

Investigations are shown.

Question 101

Hb 14 g/dlESR 5 mm/hTSH 2 iu/lChest X-ray Normal

What is the most useful investigation in elucidatingthe diagnosis?

a. Thyroid ultrasound.b. Radio-iodine thyroid scan.c. Fine-needle aspiration of the thyroid gland.d. Thyroid myoperoxidase antibodies.e. Total plasma thyroxine concentration.

A 70-year-old woman was investigated for lethargy. Onexamination she appeared slightly pale but all otheraspects of physical examination were normal.

Investigations are shown.

Question 102

Hb 6.1 g/dlWCC 3.0 � 109/lPlatelets 115 � 109/lMCV 116 flMCHC 37 pgFerritin 710 �mol/lBilirubin 35 �mol/lAST 40 iu/lLDH 1500 iu/lAlkaline phosphatase 100 iu/lAlbumin 37 g/l

What is the diagnosis?a. Hodgkin's lymphoma and haemolytic anaemia.b. Pernicious anaemia.c. Aplastic anaemia.d. Paroxysmal nocturnal haemoglobinuria.e. Autoimmune haemolytic anaemia.

Page 111: Rapid review of clinical medicine for mrcp part 2

The patient has severe symptoms despite adequate dosesof an ACE inhibitor, beta-blocker and spironolactone.His low systolic blood pressure does not allow additionalpharmacological therapy such as an angiotensin IIreceptor blocker. He has poor left ventricular functionand an interventricular conduction defect suggesting thathe may benefit symptomatically from biventricular pacing(Table A). The mitral regurgitation is a consequence ofthe left ventricular dilatation rather than the cause of it.An internal cardiac defibrillator prevents sudden death inpatients at risk of malignant ventricular arrhythmias butdoes not improve functional capacity.

Patients with cardiomyopathy and interventricularconduction defects are generally more symptomatic thanthose with normal conduction, owing to the resultingdesynchronization of ventricular contraction. Pacing bothventricles (RV is paced conventionally; LV is paced via thecoronary sinus) helps resynchronize ventricular contractionand improve functional capacity. Indications forbiventricular pacing are listed in Table A. Biventricularpacing is associated with significant improvement in qualityof life, functional capacity and reduction in hospitalizationowing to heart failure (Table B). There are preliminaryreports that suggest that overall mortality from heart failureis also improved with biventricular pacing.

110

Answer 100

d. Implant biventricular pacemaker.

Table A Indications for biventricular pacing

• Severe NYHA class III–IV failure despiteoptimal drug therapy

• QRS >130 msec• Left ventricular end diastolic diameter >55 mm• Left ventricular ejection fraction <35%

Table B Clinical effects of biventricular pacing

Improves ReducesQuality of life Total and heart failure

related hospitalizationsExercise capacity Heart rateNYHA functional classLV ejection fractionLV dimensions

Thyroid lumps may represent simple or multinodulargoitres, toxic nodules, neoplastic lesions, inflammatorymasses or thyroid cysts. Thyroid carcinomas account for5–6% of all lumps in the thyroid. Thyroid cancer is mostcommon in patients aged under 30 years or over 60years. Prior radiation to the head and neck is a recog -nized risk factor for thyroid carcinoma. Patients with afamily history of thyroid carcinoma are at a higher risk ofthyroid malignancy than the general population.

A painless swelling in the neck associated with cervicallymphadenopathy is highly suggestive of papillarycarcinoma of the thyroid gland. Papillary carcinoma ofthe thyroid invades local structures quickly and usuallypresents early as a result. The diagnosis of thyroidcarcinoma is usually made by histological sampling of afine-needle thyroid aspirate. Prior thyroid ultrasound isnot necessary. Indeed in patients with a normal TSH,fine-needle aspiration is the investigation of choice toinvestigate thyroid lumps. In patients with a low TSH aradio-iodine uptake scan is performed to confirm the

presence of a hot nodule. Since these are never malignanta fine-needle aspiration is not warranted.

Unfortunately, fine-needle aspiration cannot dis -tinguish between follicular and Hürthle cell adenomasand carcinomas. In these cases the biopsy is termedintermediate rather than obviously benign or malignant.In such cases a radio-iodine uptake scan is performed andif a cold nodule is identified then it is surgically excised.

In patients suspected to have Hashimoto’s thyroiditisclinically, or on the basis of high thyroid peroxidaseantibodies, a thyroid uptake scan should be performedbefore fine-needle aspiration as this may reveal func -tioning thyroid tissue in the entire lobe and obviate theneed for biopsy. This is particularly important as patientswith Hashimoto’s thyroiditis have Hürthle’s cells onhistology, which may be mistaken for a Hürthle cellcarcinoma.

While ultrasonography would provide more detailabout the morphology of the gland, it is not diagnostic.Thyroid scintigraphy is sometimes used by some centresto ascertain the functional status of the gland, but in thisparticular situation it would not provide a diagnosis.

Answer 101

c. Fine-needle aspiration of the thyroid gland.

Page 112: Rapid review of clinical medicine for mrcp part 2

Clinical Cases 111

A 64-year-old man presented immediately afterrecovering from a 20-minute episode of dysphasia andweakness of the right side of the face and arm. He hadexperienced two episodes of transient loss of vision in theleft eye in the past three weeks. There was a past medicalhistory of a myocardial infarction two years ago. Hismedication comprised 75 mg of aspirin daily. He did notsmoke.

On examination there was no evidence of residualneurological deficit. Fundoscopy was normal. The pulserate was 80 beats/min and regular in nature. His bloodpressure measured 138/86 mmHg in both arms. Bothheart sounds were normal. Auscultation over the carotidarteries revealed a bruit over the left carotid artery.

Investigations are shown.

Question 103

Hb 15 g/dlWCC 8 � 109/lPlatelets 300 � 109/lESR 20 mm/hSodium 138 mmol/lPotassium 4.1 mmol/lUrea 6 mmol/lCreatinine 100 mmol/lGlucose 4.7 mmol/lTotal cholesterol 6 mmol/lCT scan brain NormalCarotid Doppler Stenosis (40%) in left

internal carotid artery. Stenosis (80%) in right external carotid artery

Which three treatments have been shown to reducethe risk of recurrent stroke in this type of patient?

a. Add clopidogrel to current therapy.b. Add dipyridamole to current therapy.c. Substitute dipyridamole for current therapy. d. Increase dose of aspirin to at least 150 mg.e. Perform left carotid endarterectomy.f. Perform right carotid endarterectomy.g. Start ACE inhibitor after three days.h. Anticoagulate with warfarin.i. Start therapy with a statin drug.

This patient with lethargy has a markedly raised MCVand pancytopenia. The best answer is pernicious anaemia. Although paroxysmal nocturnal haemoglobinuria canalso be associated with pancytopenia and raised MCV,the actual MCV in PNH is less than in perniciousanaemia. The raised MCV in PNH is due to reticulocytes(newly formed red cell) arising from rapid red cellturnover secondary to haemolysis, whereas in perniciousanaemia the raised MCV is due to large immature redcells (erythroid precursors) secondary to immaturenuclear development. Remember hydroxycobalamin isrequired for thymidine synthesis, an important DNA

nucleoside. Erythroid precursors are larger thanreticulocytes. Indeed a MCV >115 fl should always raisethe possibility of abnormal DNA maturation in red cells,e.g. B12 deficiency, folate deficiency, drugs interferingwith DNA synthesis.

The raised ferritin is due to iron overload resultingfrom reduced red cell synthesis. The raised LDH andAST are due to haemolysis. Immature red cell precursorsundergo extravascular haemolysis. Aplastic anaemiawould explain the pancytopenia and raised ferritin butnot the raised MCV or haemolysis. Hodgkin’s lymphomawith haemolysis could explain all of the data given in thequestion but the MCV favours pernicious anaemia. (SeeAnswer 394.)

Answer 102

b. Pernicious anaemia.

Page 113: Rapid review of clinical medicine for mrcp part 2

112

The patient presents with a transient ischaemic attackwhich is defined as a neurological deficit lasting <24hours that is attributed to focal cerebral or retinalischaemia. The causes of TIAs are the same as those ofstroke and include cardiac intramural thrombi, carotidartery disease and disease of the intracranial arteries.Therefore strategies for the prevention of recurrent TIAsare similar to those for stroke.

Aspirin is the drug of choice in patients presentingwith TIA or stroke unless there are obviouscontraindications. Aspirin reduces the long-term risk ofstroke and cardiovascular events after a TIA or stroke,with an overall relative risk reduction of 22%. The doseof aspirin used ranges from 75 mg to 1300 mg, but smalldoses are as effective as large doses and are associatedwith a lower rate of gastrointestinal effects. The dosemost commonly prescribed in the UK is 75 mg daily.The thienopyridines clopidogrel and ticlopidine are notsuperior to aspirin in secondary prevention. There are nodata on these drugs in primary prevention.

Before commencing therapy, all patients should haveimaging of the brain (usually CT scan) to rule outunsuspected cerebral pathology as a cause for a TIA, suchas a brain tumour or a subdural haematoma, wheretherapy with aspirin is contraindicated (algorithm).

Imaging of the carotid arteries using carotid Dopplerstudies is performed after imaging of the brain to excludea significant internal carotid artery stenosis. Carotidendarterectomy is of proven benefit in patients with aninternal carotid artery stenosis of ≥70% who have had anon-disabling stroke or a TIA attributable to thestenosis. The optimal timing of surgery is currentlyunknown but greatest benefit appears to be derived ifsurgery is performed within the first two weeks of a non-disabling stroke, provided there are no contraindicationsto surgery.

Anticoagulation has not been evaluated specifically inpatients with TIA but has been extensively tested afterischaemic stroke. In patients with atrial fibrillation, long-term oral anticoagulation reduces the risk of recurrentstroke. In the absence of AF, oral anticoagulation withwarfarin is not superior to aspirin in reducing the risk ofrecurrent ischaemic stroke.

Answer 103

b. Add dipyridamole to current therapy.g. Start ACE inhibitor after three days.i. Start therapy with a statin drug.

Management of first presentation of

TIA or stroke

*In patients with AF anticoagulate immediatelywith warfarin in the case TIA. In patients withembolic stroke treat with aspirin for two weeksand then switch to warfarin.

Transient ischaemic attack or stroke

CT scan or MRI scan brain

Embolic cerebral event

Carotid Doppler study

Internal carotid artery stenosis ≥70% in appropriate carotid artery

Yes No

Consider carotidendarterectomy

Considermedical therapy

Aspirin 300 mg first dosefollowed by 75 mg*Statin if cholesterol >5 mmol/lTreat hypertension after 72 hoursAim for BP ≤130/85mmHgAdvice regarding smokingcessation, weight reduction andmoderation of alcohol intake

Page 114: Rapid review of clinical medicine for mrcp part 2

Clinical Cases 113

All patients presenting with TIA or stroke requiremeticulous control of blood pressure. Sudden loweringof the blood pressure may precipitate stroke in patientswith TIAs due to carotid artery stenoses or potentiateneurological disability in patients with stroke. It isgenerally recommended that blood pressure should bereduced cautiously 72 hours after the neurological event.The aim is to keep blood pressure below130/80 mmHg. All antihypertensive agents protectagainst stroke; however, the current literature suggeststhat ACE inhibitors may reduce the risk of furthercerebrovascular events even in patients who werenormotensive at presentation.

Lipid-lowering therapy with the statin class of drugshas been effective in primary and secondary prevention ofstroke. The aim is to treat cholesterol levels above

5 mmol/l in patients with TIA or stroke. In asympto -matic patients treatment of hyper-cholesterolaemia isdependent upon age, and other risk factors for stroke.

It is important not to underestimate the importance oflifestyle modification, and advice regarding smokingcessation, exercise, moderation of alcohol consumption,and weight control is mandatory.

Some patients continue to have TIAs or furtherstrokes despite the use of aspirin. In such patients theaddition of extended-release dipyridamole has beenshown to reduce the risk of further cerebrovascularembolic events. (European Stroke Prevention Study 2.) Atransoesophageal echocardiogram should also beperformed to exclude a patent foramen ovale as a cause ofparadoxical emboli.

A 36-year-old woman presented with a four-week historyof intermittent upper abdominal pain and vomiting. Herbowel movements were unaffected and were normal.There was no blood or mucus in the stool. Her appetitewas reduced. She had lost 2 kg in weight. She had a pastmedical history of ulcerative colitis which was diagnosedat the age of 14 years, and was stable on sulphasalazine.In the past she had received several courses of steroids foracute exacerbation of her colitis. Her only other drughistory was that she was currently on the contraceptivepill. She had been married for 10 years and had two

children, aged 7 and 3 years. She smoked 15 cigarettesper day and consumed 1–2 units of alcohol per week.

On examination, she appeared unwell. Her tonguewas dry and there was loss of skin turgor. She had mildlower limb pitting oedema. The heart rate was 98beats/min and blood pressure was 100/65 mmHg. Thetemperature was 36.8°C (98.2°F). The abdomen wasslightly distended, and there was generalized tenderness.The liver was palpable 4 cm below the costal margin, andwas tender. There was no evidence of a palpable spleen orany other palpable masses in the abdomen. Percussion ofthe abdomen revealed shifting dullness. Rectalexamination was normal. Examination of the respiratoryand cardiovascular system was normal.

Investigations are shown.

Question 104

Hb 13 g/dlWCC 8 � 109/lPlatelets 190 � 109/lSodium 131 mmol/lPotassium 3.1 mmol/lUrea 7.2 mmol/lCreatinine 100 �mol/lBilirubin 20 μmol/lAST 24 iu/lALT 21 iu/lAlkaline phosphatase 150 iu/l Albumin 29 g/lChest X-ray Normal heart size

Clear lung fields

1. What two investigations would you request to ascertain the cause of the presentation?

2. What is the diagnosis?a. Primary sclerosing cholangitis.b. Carcinoma of the colon complicated by

hepatic metastases.c. Hepatic vein thrombosis.d. Cirrhosis of the liver.e. Chronic active hepatitis.

Page 115: Rapid review of clinical medicine for mrcp part 2

114

The patient has a long history of relatively quiescentulcerative colitis and presents with a short history ofintermittent upper abdominal pain, vomiting, tenderhepatomegaly and ascites. There is no history of weightloss preceding the illness. However, she is a smoker andon the oral contraceptive pill, both of which are recog -nized risk factors for venous thrombosis. Ulcerativecolitis itself is associated with an increased risk of venousthrombosis owing to increased fibrinogen levels and theelevated plasma viscosity. The most likely diagnosis inthis patient is hepatic vein thrombosis. Although portal

vein thrombosis may also occur in ulcerative colitis, theabnormal LFT and hepatomegaly would be against thediagnosis. The differential diagnosis comprises carcinomaof the colon with hepatic metastases, CAH, cirrhosis ofthe liver and sclerosing cholangitis. Carcinoma of thecolon is ten times more common in a patient with long-standing ulcerative colitis than in the general population;however, it is unusual for a patient with relatively well-controlled colitis to present with extensive metastatic liverdisease without a prodrome of ill health or bleeding perrectum. Primary sclerosing cholangitis is stronglyassociated with ulcerative colitis. Some 75% of all caseswith primary sclerosing cholangitis occur in patients withulcerative colitis. The activity of the colitis is inverselyrelated to the severity of sclerosing cholangitis. It isusually asymptomatic in the early stages, the onlyindicator being a raised alkaline phosphatase. Pruritus,jaundice, abdominal pain and weight loss are features ofadvanced cholangitis. Rapid deterioration may occurwhen sclerosing cholangitis is complicated by acholangiocarcinoma. If this patient’s presentation weredue to primary sclerosing cholangitis or cholangio -carcinoma complicating sclerosing cholangitis, one wouldexpect a higher bilirubin level and alkaline phosphataselevel, particularly if the associated cirrhosis was severeenough to cause hepatocellular failure, as is suggested bythe low albumin and ascites. Chronic active hepatitis isalso recognized in ulcerative colitis, but the normaltransaminase level is against the diagnosis.

The investigation of choice is liver US with Dopplerstudies on hepatic venous flow. Liver biopsy and hepaticvenography are also useful in making the diagnosis. Aliver isotope scan may demonstrate preservation of theRiedel’s lobe.

All of the complications mentioned above may alsocomplicate Crohn’s disease; however, primary sclerosingcholangitis, cholangiocarcinoma and the predispositionto venous thromboses is much more common inulcerative colitis. Perianal ulcers are much more commonin Crohn’s disease and, in addition, patients with Crohn’sdisease have a higher incidence of gallstones than thosewith ulcerative colitis. The complications of ulcerativecolitis are shown (Table). With the exception ofankylosing spondylitis and hepatobiliary disease, all thecomplications of ulcerative colitis are relieved byproctocolectomy.

Answer 104

1. i. Liver ultrasound and Doppler studies. ii. Liver biopsy via internal jugular vein.

2. c. Hepatic vein thrombosis.

Complications of ulcerative colitis

Gastrointestinal• Haemorrhage• Toxic dilatation• Perforation• Carcinoma of the colon• Oral and anal ulcers

Hepatobiliary• Fatty infiltration of the liver• CAH• Cirrhosis of the liver• Sclerosing cholangitis

Dermatological• Erythema nodosum• Pyoderma gangrenosum

Ophthalmic• Episcleritis• Scleritis• Anterior uveitis

Rheumatological• Seronegative arthritis of the small joints• Ankylosing spondylitis

Haematological• Predisposition to venous thromboses

Page 116: Rapid review of clinical medicine for mrcp part 2

Clinical Cases 115

A 40-year-old woman presented with a five-day history ofrecurrent falls and an unsteady gait. She had a pasthistory of a stroke causing a right-sided hemiparesis,which resolved spontaneously after a few days. Thepatient underwent intensive investigation followingpresentation with the stroke including carotid Dopplerstudies, transoesophageal echocardiography and CT scanof the brain, which were normal.

On examination she had a broad-based gait. There wasevidence of dysdiadochokinesia in both upper limbs andabnormal heel–shin testing. The lower limb reflexes werebrisk and the plantar response was extensor. The heart rate

was 80 beats/min and regular. The blood pressure was130/90 mmHg. Both heart sounds were normal.

Question 105

What is the most probable diagnosis?a. Systemic lupus erythematosus with cerebral

involvement.b. Multiple sclerosis.c. Multiple paradoxical emboli.d. Friedreich’s ataxia.e. Multiple cerebral metastases.

A 30-year-old male presented with weight loss andpalpitation for six months. He had lost approximately 4 kg. The patient had been well previously. He was nottaking any medication. There was no family history of note.Apart from an orchidopexy affecting the right testicle ten

years ago, there was no past history of note. Onexamination he had a resting tachycardia and evidence ofbilateral gynaecomastia. There were no other abnormalities.

Investigations are shown.

Questions 106 and 107

Hb 14 g/dlWCC 6 � 109/lPlatelets 180 � 109/lSodium 135 mmol/lPotassium 4.3 mmol/lUrea 5 mmol/lAlbumin 38 g/lAlkaline phosphatase 190 iu/l (NR 25–115 iu/l)Alpha-fetoprotein 8 ku/l (NR <10 ku/l)Human chorionic 1250 iu/l

gonadotrophinTSH <0.01 mu/lThyroxine 300 nmol/l

(NR 60–160 nmol/l)Testosterone 60 nmol/l

(NR 10–35 nmol/l)Oestradiol 1300 pmol/l

(NR 500–1100 pmol/l)

Question 106

What is the diagnosis?a. Leydig cell tumour.b. Choriocarcinoma.c. Teratoma.d. Seminoma.e. Thyroid carcinoma.

Question 107

What investigation is required to help make thediagnosis?

a. Ultrasound testes.b. CXR.c. CT thorax.d. FSH level.e. FNA thyroid.

A 42-year-old man required ventilation for a prolongedperiod during an episode of septicaemia. Following thishe developed difficulty with walking and required the aidof an assistant to mobilize. On neurological examinationthere was weakness on dorsiflexion of the toes, as well asankle eversion. The patient also had reduced sensation,affecting the anterior lateral aspects below the knee andthe dorsum of the foot.

Question 108

What is the neurological diagnosis?a. L5 radiculopathy.b. L4 radiculopathy.c. Common peroneal nerve palsy.d. Femoral nerve palsy.e. Guillain–Barré syndrome.

Page 117: Rapid review of clinical medicine for mrcp part 2

116

The patient has had a previous right hemiparesis and nowpresents with ataxia. Neurological examination revealsfindings consistent with a cerebellar syndrome and bilateralpyramidal tract involvement. The differential diagnosis ofcerebellar and pyramidal tract involvement includesmultiple sclerosis, subacute combined degeneration of thespinal cord, Friedreich’s ataxia, multiple cerebral metastasesand multiple cerebral infarcts.

The most probable diagnosis is multiple sclerosis, ademyelinating disorder characterized by involvement ofthe optic tracts, pyramidal tracts, cerebellar peduncle andthe posterior columns of the spinal cord.

The normal CT scan of the brain and carotid Dopplerstudies are against the diagnosis of multiple cerebralmetastases and infarctions. A normal transoesophagealechocardiogram is against the diagnosis of paradoxicalemboli. Whereas SLE may manifest with multipleneurological manifestations, the patient does not haveany other manifestations of a multi-system disorder. Withspecific reference to SLE there is no history of arthralgia,myalgia or a rash. Whilst Friedreich’s ataxia may beassociated with pyramidal tract and cerebellar involve -ment, the condition usually presents at a much youngerage and is associated with progressive neurologicaldeterioration. The condition also affects the dorsal rootganglia, which causes absent peripheral reflexes.

Answer 105

b. Multiple sclerosis.

The patient had weight loss, palpitation and suppressedTSH, suggesting hyperthyroidism. Furthermore he hasgynaecomastia, raised testosterone and oestrogen levelsas well as a high human chorionic gonadotrophin level.This combination is highly suggestive of a chorioniccarcinoma; chorionic carcinomas are characterized byhigh HCG levels. The alpha subunit in HCG is similar tothat found in TSH, FSH and LH, therefore patients withchoriocarcinoma may be hyperthyroid and have raisedtestosterone and oestrogen.

Testicular tumours can be divided into germ-celltumours and sex-cord stromal cell tumours. Germ-celltumours include seminomas, choriocarcinomas,teratomas, embryonal cell cancer, and yolk sac tumours.Sex-cord tumours comprise Leydig cell tumours andtumours affecting the cells of Sertoli. 95% of all testiculartumours are germ-cell tumours. 50% of all germ-celltumours are seminomas. Seminomas are most commonin the fourth decade. Tumour markers are usuallynormal. Testicular teratomas are relatively rare. They maybe found in prepubertal males and in adults. Tumourmarkers are usually negative. Choriocarcinomas areassociated with high HCG levels.

Leydig cells are the commonest sex-cord stromaltumours that have the ability to produce both oestrogenand testosterone.

Answers 106 and 107

Answer 106b. Choriocarcinoma.

Answer 107a. Ultrasound testes.

Weakness of dorsiflexion and eversion of the foot andreduced sensation over the antero-lateral aspects of thelower leg and dorsum of the foot are typical features ofcommon peroneal nerve palsy. The common peronealnerve crosses over the fibula, where it may become

damaged owing to acute or chronic compression.Common peroneal nerve palsy occurs followingcompression due to plaster casts below the knee,excessive crossing of the legs, or prolonged pressure onthe fibula during general anaesthesia or unconsciousness.The nerve may be also be damaged during trauma oraffected by diabetic neuropathy.

Answer 108

c. Common peroneal nerve palsy.

Page 118: Rapid review of clinical medicine for mrcp part 2

Clinical Cases 117

A 53-year-old male was investigated for recurrentepisodes of sinusitis and earache that did not respond toconventional antibiotics. He had experienced several

attacks in the past 18 months. Just three monthspreviously he experienced two episodes of epistaxis thatwere managed at home. He was generally unwell andtired easily. He had taken six weeks of sick leave fromwork for malaise and fever in the past six months.According to his wife he seemed to be losing weight. Onfurther questioning the patient complained of a dry non-productive cough usually at the same time that he hadeither sinusitis or earache. He denied haemoptysis ornight sweats. He worked as a postman. He did notsmoke and consumed alcohol on an infrequent basis.Apart from a holiday in Kenya three years ago, he hadnot left the country for ten years. There was a familyhistory of adult onset asthma. His father had died frombronchial carcinoma.

On examination, he was thin. There was no clubbingor lymphadenopathy. His heart rate was 98 beats/min.The blood pressure measured 148/96 mmHg. The JVPwas not raised. Cardiovascular examination was normal.Examination of the upper respiratory tract revealedcrusting of the inner aspect to the nostrils. The throatwas normal. Auscultation of the lung fields was normal,as was examination of the abdomen. On neurologicalexamination, Rinne’s test was positive in the right ear.There was no skin rash.

Investigations are shown.

Question 109

Hb 11 g/dlWCC 15 � 109/l

(neutrophils 12 � 109/l,lymphocytes 2 � 109/l,eosinophils 0.35 � 109/l)

Platelets 490 � 109/lMCV 80 flESR 60 mm/hCRP 90 g/lSodium 138 mmol/lPotassium 4.1 mmol/lUrea 7 mmol/lCreatinine 110 �mol/lBilirubin 11 �mol/lAST 23 iu/lALT 25 iu/lAlkaline phosphatase 156 iu/l (NR <115 iu/l)Albumin 31 g/lImmunology:

pANCA Positive(myeloperoxidase)

cANCA NegativeANF NegativeC3 0.9 g/l (NR 0.55–1.2 g/l)C4 0.3 g/l (NR 0.2–0.5 g/l)

Chest X-ray Right middle lobe consolidation

Urinalysis 24-hour protein 0.8gBlood ++Casts absent

What is the most likely diagnosis?a. Classic polyarteritis nodosa.b. Churg–Strauss syndrome.c. Microscopic polyangiitis.d. Mixed essential cryoglobulinaemic vasculitis.e. Wegener’s granulomatosis.

A 40-year-old homosexual male presented to theAccident and Emergency Department with a four-dayhistory of sore throat and fever. On examination he hadcervical and axillary lymphadenopathy and widespreadmaculopapular rash. An HIV antibody test was negative.

Question 110

Which one of the following investigations would youperform next to help identify the cause of his illness?

a. Blood film.b. Monospot test.c. HIV viral RNA load.d. CD4 count.e. CT scan of the thorax and abdomen.

Page 119: Rapid review of clinical medicine for mrcp part 2

118

The presentation is consistent with HIV seroconversion.Most patients can be diagnosed by conventional ELISAtests, which identify the presence of HIV antibodies bythe time the patient has features of HIV seroconversion.However, if HIV antibody testing proves negative, then

the diagnosis can be confirmed by measuring HIV viralRNA load. HIV viral load is not routinely used todiagnose HIV infection because it is labour intensive, butit is useful in diagnosing acute HIV infection, helping toresolve indeterminate HIV antibody results and innewborn babies where there is passive transfer ofantibody from the mother to the baby. A low CD4 countalone is not specific for HIV infection.

The patient presents with specific symptoms of upper andlower respiratory tract involvement. Additionally, he hasmalaise, arthralgia, weight loss and raised inflammatorymarkers suggestive of a systemic illness. The presence ofblood and protein in the urine indicates renal involve -ment. A normal creatinine concentration does notpreclude renal involvement. The differential diagnosis isthat of a multi-system disease capable of involving theupper and lower respiratory tract as well as the kidneys.The presence of a positive ANCA test is suggestive of asmall-vessel vasculitis. Possibilities include Wegener’sgranulomatosis, microscopic polyangiitis andChurg–Strauss syndrome.

Wegener’s granulomatosis and microscopicpolyangiitis are both small-vessel vasculitides associatedwith ANCA. Both are characterized by upper and lowertract involvement (sinusitis, epistaxis, otitis media,haemoptysis) and renal involvement. Other featurescommon to both include episcleritis, peripheral/centralnervous system involvement and effects on thegastrointestinal tract. While manifestations of the upperand lower respiratory tract involvement may be identicalin both of these small-vessel vasculitides, the presence ofgranulomata in small vessels in biopsy specimens isspecific for Wegener’s granulomatosis. The ANCAsubtype may also aid differentiation between Wegener’sgranulomatosis and microscopic polyangiitis. Wegener’sgranulomatosis is characterized by the presence of

cANCA (antigen is proteinase 3) whereas microscopicpolyangiitis is characterized by presence of pANCA(antigen is myeloperoxidase). The presence of pANCA inthis case favours the diagnosis of microscopicpolyangiitis.

Churg–Strauss syndrome is a small-vessel vasculitisaffecting the upper and lower respiratory tract as well asthe skin and the peripheral nervous system. Nasal polyps,allergic rhinitis and adult-onset asthma usually predatethe vasculitis by many years. Renal involvement is usuallymild but can be severe. A mild eosinophilia (usually <2 �109/l) is a recognized feature. Patients have a positivepANCA directed specifically against myeloperoxidase.(See Questions 4, 396.)

Classic PAN is vasculitis that can affect medium andsmall vessels. It is characterized by micro-aneurysms,tissue infarction, haemorrhage and organ dysfunction.Fever, malaise, gastrointestinal, cardiac, CNS and softtissue involvement is usual. The diagnosis is best made byarteriography. Renal biopsy is usually not diagnosticunless a medium-sized vessel is included in the biopsyspecimen. The glomeruli are normal. ANCA is charac -teristically absent. (See Question 174.)

Mixed essential cryoglobulinaemia hepatitis C virus isimplicated in 80% of cases. Clinical features includepupura, arthralgia, membranoproliferative glom -erulonephritis, hepatosplenomegaly and neurologicalmanifestations. Rheumatoid factor levels are often high.Complement levels are low. ANCA is usually absent.(See Question 15.)

Answer 109

c. Microscopic polyangiitis.

Answer 110

c. HIV viral RNA load.

Page 120: Rapid review of clinical medicine for mrcp part 2

Clinical Cases 119

A 68-year-old male complained of dyspnoea on minimalexertion. Respiratory function tests are shown.

Question 111

FEV1 2.1 l (predicted 2–3 l)FVC 2.5 l (predicted 2.8–4.4 l)TLC 4.2 l (predicted 5–7 l)TLCO 90% predicted value

With what are the lung function tests mostconsistent?

a. Extrinsic allergic alveolitis.b. Pulmonary fibrosis.c. Asthma.d. Ankylosing spondylitis.e. Pulmonary hypertension.

A 66-year-old female patient presented with transientbilateral loss of vision lasting a few seconds. A few dayslater she developed a right-sided hemiparesis. There wasno history of head injury or headaches. She had a four-year history of hypertension. The patient had smoked for

40 years. Physical examination revealed a right-sidedhemiparesis but no other abnormality. The heart rate was80 beats/min and regular. The blood pressure measured160/100 mmHg.

The following test results were obtained:

Question 112

12-lead ECG Normal

2-D echocardiogram Normalwith colour flow

CT scan brain Left middle cerebral artery infarction

What is the investigation of choice to ascertain thecause of her presentation?

a. Carotid Doppler studies.b. Transoesophageal echocardiograph.c. Carotid angiography.d. MRI scan brain.e. Thrombophilia screen.

A 79-year-old man was admitted with sudden onset ofchest pain and breathlessness. On examination he wascyanosed. He had had a total hip replacement ten dayspreviously. The patient smoked ten cigarettes per day andhad hypertension for which he took nifedipine. Onexamination the heart sounds were normal and ausculta -tion of the lungs revealed a clear chest.

Investigations are shown.

Question 113

Hb 13 g/dlWCC 13 � 109/lPlatelets 250 � 109/lCRP 28 g/lECG Sinus tachycardia and LBBBArterial blood gases (air):

pH 7.49PaCO2 3.1 kPaPaO2 8.8 kPaHCO3 28 mmol/l

What is the most probable diagnosis?a. Pulmonary embolism.b. Acute myocardial infarction.c. Fat embolism syndrome.d. Pneumonia.e. Pulmonary oedema secondary to hypertensive

heart disease.

Page 121: Rapid review of clinical medicine for mrcp part 2

120

The patient has evidence of multiple transient ischaemicepisodes followed by a right-sided hemiparesis. CT scanof the brain reveals a left middle cerebral arteryinfarction, which is consistent with the final presentation.There is no evidence of any other abnormality in thebrain. Possible causes of multiple cerebral emboli in thisparticular case include thrombi from the carotid arteries,calcific plaques from a degenerative aortic valve, andparadoxical emboli from a patent foramen ovale. In theabsence of atrial fibrillation, the commonest cause ofthromboembolism to the brain is atheromatous carotidartery disease, particularly in patients with hypertension.

The normal transthoracic echocardiogram certainlyexcludes calcific thromboemboli from degenerative aorticvalve disease, but a patent foramen ovale cannot beexcluded with certainty. However, in patients aged over40, carotid artery disease is a much commoner cause ofstroke than a patent foramen ovale.

The initial investigation of choice is a carotid Dopplerstudy (see Answer 166). In the absence of significantcarotid artery disease, a transoesphageal echocardiogramshould be performed to exclude a patent foramen ovale.A thrombophilia screen is indicated in young patientswith stroke and in those with a previous history ofarterial and venous thrombosis or a family history ofarterial or venous thrombosis.

The patient has a restrictive lung defect (FEV1/FVCratio approx. 80%), a low total lung capacity and arelatively normal transfer factor. These findings areconsistent with either a thoracic cage deformity, a

neuromuscular defect affecting the respiratory muscles ormassive pleural fibrosis. From the options given,ankylosing spondylitis is the best answer.

Extrinsic allergic alveolitis and pulmonary fibrosis areassociated with a low transfer factor. Asthma causes anobstructive respiratory defect and pure pulmonaryhypertension does not usually affect the lung volumes.

Answer 111

d. Ankylosing spondylitis.

Answer 112

a. Carotid Doppler studies.

The presentation with sudden chest pain, dyspnoea,hypoxia and hypocarbia ten days after a hip replacementis highly suggestive of pulmonary embolism. The ECGreveals LBBB, which may be due to silent coronarydisease or long-standing hypertension. The ECG inmassive pulmonary embolism classically reveals rightventricular strain pattern or RBBB. Although it ispossible that the presentation may represent acute

myocardial infarction, the blood gases cannot beexplained by the absence of pulmonary oedema, yet hischest is clear on auscultation.

Fat embolism syndrome is a condition that may occurafter bony fractures (more common with closed thanopen fractures). It is characterized by hypoxaemia,petechial rash and neurological abnormalities usuallywithin 72 hours (but rarely before 12 hours) after theprocedure. The patient above does not fulfil the diag -nosis of fat embolism syndrome.

Answer 113

a. Pulmonary embolism.

Page 122: Rapid review of clinical medicine for mrcp part 2

Clinical Cases 121

A 56-year-old woman was admitted for a left hipreplacement. She had mechanical mitral valve prosthesis.Two days before surgery the patient was commenced onintravenous heparin and was continued on a dailyinfusion of 28,000 units per day for the next 7 days. Onthe fifth post-operative day she complained of a painfullower left limb. On examination the lower limb wasswollen and tense. Doppler venography confirmed anileo-femoral deep-vein thrombosis. The patient wascommenced on warfarin but developed gangrene of theskin affecting the toes after 12 hours.

Investigations are shown.

Questions 114 and 115

Hb 10 g/dlWCC 12 � 109/lPlatelets 30 � 109/lAPTT 94 s (control 45 s)INR 3.8Factor V Leiden Absent

mutation

Question 114

What is the diagnosis?a. Protein S deficiency. b. Protein C deficiency.c. Heparin-induced thrombocytopenia type I.d. Heparin-induced thrombocytopenia type II.e. Anti-thrombin III deficiency.

Question 115

What is the next management step?a. Stop warfarin and switch to danaparoid.b. Switch to therapeutic dose of low-molecular-

weight heparin.c. Give intravenous protein C concentrate.d. Give intravenous protein S concentrate.e. Stop all anticoagulation until platelet count is

>100 � 109/l.

A patient with psoriasis was referred to the psychiatristand found to have manic depression. He was started onmedication that resulted in severe exacerbation of therash shown (116).

Question 116

Which one of the following medications is the mostlikely cause for the deterioration?

a. Amitryptilline.b. Paroxetine.c. Risperidone.d. Lithium.e. Haloperidol.

111166

Page 123: Rapid review of clinical medicine for mrcp part 2

122

The diagnosis is heparin-induced thrombocytopenia typeII, also known as heparin-induced thrombocytopenicthrombosis. In contrast with HIT type I, which is atransient isolated thrombocytopenia usually occurringafter 48 hours of therapy with unfractionated heparin,HIT type II is an immune-mediated disorder resulting inthe formation of antibodies against the heparin-plateletfactor 4 complex. HIT type II has a prevalence ofbetween 0.3 and 3% of patients who have been onheparin for between 4 and 10 days. The condition rarelyoccurs after 10 days of treatment.

HIT II is characterized by an increased level of IgGand IgM antibodies to heparin, which results in bothplatelet activation causing venous and arterial thrombosesand thrombocytopenia due to immune-mediateddestruction of platelets. Venous thromboses are morecommon than arterial ones. Manifestations include deep-

vein thrombosis, pulmonary embolism, sinus veinthrombosis, myocardial infarction, cerebrovascularthrombosis and infarction of any other organ. Anotherdifferent manifestation is skin necrosis. The conditionshould be suspected in anyone who develops anabnormally low platelet count or a >50% drop from theoriginal platelet count in association with a thromboticepisode a few days after starting heparin. The plateletcount rarely drops below 20,000 (the usual nadir isapproximately 60,000), therefore bleeding is rare.

It is important to note that there are reports ofdelayed onset HIT type II that may occur nine days afterheparin has been stopped.

The diagnosis is clinical, being based upon athrombocytopenia in association with heparin therapyand an associated arterial or venous thrombus. It isconfirmed by demonstrating heparin-induced plateletantibodies (serotonin reactive assay, heparin-inducedplatelet aggregation or solid phase immunoassay).

Type II HIT can be prevented by using low-molecular-weight heparin or limiting the use of heparinto less than five days (i.e switching to warfarin early iflong-term anticoagulation is required).

Answers 114 and 115

Answer 114d. Heparin-induced thrombocytopenia type II.

Answer 115a. Stop warfarin and switch to danaparoid.

Management of HIT (type II)

1. Stop heparin.2. Don’t switch from unfractionated heparin to LMWT heparin once the diagnosis is made (10% cross

reactivity).3. Danaparoid (heparinoids) or lepirudin (recombinant hirudin) should be started even if the platelet count is

very low unless there is active bleeding. These drugs are the mainstay of therapy for HIT type II.4. Don’t use warfarin alone if deep-vein thrombosis (can get skin necrosis due to transient acquired protein C

deficiency).5. Don’t give warfarin until platelet count is >100 � 109/l.

The patient has developed severe psoriasis. Psoriasis maybe exacerbated by drugs and infections. With respect todrugs, the commonest culprits are beta-blockers, lithium

and antimalarial drugs. Other drugs include ACEinhibitors and NSAIDs.

Viral (including HIV) and bacterial infections arerecognized exacerbating factors for psoriasis.Streptococcal infections have been particularly implicatedin guttate psoriasis.

Answer 116

d. Lithium.

Page 124: Rapid review of clinical medicine for mrcp part 2

Clinical Cases 123

A 14-year-old Asian boy diagnosed with pulmonarytuberculosis was commenced on antituberculous drugsconsisting of isoniazid, rifampicin, pyrizinamide andethambutol. Four days after commencing treatment he

presented with malaise, myalgia, nausea and anorexia. Onexamination he was febrile with a diffuse erythematousrash. The blood pressure was 90/55 mmHg and therewere crackles and bronchial breathing audible over theright upper lobe.

Investigations are shown.

Questions 118 and 119

Hb 14 g/dlWCC 12 � 109/lNeutrophils 7 � 109/lLymphocytes 2.5 � 109/lEosinophils 4 � 109/lBasophils 0.1 � 109/lMonocytes 0.4 � 109/lPlatelets 300 � 109/lESR 120 mm/hSodium 130 mmol/lPotassium 6.5 mmol/lChloride 87 mmol/lUrea 26 mmol/lCreatinine 300 �mol/lGlucose 9 mmol/lUrinary microscopy White blood cells,

red blood cells and white cell casts

24-hour urinalysis Sodium 60 mmol/lProtein 0.8 g

Question 118

What is the cause of this patient’s renal dysfunction?a. Renal tuberculosis.b. Acute tubular necrosis.c. Acute interstitial nephritis.d. Acute post-infectious glomerulonephritis.e. Rhabdomyolysis.

Question 119

What is the most important initial step in hismanagement?

a. Renal biopsy.b. Haemodialysis.c. Corticosteroids.d. Discontinuation of isoniazid.e. Discontinuation of rifampicin.

A 69-year-old woman presented with pain in hershoulders and upper back, polydipsia and polyuria.

Investigations are shown.

Question 117

Hb 9 g/dlWCC 14 � 109/lPlatelets 140 � 109/lMCV 89 flSodium 129 mmol/lPotassium 3.4 mmol/lUrea 26 mmol/lCreatinine 290 �mol/lCalcium 2.8 mmol/lPhosphate 1.9 mmol/lTotal protein 98 g/lAlbumin 34 g/l

1. What is the diagnosis?a. Multiple myeloma.b. Aplastic anaemia.c. Hyperparathyroidism.d. Chronic renal failure.e. Vitamin D toxicity.

2. What is the commonest cause of renal failure in this condition?a. Hypercalcaemia.b. Amyloid deposition.c. Tubular obstruction by light chain.d. Analgesic nephropathy.e. Hyperuricaemia.

3. Give one possible cause for the low potassium. a. Vomiting.b. Diarrhoea.c. Diuretic therapy.d. Proximal renal tubular acidosis.e. Distal renal tubular acidosis.

Page 125: Rapid review of clinical medicine for mrcp part 2

124

The patient presents with fever, rash, eosinophilia andrenal impairment shortly after commencing anti-tuberculous therapy. The findings are consistent withacute interstitial nephritis secondary to rifampicin. AIN ismost commonly due to drug therapy (Table).

The onset of AIN after drug therapy ranges from threeto five days following a second exposure, to as long asseveral weeks with a first exposure. However, the latentperiod may be as short as one day with rifampicin or aslong as 18 months with an NSAID.

Patients typically present with an acute rise in theplasma creatinine concentration temporally related to anoffending drug or infection. Fever occurs in most casesand may be accompanied by a rash. The urine sedimentusually reveals white cells, red cells, and white cell casts.Eosinophilia and eosinophiluria are present in over 75%of cases with the exception of disease induced by NSAIDswhere fever, rash, and eosinophilia are typically absent.Most patients have normal or only mildly increasedprotein excretion (<1 g/day) although some olderindividuals have significant proteinuria (approximately 3g/day). Concurrent nephrotic syndrome due to minimalchange disease is often seen with NSAIDs. Signs oftubulo-interstitial damage, such as the Fanconi syndromeand renal tubular acidosis, may also be present.

The diagnosis should be suspected from the temporalrelation to an offending drug and the characteristiclaboratory findings. The urinary findings usuallydistinguish AIN from other causes of acute renal failuresuch as glomerulonephritis and acute tubular necrosis.Acute glomerulonephritis is usually associated with thepresence of red cell casts in the urine. In acute tubularnecrosis urinalysis typically shows granular and epithelialcell casts and free epithelial cells. Even when none ofthese other conditions appears to be present, examinationof the urine is still important because some of the drugs

listed below can produce other forms of acute renalfailure.

Genitourinary tuberculosis usually has an insidiouspresentation, with dysuria and gross haematuria being themost common symptoms, and thus not the most likelydiagnosis in this case. Patients with rhabdomyolysistypically present with the triad of pigmented granularcasts in the urine, pigmenturia due to myoglobinuria anda marked elevation in the plasma level of creatine kinase.

Although the clinical picture is often highly suggestiveof AIN, the diagnosis can be confirmed only by renalbiopsy. In the absence of severe disease, it is acceptable tobegin by observing the response to discontinuation of thesuspected offending drug. No further evaluation ortherapy is required if renal function begins to improvewithin several days.

Indications for biopsy generally include uncertainty asto the diagnosis, advanced renal failure or lack ofspontaneous recovery following cessation of drugtherapy. An alternative – particularly in those who arepoor candidates for renal biopsy – is to initiate a trial ofcorticosteroids (such as 1 mg/kg of prednisone per day).Patients with AIN typically begin to improve within oneto two weeks and rapidly return to their baseline plasmacreatinine concentration.

Answers 118 and 119

Answer 118c. Acute interstitial nephritis.

Answer 119e. Discontinuation of rifampicin.

Causes of acute tubulo-interstitial nephritis

Drug-induced• Non-steroidal anti-inflammatory drugs• Penicillins • Cephalosporins• Sulphonamides• Loop and thiazide-type diuretics

Non drug-induced• Sarcoidosis• Legionella infection• Leptospirosis• Streptococcal infection

The diagnosis of myeloma is based on the very highglobulin count (albumin subtracted from total proteincontent), anaemia, renal failure and hypercalcaemia.

The question tests understanding of the renal andbiochemical complications of myeloma.

The causes of anaemia, renal failure and otherbiochemical abnormalities in myeloma are covered in theanswers above. (See also Answer 38.)

Answer 117

1. a. Multiple myeloma.2. c. Tubular obstruction by light chain.3. d. Proximal renal tubular acidosis.

Page 126: Rapid review of clinical medicine for mrcp part 2

Clinical Cases 125

A 24-year-old woman presented with a swollen left lowerlimb when she was eight weeks pregnant. Lower limbvein Doppler studies confirmed a femoral vein deep-veinthrombosis. There was no family history ofthrombophilia.

A 56-year-old male was admitted with weight loss andfatigue. He consumed 40 units of alcohol per week. Onexamination he was pale and had a palpable spleen 10 cmbelow the costal margin.

Investigations are shown.

What is the diagnosis?a. Myelofibrosis.b. Polycythaemia rubra vera.c. Myelodysplasia.d. Chronic myeloid leukaemia.e. Hypersplenism secondary to cirrhotic portal

hypertension.

Hb 8 g/dlWCC 23 � 109/lPlatelets 500 � 109/lMCV 90 flNeutrophils 58%Lymphocytes 2%Monocytes 1%Eosinophils 1%Basophils 2%Neutrophil alkaline 25 iu/l

phosphatase (NR 35–100 iu/l)

Question 122

What is the management of the patient?a. Consider termination of pregnancy.b. Start subcutaneous fractionated heparin and

continue for the entire pregnancy, maintaining an antifactor Xa >1.0 (four hoursafter injection). After pregnancy givewarfarin for six weeks.

c. Start fractionated heparin initially and switch to warfarin in the second trimester. Continue warfarin until the middle of the third trimester and then switch patient back to heparin until term.

d. Start warfarin and continue throughoutpregnancy.

e. Use high-dose aspirin during pregnancy andswitch to warfarin for six weeks post-partum.

A 67-year-old man presented with central chest painassociated with ST segment depression in the inferior andlateral leads. He had smoked 20–30 cigarettes per day forover 40 years. Troponin T 12 hours after admission was0.4 ng/l (NR <0.05 ng/l). The patient remained painfree for three days and the ECG changes resolved. Hewas treated with antiplatelet agents, a statin, ACEinhibitor and a beta-blocker.

Question 121

What is the next step in his management?a. Echocardiography prior to discharge.b. Coronary angiography prior to discharge.c. Exercise stress test prior to discharge.d. CRP.e. Discharge home and arrange outpatient exercise

test in six weeks.

Question 120

Page 127: Rapid review of clinical medicine for mrcp part 2

126

The patient has chest pain and raised troponin in theabsence of ST elevation. By definition this patient hassuffered a non-ST elevation myocardial infarction. Suchpatients have a relatively low risk of in-hospital mortalitycompared with ST segment elevation myocardialinfarction; however, six-month mortality exceeds that ofpatients with STEMI and is around 20%. Therefore, it isprudent that all appropriate patients with NSTEMI havecoronary angiography and revascularization (if required)prior to discharge from hospital. The same applies topatients who present with chest pain and marked STsegment depression even if the serum cardiac troponin isnot elevated (Table A).

In the interim the management of both groups ofpatients is essentially the same and consists of anti-thrombotic agents (aspirin and clopidogrel together),fractionated heparin, and IIb/IIIa platelet receptorblocking agents (Table B). High-dose statins may play arole in plaque pacification in the peri-infarction period.In addition beta-blockers may reduce myocardial oxygendemand. GTN infusion is useful in reducing symptoms ofacute myocardial ischaemia.

The benefits of aspirin in myocardial infarction arewell established. Over the past few years the CURE studydemonstrated that the addition of clopidogrel to aspirin

in patients with non-ST elevation acute coronarysyndrome was associated with a lower rate of adversecardiac events in the ensuing nine months. Both drugsare initiated on admission. Aspirin is continuedindefinitely whereas clopidogrel should be continued forup to 12 months. Glycoprotein IIb/IIIa inhibitors arestarted at the same time as antithrombotic therapy andsubcutaneous fractionated heparin in all patients withNSTEMI and in patients with unstable angina whocontinue to have pain and/or dynamic ECG changesrepresentative of myocardial ischaemia. Eptifibatide andtirofiban are the most commonly used IIb/IIIaglycoprotein receptor inhibitors. IIb/IIIa glycoproteinreceptor blockers are continued for up to 72 hours, bywhich time most patients should have undergonecoronary angiography.

Answer 121

b. Coronary angiography prior to discharge.

Table A Indications for in-patient coronaryangiography in patients with non-ST elevationcoronary syndromes

• Raised serum troponin T or I• Chest pain with dynamic ST segment changes

of myocardial ischaemia• Chest pain with clinical evidence of left

ventricular dysfunction• Chest pain associated with malignant ventricular

arrhythmias

Table B Management of non-ST elevationmyocardial infarction

1. Relieve pain as in ST elevation myocardialinfarction

2. Aspirin loading dose of 300 mg followed by75–150 mg daily indefinitely

3. Clopidogrel loading dose 300 mg followed by75 mg daily for 12 months

4. Fractionated heparin, usually enoxaparin(subcutaneously), until coronary angiography.

5. IIb/IIIa glycoprotein receptor blockers for upto 72 hours. Ideally most patients should havehad coronary angiography within 72 hours ofpresentation

6. Initiate high-dose statin therapy7. Beta-blocker therapy on admission as with ST

elevation myocardial infarction8. Coronary angiography in all patients where

general health and comorbidities allow for theprocedure to be performed safely

9. ACE inhibitor prior to discharge

The patient has large splenomegaly and a high white cellcount consisting predominantly of neutrophils. Thedifferential diagnosis is between CML and myelofibrosis.Both conditions may be associated with much higherwhite cell counts; however, CML can be differentiated

from myelofibrosis by karyotyping for Philadelphiachromosome, measurement of neutrophil leucocytealkaline phosphatase level and bone marrow analysis.Patients with CML usually have the Philadelphiachromosome abnormality and low neutrophil alkalinephosphatase levels. Further support for the diagnosis ofCML in this question is the relatively high basophilcount. (See Question 392.)

Answer 120

d. Chronic myeloid leukaemia.

Page 128: Rapid review of clinical medicine for mrcp part 2

Early use of high-dose statins, such as simvastatin40 mg or atorvastatin 40–80 mg, has been shown toreduce adverse cardiac events 30 days after the initialinsult. There are no trials assessing the effect of beta-blockers in NSTEMI; however, given the magnitude ofprognostic benefit of this group of drugs in patients withSTEMI, it seems reasonable to deduce that they areprobably effective in the management of patients withNSTEMI.

It is unclear whether early initiation of an ACEinhibitor alters the immediate prognosis in patients withNSTEMI unless the patient has left ventriculardysfunction. However, the long-term benefits of ACEinhibitor therapy in patients with coronary artery diseaseare now well established (EUROPA and HOPE studies)and it is recommended that all patients presenting withNSTEMI are initiated on an ACE inhibitor prior todischarge home, provided there are no contraindications.

Clinical Cases 127

The patient requires anticoagulation but warfarin iscontraindicated in the first 13 weeks of pregnancy as it isteratogenic (skeletal, cartilage and foetal CNSabnormalities). Heparin, on the other hand, does notcross the placenta and has not been associated withteratogenic effects. The recommendation is to use eitherunfractionated or low-molecular weight heparin, at leastin the first 13 weeks.

Following this there are three options, as follows:

1. Continue dose-adjusted unfractionated heparinthroughout pregnancy, maintaining an APPT twicethat of the normal range. However, treatment withunfractionated heparin requires daily monitoring ofthe APTT, which is inconvenient and impractical.Long-term unfractionated heparin injections areassociated with a high risk of osteoporosis.

2. Continue weight-adjusted low-molecular weightheparin aiming for a 4-hour post injection antifactorXa of 1.0 u. Antifactor Xa levels need to be measuredonce or twice a month as the patient is gainingweight. This is currently the anticoagulant strategy ofchoice.

3. Start with either unfractionated or low-molecularweight heparin during the first trimester and thenswitch to warfarin until the mid-third trimester(maintenance INR of 2). During the mid-thirdtrimester the patient should be switched back toheparin until term because warfarin is associated witha higher risk of foetal haemorrhage during vaginaldelivery and the baby’s INR cannot be reversed bygiving the mother fresh-frozen plasma during labour.

In all three cases above the patient should beanticoagulated with warfarin for six weeks followingdelivery, as these women have low antithrombin IIIlevels, increase in factors I, II, VII, VIII and X and haveincreased venous stasis.

Aspirin is not effective in preventing propagation of avenous thrombus or preventing pulmonary embolism inpregnancy.

Answer 122

b. Start subcutaneous fractionated heparin andcontinue for the entire pregnancy, maintaining an antifactor Xa >1.0 four hours after injection of heparin. After pregnancy give warfarin for six weeks.

A 70-year-old obese male saw his GP and was noted tohave a blood pressure of 170/98 mmHg. The fundi werenormal. Urinalysis was normal. Urea, electrolytes andblood sugar were normal. The total cholesterol measured6 mmol/l. The patient was advised to lose weight andadhere to a low-salt diet. Blood pressure readings overthe next six months were 170/96 mmHg, 168/96mmHg and 166/96 mmHg.

Question 123

What is the treatment for the hypertension?a. Continue lifestyle modification for another six

months.b. Bendroflumethiazide (bendroflumethazide).c. Ramipril.d. Atenolol.e. Doxazocin.

Page 129: Rapid review of clinical medicine for mrcp part 2

128

The patient has moderate to severe hypertension andrequires l ifestyle modification advice as well aspharmacological therapy to help control his bloodpressure. Hypertension in the elderly is best treated withcalcium channel blockers or a thiazide diuretic. The useof thiazide diuretics as first line in most elderly patientswith hypertension is supported by the recently publishedALLHAT study, which was the largest study ever in thehypertensive population. The study showed that thiazidediuretics were more effective than beta-blockers, calciumchannel blockers and angiotensin-converting enzymeinhibitors at reducing cardiovascular mortality indiabetics and non-diabetics.

All patients should receive advice regarding lifestylemodification to help reduce the blood pressure (Table A).Lifestyle modification alone without antihyper tensivedrug therapy for a period of six months to one year isreserved for patients with mild hypertension (BP140–159/ 90–99 mmHg) who do not have any evidenceof end-organ damage such as retinopathy, abnormal renalfunction, proteinuria, or any history of diabetes,nephropathy, stroke, coronary artery disease or heartfailure (Table A). Most patients with hypertensionrequire antihypertensive drug therapy.

The British Hypertension Society recommends thatany one of the five major classes of antihypertensive drugs(thiazides, beta-blockers, calcium channel blockers, ACEinhibitors and angiotensin II receptor blockers) may beused as first-line monotherapy, although most patients

require two or even three antihypertensive drugs to helpcontrol blood pressure. All of these groups of drugs havebeen shown to reduce cardiovascular mortality andstroke. The aim is to bring down the blood pressure to≤140/90 mmHg. In patients with diabetes, stroke,coronary disease, heart failure or nephropathy the goalsare more stringent and the aim is to reduce bloodpressure to ≤130/85 mmHg.

During initiation of antihypertensive drugs, the ABCDmethod of prescription is useful for stepwise managementof hypertension. The system is based on the fact thatmost young patients (aged ≤55 years) and non-blackpatients have high plasma renin levels and respond best to drugs primarily targeting therenin–angiotensin–aldosterone system when used asmonotherapy. These patients respond to ACE inhibitors,Angiotensin receptor blockers, or Beta-blockers asmonotherapy (i.e. the A and B drugs). In contrast olderpatients and black patients (Afro-Caribbean in origin)respond better to drugs promoting natriuresis andreducing plasma volume such as Calcium channelblockers and Diuretics (i.e. the C and D drugs) whenused as single agents (Table B). Addition of an A or Bdrug to a patient whose blood pressure is still elevatedwhile taking a C or D drug and vice versa has an additiveeffect because in combination, these drugs counteractdifferent mechanisms responsible for high blood pressure.

This particular system should not be applied to everycase as there may be compelling indications to use A or Bclass drugs in patients aged >55 years or black patients ifthere is a history of diabetes (ACE inhibitor or A IIreceptor blocker) or heart failure (all of the A or B drugs).

Answer 123

b. Bendroflumethiazide (bendroflumethazide).

Table A Lifestylemodification

• Lose weight to bringBMI down below 30

• Reduce alcoholconsumption

• Regular exercise for30–45 min three timesper week

• Reduce salt intake to6 g/day

• Increase potassium to90 mmol/day

• Smoking cessation

Table B The stepwise management of hypertension using the ABCD system

Young Age ≥55 yearsNon-black patients Black patients

Step 1 A or B C or D

Step 2 A or B + C or D

Step 3 A C D

Step 4 A C D+

Spironolactone or doxazocin

Page 130: Rapid review of clinical medicine for mrcp part 2

Clinical Cases 129

A 54-year-old non-insulin-dependent diabetic patientwas referred to the blood pressure clinic for assessmentand control of blood pressure. He was a non-smoker.He had a body mass index of 28. The blood pressure inclinic measured 180/98 mmHg on three successiveoccasions. His medication comprised metformin 1 gtwice daily, simvastatin 40 mg od, aspirin 75 mg andamlodipine 10 mg od. Renal function was normal buturinalysis revealed proteinuria +. The HbA1c was 7%.The total cholesterol was 5.2 mmol/l. The 12-leadECG revealed voltage criteria for left ventricularhypertrophy.

What is the most effective initial pharmacological stepin reducing cardiovascular mortality?

a. Weight loss.b. Increase dose of statin.c. Start atenolol.d. Start bendroflumethiazide.e. Start losartan.

Question 125

A 70-year-old male presented with a four-week history ofincreasing dyspnoea and swollen ankles. There was noevidence of ischaemic heart disease, diabetes or alcoholabuse. On examination he had clinical and radiologicalevidence of pulmonary oedema. The blood pressuremeasured 180/100 mmHg. He was treated withintravenous furosemide with good results.

Investigations are shown.

Question 124

Hb 10 g/dlWCC 10 � 109/lPlatelets 200 � 109/lSodium 139 mmol/lPotassium 5.1 mmol/lUrea 14.9 mmol/lCreatinine 250 �mol/lAlbumin 29 g/lChest X-ray after diuretic treatment (124)ECG Sinus rhythm;

small complexes; Q waves inferior and lateral leads

Urinalysis Protein ++Renal ultrasound Normal kidneys

What is the most probable cause for the heart failure?a. Hypertensive heart disease.b. Anaemia.c. Ischaemic heart disease.d. Cardiac amyloid.e. Renal artery stenosis.

112244

Page 131: Rapid review of clinical medicine for mrcp part 2

130

The most effective method of reducing cardiovascularmortality in this patient with non-insulin-dependentdiabetes would be to control the blood pressureeffectively. Weight loss alone in a patient with thismagnitude of hypertension would not be enough toreduce blood pressure.

While all groups of antihypertensive drugs listed havebeen shown to be equally efficacious in bringing theblood pressure down in 40–60% of patients when used asmonotherapy, there are clinical settings in which certaingroups of drugs may have additional benefits oncardiovascular mortality. In patients with non-insulin-dependent diabetes mellitus, both angiotensin II receptorblockers and angiotensin-converting enzyme inhibitorshave been particularly effective in reducing cardiovascularmortality, as shown in the HOPE study and LIFEstudies, respectively.

The LIFE trial evaluated the specific cardiovascularbenefits of losartan in high-risk patients with moderateto severe hypertension (rest ing BP 160–200/95–115 mmHg) and electrocardiographic evidence ofleft ventricular hypertrophy.

Patients were randomly assigned to either losartan oratenolol therapy, with dose increases and the addition ofhydrochlorthiazide (as well as other agents) to attain the

target blood pressure levels (resting BP<140/90 mmHg). The initial and subsequent doses ofboth losartan and atenolol were 50 and 100 mg/day,respectively. The fall in blood pressure was similar in thetwo groups (30/17 mmHg).

The study showed that losartan was more effectivethan atenolol in reducing mortality, primarily owing to areduction in fatal or non-fatal stroke. The benefit fromlosartan was more pronounced among diabetics com -pared to non-diabetics. The benefit was most prominentin diabetic patients who had not previously received anti-hypertensive therapy.

The recently published ALLHAT study, which was thelargest study ever in the hypertensive population, showedthat thiazide diuretics were more effective than beta-blockers, calcium channel blockers and angiotensin-converting enzyme inhibitors at reducing cardiovascularmortality in diabetics and non-diabetics. However,patients in the ALLHAT study did not have severehypertension and not all had ECG criteria for leftventricular hypertrophy. Furthermore, the ALLHATstudy did not evaluate angiotensin receptor blockersspecifically. In this particular case one would add up to100 mg losartan initially and introduce a thiazide diureticif the BP was still >140/90 mmHg.

The patient will also require additional cholesterollowering therapy in the form of a statin; however, in thiscase BP reduction is the priority.

Answer 125

e. Start losartan.

The commonest causes of heart failure in the UK areischaemic heart disease and hypertension. However, theclue to the answer in this question is in the interpretationof the chest X-ray, which shows expansile lytic lesions inthe ribs consistent with the diagnosis of multiplemyeloma. A significant proportion of patients withmultiple myeloma develop amyloidosis. Cardiac amyloidis associated with restrictive dilated cardiomyopathy,conduction disturbance and angina due to amyloiddeposits in the coronary arteries. The characteristicfeatures of cardiac amyloid on the 12-lead ECG includesmall complexes, pseudo-infarcts (q-waves) and heartblock (see Question 389). Amyloidosis commonly causesrenal failure in multiple myeloma as in this case (seeQuestion 38).

Almost 80% of patients have evidence of boneinvolvement from myeloma at the time of diagnosis.

Bone involvement is characterized by expansile lyticlesions, osteopenia and pathological fractures. Osteolyticlesions are thought to be due to stimulation ofosteoclastic activity. The vertebral bodies, skull thoraciccage, pelvis and proximal aspects of the humeri andfemora are the commonest sites of bone lesions. Themost serious complication of bone involvement is cordcompression from vertebral body collapse.

Technetium bone scans, which primarily detectosteoblastic activity, are not as useful as conventionalplain radiographs (skeletal survey) at identifying theextent of bone involvement in multiple myeloma.

All patients with multiple myeloma should be treatedwith bisphosphonates to reduce the risk of skeletalcomplications. Treatment with bisphosphonates reducesbone pain, and the rate of pathological fractures and cordcompression. A small proportion of patients onbisphosphonates for myeloma may develop nephroticsyndrome due to focal segmental glomerulosclerosis.

Answer 124

d. Cardiac amyloid.

Page 132: Rapid review of clinical medicine for mrcp part 2

Clinical Cases 131

A 69-year-old male was seen in the renal cliniccomplaining of fatigue and dyspnoea. He had chronicrenal impairment secondary to focal segmentalglomerulonephritis and had been on haemodialysis for sixmonths. The patient was noted to have been anaemicthree months ago for which he was commenced on oraliron supplements. He had controlled hypertension butno other medical history. Medication comprisedamlodipine 10 mg od, ferrous sulphate 200 mg bd andsimvastatin 40 mg od.

On examination he appeared pale. There was noevidence of oedema. The heart rate was 90 beats/min andregular. The blood pressure measured 140/80 mmHg.The JVP was not raised. On examination of theprecordium the apex was not displaced. Auscultation of theheart was normal and the chest was clear.

Investigations are shown.

Question 126

Hb 9.9 g/dlWCC 5 � 109/lPlatelets 200 � 109/lMCV 88 flSodium 138 mmol/lPotassium 5.1 mmol/l Urea 20 mmol/lCreatinine 340 �mol/lCalcium 2.0 mmol/lPhosphate 1.3 mmol/lAlbumin 38 g/lGlucose 4 mmol/lIron 9 mmol/l (NR 14–32 mmol/l)TIBC 50 mmol/l (NR 40–80 mmol/l)Ferritin 36 mg/l (NR 15–250 mg/l)ECG Left ventricular hypertrophyChest X-ray Slight cardiac enlargement;

clear lung fields

How would you treat the anaemia?a. Transfuse two units of packed red cells.b. Increase dose of oral iron therapy.c. Intravenous iron therapy.d. SC erythropoietin.e. IV iron and SC erythropoietin.

A 17-year-old boy was referred to the local paediatricianbecause he had not grown very much in the past four years.Apart from feeling more lethargic than his school friends,he gave no other history of note. He measured 1.53 m. Hehad never needed to shave. His younger brother, aged 16,

measured 1.73 m and had already started shaving. Therewere no other siblings. Both parents were well. The fatherwas 1.76 m tall and the mother was 1.63 m tall.

On examination, he was slightly pale. He did not haveany facial hair. He was below the third centile for heightand was just at the 10th centile for weight. He did nothave any evidence of secondary sexual characteristics, andhis testes were small. The blood pressure was115/70 mmHg.

Investigations are shown.

Question 127

Hb 10 g/dlWCC 7 � 109/lPlatelets 200 � 109/lMCV 102 flSodium 138 mmol/lPotassium 4.3 mmol/lUrea 4 mmol/lCalcium 2.1 mmol/lPhosphate 1.1 mmol/lAlbumin 38 g/lGlucose 4.3 mmol/l

List two possible diagnoses.a. Coeliac disease.b. Growth hormone deficiency.c. Constitutional delay in puberty.d. Hypothyroidism.e. Vitamin D resistant hypophosphataemic rickets.

Page 133: Rapid review of clinical medicine for mrcp part 2

132

Treatment algorithm for anaemia in chronic renal failure

The patient has chronic renal failure and is anaemic.Anaemia is a common complication of chronic renalfailure. The cause of anaemia in CRF is multifactorial butdeficiency of erythropoietin plays an important role.Anaemia is associated with a reduced quality of life andworse clinical outcomes as well as a higher prevalence ofleft ventricular hypertrophy and left ventricular cavity sizein patients with chronic renal failure.

Anaemia is defined as an Hb <11g/dl in pre-pubertalchildren and pre-menopausal women and an Hb<12 g/dl in adult males and post-menopausal women.Patients with chronic renal failure should be maintainedon an Hb of at least 11–12 g/dl.

Other causes of anaemia, including iron deficiency,should be excluded before attributing the anaemia tochronic renal failure (Table). Iron stores should bereplenished before considering therapy witherythropoietins. In patient with chronic renal failure, themost useful markers of iron stores are the serum ferritinand the transferrin saturation which = Fe/TIBC � 100.A serum ferritin <100 mg/l and a TSAT ≤20% isindicative of absolute iron deficiency in a patient with

chronic renal failure. The patient in question hasevidence of absolute iron deficiency and requires ironsupplements before considering the need for EPO.Patients with iron deficiency usually require intravenousiron supplements to replenish iron stores, particularlypatients receiving haemodialysis. A treatment algorithmfor anaemia in chronic renal failure is shown below.Patients who are not iron deficient generally respond toerythropoietin.

Answer 126

c. Intravenous iron therapy.

Causes of anaemia in renal failure

• Erythropoietin deficiency• Chronic blood loss (iron deficiency)• Hypothyroidism• Vitamin B12 or folate deficiency• Chronic infection or inflammation• Aluminium toxicity• Malignancy• Haemolysis• Myeloma• Bone marrow infiltration• Marrow aplasia

Haemoglobin <11 g/dl

Haematinics screen

Serum ferritin <100 mg/l�

TSAT <20%

Treat with 200 mg ironsucrose weekly every three

weeks

Yes

No

Start EPO300 iu/week

Check monthlyresponse

Achieve andmaintain target Hb

No

Check haematinics onemonth post-IV iron

Serum ferritin <100 mg/l�

TSAT >20%

Yes

Hb <11 g/dlCommence EPO

Page 134: Rapid review of clinical medicine for mrcp part 2

Clinical Cases 133

A 15-year-old male presented with a four-day history ofsevere right-sided headaches affecting the orbit and rightmaxillary area. On examination he had acneform lesionsaround the nose and cheeks. Shortly after admission hebecame drowsy and developed a high temperature. Hecomplained of diplopia. Subsequent physical examinationrevealed swelling of the right eye, a partial ptosis of theright eye and a lateral gaze palsy affecting the same eye.Fundoscopy revealed papilloedema. There was noevidence of nuchal rigidity.

Question 128

What is the diagnosis?a. Septic cavernous sinus thrombosis.b. Viral encephalitis.c. Bacterial meningitis.d. Cerebral abscess.e. Cogan’s syndrome.

A 60-year-old man presents with general malaise anddrowsiness. On examination, the blood pressure was120/70 mmHg. Investigations are shown.

Question 129

Sodium 108 mmol/lPotassium 4 mmol/lUrea 3 mmol/lCreatinine 56 �mol/lBicarbonate 23 mmol/lRandom blood sugar 4.2 mmol/lUrine glucose Not detectedUrine ketones Not detected

What is the diagnosis?a. Addison’s disease.b. Hypothyroidism.c. SIADH secretion.d. Ectopic ACTH-secreting tumour.e. Gastric outlet obstruction.

This is a difficult question. There is evidence of shortstature, delayed puberty and a macrocytosis. These threefeatures can be best explained by malabsorption. Thecommonest cause of malabsorption in the Western worldis coeliac disease. The macrocytosis in this respect is dueto folate deficiency. It is unusual, however, not to alsohave biochemical evidence of osteomalacia. Thedifferential diagnosis here is hypothyroidism, which may

explain the short stature, delayed puberty andmacrocytosis; however, the problem has been present forapproximately four years and one would expect a historyof performing poorly at school or the presence of dry skinon examination. In addition, many children withhypothyroidism are relatively overweight in relation totheir height.

GH deficiency or gonadotrophin deficiency couldexplain short stature and delayed puberty, but they donot explain the macrocytosis. As with hypothyroidism,children with GH deficiency are relatively plump.

Answer 127

a. Coeliac disease.

Page 135: Rapid review of clinical medicine for mrcp part 2

134

The patient has septic dural sinus thrombosis syndrome.Three main types of dural sinus thrombosis arerecognized, notably cavernous sinus thrombosis, lateralsinus thrombosis and superior saggital sinus thrombosis.

The commonest variety is the cavernous sinusthrombosis syndrome, possibly because the facial veins draininto the sinus and the most common source of infection isdue to squeezing of nasal furuncles without antibioticcover. Other sources of infection include otitis media,sinusitis and dental infections (Table). The commonestorganism implicated is Staphylococcus aureus. Patients withcavernous sinus thrombosis present with severe peri-orbitalheadache, which also affects areas innervated by theophthalmic and maxillary branches of the trigeminal nerve.Fever and peri-orbital oedema usually develop afterwards.Ocular swelling, chemosis, ophthalmoplegia and drowsinessare recognized complications. Ophthalmoplegia is due tocompression of the third, fourth and sixth cranial nerves.Headache and associated ophthalmoplegia should alwaysalert the clinician to the possible diagnosis of cavernoussinus thrombosis.

The diagnosis of cavernous sinus thrombosis is eitherwith high resolution CT scan of the orbit with contrast ora gadolinium-enhanced MRI scan of the orbit.Management comprises intravenous flucloxacillin and

intravenous heparin. Corticosteroids may be used (after theinfection has been cleared with antibiotics) to improveinflammation around the cranial nerves. Corticosteroids aremandatory if pituitary infarction, a recognizedcomplication of cavernous sinus thrombosis, occurs.

Lateral sinus thrombosis is rare and almost alwayscomplicates otitis media. It is characterized by fifth andsixth nerve palsy and papilloedema. It may becomplicated by hydrocephalus. Treatment is as above buta radical mastoidectomy is also necessary.

Superior saggital venous thrombosis may complicatesinusitis. Thrombosis of the anterior segment of the sinuscauses headaches, which resolve on the development ofcollaterals. Complete thrombotic occlusion of thesuperior saggital sinus is invariably fatal, resulting in largevenous cortical infarction.

Cogan’s syndrome is a chronic inflammatory disordercharacterized by interstitial keratitis and vestibulo-auditory dysfunction, which may be associated with asystemic vasculitis.

Answer 128

a. Septic cavernous sinus thrombosis.

Causes of dural sinus thrombosis

• Infections of the face, ear and sinuses• Hereditary thrombophilic states• Hyperviscocity states• Oral contraceptive pill/pregnancy• Behçet’s disease

Syndrome of inappropriate ADH secretion (see Table forcauses) is the most likely diagnosis when the serumsodium is <115 mmol/l. Hypothyroidism can causefailure to excrete water, and produce a similarbiochemical picture to SIADH. The diagnosis isconfirmed by demonstrating a low plasma osmolality andan inappropriately high urine osmolality.

Answer 129

c. SIADH secretion. Causes of SIADH

Central nervous system• Head injury• Encephalitis/meningitis• Cerebral abscess• Cerebral neoplasm• Cerebral haemorrhage

Respiratory system• Bronchial carcinoma (particularly oat cell)• TB• Pneumonia (Legionnaire’s is the college favourite)• Empyema

Drugs• Chlorpropamide• Chlorpromazine• Carbamazepine• Opiates• Vincristine

Miscellaneous• Guillain–Barré syndrome• Acute intermittent porphyria• Carcinoma pancreas/thymoma

Page 136: Rapid review of clinical medicine for mrcp part 2

Clinical Cases 135

Questions 130–132

A 32-year-old woman who was 12 weeks into her firstpregnancy presented with headache and malaise. Herblood pressure in the antenatal clinic four weekspreviously was 120/70 mmHg. The current bloodpressure reading was 220/112 mmHg.

Investigations are shown.

Hb 13 g/dlWCC 7 � 109/lPlatelets 180 � 109/lESR 12 mm/hSodium 140 mmol/lPotassium 4 mmol/lUrea 6 mmol/lCreatinine 110 �mol/lAutoantibody screen NormalECG NormalUrinalysis NormalRenal ultrasound 12 cm kidney on the

right and a 7 cm kidney on the left

Question 130

What is the most probable cause for the hypertension?a. Atheromatous left-sided renal artery stenosis.b. Fibromuscular dysplasia of the left renal artery.c. Unilateral reflux nephropathy affecting left

kidney.d. Congenital atrophic kidney.e. Pre-eclampsia.

Question 131

If the patient has papilloedema what is the drugtreatment of choice?

a. IV labetolol.b. Oral methyldopa.c. Oral bendroflumethiazide.d. Oral nifedipine.e. IV nitroprusside.

Question 132

Which one of the following drugs should not be usedto treat hypertension in pregnancy?

a. Labetolol.b. Methyldopa.c. Lisinopril.d. Nifedipine.e. Hydralazine.

A 22-year-old male was admitted to hospital after anepisode of haematemesis preceded by profuse vomiting.The patient had been on an alcoholic binge drinkingsession 12 hours previously. On examination his heartrate was 90 beats/min. The blood pressure was100/60 mmHg. The Hb was 15 g/dl. An uppergastrointestinal endoscopy performed 24 hours afteradmission was normal.

Question 133

What is the next management step?a. Start ferrous sulphate.b. Start lansoprazole.c. Allow home.d. Coeliac axis angiogram prior to discharge.e. Technetium scan for Meckel’s diverticulum.

Page 137: Rapid review of clinical medicine for mrcp part 2

136

The onset of severe hypertension within a few weeks ofpregnancy in the setting of unilateral kidney disease ishighly suggestive of renal artery stenosis. Two distinctforms of renal artery stenosis are recognized, notablyatheromatous renal artery disease, which is more commonin middle-aged males with risk factors for atherosclerosis,and fibromuscular dysplasia, which usually occurs inyounger females and is characterized by narrowing of thedistal main renal artery or the intrarenal arteries.

The most probable diagnosis in this particular case isfibromuscular dysplasia affecting the left renal artery.Affected patients may present with severe exacerbationsof an otherwise relatively stable (normal or slightlyelevated) blood pressure. In pregnancy, for example, theincrease in blood volume results in increased vascularoxidative stress in the kidney in patients with renal arterystenosis, which is a stimulus for increased production ofangiotensin II, a potent vasoconstrictor. Increasedangiotensin II levels have the effect of large increases inblood pressure and precipitation of left ventricular failureor encephalopathy.

While unilateral pyelonephritis is a recognized cause ofunilateral small kidney, it does not usually result indisturbed renal function unless there is an intrinsicabnormality affecting the contralateral kidney.

Pre-eclampsia usually occurs after 20 weeks’ gestation,complicates 5–6% of all pregnancies and is much morecommon in women with pre-existing hypertension. It isdefined as hypertension and proteinuria (>0.3 g/24 hours). Up to 2% of patients with pre-eclampsiadevelop seizures and the condition is then termed

eclampsia. Pre-eclampsia may not be associated with anysymptoms, but in severe cases women may present withheadache, papilloedema and abdominal pain due to liveroedema or hepatic haemorrhage. The management ofhypertension in pre-eclampsia does not halt theprogression of the disease but does reduce the risk ofcomplications such has cerebral haemorrhage. Delivery isthe definitive method of treating the progression of pre-eclampsia but the usual practice is to allow the pregnancyto proceed as long as possible with control ofhypertension without unduly risking the mother or thefoetus. The management of hypertension in pre-eclampsia and all other causes of hypertension inpregnancy is outlined below.

In the presence of severe symptoms or evidence ofend-organ damage such as heart failure, renal failure orraised intracranial pressure, the management of severehypertension (BP >170/110 mmHg) includes either IVhydralazine or labetolol. Even in the absence of end-organ damage or symptoms there is evidence that failureto treat severe hypertension increases the risk of cerebralhaemorrhage and should be treated. IV nitroprusside iscontraindicated in pregnancy.

In patients who do not have severe hypertension, thedrug of choice is methyldopa. Second-line agents arehydralazine and nifedipine, which should be added if theblood pressure is not adequately controlled withmethyldopa. Third-line agents include alpha- and beta-blockers (labetolol and oxprenolol). Beta-blockers aregenerally avoided in the first half of pregnancy as they areassociated with foetal growth retardation. Thiazidediuretics may be used to treat hypertension in womenwith pre-existing hypertension but should be avoided inpatients with pre-eclampsia, as many such patients arevolume depleted.

ACE inhibitors and possibly also angiotensin IIreceptor blockers are contraindicated in pregnancy.Although there is no evidence that they are teratogenic,their use has been associated with foetotoxic effects suchas growth retardation.

Answers 130–132

Answer 130 b. Fibromuscular dysplasia of the left renal artery.

Answer 131a. IV labetolol.

Answer 132 c. Lisinopril.

Patients with a bleeding Mallory–Weiss tear, but withoutrisk factors for re-bleeding, clinical features indicatingsevere bleeding, or active bleeding at endoscopy willalmost always heal spontaneously. They can be managedmedically with a brief hospitalization of 24 hours.

Patients with an actively bleeding Mallory–Weiss tearat endoscopy should be treated with endoscopichaemostasis (either adrenaline injection orthermocoagulation). They should be hospitalized for atleast 48 hours because re-bleeding generally occurswithin 24 hours of the initial therapeutic endoscopy.

Answer 133

c. Allow home.

Page 138: Rapid review of clinical medicine for mrcp part 2

Clinical Cases 137

A 38-year-old man was recently diagnosed withpolycystic kidney disease after being investigated forhypertension. His father and brother also hadhypertension but had never been screened for polycystickidney disease. His paternal uncle had died from heartfailure. There was no family history of sudden death orcerebral haemorrhage. His blood pressure measured150/94 mmHg.

A 48-year-old post-menopausal woman with asthma wastaking hormone replacement therapy to preventprogression of osteoporosis, which was thought to bedue to steroid therapy. While on hormone replacementtherapy she developed a deep-vein thrombosis.

Question 135

Which of the following statements is correct?a. Aggressive management of hypertension will

reduce the rate decline of renal function.b. There is a high chance that the patient will be

dialysis-dependent by the age of 60 years.c. The patient should undergo magnetic resonance

angiography of the brain to exclude berry aneurysms.

d. Angiotensin-converting enzyme inhibitortherapy is beneficial in retarding the rate of decline of renal function.

e. Renal failure is the commonest cause of deathin this condition.

Question 134

Which alternative treatment would you use to treatthe osteoporosis?

a. 1,25-Dihydrocholecalciferol.b. Calcitonin.c. Alendronate.d. Raloxifene.e. Vitamin D and calcium supplements.

A 20-year-old male was admitted with hypertension andmalaise. His blood pressure on admission was210/124 mmHg. The mini-mental test score was10/10. Both heart sounds were normal and the chest

was clear. There was mild ankle swelling. Inspection ofthe fundi revealed grade I hypertensive retinopathy.

Investigations are shown.

Question 136

Hb 11 g/dlWCC 12 � 109/lPlatelets 120 � 109/lESR 60 mm/hSodium 138 mmol/lPotassium 7.4 mmol/lUrea 28 mmol/lCreatinine 500 �mol/lGlucose 4 mmol/l24-hour urinalysis Volume 400 ml

Protein 1.4 gBlood ++

What is the immediate management step?a. Haemodialysis.b. IV actrapid and dextrose 50%.c. IV labetolol.d. IV calcium gluconate.e. High-dose IV furosemide.

Page 139: Rapid review of clinical medicine for mrcp part 2

138

The features of polycystic kidney disease are covered inQuestion 382. Hypertension is common in patients withPKD, and patients with hypertension have a faster declinein renal function than patients who are normotensive.However, there is no conclusive trial demonstrating thattreatment of hypertension reduces the rate of decline ofrenal function. ACE inhibitors and AIIRBs do not havean effect on renal function but may improve prognosis bypreventing left ventricular hypertrophy.

Cardiovascular disease is the commonest cause ofdeath in PKD. Renal failure is the most serious renalcomplication of PKD. The median age of end-stage renalfailure is 54 years; in patients with PKD1 and PKD2 it is74 years. There is a high probability that this patient hasPKD1 since PKD1 accounts for the vast majority of casesof PKD and has an earlier presentation. The most seriousextra-renal manifestation of PKD is subarachnoidhaemorrhage due to intracerebral aneurysms. Patientswith PKD should be screened for intracranial aneurysmswith magnetic resonance angiography if there is a familyhistory of subarachnoid haemorrhage.

Answer 135

b. There is a high chance that the patient will be dialysis-dependent by the age of 60 years.

All patients with or at risk of osteoporosis should have adiet containing 800 iu of vitamin D and takesupplemental calcium 1.2 g per day. Regular exercise (30minutes three times per week) and cessation of smokingare also recommended.

Drug treatment of choice in established osteoporosisis a bisphosphonate, of which alendronate has been usedmost successfully. Other drugs include selectiveoestrogen receptor modulators such as raloxifene,oestrogen itself, calcitonin and vitamin D. The selectivereceptor modulator raloxifene has been shown to beeffective in the treatment of osteoporosis, and – unlike

oestrogen – does not promote endometrial hyperplasia orvaginal bleeding. It may reduce the risk for breast cancerbut the risk of venous thromboembolism is similar tothat of oestrogens; therefore, the drug should not beused in this particular case.

Since the Women’s Health Initiative study, whichdemonstrated that HRT did not reduce the risk ofcardiovascular disease and increased the risk of stroke,breast cancer and venous thromboembolism, oestrogentherapy is no longer first line and is only reserved forpatients who do not tolerate other treatments. Calcitoninis effective but less practical to take because it isadministered nasally and furthermore, calcitonin therapyexhibits tachyphylaxis.

Answer 134

c. Alendronate.

The patient has life-threatening hyperkalaemia, which isdefined as a serum potassium level ≥7 mmol/l orhyperkalaemia <7 mmol/l that is associated with typicalelectrocardiographic changes or muscle weakness, thelatter occur owing to antagonistic effects of potassium onthe membrane potential. In these circumstances theimmediate treatment is to stabilize the membranepotential with intravenous calcium gluconate. Intravenouscalcium acts within minutes. Its effects last for up to 60minutes, which allows definitive treatment withintravenous dextrose and insulin to take effect (Table).

Answer 136

d. IV calcium gluconate. Management of hyperkalaemia

Method AgentAntagonism of Calcium gluconatemembrane actions of potassiumDrive extra-cellular Dextrose and insulinpotassium into cells Sodium bicarbonate if acidotic

Beta-agonistsRemoval of Diureticspotassium from cells Cation exchange resins

Haemodialysis if severe or in patients with renal failure

Page 140: Rapid review of clinical medicine for mrcp part 2

Clinical Cases 139

A 5-year-old male was admitted for investigation of ankleswelling and abdominal distension that had occurred overthree months. His appetite was satisfactory, and there wasno history of diarrhoea, breathlessness, arthralgia orsyncope. He was born by a normal vaginal delivery andweighed 3.6 kg at birth. He was well as a neonate and hadachieved his milestones normally thus far. Apart fromhaving mild eczema there was no past medical history ofnote. His father was a solicitor, and his mother had justreturned to full-time teaching after giving birth to his 16-month-old brother, who was the only other sibling.There was no history of travel abroad or any drug history.

On examination, he had facial swelling. There was nopallor or cyanosis. The most striking feature was pittingoedema of the lower limbs from the ankles to the thighs.The heart rate was 100 beats/min and regular. The bloodpressure was 95/55 mmHg. The JVP was not raised.Palpation of the precordium was normal. On auscultation,heart sounds I and II were normal, but there was anadditional third heart sound. Examination of therespiratory system revealed dullness to percussion at theright lung base and reduced air entry on auscultation.Abdominal examination demonstrated a non-tender,distended abdomen with shifting dullness. There was nopalpable organomegaly.

Investigations are shown.The patient was treated with 60 mg prednisolone and

40 mg furosemide. The oedema started improving afterone week, and renal function remained normal. After twoweeks he was discharged and seen at weekly intervals. Inthe sixth week the oedema had completely subsided, andhis abdomen was soft and non-distended. A 24-hoururinary protein estimation was 0.7 g. The furosemide wasstopped and plans were made to review the patient againin two weeks, but three days later he was admitted withright upper quadrant pain and abdominal distension. Onexamination, the JVP was not raised. There was palpabletender hepatomegaly and shifting dullness. There was nolower-limb oedema.

Further investigations are shown.

113388

Hb 10 g/dlWCC 5 � 109/lPlatelets 166 � 109/lMCV 80 flUrinalysis Microscopy revealed

occasional transitional cells24-hour urinary protein 5.2 gSelective protein clearance

ratio was lowAscitic fluid Protein 8 g/lChest X-ray NormalSodium 135 mmol/lPotassium 3.7 mmol/lUrea 2.1 mmol/lCreatinine 65 μmol/lBilirubin 10 μmol/lAST 19 iu/lAlkaline phosphatase 80 iu/lTotal protein 36 g/lAlbumin 13 g/lGlucose 4 mmol/l

Bilirubin 15 μmol/lAST 90 iu/lAlkaline phosphatase 89 iu/lAlbumin 35 g/l

1. What was the original cause of oedema?2. What was the subsequent cause of abdominal pain

and distension?3. List two tests which will help confirm the cause of

abdominal pain.4. In one step, what is the management?

What is the rhythm disturbance in this rhythm strip(138)?

Question 137

Question 138

Page 141: Rapid review of clinical medicine for mrcp part 2

140

The combination of hypoalbuminaemia, heavyproteinuria and oedema is strongly suggestive of thediagnosis of nephrotic syndrome, which in youngchildren is most often due to minimal changeglomerulonephritis. Minimal change glomerulonephritisis thought to be an auto-immune disorder, and usuallyresponds extremely well to steroids and otherimmunosuppressants such as cyclophosphamide andchlorambucil. There is a well-recognized association ofthe disorder with Hodgkin’s lymphoma, bee stings, cow’smilk allergy and ingestion of NSAIDs. Minimal changenephropathy is found in 80% of children with nephroticsyndrome. Treatment is with high-dose steroids, anddiuretics are often necessary to treat the oedema.Relapses are common and are treated with steroids untilthe urine is completely free of protein for three days.Steroids are prescribed for a maximum of four weeks.Recurrent relapses are managed with cyclophosphamide

or chlorambucil. Complications of nephrotic syndromeinclude susceptibility to infection due to loss ofimmunoglobulins in the urine, protein malnutrition,hyperlipidaemia, arterial and venous thromboses andacute renal failure.

Thrombosis is a frequent and serious complication;corticosteroids and diuretic therapy contribute tothrombosis. Whole blood viscosity is increased, and thereis alteration in the concentration of clotting factors in theblood. The levels of antithrombin III and plasminogenare reduced, increasing the tendency to thrombosis. Inthis case the onset of abdominal pain, tender right upperquadrant, ascites and abnormal LFT reflect hepatic veinor portal vein thrombosis. Patients may present rapidlywith fulminant hepatic failure or with gradual-onsetascites and jaundice. Early revascularization with athrombolytic agent is important to prevent hepaticinfarction and fibrosis leading to chronic portalhypertension. In patients where thrombolysis iscontraindicated, a shunt procedure is necessary.

The diagnosis can be confirmed by ultrasonographyand Doppler. The hepatic veins, portal vein and inferiorvena cava should be examined by venography. Liverbiopsy helps make the diagnosis, and is also important toassess hepatic necrosis and fibrosis.

There is an irregular narrow complex tachycardia, and thedifferential diagnosis is between atrial fibrillation, atrialflutter with varying atrioventricular block, and an atrialtachycardia with varying atrioventricular block. The clueis in the rhythm strip which reveals a ‘saw-tooth’ patternto the ECG complexes, indicating flutter. The RR-interval is not constant, therefore there is flutter withvarying block. In this case there are between two andthree flutter waves between each RR-interval, and hencethe diagnosis is atrial flutter with 2:1/3:1 atrioventricularblock. The causes of atrial flutter are the same as thosefor AF. The most effective treatment for atrial flutter isDC cardioversion, delivering small energy shocks ofbetween 50 and 100 J. Chemical cardioversion isdifficult, but the most effective drugs for this purpose areflecainide and amiodarone. Like AF, patients with atrialflutter should be anticoagulated to prevent the risk ofthromboembolism.

Causes of atrial flutter/fibrillation

• Ischaemic heart disease• Hypertension• Mitral stenosis• Thyrotoxicosis• Myocarditis• Cardiomyopathy• Pericarditis• Bronchial carcinoma• Severe pneumonia• Chronic lung disease• Pulmonary emboli• Wolff–Parkinson–White syndrome• Alcohol abuse• Cocaine abuse

1. Nephrotic syndrome.2. Hepatic vein thrombosis.3. i. Ultrasound or CT scan of the liver looking

particularly at the hepatic and portal veins.ii. Liver biopsy.

4. Thrombolysis to prevent severe hepatic congestion.

Atrial flutter with 2:1/3:1 atrioventricular block.

Answer 137

Answer 138

Page 142: Rapid review of clinical medicine for mrcp part 2

Clinical Cases 141

A 66-year-old retired school teacher presented with a six-month history of progressive weakness, fatigue andbreathlessness which was accompanied by weight loss of5 kg. He had difficulty climbing stairs because his ‘legswould not carry him’, and had noticed that he haddifficulty holding light objects with his left hand withoutdropping them. Over the past week his wife had noticedthat his speech appeared slurred and nasal, and he washaving difficulty swallowing his meals. He had a pasthistory of pernicious anaemia for which he was takingregular intramuscular B12 injections. On examination, hewas thin. There was no evidence of pallor or clubbing.The patient had dysarthria. Examination of the cranialnerves revealed normal eye movements, but tonguemovement was sluggish and there was reduced palatalmovement. The jaw jerk was brisk. On examination of hislimbs there was wasting and fasciculation of the smallmuscles of the left hand. The tone and power in the leftupper limb was generally reduced. The upper-limbreflexes were brisk. There was wasting and fasciculation inboth thigh muscles. The tone was increased and thepower was reduced. The ankle and knee jerks were brisk,and there was obvious clonus at the ankle joint. Sensationwas normal and Romberg’s test was negative. Examina -tion of the fundi revealed bitemporal pallor.

Investigations are shown.

Hb 12 g/dlWCC 5 � 109/lPlatelets 180 � 109/lMCV 86 flSodium 137 mmol/lPotassium 4.1 mmol/lUrea 6 mmol/lCreatinine 110 μmol/lChest X-ray Normal

An 86-year-old male was found collapsed in his home byhis neighbour after his son raised concern about notbeing able to contact him by telephone. There was nosignificant past medical history. He was independent. Hewas last seen two days earlier when he joined hisneighbour for Christmas dinner.

On examination, he was unconscious and had aGlasgow coma scale of 5 out of 15. There was no nuchalrigidity. Pupillary reflexes were sluggish, but examinationof the fundi was normal. Tone was slightly increased inall the limbs. The peripheral reflexes were present andboth plantar reflexes were flexor. The heart rate was 40 beats/min, and blood pressure was 80/40 mmHg.Heart sounds were normal, and the chest was clear.

An ECG was performed in the Accident and Emer -gency Department (140).

1. What is the diagnosis?a. Cervical myelopathy.b. Subacute combined degeneration of the spinal

cord.c. Multiple sclerosis.d. Motor neurone disease.e. Polymyositis.

2. What two investigations could be performed to confirm your suspicion?a. Anti-Hu antibodies.b. MRI scan brain.c. CSF analysis for oligoclonal bands.d. Visual evoked potentials.e. Nerve conduction studies.f. MRI scan of cervical spine.g. Serum creatinine kinase.h. Serum B12 concentration.i. Muscle biopsy.j. Electromyography.

1. List four abnormalities on the ECG.2. Which single investigation should the medical

officer have performed?3. In one sentence, what is the management of this

case?

Question 139

Question 140 114400

Page 143: Rapid review of clinical medicine for mrcp part 2

142

There is evidence of combined lower motor neurone andupper motor neurone lesions and a pseudobulbar palsy.There is no sensory abnormality, and ocular movements arenormal. These findings are characteristic of motor neuronedisease, which is characterized by a progressivedegeneration of the motor neurones in the cortex, spinalcord and motor nuclei of the cranial nerves. The breath -lessness is probably due to involvement of the respiratorymuscles. The bitemporal pallor is a ‘red herring’, and al -though it may have led some readers to diagnose multiplesclerosis, it is important to note that bitemporal pallor is anormal finding in some individuals. Moreover, wasting andfasciculation are not features of multiple sclerosis be cause itdoes not affect lower motor neurones. Although the patienthas a history of pernicious anaemia, his blood count doesnot suggest B12 deficiency and therefore subacutedegeneration of the spinal cord is unlikely. The absence of asensory neuropathy is also against the diagnosis.

The cause of motor neurone disease is unknown. Inapproximately 15% of cases the disease is familial and is

due to mutations in the copper/zinc superoxide dis -mutase gene on chromosome 21. The incidence is1/50,000, and the male-to-female ratio is 1.5:1. Thereare three distinct patterns known as progressive muscularatrophy, amyotrophic lateral sclerosis and progressivebulbar palsy which are characterized by predominantlylower motor lesions, predominantly upper motor neuronelesions, and bulbar and pseudobulbar palsy, res pectively.Usually, a combination of all three is present. Female sex,bulbar onset and old age are poor prognostic markers.The disease usually progresses inexorably and death isusually from bronchopneumonia within five years of onsetof symptoms.

The diagnosis is clinical, but electromyography charac -teristically reveals a reduced number of action potentialsin the muscles which have an increased amplitude andduration. Nerve conduction studies reveal normal motorconduction and exclude an underlying neuropathy,particularly in patients with the progressive muscularatrophy variety of the disorder.

There are no curative therapies, but glutamate antagon -ists such as riluzole appear promising, as does ciliaryneurotrophic factor, which has been shown to promotesurvival of cultured rat and human motor neurones. Bothagents are currently being tested on affected humans.

Hypothermia is common among the elderly. It is usuallyattributable to cold environment (no heating at home),inadequate clothing, poor nutrition, neuroleptic drugs,alcohol and hypothyroidism. Hypothermia is defined asa fall in the core temperature to below 35°C (95°F).Severe hypothermia causes impaired consciousness andcardiac embarrassment. Bradycardia, hypotension,

hypopnoea, sluggish pupillary and peripheral reflexes,muscle rigidity and coma are recognized features ofsevere hypothermia. Ventricular fibrillation and asystoleare the usual causes of death.

ECG abnormalities include bradycardia, atrial fibril -lation with a slow ventricular rate, prolonged PR- andQT-intervals and J-waves. A baseline artefact due tomuscle tremor may also be seen, and should help makethe diagnosis in the examination situation.

Gentle rewarming is indicated. If the temperature isabove 32°C (89.6°F), space blankets and warm oral fluidsare usually sufficient. If the temperature is lower thanthis, and the patient is unconscious, then warmed intra -venous fluids should be given. In this situation there is adanger of severe metabolic acidosis due to sluggishcirculation, which should be corrected promptly. The aimis to increase the temperature by 1°C (1.8°F) per hour.

1. d. Motor neurone disease.2. e. Nerve conduction studies.

j. Electromyography.

1. i. Bradycardia.ii. Tremor artefact.iii. J-waves.iv. Prolonged QT-interval.

2. Rectal (core) temperature with a low-reading thermometer.

3. Gradual rewarming using space blankets and possibly warmed intravenous fluids, depending on the severity of hypothermia.

Answer 139

Answer 140

Page 144: Rapid review of clinical medicine for mrcp part 2

Clinical Cases 143

Question 141

A 21-year-old chef presented with a two-week history ofgeneral malaise, cough and headaches. He had just returnedfrom a ten-day holiday in a hotel resort in Spain. Heconsulted his GP on arrival, who diagnosed a respiratorytract infection and prescribed erythromycin. Despite this,the patient’s symptoms continued to persist. He had nowdeveloped a foul-smelling, blood-stained nasal dischargeand was deaf in his left ear. His cough was worse, but wasnon-productive. He felt ‘hot and cold’ and had a reducedappetite due to nausea. He was a non-smoker andconsumed 20 units of alcohol per week.

On examination, he appeared relatively well. There wasno skin rash, pallor, cyanosis, clubbing, lymphadenopathyor ankle oedema. He was tender over his frontal sinuses,and had crusting around the nasal septum on the right side.Inspection of the outer ears and tympanic membranes wasnormal. The Rinne’s test revealed reduced air and boneconduction on the left side. Weber’s test lateralized to theright side. Apart from the abnormal hearing tests, all otherneurological examination was normal. The heart rate was96 beats/min and blood pressure 140/90 mmHg. Thepatient was tachypnoeic on mild exertion, and had arespiratory rate of 20/min. Chest expansion was moderate.The trachea was central. On auscultation of the lung fieldsthere was evidence of bronchial breathing at the left lungbase and a few inspiratory crackles at the right lung base.Examination of the heart and abdomen was normal.

Investigations are shown.

Hb 11 g/dlWCC 14 � 109/l (neutrophil leucocytosis)Platelets 480 � 109/lESR 110 mm/hCRP 209 g/lSodium 134 mmol/lPotassium 4.1 mmol/lUrea 6 mmol/lCreatinine 89 μmol/lUrinalysis Protein +1

Blood not detectedChest X-ray (141a)

114411aa

1. Describe the abnormalities on the chest X-ray.

2. How would you interpret the findings from the Rinne’s and Weber’s tests?

3. What two tests could you perform to help achieve the diagnosis?

4. What is the underlying diagnosis?

Page 145: Rapid review of clinical medicine for mrcp part 2

144

Wegener’s granulomatosis is a necrotizing granulomatousvasculitis affecting the upper and lower respiratory tractsand the kidneys, although several other organs can beaffected. Granulomas occur in small arterioles. Thediagnosis should be suspected when two of these systemsare involved in the presence of systemic features such asmalaise, night sweats, fever and weight loss. This patienthas features of upper and lower respiratory tractinvolvement which include frontal sinusitis, purulent nasaldischarge and nasal septal crusting, dry cough and evidenceof gross respiratory involvement on the chest X-ray. Thedeafness in the left ear is indicative of 8th nerve

involvement. The kidneys do not appear to be significantlyaffected. The trace of protein may be a response to a febrileillness. Upper respiratory involvement can also lead tohoarseness and stridor due to vocal cord and trachealinvolvement, respectively, and lower respiratoryinvolvement may present with pleurisy and haemoptysis.Renal involvement can vary from asymptomatic proteinuriaand haematuria to fulminant renal failure due to crescenticglomerulonephritis. Other organs affected include the eye,cardiovascular system, central nervous system and the skin.In the eye, the disease can present with scleritis, uveitis andpseudotumour due to retro-orbital granuloma formation.Cardiovascular involvement is rare. The most commonmanifestation is pericarditis, but conduction tissueinvolvement produces tachy- and bradyarrhythmias.Involvement of the central nervous system is also rare andcomprises a mononeuritis multiplex, isolated cranial nervepalsies and meningeal involvement.

The diagnosis is based on demonstrating a necrotizinggranulomatous vasculitis biopsy of the affected tissue.The best yield of a positive biopsy comes from the kidneyor the lungs when they are involved. ANCA are presentin over 80% of untreated cases. Immunofluorescencestudies have revealed two main types of ANCA which aredetermined by the pattern of cytoplasmic staining. c-ANCA stains the outer cytoplasm, whereas p-ANCAstains the perinuclear cytoplasm. It is now clear that c-ANCA binds the enzyme proteinase 3 (PF3) and isspecific for Wegener’s granulomatosis. p-ANCA isheterogeneous. One subset binds the cytoplasmic enzymeMPO, which is positive in microscopic polyangiitis andidiopathic crescentic glomerulonephritis. Other subsets ofp-ANCA bind elastase, lactoferrin, lysozyme andcathepsin G and are present in a variety of auto-immuneconditions including CAH, sclerosing cholangitis,ulcerative colitis and SLE (141b).

The management of Wegener’s granulomatosis is withsteroids and other immunosuppressants such ascyclophosphamide. Disease progress is monitored byestimating ANCA titres. There is good correlationbetween ANCA titres and underlying inflammatoryactivity in most patients.

p-ANCA

Anti-MPO subset Non-specific subset

ANCA114411bb

c-ANCA(PF3)

Wegener’s granulomatosisMicroscopic polyarteritis

nodosaIdiopathic crescenticglomerulonephritis

A variety of autoimmune diseases:SLE

Mixed connective tissue diseaseUlcerative colitis

RTACFA

1. The chest X-ray demonstrates alveolar shadowing affecting the right middle and lower lobes and in the left lower lobe. In addition, there is a round cavitating lesion in the left lower lobe. The findings are consistent with either a purulent pneumonia with left lower lobe abscess formation, orpulmonary haemorrhage and a cavitating granuloma due to Wegener’s granulomatosis.

2. The Rinne’s test reveals reduced bone and air hearing conduction, suggesting sensorineuronal deafness on the left side. The Weber’s test does not normally lateralize to either ear, but in this case it lateralized to the right. This is in keeping with sensorineuronal hearing loss in the left ear. Because both air and bone conduction are impaired in sensorineuronal deafness, the sound from the tuning fork (which is conducted via bone in the Weber’s test) is heard loudest in the ear which is not affected. In the case of pure conduction deafness, the Weber’s test lateralizes to the affected ear. Wegener’s granulomatosis can produce conduction and sensorineuronal deafness by blockage of the Eustachian tube when it involves the upper respiratory tract and 8th cranial nerve, respectively.

3. i. Nasal or transbronchial lung biopsy.ii. c-ANCA (antiproteinase 3).

4. Wegener’s granulomatosis.

Answer 141

Page 146: Rapid review of clinical medicine for mrcp part 2

Clinical Cases 145

The following are serial LFT on a 51-year-old marriedschoolteacher who had a partial gastrectomy for aperforated peptic ulcer ten years previously. He wasasymptomatic. He was currently taking ranitidine.

He consumed approximately 4 units of alcohol per week.Apart from very slight jaundice, the physical examinationwas normal.

The following are cardiac catheter data on a 4-year-old male with cyanosis and failure to thrive:

Chamber Pressure Oxygen saturation Oxygen saturation (%)(mmHg) (%) (on air) (on 28% oxygen)

Right atrium 8 52 52Right ventricle 90/30 53 54Pulmonary artery 22/12 53 54PCWP 4Left ventricle 80/20 82 81Aorta 77/50 66 66

1. What is the most likely cause for the abnormality of the LFT?

2. Which two investigations would you perform?3. What treatment should be considered?

1987 1988 1990 1992 1993Alanine aminotransferase (normal 5–30 iu/l) 30 36 40 44 49Aspartate aminotransferase (normal 10–40 iu/l) 42 65 69 73 78Alkaline phosphatase (normal 25–115 iu/l) 100 112 118 125 135Bilirubin (normal <17 μmol/l) 17 20 24 29 36

1. List four abnormalities.2. What is the underlying

diagnosis?3. Explain the lack of rise

in oxygen tension after inspiring 28% oxygen.

A 28-year-old male presented with a three-day history ofrigors and pleuritic pain affecting the left lung, wherethere were signs of consolidation. There was no evidenceof lymphadenopathy or hepatosplenomegaly.

Investigations are shown.

Hb 12 g/dlWCC 24 � 109/lPlatelets 149 � 109/lNeutrophils 85%Normoblasts 7%Myeloblasts 3%Myelocytes 2%Lymphocytes 3%

1. What is the characteristic term given to this blood picture?

2. Which two features on the blood film would confirm your suspicions?

Question 142

Question 143

Question 144

Page 147: Rapid review of clinical medicine for mrcp part 2

146

The patient has had major surgery in the past that almostcertainly required blood transfusion. Both hepatitis B and Care readily contracted from blood transfusions. Sincescreening donated blood for hepatitis B was initiated in the1980s, hepatitis C has become the most common cause ofpost-transfusion hepatitis in the Western world.

Almost 20% of patients with hepatitis B infection willdevelop CAH. The cause is unknown, but is thought to bea defective T-cell immune response. Most patients af fectedare male. The condition is usually asymptomatic, or maypresent as very slowly progressive hepatitis. About 50% ofthose affected present with established chronic liver disease.

CAH may also occur following hepatitis C infection.The acute hepatitic event is usually very mild, and oftengoes unnoticed. The sequelae are similar to those of hepa -titis B infection; however the incidence of CAH is almost50%. Other causes of CAH are given (Table). There isnothing in the patient’s history to suggest any of theseother causes.

The aminotransferases and bilirubin are usually modestlyraised and the alkaline phosphatase is very slightly raised.The blood transaminase level bears no correlation with the

severity of the hepatic damage. In hepatitis B-related CAH,the hepatitis B surface antigen and the e antigen is present.In hepatitis C-related CAH, antihepatitis C antibodies arepresent. The histological landmark of CAH is piecemealnecrosis. There is destruc tion of the liver cells between theinterface of the hepatic parenchyma and the connectivetissue. In hepatitis B-related CAH, the presence of hepatitisB surface antigen in the hepatocytes produces a ground-glass appearance on haema toxylin and eosin staining.

Patients with evidence of CAH should be treated withintravenous interferon for several months to prevent viralreplication and allow seroconversion. The success rate isbelow 30%, and relapse often occurs soon after cessation oftherapy.

Other causes of CAH

Autoimmune• Lupoid hepatitis• Anti-LKM antibody-associated CAH

Drugs• Methyl dopa• Isoniazid• α-1 antitrypsin deficiency• Wilson’s disease• Inflammatory bowel disease (usually ulcerative

colitis)

(See Interpretation of Cardiac Catheter Data, page 418.)

1. Chronic active hepatitis due to hepatitis B or hepatitis C.

2. i. Hepatitis B and C serology.ii. Liver biopsy.

3. Interferon therapy.

1. i. Elevated right ventricular pressure.ii. Pressure drop across the pulmonary valve,

suggesting pulmonary stenosis.iii. Low oxygen saturation in the left ventricle,

suggestive of a VSD with a right-to-left shunt.iv. Saturation in the ascending aorta much lower

than the left ventricle, indicating an overriding aorta.

2. Fallot’s tetralogy.3. The right-to-left shunt means that the majority of

the blood does not pass through the lungs to be oxygenated.

A leukaemoid reaction is a descriptive term given to aleucoerythroblastic blood picture which follows sepsis. Insome cases of overwhelming infection, the ‘sick’ marrowreleases immature cells into the blood stream; thus, thereis an abundance of myelocytes and normoblasts. In con -trast to a true leucoerythroblastic anaemia where the mar -row is characteristically replaced by neoplastic cells, thereis no lymphadenopathy or hepatosplenomegaly.

1. ‘Leukaemoid reaction’.2. i. The presence of toxic granulations or Dohle

bodies in the white cells.ii. Neutrophils exhibiting a toxic shift to the left,

i.e. three or fewer segments of the nucleus.

Answer 142

Answer 143

Answer 144

Page 148: Rapid review of clinical medicine for mrcp part 2

Clinical Cases 147

A 30-year-old male presented with vertigo and deafnessin the left ear. His trunk was examined (145a), and hehad an audiogram as part of the investigation for hissymptoms (145b). There was no air–bone gap.

A 17-year-old male was referred with poorly developedsecondary sexual characteristics. He was a slow developer,and had been to a special school for his entire education.He had a brother who was severely handicapped andwheelchair-bound who had died at the age of 17 years.

On examination, there was absence of axillary and pubichair. His penis and testicles were small. There was bilateralgynaecomastia. Neurological examination was normal.

Investigations are shown.

Hb 12 g/dlWCC 5 � 109/lPlatelets 190 � 109/lSodium 137 mmol/lPotassium 3.7 mmol/lUrea 5 mmol/lCreatinine kinase 350 iu/lTestosterone 4 nmol/lLH 35 iu/l (normal range 1–10 iu/l)FSH 30 iu/l (normal range 1–7 iu/l)

1. What is the audiographic abnormality?2. What is the unifying diagnosis?

3. What other abnormalities would you expect to find on neurological examination?

1. What is the endocrine abnormality?2. What is the most probable diagnosis?3. How would you confirm the diagnosis?4. How would you explain the abnormal creatinine

kinase level?

114455bb

125 250 500 1000 2000 4000 8000 Frequency (Hz)

125 250 500 1000 2000 4000 8000 Frequency (Hz)

-10

0

10

20

30

40

50

60

70

80

90

100

110

120

130

140

Hea

ring

leve

l (dB

)

-10

0

10

20

30

40

50

60

70

80

90

100

110

120

130

140

Hea

ring

leve

l (dB

)

Right

Left

Right

Left

Unmasked

AC

BC

Question 145

Question 146

114455aa

Page 149: Rapid review of clinical medicine for mrcp part 2

148

Neurofibromatosis is inherited as an autosomal dominantcondition. Abnormalities on two chromosomes maycause disease with either a predominantly peripheral orcentral manifestation (Table B). Regular audiometry is

mandatory to detect acoustic neuroma early. MRI of thebrain revealed an acoustic neuroma (145c).

(See Audiograms, page 416.)

The most common cause of primary hypogonadism isKleinfelter’s syndrome, which has an incidence of1/1,000 births. Patients are tall, lack secondary sexualcharacteristics, have gynaecomastia, and are often men -tally subnormal. The condition is characterized by failureof development of Leydig cells and seminiferous tubules.Other causes of primary hypogonadism include Prader–Willi syndrome, Lawrence–Moon–Biedel syndrome,

Leydig cell agenesis (Castelli’s syndrome), 5α-reductasedeficiency, mumps orchitis, renal failure, sickle celldisease, alcohol excess, chemotherapy and radiotherapy(Table, Answer 260).

Patients have XXY karyotype due to non-dysjunctionand have the potential of being carriers for X-linkedrecessive conditions (as with females) such as Duchennemuscular dystrophy, glucose-6-phosphate dehydrogenasedeficiency and haemophilia A. Kleinfelter patients areinfertile. They are capable of having erections andejaculation, but the semen does not contain spermatozoa.

Kleinfelter’s syndrome is associated with a higherincidence of carcinoma of the breast, hypothyroidism,diabetes mellitus and chronic respiratory disease than thegeneral male population.

Table B Other associations of neurofibromatosis

Chromosome 17 (peripheral abnormalities)• Renal artery stenosis• Pulmonary fibrosis• Cardiomyopathy (hypertrophic, restrictive and

dilated)• Fibrous dysplasia of bone• Phaeochromocytoma

Chromosome 22 (central abnormalities)• Acoustic neuroma• Meningioma• Optic glioma• Ependymoma

1. There is severe hearing loss at progressively higher frequencies in the left ear.

2. The trunk demonstrates café au lait spots, a feature of neurofibromatosis. An important association of neurofibromatosis is acoustic neuroma, which is a recognized cause of sensorineuronal deafness (Table A).

3. On neurological examination, the patient may also have an absent ipsilateral corneal reflex and ipsilateral cerebellar signs. In a minority of cases acoustic neuromas may be bilateral.

1. Primary hypogonadism (hypergonadotrophic hypogonadism).

2. Kleinfelter’s syndrome.3. Demonstration of Barr body on buccal smear or

karyotyping.4. Carrier for Duchenne muscular dystrophy.

Table A Causes of sensorineuronal deafness

Unilateral (lesion in the cerebellopontine angle)• Meningioma• Acoustic neuroma• Granuloma• Metastasis

Bilateral• Degenerative (presbycusis)• Aminoglycosides• Amphotericin• High-dose loop diuretics• Mumps• Rubella

Answer 145

Answer 146

114455cc

Page 150: Rapid review of clinical medicine for mrcp part 2

Clinical Cases 149

A 17-year-old female was admitted with a seven-hourhistory of severe central abdominal pain radiating to herback, which was accompanied by bilious vomiting. Shehad been constipated for 72 hours. There was nosignificant past medical history; in particular there was nohistory of abdominal surgery. She was a non-smoker andhad a very occasional glass of wine. She was an only child.Her father died when she was aged two. She was notcertain about the cause of death, but mentioned that hehad a history of psychosis and epilepsy. Her mother hadgallstones, but was otherwise well. She took paracetamolfor infrequent ‘period pains’. The only other drug historywas that of the contraceptive pill, which had beencommenced 10 days previously.

On examination she was unwell and distressed with pain.She was sweating profusely. Her heart rate was130 beats/min and blood pressure 190/110 mmHg.Examination of the precordium revealed normal heartsounds. Peripheral pulses were easily palpable. The chestwas clear. The abdomen was generally very tender, withguarding around the umbilical area. The hernial orifice wasnormal. There was a positive succussion splash and thebowel sounds were present. Rectal examination revealed anempty rectum. Neurological examination was normal.

The patient was prescribed pethidine for her pain andmetoclopramide for the vomiting. She receivedintravenous saline. A few hours later she complained ofweakness affecting her left shoulder. She was unable toabduct or adduct the shoulder. Shortly afterwards shedeveloped weakness in both her legs, in addition tourinary retention. She was due to be catheterized whenshe had a grand mal epileptic seizure.

Investigations are shown.

Hb 13 g/dlWCC 19 � 109/lPlatelets 400 � 109/lESR 12 mm/hSodium 121 mmol/lPotassium 3 mmol/lUrea 5 mmol/lCalcium 2.5 mmol/lAlbumin 44 g/lBilirubin 15 μmol/lAST 44 iu/lBlood glucose 4 mmol/lAbdominal X-ray NormalChest X-ray NormalCT scan brain NormalLumbar puncture 1 lymphocyte/mm3

No red cellsProtein 0.3 g/lGlucose 3.3 mmol/l

1. What is the diagnosis?2. Which two investigations would you perform to

confirm the diagnosis?3. What is the precipitating factor?4. List four management steps which you would take

in this patient.5. How will you manage her epileptic seizures?

A 68-year-old male presented with a 48-hour history ofright upper quadrant pain, dark urine and pale stools.Shortly after admission he had an investigation (148).

1. What investigation has been performed?2. What does it reveal?3. What is the diagnosis?

Question 147

Question 148 114488

Page 151: Rapid review of clinical medicine for mrcp part 2

Acute intermittent porphyria is the most common type ofacute porphyria. It is a rare condition caused by a geneticmutation in the gene encoding the enzyme PBG deaminaseon chromosome 11, and is inherited as an autosomaldominant trait. A family history is usually present, but in upto one-third of cases this may be absent, either because thecondition has not been diagnosed in first-degree relatives,or because the condition is running a latent course. It is fivetimes more common in females than males, and usuallypresents between the ages of 14 and 30 years. The absenceof PBG deaminase leads to elevated δ-ALA and PBG in theblood stream (147), both of which are responsible for theclinical manifestations of the condition. AIP is characterizedmainly by gastrointestinal and neurological symptoms.Unlike the chronic porphyrias, skin photosensitivity isextremely unusual in AIP. Attacks are usually precipitatedby drugs, alcohol, fasting and sepsis. Several drugs, particu -larly barbi turates and other anti convulsants, have beenimplicated. The oral contraceptive pill has also beenassociated with acute attacks of porphyria and is the mostlikely precipita ting factor in this case. The most commonsymptoms are acute abdominal pain which can simulate asurgical emergency, nausea, vomiting and constipation.Abdominal guarding may be present, but bowel sounds areusually present. There is often evidence of delayed gastricemptying, and it is not unusual to elicit a succussion splashdue to a stomach full of gastric contents. The abdominal X-ray is normal. Neurological manifesta tions of the diseaseinclude a peripheral and autonomic neuropathy. Auto nomicneuropathy most commonly mani fests as tachycardia whichis out of proportion to the extent of abdominal pain, hyper -tension and excess sweating. In approximately 50% of casesthere is evidence of a motor neuropathy which often affectsthe proxi mal muscles. Both bulbar palsy and quadriplegiaare recognized mani festations. Respiratory muscle paralysis

is life-threatening, and requires assisted ventilation. In thiscase there was involve ment of the left shoulder girdle andthen a paraparesis. Epilepsy occurs when the disease is in itsmost aggressive phase. It often heralds the onset of hypo -natraemia due to SIADH.

The management is by withdrawing the offending drug,if this is applicable. A high calorie intake reduces precursorsof haem synthesis in the blood by reducing the activity ofthe rate-limiting enzyme ALA synthetase. Intravenoushaematin is used in the acute situation and also acts byinhibiting the action of ALA synthetase. The control ofabdominal pain requires narcotic analgesia. Vomiting can beprevented by insertion of a nasogastric tube to empty thestomach contents.

Metoclopramide is another drug which can precipitateacute porphyria, and should be avoided. Beta-blockers areused to manage the tachycardia and hypertension.

The management of epilepsy is by trying to abate thecondition with a high calorie intake and by fluid restrictionto prevent further hyponatraemia from SIADH. Epilepsy inthe acute situation is managed with chlormethiazole, whichis one of the very few anticonvulsant agents not known tocause acute porphyria. Prevention of the condition is mainlybased on avoiding precipitating drugs and recognizingwarning symptoms early.

The diagnosis is made by demonstrating high levels of δ-ALA and PBG in the urine. The addition of Ehrlich’s dyeto the urine will cause a red discoloration in the presence ofPBG; however, this phenomenon is also observed in the

Succinyl CoA + Glycine

ALA

PBG

Uroporphyrinogen

Coproporphyrin

Protoporphyrin

Haem

150

Answer 147

1. Acute intermittent porphyria (AIP). The combination of abdominal pain, neurological features, hypertension and tachycardia in a young person should raise the suspicion of AIP.

2. i. Urinary PBG.ii. Assay red cells for the enzyme PBG deaminase.

3. The oral contraceptive pill.4. i. Stop oral contraceptive pill.

ii. High calorie intake with intravenous infusion 500 ml of 50% dextrose per 24 hours via a large central vein in this case. The normal regime is 2 litres of 20% dextrose per 24 hours; however, in the case of syndrome of inappropriate ADH secretion, fluid restriction demands the use of smaller volumes of high-concentration dextrose.

iii. Start intravenous haematin.iv. Intravenous beta-blocker to control tachycardia

and hypertension.5. Intravenous diazepam or chlormethiazole.

Deficientin AIP

Deficientin PCT

*Inhibited by lead

Pathway for haem biosynthesis114477

ALA synthetase

ALA dehydratase*

PBG deaminase

Uroporphyrindecarboxylase

Coproporphyrin oxidase*

Ferrochelatase*

Metabolites in light typeEnzymes in bold type

Page 152: Rapid review of clinical medicine for mrcp part 2

Clinical Cases 151

presence of urobilinogen in the urine. Differentiation ofurobilinogen from PBG involves the addition of chloro -form, which will cause PBG to separate out in the aqueouslayer as a purple sediment, whereas the uro bilinogen issoluble and becomes colourless. This is the basis of theWatson–Schwartz test. The urinary levels of ALA and PBG

are always elevated during an attack, though they may benormal when the patient is well. The red cells can also beassayed for the enzyme PBG deaminase, which is very lowin AIP. The enzyme ALA dehydratase is elevated. (SeeAnswer 400).

1. The investigation is a percutaneous transhepatic chol angiogram.2. Both the common bile duct and the internal hepatic ducts are dilated. There is a large filling defect at the distal

end of the common bile duct.3. A gall stone in the common bile duct causing obstructive jaundice.

Answer 148

A 25-year-old female presented to her GP after noticingasymmetry of her face (149a). For the past two days she hadalso experienced dizziness and lethargy, particularly onexertion. She had no past medical history. Six weekspreviously she had been to visit friends in North America.While on holiday, she developed a painful rash on her leftarm which she described as a small red nodule whichgradually expanded over the next few days before subsiding.A friend had taken a photo graph of the rash (149b).

On examination, she had a temperature of 37°C (98.6°F)and a slow pulse. Examination of the ears was normal. Thedoctor was concerned and performed an ECG (149c).

114499aa 114499bb

114499cc

1. What is the cause of the abnormality on her face?2. What is the characteristic name given to the rash?3. Give two other dermatological manifestations of

this disorder.

4. What is the ECG diagnosis?5. What is the overall diagnosis?6. List two therapeutic steps in the management of

this patient.

Question 149

Page 153: Rapid review of clinical medicine for mrcp part 2

152

The patient presented with a facial palsy and has ECGevidence of third-degree atrioventricular block. In addi -tion, she had a macular rash which appears to be fadingin the centre (149b) and is characteristic of erythemachronicum migrans. While there are several medicalconditions which may involve the cardiovascular andcentral nervous system simultaneously (Table), the rashmakes Lyme disease the most likely diagnosis.

Lyme disease is caused by the spirochaete, Borreliaburgdorferi. It is transmitted by hard-bodied ticks (Ixodesspecies). The disease has a wide clinical spectrum, rangingfrom asymptomatic infection to multi-systeminvolvement. The first presentation is a characteristic skinlesion at the site of the tick bite, which begins as a maculebetween 2–40 days after exposure, and expands withcentral clearing. This is termed erythema chronicummigrans and may be associated with fever and regional

lymphadenopathy. Early localized infection may presentas Borrelia lymphocytoma, which is a bluish-red noduleon the ear lobe or nose. The second stage of the diseaseoccurs several weeks or months after exposure, and is dueto spread of the organism via the blood or the lym -phatics. At this stage there may be fever and flu-likesymptoms, several skin lesions similar to erythemachronicum migrans and cardiac and neurological involve -ment. Cardiac disease occurs in 5% of affected individualsand is characterized by varying degrees of heart block andmyopericarditis. It is usually brief and seldom lasts morethan one week. Temporary pacing is only indicated inpatients who are symptomatic from heart block, as in thiscase. Neurological disease com prises a mildmeningoencephalitis, peripheral neuritis, cranial nerveinvolvement, particularly the facial nerve andBannworth’s syndrome (lymphocytic meningo radiculitis).

Chronic Lyme disease may manifest years later as anerosive oligoarthritis or as acrodermatitis chronica atro -phicans, which is characterized by atrophy of the skin onthe lower limbs.

The diagnosis is clinical and confirmed by serologicaltesting. Serology may be negative during the early ery -thema chronicum migrans stage. All stages of the diseaseare amenable to therapy with antibiotics, and the clinicalresponse is usually good. The antibiotics used withsuccess include doxycycline, benzyl penicillin, cefotaxime,amoxycillin and azithromycin.

Conditions simultaneously affecting the cardiovascular and central nervous system

Infections• TB• HIV• Syphilis• Lyme disease• Rheumatic fever• Infective endocarditis

Multi-system conditions• Connective tissue diseases• Vasculitides• Sarcoidosis

Toxins• Alcohol

Drugs• Amphetamines• Tricyclic antidepressant drugs• Lithium

Neuroectodermal syndromes• Neurofibromatosis• Tuberose sclerosis

Hereditary neuropathies• Friedreich’s ataxia• Dystrophia myotonica• Duchenne muscular dystrophy

Miscellaneous• AIP• Guillain–Barré syndrome

1. Left facial nerve palsy.2. Erythema chronicum migrans.3. i. Borrelia lymphocytoma.

ii. Acrodermatitis chronica atrophicans.4. Complete heart block. (Third-degree

atrioventricular block.)5. Lyme disease.6. i. Therapy with doxycycline, benzyl penicillin,

azithromycin, or cefotaxime.ii. Temporary cardiac pacing.

Answer 149

Page 154: Rapid review of clinical medicine for mrcp part 2

Clinical Cases 153

A 69-year-old male was admitted to the Coronary CareUnit after presenting with a three-hour history of chestpain, palpitations and breathlessness. He had a pasthistory of hypertension and three myocardial infarctions.His regular medication comprised aspirin 150 mg oncedaily, captopril 50 mg three times daily, furosemide

40 mg once daily, and simvastatin 10 mg once daily. AnECG per formed on admission is shown in 150a. (Do notanswer 4 until you have finished answering 1–3.)

115500aa

A deaf couple both have a hereditary form of deafness.They have recently married, and want advice on thechances of their children being affected. The family treeof both parents is shown (151).

115511A B

1. What is the ECG diagnosis?2. Give four reasons in favour of your diagnosis.3. What is the drug of choice?4. Shortly after administering the drug, the patient

becomes very clammy and cyanosed. His conscious level deteriorates and his blood pressure recording is 70/40 mmHg. What is the next step in his management?

1. What is the pattern of inheritance for family A?a. Autosomal dominant. b. Autosomal recessive.c. X-linked recessive.d. X-linked dominant.e. Autosomal dominant with incomplete

penetrance.

2. What is the inheritance for family B?a. Autosomal dominant. b. Autosomal recessive.c. X-linked recessive.d. X-linked dominant.e. Autosomal dominant with incomplete

penetrance.

3. Assuming that both families have the same mutation causing deafness, what is the chance of their children being affected?a. None.b. 1/4.c.1/2.d. 100%.e. 3/4.

Affected male Unaffected male*

Affected female Unaffected female*

* Does not exclude inheritance of an autosomalrecessive gene in either sex or an X-linked recessivedisease in a female

Question 150

Question 151

Page 155: Rapid review of clinical medicine for mrcp part 2

154

Factors favouring VT rather than supraventriculartachycardia include all of those in (150b) above (see alsoTable). The diagnosis of VT rather than SVT in a patientwith a broad complex tachycardia is also more likely whenthere is a history of ischaemic heart disease orcardiomyopathy.

Most VT has a RBBB morphology because it arises fromthe left ventricle; however, VT from the right ventricle isalso recognized, in which case it has a LBBB morphology.Examples of right ventricular VT include right ventricularoutflow tachycardia, congenital heart disease with right-to-left shunts, and arrhythmogenic right VT.

The management of sustained VT is dependent onhaemodynamic status. Patients who are haemodynamicallystable (not hypotensive or clammy) should be treated withamiodarone. Other effective drugs include sotalol,procainamide and lignocaine. Sotalol and procainamideare more effective than lignocaine. Amiodarone is the leastnegatively inotropic agent and is the drug of choice whenthere is suspicion that left ventricular function may beimpaired. Patients with haemodynamic collapse should becardioverted immediately.

In both families neither parent is affected, which excludesan autosomal dominant or an X-linked dominant conditionin any of the parents and an X-linked recessive condition inboth fathers (the presence of a defective X-linked gene in the fathers would manifest as abnormalphenotype). This leaves the possibility of an autosomalrecessive condition, or that the mothers are carriers of anX-linked condition. However, an X-linked recessive mode

of inheritance is excluded by the fact that there is anaffected daughter in one family. If a mother is a carrier foran X-linked recessive condition there is a 50% chance thather daughters will be carriers, but none will have thedisease, and there is a 50% chance that her sons will beaffected. If both parents are carriers of autosomal recessivegenes for deafness, then there is a 25% chance that theirchildren will be affected, irrespective of their sex.

Both the couples in question are affected, whichimplies that both carry two autosomal recessive genes fordeafness. The chance of their children being affected is100%, because they will inherit one autosomal recessivegene for deafness from each parent.

115500bb

Features of ventricular tachycardia

• Broad complex (QRS usually >140 ms)• Extreme axis deviation• Positive or negative concordance in the

precordial leads• RSr pattern in V1• Deep S-wave in V6• Fusion and capture beats• Dissociated P-waves

1. Ventricular tachycardia (VT).2. i. Broad complex tachycardia.

ii. Extreme axis deviation.iii. Concordance in the precordial leads.iv. Dissociated P waves (150b).

3. Intravenous amiodarone.4. DC cardioversion.

1. b. Autosomal recessive.2. b. Autosomal recessive.3. d. 100%.

Answer 150

Answer 151

Page 156: Rapid review of clinical medicine for mrcp part 2

Clinical Cases 155

A 67-year-old patient with diet-controlled diabetes wasfound collapsed at the bottom of the stairs in his homeby his daughter. The only other medical history availablewas that he had a history of hypertension, for which hewas taking bendroflumethiazide. On examination he wasdrowsy. His heart rate was 41 beats/min, and bloodpressure 140/90 mmHg. The temperature was 36.8°C(98.2°F). The JVP was not raised. The heart sounds werenormal, and his chest was clear. Abdominal examinationwas normal. His left lower limb was externally rotatedand painful to move, and there was extensive bruising ofthe buttock and left thigh.

Investigations are shown.

A 5-year-old male was admitted with a two-day history ofblood-stained diarrhoea. Three days after, he feltnauseous and generally unwell. On examination he had

periorbital oedema and a blood pressure of150/95 mmHg.

Investigations are shown.

115522Hb 8 g/dlWCC 13 � 109/lPlatelets 36 � 109/lPT 13 s (control 13 s)APTT 34 s (control 36 s)Blood film (152)Sodium 138 mmol/lPotassium 5.9 mmol/lCreatinine 130 μmol/lUrea 11 mmol/l

Hb 9.8 g/dlWCC 14 � 109/lPlatelets 350 � 109/lMCV 84 flClotting NormalSodium 135 mmol/lPotassium 6.8 mmol/lUrea 16 mmol/lCreatinine 670 μmmol/lBicarbonate 15 mmol/lCalcium 2.0 mmol/lPhosphate 2.8 mmol/lBilirubin 14 μmol/lAST 26 iu/lAlkaline phosphatase 100 iu/lAlbumin 40 g/lGlucose 9 mmol/lUrinalysis Blood +++

Protein +Chest X-ray NormalX-ray of left hip Fracture and dislocation of

the neck of femurECG Minor T-wave abnormalities

in the lateral leads

1. What is the diagnosis?a. Nephrotic syndrome secondary to minimal

change glomerulonephritis.b. IgA nephritis.c. Haemolytic uraemic syndrome.d. Thrombotic thrombocytopenic purpura.e. Chronic pyelonephritis.

2. What investigation would you request to confirm your suspicion?a. 24 hour urine collection.b. Intravenous pyelogram.c. Stool culture for Escherichia coli 0157:H7.d. Renal biopsy.e. Auto-antibody screen.

1. What is the diagnosis?2. List two investigations you would perform to

confirm the cause of the metabolic abnormality.

Question 152

Question 153

Page 157: Rapid review of clinical medicine for mrcp part 2

156

E. coli 0157:H7 is most commonly transmitted by eatingunder-cooked beef. The bacterium produces a toxin thatis responsible for the gastrointestinal effects, which includecolicky pain and diarrhoea, which may be blood-stained.Fever is very mild or completely absent. Symptoms ofinfection usually subside after about a week; however, theHUS develops in approximately 6% of patients. This ischaracterized by an MAHA, which is a result ofendothelial damage by the toxin, leading to adherence offibrin strands which trap and fragment red cells andplatelets. This produces a characteristic blood picture ofanaemia, red cell fragments, thrombocytopenia andincreased reticulocytes. Small vessel fibrin deposition andthrombosis resulting from MAHA lead to renal damageand oliguric renal failure. Bleeding and bruising arecommon. In pure MAHA clotting is usually normal;however, MAHA often coexists with disseminatedintravascular coagulation, where clotting is impaired andthe serum fibrinogen degradation product level iselevated. Treatment of HUS complicated by MAHA andrenal failure is supportive. Blood transfusions are often

necessary. Some 50% of patients require dialysis. Themortality rate is 3–5%.

A variant of HUS occurring in the same circumstancesand characterized by neurological involvement is knownas thrombotic thrombocytopenic purpura. This is causedby a circulating inhibitor for the Von Willebrand Factorprotease enzyme. This results in uncontrolled thrombotictendencies in small vessels.

The combination of MAHA and DIC occurs in sepsis,malignancy and obstetric emergencies such as septicabortion, abruptio placentae and amniotic fluid embolism.

The presence of blood in the urine is against thediagnosis of nephrotic syndrome.

There are two important clues for diagnosing rhabdo -myolysis. Firstly, the patient has had a fall. Secondly,there is renal failure with a disproportionately raisedcreatinine level. Other causes of rhabdomyolysis aretabulated below (Table A). The common pathology is

myositis, skeletal muscle trauma or infarction leading toraised myoglobin levels in the blood. Myoglobin is toxicto the renal tubules and precipitates renal failure. Inrhabdomyolysis, the serum calcium is low because freecalcium becomes bound by myoglobin. The serumpotassium and phosphate are high because large amountsof these electrolytes are released from damaged musclecells (Table B)

The management involves adequate hydration andalkalinization of the urine to reduce precipitation ofmyoglobin in the renal tubules. Loop diuretics should beavoided as they result in an acidic urinary pH.

Table A Causes of rhabdomyolysis

• Muscle trauma/infarction• Electrocution• Hypothermia• Status epilepticus• Neuroleptic malignant syndrome• Ecstasy/amphetamine abuse• Burns• Septicaemia• Therapy with statins• Very strenuous exercise (e.g. marathon running)

Table B Causes of low calcium and high phosphate

• Chronic renal failure• Hypoparathyroidism• Pseudohypoparathyroidism• Magnesium deficiency*• Rhabdomyolysis

* Due to end-organ resistance to the effects ofPTH

1.c. Haemolytic uraemic syndrome.2.c. Stool culture for Escherichia coli 0157:H7.

1. Rhabdomyolysis. 2. i. Measuring creatinine kinase levels, which are

elevated due to skeletal muscle damageii. Detection of myoglobin in the urine.

Causes of MAHA

• HUS• Thrombotic thrombocytopenic purpura• Accelerated hypertension• Septicaemia• Vasculitides• Connective tissue diseases• Mucinous adenocarcinoma• Burns• Prosthetic valve-induced haemolysis• Drugs (cyclosporin, mitomycin)

Answer 152

Answer 153

Page 158: Rapid review of clinical medicine for mrcp part 2

Clinical Cases 157

A 16-year-old male presented with a four-day history of adry cough, right-sided pleuritic chest pain and increasingbreathlessness. On examination, he had a temperature of38°C (100.4°F). He had a widespread rash affecting hisarms and legs and his face (154).

Investigations are shown.

115544

Hb 9 g/dlWCC 5 � 109/lPlatelets 150 � 109/lBlood film Red cell agglutinationSodium 131 mmol/lPotassium 4.0 mmol/lUrea 4.0 mmol/lCreatinine 70 μmol/lBilirubin 26 μmol/lAST 60 iu/lALT 83 iu/lAlkaline phosphatase 110 iu/lAlbumin 36 g/lChest X-ray Right middle lobe

consolidation

A 14-year-old female was admitted for investigation ofintermittent colicky abdominal pain associated withvomiting. She had experienced several episodes in thepast six months requiring hospital admissions. Physicalexamination during these episodes revealed a distendedabdomen which was tympanic to percussion. Bowelsounds were tinkling. Hernial orifice examination wasnormal. The patient usually developed two painful 4 cmraised lesions on her left hand and forearm respectivelywhich were only present while she was symptomatic. Shewas never in hospital for more than 48 hours, duringwhich time she was managed with intravenous fluids andresting the bowel. She always made a spontaneousrecovery. In between episodes of abdominal pain she wasperfectly well. She had a 5-year-old cousin who had alsorecently been admitted to hospital on three occasionswith the same symptoms.

These investigations were performed while the patientwas symptomatic.

The patient had a small bowel meal when her symp -toms had recovered, which was also normal.

Hb 12 g/dlWCC 6 � 109/lPlatelets 390 � 109/lU&E NormalLFT NormalThyroid function tests NormalBone biochemistry NormalBlood glucose 5 mmol/l Serum amylase NormalSerum lead UndetectableUrinary δ-ALA UndetectableUrinary PBG UndetectableSerum immunoglobulins Normal

1. What is the rash?2. What is the cause of the low sodium?3. What is the cause of his anaemia?4. Which single investigation would you perform to

confirm the cause of the anaemia?5. What is the cause of his chest X-ray findings?6. Which two tests would you perform to confirm the

diagnosis?

1. What is the cause of the abdominal pain?2. How would you manage her symptoms if she

remained symptomatic for several days, or if her symptoms deteriorated?

Question 154

Question 155

Page 159: Rapid review of clinical medicine for mrcp part 2

158

The patient has a right middle lobe pneumonia and a rashwhich is characterized by blistering target lesions (ery -thema multiforme). The patient has an anaemia, and theagglutination on the blood film points to an immunehaemolytic anaemia. Mycoplasma pneumonia is associatedwith both erythema multiforme and a cold autoimmunehaemolytic anaemia. Other causes of erythema mult iformeand complications of Mycoplasma pneumonia are listed inTables A and B. The low sodium is a con sequence ofSIADH complicating pneumonia. The causes of SIADHare discussed in Answer 129.

Table A Causes of erythema multiforme

Infections• Mycoplasma pneumonias• Herpes simplex virus• Orf• TB

Drugs• Penicillins• Sulphonamides• Barbiturates• NSAID

Miscellaneous• Connective tissue diseases• Vasculitides• Internal malignancy

Table B Complications of Mycoplasma pneumonia

Neurological Aseptic meningitisEncephalitisTransverse myelitisGuillain–Barré syndromePeripheral neuropathy

Cardiac MyocarditisPericarditis

Haematological ThrombocytopeniaCold autoimmune haemolytic

anaemia

Locomotor MyositisArthritis

Skin Erythema multiforme

Gastrointestinal Diarrhoea/vomiting

Endocrine SIADH

Miscellaneous Bullous myringitis

Hereditary angio-oedema is inherited as an autosomaldominant trait. It is associated with C1 esterase inhibitordeficiency, resulting in high C1 esterase levels. Deficiencyof C1 esterase inhibitor (which modulates complement)can lead to angio-oedema. A positive family history isvery common, although there is a similar conditionassociated with an acquired lymphoproliferative disorderin which C1 esterase inhibitor levels are also low. Clinicalfeatures can occur late in adult life. Attacks are preceded

with a painful macular rash, or even a rash simulatingerythema marginatum. Triggering factors include traumato the skin. Laryngeal oedema can be life-threatening.Abdominal pain occurs due to visceral oedema. Attackscan exceed 72 hours. Treatment is supportive, althoughfresh-frozen plasma can be given to abort severe or pro -longed attacks. Inhibitors of plasmin, such as danazol, areused for long-term treatment. Tranexamic acid is alsoeffective through increasing C1 esterase inhibitor levels.Hydrocortisone and chlorpheniramine are not as effectivein the management of this condition as they are in acuteanaphylaxis. The diagnosis is made by quantifying the C1esterase level (high) and C1 esterase inhibitor level (low).C2 and C4 levels are also low.

1. Erythema multiforme.2. Syndrome of inappropriate ADH secretion.3. Autoimmune haemolytic anaemia.4. Direct Coomb’s test at 4°C (39.2°F).5. Mycoplasma pneumonia.6. i. Mycoplasma serology.

ii. Cold agglutinin estimation.

1. Hereditary angio-oedema.2. Use fresh-frozen plasma or C1 esterase inhibitor

concentrates.

Answer 154

Answer 155

Page 160: Rapid review of clinical medicine for mrcp part 2

Clinical Cases 159

A 30-year-old male presented with a four-day history ofpassing dark urine. He had passed dark urine intermit tentlyfor six months, though such passage had never exceeded24 hours, except on this occasion. More recently, hecomplained of malaise, lethargy and weak ness. His GPthought he appeared pale, and prescribed iron tablets. Twodays before coming to the hospital, the patient developedsharp right lower chest pain which was worsened oninspiration, and a cough productive of bright red sputum.On further questioning, he volunteered a six-monthhistory of intermittent abdominal pain and episodes ofabdominal distension.

He had never taken any regular medication and hadnot travelled abroad for a year. He consumed 5–7 litresof lager on weekends, and occasionally had a glass ofwine with a meal.

On examination, he was pale and slightly jaundiced. Hehad a temperature of 37.5°C (99.5°F). There was nolymphadenopathy. The JVP was not raised. Examination ofthe respiratory system revealed dullness of the right baseassociated with reduction of air entry. Abdominal examina -tion demonstrated firm, palpable hepatomegaly 4 cm belowthe costal margin. There were no other palpable masses.There was no clinical evidence of peri pheral oedema.

Investigations are shown.

Hb 7.3 g/dlWCC 2.8 � 109/lPlatelets 200 � 109/lMCV 93 flReticulocyte count 15%Direct Coomb’s test NegativeChest radiograph Atelectasis at the right

lung baseUrinalysis Urobilinogen +++

Haemosiderin ++Cell 0

Sodium 136 mmol/lPotassium 4.8 mmol/lUrea 5.3 mmol/lBilirubin 33 μmol/lAST 40 iu/l Alkaline phosphatase 94 iu/l Albumin 36 g/l

An 18-year-old male developed sharp left-sided chestpain on running up a flight of stairs, followed by slightbreathlessness on exertion. On examination, he appearedrelatively well. His heart rate was 90 beats/min, and therespiratory rate was 18/min. On auscultation of the heartthere was a clicking sound synchronous with the heartsounds.

1.What is the cause of the dark urine?a. Bilirubinuria.b. Urobilinogenuria.c. Myoglobinuria.d. Haemoglobinuria.e. Haemosiderinuria.

2. What is the cause of jaundice?a. Haemolysis.b. Hepatitis.c. Gall stones.d. Hepatic metastases.e. Pancreatitis.

1. What is the most probable cause of his symptoms?a. Mitral valve prolapse.b. Pericarditis.c. Left-sided apical pneumothorax.d. Pulmonary embolism.e. Aortic dissection.

2. What investigation would you perform to confirm the diagnosis?a. Echocardiogram.b. ECG.c. Serum cardiac troponin measurement.d. CT pulmonary angiogram.e. Chest X-ray in end expiration.

Question 156

Question 157

Page 161: Rapid review of clinical medicine for mrcp part 2

160

Bilirubinuria does not occur in haemolytic anaemiabecause bilirubin is in the unconjugated form andtherefore is not water-soluble and cannot be excreted inthe urine. Urobilinogen does stain urine, but not enoughto give it the red colour of haemoglobinuria. Myoglobin isproduced in large amounts in rhabdomyolysis, and while itmay produce red discoloration of urine, there is noevidence for rhabdomyolysis in this case. Haemosiderinuriaoccurs in intravascular haemolyis, but can only be detectedon staining the urine with Prussian blue.

Hepatitis, gallstones and pancreatitis do not accountfor the severity of the anaemia. Although haemolyis andthrombotic episodes can be present in a patient withmalignancy, it does not account for the dark urine that ispresent in the absence of red cells in the urine.

The diagnosis is PNH. This is an acquired disorder ofthe haematopoietic cells which usually occurs after anepisode of aplastic anaemia, leading to increasedsusceptibility of red cells to complement lysis. White cellsand platelets are also affected. The onset of the disease isoften insidious. The disorder is characterized by non-immune haemolytic anaemia, variable pancytopenia andthrombotic episodes. The MCV may be normal,depending on the extent of reticulocytosis. Themolecular basis of the membrane defect has recently beenunfolded and attributed to abnormalities in the synthesisof the phospholipid GPI, which serves an anchoringfunction for important cell surface proteins such as CD59on red cells, which is a complement lysis inhibitor. Lackof GPI does not permit red cells to express the antigenCD59 on their cell surface, leading to complement lysis.The thrombotic tendency results from inappropriateactivation of platelets that are abnormally sensitive tocomplement. Intravascular haemolysis is reflected by an

increased reticulocyte count, raised bilirubin levels,reduced serum haptoglobins, methaemoglobinaemia,increased urinary urobilinogen, haemoglobinuria andhaemosiderinuria. The latter is only detectable by stainingthe urine with Prussian blue agent. Haemolysis isincreased during sleep, hence the term PNH. It is notclear why haemolysis should increase during sleep.Haemolysis is usually worse after infections, followinginoculations, blood transfusions and surgery. Anaemiacauses fatigue and dyspnoea. Some patients present withintermittent abdominal pain which is worse duringepisodes of active haemolysis. There is mild jaundice andintermittent passage of dark urine. On examination thepatient may be pale and jaundiced. The spleen issometimes palpable and the liver may be enlarged.

Complications include venous thrombosis, increasedsusceptibility to infection due to granulocytopenia, irondeficiency, pigment gallstones and transformation toacute myeloid leukaemia. Thrombosis is the mostfrequent complication. Small vessel thrombosis may bethe cause of abdominal pain, and swelling may be due tointermittent ascites, as in this patient. Portal vein orhepatic vein thrombosis is a serious complication.Pulmonary embolism is a recognized complication. Inthis case the pleuritic chest pain and haemoptysis may bedue to pulmonary embolism or infection, the latterworsening the haemolytic event. Acute myeloidleukaemia complicates a few cases of PNH and ischaracterized by rapid pancytopenia and blast cells in theperipheral blood film.

The diagnosis of PNH is made by addition of acidifiedserum to the red cells, which will cause rapid haemolysis(the Ham’s test). The reason for acidification of theserum is to activate the alternative complement pathway.

The definitive treatment is marrow transplantation.Leucocyte-depleted blood transfusions are oftennecessary.

A systolic click that is synchronous with heart sounds is arare but recognized sign of a small left apicalpneumothorax. It is often referred to as Hamman’s sign.A differential diagnosis of this sign is the systolic click of

mitral valve prolapse; however, this is not consistent withchest pain and breathlessness on exertion of sudden onsetin a young man.

An inspiratory X-ray alone may fail to detect smallapical pneumothoraces. The management is conservative.Patients are advised not to engage in strenuous exertionfor at least two weeks.

1. d. Haemoglobinuria.2. a. Haemolysis.

1. c. Left-sided apical pneumothorax.2. e. Chest X-ray in end expiration.

Answer 156

Answer 157

Page 162: Rapid review of clinical medicine for mrcp part 2

Clinical Cases 161

A 15-year-old female was investigated for short stature.She measured 1.22 m tall. Her sister aged 14 was 1.53 mtall. She was born three weeks prematurely, and wasnoted to have a divergent squint affecting her left eye inchildhood. She had achieved her milestones in a normalmanner, but school performance had been belowaverage. On examination, the patient weighed 42 kg. Shehad a left-sided divergent squint. Her neck was short andher hands were small. Breast development had just begunand there was scanty pubic and axillary hair growth. Theheart rate was 90 beats/min and regular, and the bloodpressure was 110/70 mmHg. The heart sounds werenormal and the chest was clear. Abdominal and neuro -logical examination was also normal.

Initial investigations are shown.

A 33-year-old male returned from Mombasa, East Africawith a high fever, headaches and rigors. On examination,he had a temperature of 40°C (104°F). A blood filmperformed on admission is shown (158a).

115588aa

Hb 13 g/dlWCC 4.2 � 109/lPlatelets 410 � 109/lSodium 136 mmol/lPotassium 3.8 mmol/lUrea 5.1 mmol/lCreatinine 90 μmol/lGlucose 5.2 mmol/lCalcium 1.96 mmol/lPhosphate 1.3 mmol/lAlkaline phosphatase 90 iu/lAlbumin 39 g/lTSH 2 mu/lThyroxine 87 nmol/l

(normal 60–160 nmol/l)LH 4 iu/l (normal 1–12 iu/l)FSH 6 iu/l (normal 1–9 iu/l)GH 2 mu/l X-ray Left wrist, lateral skull –

normal bone age

What is the diagnosis?

1. List two possible causes for her short stature.2. Give one test you would perform to confirm the

diagnosis.

Question 158

Question 159

Page 163: Rapid review of clinical medicine for mrcp part 2

162

The two malaria parasites commonly shown on bloodfilms in higher examinations are P. falciparum andP. vivax (for comparison, the blood films of both parasitesare shown side-by-side [158b, c]). P. falciparum can bedifferentiated from P. vivax by the following features:

• The parasite count is high, whereas in P. vivax it islow.

• The affected cells are small and/or crenated, whereasin P. vivax the cells are large.

• The ring form has two chromatin dots, whereas P. vivax has just one.

• The actual ring is thin, whereas in P. vivax it is thick.

A low serum calcium, raised phosphate and normalalkaline phosphatase are suggestive of either idiopathichypoparathyroidism or pseudohypoparathyroidism. Theshort stature, short neck and squint are characteristic ofpseudohypoparathyroidism. In addition, the patient hassmall hands – presumably secondary to short metacarpals– which are also associated with pseudohypoparathy -roidism. However, these morphological changes are alsoinfrequently present in idiopathic hypoparathyroidism.Pseudohypoparathyroidism results from resistance ofsome target tissues to the actions of PTH. The disordermay be asymptomatic, present with lethargy, neuromus -cular irritability or psychosis. Neuromuscular irritability is

only present in profound hypocalcaemia, and consists ofparaesthesia of the face and extremities, carpopedalspasm, and latent tetany. Convulsions are relativelycommon.

The serum PTH distinguishes between idiopathichypoparathyroidism and pseudohypoparathy roidism. Inidiopathic hypoparathyroidism, PTH level is extremelylow or absent. The diagnosis of pseudohyoparathyroidismis also confirmed by measuring urinary cyclic AMP andphosphate concentration after infusion of PTH(Howard–Ellsworth test). The normal response is anincrease in both cyclic AMP and phosphate in the urineafter PTH. In patients with end-organ resistance to PTH,i.e. pseudohypoparathyroidism, there is no such response.In a small proportion there is a normal response (type IIpseudohypoparathyroidism). In these cases the diagnosisis based on the identification of a relatively high level ofPTH in relation to the low serum calcium level.

115588bb 115588cc

Plasmodium falciparum malaria.

1. i. Pseudohypoparathyroidism.ii. Idiopathic hypoparathyroidism.

2. Serum PTH level.

Answer 158

Answer 159

Page 164: Rapid review of clinical medicine for mrcp part 2

Clinical Cases 163

116600aa

116600bb

A 55-year-old West Indian male presented withsevere anterior chest pain. On arrival, he was hypo -tensive. An ECG performed in the Accident andEmergency Department is shown (160a). Shortlyafter admis sion, the patient had an echo cardiogram(160b).

A 70-year-old male presents to the Accident andEmergency Department in the morning with a painfulswelling on the anterior aspect of his left thigh which hehad noticed on the previous evening. The swelling hadbecome bigger and he was having difficulty walking dueto the pain. He denied any recent trauma to the leg.Apart from a past history of TB 30 years ago, there wasno other history of note. He was widowed three yearsago and lived alone. He took paracetamol infrequentlyfor aches and pains in his limbs.

On examination, he appeared unkempt and was thin.There was no pallor or oedema. Vital parameters werenormal and he was apyrexial. There was a 6 cm, tenderswelling on the anterior aspect of the left thigh. Therewas also evidence of widespread bruising on his forearmsand legs.

Investigations are shown.

Hb 11 g/dlWCC 5 � 109/lPlatelets 170 � 109/lMCV 90 flPT 14 s (control 14 s)APTT 45 s (control 45 s)BT 12 min (control <10 min)

1. What is the complete diagnosis?2. What would be the next step in your

management?

What is the diagnosis?a. Vitamin C deficiency (scurvy).b. Folate deficiency.c. B12 deficiency.d. Von Willebrand’s disease.e. Chronic liver disease.

Question 160

Question 161

Page 165: Rapid review of clinical medicine for mrcp part 2

164

Patients with proximal dissections should undergo urgentsurgical repair, but those with distal dissection are treatedconservatively. Repair is by resection of the ascendingaorta and replacement with dacron. In the interim, itwould be important to relieve the severe pain withopiates and insert a temporary pacing wire to treat thecomplete heart block and help maintain circulation as heis hypotensive. In acute dissection, hypotension is due torupture of the proximal dissection into the pericardialspace, which is not present in this case; therefore,complete heart block is contributing to the hypotensionin this case. In contrast with this case, most patientspresenting with acute dissection are hypertensive and inthese circumstances it is prudent to treat the hypertensionaggressively to obtain a systolic blood pressure equal toor less than 100 mmHg. Drugs most commonly usedinclude intravenous beta-blockers, for example, labetolol,or intravenous nitroprusside. It is very important torecognize that the cause of myocardial infarction is aproximal dissection, and not coronary thrombosis. Whilethrombolytic therapy has revolutionized the managementof acute coronary thrombosis, it is an absolutecontraindication in a patient who has sustained amyocardial infarction secondary to aortic dissection.

Aortic dissection is catastrophic, with a mortality rate of1% per hour for the first 48 hours in untreated patients.Some 95% of dissections originate at the site of theintimal tear. Most tears occur in the ascending aorta(65%), followed by the proximal descending aorta (20%),in the transverse arch (10%) and in the distal thoracicaorta (5%). Predisposing factors are tabulated above.

Clinical features of acute dissection include verysudden onset of chest pain, either in the chest or betweenthe shoulder blades. Syncope from tamponade,neurological deficit from involvement of the carotidartery, abdominal pain from mesenteric ischaemia,ischaemic upper limbs from brachiocephalic and leftsubclavian involvement and acute renal failure frominvolvement of the renal arteries are other presentations.

The diagnosis may be made by transthoracicechocardiography; however, transoesophagealechocardiography is the ‘gold standard’ test in thediagnosis of suspected dissection. MRI scanning issuperior to transoesophageal echocardiogaphy, but itsmain disadvantage is that it is not portable.

Conditions predisposing to aortic dissection

• Hypertension in patients of Afro-Caribbeanorigin

• Marfan’s syndrome• Ehlers–Danlos syndrome• Pregnancy• Coarctation of the aorta• Relapsing polychondritis

The normocytic anaemia, bruising and abnormalbleeding time in an elderly patient point to the diagnosisof vitamin C (ascorbic acid) deficiency, or scurvy.Vitamin C is abundant in fresh greens, citrus fruits, liverand kidney. Nutritional deficiency is relatively common inthe elderly, who eat predominantly tinned foods. VitaminC is required for the hydroxylation of proline in collagensynthesis. Deficiency results in abnormal connectivetissue, resulting in easy bruising, bleeding gums,periosteal haemorrhage and muscle haematomas. Thecharacteristic signs of vitamin C deficiency are

perifollicular haemorrhages, widespread bruising and‘corkscrewing’ of the body hair. The latter may benormal in the elderly. Patients may have coexistent ironand folate deficiency. The marrow is megaloblastic whenthere is folate deficiency. The bleeding time is prolonged,although platelet function is normal. The diagnosis ismade by the demonstration of a low platelet or leucocyteascorbate concentration. Treatment is with high doses ofascorbic acid.

The normal MCV is not consistent with the diagnosisof B12 deficiency or chronic liver disease. There is noprevious history of haemorrhage to indicate VonWillebrand’s disease.

1. Acute inferior myocardial infarction with complete heart block complicating an acute proximal aortic dissection. The echocardiogram demonstrates an enlarged aortic root with an obvious dissection flap.(See Echo cardiography, page 421.)

2. Immediate referral to the cardiothoracic surgeons.

a. Vitamin C deficiency (scurvy).

Answer 160

Answer 161

Page 166: Rapid review of clinical medicine for mrcp part 2

Clinical Cases 165

A 48-year-old female had an 11-month history ofabdominal cramps and diarrhoea. She moved her bowelstwice daily; the stool was loose and foul-smelling. Therewas no history of bleeding per rectum or tenesmus. Herappetite was unchanged, but she had lost 8 kg in weight.Two years ago she had a total abdominal hysterectomyand salpingo-oophorectomy for carcinoma of the cervix,followed by a course of external radiotherapy. She hadbeen followed up regularly by her gynaecologist, and wasfree from recurrence of the malignancy. She was marriedwith two sons, aged 10 and 12. She worked as asecretary. She consumed 1 unit of alcohol daily and was anon-smoker. There was no history of travel abroad. Shehad a long history of Raynaud’s phenomenon. She tookparacetamol for very infrequent headaches.

On examination she was thin. There was no pallor,clubbing, lymphadenopathy or oedema. The temperaturewas normal. Inspection of the oral cavity revealed twosmall, non-tender, shallow ulcers. The abdomen was thin.There was a lower, midline scar. Mild tenderness waselicited on palpation of the right iliac fossa. Rectalexamination was normal. Investigations are shown.

Hb 10g/dlWCC 8 � 109/lPlatelets 199 � 109/lMCV 106 flESR 16 mm/hSerum B12 98 pmol/l Serum folate 18 μg/lCalcium 2.12 mmol/lPhosphate 0.9 mmol/lAlkaline phosphatase 112 u/lAlbumin 38 g/lU&E NormalTotal thyroxine 110 nmol/lTSH 1.9 mu/lAbdominal X-ray NormalChest X-ray Normal

Hb 10.5 g/dlWCC 6 � 109/lPlatelets 300 � 109/lMCV 61 flMCHC 28 g/lMCH 19 pgRed cell count 6.8 � 1012/lBlood film Hypochromia, anisocytosis,

poikilocytosis, target cellsSodium 136 mmol/lPotassium 4.1 mmol/lUrea 4 mmol/lCreatinine 72 μmol/l

1. What is the diagnosis?a. Crohn’s disease.b. Coeliac disease.c. Small bowel lymphoma.d. Radiation enteritis.e. Tuberculous ileitis.

2. Which investigation would you perform to reach a diagnosis?a. Schilling test.b. Duodenal biopsy.c. Small bowel meal.d. 14CO2 breath test.e. Duodenal aspiration for culture.

1. What is the most probable diagnosis?a. β-thalassaemia major.b. Anaemia of chronic disease.c. β-thalassaemia minor.d. Iron deficiency anaemia.e. Paroxysmal nocturnal haemoglobinuria.

2. What two investigations should be performed?a. Blood film.b. Ham’s test.c. Serum ferritin.d. Serum B12.e. Serum folate.f. Thyroid function.g. Haemoglobin electrophoresis.h. Coomb’s test.i. Liver function tests.j. Red cell sequestration study.

Question 162

Question 163

An 18-year-old female had blood tests as part of a routinemedical (below).

Page 167: Rapid review of clinical medicine for mrcp part 2

166

The patient has a malabsorption syndrome. The B12 levelis low in the presence of a normal folate level. Thissuggests either selective B12 malabsorption due toterminal ileal disease (Crohn’s disease, terminal ilealresection, ileal TB), pernicious anaemia or excessutil ization of B12 within the gut itself (bacterialovergrowth which occurs in the presence of a structuralabnormality of the small bowel, e.g. after Billroth IIgastric surgery, jejunal diverticula, Crohn’s disease,intestinal fistulae, radiation enteritis and small bowelinvolvement in systemic sclerosis, amyloidosis anddiabetic autonomic neuropathy).

Symptoms of malabsorption are absent in perniciousanaemia. The differential diagnosis is, therefore, betweenterminal ileal disease or bacterial overgrowth. There is nomention of small bowel surgery in the question. Ileal TBis unlikely in the absence of a febrile illness, night sweats,only slightly elevated ESR, absent past history or contact

with TB, and a normal chest X-ray. Although Crohn’sdisease is a possibility, the most likely answer is radiationenteritis. Radiation enteritis can complicate radiotherapyto the abdominal region. In the acute phase this mayremain silent or present as diarrhoea. The sequel isfibrotic strictures or adhesions causing cramp-likeabdominal pain. Strictures predispose to bacterialovergrowth, which in turn leads to steatorrhoea due todeconjugation of bile salts. Vitamin B12 is metabolized bythe bacteria, whereas folate absorption is left unhindered.Moreover, some bacteria actually produce folate, oftenleading to elevated levels in the blood. Investigationinvolves small bowel meal, which will demonstrate thestructural abnormality. Hydrogen or carbon dioxidebreath tests will confirm the diagnosis.

Small bowel meal is regarded as the investigation ofchoice in this question because it will detect Crohn’sdisease, ileal TB, radiation enteritis and small bowellymphoma. The latter occasionally complicatesradiotherapy, but it does not lead to a selective B12 deficiency.

The best answer is β-thalassaemia minor. The clue lies inthe fact that the MCV is disproportionately low com -pared with the Hb. In β-thalassaemia trait (minor), thecells are small due to defective globin synthesis and areremoved relatively rapidly from the circulation by thespleen. As a consequence, the marrow produces largeamounts of red cells and, provided that iron and folatestores remain replenished, the Hb is only modestlyreduced but the cells are relatively small. The red cellcount itself, how ever, may be high. In iron deficiency,the marrow is unable to produce red cells to maintain anormal Hb because haem synthesis is incomplete withoutiron. Therefore, the number of cells produced (and hencethe red cell count) is small compared with β-thalassaemiatrait; however, the size is similar. By the time the irondeficiency has become severe enough to cause a reduc -tion in the MCV as low as 61 fl, the Hb is much lowerthan in β-thalassaemia trait.

In β-thalassaemia major the Hb, red cell count andMCV are all very low, and such patients require life-longblood transfusions to maintain a Hb around 10 g/dl. α-thalassaemia trait may present with a similar picture asβ-thalassaemia trait if two of the four genes which code

for α-globin chains are defective. When only one gene isdefective the Hb and MCV are usually normal.

The investigation of choice is Hb electro phoresis. Thisis normal in a patient with iron deficiency, revealingapproximately 98% HbA (i.e. normal Hb comprising twoα- and two β-globin chains) and 2% HbA2 (two α- andtwo δ-globin chains). In β-thalassaemia trait the HbA isreduced to approximately 90%, the HbA2 is slightlyincreased to 6%, and in addition may be the presence ofsome HbF (fetal Hb; two α- and two γ-globin chains).The reason for this is that in β-thalassaemia trait thedeficiency of normal β-globin means that α-globin chainscombine with either γ-globin or δ-globin. In β-thalassaemia major, where there is no normal β-globinproduction, almost all the Hb is HbF.

The serum ferritin is also useful because it is low in irondeficiency but normal or increased in β-thalas saemia trait,unless there is coexisting iron deficiency. The slightlyelevated ferritin in some patients with β-thalassaemia traitmay be explained by the fact that defective β-globinsynthesis prevents the marrow utilizing all the iron whichis absorbed from the gastrointestinal tract, resulting insome iron accumulation.

Patients with megaloblastic marrow as seen in B12 orfolate deficiency and those with hypothyroidism andchronic liver disease have a raised MCV. Patients withanaemia of chronic disease generally have a normal MCV.There is nothing in this patient’s history to indicateparoxysmal nocturnal haemoglobinuria, therefore a Ham’stest is not necessary.

1. c. β-thalassaemia minor.d.Iron-deficiency anaemia.

2. c. Serum ferritin.g. Haemoglobin electrophoresis.

Answer 162

1. d. Radiation enteritis.2. c. Small bowel meal.

Answer 163

Page 168: Rapid review of clinical medicine for mrcp part 2

Clinical Cases 167

A 76-year-old female developed gradual onset of breath -lessness over the past three weeks. On admission, she hadstridor. Her respiratory rate was 30/min, and she was notcyanosed. Her trachea was slightly deviated to the right,and there was a mass just palpable in front of it, whichmoved slightly with swallowing. The lower margins ofthe mass could not be defined. Auscultation of the lungsrevealed stridor-type sounds transmitted from the upperairways. She was managed with oxygen and saline nebu -lizers in the Accident and Emergency Department, andfelt considerably better. She mentioned having a problemwith her thyroid gland, and over 20 years ago was toldshe did not need any treatment. She had lost 5 kg inweight over the past two months.

She had a chest radiograph (164a) in the Accidentand Emergency Department.

Lung function tests and a thyroid scan were per -formed the following day. Results of the flow loop curve(164b) and thyroid scan (164c) are shown.

116644aa 116644bb

116644cc

1. What is the abnormality on the chest radiograph? List four possible causes for this appearance.

2. What is the abnormality on the flow loop curve and what does this suggest?

3. Give three possible causes for the abnormality in the flow loop curve in our patient.

4. What is the abnormality on the thyroid scan?5. List two tests which would help confirm the cause

of her breathlessness.

Expiration

Inspiration

0 2 4 6Change in lung volume (l)

10

8

6

4

2

0

2

4

6

8

10

Flow (l/s)

Question 164

Question 165

The blood results shown were performed two monthsapart in a patient with HIV infection.

What is the most likely cause for the discrepancy between the results?

a. Development of B12 deficiency.b. Development of folate deficiency.c. Development of systemic lupus erythematosus.d. The patient has become pregnant.e. Zidovudine toxicity.

October December Hb (g/dl) 11.8 7.6WCC (� 109/l) 3.0 3.1Platelets (� 109/l) 160 90MCV (fl) 86 110Reticulocyte count (%) 1.5 4CD4 24 120

Page 169: Rapid review of clinical medicine for mrcp part 2

168

The patient has become more anaemic, has developed amacrocytosis, thrombocytopenia and an increase in theCD4 count. The most likely cause of these abnormalities istreatment with AZT. AZT is a nucleoside analogue thatinhibits the viral enzyme reverse transcriptase and is a DNAchain terminator. It is widely used in symptomatic HIVinfection. In such patients it has been shown to reducemortality by reducing the incidence of opportunisticinfections owing to a significant but transient improvementin the CD4 count. The drug has important haematologicalside-effects, which include a reduction in all cell lines and amegaloblastic change in the marrow. The general side-effects of the drug include nausea, vomiting, myalgia,headaches, neuropathy and myopathy.

Other drugs which cause pancytopenia andmacrocytosis in HIV patients are co-trimoxazole (used inthe treatment of Pneumocystis carinii infection) and2',3'-deoxynucleoside analogues other than AZT, such asdideoxyadenosine and dideoxyinosine. The latter drugshave less marrow toxicity.

Marrow suppression may occur in HIV as a directresult of the virus.

Another important haematological manifestation ofHIV in relation to the MRCP and similar examinations isthe presence of a circulating anticoagulant in some cases.All the other options except pregnancy may result in apancytopenia and raised MCV. While pregnancy maycause anaemia it would usually have no bearing on thewhite cells or platelets in the early stages.

There is a history of a chronic thyroid disorder in this elderlywoman. She is clinically and biochemically euthyroid, andhas a palpable mass in the suprasternal area which moveswith swallowing. The most probable cause for the mass inthe superior mediastinum is a retrosternal extension of agoitre. The reason for sudden deterioration is trachealcompression due to rapid enlargement of the goitre. This

may occur as a result of haemorrhage or malignant changewithin the goitre. An additional cause of trachealcompression in this situation is compression from malignantlymph nodes. Tracheal compression causes stridor when it issevere. It is potentially a medical emergency requiring rapidrelief of the compression, the speed of which is determinedby the extent of respiratory embarrass ment.

Carcinoma of the thyroid complicates approximately 4%of multi-nodular goitres. Most patients are asymptomatic,but some experience symptoms as a result of local com -pression of structures which include dysphagia, dysphonia,stridor and haemoptysis. Technetium scanning is the mostwidely used imaging agent of a goitre. Lack of uptake oftechnetium in parts of the thyroid gland is suggestive ofhypofunction. Such ‘cold’ nodules are associated withcarcinoma of the thyroid in 20% of cases. Cold areas canalso occur with cysts, colloid adenomas, degenerativenodules and thyroiditis. The differen tiation from a cyst ispossible with US scanning. This patient has a very large areain the left lobe where there is no uptake (cold area). An USwill confirm whether this is a cystic or a solid lesion; how -ever, in this case the symptom of stridor raises the suspicionof sinister pathology and therefore a CT scan of the neckand thorax is more likely to provide extra information aboutthe gland, its pathology and the extent of direct invasionfrom a probable malignant process. Fine-needle aspirationof the lesion is an invaluable means of making a histologicaldiagnosis, provided an experienced cyto pathologist isavailable to interpret the histology. The main limitation ofthis procedure is that it does not neces sarily allowdifferentiation between a follicular adenoma and follicularcarcinoma, and in a few cases false-negative results aregenerated due to sampling error. The results of the aspirateshould be interpreted in the context of the patient’spresentation and results from other imaging techniques.

1. The lateral chest X-ray reveals a mass in the superior mediastinum. Possible causes of a superior mediastinal mass include (i) retrosternal goitre, (ii) thymoma, (iii) lymphoma, (iv) malignant lymphadenopathy, (v) teratoma, and (vi) aneurysm of the aortic arch. The most likely cause of the mass in this situation is a retrosternal goitre.

2. There is evidence of reduced airflow rate which is more pronounced in the inspiratory phase of respiration. This is suggestive of extrathoracic large airways obstruction. Typical examples include retrosternal goitre and its complications, compression by malignant lymph nodes and bilateral vocal cord paralysis.

3. i. Compression from a retrosternal multinodular goitre which has undergone malignant change.

ii. Compression from a retrosternal multinodular goitre due to spontaneous haemorrhage withinthe gland.

iii. Compression from locally metastasized lymphatic deposits from carcinoma of the thyroid.

4. A large ‘cold nodule’ in the left lobe of the thyroid gland.

5. i. CT scan of the neck and upper thorax.ii. Fine-needle aspirate of the thyroid gland for

histology.

Answer 164

e. Zidovudine toxicity.

Answer 165

Page 170: Rapid review of clinical medicine for mrcp part 2

Clinical Cases 169

A 30-year-old Bangladeshi female was admitted to hospitalwith a two-month history of increasing lethargy and ankleswelling. One week before admission she developed rightupper quadrant pain, nausea and anorexia. She hadimmigrated from Bangladesh two years ago, and ever sincecoming to this country she had noticed a troublesomecough which was productive of yellow sputum and forwhich she had been prescribed several courses of oralantibiotics. She lived with her husband in a rented flat.

On examination she was thin and slightly pale. Hertemperature was 37.5°C (99.5°F). The heart rate was110 beats/min and regular; the blood pressure was95/55 mmHg. The JVP was grossly elevated. Onexamination of the precordium the apex was not palpable,and on auscultation both heart sounds were soft. Onexamination of the respiratory system there were harshbreath sounds on auscultation of the left upper zone.Abdominal examination demonstrated smooth and tenderhepatomegaly palpable 6 cm below the costal margin. Inaddition, there was clinical evidence of moderate ascites.The lower limbs had pitting oedema to the shins.

Investigations are shown.

116666aa 116666bb

Hb 11 g/dlWCC 10 � 109/lPlatelets 180 � 109/lMCV 85 flESR 80 mm/hSodium 129 mmol/lPotassium 4.3 mmol/lUrea 8 mmol/lCreatinine 90 μmol/lGlucose 5 mmol/lEchocardiogram (166a)Sputum microscopy (166b)

The data are from cardiac catheter traces of a 14-year-oldmale, taken five years apart.

5 years ago PresentPressure Satn (%) Pressure Satn (%)(mmHg) (mmHg)

Superior vena cava 68 49Right atrium 8 68 12 50Right ventricle 40/5 94 120/20 50Pulmonary artery 40/18 94 115/55 50Pulmonary capillary

wedge pressure 6 8Left ventricle 110/4 95 115/15 57Aorta 115/60 95 100/50 50

1. What is demonstrated on the echocardiogram?2. What is the most probable cause for the abnormality

on the echocardiogram?3. List two therapeutic steps in your management of

this patient.4. What other step would you take after initiating

therapy for the patient?

1. List three abnormalities in the catheter performed five years ago.

2. List three abnormalities in the catheter performed most recently.

3. What is the unifying diagnosis?

Question 166

Question 167

Page 171: Rapid review of clinical medicine for mrcp part 2

170

The most likely underlying cause is TB, considering that thewoman has a productive cough and the sputum, which isstained with the Ziehl–Neelsen stain, reveals acid-fast bacilli.In the UK, the incidence of TB in immigrants from theAsian subcontinent is 40 times higher than in the nativewhite population. Primary TB is usually without symptomsin the majority of individuals, although in a few cases theremay be a cough, wheeze and fever. The primary complexheals with calcification; however, there may be reactivationof infection several years later due to malnourishment orconditions predis posing to immunosuppression resulting inpulmonary TB and possible dissemination to other organsvia the lym phatics and the bloodstream.

The diagnosis is based on identifying the organismfrom pericardial fluid. In the simplest form this may bedone by staining the fluid with Ziehl –Neelsen to revealacid-fast bacilli; however, the yield is only 25% with thismethod. The organism may be cultured on Lowenstein-Jensen medium but this can take over eight weeks. Thequickest method of making the diagnosis in the event ofa negative acid-fast stain is by PCR to tuberculous protein or by demonstrating evidence of the disease elsewhere, for example in the patient’s sputum.

Treatment is with empirical antituberculous therapy.Because TB is spread from person to person, and effectivetracing of close contacts helps limit spread of the disease, itis important to identify all individuals who may have comeinto close contact with the patient or those who share thesame bathroom facilities. Individuals who are traced shouldhave a chest X-ray and a tuberculin test. In adults, a normalchest X-ray positive test is a sign of immu nity and nothingmore needs to be done. In children who have not had aBCG vaccine, a positive tuber culin test is taken as evidenceof infection, and treat ment is instituted. If the test isnegative, it is repeated again in six weeks and if remainsnegative the patient is given the BCG vaccine. In childrenunder the age of one year who have a family member withTB, isoniazid is given as chemoprophylaxis for six months,together with immunization with a strain of BCG which isresistant to isoniazid.

Causes of pericardial effusion

• Any infective pericarditis• Uraemia• Post myocardial infarction• Post cardiotomy• Dressler’s syndrome• SLE• Rheumatoid arthritis• Malignancy• Hypothyroidism• Trauma• Aortic dissection• Drugs, e.g. practolol, phenylbutazone,

procainamide, hydralazine

(See Interpretation of Cardiac Catheter Data, page 418.)

1. The echocardiogram demonstrates a large pericardial effusion. (See Table for causes.) (See Echocardiography, page 421.)

2. Tuberculous pericardial effusion.3. i. Pericardiocentesis.

ii. Empirical antituberculous therapy with rifampicin, isoniazid and ethambutol. In patients of ethnic origin, or where multi-resistant bacilli are suspected, there is a case for adding pyrazinamide.

4. Contact tracing.

1. i. Elevated right ventricular pressure.ii. Elevated pulmonary artery pressure.iii. Increase in oxygen saturation in the right

ventricle, suggesting a left-to-right shunt at the level of the ventricle.

2. i. Huge increase in right ventricular pressure in the following five years.

ii. Huge increase in pulmonary artery pressure in the following five years.

iii. Reduction in oxygen saturation in the left ventricle, suggesting a right-to-left shunt at the level of the ventricles.

3. There has been a large increase in right ventricular pressure, leading to shunt reversal. The patient has developed Eisenmenger’s phenomenon.

Answer 166

Answer 167

Page 172: Rapid review of clinical medicine for mrcp part 2

Clinical Cases 171

A 45-year-old male was referred to the local psychiatristby his GP with a three-month history of lethargy, loss ofappetite, nausea, early morning wakening and loss oflibido. He had recently lost his job as a clerk for makingthree serious errors in the space of a fortnight. There wasno history of headaches or weight loss. He hadexperienced intermittent abdominal pain after mealswhich was investigated with upper gastrointestinalendoscopy four weeks earlier and attributed to a smallgastric ulcer. Biopsy from the ulcer demonstrated an in -flammatory infiltrate but there was no evidence ofmalignancy or Helicobacter pyloris. He had a long historyof constipation which had not caused him any concern,and more recently complained of pains in his hands andfeet. He was not sleeping well and awoke between twoand three times a night to pass urine, which was ex -tremely unusual for him. His wife was concerned that hewas worried about loss of earnings and had become verydepressed. He had no other significant past medicalhistory. He was a non-smoker and consumed half a bottleof wine daily.

On examination, he had a reduced affect. There wasno pallor, clubbing or lymphadenopathy. He was apy -rexial. The heart rate was 96 beats/min and blood pres -sure 160/95 mmHg. Examination of his eye is shown(168a). On abdominal examination, a mass wasidentified in the right iliac fossa which was indentible.There was no tenderness and the bowel sounds wereaudible. A rectal examination was not performed. Theexamination of the cardiovascular and respiratory systemwas normal. He had painful hands, but there was noevidence of joint swelling or restriction of movements.Examination of his central nervous system, including hishigher mental function was entirely normal.

Investigations are shown.

116688aa

Hb 15 g/dlWCC 5 � 109/lPlatelets 306 � 109/lESR 18 mm/hSodium 136 mmol/lPotassium 4.2 mmol/lBicarbonate 19 mmol/lChloride 115 mmol/lUrea 7 mmol/lCreatinine 89 μmol/lBilirubin 12 μmol/lAST 24 iu/lAlkaline phosphatase 300 iu/lTotal protein 78 g/lAlbumin 44 g/lX-ray of hands (168b)

1. List two tests you would perform to come to a rapid diagnosis.

2. What is the definitive treatment?3. How would you manage the patient after he has

had the definitive treatment?

Question 168

116688bb

Page 173: Rapid review of clinical medicine for mrcp part 2

172

Answer 168

The symptoms of hypercalcaemia are rather non-specific,and include nausea, anorexia, weight loss, abdominal pain,bone pain, polyuria, polydipsia, lack of concentration,depression and occasionally acute psychosis. Theabdominal pain may be non-specific or due to constipa -tion, peptic ulcer disease, pancreatitis and renal colic, all ofwhich are recognized complications of hyperpara thyroid-ism. It is worth remembering that while nephrocalcinosisis also a complication of hyperpara thyroidism, it is notassociated with abdominal pain. This man has several ofthese symptoms, as well as recent peptic ulcer disease. Theeye in 168a reveals corneal calcification which occurs inhypercalcaemic states. The calcification is usually seen atthe 3 and 9 o’clock positions. The hand X-ray (168b)demonstrates periosteal bone resorption which is charac -teristic of hyperpara thyroidism. Hyperparathyroid-relatedbone disease is further supported by the raised alkalinephosphatase level. Bone changes are seen in 2% of cases ofhyperparathyroidism. Cystic lesions may be present, andcan result in pathological fractures. Extensive bone resorp -tion affecting the skull gives it a characteristic ‘salt-and-pepper’ appearance on the skull X-ray.

Hyperparathyroidism is diagnosed by demonstratingelevated or normal levels of PTH in the presence of a highserum calcium. Primary hyperparathyroidism is usuallysecondary to a single, benign parathyroid adenoma.Hyperparathyroidism due to carcinoma of the parathyroidglands is extremely rare. Hyperparathyroidism can be partof MEN syndromes. Secondary hyperparathyroidism resultsfrom chronic calcium defi ciency, and is a compensatingmechanism whereby all four parathyroid glands undergohyperplasia. In this situation, the calcium may be low ornormal, but the PTH level is elevated. The condition istreated with vitamin D and calcium supplements. Tertiaryhyperparathyroidism compli cates some cases of secondaryhyperparathyroidism where auto nomous production ofPTH occurs, leading to hypercalcaemia which is notcontrolled with vitamin D or calcium supplements. In thiscase, there is no underlying disorder to predispose tosecondary or tertiary hyperpara thyroidism, and so the mostlikely answer is primary hyperparathyroidism.

Other causes of hypercalcaemia are given below(Table). Myeloma is unlikely in the presence of a normalfull blood count, ESR and globulin level. Sarcoidosis isassociated with hypercalcaemia and band keratopathy, but

is less likely in the presence of a normal chest X-ray. More -over, it would not explain the X-ray changes in the hands.This patient is not taking regular antacids, which excludesthe milk-alkali syndrome, a condition caused by chronicingestion of calcium-based antacids and milk for the reliefof dyspepsia or peptic ulcer disease. Metastatic bonedisease is a possibility, but the majority of patients withbony metastases have overt metastatic deposits, particu -larly in areas where pain is experienced. In such cases, thealbumin level is also low due to cachexia and the plasmabicarbonate is normal or high because bony destructionoften releases bicarbonate from the bone. In hyperpara -thyroidism in the absence of renal disease, a mild meta -bolic acidosis is common. It is due to RTA and is reflectedby a slightly low bicarbonate and a high chloride level.

In the past, other causes of hypercalcaemia – particu -larly malignancy – could be differentiated from hyper -parathyroidism by the hydrocor tisone suppression test,where high doses of hydro cortisone given daily for a weekwould fail to reduce the serum calcium level in hyper -parathyroidism but not in other cases; however, the moresensitive PTH assays used nowadays have made this testobsolete.

Treatment of primary hyperparathyroidism is with totalparathyroidectomy following trans plantation of a smallamount of parathyroid tissue into the forearm muscles.Following surgery, calcium levels have to be monitoredbecause they fall rapidly and tetany can be precipitated.Hypocalcaemia can persist for several months during whichtime it is important to continue vitamin D supple ments.

(See Calcium Biochemistry, page 415.)

Causes of hypercalcaemia

1° hyperparathyroidism Paget’s disease (immobility)

3° hyperparathyroidism Familial hypocalcuric hypercalcaemia

SarcoidosisMultiple myelomaThiazide diureticsMilk-alkali syndromeAddison’s diseaseAcromegalyPhaeochromocytomaThyrotoxicosisVitamin D intoxication

1. i. Serum calcium.ii. PTH level.

2. Parathyroidectomy.3. Vitamin D supplements.

Page 174: Rapid review of clinical medicine for mrcp part 2

Clinical Cases 173

Question 169

A 44-year-old female was persuaded to come into hos -pital after she was seen in the out-patient clinic withjaundice. She was diagnosed as having auto-immuneCAH five months ago and was suc cessfully treated withhigh-dose steroids, which had gradually been withdrawn.She had not been taking any medication before admissionand denied alcohol consumption. On examination, shewas thin and deeply jaundiced. She was alert andorientated, and did not demonstrate any signs ofcognitive impairment. Abdominal examina tion wasessentially normal. Investigations on admission areshown. Two days following admission, the patientbecame suddenly very aggressive, and soon aftercollapsed and was comatosed.

Hb 12.3 g/dlWCC 8 � 109/lPlatelets 45 � 109/lMCV 109 flPT 59 s (control 13 s)APTT 90 s (control 45 s)Bilirubin 300 μmol/lAST 350 iu/lAlkaline phosphatase 340 iu/lAlbumin 26 g/l

Which urgent investigation would be most useful in determining the cause of collapse?

a. CT scan of the head.b. EEG.c. Drug screen for sedatives.d. Blood cultures.e. Blood glucose.

Question 170

This pedigree chart (170) is from a family with a veryrare condition.

1177001. What is the mode of inheritance?

a. Autosomal dominant.b. Autosomal recessive.c. X-linked dominant.d. X-linked recessive.e. Autosomal dominant with incomplete

penetrance.

2. What are the chances of A being affected?a. None. d. 50%.b. 100%. e. 75%.c. 25%.

3. What are the chances of B being affected?a. None. d. 50%.b. 100%. e. 75%.c. 25%.

4. What are the chances of C being affected?a. None. d. 50%.b. 100%. e. 75%.c. 25%.

A B

C

Affected male Unaffected male*

Affected female Unaffected female*

* Does not exclude inheritance of an autosomalrecessive gene in either sex or an X-linked recessivedisease in a female

Page 175: Rapid review of clinical medicine for mrcp part 2

174

Answer 169

The patient was admitted with gross hepatic dysfunction.She had previously been successfully treated with corti -costeroids, and had a relapse after they were graduallywithdrawn. She became aggressive and then comatosed.The most probable diagnosis is hepatic encephalopathy.Precipitating factors include gastroin testinal haemorr -hage, sepsis, hypokalaemia, drugs promoting catabolism,sedatives and hypoglycaemia. In hepatic encephalopathyit is possible for both hypo glycaemia and hyperglycaemiato occur because glucose homeostasis is disturbed. Glyco -gen is neither efficiently stored nor mobilized andgluconeogenesis is impaired. Early features of encepha -lopathy include confusion, irritation, aggressive ness orpsychosis. Coma can ensue rapidly. The test which wouldbe most useful in determining the cause of collapse is the

serum blood glucose, because its correction and thera -peutic effect are almost immediate. There is nothing inthe history or examination to suggest sepsis, although itmust be noted that patients with chronic liver disease donot always have a pyrexia. There is no history of sedativedrug ingestion, and in practice screening for sedativedrugs is unhelpful because it seldom influences themanagement. A cerebral haemorrhage is possible in thepresence of a raised PT and low platelets. If the bloodglucose is normal, a cerebral haemorrhage should beexcluded by CT scanning of the head. If this is normal,the diagnosis is encephalopathy. An EEG is not necessaryto diagnose encephalopathy, but at the onset of encepha -lopathy the EEG shows an alpha rhythm and as drowsi -ness worsens this is replaced by lower frequency thetaactivity. In deep coma, high-amplitude delta waves arepresent.

e. Blood glucose.

In X-linked conditions there is no male-to-male trans -mission. In X-linked dominant conditions, the femaleoffspring of affected males have an abnormal phenotype,and in X-linked recessive conditions all female offspring

are carriers but do not have the disease. In X-linkeddominant conditions, an affected female will transmit thedisorder to 50% of her daughters and 50% of her sons. InX-linked recessive conditions, an affected female willtransmit the disease to 50% of her sons, and 50% of thedaughters will be carriers, i.e. none will have the diseasephenotype.

Examples of X-linked dominant conditions includevitamin D-resistant rickets, Rett’s syndrome and incon -tinentia pigmentii.

1. c. X-linked dominant.2. a. None.3. b. 100%.4. d. 50%.

Question 171

A 65-year-old male was admitted with a six-week history ofprogressive dyspnoea. One week before admission he haddeveloped dyspnoea at rest, orthopnoea and paroxysmalnocturnal dyspnoea. He had lost his appetite, and had lostover 3 kg in weight in six weeks. He had a coughproductive of yellow sputum initially which was treatedwith antibiotics; however, the cough persisted and threedays before admission he had an episode of frank haemop -tysis. The patient had enjoyed a very active and indepen -dent life and, apart from a truncal vagotomy and drainageprocedure for a perforated duodenal ulcer 20 years ago, hehad been very well. He was a non-smoker.

On examination, he appeared pale and was dyspnoeic.The heart rate was 144 beats/min and irregular; the bloodpressure was 160/100 mmHg. The JVP was not raised. Therespiratory rate was 30/min and temperature was 38°C(100.4°F). The heart sounds appeared normal; there wereno audible murmurs. Auscultation of the lung fields revealedfine inspiratory crackles both anteriorly and posteriorly.Examination of the abdomen and central nervous systemwas normal.

A subclavian line was inserted which revealed a CVP of3 cmH2O. A urinary catheter drained a residual urinevolume of 300 ml.

Answer 170

Page 176: Rapid review of clinical medicine for mrcp part 2

Clinical Cases 175

Investigations were as follows:

The patient was treated with intravenous cefuroximeand erythromycin, and gently transfused with 3 units ofpacked cells. The temperature came down, CVP increasedto 9 cmH2O, and urine output averaged 420 ml/24 h.The Hb following transfusion was 6.4 g/dl. While on theward, the patient had three episodes of frank haemoptysis.

Serial chest X-rays over four days remained unchanged.He reverted to sinus rhythm three days after commencingdigoxin.

Serum biochemistry on day four is shown.

Hb 5.3 g/dlWCC 17 � 109/lPlatelets 249 � 109/lMCV 89 flMCHC 34 g/dlESR 102 mm/hSodium 131 mmol/lPotassium 5.7 mmol/lUrea 29 mmol/lCreatinine 560 μmol/lCalcium 2.1 mmol/lPhosphate 1.5 mmol/lAlbumin 33 g/lBilirubin 14 μmol/lAST 19 iu/lAlkaline phosphatase 113 iu/lLDH 1320 iu/lThyroid function tests NormalCRP 316 mg/lPT 12 s (control 12 s)APTT 43 s (control 41 s)Renal US Normal-sized kidneys with

echogenic renal parenchymaNo evidence of obstruction

Arterial blood gases (on 40% oxygen):pH 7.32PaCO2 5.6 kPaPaO2 14 kPaBicarbonate 14 mmol/lO2 saturation 99%

Blood cultures No growthUrinalysis Blood +++

Protein +++Pus cells +++Granular casts +++

Urine culture No growthECG Atrial fibrillation;

ventricular rate of 154/minNon-specific T-wave changes

Chest X-ray (171)Echocardiogram Biatrial enlargement

Mild left ventricular hypertrophy with good systolic functionThe mitral valve appeared normal

117711

Sodium 131 mmol/lPotassium 6.0 mmol/lUrea 35 mmol/lCreatinine 780 μmol/l

1. Describe the chest X-ray and give two possible pathological causes for this appearance.

2. List three possible diagnoses.3. List at least four further tests which would be

useful in confirming the diagnosis.4. The patient was given 120 mg of intravenous

furosemide, with no increase in urine output. What would be the next step in your management?

Page 177: Rapid review of clinical medicine for mrcp part 2

176

The history of continuing haemoptysis and failure of thehaemoglobin to rise sufficiently after blood transfusionstrongly suggest that the chest X-ray appearances aresecondary to pulmonary haemorrhage rather than anyother cause. The presence of normal-sized kidneys is sug -gestive of an acute renal process. In chronic renal disease,the kidney size is usually small, although there are a fewexceptions (Table A). Pulmonary haemorr hage and acuterenal failure are best explained by a systemic vasculitis.The best answers are, therefore, anti-GBM, poly arteritisnodosa or Wegener’s granulomatosis in decreas ingfrequency. Anti-GBM disease is a rare condition whereantibodies to the basement membrane of the glomeruliand alveoli lead to acute nephritis and pulmonaryhaemorrhage, respectively. The disease presents withcough, fatigue, dyspnoea or haemoptysis followed days,weeks or months later, by glomeru lonephritis.

Wegener’s granulomatosis would be the best answer ifthere was additional upper respiratory tract involvement.Although it is also possible, involvement of the upperrespiratory tract is much more common in Wegener’sthan in any of the other vasculitides. SLE is rare in malesand in this age group. Pulmonary haemorrhage is muchless common in SLE than in the other vasculitidesmentioned above. Although Legionnaire’s disease ispossible, there is no history of travel abroad or being inan environment with air conditioning, and it does noteasily explain the pulmonary haemorrhage. Fever is arecognized feature of both infection and the vasculitides.The initial cough productive of yellow-brown sputummay have been secondary to a respiratory tract infectionwhich is of interest because pulmonary haemorrhage ismore common in anti-GBM disease if there has been arecent respiratory infection. Other factors predisposing to

pulmonary haemorrhage in anti-GBM disease aresmoking, coexisting pulmonary oedema and exposure toorganic solvents.

Other conditions which affect the respiratory and renalsystem include IgA nephritis, which occurs followingupper respiratory tract infections. Respiratory involve -ment can occur as an indirect consequence of glomerulo -nephritis if the ensuing renal failure is complicated byfluid overload, producing pulmonary oedema. A raresituation where a primarily renal disorder affects therespiratory system to produce haemoptysis is renal veinthrombosis complicated by pulmonary embolism.

The tests should be based around making the mostprobable diagnosis where two or three diagnoses arepossible. The presence of anti-GBM antibodies will makethe diagnosis of anti-GBM disease, which can be con -firmed on renal biopsy, and immunofluorescence studieswhich will demonstrate IgG deposits around the GBM. A lung biopsy would be dangerous in the presence offrank pulmonary haemorrhage. High titres of serumANCA may support a vasculitic process, particularly if the

Answer 171

1. The chest X-ray reveals bilateral, widespread alveolar shadowing with sparing of the apices.Possible causes of alveolar shadowing on the chest X-ray include:i. Pulmonary oedema.ii. Pulmonary haemorrhage.iii. Pulmonary consolidation.

In this case, pulmonary oedema is unlikely if the apices are spared, and this could only be explained if there was pre-existing, severe bilateral apical emphysema.

2. The patient has a relatively acute illness involving the respiratory and renal system. Possible cause of a systemic illness causing respiratory and renal impairment include:

The vasculitides (in order of decreasing frequency):• Anti-GBM disease or Goodpasture’s syndrome.• Polyarteritis nodosa.• Wegener’s granulomatosis.• Systemic lupus erythematosus.

Infection:• Legionnaire’s disease and secondary interstitial

nephritis.• Pneumococcal pneumonia with mycotic emboli

to the kidneys.

3. i. Anti-GBM antibodies.ii. ANCA.iii. Renal biopsy with immunofluoresence studies.iv. Transbronchial lung biopsy if the patient’s

respiratory status permits.v. Antinuclear factor.vi. Legionella antigen in the urine or Legionella

RMAT on the serum.vii. TLCO.4. Haemodialysis.

Table A Causes of chronic renal impairmentwith normal-sized kidneys

• Myeloma• Diabetes mellitus• Amyloidosis• Polycystic kidney disease (kidneys are usually

huge by the time renal impairment becomesapparent)

Page 178: Rapid review of clinical medicine for mrcp part 2

Clinical Cases 177

anti-GBM antibody is absent. Again, a renal or lungbiopsy with immuno fluoresence should help make thefinal diagnosis. The TLCO is raised in pulmonaryhaemorrhage. It is useful in the assessment ofhaemorrhage in the acute situation because the chest X-ray appearance takes several days to improve, even if thehaemorrhage has ceased. The causes of raised TLCO arelisted (Table B).

If Legionnaire’s disease is suspected, and a rapiddiagnosis is required, the best tests are estimation of theLegionella antigen and the RMAT. Legionella antigen ispresent in the urine of 90% of patients within 48 hours.The RMAT can be per formed on the serum and respira -tory secretions, and a result is available on the same day.Testing for raised levels of Legionella antibodies is pos -sible in this case because the disease has been present fora little while, but if the illness had started in the pre viousweek then this test would not be useful in making a rapiddiagnosis as it can take up to 10 days for the Legionellaantibodies to rise appreciably.

The patient is oliguric, acidotic, has a rapidly rising

creatinine, and has developed hyperkalaemia. Haemo -dialysis is indicated in all of these situations and when thepatient shows evidence of fluid overload. It allows theopportunity for plasmapheresis, which is successful insome patients with anti-GBM disease, particularly withrespect to pulmonary haemorrhage. However, results onthe renal system are not very satisfactory if plasmapheresisis initiated after significant renal impairment has occur -red. Subsequent management will be with high-dosesteroids and cyclophosphamide.

Question 172

An 18-year-old male was admitted following a collapse ata local night club. On examination he was drowsy. Histemperature was 40°C (104°F), and he was sweatingprofusely. His heart rate was 120 beats/min, and regular.The blood pressure was 160/110 mmHg. His pupilswere dilated and reacted poorly to light.

Investigations are shown.

Hb 11 g/dlWCC 11 � 109/lPlatelets 100 � 109/lPT 20 s APTT 70 sTT 16 sSodium 125 mmol/lPotassium 7.4 mmol/lUrea 10 mmol/lCreatinine 123 μmol/lPhosphate 1.7 mmol/lUrine Blood +++

1. What is the underlying cause of his presentation?2. Explain his serum potassium and phosphate levels.3. What is the cause of his abnormal clotting?4. List two therapeutic steps in the management of his

temperature.5. Give two explanations for the hyponatraemia.

Table B Causes of raised TLCO

• Pulmonary haemorrhage• Polycythaemia• Left-to-right shunts• Asthma

Page 179: Rapid review of clinical medicine for mrcp part 2

178

The history is of collapse at a local night club, and onexamination the patient has a very high temperature, issweating profusely, has dilated pupils, and is hypertensive.The blood tests reveal evidence of renal failure with adisproportionately high potassium and phosphate, hint -ing that the patient may have rhabdomyolysis; thehaematological tests suggest underlying DIC. In thiscase, the most probable diagnosis is Ecstasy abuse. Thecase would have been more difficult if he was hypotensivebecause then the differential diagnosis would haveincluded septicaemia, encephalitis or meningitis, all ofwhich must also be borne in mind when attending to ayoung patient with any of these features.

Ecstasy abuse has become an increasingly commonproblem over the past few years, and therefore know -ledge of its recognition and management have becomealmost mandatory. The clinical features are basically thoseof a large concentration of circulating catecholamines inthe blood. In the acute situation it may present ascollapse, convulsions and hyperpyrexia, i.e. a core tem -perature of 39–42°C (102.2–107.6°F) (see Table forcauses of hyperpyrexia). Other features com prise profusesweating, tachycardia, hypertension and dilated pupils.

The blood pressure may also be low due to shockfrom excessive fluid loss secondary to perspiration.Complications of Ecstasy abuse include rhabdomyolysis,acute renal failure (either from reduced renal perfusion ormyoglobin-induced tubular damage), DIC, acute hepati -tis, myocardial infarction and cerebrovascular accident.

Hyponatraemia is a common problem and may be dueto excess sodium loss from the skin during perspiration.The problem is compounded because most Ecstasyabusers are advised to drink large amounts of water whichmay replenish fluid, but not the actual salt loss. Further -more, Ecstasy is thought to cause SIADH which may

exacerbate the hyponatraemia. In some cases the sodiumconcentration may be low enough to precipitate cerebraloedema and intractable seizures.

The management of the patient involves cooling withtepid sponging, and rectal paracetamol. Intravenousdantrolene is useful in controlling the hyperpyrexia.Diazepam is effective in controlling seizures. Fluidreplacement will depend on the sodium concentrationand the CVP. The general rule is to administer 1 litre ofsaline to all patients with a high temperature and lowblood pressure before inserting a central venous catheter,irrespective of the sodium concentration. If the bloodpressure goes up and the heart rate comes down, this isfollowed by another litre. After this, any further fluidreplacement is determined by the CVP and the sodiumconcentration. If the sodium is low and the CVP is low,the patient is infused with normal saline until the CVP isnormal. Following this, the sodium and fluid intake andoutput is observed carefully and fluid restriction isinstituted to help combat possible SIADH. In the patientwhere the sodium is low and the venous pressure ishigh/normal, fluid restriction is instituted for the samereasons; however, if the sodium is low enough to causecoma and seizures, then therapy is with mannitol orhypertonic saline and furosemide. In patients who arehypertensive on admission, fluid replacement should beguided by the CVP from the beginning, and all otherrules are the same as those for patients who are hypoten -sive at presentation.

Causes of hyperpyrexia

• Septicaemia• Viral infections• Malaria• Neuroleptic malignant syndrome• Malignant hyperpyrexia• Ecstasy or other amphetamine abuse• Cocaine abuse• Malignancy• Aspirin toxicity • Prostaglandins

1. ‘Ecstasy’ (3, 4-methylenedioxymetamphetamine; MDMA) abuse.

2. Rhabdomyolysis.3. DIC.4. i. Cooling/tepid sponging.

ii. Intravenous dantrolene.5. i. Syndrome of inappropriate antidiuretic hormone.

ii. Excess sodium loss from skin during profuse perspiration.

Answer 172

Page 180: Rapid review of clinical medicine for mrcp part 2

Clinical Cases 179

Question 173

117733aa 117733bb

1. What is shown on inspection of this patient’s arm pit?

2. Comment on the dexamethasone suppression test.

3. What is the differential diagnosis for the patient’spresentation?

4. Which single biochemical test would you perform to confirm the diagnosis?

5. What is demonstrated on the CT scan of the abdomen?

6. Which other condition does the patient have?7. What is the management of this patient?

A 52-year-old male was investigated for obesity of recentonset, polydipsia and polyuria. On examination, he had ablood pressure of 180/105 mmHg, and was obese.Examination of the arm pit is shown (173a). He under -went routine investigations which are shown.

Hb 14 g/dlWCC 12 � 109/lPlatelets 230 � 109/lSodium 139 mmol/lPotassium 3.2 mmol/lUrea 7 mmol/lBicarbonate 30 mmol/lGlucose 16 mmol/l

Serum cortisol (nmol/l) 9.00 a.m. Midnight940 1,000

After dexamethasone 900 8002 mg q.i.d. for 48 h

Chest X-ray NormalCT scan abdomen (173b)

Page 181: Rapid review of clinical medicine for mrcp part 2

180

Acanthosis nigricans is characterized by velvety hyper pigmen-tation and overgrowth of skin affecting the neck, axilla, groinor the face. The condition may be a normal variant in theAsian race. It is associated with internal malig nancy,particularly carcinoma of the stomach and the lung andlymphoma. It is also associated with a variety of endocrinedisorders causing insulin resistance and hyper androgenicstates (Table A). In this case, it is due to Cush ing’s syndrome(Table B). Complications are shown in Table C.

The high-dose dexamethasone test suppresses theserum cortisol in over 60% of patients with Cushing’s

Answer 173

1. The arm pit demonstrates acanthosis nigricans.2. The high dose of dexamethasone fails to suppress the

elevated cortisol down to normal.3. The differential diagnosis is between ectopic ACTH

and an autonomous glucocorticoid-secreting adrenal tumour (173d).

4. Plasma ACTH level, which will be elevated in ectopic ACTH secretion and suppressed in a cortisol-secreting tumour.

5. The CT scan demonstrates an adrenal tumour on the left side (see also detail, 173c, arrowed).

6. Diabetes mellitus.7. Surgical excision of the affected adrenal gland.

Table B Causes of Cushing’s syndrome

• Pituitary basophil adenoma• Ectopic ACTH secretion• Adrenal adenoma• Adrenal carcinoma• Iatrogenic

Table A Causes of acanthosis nigricans

• Normal variant (Asian race)• Obesity• Carcinoma stomach, lung• Lymphoma• Cushing’s syndrome• Acromegaly• Insulin-resistant diabetes• Polycystic ovary syndrome• Androgen-secreting tumours

117733dd

Pituitary Cushing’s disease

Cortisol-secreting tumourEctopic ACTH secretion

Low dosedexamethasone

0.5 mg four timesdaily for 2 days

High dosedexamethasone2 mg four timesdaily for 2 days

High midnight cortisol

Suppression of cortisol to

normal levels

Failure to suppress cortisol

CheckserumACTH

ObesityAlcohol abuse

Anxiety

DepressionPregnancy

PCOS

Pituitary Cushing’s diseaseCortisol-secreting tumourEctopic ACTH secretion

Elevated

Suppressed

Failure to suppress cortisol

Suppression of cortisol to

normal levels

Ectopic ACTH

Cortisol-secreting adrenal tumour

117733cc

Page 182: Rapid review of clinical medicine for mrcp part 2

Clinical Cases 181

syndrome (bilateral hyperplasia of the adrenal glands dueto autonomous production of ACTH by a pituitarybasophil adenoma); however, nearly 40% of cases fail to

suppress. The serum cortisol in patients with autonomousglucocorticoid-secreting adenomas and those with ectopicACTH production does not suppress to high-dose dexa -methasone. In the normal clinical situation, the differentialdiagnosis between the three conditions, based on thedexamethasone suppression test, is not necessarily resolved;however, in the examination situation it is almost alwaysassumed that suppression with high-dose dexa methasoneexcludes Cushing’s syndrome.

Most tumours are benign; however, in rare instancesadrenal carcinoma may result in Cushing’s syndrome.Malignant tumours are highly aggressive and the prognosisis poor.

Table C Complications of Cushing’s syndrome

• Obesity• Diabetes mellitus• Hypertension• Premature coronary

artery disease• Osteoporosis

• Myopathy• Psychosis• Immunosuppression• Poor wound healing• Hypokalaemia

A 69-year-old male was referred with a five-monthhistory of pains in his shoulders and thighs, night sweatsand weight loss of approximately 3 kg. The pain wasworse in the mornings and during episodes of the nightsweats. Two months previously the patient was seen inthe Accident and Emergency Department for ongoingtesticular pain, but no obvious cause was found.

On examination, he appeared unwell and pale. He wasapyrexial. There was no lymphadenopathy. His handswere examined (174). The heart rate was 104 beats/minand regular, and the blood pressure was 170/100 mmHg.The heart sounds were normal and the chest was clear.Abdominal examination was normal. On examina tion ofthe genitalia, there was testicular tender ness, but noswelling or erythema. There was no inguinal lympha -denopathy. Neurological examination demon stratednormal power, although hip extension was difficult due topainful quadriceps muscles. There was no fasciculation.Both ankle jerks, the left knee jerk and right supinatorreflex were absent. There was loss of light touch sensationin both toes, extending to the medial border of the dorsalaspect of the left foot.

Investigations are shown.

117744Hb 9.5 g/dlWCC 15.1 � 109/lPlatelets 591 � 109/lMCV 90 flESR 104 mm/hCRP 82 g/lSodium 136 mmol/lPotassium 5.3 mmol/lUrea 6.5 mmol/lCreatinine 99 μmol/lCalcium 2.31 mmol/lPhosphate 0.9 mmol/lAlbumin 35 g/lBilirubin 7 μmol/lAST 22 iu/lGamma GT 43 iu/l Alkaline phosphatase 162 iu/lTotal protein 77 g/lGlucose 5.4 mmol/lANF NegativeRheumatoid factor Not detectedImmunoelectrophoresis Polyclonal increase in

gamma-globulin region No paraprotein

Blood cultures � 6 No growthUrinalysis Blood ++

24 hour urine protein 2.4gUrine culture Negative. AAFB not

detected on early morning urine specimens

Chest X-ray Normal

1. Give at least two possible diagnoses.2. List at least three useful investigations.

Question 174

Page 183: Rapid review of clinical medicine for mrcp part 2

182

This is a case where several diagnoses are possible. Thepatient is in his seventh decade and has evidence of generalmalaise, night sweats, weight loss and myalgia, associatedwith a very high ESR (Table C). There is clinical evidenceof peripheral neuropathy. The inflam matory markers areelevated and the possible dif ferential diagnoses in an elderlypatient includes PAN, malignancy (including lymphoma),PMR and sepsis. TB is a possibility; however, a completely

normal chest X-ray makes post-primary infection lessprobable. The main differentials are PAN and PMR.Factors differentiating between PAN and PMR have beentabulated below (Table A). Neuropathy can only beexplained by PAN. The slide reveals livedo reticularis,which is a skin mani festation associated with connectivetissue disease and the vasculitides. It is due to randomspasm of cutaneous arterioles and secondary dilatation ofcapillaries and venules. The presence of livedo reticularisfavours the diagnosis of PAN, which is a non-granulomatous vascu litis affecting medium-sized arteries. Itis more common in males, and is most prevalent in thesixth and seventh decades. It may manifest with myalgia,night sweats and weight loss, and is a recognized cause ofpyrexia of un known origin. The disorder can affect thekidneys, cardiovascular system, gastrointestinal tract andnervous system. Arteritis leads to aneurysm formation, withsubsequent thrombosis and infarction of the suppliedorgans. Clinical features include progressiveglomerulonephritis, myocardial infarction, mesentericischaemia, peripheral neuropathy, CVA and vasculitic areasaffecting the skin. Testicular pain is a recognizedmanifestation of PAN (Table B).

Inflammatory markers are elevated. ANCA may bedetected and lend support for a vasculitic process, but itis not specific for PAN. The diagnosis is made bybiopsying the affected organ or demonstrating micro-aneurysms on angiography. Treatment is with steroids.

Table A Differentiation between PMR and PAN in this case

PMR PANAge 7th decade onwards Middle age onwardsSex Male MaleSymptoms As above As aboveNeuropathy Absent PresentTesticular pain Absent RecognizedLivedo reticularis Absent PresentRaised ESR, CRP Common CommonRaised WCC, alkaline phosphatase Common CommonRaised ANF/Rh factor Absent Uncommon

Table B American College of Rheumatology criteria for PAN

• Weight loss of more than 4 kg• Livedo reticularis• Testicular pain• Myalgia, leg tenderness• Mono/polyneuropathy• Hepatitis B sAg positive• Arterographic abnormality• Positive biopsy

Table C Causes of a very high(>100 mm/h) ESR

• Multiple myeloma• Giant cell arteritis/polymyalgia

rheumatica• Sepsis• Occult malignancy• SLE

1. i. Polyarteritis nodosa. ii. Polymyalgia rheumatica.iii. Lymphoma.iv. Testicular seminoma.v. Post-primary TB.

2. i. Renal biopsy.ii. Skin biopsy.iii. Muscle biopsy may help in the differentiation of

vasculitis from polymyalgia rheumatica.iv. Renal or hepatic angiography to demonstrate

micro-aneurysms.v. Serum ANCA lends support to a vasculitic

process.vi. CT scan thorax/abdomen to check for intra-

abdominal malignancy, splenomegaly and para-aortic lymphadenopathy.

vii. Ultrasound of the testes.viii. Bone marrow examination/culture for AAFB.ix. Mantoux test.

Answer 174

Page 184: Rapid review of clinical medicine for mrcp part 2

Clinical Cases 183

A 45-year-old female on treatment for ulcerative colitiscomplained of increasing tiredness, while her bowelproblems seemed well controlled.

Some of her results are shown.117755

Hb 7.5 g/dlMCV 95 flMCHC 34%WCC 10.5 � 109/lNeutrophils 8.4 � 109/lPlatelets 290 � 109/lReticulocytes 9.8%Blood film (175)

A 26-year-old male presented to the Accident andEmergency Department with a stiff jaw and being unableto open his mouth. Three days previously he wasimmunized with tetanus toxin after lacerating his fingerat work. On examination, he had evidence of ‘lock jaw’.His injured finger was swollen, painful and exuding pus.

Question 175

1. What type of anaemia do these results suggest?a. Iron deficiency anaemia.b. Anaemia of chronic disease.c. Oxidative haemolytic anaemia.d. Immune haemolytic anaemia.e. Anaemia secondary to folic acid deficiency.

2. What is the likely cause?a. Azathioprine-induced folic acid deficiency.b. Chronic blood loss secondary to ulcerative colitis.c. Splenic atrophy.d. Septicaemia secondary to steroid-induced

immunosuppression.e. Sulphasalazine-induced anaemia.

What is the immediate management?a. Debridement and cleansing of the wound.b. Injection of tetanus antitoxin into the wound.c. Oral penicillin V.d. Intramuscular human tetanus immunoglobulin.e. Intravenous pancuronium.

Question 176

Page 185: Rapid review of clinical medicine for mrcp part 2

184

Tetanus is caused by the anaerobic, Gram-positivebacillus Clostridium tetani. The organism producesspores that are commonly found in soil and faeces ofdomestic animals. Soil is the natural habitat for C. tetani.The organism gains entry into the body throughtraumatic wounds, where it multiplies under anaerobicconditions and produces tetanospasmin, a potentneurotoxin. The toxin reaches the spinal cord via theblood or by retrograde axon transport and increasesexcitability in motor neurones by interfering with thefunction of inhibitory neurones. The toxin may alsoproduce overactivity of the sympathetic nervous system.The incubation period after injury is less than 14 daysand can be as early as 2 days.

The dominant features of tetanus are rigidity andreflex spasms. Rigidity may affect any muscle, includingthe jaw muscles, so the mouth cannot be fully opened, asin this case. This is known as ‘trismus’ or ‘lock jaw’.Stiffness may affect the facial muscles, altering facialappearance. Risus sardonicus (pursing of the lips withretraction of the angles of the mouth) is a characteristicfeature of this phenomenon. Pharyngeal involvement mayproduce dysphagia early in the disease. Rigidity of themuscles of the back causes the body to curve backexcessively with the head fully retracted (ophisthotonus).Reflex spasms are a sudden exacerbation of underlyingrigidity which last 1–2 seconds. The time of onset ofspasms from the first symptoms of rigidity is known as

the ‘period time’. This is usually within 72 hours. Theshorter the period time, the worse the prognosis. Largegroups of muscles are affected at any one time.Relaxation may take a few seconds, which is particularlyhazardous when thoracic muscles are involved becauserespiration is impeded until relaxation of the musclesoccurs. Laryngeal spasm is probably the most dangerousevent in tetanus.

Autonomic overactivity has effects on thecardiovascular system. Tachycardia, massive swings inblood pressure and cardiac arrhythmias are recognizedcomplications. Haemodynamic instability may producehypoperfusion of vital organs leading to multi-systemfailure and death. Most deaths are from respiratoryfailure, either due to laryngeal spasms, respiratory musclefatigue and paralysis or pulmonary aspiration.

The mainstay of treatment in an affected individual is togive intramuscular human immunoglobulin immediately.Human immunoglobulin will remove circulating tetano -spasmin from the blood. Fixed neurotoxin has to beremoved from the wound by aggressive cleansing and, ifnecessary, by surgical debridement of the wound. Despitesuch measures the organism may not be entirely removedfrom the wound and added protection with intravenouspenicillin is mandatory. Diazepam is useful in controllingspasms. In severe cases total paralysis with curarization(using intravenous pancuronium) and artificial ventilatorysupport are necessary to reduce mortality from respiratoryfailure.

Prevention is by active immunization and meticulouswound cleansing as soon as possible after trauma.

The patient has a low Hb and a high reticulocyte count,suggesting either haemolysis or recent blood loss. Theblood film is diagnostic. It is stained with methyl violet toreveal small inclusions within red cells termed Heinzbodies, which are essentially precipitated Hb. Theseinclusion bodies are visualized with methyl violet stain.They occur in patients with genetic Hb abnormalities(unstable Hb), or when the iron within the haemmolecule is oxidized from the ferrous state (Fe2+) to theferric state (Fe3+) to produce methaemoglobin. Causes ofmethaemoglobinaemia are discussed in Answer 316. Inthis case, therapy for ulcerative colitis with sulphasalazine

(sulphonamide) is the culprit. Patients takingsulphonamides may develop a chronic haemolyticanaemia owing to the oxidative action of the drugs onthe Hb. Haemolysis is particularly severe in patients withglucose-6-phosphate dehydrogenase deficiency.Management involves drug withdrawal; however, if this isnot possible then a lower dosage should be prescribed,together with iron and folate supplements. Azathioprineis not associated with oxidative haemolytic anaemia.Splenic atrophy is also associated with inclusions withinred cells termed Howell–Jolly bodies, which representparticles of denatured DNA. They are visible when theblood film is stained with Wright’s stain. In contrast,Heinz bodies cannot be seen when stained with Wright’sstain because they are the same colour as the red cell.

1. c. Oxidative haemolytic anaemia.2. e. Sulphasalazine-induced anaemia.

1. d. Intramuscular human tetanus immunoglobulin.

Answer 175

Answer 176

Page 186: Rapid review of clinical medicine for mrcp part 2

Clinical Cases 185

Question 177

A 33-year-old Indian male presented with a four-weekhistory of a painful swollen knee, which he attributed to afall during a game of tennis. The knee was painful afterthe fall, but over the next two weeks he developedswelling and increasing pain which was only partiallyrelieved by paracetamol. He had never had a similarproblem in the past. Apart from feeling slightly morebreathless than usual while playing tennis over the pastthree months, he had no other complaints. There was nosignificant family history except that his grandparents hadboth had TB. He could not ever recall having had a BCGvaccine, but did have several chest X-rays in his earlyteenage years. On examination, he appeared well. He hada temperature of 37.5°C (99.5°F). Examination of hisupper arms did not reveal a BCG scar. Examination of theleft knee demonstrated a painful erythematous joint witha large synovial effusion. On examination of therespiratory system there were some harsh breath sounds inanterior aspect of the chest. The attending rheumatologistrequested some investigations, which are shown.

On the basis of these investigations, the patient wascommenced on empirical antituberculous therapy. He wasreviewed four months later and had an ESR check, whichmeasured 57 mm/h. The swelling in the left knee hadsubsided, but the changes on the chest X-ray persisted.Six months later he presented to the Accident andEmergency Department with swelling in his right kneeand both ankles. In addition, he complained of pain inthe cervical area and lower back pain that radiated intothe buttocks and was worse in the mornings.

Hb 12 g/dlWCC 7 � 109/lPlatelets 308 � 109/lESR 107 mm/hChest X-ray (177a)Rheumatoid factor NegativeX-ray of left knee Soft-tissue swelling

No bony involvementAspirate of Yellow fluid; low viscosity;

knee joint WCC 50/mm3; culture sterile

Synovial biopsy Thickened synovium macroscopically with histological evidence ofincreased inflammatory infiltrate

1. What is the diagnosis?2. How would you explain the changes on the

chest X-ray?3. Give two diagnostic tests.4. Apart from diagnostic tests, which other

investigation would you perform to help his further management?

5. What is the management of his condition?

117777aa

Page 187: Rapid review of clinical medicine for mrcp part 2

186

While the inflamed knee joint in association with theabnormal chest X-ray may be highly suggestive of TB inan Asian patient, the failure to improve with anti-tuberculous therapy is against this diagnosis. In a youngmale with a synovitis of the knee joint followed by pain inthe ankle joints and in the lumbar and cervical area, thedifferential diagnosis is between rheumatoid arthritis,adult Still’s disease and a seronegative arthropathy,particularly ankylosing spondylitis. The absence of therheumatoid factor is against the diagnosis of rheumatoidarthritis. Still’s disease is not sufficient to explain thechest X-ray abnormalities, which in this case are due to

apical fibrosis (177a) (Table A lists other causes), a well-recognized complication of ankylosing spondylitis. Othercomplications of ankylosing spondylitis are listed below(Table B). In addition, there is an increased incidence ofscleritis and scleromalacia.

Ankylosing spondylitis is a chronic inflammatorydisorder that predominantly affects the spinal joints. It isstrongly associated with the presence of HLA B27, whichis present in 95% of all cases, and the disease is moreprevalent in males in their second and third decades. Theexact aetiology of the disorder is unknown. Most patientspresent with a sacroiliitis causing back pain and stiffness,which is worse in the mornings (Table C). There is acharacteristic loss of lumbar lordosis and a reduced abilityto flex the spine. An asymmetric peripheral large jointarthropathy may be the presenting feature in somepatients – as in this case. Enthesopathy causes pain in thetendons, particularly the Achilles tendon. Weight loss,low-grade fever and high inflammatory markers in theblood are recognized. Extra-articular complications occurin approximately 15% of patients. The diagnosis is aclinical one, but is helped by the demonstration ofsacroili itis on the X-ray of the pelvis and thedemonstration of thickening and calcification of theinterspinous ligament, which may give the appearance ofa bamboo spine (177b).

Table B Ankylosingspondylitis is easilyremembered as the ‘A’disease

• Arthritis• Atlano axial subluxation• Arachnoiditis (spinal)• Apical fibrosis• Anterior uveitis• Aortitis• Aortic regurgitation• Atrioventricular block• Amyloidosis• IgA nephropathy• Achilles tendinitis• Plantar fAsciitis

Table A Causes of apicallung fibrosis

• TB• Extrinsic allergic alveolitis• Ankylosing spondylitis• Allergic bronchopulmonary

aspergillosis• Radiation• Sarcoidosis• Histiocytosis X

Table C Causes ofsacroiliitis

• Ankylosing spondylitis• Reiter’s syndrome• Psoriatic arthropathy• Enteropathic arthritis

(inflammatory boweldisease)

• Whipple’s disease• Osteoarthritis

1. Ankylosing spondylitis.2. Bilateral apical fibrosis.3. i. X-ray of the sacroiliac joints, which will

demonstrate sacroiliitis and may demonstrate squaring of the lumbar vertebrae with thickening and calcification of the interspinous ligaments.

ii. HLA typing for B27; however, one must remember that this is found in 5% of the general population.

4. Respiratory function tests given the history of breathlessness on exertion and the lung fibrosis.

5. NSAID and physiotherapy.

117777bb

Answer 177

Page 188: Rapid review of clinical medicine for mrcp part 2

Clinical Cases 187

A 21-year-old female developed a sudden left hemi paresis followed by loss ofvision in her left eye 24 hours afterwards. For three months she had feltgenerally l istless, experienced frequent night sweats and had lost 4 kg in weight. Over the past two weeks she had become breathless on exertionand could not sleep unless she was propped up on three pillows. She lostconsciousness on two occasions and was seen in the local Accident andEmergency Department with a suspected epileptic seizure three days earlier, butwas not admitted. The medical officer had arranged an out-patient appointmentwith a cardiologist because he had heard an interesting murmur. The patient hadenjoyed a healthy life before this illness and had never travelled abroad.

On examination, she was unwell. The temperature was 37.6°C (99.7°F). Shehad a heart rate of 110 beats/min, which was regular, and the blood pressurewas 90/60 mmHg. The JVP was elevated 4 cm above the sternal angle. Thecardiac apex was not displaced. On auscultation, the heartsounds were normal but there was a soft mid-diastolicmurmur in the mitral area, as well as a mitral regurgitantmurmur. Auscultation of the lung fields revealed fineinspiratory crackles at both lung bases. Neurologicalexamination demonstrated a left hemi paresis. The speechwas intact. Examination of the left fundus is shown(178a). Investigations are shown.

117788bb

Hb 10 g/dlWCC 8 � 109/lPlatelets 360 � 109/lESR 90 mm/hSodium 135 mmol/lPotassium 4.1 mmol/lUrea 8 mmol/lGlucose 5 mmol/lECG (178b)Chest X-ray Cardiac silhouette slightly enlarged

Upper lobe vein distension and bibasal interstitial lung shadowing

Question 179

A 15-year-old male is admitted for parathyroidectomy.His father had a parathyroidectomy 20 years previously.

Pre-operative blood and post-operative blood (twomonths later) results are shown.

1. What is the diagnosis?a. Mitral valve

endocarditis.b. Left atrial

myxoma.c. Systemic lupus

erythematosus.d. Polyarteritis

nodosa.e. Sarcoidosis.

2. What investigation is required to confirm the diagnosis?a. Serial blood

cultures.b. Serum ANCA.c. Double stranded

DNA antibodies.d. Coronary

angiography.e. Echocardiography.

1. What is the diagnosis?2. How would you confirm the diagnosis?

117788aa

Question 178

Pre-operative: Calcium 2.8 mmol/lPhosphate 0.8 mmol/lAlbumin 44 g/lPTH 40 ng/l

Post-operative: Calcium 2.9 mmol/lPhosphate 0.9 mmol/lAlbumin 43 g/lPTH Undetectable

Page 189: Rapid review of clinical medicine for mrcp part 2

188

The patient presents with a fever, systemic illness, amurmur and retinal artery occlusion. The differentialdiagnosis is between atrial myxoma and endocarditisaffecting a stenosed mitral valve. Both may present inidentical fashion and may be associated with left atrialenlargement and right ventricular hypertrophy, as seenon the 12-lead ECG. In this particular case mitralstenosis is less likely for three main reasons: (i) there is nopast history of rheumatic fever to account for mitralstenosis (congenital mitral stenosis is rare and would havepresented at a much younger age); (ii) endocarditis is rarein mitral stenosis owing to the low pressure proximal tothe valve surface; and (iii) it is unusual for a patient withmitral stenosis to be in sinus rhythm in the presence ofsymptomatic mitral stenosis. Left atrial myxoma (178c)would explain the dyspnoea, constitutional upset, fever,cardiac signs and the cerebral emboli.

Atrial myxomata are the most common tumours of theheart, with 75% being found in the left atrium. Thetumours are benign and characteristically attached to thefossa ovalis by a pedicle. They are highly mobile andprolapse to and fro through the mitral valve orifice. Themovement through the valve produces an added sound indiastole known as a ‘tumour plop’, which sounds like athird heart sound, and a mid-diastolic murmur, which

may be confused with mitral stenosis. However, thirdheart sounds do not occur in mitral stenosis.Breathlessness owing to increased left atrial pressure iscommon. Mechanical obstruction of the mitral valveorifice may cause syncope or even sudden death.Constitutional upset and pyrexia are common, as areraised inflammatory markers and hypergamma-globulinaemia. Haemolysis is also recognized. Systemicemboli from the gelatinous tumour may complicate atrialmyxoma. Arrhythmias, including atrial fibrillation, areuncommon. The diagnosis is made by echocardiographyand the management is surgical excision.

117788cc1. b. Left atrial myxoma.2. e. Echocardiography.

Familial benign hypocalcuric hypercalcaemia is anautosomal dominant disorder which is characterized bylife-long, asymptomatic hypercalcaemia. However, somepatients develop acute pancreatitis, gallstones andchondrocalcinosis. The abnormality is in the calcium-sensing receptor, a G-protein-coupled receptor. It may bedifficult to distinguish from primary hyperparathyroidism,and in 20% of cases the PTH is mildly elevated. The serumphosphate values are usually normal or slightly reduced. Incontrast to other causes of hypercalcaemia, the 24-hour

urinary calcium is reduced; 75% of patients have values<2.5 mmol/l, and 95% have values <5 mmol/l. The serumcalcium level may vary from mild elevation to markedhypercalcaemia with values exceeding 3.5 mmol/l. TheCaCl/CrCl ratio can be used to improve differentiationbetween hyperparathyroidism and familial benignhypocalcuric hypercalcaemia:

CaCl/CrCl = UrCa � P Cr/P Ca � UrCr

Some 80% of patients with familial benignhypocalcuric hypercalcaemia have a ratio of <0.01,whereas more than 70% of those with primaryhyperparathyroidism have values above this cut-off.

The management is conservative.

1. Familial benign hypocalcuric hypercalcaemia.2. 24-hour urinary calcium estimation or a urinary

calcium clearance to creatinine clearance (CaCl/CrCl) ratio.

Answer 178

Answer 179

Page 190: Rapid review of clinical medicine for mrcp part 2

Clinical Cases 189

A 16-year-old female was investigated for lethargy andpolyuria. Her blood pressure measured 116/70 mmHg.

Investigations are shown.

Question 181

Hb 13 g/dlWCC 7 � 109/lPlatelets 300 � 109/lSodium 129 mmol/lPotassium 3.0 mmol/lUrea 7 mmol/lCreatinine 80 �mol/lBicarbonate 33 mol/lCalcium 2.0 mmol/lPhosphate 1.3 mmol/lAlbumin 40 g/lGlucose 4 mmol/l24-hour urinalysis:

Sodium 30 mmol/l (<20 mmol/l in hyponatraemia)Potassium 40 mmol/l (<10 mmo/l in presence of hypokalaemia)Calcium 10.6 mmol (NR <7.5 mmol/24h)Chloride 42 mmol/l (NR <25 mmol/l)

Serum renin 960 pmol/l (NR 100–500 pmol/l)

What is the diagnosis?a. Laxative abuse.b. Addison’s disease.c. Thiazide diuretic abuse.d. Self-induced vomiting.e. Bartter’s syndrome.

A 43-year-old male was admitted with a 2-hour historyof malaise and nausea. A 12-lead ECG revealed an acuteinferior myocardial infarction and third degree AV block.He had type 1 diabetes mellitus. The heart rate at restwas 34 beats/min. The blood pressure was80/50 mmHg. Both heart sounds were normal. Atemporary pacing wire was inserted via the rightsuperficial femoral vein and the patient was successfullypaced. The BP improved to 90/60 mmHg.

Blood results were as follows:

Question 180

Hb 13 g/dlWCC 17 � 109/lPlatelets 380 � 109/lSodium 140 mmol/lPotassium 4.2 mmol/lUrea 8 mmol/lCreatinine 126 �mol/lGlucose 17 mmol/l

What is the next management step?a. IV heparin.b. IV recombinant tissue plasminogen activator.c. IV metoprolol.d. IV streptokinase.e. IV glyceryl trinitrate.

Page 191: Rapid review of clinical medicine for mrcp part 2

190

The patient has hypokalaemia and metabolic alkalosisassociated with a normal blood pressure. The differentialdiagnosis is between self-induced vomiting, chronicthiazide diuretic use (or abuse) and Bartter’s syndrome.Although laxative abuse can also be associated withhyponatraemia and hypokalaemia, most patients whoabuse laxatives have a metabolic acidosis rather thanalkalosis (loss of bicarbonate via the gastrointestinaltract). Addison’s disease is usually associated withhypotension, hyperkalaemia, reduced potassiumexcretion in the urine and metabolic acidosis.

The differentiation between thiazide drug abuse, self-induced vomiting and Bartter’s syndrome is dependentupon the history and the urinary electrolytes. Patientswho lose sodium or potassium via the gastrointestinaltract usually have low urinary sodium (<10 mmol/l) andpotassium (<20 mmol/l) levels. The patient in questionhas inappropriately high urinary sodium and potassium,indicating primary renal tubular dysfunction or lack ofmineralocorticoid. Mineralocorticoid deficiency ischaracterized by low serum sodium, high serumpotassium and metabolic acidosis.

The remaining differential diagnosis is betweenthiazide diuretic abuse and Bartter’s syndrome. Thiazidesmay cause all of the biochemical abnormalities in theblood shown in the question. Raised plasma renin is dueto hypovolaemia or hyponatraemia. Furthermorethiazides are also associated with high urine sodium,potassium and chloride levels. However, urinary calciumis low in patients taking thiazide diuretics since thiazidesincrease absorption of calcium in the distal convoluted

tubule. Chronic frusemide abuse can cause a metabolicsyndrome identical to Bartter’s syndrome. Unlikethiazide diuretics, loop diuretics promote calciumexcretion by the kidneys by inhibiting sodium re -absorption in the loop of Henle.

Bartter’s syndrome is a rare syndrome with acharacteristic set of metabolic abnormalities that includehypokalaemia, metabolic alkalosis, hyponatraemia,hyperaldosteronism, and hyperplasia of the juxta -glomerular apparatus (source of renin in the kidney).Most cases present early in life and may be associatedwith short stature and mental retardation. The conditionis due to abnormal sodium concentration ability in theloop of Henle. This results in increased sodium andchloride excretion via the kidney. The resultinghyponatraemia that is associated with hyperreninaemiaand hyperaldosteronism causes hypokalaemia andmetabolic alkalosis. Because urinary calcium reabsorptionin the loop of Henle is dependent on sodium absorption,urinary calcium excretion is also increased. Similarlymagnesium reabsorption is impaired and may result inhypomagnesaemia. Urinary calcium is typically>40 mmol/l. In relation to the differential diagnosisabove, self-induced vomiting is associated with lowurinary calcium levels (<25 mmol/l).

Prostaglandin levels are raised in Bartter’s syndrome,which may explain why these patients are normotensive.Treatment is aimed at minimizing the effects ofprostaglandins and aldosterone. Non-steroidal anti-inflammatory drugs and high dose spironolactone oramiloride are the drugs of choice. Magnesiumsupplements may also be required in those patients withhypomagnesaemia.

There is good evidence that the patient should eitherundergo primary coronary angioplasty or receivethrombolysis to reduce his immediate risk of suddendeath. The angioplasty option is not provided, thereforethe candidate is left to decide between streptokinase and

recombinant TPA. Streptokinase carries the risk of anallergic/anaphylactic reaction, which may compromisethe blood pressure further. Streptokinase should be usedwith caution in a patient with hypotension. RecombinantTPA is not usually associated with such reactions and ismarginally superior to streptokinase in achieving vesselpatency.

Answer 180

b. IV recombinant tissue plasminogen activator.

Answer 181

E. Bartter’s syndrome.

Page 192: Rapid review of clinical medicine for mrcp part 2

Clinical Cases 191

A 60-year-old obese male was referred to agastroenterologist with a 4-month history of right upper

quadrant pain and abnormal liver function tests. He didnot have nausea, vomiting or steatorrhoea. He consumed24 units of alcohol per week. He had an 8-year history ofnon-insulin-dependent diabetes mellitus and had beendiagnosed as having hypertension 1 year previously. Hehad chronic pain in both knees that was attributed toosteoarthritis.

On examination he was obese. He measured 1.8 mand weighed 105 kg. He was not jaundiced and therewere no peripheral stigmata of chronic liver disease. Theheart rate was 70 beats/min and regular. The bloodpressure measured 150/90 mmHg. Both heart soundswere normal and the chest was clear. Abdominalexamination revealed a palpable liver edge 4 cm belowthe costal margin. The spleen was not palpable.

Investigations are shown.

Question 182

Hb 14 g/dlWCC 6 � 109/lPlatelets 200 � 109/lSodium 136 mmol/lPotassium 4.1 mmol/lUrea 6 mmol/lCreatinine 100 �mol/lAST 60 iu/lALT 78 iu/lAlkaline phosphatase 180 iu/lBilirubin 12 μmol/lAlbumin 38 g/lTotal cholesterol 7.2 mmol/lTriglyceride 6.1 mmol/lBlood glucose 13 mmol/lSerum ferritin 400 �g/lSerum iron 20 �mol/lTIBC 60 �mol/lIgG 22 g/lIgA 4.8 g/lIgM 3.1 g/lHep B sAg Not detectedHCV antibodies Not detected

What is the most probable diagnosis?a. Autoimmune hepatitis.b. Non-alcoholic steatohepatitis.c. Primary biliary cirrhosis.d. Haemochromatosis.e. Wilson’s disease.

This is a chest X-ray (183) of a 70-year-old Indian femalewho presented with dyspnoea and a low-grade fever.

Question 183

What is the next most important management step?a. Commence IV cefuroxime and oral

clarithromycin.b. Arrange urgent bronchoscopy.c. Commence antituberculous treatment.d. Aspirate left-sided pleural effusion and send fluid

for microscopy and culture.e. Start intravenous diuretics.

118833

Page 193: Rapid review of clinical medicine for mrcp part 2

192

The patient is obese. He has non-insulin-dependentdiabetes mellitus, hypertension and mixed hyper -lipidaemia. The findings are compatible with insulinresistance or metabolic syndrome. Additionally, thepatient has right upper quadrant pain, hepatomegaly anda transaminitis in the absence of alcohol abuse, specificdrug therapy or hepatitis B or C infection. The mostprobable diagnosis is non-alcoholic steatohepatitis.

NASH is associated with obesity and the metabolicsyndrome. It is characterized by fatty infiltration of theliver associated with hepatitis or even cirrhosis in theabsence of alcohol abuse, specific drug therapy(amiodarone, tamoxifen, oestrogens) or chronic viralhepatitis. Liver biopsy is identical to alcoholic hepatitis.Both AST and ALT are raised. The AST/ALT ratio isusually <1, unlike in alcohol abuse where the ratio isusually >2. Alkaline phosphatase is only modestly raised

but hyperbilirubinaemia is rare. Serum ferritin is raised asin certain other chronic liver disorders such as alcoholabuse and chronic viral hepatitis. The diagnosis of NASHis confirmed by liver biopsy (Table). Patients with alcoholabuse or chronic hepatitis B/C infection should not bediagnosed as having NASH. Other differential diagnosesare tabulated below.

The condition is more common in females. Mostpatients present with abnormal liver function testsalthough malaise and right upper quadrant pain arerecognized presenting features. Hepatomegaly iscommon. Insulin resistance is thought to play a primaryrole in NASH.

The prognosis is relatively benign although somepatients, particularly elderly diabetic females, may developcirrhosis. There is no specific treatment but gradualweight loss has been shown to improve liver function.The role of insulin sensitizing drugs such as metformin isnot well established. Treatment with β1 cannabanoidreceptor blockers appears promising.

Answer 182

b. Non-alcoholic steatohepatitis.

Differential diagnosis of NASH

Condition Differentiating features*Alcohol-related liver disease History of alcohol abuse/AST:ALT >2Chronic viral hepatitis (B or C) VirologyAuto-immune hepatitis Autoantibody screen shows raised SMA or ANA

Hypergammaglobulinaemia is also a featurePrimary biliary cirrhosis Predominant cholestasis on blood tests and positive antimitochondrial

antibodiesHaemochromatosis Serum TIBC >90%; ferritin >300Primary sclerosing cholangitis History of inflammatory bowel disease

Predominantly cholestatic picture

HypergammaglobulinaemiaDrugs History

*In the absence of liver biopsy

There is a large area of opacification affecting the leftlung without any visible lung markings, and markedmediastinal shift to the right. The diagnosis is consistent

with a large pleural effusion. Given her ethnicity it ispossible that this may represent a tuberculous pleuraleffusion, although a malignant pleural effusion cannot beexcluded by the history. The next management stepwould be to perform a diagnostic/therapeutic pleuralaspirate before starting any antibiotic treatment.

Answer 183

d. Aspirate left-sided pleural effusion and send fluid for microscopy and culture.

Page 194: Rapid review of clinical medicine for mrcp part 2

Clinical Cases 193

A 24-year-old man presented with sudden onset ofpalpitations while walking to work. He had no pastmedical history of note. He consumed two units ofalcohol per day. He did not smoke or abuse illicit drugs.

On examination his heart rate was 132 beats/min andirregular. The blood pressure measured 105/70 mmHg.Both heart sounds were normal and his chest was clear.

Investigations are shown.

Question 184

Hb 15 g/dlWCC 5 � 109/lSodium 138 mmol/lPotassium 3.8 mmol/lBlood glucose 4.2 mmol/lSerum TSH 2 mu/lECG Atrial fibrillation with

a ventricular rate of 132 beats/min but no other abnormality

Chest X-ray NormalEchocardiogram Normal study

What is the most effective drug in restoring sinusrhythm quickly in this patient?

a. IV digoxin.b. IV flecanide infusion.c. IV esmolol.d. Oral sotalol.e. IV amiodarone.

An 18-year-old male navy cadet presented with a 2-dayhistory of headache, fever and sore throat. On exam -ination he was noted to have cervical lymphadenopathy.His heart rate was 80 beats/min and BP was170/100 mmHg. There was a soft systolic murmur atthe left lower sternal edge, which did not radiate. Allother examination was normal.

Investigations are shown.

He was treated with antibiotics for one week and hemade an unremarkable recovery. Review in the out-patients clinic following this revealed the followingresults:

Question 185

Hb 13.6 g/dlWCC 12.0 � 109/lPlatelets 180 � 109/lESR 36 mm/hSodium 136 mmol/lPotassium 4.8 mmol/lUrea 7.3mmol/lCreatinine 143 �mol/lBlood cultures No growthThroat swab culture No growth Urinalysis 10–20 rbc ++,

granular casts, no growth

Hb 14.1 g/dlWCC 8.2 � 109/lPlatelets 191 � 109/lSodium 136 mmol/lPotassium 4.1 mmol/lUrea 5.1 mmol/lCreatinine 92 �mol/lANA and ds DNA binding Negative Urinalysis Blood ++

Protein ++

What diagnosis would you consider most likely?a. Post-streptococcal glomerulonephritis.b. Berger’s nephritis.c. Infective endocarditis.d. Henoch–Schönlein purpura.e. Focal segmental glomerulonephritis.

Page 195: Rapid review of clinical medicine for mrcp part 2

194

The patient has atrial fibrillation (AF) that is definitely lessthan 48 hours in duration. In such patients sinus rhythmcan be safely restored immediately without prioranticoagulation if the patient does not have left ventriculardysfunction or rheumatic mitral valve disease (see below).

In patients with haemodynamic embarrassment due toatrial fibrillation with a rapid ventricular rate thetreatment of choice is urgent electrical cardioversion, as itis more effective than pharmacological therapy atrestoring sinus rhythm.

In patients who are haemodynamically stable and donot have dilated cardiomyopathy or suspicion ofrheumatic mitral valve disease, either electricalcardioversion or anti-arrhythmic drugs may be used torestore sinus rhythm. The drugs commonly used are classI and class III anti-arrhythmic agents. The class III anti-arrhythmic agents commonly used are sotalol andamiodarone. Recent evidence suggests that sotalol is notsuperior to conventional beta-blockers at restoring sinusrhythm. Amiodarone is not as effective as some of theclass I anti-arrhythmic drugs and should be reserved foruse in patients with underlying cardiomyopathy.

Commonly used class I anti-arrhythmic agents used torestore sinus rhythm in atrial fibrillation include flecanide,propafenone and dofetilide. Of these, flecanide is themost effective agent at restoring sinus rhythm early and iseffective in 72–95% of cases. Flecanide is avoided inpatients who have any form of cardiomyopathy or haveatrial fibrillation in the context of a myocardial infarction.

Digoxin is not effective at restoring sinus rhythm, butplays an important role in controlling ventricular ratewhen given with beta-blockers or calcium antagonists

with AV nodal blocking properties such as verapamil ordiltiazem. Beta-blockers are less effective than flecanide atrestoring sinus rhythm.

Anticoagulation in atrial fibrillationPatients with AF associated with haemodynamic collapseshould have electrical cardioversion irrespective of prioranticoagulation. Patients with stable AF of <48 hoursduration may undergo electrical or pharmacologicalcardioversion without prior anticoagulation if they do nothave any evidence of left ventricular dysfunction orrheumatic mitral valve disease on echocardiography. Patientswith left ventricular dysfunction or rheumatic mitral valvedisease should have TOE prior to restoring sinus rhythm.

Patients who have been in stable atrial fibrillation for>48 hours should undergo transoesophagealechocardiography to exclude thrombus in the left atrialappendage before restoring sinus rhythm. If a thrombusis identified, the patient should be anticoagulated for atleast three weeks, following which a TOE should berepeated to ensure resolution of thrombus beforeattempting restoration of sinus rhythm. In the absence oftransoesophageal echocardiography the patient should beanticoagulated for at least three weeks before safelyattempting to restore sinus rhythm.

After successful restoration of sinus rhythm the patientsshould continue anticoagulation for four weeks.Anticoagulation may be stopped after four weeks if thepatient remains in sinus rhythm and does not have riskfactors for further episodes of atrial fibrillation (such as leftventricular dysfunction or rheumatic mitral valve disease).Patients who have suffered prior systemic thrombo -embolism secondary to AF should continue antico -agulation despite restoration of sinus rhythm, as onecannot predict if the patient will revert to atrial fibrillation.

Answer 184

b. IV flecanide infusion.

There is a short history of upper respiratory tract infectionwith transient deterioration in renal function and persistentproteinuria and microscopic haematuria. The most probablediagnosis is Berger’s nephritis (IgA nephritis). The history isnot long enough for classic post-streptococcalglomerulonephritis, which usually occurs 10–14 days afterthe initial respiratory infection. Apart from the nephritisthere is nothing to suggest Henoch–Schönlein vasculitissuch as a rash, arthralgia or abdominal symptoms. Althoughthere is a soft murmur at the lower sternal edge, thepreceding symptoms of an upper respiratory tract infectionfavour IgA nephritis rather than nephritis complicatinginfective endocarditis. Treatment is not usually required.

Proteinuria exceeding 1g per 24 hours or the persistence ofhaematuria would be an indication for renal biopsy toexclude another cause of nephritis.

Patients with chronic proteinuria following IgAnephropathy should receive ACE inhibitors or AIIRBs.There is some evidence currently that the addition of anAIIRB to ACE inhibitor provides additionalantiproteinuric effects. The blood pressure should bemanaged aggressively. Steroids and other immuo -suppressive therapy are reserved for patients with nephriticsyndrome or an active and persistent nephritis.

Apart from being secondary to viral illness, IgAnephritis has also been associated with cirrhosis of the liver,coeliac disease, HIV infection, certain bacterial infections(Staphylococcus aureus) and Wegener’s granulomatosis. Afamilial form of IgA nephritis is also recognized.

Answer 185

b. Berger’s nephritis.

Page 196: Rapid review of clinical medicine for mrcp part 2

Clinical Cases 195

A 45-year-old man was involved in a motorbike accidentresulting in a fractured femur. He underwent internal

fixation of the hip. While in theatre the temperature roseto 39°C (102.2°F). The heart rate was 140 beats/min.There was evidence of hypertonia. Blood results were asfollows:

Question 186

Arterial blood gases:pH 7.3PCO2 6.2 kPaPO2 12 kPaBicarbonate 18 mmo/l

Sodium 136 mmol/lPotassium 7.1 mmol/lUrea 7 mmol/lCreatinine 130 �mol/l

What is the immediate management step?a. Stop anaesthesia.b. IV dantrolene.c. Haemodialysis.d. IV bromocryptine.e. IV calcium gluconate.

A 70-year-old male who presented with anterior chestwall pain had a chest X-ray (187).

Question 187

What is the diagnosis?a. Cannon ball metastases.b. Multiple myeloma.c. Bony metastases.d. Left lower lobe collapse.e. Fractured ribs on the right side.

A 69-year-old woman attended her GP complaining oflethargy and pruritus. She also complained of difficultyon climbing stairs because she felt her thighs could notcarry her. She had always led a very healthy lifestyle andwas not taking any medications. On examination sheappeared slightly icteric and had xanthelasma. There were

several scratch marks on the skin. The heart rate,temperature and blood pressure were normal. The liverwas palpable 3 cm below the costal margin and thespleen could just be felt. There was no evidence oflymphadenopathy.

Investigations are shown.

Question 188

Hb 11 g/dlWCC 6 � 109/lPlatelets 120 � 109/lSodium 133 mmol/lPotassium 4.1 mmol/lUrea 3 mmol/lCreatinine 110 �mol/lBilirubin 56 mmol/lAST 58 iu/lAlkaline phosphatase 230 iu/lTotal protein 70 g/lAlbumin 32 g/l

Give two explanations for the raised alkaline phosphatase.a. Cholestasis.b. Haemolysis.c. Bony metastases.d. Hypothyroidism.e. Osteomalacia.f. Osteoporosis.g. Hepatic metastases.h. Choriocarcinoma.i. Paget’s disease.j. Hepatitis.

118877

Page 197: Rapid review of clinical medicine for mrcp part 2

196

Hyperpyrexia, hypertonia and hyperkalaemia duringanaesthesia raise the possibility of malignant hyper -pyrexia.

Malignant hyperthermia is a rare genetic disorder,which manifests following treatment with anaestheticagents, most commonly succinylcholine and halothane.The onset of malignant hyperthermia is usually within one hour of the administration of general anaesthesia butrarely may be delayed as long as 11 hours. 50% of casesare inherited in an autosomal dominant fashion.Susceptible patients with autosomal dominant diseasehave any one of eight distinct mutations in the ryanodinereceptor calcium channel receptor found in thesarcoplasmic reticulum of skeletal muscle.

In the presence of anaesthetic agents, alterations in thehydrophilic, amino-terminal portion of the ryanodinereceptor result in uncontrolled efflux of calcium from thesarcoplasmic reticulum with subsequent tetany, increasedskeletal muscle metabolism, and heat production.

Early clinical findings in malignant hyperthermiainclude muscle rigidity (especially masseter stiffness),sinus tachycardia, increased CO2 production, and skincyanosis with mottling. Marked hyperthermia (up to45°C [113°F]) occurs minutes to hours later; core bodytemperature tends to rise 1°C every 5 to 60 minutes.Hypotension, complex dysrhythmias, rhabdomyolysis,electrolyte abnormalities, disseminated intravascularcoagulation, and mixed acidosis commonly accompanythe elevated temperature. Rectal temperature should bedetermined in all patients. Abnormalities of vital signs insevere hyperthermia include sinus tachycardia,tachypnoea, a widened pulse pressure, and hypotension.

The diagnosis of malignant hyperthermia can beconfirmed by an in vitro muscle contracture testfollowing recovery from the acute hyperthermic episode.Abnormal augmentation of in vitro muscle contractionfollowing treatment with halothane or caffeine isdiagnostic of the disorder.

Management of malignant hyperthermia requiresimmediate cessation of any triggering anaesthetic agent,ensuring adequate airway, breathing, and circulation;initiation of rapid cooling; and treatment ofcomplications such as DIC and rhabdomyolysis. Alpha-adrenergic agonists should be avoided, since the resultantvasoconstriction decreases heat dissipation. Continuouscore temperature monitoring with a rectal or oesophagealprobe is mandatory, and cooling measures should bestopped once a temperature of 39.5°C (103°F) has beenachieved.

Dantrolene administration is the mainstay oftreatment of malignant hyperthermia, and should beinitiated as soon as the diagnosis is suspected. Since theintroduction of dantrolene, the mortality of thefulminant syndrome has fallen from close to 70% to lessthan 10%. Dantrolene is a non-specific skeletal musclerelaxant that acts by blocking the release of calcium fromthe sarcoplasmic reticulum. This, in turn, decreases themyoplasmic concentration of free calcium and diminishesthe myocyte hypermetabolism that causes clinicalsymptoms. The drug is most effective when given early inthe illness (i.e. before hyperthermia occurs), whenmaximal calcium can be retained within the sarcoplasmicreticulum.

Although the patient also has dangerous hyper -kalaemia and would normally also receive intravenouscalcium gluconate to reduce the risk of cardiac arrest, thepriority is to stop the causal anaesthetic agent first.

Answer 186

a. Stop anaesthesia.

The chest X-ray shows multiple sclerotic areas affectingthe ribs. These areas are characterized by loss ofdistinction between the cortex and the medulla. Thecommonest cause of sclerotic metastases in a male patientis carcinoma of the prostate.

Answer 187

c. Bony metastases.

The patient has primary biliary cirrhosis. Vitamin Ddeficiency is a well recognised complication.

Answer 188

a. Cholestasis.e. Osteomalacia.

Page 198: Rapid review of clinical medicine for mrcp part 2

Clinical Cases 197

A 66-year-old white female slipped and fell on an icywalkway. When seen in the Accident and EmergencyDepartment she complained of pain in the left hip. Herleft leg was shortened and externally rotated. X-raysconfirmed a diagnosis of fractured neck of the femur, forwhich she underwent arthroplasty. Two days aftersurgery, she complained of difficulty breathing. On

examination her heart rate was 110 beats/min,respiratory rate was 20/min, and BP was 90/50 mmHg.The JVP was elevated and there was a right parasternalheave. Auscultation revealed a gallop rhythm and loudP2. She had a fever of 38.5°C (101.3°F). Inspection ofthe trunk revealed reddish-brown non-palpable petechiae.There were also subconjunctival and oral haemorrhagesand petechiae. ECG showed sinus tachycardia. CXRshowed diffuse bilateral pulmonary infiltrates.Investigations are shown.

Question 189

Arterial blood gases:PaO2 8 kPaPaCO2 4 kPapH 7.55

Haemoglobin 8.0 g/dlMCV 80 flWCC 11 � 109/lPlatelets 100 � 109/lFibrinogen 0.95 g/l (NR 2–4 g/l)

What is the most likely diagnosis?a. Meningitis.b. Pulmonary embolism.c. Fat embolism.d. Subacute bacterial endocarditis.e. Endotoxic shock.

A 26-year-old female patient developed severe abdominalpain and vomiting. The haemoglobin on admission was9.6 g/dl. A blood film is shown (191).

Question 191

What is the cause of her abdominal pain?a. Biliary colic.b. Small bowel obstruction.c. Lead poisoning.d. Haemolytic uraemic syndrome.e. Mesenteric ischaemia.

A 30-year-old woman with a history of epilepsy sincechildhood was 12 weeks pregnant. Her epilepsy had beenvery well controlled on carbamazepine 200 mg tds andshe had been completely free from epileptic seizures foreight years.

Question 190

What is the best management of her seizures duringpregnancy?

a. Continue with the current dose ofcarbamazepine.

b. Stop the carbamazepine.c. Halve the dose of carbamazepine.d. Switch to sodium valproate.e. Switch to phenytoin.

119911

Page 199: Rapid review of clinical medicine for mrcp part 2

198

Fat embolism is a clinical diagnosis. In many patients thediagnosis is missed owing to subclinical illness orconfounding injury or illness. Mortality rate is 10–20%. It isusually caused by trauma to a long bone or the pelvis,including orthopaedic procedures, as well as parenteral lipidinfusion or recent corticosteroid administration (Table).

Early persistent tachycardia may herald the onset of thesyndrome. Patients become tachypnoeic, dyspnoeic andhypoxic owing to ventilation–perfusion abnormalities12–72 hours after the initial injury. A high fever iscommon. Between 20 and 50% of patients develop reddish-brown non-palpable petechiae over the upper body,particularly in the maxillae, within 24–36 hours of insult orinjury; these resolve quickly. The presence of petechiae inthis setting is virtually diagnostic of fat embolism.Subconjunctival and oral haemorrhages are also recognized.

Central nervous system dysfunction initially manifestsas agitated delirium but may progress to stupor, seizures,or coma and frequently is unresponsive to correction ofhypoxia. Retinal haemorrhages with intra-arterial fatglobules are visible upon fundoscopic examination.

Arterial blood gas shows an otherwise unexplainedincrease in pulmonary shunt fraction alveolar-to-arterialoxygen tension difference.

Thrombocytopenia, anaemia, and hypofibrino-genaemia are indicative of fat embolism syndrome;however, they are non-specific.

Serial chest X-rays reveal increasing diffuse bilateralpulmonary infiltrates within 24–48 hours of the onset ofclinical findings. Nuclear medicine ventilation–perfusionimaging of the lungs may be normal or may demonstratesubsegmental perfusion defects.

Medical care is supportive in nature and includesmaintenance of adequate oxygenation and ventilation,stable haemodynamics, blood products as clinicallyindicated, hydration, prophylaxis of deep venousthrombosis and stress-related gastrointestinal bleeding,and adequate nutrition.

Answer 189

c. Fat embolism.

Causes of fat embolism

• Blunt trauma (associated with 90% of FES cases)• Acute pancreatitis• Diabetes mellitus• Burns• Joint reconstruction• Liposuction• Cardiopulmonary bypass• Decompression sickness• Parenteral lipid infusion• Sickle cell crisis

The blood film shows numerous spherocytes (red cellswithout the central punched out doughnut ring). Thepresence of more than 50% spherocytes on the blood film

is highly suggestive of hereditary spherocytosis. Patientswith hereditary spherocytosis have chronic haemolysisand are prone to pigment gallstones at a young age. Thecommonest complications of gallstones are biliary colic,cholecystitis and pancreatitis.

Answer 191

a. Biliary colic.

Most anticonvulsant drugs have teratogenic properties.However, in patients with epilepsy the benefits oftreatment with anticonvulsant drugs outweigh the risksof teratogenicity, and anticonvulsant therapy iscontinued. The risk of teratogenicity is reduced if thetreatment of epilepsy is limited to a single drug inpregnancy.

Neural tube defects are probably the most seriousteratogenic effect of anticonvulsants and are particularly

important in patients taking carbamazepine,oxycarbamazepine, phenytoin and valproate. The risk ofneural tube defects may be reduced by folic acidsupplements. Pregnant females taking anticonvulsantmedications should be advised to take folic acid supple -ments before and during pregnancy.

Women with epilepsy wishing to become pregnantshould be informed of the potential consequences ofanticonvulsant medications on the foetus. Women whoare already pregnant should be counselled and offeredantenatal screening with alpha-fetoprotein measurementsand second-trimester foetal ultrasound scan.

Answer 190

a. Continue with the current dose of carbamazepine.

Page 200: Rapid review of clinical medicine for mrcp part 2

Clinical Cases 199

A 29-year-old Tanzanian woman presented with a one-week history of increasing dyspnoea, orthopnoea andparoxysmal nocturnal dyspnoea. She had undergone aCaesarean section for an uncomplicated twin pregnancytwo weeks previously. This was her first pregnancy. Theblood pressure throughout the pregnancy was normal. Thefirst trimester was complicated by hyperemesis, whichresolved spontaneously. The patient had noticed increasingdyspnoea and swollen ankles just prior to delivery butattributed this to her advanced pregnant state.

A murmur was heard on routine examination in thesecond trimester, which was investigated with a 12-leadECG and echocardiogram that were both reported asnormal. There was no family history of cardiac disease.

On examination the pulse was 120/min and regular.The temperature was 37.2°C (99.0°F). The bloodpressure measured 100/60 mmHg. The JVP was raised.Palpation of the precordium revealed a displaced andprominent apex. On auscultation both heart sounds werenormal. In addition there was a systolic murmur of mitral

regurgitation and a third heart sound. Auscultation of thelungs revealed inspiratory crackles at both bases.

Investigations are shown.

Question 193

Hb 10.8 g/dlWCC 13 � 109/lPlatelets 368 � 109/lESR 38 mm/hCRP 13 g/lCXR Pulmonary oedema12-lead ECG (193a)Echocardiogram Dilated left ventricle with

globally impaired systolic functionEjection fraction 35% Mitral regurgitation No obvious vegetations on the valves

Blood cultures AwaitedArterial blood gases:

pH 7.38PaCO2 4.3 kPaPaO2 7.3 kPaBicarbonate 25 mmol/l

What is the most probable diagnosis?a. Idiopathic dilated cardiomyopathy.b. Infective endocarditis.c. Forward output heart failure due to anaemia.d. Viral myocarditis.e. Peri-partum cardiomyopathy.

A 20-year-old woman gave birth to a full term baby whowas floppy at birth and required resuscitation followed byassisted ventilation. The mother had ptosis, muscleweakness and bilateral cataracts. Her father also hadpremature cataracts.

Question 192

What is the baby’s diagnosis?a. Galactosaemia.b. Hypoparathyroidism.c. Hypothyroidism.d. Myasthenia gravis.e. Myotonic dystrophy.

119933aa

Page 201: Rapid review of clinical medicine for mrcp part 2

200

The patient presents with pulmonary oedema in thepuerperium associated with systolic heart failure. She hadpreviously documented normal LV function and there isno obvious cause for her heart failure other thanpregnancy. Although she has a low-grade fever and amurmur of mitral regurgitation, the CRP is not highenough to indicate overwhelming sepsis, and theechocardiogram does not show any vegetations or valvedysfunction as would be expected in this situation.

Peri-partum cardiomyopathy affects 1 in 3,000 live-birth pregnancies. The aetiology is unknown but thecondition is much more common in African women.Other predisposing factors include advanced age, multiplepregnancy, multi-parity and a history of hypertension inpregnancy. The condition presents in the last month ofpregnancy or within five months post delivery, whereasfemales with idiopathic dilated cardiomyopathy present inthe second trimester when the burden of volume overloadis greatest (193b).

Diagnosis requires the following four criteria:1. Presentation in the last month or within five months

of delivery.2. Absence of an obvious cause for heart failure.3. Previously normal cardiac status.4. Echocardiographic evidence of systolic left ventricular

dysfunction.

The patient fulfils all four criteria. The ECG may showmultiple abnormalities that include voltage criteria for leftventricular hypertrophy and ST/T wave abnormalities, asin this case. There are no symptoms of myocardial

ischaemia as expected in a thrombotic myocardialinfarction that may rarely complicate pregnancy.

The management is with conventional heart failuretherapy. ACE inhibitors should be avoided duringpregnancy. Patients with very dilated or poorly con tractingleft ventricles should be anticoagulated, as the risk ofintramural thrombus is high in peripartum cardio -myopathy. The prognosis is guarded with up to 25% offemales dying in the first three months. Approximately 50%recover full cardiac function within six months. Around 5%of females require cardiac transplantation. Futurepregnancies should be avoided in females who have notrecovered normal cardiac size and function.

Answer 193

e. Peri-partum cardiomyopathy.

Peripartumcardiomyopathy

Morecommon in

West Africans1/3,000live births

Advancedage >30

? Viralmyocarditis

Lake saltSeleniumdeficiency

Increasedparity

Treat withconventionalheart failuretherapy and

anticoagulation? IV immuno -

globulin

50% recover10% diewithin2 years

4% requiretransplant

Diagnosis basedon echo and

the absence ofan alternative

causeRare before

36/52

The mother has ptosis, muscle weakness and bilateralcataracts indicating that she has myotonic dystrophy. Herbaby is also affected. Whereas the classical form of dystrophiamyotonica presents in adolescence or young adulthood,babies born to mothers with dystrophia myotonica may havethe manifesting features of myotonic dystrophy at birth, i.e.hypotonia and respiratory muscle weakness.

Dystrophia myotonica is inherited as an autosomaldominant trait. The condition is due to mutationcomprising a triple nucleotide repeat (CTG) expansion inthe gene encoding a serine threonine protein kinase onchromosome 19. Another locus on chromosome 3comprising a CCGT repeat has also been identified.

Affected individuals usually present with myotonia withsubsequent progressive muscle weakness and wastingaffecting the face, neck and distal limb muscles. Cataracts,frontal balding, diabetes due to insulin resistance, cardiacconduction tissue disease, cardio myopathy, hypogonadismand low intelligence are also recognized features.

The severity of the condition appears to be related tothe number of triple nucleotide repeats. The higher thenumber or repeats the more severe the manifestations.There can be further amplification of the triplenucleotide repeats in successive generations, resulting inincreased disease severity in future generations (geneticamplification). Myotonia is treated primarily withmexiletine, although phenytoin, procainamide andcarbamazepine and quinine have also been used.Weakness is treated with mechanical devices.

Answer 192

e. Myotonic dystrophy.

119933bb

Page 202: Rapid review of clinical medicine for mrcp part 2

Clinical Cases 201

Question 194

What are the chances of A being affected?a. None.b. 100%.c. 50%.d. 25%.e. 12.5%.

Question 195

What are the chances of B being a carrier?a. None.b. 100%.c. 50%.d. 25%.e. 12.5%.

Questions 194 and 195

Symbol definitions

Clear

Affected

Possibly affected? ?

?B

?A

A 60-year-old female, who was diagnosed as havingrheumatoid arthritis eight years previously, presentedwith a two-month history of progressive dyspnoea. Shewas on a non-steroidal agent and methotrexate forcontrol of her rheumatoid arthritis. There was no historyof cough productive of sputum.

On examination her heart rate was 80 beats/min andregular. Her blood pressure measured 106/68 mmHg.The JVP was not raised. Both heart sounds were normaland the chest was clear. The oxygen saturation on air was96% but desaturated to 85% after walking for twominutes. The chest X-ray was normal.

Question 196

What investigation would you perform first toinvestigate the cause of her breathlessness?

a. Pulmonary function tests.b. Ventilation/perfusion scan.c. Bronchial washings for Pneumocystis jerovecii. d. High-resolution CT scan.e. Echocardiogram.

Page 203: Rapid review of clinical medicine for mrcp part 2

202

Answers 194 and 195

B

Generation 1

Generation 2

A

Generation 3

Generation 4

Symbol definitions

Clear

Affected

Possibly affected? ?

This is a family pedigree with an X-linked recessivecondition. The mother of A in generation 2 is a definitecarrier as his brother is affected. Since females pass the

abnormal X-chromosome to 50% of all their childrenthere is a 50% chance that A will have the disease. (Acannot be a carrier as this is an X-linked condition.) Thegrandmother of B (see generation 2) is a definite carrieras her son is affected (generation 3). Therefore there is a50% (or a 1/2) chance of B’s mother being a carrier. Thechances of B being a carrier are 1/2 � 1/2 since a carrierfemale will only pass the abnormal gene to half of all heroffspring. B will not be affected, as this is an X-linkedrecessive condition.

Answer 194c. 50%.

Answer 195d. 25%.

This is a difficult question. Progressive dyspnoea andoxygen desaturation during exercise in patients withrheumatoid arthritis are suggestive of pulmonary fibrosis.However multiple pulmonary emboli may also present ina similar fashion. The presence of sharp chest pains doesnot differentiate small pulmonary emboli from pulmonaryfibrosis due to rheumatoid arthritis because sharp chestpain is common to both. The absence of inspiratorycrackles and a normal chest X-ray are not characteristic of

rheumatoid lung disease such as pulmonary fibrosis andobliterative bronchiolitis, respectively. While it is stillpossible that the patient’s symptoms are related directlyto rheumatoid arthritis or the treatment of rheumatoidarthritis, the diagnosis of pulmonary embolism should beconsidered. Therefore the first investigation in a patientpresenting in this fashion is a ventilation/perfusion scanor a CT pulmonary angiography. The identification andtreatment of pulmonary embolism should prevent furtherpulmonary emboli. The absence of fever is against thediagnosis of Pneumocystis jerovecii pneumonia.

Answer 196

b. Ventilation/perfusion scan.

Page 204: Rapid review of clinical medicine for mrcp part 2

Clinical Cases 203

A 57-year-old woman presented with a six-hour historyof severe central chest pain. The 12-lead ECG revealedST elevation in the antero-lateral leads. Apart from ahistory of a perforated duodenal ulcer requiring surgeryfive weeks previously, there was no other past medicalhistory of significance.

Question 197

What would be the most effective treatment in themanagement of this patient?

a. Aspirin and IV heparin.b. IV tenectoplase.c. Aspirin and beta-blocker.d. Aspirin, clopidogrel and heparin.e. Primary coronary angioplasty.

A 50-year-old West African woman was admitted tohospital with a 48-hour history of generalized headacheand dyspnoea at rest. The patient had a cough productiveof white frothy sputum. There was no history of fever,haemoptysis or drowsiness. The patient had been treatedfor hypertension for five years; blood pressure controlover the past two years had been satisfactory. She wasknown to have mild asthma that was usually precipitatedby cold weather. She had Raynaud’s disease that wascontrolled with nifedipine and had recently been referredto a gastroenterologist for investigation of difficulty inswallowing. Current medications included nifedipine SR20 mg bd and ranitidine 150 mg bd.

On examination she appeared unwell. Hertemperature was 36.8°C (98.2°F) and her heart rate was110 beats/min and regular. The blood pressure was220/130 mmHg. The JVP was not raised. Onexamination of the precordium, the apex was notdisplaced. Auscultation revealed normal heart sounds andan added fourth heart sound at the apex. The respiratoryrate was 28/min. Auscultation of the lungs revealedbibasal inspiratory crackles. The ankles were mildly

swollen. Neurological examination was normal with theexception of the fundi, which revealed grade hardexudates, cotton wool spots and flame-shapedhaemorrhages.

Investigations are shown.

Question 198

Hb 11 g/dlWCC 11 � 109/lPlatelets 100 � 109/lBlood film Shistocytes

MicrospherocytesSodium 138 mmol/lPotassium 4.8 mmol/lUrea 28 mmol/lCreatinine 490 μmol/lECG Left ventricular hypertrophy Chest X-ray Bilateral basal alveolar shadows

and small pleural effusionsUrinalysis Protein ++

Blood 0

What is the best treatment for her hypertension?a. Prednisolone.b. IV nitroprusside.c. Plasma exchange.d. ACE inhibitor.e. Labetolol.

Page 205: Rapid review of clinical medicine for mrcp part 2

204

The patient has a history of well-controlled hypertensionand then presents with headache and breathlessness. Thepresence of grade III hypertensive retinopathy andpulmonary oedema in association with severehypertension is consistent with the development ofaccelerated or malignant hypertension.

While there are potentially several conditionsassociated with malignant hypertension – such as renalartery stenosis, hypertensive nephrosclerosis, haemolyticuraemic syndrome, anti-phospholipid syndrome – thepresence of Raynaud’s syndrome and possibleoesophageal problems is highly suggestive of systemicsclerosis. In the context of this history the micro-angiopathic haemolytic blood picture reflects malignanthypertension rather than haemolytic uraemic syndromeor a vasculitis.

Renal disease is frequently present in patients withscleroderma. Up to 80% of patients have renal disease and50% actually have signs of renal problems, e.g. abnormalelectrolytes, hypertension and proteinuria. Around 10% of

patients develop scleroderma renal crisis owing to fibrinoidnecrosis within the interlobular arteries and the glomeruli.It is characterized by acute renal failure, abrupt onset ofhypertension or sudden acceleration of otherwise mildhypertension. Proteinuria is common but haematuria is nota feature.

Risk factors for the development of sclerodrema renalcrisis include diffuse skin involvement in association withthe scleroderma, use of corticosteroids, cool weather andblack race.

Blood pressure control is the mainstay of treatment.ACE inhibitors are the drugs of choice and lead to BPimprovement in up to 90% of patients by inhibitingangiotensin II mediated vasoconstriction. ACE inhibitorshave significantly improved the prognosis in thiscondition. Best benefit is achieved by gradual BPreduction (10–15 mmHg per day) until diastolic BPreaches 85–90 mmHg. Rapid reduction in BP mayworsen renal function, therefore IV nitroprusside andlabetolol are relatively contraindicated. Despite treatmentwith ACE inhibitors up to 50% of patients develop end-stage renal failure.

Answer 198

d. ACE inhibitor.

The patient’s chest pain and ST elevation in the anteriorleads are consistent with the diagnosis of acute anteriormyocardial infarction. The usual treatment would be tothrombolyse the patient. However, the patient does havea major contraindication as she had major surgery onlyfive weeks previously (Table). The safest and mosteffective therapy in this situation would be to performcoronary angiography with a view to primary angioplasty.Patients who undergo primary angioplasty are usuallygiven intravenous abciximab (which itself may promotehaemorrhage) to reduce the risk of acute stent occlusion;however, the drug would be used with great caution in apatient who had recently had major surgery.

While aspirin and heparin have been shown to reducemortality in acute myocardial infarction, they are not aseffective as thrombolysis or angioplasty.

Answer 197

e. Primary coronary angioplasty. Contraindications for thrombolysis

Absolute contraindications for thrombolysis• Aortic dissection• Cerebral haemorrhage (ever)• Intracranial neoplasm• Active severe bleeding at time of presentation

Major contraindications• CVA (embolic) within three months• Major surgery within three months• INR more than 4 in patient taking warfarin or

equivalent

Relative contraindications• Pregnancy• Uncontrolled hypertension• Concomitant use of anticoagulant drugs• Active peptic ulcer disease

Page 206: Rapid review of clinical medicine for mrcp part 2

Clinical Cases 205

A 70-year-old man was awoken with retrosternal chest painradiating into his left arm in the early hours of the morning.The pain was persistent for 5 hours, and unrelieved by takingantacids before finally subsiding. The following day heconsulted his daughter over the telephone, who persuadedhim to make an urgent appointment with his GP. There wasa past history of mild hypertension and hypothyroidism. Thelatter was diagnosed over a year ago after the patientpresented with lethargy and was treated with 25 mg ofthyroxine; however, the drug was stopped by the patientwithin three months because he felt well again. He was anon-smoker and consumed alcohol on a very infrequentbasis. Over the past two months he had felt lethargic andsaw another doctor, who diagnosed depression. He livedalone. He had been widowed three years previously. Hetook antacids for infrequent indigestion.

On visiting his GP, an ECG was performed (199).The patient was referred to the local hospital immediatelyfor further evaluation. On arrival at hospital he appearedpale, but was well perfused. His heart rate was50 beats/min, and blood pressure was 150/95 mmHg.The heart sounds were normal and the chest was clear.There was no clinical evidence of cardiac failure.Neurological examination was normal.

Further investigations are shown.

Question 199

Hb 9.6 g/dlWCC 4 � 109/lPlatelets 150 � 109/lMCV 108 flSodium 132 mmol/lPotassium 3.8 mmol/lUrea 3 mmol/lCreatinine kinase 300 iu/lTroponin T 5.3 ng /l (NR < 0.1 ng/l)TSH 21 mu/lThyroxine 30 nmol/lThyroid microsomal Positive 1/3,000

antibodiesMorning cortisol 800 ng/lLH 8 iu/l (normal, 1–10 iu/l)FSH 5 iu/l (normal, 1–7 iu/l)Prolactin 550 mu/l

1. Give one reason for the elevated troponin.

a. Anterior ST elevation MI.b. Non-ST elevation

anterolateral infarction.c. Inferior ST elevation MI.d. Hypothyroidism.e. Intramuscular injection.

2. What is the most likely cause of the abnormal thyroid function tests?

a. Secondary hypothyroidism.b. Tertiary hypothyroidism.c. Auto-immune

hypothyroidism.d. de Quervain’s thyroiditis.e. Sick euthyroid syndrome.

3. What single investigation would you perform to investigate the cause of this patient’s anaemia?

a. Serum B12 level.b. Serum folate.c. Gastric parietal antibodies.d. Serum ferritin.e. Hb electrophoresis.

119999

A 50-year-old woman presented with a two-day historyof increasing difficulty with swallowing and regurgitationof food. Over the preceding two to three months she hadalso experienced difficulty climbing stairs, raising herarms above her head, and arising from a sitting or lyingposition, with some associated lower back pain. Onexamination she was found to have symmetrical proximalmuscle weakness with normal tone, sensation and deeptendon reflexes. Cranial nerve examination revealedweakness of the bulbar muscles.

Question 200

What is the most likely cause of her muscle weakness?a. Motor neurone disease.b. Myasthenia gravis.c. Hypothyroidism.d. Polymyositis.e. Polymyalgia rheumatica.

Page 207: Rapid review of clinical medicine for mrcp part 2

206

The patient presents with a non-ST elevation anterolateralmyocardial infarction as evidenced by the ST depressionand elevated troponin level. The blood results suggest thathe has primary hypothyroidism. The very high titre of anti-microsomal antibodies indicates atrophic auto-immunehypothyroidism or Hashimoto’s hypothyroidism.Hypothyroidism is a well-recognized cause of secondaryhypercholesterolaemia (Table A for other causes) andischaemic heart disease. Other cardiac manifestations of thecondition include pericarditis, dilated cardiomyopathy andpericardial effusion. In this case, the raised creatinine kinaselevel could be explained by myocardial infarction and byhypothyroid induced myositis. Other causes of raisedcreatinine kinase are listed (Table B). Kleinfelter’s

syndrome is associated with hypothyroidism, but thenormal LH level (usually high in Kleinfelter’s syndrome) isagainst the diagnosis.

Hypothyroidism may cause normochromic normocyticanaemia, macrocytic anaemia or microcytic anaemia. Thelatter is usually seen in menstruating women who developmenorrhagia as a consequence of hypothyroidism.Macrocytosis in hypothyroidism may be directly due tohypothyroidism or associated pernicious anaemia, which iscommon in patients with auto-immune hypothyroidism.Pernicious anaemia is best diagnosed by the demonstrationof low serum B12 and the presence of gastric, parietal andintrinsic factor antibodies. Coincidental folate deficiencymay also cause macrocytosis.

Answer 199

1. b. Non-ST elevation anterolateral infarction.2. c. Auto-immune hypothyroidism.3. a. Serum B12 level.

Table A Causes of secondary hypercholesterolaemia

• Alcohol abuse • Obesity• Diabetes mellitus • Chronic liver disease• Hypothyroidism • Myeloma• Primary biliary cirrhosis • Nephrotic syndrome

Table B Causes of a raised creatinine kinase

• Normal variant in the black population (up to 300 iu/l)

• Muscle injury or trauma• Septicaemia• Leptospirosis• Polymyositis• Muscular dystrophy• Hypothyroidism• Cholesterol-lowering statin and fibrate drugs

See Answer 323.

The patient has symmetrical distribution of proximal muscleweakness as well as weakness of the bulbar muscles. Themost probable diagnosis is polymyositis. Muscle weakness isthe most common presenting feature of polymyositis. Theonset is usually insidious, with gradual worsening over aperiod of several months before medical attention is sought.Occasionally, however, it shows acute onset of symptoms.Diagnostic criteria:• Symmetric proximal muscle weakness.• Elevated plasma muscle enzymes.• Myopathic changes on electromyography.• Characteristic muscle biopsy abnormalities and the

absence of histopathologic signs of other myopathies .

There is a female to male predominance of about 2:1and, in adults, the peak incidence occurs in the fifthdecade, although all age groups may be affected.

Muscle atrophy may occur in severe, long-standingdisease. 30% develop weakness of the oropharyngealmuscles or the striated muscle of the upper one-third ofthe oesophagus, leading to dysphagia, nasal regurgi tationor aspiration.

Arthralgias and joint pain can occur. Myalgias and muscletenderness occur in 25–50% of cases. These symptoms tendto be mild, unlike those which occur in viral or bacterialmyositis, the inherited metabolic myopathies, fibromyalgia orpolymyalgia rheumatica. The latter predominantly affects thehip and shoulder girdles with accompanying morningstiffness (see Question 340).

Motor neurone disease can present with elevatedplasma muscle enzymes, dysphagia, and proximal muscleweakness. However, distal motor weakness is morecommon, there are associated pyramidal tract signs andthe EMG does not show myopathic changes.

Myasthenia gravis presents in women in their 20s and30s and men over the age of 60. Weakness often beginsin the eyes, causing ptosis and/or diplopia, but maybegin in the bulbar and neck muscles or the muscles ofthe limbs and trunk. One important feature ofmyasthenia is the marked association with fatiguability.Occasionally it can cause diffuse weakness withoutprominent fatiguability symptoms. The respiratorymuscles can be affected in advanced disease. Myastheniagravis is distinguished from myositis by the frequentpresence of facial muscle weakness, normal muscleenzymes, characteristic EMG changes, and the presenceof antiacetylcholine receptor antibodies.

Answer 200

d. Polymyositis.

Page 208: Rapid review of clinical medicine for mrcp part 2

Clinical Cases 207

Hb 15 g/dlWCC 16 � 109/lPlatelets 200 � 109/lSodium 128 mmol/lChloride 98 mmol/lPotassium 3.6 mmol/lUrea 6 mmol/lCreatinine 100 μmol/lCalcium 2.1 mmol/lAlbumin 42 g/lGlucose 6 mmol/lChest X-ray NormalAbdominal X-ray Normal

Question 202

A patient collapsed during an ECG (202a).

1. Which urgent test would you request to confirm the diagnosis?

2. Which two other investigations would you request to aid your management?

3. What is the full diagnosis?4. Give two possible explanations for the low sodium.

1. What is the cause of col lapse?2. What is the immediate manage ment?

Question 201

A 29-year-old male presented with a three-hour historyof severe epigastric pain and profuse vomiting. The painradiated into his back, and was not relieved by any of theantacid therapies which he had tried at home. There wasno previous history of abdominal pain. He was a non-smoker and consumed 1–2 units of alcohol on aninfrequent basis.

On examination, he was distressed with pain. Oninspection of the trunk there were several raised lesions(201). There was no pallor. His heart rate was 100beats/min and blood pressure 140/80 mmHg. Onexamination of the abdomen, he was very tender in theepigastrum but there was no abdominal rigidity. Bowelsounds were reduced.

Investigations are shown.

220011

220022aa

Page 209: Rapid review of clinical medicine for mrcp part 2

208

Answer 202

The rhythm is broad complex, and the QRS-complexesare very irregular and are of varying amplitude and dura -tion. This is typical of ventricular fibrillation.

There is only one effective method of reverting ven -tricular fibrillation to sinus rhythm, and that is DC cardio -version using a high-energy shock. The quicker this isgiven, the higher the chance of reverting to sinus rhythm.The window for success is around 2 minutes, follow ingwhich reversion becomes increasingly difficult. The mostcommon cause of ventricular fibrillation is myocardialinfarction; however, the cardiomyopathies, ventricularaneurysms, electrolyte imbalances (particularly hypo -kalaemia) and certain drug overdoses (e.g. cocaine ortricyclic antidepressant drugs) are also recognized causes.Following myocardial infarction, multiple ventricularectopics occurring on the preceding T-wave may cause

monomorphic ventricular tachycardia which can degener -ate into ventricular fibrillation. Post-myocardial infarctionpatients with ‘R on T’ ventricular ectopics are treatedprophylactically with a lignocaine infusion to reduce thefrequency of the ectopics.

In this case (202b), there is clear evidence of the R-wave of an ectopic beat (1) on the T-wave of thepreceding sinus beat (2).

Answer 201

The history of acute abdominal pain radiating to the backand associated with profuse vomiting is consistent withacute pancreatitis, although a perforated duodenal ulcerand cholecystitis may present similarly. The slightly lowserum calcium supports the diagnosis of acute pancreatitisbecause the associated rise in serum lipase resulting frompancreatic damage leads to an increase in the free fattyacid concentration in the blood, which has the effect ofbinding calcium and thus reducing the ‘free calcium’ inthe serum.

A very high serum amylase is usually diagnostic ofacute pancreatitis, but acute parotitis may cause similarlyhigh levels. However, in the latter case the history is ofpainful parotid glands. A modestly elevated level is seenin perforated duodenal ulcer, cholecystitis and aorticaneurysm.

A neutrophil leucocytosis is common. The serumcalcium may be low for reasons explained above. It ismandatory to check urea, electrolytes, glucose, arterialblood gases, chest X-ray and clotting. The most commoncause of mortality is renal failure and respiratory failure

due to hypovolaemia and ARDS, respectively. The latteris thought to be due to the effects of high levels oftrypsin that has leaked from the pancreas on lungsurfactant. DIC is another important cause of mortality.

In this case, the cause of pancreatitis ishypertriglyceridaemia, as evident by the multiple eruptivexanthomata on the trunk of this young male patient.Causes of acute pancreatitis are tabulated above. In theUK, the most common cause of acute pancreatitis isalcohol abuse, followed by cholelithiasis (‘gallstones’).

The low sodium is either due to vomiting or is a resultof pseudohyponatraemia resulting from hyperlipidaemia.In hyperlipidaemia and paraproteinaemias, the serumsodium is falsely low because only the sodium in freewater space is measured. True hyponatraemia can bedifferentiated from pseudohyponatraemia by measuringthe plasma osmolality, which is low in the former andnormal in the latter.

1. Serum amylase.2. i. Arterial blood gases.ii. Clotting studies.3. Acute pancreatitis secondary to hypertrigly-

ceridaemia.4. i. Vomiting.ii. Pseudohyponatraemia secondary to hyper -

lipidaemia.

Causes of acute pancreatitis

• Alcohol abuse• Cholelithiasis• Coxsackie and mumps virus• Hypercalcaemia• Hypertriglyceridaemia• Drugs: thiazides, steroids, azathioprine• ERCP• Trauma

1. Ventricular fibrillation precipitated by the ‘R on T’ phenomenon.

2. DC cardioversion using 200 J.1

2 220022bb

Page 210: Rapid review of clinical medicine for mrcp part 2

Clinical Cases 209

A 24-year-old basketball player was referred to the localcardiologist after experiencing two episodes of dizzinesswhile playing basketball in one week. During the lastepisode he almost lost consciousness. There were noother associated symptoms. He had similar dizzy episodesone month previously, but attributed these to a coryzalillness. There was no history of headaches. He had nopast medical history of significance. His father died inMexico at the age of only 27 years but the patient wasnot sure about the cause of death. He had a step brotherand sister who were well.

On examination, he appeared well. He weighed 90 kgand was 1.85 m tall. His heart rate was 68 beats/min andhis blood pressure was 130/60 mmHg. The JVP was notraised. On examination of the precordium, he had adouble apical impulse. On auscultation, there was a loudpansystolic murmur in the mitral area, and an ejectionsystolic murmur at the left lower sternal edge. His chestwas clear. Examination of his peripheral pulses wasnormal, as was examination of the central nervous system.

The patient had a 12-lead ECG, echocardiogram and24-hour ECG (203a–d).

Question 203

1. What is the significance of the double apical impulse?

2. With respect to auscultation, what other manoeuvres could you have performed to try and ascertain the cause of the murmurs?

3. What are the abnormalities on the ECG and how would you interpret them?

4. What are the abnormalities on the two-dimensional and M-mode echocardiograms?

5. What is the significance of the abnormality on the 24-hour ECG?

6. What is the diagnosis?7. What implications (if any) does this have on the

patient’s career?

220033aa 220033bb

220033cc 220033dd

Page 211: Rapid review of clinical medicine for mrcp part 2

210

Molecular genetics of hypertrophic cardiomyopathy

Chromosome Gene affected Chromosome Gene affected14 β-myosin heavy chain 11 Myosin-binding protein C1 Troponin T 7 ?19 Troponin I 3 Myosin essential light chain15 α-tropomyosin 12 Myosin regulatory light chain

Answer 203

Hypertrophic cardiomyopathy has a prevalence of0.1–0.2%. It is inherited as an autosomal dominant trait inover 70% of cases, but sporadic cases are recognized. It isa genetically heterogeneous condition which is caused bymutations within genes encoding the sarcomeric proteins.The disease has been assigned to eight separate chromo -somal loci (Table). Mutations within genes encoding β-myosin heavy chain and troponin T account for overhalf of all patients with HCM. Some 50% of patients have

chest pain and breathlessness; 20% present with dizzinessor syncope. Sudden death is frequently the first presen ta-tion, particularly in adolescents, young adults and athletes.The overall mortality rate is 3%. Sudden death commonlyoccurs during or immediately after strenuous exertion.The diagnosis is made using echocardiography to demon -strate left ventricular hypertrophy, which is usually asym -metric, but can be concentric in up to 40% of cases.

1. A double apical impulse is the result of a palpablefourth heart sound which occurs when there isforceful left atrial contraction against a non-compliant left ventricle. This sign is often present inconditions which produce significant left ventricularhypertrophy. Examples include aortic stenosis,hypertrophic cardiomyopathy and occasionallysevere left ventricular hypertrophy due tohypertension.

2. In the absence of hypertension, the differentialdiagnosis is between aortic stenosis andhypertrophic cardiomyopathy. In hypertrophiccardiomyopathy, an obstructive element is presentin approximately 30% of patients. The obstruction isvariable and thought to be due to anterior motionof the mitral valve towards the hypertrophiedinterventricular septum in systole, thus narrowingthe left ventricular outflow tract. The exactmechanism for this is not clearly understood. Theobstruction is worsened by manoeuvres whichreduce LVEDP, and in these circumstances themurmur is louder. The Valsalva manoeuvre(blowing against a closed glottis) increasesintrathoracic pressure and therefore reduces venousreturn to the heart, reducing LVEDP. Standingfrom a squatting position has similar consequences.In aortic stenosis these manoeuvres reduce theintensity of the systolic murmur.

3. The ECG reveals sinus rhythm with voltage criteriafor left ventricular hypertrophy and deep S-waves inthe septal (V2–V3) leads. In addition, there is T-wave inversion in the inferior leads. Regular physicaltraining can also produce left ventricularhypertrophy; however, the extent of hypertrophy

seldom exceeds the upper limit of normal. While itis common to have voltage criteria of left ventricularhypertrophy in athletes, accompanying T-waveinversion in any lead other than V1–V2 should raisethe suspicion of organic heart disease and the needfor further investigation.

4. The two-dimensional echocardiogram revealsasymmetrical left ventricular hypertrophy. Theseptum is grossly hypertrophied. The left ventricularcavity size is small. Both these features suggest thediagnosis of hypertrophic cardiomyopathy. The M-mode study demonstrates systolic anterior motionof the mitral valve, which is also suggestive ofhypertrophic cardiomyopathy. Otherechocardiographic features of hypertrophiccardiomyopathy include an enlarged left atrium andevidence of diastolic dysfunction on Dopplerstudies. The systolic function is usually excellent.

5. The 24-hour ECG demonstrated a run of non-sustained ventricular tachycardia. This occurs inapproximately 25% of patients with hypertrophiccardiomyopathy, and is a high-risk marker of suddendeath. Other markers of high risk include syncopalepisodes and a family history of sudden death. Thepatient is experiencing syncopal episodes. Thepatient’s father died aged 27, and in this context wehave to assume that he may have had hypertrophiccardiomyopathy. This combination classifies thepatient into the high-risk category for sudden death.These patients are treated with an implantableautomatic cardiovertor defibrillator.

6. Hypertrophic cardiomyopathy.7. The diagnosis of hypertrophic cardiomyopathy in an

athlete leads to disqualification from competitivesport, in order to minimize the risk of sudden death.

Page 212: Rapid review of clinical medicine for mrcp part 2

Clinical Cases 211

Question 204

A 68-year-old male was admitted for investigation ofincreasing weakness and recent onset of jaundice. There wasno other relevant past medical history. In particular, therewas no travel abroad and he was not taking any medication.

On examination, he had non-tender lymphadenopathyin the cervical, axillary and inguinal areas. Abdominalexamination revealed an enlarged spleen 6 cm below thecostal margin.

Investigations are shown.

Hb 10.1 g/dlWCC 60 � 109/lPlatelets 190 � 109/lReticulocyte count 12%MCV 101 flBlood film (204)Bilirubin 39 μmol/lAST 20 iu/lAlkaline phosphatase 70 iu/lAlbumin 36 g/lTotal protein 56 g/lChest X-ray NormalUrinalysis Urobilinogen ++

Bilirubin 0

A 19-year-old medical student was admitted to theAccident and Emergency Department with headache anddrowsiness on a cold winter’s morning. He shared ahouse with three fellow students, who found him in bed

after he failed to answer several wake-up calls to attend ananatomy viva. On examination, he was flushed anddrowsy. He was apyrexial. There was no evidence of askin rash. His heart rate was 100 beats/min and regular,and the blood pres sure was 120/80 mmHg. Neurologicalexamination revealed a Glasgow coma score of 11/15.There was no nuchal rigidity. The cranial nerves, motorand sensory systems were normal. Examination of allother systems was normal.

Investigations are shown.

Hb 14 g/dlWCC 9 � 109/lPlatelets 370 � 109/lSodium 135 mmol/lPotassium 4.4 mmol/lUrea 4 mmol/lCreatinine 80 �mol/lArterial blood gases:

pH 7.37PaO2 7.2 kPaPaCO2 3.8 kPaBicarbonate 23 mmol/lO2 saturation 98%

Chest X-ray NormalECG Sinus tachycardia

1. List three abnormalities on the blood film.2. Explain the cause of the phenomenon indicated by

the arrow.3. What is the cause for the jaundice?4. Give two possible explanations for the raised MCV.5. Which single investigation would you perform to

ascertain the cause of this patient’s anaemia?

What is the diagnosis?a. Herpes simplex encephalitis.b. Tricyclic antidepressant overdose.c. Subarachnoid haemorrhage.d. Carbon monoxide poisoning.e. Hypothermia.

Question 205

220044

Page 213: Rapid review of clinical medicine for mrcp part 2

212

Answer 204

Chronic lymphatic leukaemia usually affects the middle-aged and the elderly. It is a malignancy of B-celllymphocytes. The clinical features are insidious, and thediagnosis may be made incidentally on a blood film. Themain features are lethargy, night sweats, weight loss andrecurrent infections due to hypogammaglobulinaemia.Physical examination reveals generalized lymphadenopathyand hepatosplenomegaly. The characteristic blood pictureis of a lymphocytosis and a mild normochromicnormocytic anaemia. Smudge or smear cells are common.As disease progresses, the anaemia becomes more severe,partly owing to marrow suppression and partly owing to an

auto-immune haemolytic anaemia, and athrombocytopenia becomes evident. Patients presentingwith low platelets have advanced disease and a poorprognosis. Folate deficiency may occur secondary toincreased cell turnover and haemolysis. Therapy iscommenced in symptomatic patients. The drug of choice ischlorambucil. Severely immunosuppressed individuals maybe treated with intravenous gammaglobulin preparations.

Causes of autoimmune haemolytic anaemia for higher examinations

Warm(IgG antibody attaches to red cells at 37°C (98.6°F))

• SLE• Lymphoma• Carcinoma• Chronic lymphatic leukaemia• Methyl dopa

Cold(IgM antibody attaches to red cells below 37°C (98.6°F))

• Mycoplasma pneumonia• Infectious mononucleosis• Paroxysmal cold haemoglobinuria (IgG)• Lymphoma

1. i. Numerous mature lymphocytes.ii. Smudge or smear cell.iii. Red cell agglutination.

2. Smudge or smear cells are an artefact phenomenon which arises during blood film preparation due to the fragility of lympho cytes in chronic lymphatic leukaemia.

3. Warm auto-immune haemolytic anaemia (see Tablefor other causes).

4. i. Reticulocytosis secondary to haemolysis.ii. Megaloblastic anaemia due to folate deficiency

from increased cell turnover.5. Direct Coomb’s test.

The clue for the diagnosis of carbon monoxide poisoningcomes from the arterial blood gases, which reveal hypoxiabut a normal oxygen saturation. Carbon monoxidedisplaces oxygen from Hb and displaces the oxygendissociation curve to the left. It also inhibits cellularrespiration by inhibiting the cytochrome oxidase system.This results in tissue hypoxia, which may manifest asheadaches, dizziness, convulsions, nausea and vomiting.On examination, patients may have tachycardia,tachypnoea, ataxia and cherry red discoloration of thelips; however, the latter is a relatively rare and latemanifestation of the condition. Retinal haemorrhages area recognized complication of carbon monoxidepoisoning. Late complications include basal gangliainfarction and parkinsonism, and sometimesneuropsychiatric symptoms. Oxygen saturation is normalbecause automated biochemical and pulse oximeteranalysers cannot differentiate between oxyhaemoglobinand carboxyhaemoglobin; however, PaO2 is low whenthere is significant carbon monoxide poisoning. It isworth noting that arterial PaO2 is often normal in

patients with carbon monoxide poisoning; therefore, ahigh index of suspicion is needed to diagnose thecondition. Carbon monoxide poisoning is caused bycombustion engines, faulty stoves or paraffin heaterswhen facilities for ventilation are poor.

The diagnosis is made by direct measurement ofarterial carboxyhaemoglobin level. Levels of 40–60% areassociated with neurological symptoms and signs(described above), as well as cardiac brady- andtachyarrhythmias. Levels above 60% are associated withcoma and death.

The management of carbon monoxide poisoninginvolves administration of 100% oxygen via a tight-fittingmask. This reduces the half-life of carboxyhaemoglobinfrom 4 hours to approximately 90 minutes. During thisperiod, carboxyhaemoglobin levels should be measured 2-hourly, and the oxygen continued until thecarboxyhaemoglobin level is below 10%. Patients withneurological signs and symptoms, ECG abnormalities,myocardial ischaemia, pulmonary oedema and shock requirehyperbaric oxygen (3 atm) at a specialized centre. Somewould agree that any individual with a carboxyhaemoglobinlevel exceeding 25% should also be considered forhyperbaric oxygen, even if symptoms are absent.

Answer 205

d. Carbon monoxide poisoning.

Page 214: Rapid review of clinical medicine for mrcp part 2

Clinical Cases 213

Question 206

A 38-year-old businessman was admitted to hospital withnausea, vomiting and drowsiness. He had been inThailand for 10 days and returned back to England twodays previously. He was well in Thailand, but on arrivalto England developed general malaise and a headache.He attributed his symptoms to excess alcohol ingestionwhile abroad, and attempted to relieve his symptoms withtwo paracetamol tablets. He continued to have aheadache the following morning, and took another twotablets of paracetamol, without much relief of hissymptoms. Following this, he took two paracetamoltablets every four hours. He gradually became moreunwell, nauseous and started vomiting. There was noneck stiffness, photophobia or diarrhoea. He normallyconsumed two bottles of wine and occasional spirits perday and smoked 30 cigarettes a day. While abroad, hehad drunk more alcohol than usual. There was nosignificant past medical history.

On examination, he was tanned and had a flushedface. He was mildly icteric. He had a fever of 39°C(102.2°F). He was slightly drowsy, but could hold anormal, coherent conversation. There was no evidence ofasterixis. Examination of the fundi was normal. The heartrate was 110 beats/min and blood pressure 105/70mmHg. Examination of the precordium revealed a verysoft third heart sound. His chest was clear. Abdominalexamination revealed a slightly tender palpable liver edge4 cm below the costal margin. The spleen was notpalpable.

Investigations are shown.

Hb 15.8 g/dlWCC 9 � 109/lPlatelets 105 � 109/lMCV 102 flPT 20 s (control 14 s)Malaria parasite Not detectedSodium 133 mmol/lPotassium 3.5 mmol/lUrea 5 mmol/lCreatinine 98 μmol/lBilirubin 30 μmol/lAST 1500 iu/lAlkaline phosphatase 110 iu/lSerum ferritin 500 μg/lChest X-ray Normal

A 42-year-old female presented withincreasing dyspnoea on exertion. Shehad a past history of rheumatic fever.Left and right cardiac catheter datawere as follows:

Chamber Pressure (mmHg) Oxygen saturation (%)Right atrium 10 68Right ventricle 40/10 68Pulmonary artery 40/22 68PCWP 16Left ventricle 160/10 96Aorta 110/70 96

1. Which test would you perform immediately?a. Blood cultures.b. Hepatitis serology.c. Blood ethanol level.d. Serum paracetamol level.e. Liver biopsy.

2. What is the cause of the deterioration of his symptoms?a. Paracetamol poisoning/viral hepatitis.b. Paracetamol poisoning/alcohol abuse.c. Haemochromatosis/hepatoma.d. Viral hepatitis/paracetamol intoxication.e. Haemochromatosis/paracetamol poisoning.

What statement fits best with the data provided?a. Mitral stenosis, aortic stenosis and pulmonary hypertension.b. Mitral regurgitation and pulmonary hypertension.c. Aortic stenosis and pulmonary hypertension.d. Atrial septal defect, aortic stenosis and pulmonary hypertension.e. Mixed aortic valve disease.

Question 207

Page 215: Rapid review of clinical medicine for mrcp part 2

214

Answer 207

The right ventricular pressure and pulmonary arterypressures exceed 35mmHg, indicating pulmonaryhypertension. The PCWP exceeds the LVEDP by 6 mmHg, indicating significant mitral stenosis. There is apressure drop of 50 mmHg across the aortic valve,indicating moderate to severe aortic stenosis.

There is no evidence of step-up in oxygen saturation inthe right heart to indicate a left-to-right intracardiac shunt.(See Interpretation of Cardiac Catheter Data, page 418.)

Answer 206

There is a history of alcohol abuse and consumption of 6–7 g of paracetamol in 24 hours. Ingestion of 15 g ofparacetamol is regarded as a significant overdose in normalhealthy individuals; however, the dose of paracetamolrequired to cause hepatotoxicity in individuals with chronic,heavy alcohol ingestion is much lower. This is explained bythe ability of alcohol to induce hepatic microsomal enzymesand to deplete hepatic intracellular levels of glutathione.Paracetamol itself is not hepatotoxic, but it is usuallymetabolized to NAPQI, a potent oxidative agent withpotential to cause massive hepatic necrosis. The metaboliteis normally rapidly conjugated with glutathione, whichrenders it inactive and it is excreted. In patients with heavyalcohol ingestion the detoxifying microsomal enzymeconcentration is increased to metabolize alcohol. Althoughthis is a beneficial effect when dealing with alcohol, it canhave serious consequences when large amounts ofparacetamol are ingested because it leads to rapidmetabolism of the drug to NAPQI. The hepatocytes arerelatively depleted of glutathione owing to hepatic damagefrom alcohol and soon become completely depleted in anattempt to conjugate NAPQI. The result is an excess ofNAPQI, causing hepatic necrosis, which can have profoundeffects if hepatic reserve has already been severely affectedby alcohol abuse. Alcohol abusers are more likely to developmassive hepatic necrosis and have a worse prognosis fromparacetamol overdose.

Hepatitis A infection is possible and in the early stagesdoes produce general malaise, fever, nausea and elevatedhepatic transaminases. Hepatitis A has an incubationperiod of two weeks and has a hepatitic and a cholestaticphase. It is feasible that the patient contracted the virusabroad and is now in the hepatitic phase; however, giventhe history of alcohol abuse and self-administration of asignificant amount of paracetamol, this is less likely thanthe answer given above.

The diagnosis of haemochromatosis is raised owing tothe high serum ferritin level of 500 �g /l; however, highferritin levels and deposition of iron in the liver also occurin alcohol-related disease and patients with regular bloodtransfusions (secondary haemochromatosis). Liver biopsyis the main way of distinguishing between primary andsecondary haemochromatosis. In secondary haemo-chromatosis the biopsy will reveal changes of alcoholichepatitis (Mallory bodies) as well as iron deposition.Moreover, primary haemochromatosis is not usuallyassociated with significant derangement of LFT unlessthe disease is very advanced or there is development ofhepatocellular carcinoma, which is a recognizedcomplication of cirrhosis of the liver due to any cause,but particularly haemochromatosis. In this case therewould be a history of poor health before travellingabroad, as well as other features of haemochromatosissuch as diabetes mellitus and hypogonadism, which areabsent. The LFT usually demonstrate a high alkalinephosphatase and a raised bilirubin in the presence ofhepatocellular carcinoma.

1. d. Serum paracetamol level.2. b. Paracetamol poisoning/alcohol abuse.

a. Mitral stenosis, aortic stenosis and pulmonary hypertension.

Page 216: Rapid review of clinical medicine for mrcp part 2

Clinical Cases 215

Question 208

A 15-year-old male was admitted with suddenonset of rapid palpitations which were notassociated with chest pain or dizziness. Onexamination, he was well-perfused and hisblood pressure was 110/65 mmHg. An ECGperformed in the Accident and EmergencyDepartment is shown (208a). He was treatedby the medical officer, and his ECG after wardsis shown (208b).

A 33-year-old male with recently diagnosed acute schizo -phrenia was referred to the Accident and EmergencyDepartment after he was found collapsed on thepsychiatric ward. On examination, the patient was verydrowsy and appeared rigid. The heart rate was120 beats/min, and regular; blood pres sure was110/60 mmHg.

The temperature was 40°C (104°F). Examination ofthe cardio vascular, respiratory and abdominal system wasnormal. Neurological examination demonstrated general -ized rigidity of all muscle groups.

Investigations are shown.

Hb 15 g/dlWCC 10 � 109/lPlatelets 300 � 109/lUrinalysis Protein +Sodium 138 mmol/lPotassium 4 mmol/lUrea 10 mmol/lCreatinine 160 μmol/l

1. What is shown in the 12-lead ECG while the patient is having palpitations?

2. List two forms of treatment with which the medical officer may have attempted to treat the palpitation.

3. Which cardiac condition does this patient have?

4. List four forms of drug therapy which could be safely used to prevent the patient’s symptoms.

5. What is the definitive treatment in a young patient with this disorder?

1. What is the diagnosis?2. Which investigation would you perform to confirm

the diagnosis?3. Briefly, list three management steps.

Question 209

220088aa

220088bb

Page 217: Rapid review of clinical medicine for mrcp part 2

216

Answer 208

Wolff–Parkinson–White syndrome (WPW), a congenitalillness with a prevalence of 1 in 1,500, is charac terized byan accessory conduction pathway between atria andventricles (bundle of Kent) which predisposes to anelectrical re-entrant circuit and tachycardia. During sinusrhythm the electrical impulse conducts rapidly over theaccessory conduction pathway and depolarizes theventricles prematurely, resulting in a short PR-intervaland a wide QRS-complex with slurred upstroke, oftencalled a delta wave. Anterograde con duction through theaccessory pathway and retrograde conduction via theatrioventricular node may predispose to two types oftachyarrhythmias which are atrioventri cular re-entranttachycardia or atrial fibrillation. Two variants of WPW arerecognized: type A (left-sided path way), with an upright(positive) delta wave in V1, as in this case; and type B(right-sided pathway), with negative delta wave in V1(208c). The former is more common.

In atrioventricular tachycardia the ventricular rate isbetween 140–280 beats/min and the P-wave is seen justafter the QRS-complex, usually in the ST-segment Inatrial fibrillation, there is a rapid irregular rhythm withsome QRS-complexes exhibiting delta waves.

Management of atrioventricular re-entrant tachycardiainvolves vagotonic manoeuvres such as carotid sinusmassage, Valsalva manoeuvre, eyeball compression orintravenous adenosine. AF does not usually respond tovagotonic manoeuvres, but may be abolished by adenosine.DC cardioversion is reserved for patients who arehaemodynamically unstable and unresponsive to adenosine.Drugs to prevent arrhythmias include class I and class IIIantiarrhythmic agents (e.g. sotalol). Both digoxin andverapamil are contraindicated in WPW as they increaseconduction through the accessory pathway and increase therisk of malignant ven tricular arrhythmias. Radiofrequencyablation of the acces sory pathway is the definitive treatmentfor AF or frequent atrioventricular re-entrant tachycardia.

The neuroleptic malignant syndrome is a rare but life-threatening condition that can occur with any anti-psychotic drug, irrespective of dose. It is most commonwhen starting treatment or increasing the dose. Symptomsinclude fever, muscle rigidity, confusion and impairedconsciousness. Autonomic instability is well recognized andis associated with excessive sweating, labile blood pressureand tachycardia. The diagnosis is made by demonstrating agrossly elevated creatinine kinase. Rhabdomyolysis mayprecipitate renal failure, which can be avoided by adequatehydration. Other complications include aspirationpneumonia and respiratory failure. The management of this

condition involves stopping the neuroleptic drugimmediately, antipyretics and rehydration. The drugdantrolene may be useful in some cases. Collapse in apsychiatric patient is a common scenario in the exam. Whilemany general medical conditions may cause collapse inpatients with psychiatric disease, specific things to considerare tabulated below.

1. An atrioventricular tachycardia with LBBB.2. i. Vagotonic manoeuvres such as carotid sinus

massage, Valsalva manoeuvre, compres sion of the eyeball.

ii. Intravenous adenosine.3. Wolff–Parkinson–White syndrome (type A).4. i. Sotalol.

ii. Flecanide.iii. Disopyramide.iv. Amiodarone.

5. Radiofrequency ablation of the accessory pathway.

1. Neuroleptic malignant syndrome secondary to the use of phenothiazines.

2. Serum creatinine kinase.3. i. Stop drug.

ii. Intravenous diazepam.iii. Adequate hydration to prevent renal failure.

Specific things to consider in a collapsed patienton a psychiatric ward• Neuroleptic malignant syndrome• Drug overdose• Torsades de pointes (polymorphic VT)

secondary to neuroleptic agents, lithium ortricyclic antidepressant overdose

• Epilepsy secondary to neuroleptic agents• Epilepsy secondary to hyponatraemia of

psychogenic polydipsia• Diabetic emergencies secondary to olanzepine

Answer 209

220088cc

Page 218: Rapid review of clinical medicine for mrcp part 2

Clinical Cases 217

Question 210

A 59-year-old male presented with a three-week history ofcough and progressive breathlessness and three episodesof haemoptysis in 24 hours. At the onset of the initialsymptoms he visited his GP, who diag nosed a respiratorytract infection and prescribed a course of broad-spectrumantibiotics. The cough per sisted and the patient remainedbreathless. His effort tolerance had become significantlyreduced from walking two miles daily to being breathlessat rest. He had felt increasingly lethargic and nauseous fora week before being seen in the Accident and EmergencyDepartment. Twenty-four hours previously he hadcoughed an egg-cup-full of fresh clots of blood on threeseparate occa sions. There was no history of sore throat ornight sweats. Prior to a week previously, his appetite wasgood. There was no history of weight loss. He had a pasthistory of haemorrhoids which were surgically ligated twoyears ago.

The patient was married with one son and worked as amechanic. He smoked 10 cigarettes per day andconsumed 2–3 units of alcohol per week. There was nohistory of travel abroad. There was no family history ofnote, with the exception that his mother had beentreated for pulmonary TB 16 years ago.

On examination, he appeared pale and centrallycyanosed. There was no evidence of clubbing or lympha -denopathy. There was bilateral pitting ankle oedema. Theheart rate was 110 beats/min, and regular, and bloodpressure was 176/105 mmHg. The JVP was raised 4 cmabove the sternal angle. On examination of the respira -tory system chest expansion was symmetri cally reduced.The trachea was central. Percussion note was dull at bothbases, and on auscultation there was widespread bron -chial breathing at both mid-zones and bases. Precordialand abdominal examination was normal.

Investigations are shown.

Hb 7.2 g/dlWCC 12 � 109/lPlatelets 500 � 109/lSodium 137 mmol/lPotassium 5.2 mmol/lUrea 19 mmol/lCreatinine 400 μmol/lCalcium 2.1 mmol/lPhosphate 2 mmol/lAlbumin 40 g/lECG Sinus tachycardia. Partial RBBBChest X-ray Alveolar shadowing affecting both

lower zonesUrinalysis Blood +++

Protein ++Numerous blood cells and red cell

castsLung function:

FEV1 65% predictedFVC 60% predictedTLC 68% predictedRV 66% predicted TLCO 110% predicted

Question 211

A 60-year-old male presents with an episode of amaurosisfugax. On examination, apart from splenomegaly, thereare no abnormal neurological or other clinical findings.

Investigations are shown.

Red cell count 8 � 1012/lHb 14.2 g/dlPCV 0.55MCV 69 flMCH 18.2 pgWCC 15 � 109/lNeutrophils 88%Lymphocytes 8%Monocytes 4%Platelets 475 � 109/lBlood film Hypochromia ++

1. Comment on the respiratory function tests.2. What is the most likely diagnosis?3. List two tests you would request to confirm the

diagnosis.4. What are the four predisposing factors for

producing the changes seen in the chest?

What is the diagnosis?a. Pseudopolycythaemia.b. Secondary polycythaemia.c. Primary polycythaemia (polycythaemia rubra vera).d. Chronic myeloid leukaemia.e. Essential thrombocythaemia.

Page 219: Rapid review of clinical medicine for mrcp part 2

218

The patient has evidence of acute glomerulonephritis. Thebreathlessness, haemoptysis, raised TLCO and bi lateralalveolar shadows on the chest X-ray are highly sug gestiveof pulmonary haemorrhage. The combination of acutenephritis and pulmonary haemorrhage may occur in anti-GBM disease, Wegener’s granulomatosis, micro scopicpolyarteritis nodosa and SLE. In anti-GBM disease,patients who are smokers are more predisposed to lunghaemorrhage, as are patients who are exposed to hydro -carbons. The patient is both a smoker and has beenexposed to hydrocarbons from his occupation as a garagemechanic. The most probable diagnosis is anti-GBMdisease.

Anti-GBM disease is rare, with prevalence 1 case permillion. The male to female ratio is 2:1. The age of onsethas two peaks which are the third and the seventh

decades. There is a strong association between HLA DR2and anti-GBM disease, the antigen being present inapproximately 80% of affected patients. Other HLAassociations include HLA DR4 and HLA DR2. Circu -lating anti-GBM antibodies are responsible for both thenephritis and pulmonary haemorrhage.

Clinical features are those of rapidly progressivenephritis and/or pulmonary haemorrhage. Fatigue andbreathlessness may pre-date the onset of nephritis orfrank haemoptysis. Glomerulonephritis is of the rapidlyprogressive type, and renal function may be completelylost within a week. Some 60% of patients have pulmonaryhaemorrhage. Smokers, patients with coexisting lowerrespiratory tract infections and pulmonary oedema, andpatients exposed to hydrocarbons are more prone tohaemorrhage.

The chest X-ray may reveal alveolar shadowing in thepresence of pulmonary haemorrhage. The differentiationfrom pulmonary oedema (which looks the same) requiresdetermination of the TLCO, which is raised in haemorr -hage and reduced in oedema.

Investigations include renal biopsy with immuno -fluorescent agent which demonstrate rapidly progressiveglomerulonephritis and linear IgG deposits along thebasement membrane. Transbronchial lung biopsy may beuseful, but has a lower histological yield.

Management is with early plasmapharesis, methylpred -nisolone and cyclophosphamide.

The patient has a normal Hb but the PCV and the red cellcount are elevated. Furthermore, he has associated leuco -cytosis, thrombocytosis and spleno megaly, which shouldraise the suspicion of PRV. The normal (as opposed tohigh Hb) is due to coexisting iron defi ciency. The diag -nosis of iron deficiency is supported by the low MCV.

PRV has an insidious onset. It rarely occurs before thefifth decade. Symptoms include lethargy, depression,

vertigo, tinnitus and amaurosis fugax. On examination,the patient may have a plethoric facial complexion, hype r-tension, and a palpable spleen in approximately 70% ofcases. Complications include bleeding due to throm bo-cythaemia, stroke from hyperviscosity, and gout fromincreased cell turnover. Bleeding is most commonly fromthe upper gastro intestinal tract. PRV may progress tomyelofibrosis or acute myeloid leukaemia.

Management is by venesection, although this mayresult in iron deficiency. Busulphan or hydroxyurea areeffective in controlling the thrombocytosis.

Answer 211

Answer 210

1. Restrictive lung defect with a raised TLCO. (See Respiratory Function Tests, page 419.)

2. Goodpasture’s syndrome. Other possible diagnoses include microscopic polyarteritis nodosa and Wegener’s granulomatosis.

3. i. Renal biopsy.ii. Anti-GBM disease.

4. i. Smoking.ii. Lower respiratory tract infection.iii. Pulmonary oedema.iv. Inhalation of organic solvents.

c. Primary polycythaemia (polycythaemia rubra vera).

Page 220: Rapid review of clinical medicine for mrcp part 2

Clinical Cases 219

Question 212

A 54-year-old Turkish male was admitted to hospital witha three-week history of malaise, headaches, pain in hisarms, legs and lower back and night sweats. His appetitewas poor, and he had lost almost 3 kilograms in threeweeks. He worked as a butcher. He had returned from aholiday in Turkey six weeks ago where he lived on hisbrother’s farm for two weeks.

On examination, he was unwell. He had generalizedcervical lymphadenopathy. His temperature was 38.5°C(101.3°F). The blood pressure was 120/60 mmHg.Cardiovascular examination revealed a soft systolic murmurat the apex. Examination of the respiratory system wasnormal. On examination of his abdomen he had a palpablespleen 4 cm below the costal margin. The liver edge was alsopalpable 3 cm below the costal margin. Genital examinationdemonstrated an erythematous scrotum with a swollen,tender left testicle. Examination of the central nervoussystem, including the fundi, was normal. His lower back wastender, but movements were not restricted. Similarly, hislimbs were tender but there was no fasciculation and powerwas preserved. He had tender, red, macular lesions on his

shins which varied between 2–5 cm in diameter.Investigations are shown.

Hb 11.8 g/dlWCC 2 � 109/l (neutropenia)Platelets 440 � 109/lESR 66 mm/hCRP 210 mg/lU&E NormalLFT NormalChest X-ray Few calcified paratracheal lymph

nodes

Question 213

A 24-year-old Jamaican painter and decorator wasadmitted with acute colicky central abdominal painassociated with vomiting. The only past history was thatof a viral illness associated with a rash two weekspreviously. On examination, he appeared pale. The heartrate was 110 beats/min and the blood pressure

140/90 mmHg. The abdomen was generally tender, butthere was no guarding, and bowel sounds wereinfrequent. Examination of the central nervous systemrevealed reduced power and tone in the lower limbs, andabsent ankle and knee reflexes.

Investigations are shown.

Hb 10 g/dlWCC 7 � 109/lPlatelets 170 � 109/lMCV 63 flMCHC 28 g/dlBlood film (213)Urinary d-ALA 100 mmol/day (normal range

11–57 mmol/day)Sodium 134 mmol/lPotassium 3.2 mmol/lBicarbonate 15 mmol/lChloride 115 mmol/lUrea 7 mmol/lCreatinine 80 �mol/lCalcium 2.32 mmol/lAlbumin 40g/lGlucose 4 mmol/lCT of brain Normal

1. What is the diagnosis?2. List three tests you would perform to help confirm

the diagnosis.3. What is the management?

What is the diagnosis?a. Sarcoidosis.b. Acute intermittent porphyria.c. Lead poisoning.d. Arsenic poisoning.e. Polyarteritis nodosa.

221133

Page 221: Rapid review of clinical medicine for mrcp part 2

220

Answer 212

The history of malaise, myalgia, headaches and weight lossin association with neutropenia, hepatospleno megaly anderythema nodosum is highly suggestive of brucellosis.

Brucellosis is caused by Brucella, a Gram-negativebacillus, which may be contracted from cows, goats, pigsor sheep. It has a wide animal reservoir. Three mainspecies cause infection in man: B. abortus (cattle),B. melitensis (goats/sheep), and B. suis (pigs). Theorganism usually gains access into the human bodythrough the gastrointestinal tract as a result of consumingunpasteurized milk; however, contact with animalcarcasses may also increase risk of oral ingestion of thebacterium. Brucellosis is virtually eliminated in the UKdue to the strict policy of pasteurization of milk. Once inthe gastro intestinal tract, the bacilli travel to the

lymphatics and infect lymph nodes, and eventually thereis haemato genous spread to many other organs.

The onset is usually insidious. General malaise, head aches,myalgia, weakness and night sweats are common. There is anundulant high fever. Lympha denopathy is common.Hepatosplenomegaly may be present, but an acute hepatitis israre. Splenomegaly usually indicates severe infection. Spinaltenderness is relatively common. Haematogenous spread maybe com plicated by arthritis, orchitis, endocarditis,osteomyelitis, and meningo encephalitis.

Brucellosis may become chronic when it is undiagnosedand is associated with fatigue, myalgia, depression, andoccasionally fever. Palpable splenomegaly is characteristic.In a few cases the infection may be localized to specificorgans such as the bones, heart or central nervous system.In these circumstances, systemic features are absent in over60% of cases and antibody titres are low.

The diagnosis is based on positive blood cultures andrising anti-Brucella immunoglobulin titres. Blood culturesare positive in only half the cases. The Brucellaagglutination test is positive within four weeks of the onsetof illness. Treatment is with tetracycline and rifam picin forsix weeks, although co-trimoxazole is also effective.

Answer 213

In a young man with abdominal pain, vomiting,neuropathy and anaemia, the most likely diagnosis is leadpoisoning, although several other conditions may also causethe same symptoms (Table). The diagnosis is confirmed bymeasuring the serum lead concentration. A leadconcentration above 4 mmol/l is toxic. Treatment is withD-penicillamine therapy.

Acute lead poisoning is rare, but chronic poisoning mayoccur in scrap-metal workers or plumbers, in individualsingesting water from lead pipes, in children ingesting oldlead-based paint in the house, and painters and decorators.Lead interferes with haem and globin synthesis (see Answer147). The effects are a microcytic anaemia and raisedprecursors of haem synthesis. The latter cause abdominalsymptoms, autonomic and motor neuropathy which areidentical to those seen in AIP. In both lead poisoning andAIP the �-ALA is elevated; however, lead poisoning isdifferentiated from AIP by the relatively severe anaemia,basophilic stippling affecting the red cells and the elevatedproto- and coproporphyrins. Basophilic stippling is causedby aggregates of RNA resulting from inhibition of theenzyme pyrimidine-5-nucleotidase. Other features includehaemolytic anaemia, lead encephalopathy (which ischaracterized by a high CSF protein), a blue line on thegums, and dense metaphyseal bands at the end of long

bones in children (known as lead lines). Proximal RTA isrecognized and comprises a hypokalaemic, hyperchloraemicacidosis, as well as loss of amino acids in the urine.

Although sarcoidosis may be associated with abdominalpain and polyneuropathy and is more common in patientsof Afro-Caribbean origin, there is no evidence of pulmonarysymptoms, bone pain, iritis or hypercalcaemia. The raised �-ALA cannot be explained by sarcoidosis.

Arsenic poisoning may affect patients working in somemetal and glass production industries and patients who havedeliberately or accidentally ingested certain rodenticides. Itpresents acutely with severe abdominal cramps and profusediarrhoea. Chronic exposure usually presents with a painfulperipheral neuropathy, hyperkeratosis, microcytic anaemiaand white horizontal lines on the nails (Mee’s lines).Arsenic does not interfere with haem biosynthesis. (SeeQuestion 257.)

Causes of abdominal pain and neuropathy

• AIP• Lead poisoning• Arsenic poisoning• Guillain–Barré syndrome• Polyarteritis nodosa• Diabetic ketoacidosis• Intra-abdominal malignancy• Sarcoidosis• Alcohol abuse

1. Brucellosis.2. i. Blood culture.

ii. Marrow culture.iii. Brucella agglutination test or Brucella

immunoglobulins (ELISA).3. Tetracycline and rifampicin; co-trimoxazole.

c. Lead poisoning.

Page 222: Rapid review of clinical medicine for mrcp part 2

Clinical Cases 221

Question 214

A 52-year-old obese hypertensive female was seen in theEmergency Department with sudden onset of palpitationsand breathlessness. She had a history of hypertension,congestive cardiac failure and non-insulin-dependentdiabetes mellitus. She was taking captopril 50 mg twicedaily, nifedipine slow release 20 mg twice daily, metformin500 mg three times daily, and furosemide 80 mg daily. Thefurosemide dose had recently been doubled by her GP tocontrol her persistently elevated blood pressure and help herbreathlessness. She had previously undergone several investi-gations to exclude a secondary cause for the hypertension.

On examination, the patient appeared breathless, butwas not cyanosed. She was well perfused. The heart rate was100 beats/min, and regular; blood pressure was 150/100mmHg. The JVP was not elevated. On examination of herprecordium, the apex was not palpable, but the heartsounds were normal. There were no murmurs. Respiratoryexamination revealed good chest expansion. The tracheawas central and percussion was normal. Auscultation of thelung fields was normal. There was no evidence of peripheraloedema, but she had an erythematous skin rash on hershins. Blood results are shown.

Sodium 134 mmol/lPotassium 4.3 mmol/lCreatinine 120 μmol/lUrea 8 mmol/lECG Sinus tachycardiaChest X-ray (214)

Arterial blood gases:pH 7.5PaCO2 1.8 kPaPaO2 15 kPa Bicarbonate 19.3 mmol/lBase excess +7

Question 215

A 25-year-old Malaysian male was admitted to hospital asan emergency when he noticed that he was completelyparalysed for two hours after waking from his sleep in theearly hours of the morning. He had experienced this type ofsevere weakness on four occasions over the past fewmonths, and remarked how they always occurred duringsleep and on waking he could not move. He drank fourunits of alcohol every night. He had recently seen his GPfor anxiety attacks and was being coun selled. He emigratedto England in 1994 and worked as a chef. He was single.All his family members were in Malaysia and were well.

On examination, he was thin; otherwise, physicalexamination was normal. Investigations are shown.

FBC NormalSodium 136 mmol/lPotassium 2 mmol/lUrea 5 mmol/lBicarbonate 23 mmol/lTSH <0.1 mu/lThyroxine 190 nmol/

1. Interpret the blood gases.2. What is the single most likely explanation for the

abnormal blood gas results?

1. What is the cause of the intermittent episodes of profound weakness?a. Epilepsy.b. Periodic paralysis.c. Cataplexy.d. Narcolepsy.e. Brugada’s syndrome.

2. List two important steps in the management of this patient.a. Implantation of an internal cardiovertor

defibrillator.b. Perform an EEG.c. Treat hypokalaemia.d. Perform electrophysiological ventricular

stimulation study.e. Start carbimazole.f. Institute IV dextrose 50% and 16 units of short

acting insulin.g. Perform MRI scan of the brain.h. 24-hour ECG monitoring.i. Give a trial of modenifil.j. Perform a sleep study.

221144

Page 223: Rapid review of clinical medicine for mrcp part 2

222

Answer 214

Respiratory alkalosis is characterized by a low PaCO2 anda high pH. Respiratory alkalosis may be seen in hyper -ventilation and artificial ventilation. In these situations itis associated with a high or normal PaO2. It is also seenin conditions predisposing to ventilation/per fusion mis -match such as pneumonia, pulmonary oedema, pul -monary embolus and parenchymal lung disease. In thisparticular case, the patient has a normal PaO2 and clearlung fields on the chest X-ray, suggesting that she ishyperventilating (see Table for causes of respiratoryalkalosis).

In all chronic respiratory alkalotic states the kidneyscompensate by excreting excess bicar bonate to produce arelative metabolic acidosis in an attempt to normalize theblood pH. Therefore, in compensated respiratory alka -losis the plasma bicarbonate is low.

Acute salicylate poisoning also produces a combinedrespiratory alkalosis and metabolic acidosis. In toxic

doses, salicylates stimulate hyperventilation througheffects on the respiratory regulatory system. Renal com -pensation leads to increased excretion of bicarbonate,resulting in metabolic acidosis. In addition, salicylatesinterfere with oxidative phosphory lation and lead to lacticacidosis, further exacerbating the metabolic acidosis.

(See Acid–base Disturbance, page 426.)

Causes of respiratory alkalosis

Due to stimulation of the central nervous system• Anxiety• Hypoxia• Salicylate poisoning• Encephalitis• Brainstem injury

Due to pulmonary disease• Asthma• Pneumonia• Lung fibrosis• Pulmonary oedema• Pulmonary embolus

Answer 215

Hypokalaemic periodic paralysis is a rare condition that isoften inherited as an autosomal dominant trait. It ischaracterized by episodic paralysis, which classicallyoccurs at night while the patient is asleep. The cause ofhypokalaemia is not clear, but shifts in potassium fromthe extracellular fluid to the intracellular fluid arethought to be responsible. Symptoms can be precipitatedby administering intravenous glucose and insulin intosuch patients, which would lend support to the potassiumshift theory. Other precipitating factors include alcohol,large carbohydrate meals, anxiety and tension. Recoveryis usually rapid, but can be expedited by intravenous

potassium supplements. Long-term treatment is withpotassium supplements or potassium-sparing diuretics.

In oriental males, aged between 20 and 50 years, thecondition is associated with thyrotoxicosis. Thesymptoms of thyrotoxicosis may precede the paralysis, ormay not be apparent. Paralysis occurs during sleep andperiods of prolonged rest. Arrhythmias associated withhypokalaemia are a recognized feature in this condition.Treatment of the thyrotoxicosis prevents further attacksof paralysis. In the meantime, the potassium needs to becorrected. Propranolol is successful in preventing attacksin approximately 60% of cases. Although thyrotoxicosis ismore common in females, periodic paralysis only affectsmales. The mechanism of hypokalaemia is the same as infamilial periodic paralysis. An important differentialdiagnosis when the patient is in the paralysed state isGuillain–Barré syndrome and acute myasthenia gravis.

Narcolepsy and cataplexy have been discussed inQuestion 67 and Brugada’s syndrome is discussed inQuestion 353.

1. The blood gases demonstrate a partially compensated respiratory alkalosis.

2. Hyperventilation.

1. b. Periodic paralysis.2. c. Treat hypokalaemia.

e. Start carbimazole.

Page 224: Rapid review of clinical medicine for mrcp part 2

Clinical Cases 223

Question 216

A 39-year-old male was admitted with malaise, lethargy andbleeding gums. Investigations on admission were as follows:

The patient received chemotherapy which he toleratedwell, but on the second day post-chemotherapy he com -plained of severe nausea. Biochemical results were as follows:

Hb 4 g/dlWCC 70 � 109/l (predominantly blasts)Platelets 26 � 109/lBlood film (216)Sodium 137 mmol/lPotassium 3.7 mmol/lUrea 4 mmol/lCreatinine 74 μmol/l

Sodium 136 mmol/lPotassium 7.2 mmol/lUrea 23 mmol/lCreatinine 300 μmol/lUrate 1.2 mmol/lPhosphate 3 mmol/l

Question 217

A 59-year-old headmaster was referred to a chest unitwith a six-month history of progressive breathlessness.His effort tolerance one year previously was very good,allowing him to jog 10 km, twice a week. Since then hehas had to give up running due to fatigue and breathless -ness. His condition had continued to deteriorate, despitethe use of steroid and salbutamol inhalers prescribed byhis GP. He was now breathless after walking just 200 m.He was a non-smoker. He had been a headmaster in aLondon public school for nearly 25 years. There was nofamily history of lung disease. He had never worked withasbestos, and did not keep pets.

On examination, the patient was cyanosed and tachy -pnoeic. His hands and feet were clubbed. The JVP wasnot raised, and heart sounds were normal. On auscul -

1. What is the diagnosis?2. Which two investigations will you perform to

confirm the diagnosis?

1. What was the haematological diagnosis?2. How do you account for the abnormal

biochemistry?3. What is the cause of the renal failure?4. State two steps which should have been taken to

prevent this complication.

FEV1 (% predicted) 50 Blood gases on air:FVC (% predicted) 52 pH 7.45FEV1/FVC (%) 89 PaCO2 3.5 kPaTLC (% predicted) 56 PaO2 6.4 kPaKCO (% predicted) 60 Bicarbonate 22 mmol/lBronchoalveolar lavage revealed increased number

of cells which were predominantly neutrophilsChest X-ray (217)

tation of the lung fields there were bilateral fine end-inspiratory crackles at both lung bases.

Investigations are shown.

221166

221177

Page 225: Rapid review of clinical medicine for mrcp part 2

224

Answer 217

In the absence of an obvious underlying cause (occu pa-tional, drugs, connective tissue disorder) for the symp -toms and signs, the most likely diagnosis is CFA, which isa disorder of un known aetiology which usually occurs inmiddle age. It is characterized by progressively worseningdyspnoea, eventually leading to respiratory failure and corpulmonale from pulmonary hypertension. The cardinalsigns include cyanosis, clubbing and fine inspiratorycrackles at both lung bases. The chest X-ray demonstratesa ground-glass appearance in the mid-zones and bases ofthe lung which progresses to nodular shadows and streakyfibrosis. The end result is a ‘honeycomb appearance of thelung on the chest X-ray’.

Lung function tests demonstrate a restrictive lungdefect and a low KCO.

Blood gases demonstrate a low arterial oxygen contentand a normal CO2 content unless the patient developsend-stage respiratory failure. The quality of high-resolu -tion CT scans obviate the need for lung biopsy todiagnose the condition in the majority of patients, but inyounger patients biopsy may be useful in differentiatingthe condition from other causes of a similar chest X-rayappearance. Bronchoalveolar lavage reveals large numbersof neutrophils, and transbronchial biopsy will demon -strate cellular infiltration and thickening and fibrosis ofthe alveolar walls.

Some 50% of patients are positive for rheumatoidfactor, and 33% are positive for ANF.

CFA is associated with other auto-immune conditionssuch as coeliac disease, primary biliary cirrhosis, ulcerativecolitis and RTA.

The management is with prednisolone. Azathioprineor cyclophosphamide may be used as steroid-sparingdrugs. Most patients also require oxygen therapy. Themedian survival is five years.

Answer 216

Causes of lung fibrosis

• CFA• Extrinsic allergic alveolitis (farmer’s lung; bird-

fancier’s disease)• Sarcoidosis• Connective tissue disease (SLE, rheumatoid

arthritis, systemic sclerosis)• Vasculitides• Drugs (busulphan, bleomycin,

cyclophosphamide)• Pneumoconiosis• Asbestosis• Neurofibromatosis• Tuberous sclerosis• Histiocytosis• Ankylosing spondylitis• TB• Radiation

1. Acute myeloid leukaemia. Myeloblasts are large round cells with a round nucleusand prominent nucleoli. In addition, they may contain cytoplasmic inclusionsknown as Auer rods, which are almost pathognomonic of the condition.

2. Tumour lysis syndrome. The large numbers of blasts destroyed by chemotherapyliberate large amounts of intracellular contents into the bloodstream, as evidencedby the high potassium and phosphate concentrations. The nucleic acid from thecell nuclei is metabolized to produce urate, which is excreted by the kidney.

3. Hyperuricaemic nephropathy. High concentrations of urate may causecrystallization within renal tubules and acute renal failure.

4. i. Pre-hydration with intravenous fluids. ii. Allopurinol, a xanthine oxidase inhibitor that inhibits the synthesis of urate.iii. Rasburicase, a recombinant urate oxidase inhibitor.

1. i. Restrictive lung defect with reduced KCO.ii. Cryptogenic fibrosing alveolitis.

2. i. High-resolution CT scan.ii. Transbronchial lung biopsy.

Page 226: Rapid review of clinical medicine for mrcp part 2

Clinical Cases 225

Question 218

A 76-year-old female was admitted after being found on thefloor by the home help. She complained of frequent dizzyspells, particularly after standing from a recumbent position.She had experienced several falls at home, but neverrequired hospital admission. She had become house bounddue to the dizzy spells and falls and was increasinglydependent on the social services. She also complained oflethargy, hoarse voice, intermittent con fusion, and weightgain of almost 7 kg over the past seven months. She had ahistory of arthritis for which she took paracetamol when inpain. She was not taking any other regular medications.

On examination, she was pale and frail. The heart ratewas 60 beats/min and regular. The blood pressure was200/60 mmHg when lying, and 90/50 mmHg whenstanding. The JVP was not raised. The apex was displacedslightly. Auscultation of the heart revealed soft first andsecond heart sounds. There were no added sounds ormurmurs. Auscultation of the lungs was normal, as wasexamination of the abdomen. Central nervous systemexamination was essentially normal, with the exception ofbilateral absence of the ankle jerks.

Investigations are shown.

Hb 10 g/dlWCC 6 � 109/lPlatelets 149 � 109/lMCV 87 flSodium 118 mmol/lPotassium 5.3 mmol/lUrea 3 mmol/lCreatinine 69 μmol/lBilirubin 12 μmol/lAST 20 iu/lAlkaline phosphatase 100 iu/lAlbumin 36 g/lTotal protein 63 g/lChest X-ray (218)ECG Normal rhythm and

complexes

Question 219

A 69-year-old male was seen by his GP with pain in hiswrist after a fall. He complained of general ized weaknessand pains in his lower limbs over the past few months.Several investigations were ordered, the results of whichare as follows:

1. What is the most likely explanation for the dizzy spells and falls?

2. What is the explanation for the electrolyte abnormality?

3. Which single test would you request to confirm this?

4. What is the unifying diagnosis?

1. Give two possible diagnoses.2. Give three reasons to support the cause of this

patient’s raised IgG level.

Hb 13 g/dlWCC 7 � 109/lPlatelets 350 � 109/lESR 55 mm/hSodium 136 mmol/lPotassium 4 mmol/lUrea 6 mmol/lCreatinine 90 μmol/lCalcium 2.0 mmol/l

Phosphate 0.7 mmol/lAlkaline phosphatase 240 iu/lIgG 23 g/l IgM 2 g/l IgA 2.4 g/l Paraprotein 6 g/lX-ray of wrist Fracture radius, small

radiolucent area adjacent to fracture

221188

Page 227: Rapid review of clinical medicine for mrcp part 2

226

Answer 219

The diagnosis of osteomalacia is relatively straightforwardon the bone biochemistry, which classically reveals hypo -calcaemia, hypophosphataemia and a raised alkaline phos -phatase, the latter being secondary to an increase in PTHrelease in response to hypocalcaemia. Skeletal mani -festations of osteomalacia include Looser’s zones, whichare linear radiolucent areas seen most commonly in thebones of the forearms, scapulae, femurs and the pelvis.The raised ESR and IgG level associated with a very mildparaproteinaemia will confuse some readers, and com mit

them to make the erroneous diagnosis of multiplemyeloma. However, there are a few pointers to suggestthat the cause for the raised IgG level is MGUS ratherthan myeloma.

MGUS affects a very small proportion of the elderly. Itis characterized by benign monoclonal proliferation oflymphocytes, resulting in a modest increase in IgG levelsand an associated paraprotein of <20 g/l (in myeloma,the paraprotein level is usually >20 g/l). In contrast withpatients suffering from myeloma, these patients are notanaemic, they do not have immune paresis, i.e. the otherimmunoglobulins are not reduced because of monoclonalproliferation, Bence Jones proteins are classically absent(may be present in <10%), and the skeletal survey isnormal. The condition was originally termed benignmonoclonal gammopathy, but it is now recognized that10% of patients develop multiple myeloma, hence therelatively new term, MGUS.

Answer 218

The postural hypotension and hyponatraemia suggest thediagnosis of Addison’s disease. For causes and furtherdetails on Addison’s disease, see Answer 273. Theadditional history of weight gain, croaky voice, andenlarged heart on the chest X-ray suggests coexistenthypothyroidism. (Note: in pure Addison’s disease theheart is characteristically small owing to chronichypovolaemia; however, if hypothyroidism is also presentthe heart may be enlarged owing to a consequentcardiomyopathy or a pericardial effusion.) Approximately10% of cases of Addison’s disease due to auto-immuneadrenalitis are associated with other endocrinedeficiencies (Table). The association of auto-immuneadrenalitis with auto-immune hypothyroidism or insulin-dependent diabetes mellitus is termed Schmidt’ssyndrome or polyendocrine deficiency type 2.

The patient’s confusion could be explained by eitherAddison’s disease or hypothyroidism. Absent ankle jerks

are not uncommon in the elderly. In hypothyroidism, theankle jerk characteristically demonstrates slow relaxation;however, there may be absent reflexes owing to asecondary peripheral neuropathy. Other neurologicalmanifestations of hypothyroidism include cerebellarsyndrome and coma.

The polyendocrine deficiency syndromes

Polyendocrine deficiency syndrome (type 2) comprisesany two of the following• Auto-immune hypoadrenalism• Auto-immune hypothyroidism• Insulin-dependent diabetes mellitus

Polyendocrine deficiency syndrome (type 1) comprisesany two of the following• Auto-immune hypoparathyroidism• Auto-immune hypoadrenalism• Mucocutaneous candidiasis, alopecia, pernicious

anaemia

Vitiligo and hypogonadism may occur in bothforms

1. Postural hypotension.2. Addison’s disease.3. Short synacthen test.4. Polyendocrine deficiency type 2 (Schmidt’s

syndrome).

1. i. Osteomalacia.ii. Monoclonal gammopathy of undetermined

significance.2. i. Not anaemic.

ii. No evidence of immune paresis.iii. Paraprotein level below 20 g/l.

Page 228: Rapid review of clinical medicine for mrcp part 2

Clinical Cases 227

A 42-year-old male presented with a four-month historyof headaches, generalized weakness and polyuria. Onexamination, the physical examination was normal withthe exception of a blood pressure of 190/105 mmHg.

Initial investigations are shown.The patient was treated with a calcium antagonist, and

underwent further investigations which led to him havingabdominal surgery to cure his hypertension. The fullmacroscopic specimen of the abnormality removed isshown (220b).

Sodium 142 mmol/lPotassium 3.3 mmol/lUrea 4.0 mmol/lCreatinine 100 μmol/lBicarbonate 36 mmol/lChloride 107 mmol/lECG (220a)

1. What is the electrolyte abnormality?2. List four abnormalities on the ECG, and two

inferences you would make from these abnormalities.

3. What was the cause of his hypertension?

4. Which two investigations would he have had to confirm the diagnosis before surgery?

5. With all of the above information at hand, which antihypertensive agent should he have been prescribed?

Question 220

222200aa

222200bb

Page 229: Rapid review of clinical medicine for mrcp part 2

228

Answer 220

Hypokalaemic alkalosis and hypertension in the MRCPand similar examinations is highly suggestive of primaryhyperaldosteronism, which is often referred to as Conn’ssyndrome. The syndrome is caused by excessive secretionof aldosterone by the adrenal gland. This leads to increased salt and water retention, causing hypertensionand clinical symptoms of hypokalaemia which includemuscle weakness, polyuria and polydipsia from nephro -genic diabetes insipidus, and paraesthesia. Acceleratedhypertension may complicate primary hyperaldo steronism,and cardiac arrhythmias may result from hypo kalaemia.The serum renin level in primary hyper aldosteronism issuppressed, whereas in conditions causing secondaryhyperaldosteronism the renin is elevated. Primary

hyperaldosteronism may be caused by an autonomousaldosterone-secreting adrenal adenoma or bilateralhyperplasia of the adrenal gland. Endocrine causes ofhypertension are tabulated (Table C).

The diagnosis is made by simultaneous measurementof serum renin and aldosterone levels in the lying andstanding positions. The renin is low, and the aldosteronelevel is high, in the supine position. On standing sud -denly, there is a paradoxical drop in the aldosterone levelin patients with an adenoma and an exaggerated rise inaldosterone levels in patients with hyperplasia. Imagingwith CT scanning is very useful in distinguishing betweenadenoma and hyperplasia. Adrenal adenomas >0.5 cm indiameter can be diagnosed by high-resolution CT scan.Adrenal vein sampling is only useful when there is doubtabout the existence of an adenoma.

Iodolabelled cholesterol scintiscan is not a very sensi -tive method for identifying Conn’s tumours.

The management of adrenal adenoma is surgical exci -sion, and the management of hyperplasia is with a high-dose aldosterone antagonist, such as spironolactone.

1. Hypokalaemic alkalosis (see Answer 275).2. i. First-degree atrioventricular block. The PR-

interval is much greater than 200 ms. There are many causes of first-degree atrioventricular block(Table A); however, in this case the most likely cause is hypokalaemia.

ii. There is voltage criteria of left ventricular hypertrophy. The sum of the S-wave in V1 and the R-wave in V5 or V6 exceeds 0.35 mV (each small square on the ECG is 0.1 mV), which conforms with the Sokolow criteria for left ventricular hypertrophy.

iii. There is ST-segment depression in the lateral leads which may be secondary to left ventricular hypertrophy or to hypokalaemia. Although myocardial ischaemia is also a recognized cause of ST-segment depression, it is unlikely in the context of this question.

iv. There are prominent U-waves (upright waves immediately following the T-wave, 220carrowed) which are suggestive of hypokalaemia. Other ECG changes in relation to metabolic abnormalities are tabulated below (Table B).

3. Primary hyperaldosteronism or Conn’s syndrome secondary to an adrenal adenoma. Conn’s tumours are bright yellow as opposed to phaeochro-mocytomas, which are a greyish-purple colour (220d).

4. i. Lying and standing plasma renin and aldosterone levels.

ii. High-resolution abdominal CT scan.5. High doses of spironolactone, an aldosterone

antagonist.

222200cc

222200dd

Page 230: Rapid review of clinical medicine for mrcp part 2

Clinical Cases 229

Question 221

A 15-year-old farmer’s daughter was admitted to theinfectious diseases unit with a five-day history of right

upper quadrant pain. On examination, she was pyrexialwith a temperature of 38.4°C (101.1°F). She appearedunwell and jaundiced.

Investigations are shown.

Table A Causes of first-degree atrioventricular block

• Ischaemic heart disease• Cardiac conduction tissue disease (sinoatrial

disease)• Rheumatic fever• Myocarditis• Drugs• Hypokalaemia• Athlete’s heart

Table B ECG changes in relation to metabolicabnormalities frequently encountered in theMRCP and similar examinations

Hypokalaemia• Peaked P-waves• Long PR-interval• Increased duration of the QRS-complex• U-waves• Long QT-interval• Flattened T-waves• ST-segment depression

Hyperkalaemia• Flat P-waves• Peaked T-waves• Increased duration of the QRS-complex• Sinusoidal QRS-complexes (pre-arrest)

Hypocalcaemia• Long QT-interval

Hypercalcaemia• Short QT-interval

Table C Endocrine causes of hypertension

• Conn’s syndrome• Cushing’s syndrome• Phaeochromocytoma• Acromegaly• Thyrotoxicosis• Hyperparathyroidism• 11-β- and 17-β-hydroxylase deficiency

Hb 8 g/dlWCC 18.8 � 109/lPlatelets 140 � 109/lReticulocyte count 20%Blood film No fragmented red cells

seenPT 36 s (control 12 s)APTT 59 s (control 45 s)Sodium 136 mmol/lPotassium 4.9 mmol/lUrea 26 mmol/lCreatinine 400 �mol/lBilirubin 50 mmol/lAST 900 iu/lAlkaline phosphatase 300 iu/lAlbumin 33 g/lHepatitis B sAg AbsentHCV Ab AbsentCMV serology AbsentMonospot test Negative

1. Suggest at least two possible diagnoses.2. Give three possible causes for the anaemia.

Page 231: Rapid review of clinical medicine for mrcp part 2

230

Answer 221

The patient has evidence of hepatitis together with renalfailure and a coagulation disorder. The clinical scenariowhich would fit this triad is fulminant hepatic failurecomplicated by the hepatorenal syndrome. The majorityof cases are due to viral hepatitis (hepatitis A, B, C andD, although EBV, CMV, Coxsackie and herpes simplexvirus have been implicated; Lassa fever, Ebola virus andMarburg virus should be considered in a patient who hasrecently returned from Africa), and drugs (paracetamol,iron, tetracycline, isoniazid, rifampicin, halothane). Otherrecognized causes include toxins (alcohol, carbontetrachloride), leptospirosis (Weil’s disease), Wilson’sdisease, and pregnancy. In a farmer’s daughterleptospirosis causing hepatic and renal failure (Weil’sdisease) is a distinct possibility, but hepatitis A and drugoverdose should also be borne in mind.

Leptospirosis is caused by the spirochaete Leptospiraicterohaemorrhagica. Rodents, particularly rats, are themost important reservoir of infection. Other hostsinclude cattle, pigs, goats, hamsters, mice, hedgehogs,foxes and skunks. The organism is excreted in the urineand may survive in the soil for several weeks. Entry intothe human host is through cuts and abrasions on theskin, or through intact mucous membranes. Occupationsmost susceptible are sewage workers, fishermen, vets andfarmers. Replication occurs in the blood and tissue andmulti-system involvement may occur. The kidneys andliver are most affected. Glomerular injury occurs firstcausing an acute interstitial nephritis and tubularnecrosis. In the liver there is evidence of centrilobularnecrosis in severe cases.

The incubation period is 7–12 days. The initial(septicaemic) phase is 4–7 days in duration. It ischaracterized by fever, headaches, myalgia, abdominalpain, vomiting, skin rash (macular, maculopapular orhaemorrhagic) or conjunctival injection. Fever is high,myalgia is present in over 85% of cases and a majorhallmark of the disease, and the headache may be severe.Persistent headache is suggestive of meningitis.Proteinuria and haematuria may be present during thisphase, and renal failure is evident in just over 50% ofpatients. Jaundice and impairment of liver function arepresent only in severe cases. Some 90% of cases areanicteric. Hepatosplenomegaly is present inapproximately 20%. Respiratory involvement is common,and manifests as a dry cough, haemoptysis and confluentshadow on the chest X-ray. The second phase of thedisease (immune phase) varies between 4 and 30 days.The patient is afebrile. Antibody titres to Leptospira arerising. Deterioration in liver and renal function maycontinue. Meningism, uveitis and a rash are common.Haematological manifestations include thrombo -cytopenia, haemolytic uraemic syndrome characterized byfragmented red cells on the blood film and intravascularhaemolysis. Endothelial injury may cause blood loss fromthe gastrointestinal tract and the lung.

Blood cultures are positive in the first week, and bythe second week the urine culture is also positive. IgMantibodies to Leptospira are detectable in the first week.There is a rising titre in the second week. Treatment iswith high-dose penicillin.

Brucellosis is also recognized in farmers. Thebacterium (Gram-negative bacillus) infects mammals,particularly cows, and can be contracted from contactwith infected urine or drinking unpasteurized milk. Acutebrucellosis is also characterized by fever, headache andmyalgia; however, it is unlikely in this case owing to theabsence of a high fever, leucopenia, and assumed absenceof hepatosplenomegaly, which is common in brucellosis.In addition, abnormal liver function or jaundice isextremely rare in brucellosis. Chronic brucellosis isassociated with lassitude, headaches, depression, nightsweats, skeletal, cardiovascular, neurological,genitourinary, pulmonary and intra-abdominalcomplications. Brucellosis has been eradicated in the UKowing to mandatory pasteurization of milk.

1. i. Leptospirosis (Weil’s disease).ii. Acute hepatitis A infection.iii. Paracetamol overdose.iv. Iron overdose.

2. i. Blood loss from endothelial damage resulting in haemorrhage.

ii. Haemolysis.iii. Disseminated intravascular coagulation causing

blood loss.

Page 232: Rapid review of clinical medicine for mrcp part 2

Clinical Cases 231

A 70-year-old male developed atrial fibrillation 12 hoursafter thrombolysis for an anterior myocardial infarction.He did not have chest pain. He had a history ofhypertension and chronic obstructive airways disease. Theheart rate measured 122 beats/min. The blood pressuremeasured 130/80 mmHg and the chest was clear.

Question 223What immediate treatment is recommended for themanagement of the atrial fibrillation?

a. IV digoxin.b. DC cardioversion.c. IV amiodarone.d. IV flecanide.e. IV metoprolol.

A 60-year-old male was investigated for a two-yearhistory of increasing breathlessness. He worked as aminer and smoked 20 cigarettes per day. His past medicalhistory included psoriasis and rheumatoid arthritis. Hismedications included salbutamol inhaler, Becotideinhaler, methotrexate and folic acid.

Investigations are shown.

Question 222

Hb 16 g/dlWCC 6.9 � 109/lPlatelets 412 � 109/lMCV 98 flSodium 138 mmol/lUrea 6 mmol/lCalcium 2.31 mmol/lAlbumin 35 g/lBilirubin 18 mmol/lAlkaline phosphatase 190 iu/lAST 30 iu/lChest X-ray (222a)CT Thorax (222b)

What is the diagnosis?a. Pleural fibrosis.b. Rheumatoid induced pulmonary fibrosis.c. Large pleural effusion.d. Emphysema.e. Caplan syndrome.

A 44-year-old male was admitted to the Coronary CareUnit following an inferior myocardial infarction. He wasa smoker and consumed up to 20 units of alcohol perday. On the third day of admission he became graduallyclammy. He was aggressive towards the nursing staff. Hisheart rate was 130 beats/min. The blood pressure was160/100 mmHg. Oxygen saturation on air was 96%.

Question 224

What is the best management of this patient?a. Intravenous thiamine.b. Intravenous chlormethiazole.c. Oral diazepam.d. Intravenous midazolam.e. Intramuscular haloperidol.

222222aa

222222bb

Page 233: Rapid review of clinical medicine for mrcp part 2

232

The patient has developed rapid atrial fibrillation shortlyafter myocardial infarction. He is surprisinglyasymptomatic. DC cardioversion is only indicated if thereis ongoing myocardial ischaemia or heamodynamiccompromise. In the absence of these features the aim isto control the ventricular rate to reduce myocardialoxygen consumption and to prevent systemicthromboembolism.

The most effective drugs to slow ventricular rate arebeta-blockers, but these are relatively contraindicated inthis patient, who also has COPD. Although diltiazem iseffective, there have been recent reports suggesting thatcalcium channel antagonists may be associated with anincreased mortality following myocardial infarction.

Digoxin is also highly effective and would be the drug ofchoice in this situation. In most cases AF following MI istransient, therefore chronic anti-arrhythmic therapy is notusually required. If the AF persists then electivecardioversion should be considered at a later date (usuallyafter six weeks in the context of a recent myocardialinfarction) ensuring that the patient has beenanticoagulated throughout.

There are no trials assessing the effect of amiodaronein the management of AF following MI. Amiodaroneshould be reserved for patients who have severalparoxysms of AF following MI (based on anecdotalevidence).

Flecanide is contraindicated after MI as it has beenassociated with an increased mortality in this particularsituation.

Answer 223

a. IV digoxin.

In a patient with rheumatoid arthritis, a history of heavysmoking and an occupational history of mining there areseveral potential causes of dyspnoea, which includerheumatoid lung disease (see Answer 320), occupationaldust induced fibrosis and obstructive airways disease. The

chest X-ray shows reduced lung markings in the leftupper and lower lobes and to some extent in the rightupper lobe. The CT scan of the thorax demonstrateslarge emphysematous bullae in the left upper lobe andsome emphysematous changes in the right upper lobe.

Answer 222

d. Emphysema.

The patient presents with features of delirium tremens(aggressive behaviour, sweating, tachycardia andhypertension), the best treatment for which is therapy withbenzodiazepines. Short-acting agents such as lorazepamare usually not recommended because they have to begiven relatively frequently. Chlormethiazole was commonlyused in the 1980s but there have been adverse effects withrespiratory suppression, therefore the drug is no longerrecommended for alcohol withdrawal. Diazepam is

advocated for use in DT but its long-acting duration maycause oversedation. The best treatment is a benzodiazepinewith an intermediate effect such as chlordiazepoxide, butthis is not one of the options given. Intravenousmidazolam may cause marked sedation and is potentiallyharmful in this situation. Neuroleptic agents reduce thethreshold for epileptic seizures. They should be avoided inDT. While all patients with alcohol dependence should betreated with thiamine to prevent Wernicke’sencephalopathy, this particular scenario calls for specificmanagement of delirium tremens.

Answer 224

c. Oral diazepam.

Page 234: Rapid review of clinical medicine for mrcp part 2

Clinical Cases 233

A 16-year-old girl was admitted with a 3-hour history ofvomiting followed by collapse. She had recently split upfrom her boyfriend and was thought to have taken anoverdose of digoxin that her father had been prescribedfor atrial fibrillation.

On examination she was drowsy and confused. Herpulse was 36 beats/min. The blood pressure was70 systolic.

Investigations are shown.

Question 226

Sodium 133 mmol/lPotassium 4.8 mmol/lUrea 5.6 mmol/lCreatinine 100 �mol/lDigoxin level 5 nmol/lECG High second degree AV block;

VR 36/min

What is the immediate management of her condition?a. Activated charcoal via nasogastric tube.b. IV atropine.c. IV isoprenaline.d. IV dextrose 50% (50 ml).e. IV digoxin specific antibody fragments (Digibind).

A 23-year-old male was admitted with a 24-hour historyof cough, fever and confusion. He had a history ofepilepsy that was well controlled on carbamazepine.Looking through his records there were several transientadmissions for deliberate self-harm, including a history ofEcstasy abuse. According to a friend he consumed up to70 units of alcohol per week.

On examination he was pale. The heart rate was110/min and regular. The blood pressure measured96/60 mmHg. There was reduced air entry over theanterior aspect of the right chest.

Investigations are shown.

Question 225

Hb 12 g/dlWCC 14 � 109/lPlatelets 200 � 109/lMCV 90 flSodium 134 mmol/lPotassium 3.8 mmol/lUrea 7 mmol/lCreatinine 120 �mol/lBilirubin 15 mmol/lAST 140 iu/lALT 182 iu/lGamma GT 312 iu/l

Arterial blood gases on air:pH 7.4PaO2 9.2 kPaPaCO2 4.8 kPaBicarbonate 21 mmol/l

Chest X-ray (225a, b)

Which antibiotic regime would you prescribe for thepatient?

a. IV cefuroxime.b. IV amoxycillin.c. IV clarithromycin.d. IV cefuroxime and clarithromycin.e. IV amoxycillin and metronidazole.

222255aa

222255bb

Page 235: Rapid review of clinical medicine for mrcp part 2

234

Answer 226

Digoxin toxicity is characterized by gastrointestinaldisturbance such as anorexia, nausea, vomiting anddiarrhoea, blurred vision, xanthopsia, dizziness,confusion and delirium. Cardiac arrhythmias are usuallythe most serious complication and are the commonestcause of mortality in digoxin toxicity. Almost anyarrhythmia is possible. Ventricular extra-systoles are thecommonest rhythm disturbance. The presence ofsupraventricular tachyarrhythmias and atrioventricularblock in a patient taking digoxin are highly suggestive ofdigoxin toxicity.

The management of digoxin therapy includesprevention of absorption of the drug, correction ofelectrolyte disturbances that promote digoxin toxicity oroccur as a result of digoxin toxicity, and management ofarrhythmias. A small proportion of patients may requiredigoxin specific antibody fragments (Table).

In severe digoxin toxicity, the inhibition of theNa+/K+ ATPase can result in hyperkalaemia that shouldbe treated conventionally with 50 ml of 50% dextroseand 16–18 units of short-acting insulin. Severebradyarrhythmias (HR <40 beats/min), as in thissituation, should be treated with IV atropine. Temporarycardiac pacing is necessary in patients unresponsive toatropine. Isoprenaline is contraindicated, as it mayprecipitate tachyarrhythmias.

Verapamil is the drug of choice in patients withsupraventricular arrhythmias. Lignocaine and phenytoinare effective in the management of ventriculartachycardia. Electrical cardioversion is indicated inpatients with haemodynamic compromise but withrelatively low voltage DC shocks, e.g. 50 J, as there is arisk of asystole.

A small group of patients require the specific digoxinbinding antibody fragments, which rapidly bind tocirculating digoxin and prevent its effects on the heart.They are indicated in patients who have taken a largeoverdose (10 mg in adults; 4 mg in children), those withplasma digoxin >13 nmol/l, and in patients with K+ level>5 mmol/l in the presence of l ife-threateningarrhythmias such as high AV block and VT/VF. They aregiven IV over 30 minutes but can be given as a bolus inthe event of a cardiac arrest.

Digoxin binding antibody fragments are smallmolecules that pass through the glomerular basementmembrane and are excreted via the kidneys. They can besafely given to patients with renal impairment includingthose who are dialysis dependent.

In this particular case the immediate management isatropine followed by temporary pacing if the heart blockpersists.

b. IV atropine.

Management of digoxin toxicity

Action Technique/drugPrevent absorption of drug Gastric lavage or

ipecacuanhaActivated charcoal

if ingestion <6–8 hours

Correct electrolytes Correct K+ and Mg2+

Manage bradyarrhythmias AtropineTemporary cardiac pacingManage SVTs VerapamilManage VT Lignocaine

PhenytoinIn very severe cases Digoxin binding

antibody fragments (see above)

The patient has evidence of a right upper lobeconsolidation, which would usually be treatedconventionally with IV cefuroxime and IV clarithromycinin accordance with the guidelines set by the BritishThoracic Society. However, he has several risk factors foraspiration, notably alcohol and recreational drug abuse.Aspiration pneumonia is caused by anaerobic bacteriathat are not effectively treated with cephalosporins,macrolides, aminoglycosides or the quinolones.

Antibiotics of choice in the management of aspirationpneumonia include a combination of amoxycillin andmetronidazole, or clindamycin or imipenem.

The coexistence of confusion and liver function testsmay have led to a diagnostic dilemma between aspirationpneumonia and pneumonia due to Legionellapneumophila. However, the possibility of an unprotectedairway (unconsciousness) in this individual means thataspiration pneumonia should be considered above thediagnosis of Legionnaire’s disease.

Answer 225

e. IV amoxycillin and metronidazole.

Page 236: Rapid review of clinical medicine for mrcp part 2

Clinical Cases 235

Question 228

A 30-year-old male with a family history of adultpolycystic kidney disease wished to be screened for thecondition. His mother was affected and a paternal aunthad recently suffered a subarachnoid haemorrhage as aresult of a ruptured berry aneurysm.

What is the standard screening investigation ofchoice?

a. CT scan abdomen.b. Renal ultrasound.c. CT scan brain.d. Genotype for APCKD-1 gene.e. Urinalysis.

Below is an ECG (229) from a patient presenting withsyncope.

Question 229

What is the most likely cause of the syncope?a. Complete heart block.b. Torsades de pointes.c. Posterior myocardial infarction.d. Rapid atrial fibrillation.e. Monomorphic ventricular tachycardia.

A 65-year-old smoker with exertional dyspnoea wasinvestigated with a chest X-ray (227).

Question 227

What is the radiological diagnosis?a. Left upper lobe consolidation.b. Left upper lobe collapse.c. Left lower lobe collapse.d. Left-sided pneumothorax.e. Left pleural effusion.

222277

222299

Page 237: Rapid review of clinical medicine for mrcp part 2

236

The patient has adult polycystic kidney disease. 86% ofpatients with APCKD have an abnormality in a geneadjacent to the alpha globin gene in chromosome 16(PKD-1 gene). The other patients with APCKD (PCKD-2)have an abnormal gene on chromosome 4. In PCKD-2cysts and end-stage renal failure occur late (in the sixth orseventh decade), thus screening with ultrasound may missyoung PCKD-2 patients. However, this is not felt to beparticularly important since most patients with thisgenotype remain well throughout life.

With respect to APCKD-1, screening is based on thedemonstration of cysts in one or both kidneys (dependingupon age) using CT scan or renal ultrasound. In childrenCT scan of the abdomen is the screening test of choice assmall cysts may be missed using ultrasound. By the age of30–35 years all patients with APCKD-1 have visible cystson renal ultrasound; therefore, for adults in or above thisage group renal ultrasound is the screening test of choice.Even in patients aged 20, the false-negative rate with renalultrasound is only 4%. See Table for ultrasound criteria fordiagnosis of polycystic kidney disease.

Genetic testing for APCKD-1 is 99% accurate but is notwidely available, and is currently reserved for screeningchildren in whom CT scan may be normal and those whoare potential transplant donors for their families.

APCKD is associated with intracranial aneurysms.Approximately 8% of patients with APCKD haveintracranial aneurysms, which is four to five times higherthan the general population. Ruptured aneurysms areresponsible for around 6% of all deaths due to APCKD.Patients with APCKD and a family history of cerebral

haemorrhage should be offered screening with magneticresonance angiography of the brain or spiral CT scan ofthe brain (see algorithm for screening for intracranialaneurysms in patients with APCKD).

Answer 228

b. Renal ultrasound.Ultrasound criteria for the diagnosis of APCKD

Age Criterion<30 at least two cysts (unilateral or bilateral)30–59 at least two cysts in each kidney≥ 60 at least four cysts in each kidney

The patient has a bifascicular block (left anteriorhemiblock and RBBB) and a prolonged PR interval.

There is a high probability that the syncopal episode isdue to complete heart block resulting from impairedconduction through the third fascicle (the left posteriorfascicle). Slow heart rates can also cause prolonged QTand torsades de pointes, but the QT interval is normal.

Answer 229

Algorithm for screening for intracranialaneurysms in patients with APCKD

Intercerebralaneurysms?

Repeat every5 years

For cerebral angiography anddiscussion with neurosurgeons

Yes

Yes

No

Screening notindicated

No

Age 18–55 years andfamily history of

intracranial aneurysmsor haemorrhage

MRA brain orspiral CT brain

scan

There are three main features that are related to loss oflung volume: elevation of the left hemidiaphragm;

elevation of the left hilum; and reduction in the spacesbetween the ribs in the left upper zone. There is a veil-like shadow affecting the upper part of the lung, whichrepresents a collapsed left upper lobe.

Answer 227

b. Left upper lobe collapse.

a. Complete heart block.

Page 238: Rapid review of clinical medicine for mrcp part 2

Clinical Cases 237

Question 230

A 48-year-old male, psychiatric patient, living in aninstitution, was witnessed to have a fit for the first time.He had been treated with chlorpromazine for the past

four years. Three hours later the patient remainedunconscious and was admitted to hospital where bloodresults were as follows:

Sodium 115 mmol/lPotassium 3.4 mmol/lUrea 1.1 mmol/lCreatinine 60 �mol/lGlucose 4.1 mmol/lPOsm 248 mOsm/lRenin 90 pmol/l (NR 100–500 pmol/l)Aldosterone 50 pmol/l (NR 100–500)TSH 3 mu/l (NR 0.5–5.0 mu/l)Urine sodium 10 mmol/lUrine osmolality 90 mOsm/l

What is the diagnosis? a. Chlorpromazine-induced ADH secretion.b. Syndrome of inappropriate ADH secretion.c. Lithium toxicity.d. Diabetes insipidus.e. Compulsive water drinking.

A 56-year-old woman presented with a four-day historyof breathlessness, cough and a high fever. She gave a longhistory of indigestion and intermittent dysphagia to solidsand fluids. On several occasions undigested food wouldbe regurgitated back shortly after she had eaten. Shewould often wake at night with episodes of coughing andspluttering. There was no history of weight loss.

On examination, she had a temperature of 39°C(102.2°F). The respiratory rate was 30/min, and theheart rate was 102 beats/min. Chest expansion wasmoderate and symmetrical. On auscultation of the lungfields there were coarse bilateral basal crackles.

The chest X-ray demonstrated bilateral lower lobeconsolidation.

Question 231

1. What is the cause of this patient’s illness?a. Hiatus hernia.b. Benign oesophageal stricture.c. Achalasia.d. Pharyngeal pouch.e. Bronchiectasis.

2. What investigation would be most useful in identifying the cause of her presentation?

a. Oesophagoscopy.b. Barium swallow.c. Oesophageal manometry.d. Bronchoscopy.e. CT scan of the thorax.

Page 239: Rapid review of clinical medicine for mrcp part 2

238

The patient has a very low serum sodium and low plasmaosmolality (calculated 242 mOsm/l). The maindifferential diagnoses are SIADH and primary polydipsia,although hypothyroidism and cortisol deficiency may alsopresent with similar plasma biochemistry. In compulsivewater drinking (primary polydipsia), ADH production iscompletely inhibited and urine osmolality is very low(<100 mOsm/l). In this particular case the very lowurine osmolality is consistent with the diagnosis ofcompulsive water drinking. This is a relatively commonproblem in patients with schizophrenia.

Although chlorpromazine can cause SIADH, the lowurine osmolality is against the diagnosis. Patients with

SIADH have concentrated urine (high POsm) and urinesodium >40 mmol/l. Patients with diabetes inspidushave dilute urine but high plasma osmolality and plasmasodium. Patients with mineralocorticoid deficiency havelow sodium, low plasma osmolality but high urinesodium and osmolality. In this case the renin andaldosterone levels are suppressed owing to the profoundhyponatraemia caused by excessive water intake. Excessgastrointestinal fluid loss would cause dehydration andhyponatraemia resulting in salt and water retention bythe kidneys and hence a high urine osmolality and lowurine sodium (<20 mmol/l).

Diabetes insipidus is characterized by a normal orelevated plasma osmolality but a low urine osmolality(Table) (see Question 401).

Answer 230

e. Compulsive water drinking.

Plasma sodium/osmolality, urinary sodium and urinary osmolality in various conditions

P Na/Osm U Na U OsmDiabetes insipidus Normal/high Low LowSIADH Low High HighHypothyroidism Low High HighHypoadrenalism Low High HighEffective volume depletion* Low/normal Low HighWater intoxication Low Low Low

*Includes dehydration from GI loss, excess perspiration, water loss, CCF, hypoalbuminaemic states. Exceptions include renal failure due to interstitial nephritis, where sodium excretion is increased even in

dehydration.

A history of dyspepsia, dysphagia, oesophageal reflux anda possible aspiration pneumonia is consistent with thediagnosis of achalasia. Hiatus hernia may cause reflux andoccasionally nocturnal cough and aspiration pneumonia;however, dysphagia is not a feature unless recurrentreflux has caused a peptic stricture. Peptic stricture maycause all the symptoms of achalasia; however, there isusually a long history of indigestion before the onset ofdysphagia. Pharyngeal pouch does not cause retrosternalpain, although dysphagia and regurgitation are common.

A barium swallow is the best method of differentiatingbetween achalasia, peptic stricture, hiatus hernia andpharyngeal pouch. Although it is relatively clear that thediagnosis is achalasia, oesophageal manometry scoresfewer marks than barium swallow or oesophagoscopybecause other causes of dysphagia – particularly

oesophageal stricture – would have to be excluded beforeentertaining a diagnosis of achalasia. In achalasia, thebarium demonstrates dilatation of the oesophagus andlack of peristalsis. The lower end gradually narrows owingto failure of the lower oesophageal sphincter to relax.Oesophageal manometry is useful for measuring motility.

Achalasia is a motility disorder affecting theoesophagus. It is characterized by lack of peristalsis andfailure of the lower oesophageal sphincter to relax.Pathologically, there is degeneration of the ganglioniccells in the nerve plexus of the oesophageal wall. Theaetiology is unknown. The disease often presents inmiddle age. There is a long history of retrosternal pain,intermittent dysphagia, oesophageal reflux and aspiration.Weight loss is unusual. Diagnosis is by a combination ofbarium swallow and oesophageal manometry. Treatmentis by dilating the lower oesophagus by pneumaticinflation. In unsuccessful cases, surgical division of thelower end of the oesophagus (Heller’s operation) isrequired.

Answer 231

1. c. Achalasia.2. b. Barium swallow.

Page 240: Rapid review of clinical medicine for mrcp part 2

Clinical Cases 239

Question 233

A 38-year-old man was investigated for lethargy andsevere lower back pain. He had a past medical history ofHodgkin’s lymphoma (stage 1A) which had been treated

with radiotherapy one year previously. On examinationhe had a tender lumbar spine and palpable lymph nodesin the left axilla and both inguinal areas.

Investigations are shown.

Hb 9 g/dlWCC 14 � 109/lPlatelets 80 � 109/lMCV 88 flReticulocyte count 0.4%Blood film Nucleated red cells;

metamyelocytes and myeloblasts

What is the cause of the anaemia? a. Anaemia of chronic disease.b. Hypersplenism.c. Haemolysis.d. Marrow infiltration.e. Folate deficiency.

A 15-year-old boy was admitted to the intensive care unitfor the second time with meningococcal septicaemia.

Investigations on admission were as follows:

Question 234

Hb 14.8 g/dlWCC 18 � 109/l (neutrophils 80%)Platelets 190 � 109/l

What investigation would you perform to identify thecause?

a. White cell count.b. Neutrophil chemotactic factor.c. Serum complement level.d. Serum immunoglobulins.e. Neutrophil antibodies.

Question 235

Below is an ECG (235) from a 78-year-old patient with a slow pulse rate.

What are the two ECG diagnoses?a. Complete heart block.b. Left atrial enlargement.c. Mobitz II second degree AV block.d. 2:1 second degree AV block.e. Left ventricular hypertrophy.f. Left axis deviation.g. Inferior myocardial infarction.h. Anterior myocardial infarction.i. First degree AV block.j. Mobitz I second degree AV block.

A 16-year-old girl who had undergone previous surgeryfor congenital heart disease presented with fatigue. Onexamination she had a heart rate of 40 beats/min. Theblood pressure was 110/70 mmHg. There was noevidence of heart failure. The ECG showed completeheart block with a ventricular rate of 40/min.

Question 232

What is the management?a. Dual chamber permanent pacemaker.b. Atrial pacemaker.c. Single chamber ventricular pacemaker.d. Reassure.e. Observe at six-monthly intervals.

223355

Page 241: Rapid review of clinical medicine for mrcp part 2

The patient has a past history of Hodgkin’s lymphomaand presents with back pain. There is evidence of anormochromic, normocytic anaemia. The blood filmshows evidence of immature red and white cells, i.e. aleucoerythroblastic blood picture. In the context of thisquestion, the most likely cause of the leucoerythroblasticblood picture is marrow infiltration (Table).

240

Answer 233

d. Marrow infiltration. Causes of a leucoerythroblastic blood picture

• Bone marrow infiltration from malignant cells• Myelofibrosis• Marble-bone disease (osteopetrosis)• Certain metabolic storage disorders,

e.g. Gaucher’s disease• TB• Septicaemia

There are two P waves to every QRS complex. Thepatient also satisfies the Sokolow–Lyon voltage criteriafor left ventricular hypertrophy.

Patients with C5 to C9 complement deficiency are proneto recurrent Neisseria infections.

Hypocomplementaemia may be inherited or acquired.Inherited hypocomplementaemia is associated with a

predisposition to auto-immune disorders and bacterialinfections (Table). Acquired causes of hypo -complementaemia include glomerulonephritis,haemolytic uraemic syndrome and its variants, severesepsis, pancreatitis, advanced renal disease and athero-embolic renal disease.

Answer 234

c. Serum complement level.

Hereditary causes of hypocomplementaemia

Deficiency DisorderC2 deficiency Auto-immune diseases e.g. SLEC1 q, r or s deficiency SLEC3 deficiency Severe recurrent infections with encapsulated bacteria shortly

after birthC4 deficiency Auto-immune disorders such as Henoch–Schönlein

syndrome, membranous glomerulonephritis, sclerodermaC5–C9 deficiencies Predisposition to Neisseria spp. infections (manifests in late teens usually)

Answer 235

d. 2:1 second degree AV block.e. Left ventricular hypertrophy.

Answer 232

The patient has had prior cardiac surgery and presentswith symptoms of fatigue and a heart rate of40 beats/min secondary to third degree AV block. Allforms of acquired third degree AV block should bepaced. The annual mortality from acquired third degreeblock is 15% in the absence of cardiac pacing.

Congenital third degree heart block may not requirepacing if the ventricular complexes are narrow and therate is >50 beats/min. In the presence of broadventricular complexes or a heart rate of <50 beats/min,pacing is indicated.

a. Dual chamber permanent pacemaker.

Page 242: Rapid review of clinical medicine for mrcp part 2

Clinical Cases 241

Question 238

A 16-year-old female was referred with primaryamenorrhoea and hirsutism. The patient had noticedexcessive hair on her face, arms and legs since the age of8 years. She had a past history of measles but had beenwell otherwise. She lived in a mountain hut with herfather and aunt. She had two older siblings who werewell. Her father was 1.8 m tall. On examination she hadexcessive acne and facial hair. The blood pressure was105/65 mmHg. On inspection of the genitalia, therewas evidence of pubic hair up to the umbilical region andmoderate cliteromegaly.

What is the diagnosis?a. Congenital adrenal hyperplasia secondary to

21-hydroxylase deficiency.b. Testosterone secreting adrenal tumour.c. Arrhenoblastoma.d. Polycystic ovarian syndrome.e. Cushing’s syndrome.

A 41-year-old schoolteacher was seen by arheumatologist for intermittent painful blue fingersduring exposure to the cold weather. She had alsorecently developed fixed flexion deformities of her fingersbilaterally and thickening on the dorsal aspects of thehands. She was subsequently commenced onpenicillamine and a non-steroidal anti-inflammatorydrug. Two months later she complained of breathlessnesson exertion and retrosternal burning pain on stoopingforward. On auscultation of the lung fields there werefine end-inspiratory crackles at both lung bases. Lungfunction tests revealed a restrictive defect with reducedtransfer factor.

Question 236

What is the most probable diagnosis? a. Penicillamine-induced lung fibrosis.b. Bronchiolitis obliterans.c. Mixed connective tissue disease.d. Systemic sclerosis.e. Pulmonary fibrosis secondary to NSAID use.

Question 237

An 83-year-old woman was admitted with central chestpain radiating to her left arm while out shopping. Shehad never experienced chest pain previously. There wasno history of smoking, hypertension or diabetes. Theserum cholesterol was 8 mmol/l.

Physical examination was normal. The 12-lead ECGshowed flattening of the T waves in leads I, V5 and V6.The troponin level 12 hours after the pain was notelevated. A diagnosis of angina was made. The patientremained stable and pain free over the next 48 hours. Anexercise test performed on the third day of admissionrevealed ST segment depression >2 mm in leads V4–V6after 4 minutes of exercise (HR 120 beats/min). Thepatient did not experience chest pain on the treadmill.

Which of the medications below is unlikely to alter herprognosis?

a. Atenolol.b. Nifedipine.c. Ramipril.d. Aspirin.e. Simvastatin.

Page 243: Rapid review of clinical medicine for mrcp part 2

242

Answer 238

The differential diagnosis of hirsutism and virilismincludes congenital adrenal hyperplasia, polycystic ovarysyndrome, androgen secreting adrenal tumour,Cushing’s syndrome, arrhenoblastoma (ovarian tumoursecreting androgens) and anabolic drug abuse. Thepatient has presented with primary amenorrhoea and hadevidence of virilism at a relatively young age, whichwould be consistent with 21-hydroxylase deficiencycongenital adrenal hyperplasia. PCOS rarely presents

with primary amenorrhoea. Androgen secreting tumoursof the ovary are malignant and would not have such along history. Furthermore, they usually present at anolder age. Androgen secreting adrenal tumours are alsousually malignant and usually also produce cortisol,therefore affected females are virile and have features ofCushing’s syndrome.

The diagnosis of congenital adrenal hyperplasiasecondary to 21-hydroxylase deficiency can be establishedby demonstrating high levels of 17-hydroxyprogesteronein the blood and an increased concentration of keto-steroids in the urine. (See Question 334.)

a. Congenital adrenal hyperplasia secondary to 21-hydroxylase deficiency.

The patient has scleroderma and associated Raynaud’sphenomenon as well as other organ involvement, notablythe lungs (pulmonary fibrosis) and the oesophagus(dysmotility/lower oesophageal sphincter dysfunction).The best diagnosis is systemic sclerosis. 90% of patientswith systemic sclerosis have gut involvement (50% aresymptomatic). Oesophageal dysmotility is the mostfrequent visceral complication of systemic sclerosis. 70%have pulmonary involvement. Renal, cardiac andmusculoskeletal involvement is well recognized. Renalinvolvement is characterized by hypertension that may beacute and severe; cardiac involvement comprisespericarditis and its complications, restrictive cardio -myopathy and myocardial fibrosis. Myocardial fibrosis is

responsible for fatal ventricular arrhythmias causingsudden death in some cases of systemic sclerosis.

The sera of patients with scleroderma may display oneof the hallmark antibodies of scleroderma: anti -centromere, antitopoisomerase-1 (Scl-70), anti-RNApolymerase, or U3-RNP antibodies. ANCAs are presentin scleroderma syndromes.

Treatment is aimed at management of Raynaud’sphenomenon (keep hands warm, calcium channelblockers, alpha-blockers). Specific therapy is based onimmune modulating drugs. Several treatments have beenused with varying degrees of efficacy and includecyclophosphamide, ciclosporin, azathioprine, metho -trexate and interferons. Steroids should be used withextreme caution as they have been shown to precipitaterenal failure.

Answer 236

d. Systemic sclerosis.

Answer 237

All patients with angina should be prescribed an anti-thrombotic drug. The antithrombotic drug of choice isaspirin, which has been shown to significantly reducemortality following myocardial infarction (ISIS 2 trial).No other antithrombotic drug has been shown to besuperior to aspirin, although clopidogrel is equallyeffective. Beta-blockers reduce myocardial oxygendemand and are prognostically useful in the event ofmyocardial infarction (ISIS 1). In patients with coronary

artery disease, ACE inhibitors prevent future myocardialinfarction and mortality (EUROPA and HOPE studies).Cholesterol lowering with statins has been shown toreduce mortality in patients with established coronaryartery disease (4S and LIPID studies) even in patientswith mildly elevated or borderline levels of totalcholesterol (CARE and REGRESS studies). Nifedipine,on the other hand, has been shown to exacerbatemyocardial ischaemia (in the absence of a beta-blocker)and in one trial was associated with increased mortality.

b. Nifedipine.

Page 244: Rapid review of clinical medicine for mrcp part 2

A 23-year-old female student consulted her GP forconstant headaches for several weeks, which were worsein the mornings. She also found that her vision hadbecome blurred recently and she had developed diplopiatwo days ago. She had no past medical history of note.The patient was taking the oral contraceptive pill.

On examination the blood pressure measured125/80 mmHg. Her body mass index was 27. Herpupils were equal and reactive to light. Visual field testingrevealed enlarged blind spots bilaterally. She has diplopiaon looking to the right and impaired abduction of theright eye. Fundoscopy revealed bilateral papilloedema. Allother cranial nerves were normal, as was the examinationof the cerebellar and peripheral nervous system.

Investigations were as follows:

Clinical Cases 243

Questions 239, 240 and 241

ESR 8 mm/hCT scan of the brain without contrast NormalLumbar puncture and CSF analysis:

Opening pressure 29 cmH2OProtein 0.4 g/lGlucose 4.5 mmol/lMicroscopy Lymphocytes 4/mm3

Question 239

What is the next investigation of choice?a. Contrast enhanced CT scan.b. MRI scan of the brain with MR venography.c. CSF analysis.d. Skull X-ray.e. Serum retinoic acid level.

Question 240

What would your next management step be?a. Anticoagulation.b. Corticosteriods.c. Stop the oral contraceptive pill.d. Neurosurgical referral.e. Nimodipine.

Question 241

The patient is seen again 48 hours later withsignificant reduction in visual acuity. What is the bestmanagement?

a. Acetazolamide.b. Dexamethasone.c. Intravenous mannitol.d. Neurosurgical referral for optic nerve

fenestration.e. Lumbo-peritoneal shunt.

A 46-year-old obese woman was recently diagnosed ashaving diabetes mellitus. Investigations on presentationare shown.

Question 242

Sodium 132 mmol/lPotassium 4.6 mmol/lUrea 12 mmol/lCreatinine 190 �mol/lFasting glucose 13 mmol/lFasting cholesterol 8.2 mmol/lFasting triglycerides 8 mmol/lHbA1c 9.4%Urinalysis Protein ++

Glucose ++Microscopy normal

What is the best management for her diabetes?a. Gliclazide.b. Insulin.c. Stringent diabetic diet.d. Metformin.e. Insulin and metformin.

Page 245: Rapid review of clinical medicine for mrcp part 2

Endocrinal risk factors• Female sex • Reproductive age group • Menstrual irregularity • Obesity – recent weight gain• Adrenal insufficiency • Cushing’s disease • Hypoparathyroidism • Hypothyroidism • Excessive thyroxine replacement in children

Miscellaneous risk factors• SLE• Chronic renal failure

Pharmacological risk factors • Cimetidine, corticosteroids • Danazol • Isotretinoin (Accutane) • Levothyroxine• Lithium• Minocycline • Nalidixic acid• Nitrofurantoin• Tamoxifen• Tetracycline• Trimethoprim-sulfamethoxazole• All-trans-retinoic acid (ATRA) • Ciclosporine• Levonorgestrel implant• Pancreatin recombinant human growth

hormone• Vitamin A in infants

The presentation of benign intracranial hypertension maysimulate that of a cerebral tumour, hence it is also knownas ‘pseudotumour cerebri’. Patients present withsymptoms of raised intracranial pressure, which compriseheadache, transient visual obscurations and diplopia. Themost signifi cant physical finding in these patients isbilateral disc oedema. The diplopia is due to unilateral orbilateral sixth nerve palsies and resolves on lowering theintracranial pressure.

The diagnostic criteria include the following:• Symptoms and signs restricted to those of elevated

intracranial pressure.• Normal neuroimaging studies (excluding non-

specific findings of raised intracranial pressure).• Increased cerebrospinal fluid pressure with a normal

composition.

The pathophysiology of this disorder is unclear. It is widelypresumed that a relative resistance to the absorption ofcerebrospinal fluid across the arachnoid villi is present. Thedisease commonly occurs in individuals who are

overweight. There is a strong predilection for women, witha female-to-male ratio of up to 10:1 (Table A).

The morbidity of BIH is mainly related to the effectsof papilloedema. If left untreated, long-standing discoedema results in an irreversible optic neuropathy withaccompanying con striction of the visual field and loss ofcolour vision. In end-stage papilloedema, central visualacuity also is involved. Permanent visual loss occurs in10% of patients.

A patient with bilateral disc swelling should undergourgent neuroimaging to rule out an intracranial mass or adural sinus thrombosis (239a). Although CT scan of thebrain is adequate in most instances, MRI is most effectivein ruling out both a mass lesion and a potential duralsinus thrombosis.

In the setting of BIH, neuroimaging will be eithernormal or demonstrate small, slit-like ventricles, enlargedoptic nerve sheaths and occasionally an empty sella.

The treatment goal for these patients is to preserveoptic nerve function while managing their increasedintracranial pressure. Optic nerve function should bemonitored carefully with the assessment of visual acuity,colour vision, optic nerve head observation and perimetry.For an overview of management see Table B and (239b,overpage).

Despite careful follow-up care and maximum medicaltreatment, some patients develop deterioration of visualfunction. In this situation, surgical intervention should beconsidered. The two procedures that can be per formed areoptic nerve sheath fenestration and lumbo-peritonealshunting procedure. Treatment of this disorder by repeatedlumbar punctures is considered to be of historic interest.

244

Answers 239, 240 and 241

Answer 239b. MRI scan of the brain with MR venography.

Answer 240c. Stop the oral contraceptive pill.

Answer 241d. Neurosurgical referral for optic nerve fenestration.

Table A Causes of benign intracranial hypertension

Page 246: Rapid review of clinical medicine for mrcp part 2

Clinical Cases 245

Table B Medical management of benign intracranial hypertension• Weight control. Weight loss is a cornerstone in the management of patients.• Treatment of related underlying diseases • Cessation of exogenous agents related to increased intracranial pressure • Treatment of headache• Reduction of the intracranial pressure in order to protect optic nerve function.• Diuretics. Acetazolamide appears to be the most effective diuretic in lowering the intracranial pressure. • Corticosteroids are used as a supplement to acetazolamide to hasten recovery in patients who present with

severe papilloedema. Significant adverse effects preclude long-term use in such patients

Investigation of bilateral papilloedema

Patients with lateral sinus thrombosis may present witha pseudotumor cerebri-like syndrome. The scenariodescribes a patient whose sex, history of smoking and oralcontraceptive use put her at risk of developing cerebralvenous sinus thrombosis, which typically presents with ashort history of headache, vomiting and papilloedema. Ifvenous infarction supervenes, focal neurology will be seen.

CT may show the characteristic delta sign – a filling defectwithin the sinus after contrast administration. Magneticresonance venography is the investigation of choice as it isan excellent method of visualizing the dural venous sinusesand larger cerebral veins.

(Continued over page)

Bilateral papilloedema

Malignanthypertension? No

Urgent neuroimaging(preferably MRI)

NoSpace-occupying

lesion or a dural sinusthrombosis?

Lumbar puncture

Raised openingpressure? Yes

Raised ICP due tospecific offending agent?

NoBIH

223399aa

Page 247: Rapid review of clinical medicine for mrcp part 2

246 Clinical Medicine

An overview of the management of benign intracranial hypertension

Perimetry

No visual loss Visual loss

1. Weight loss2. Symptomatic

headache treatment

1. Weight loss2. Acetazolamide or

furosemide

RESOLUTION Progression

Progression orintractable headache

Steroids or nervesheath fenestration

Lumbar shunt

The three main goals in the management of diabetes are:1. Good glycaemic control, which has been shown to

reduce microvascular complications.2. Treatment of nephropathy, retinopathy and diabetic

foot disease.3. Aggressive treatment of risk factors for atherosclerosis

to reduce morbidity and mortality from macro-vascular disease (Table).

This particular question tests the candidate onmanagement of hyperglycaemia. Glycaemic controlinvolves dietary modification, weight loss, regular exerciseand pharmacological therapy.

The patient in question has evidence of nephropathy andhas a mixed hyperlipidaemia. There is good evidence thatmeticulous glycaemic control will retard progression ofrenal impairment and have a beneficial effect on lipidprofile. Weight loss and stringent diabetic diet shouldalways be implemented in overweight diabetics as increasedbody mass index is linked with insulin resistance. However,this patient has nephropathy, an unsatisfactory HbA1c and

hyperlipidaemia, which necessitate additional pharmaco -logical measures to improve glycaemic control.

Gliclazide, a sulphonylurea, promotes release of insulinand potentiates its effects in the periphery. Unfortunately itis associated with weight gain. Further more, sulphonylureasonly reduce blood sugar by 20%. Metformin is an ideal drugin obese diabetics because it promotes weight reduction andaids hyperlipidaemia; however, its use is contraindicated inpatients with a serum creatinine >150 mmol/l. The safestoption here is insulin therapy.

Answer 242

b. Insulin.

Goals in diabetes mellitus management toprevent cardiovascular/cerebrovascular mortality

• Achieve BMI 28• Cessation of smoking• Regular exercise*• HbA1c ≤ 7%• BP ≤ 130/80 mmHg• Total cholesterol ≤ 5 mmol/l• LDL cholesterol ≤ 2.2 mmol/l

*30 minutes of aerobic exercise five times per week.

223399bb

Page 248: Rapid review of clinical medicine for mrcp part 2

247

Question 243

A 60-year-old woman attended the Accident andEmergency Department with central chest pain radiatingto the jaw. Risk factors for coronary artery diseaseincluded hypercholesterolaemia and a family history ofischaemic heart disease. The 12-lead ECG revealed STsegment depression in leads V2–V6. Troponin T 12hours after admission was 0.4 ng/l (NR <0.1ng/l).

What is the diagnosis?a. Acute infero-lateral myocardial infarction.b. Musculoskeletal chest pain.c. Stable angina.d. Non-ST elevation myocardial infarction.e. Unstable angina.

Questions 244 and 245

A 64-year-old male presented with sudden onset ofbreathlessness. He had experienced a transient ischaemicattack three months previously and had a history ofintermittent claudication. The chest X-ray on admissionwas consistent with pulmonary oedema. He was takingnifedipine and atenolol for hypertension. He was treated

with intravenous diuretics and glyceryl trinitrate for 48hours with good effect. He was subsequently switched tooral furosemide 40 mg od. The atenolol was stopped andsubstituted with an ACE inhibitor.

Blood tests on admission are shown.

Sodium 137 mmol/lPotassium 3.6 mmol/lUrea 8.2 mmol/lCreatinine 137 �mol/l

Blood tests one week later:Sodium 135 mmol/lPotassium 4.6 mmol/lUrea 15 mmol/lCreatinine 220 �mol/l

Question 244

What is the most likely cause for the patient’sdeterioration?

a. Initiation of furosemide.b. Initiation of an ACE inhibitor.c. Hypertensive nephropathy.d. Relative hypotension.e. Left ventricular dysfunction.

Question 245

What is the next best investigation to ascertain thecause of the abnormal renal function?

a. Renal biopsy.b. Renal angiography.c. Renal isotope scan.d. Captopril renogram.e. Echocardiogram.

Page 249: Rapid review of clinical medicine for mrcp part 2

Sudden onset of pulmonary oedema in a patient withgeneralised arteriosclerosis raises the diagnosis of renalartery stenosis in which rapid onset of pulmonary oedema,sometimes termed ‘flash pulmonary oedema’ is recognized.

An abnormal baseline creatinine (>140 μmol/l) is usuallysuggestive of the diagnosis in patients with flash pulmonaryodema. In this case, however, the diagnosis becomes clearbecause initiation of an ACE inhibitor results indeterioration of renal function, a finding that is socharacteristic of renal artery stenosis. Remember ACEinhibitors cause a profound drop in GFR by causingafferent vasodilatation of the glomerular tuft.

The gold standard test for diagnosing renal artery stenosisis direct renal angiography, contrast CT scan of the renalarteries or magnetic resonance imaging of the renal arteries.

248

The patient presents with chest pain, non-ST elevationECG changes and a raised troponin level (specific formyocardial damage). The triad is consistent with thediagnosis of NSTEMI. Since the widespread use oftroponin in the investigation of chest pain, there hasbeen some confusion with respect to the diagnosis ofmyocardial infarction. As a result, acute coronarysyndromes have been divided into three categories:1. Classic ST elevation myocardial infarction.2. NSTEMI (where patient may or may not have classic

chest pain associated with ST segment depression or Twave inversion and raised troponin*).

3. Unstable angina (troponin-negative chest pain with orwithout non-ST elevation ECG changes).

*Troponin T >0.05 mg/l; troponin I >0.3 mg/l.STEMI is managed with urgent thrombolysis or primaryangioplasty if the patient presents within 12 hours andhad no contraindications to thrombolysis. Patients whohave failed thrombolysis, or have on-going myocardialischaemia, should have in--patient coronary angiography.Patients with a positive exercise stress test followingSTEMI should also have early coronary angiography.

The immediate prognosis in STEMI is worse than thatin NSTEMI; nevertheless over the following six monthspatients with NSTEMI have a 16% mortality. Thereforepatients presenting with NSTEMI should undergo earlycoronary angiography with a view to revascularization (co-morbidities, quality of life and cognitive state allowing).

The differentiation between NSTEMI and unstableangina is retrospective and based on a troponin resultperformed 12 hours after admission. The management isessentially the same, comprising of aspirin, clopidogrel,heparin, glycoprotein IIb/IIIa receptor inhibitor, GTNand beta-blocker therapy. Aggressive management of riskfactors for coronary artery disease is important. Inunstable angina early angiography is recommended inpatients with chest pain associated with ST segmentdepression or T wave inversion.

A raised troponin is specific for myocyte necrosis butdoes not always indicate myocardial damage secondary tocoronary artery occlusion. Myopericarditis, tachy -carrhythmias, DC cardioversion, the cardiomyopathies,and large pulmonary embolus are all recognized causes ofa raised troponin level. Septicaemia may also be associatedwith raised troponin levels in up to 80% of cases. Chronicrenal impairment is associated with raised troponin levelsdue to accumulation of cardiac troponin fragments in theblood, which are identified as raised troponins by currentassays (Table).

Troponin I assays are less likely to identify thesetroponin fragments than troponin T assays; therefore,troponin I measurements may be more accurate thantroponin T measurements in patients with chronic renalfailure. However, it is well established that patients withchronic renal failure and raised troponin measurementshave a higher risk of morbidity/mortality than patientswithout raised troponin levels.

A raised cardiac troponin should be interpreted in thecontext of the presentation and the ECG. For example, apatient with meningococcal septicaemia and a raisedtroponin is not necessarily an indication for anti-thrombotic therapy and subsequent coronary angiography.

Answer 243

d. Non-ST elevation myocardial infarction.

Causes of raised cardiac troponins

• Myocardial infarction• Myopericarditis• Dilated and hypertrophic cardiomyopathy• DC cardioversion• Tachyarrhythmias• Pulmonary embolus• Exacerbation of chronic obstructive airways disease• Chemotherapy• Chronic renal impairment• Septicaemia• Subarachnoid haemorrhage• Marathon running

Answer 244b. Initiation of an ACE inhibitor.

Answer 245b. Renal angiography.

Answers 244 and 245

Page 250: Rapid review of clinical medicine for mrcp part 2

Clinical Cases 249

Question 248

A 19-year-old woman presented with lethargy andconstipation. Her appetite had increased. Onexamination the heart rate was 100 beats/min and therewas a soft systolic murmur at the left lower sternal edge.

She was not taking any medication. Her mother andsister had hypothyroidism.

Investigations are shown.

Haemoglobin 11.4 g/dlWCC 4 � 109/lPlatelets 179 � 109/lMCV 80 flSodium 138 mmol/lPotassium 3.6 mmol/lUrea 6 mmol/lTotal T4 190 nmol/l (NR 60–160 nmol/l)Total T3 3.5 nmol/l (NR 1.2–3.1 nmol/l)Free T4 20 pmol/l (NR 20–30 pmol/l)Free T3 NormalTSH 3.5 mu/l (NR 0.5–5.0 mu/l)

What is the diagnosis?a. Choriocarcinoma.b. Factitious hyperthyroidism.c. Pregnancy.d. Sick euthyroid syndrome.e. Amiodarone therapy.

The following are cardiac catheter data on a 40-year-oldmale with heart failure.

Question 246

Chamber Pressure Oxygen saturation(mmHg) (%)

Superior vena cava 60Inferior vena cava 61Right atrium 15 65Right ventricle 105/40 64Pulmonary artery 102/50 68Left ventricle 98/36 95Ascending aorta 98/60 95Descending aorta 98/60 80

What two conclusions can be drawn fromthese data?

a. Atrial septal defect.b. Ventricular septal defect.c. Primary pulmonary hypertension.d. Coarctation of the aorta.e. Patent ductus arteriosus.f. Chronic obstructive airways disease.g. Eisenmenger’s syndrome.h. Pulmonary stenosis.i. Fallot’s tetralogy.j. Patent foramen ovale.

A 31-year-old pregnant woman in her third trimestercomplained of pain and tingling affecting her right hand,forearm and shoulder. The symptoms were worse atnight but were relieved by shaking the hand. Her handfelt weaker than usual. On examination there was nowasting of the hand muscles. Abduction of the rightthumb was difficult. There was impaired touch sensationaffecting the palmar aspects of the first three digits in theright hand. The biceps, triceps and supinator reflexeswere intact.

Question 247

What is the diagnosis?a. Carpal tunnel syndrome.b. C6,7 radiculopathy.c. Ulnar nerve lesion.d. Thoracic inlet syndrome.e. Radial nerve palsy.

Page 251: Rapid review of clinical medicine for mrcp part 2

Lethargy, weight gain, constipation and increasedappetite are consistent with pregnancy in a youngwoman. Raised total T3 and T4 but normal free T3 andT4 suggest high concentrations of thyroid bindingglobulin, which are present in pregnancy. It is importantto note that high concentrations of HCG in earlypregnancy may result in subclinical hyperthyroidism sinceHCG has a common beta subunit to TSH.

Choriocarcinoma or a hydatidiform mole producehigh concentrations of HCG that induce profusevomiting. Both choriocarcinoma and hydatidiform molecan result in severe biochemical hyperthyroidism. Thediagnosis should always be considered in a pregnantwoman with hyperemesis, very high T3/T4 and lowTSH. In this case the TSH is normal, therefore thepatient is clinically euthyroid. Factitious thyrotoxicosiswould be associated with a high free T4. The biochemicalpicture is against sick euthyroid syndrome.

There is evidence of a right-to-left shunt after theascending aorta. The most likely diagnosis is a patentductus arteriosus allowing communication between theaorta (just distal to the left subclavian artery usually) and

the pulmonary artery. The left-to-right shunt leads to ahyperkinetic pulmonary circulation and an elevated rightventricular pressure. If the defect is large the rightventricular pressure may subsequently become highenough to exceed that of the left ventricle resulting inshunt reversal (right-to-left shunt). This phenomenon ofshunt reversal with cardiac defects is sometimes termedEisenmenger’s syndrome.

250

Answer 246

e. Patent ductus arteriosus.g. Eisenmenger’s syndrome.

Answer 248

c. Pregnancy.

The patient has the classic symptoms of carpal tunnelsyndrome, which is caused by compression of themedian nerve beneath the transverse carpal ligament. Itis usually bilateral and more common in females thanmales (Table).

Pregnancy is a recognized cause, presumably owing tofluid retention. Symptoms include pain in the first threedigits of the hand that may also affect the arm andshoulder. There is tingling and pain in the palmar aspectsof the first three digits that may be relieved by shakingthe hand. The symptoms are worse at night but may bepresent in the day when carrying out actions that involverepetitive flexion of the hand.

Clinical examination reveals reduced sensation affectingthe first three digits. There is weakness affecting the firstlumbrical muscle, opponens pollicis and abduction pollicisbrevis. Wasting of abductor pollicis brevis causes wastingof the thenar eminence. Tinel’s sign (precipitation of

symptoms by flexing the wrist) and Phelan’s signs(provoking symptoms by flexion of the wrist) areunreliable. Nerve conduction studies are essential. Slowsensory velocity and reduced amplitude of the mediannerve action potential are diagnostic. Treatment involvesthat of the underlying cause in some circumstances.Carpal tunnel steroid injection may prevent symptoms inpatients with arthritis, but surgical decompression is thetreatment of choice in most patients with symptoms.

Answer 247

a. Carpal tunnel syndrome.

Causes of carpal tunnel syndrome

• Pregnancy• Oral contraceptive pill• Acromegaly• Amyloidosis• Hypothyroidism• Rheumatoid arthritis• Tight plaster casts• Repetitive wrist movements

Page 252: Rapid review of clinical medicine for mrcp part 2

Clinical Cases 251

A 30-year-old male presented with severe pain in his rightflank radiating to the groin. He was diagnosed withsevere Crohn’s disease five years ago and had a terminalileal resection three years ago. He also had a history ofhypertension that was well controlled on a low-salt dietand amlodipine 5 mg daily.

An intravenous urogram was performed whichconfirmed findings consistent with a right ureteric stone.The serum calcium, phosphate and urate levels werenormal.

Questions 249 and 250

Question 249

What is the most probable consistency of the uretericstone?

a. Calcium phosphate.b. Calcium oxalate.c. Uric acid.d. Magnesium ammonium sulphate.e. Cystine.

Question 250

What initial treatment is required to prevent furtherepisodes?

a. Thiazide diuretic.b. Chronic antibiotic therapy.c. High fluid intake.d. Calcium carbonate supplements.e. Potassium citrate therapy.

A 63-year-old patient who had a cadaveric renaltransplant six months ago was admitted to the dialysisunit with nausea, vomiting and deteriorating renalfunction. He also complained of headache.

On examination he was tremulous. The heart rate was90 beats/min and regular. The blood pressure measured200/110 mmHg. The temperature was 37°C (98.6°F).The JVP was not raised. Both heart sounds were normal.Inspection of the fundi revealed grade II hypertensiveretinopathy. There were no focal neurological signs. Therenal graft was non-tender.

Investigations are shown.

The patient was commenced on dialysis but had agrand mal seizure while he was being dialysed.

Question 251

Sodium 131 mmol/lPotassium 5.6 mmol/lUrea 20 mmol/lCreatinine 400 �mol/lBicarbonate 15 mmol/lChloride 111 mmol/l

What is the diagnosis?a. Hypertensive encephalopathy.b. Ciclosporin toxicity.c. Subarachnoid haemorrhage.d. Disseminated CMV infection.e. Meningitis.

Page 253: Rapid review of clinical medicine for mrcp part 2

252

Patients with Crohn’s disease are prone tonephrolithiasis. The renal stones in Crohn’s disease aremost commonly composed of calcium oxalate, primarilyowing to enteropathic hyperoxaluria. Terminal ilealdisease is associated with reduced free fatty acidabsorption. The free fatty acids bind intraluminal freecalcium, allowing increased oxalate reabsorption.Furthermore, the presence of bile salts in the colonincreases colonic permeability to small molecules such as

oxalic acid. The condition is exacerbated by low fluidintake and metabolic acidosis.

The management of renal stones due to enteropathichyperoxaluria is as follows (Table):• Increased fluid intake to enable urine volumes of 2 l or

more per day.• Reduced fat and oxalate in diet• Calcium carbonate supplements (increase intraluminal

calcium and enable excretion of oxalate by the gut). • Cholestyramine is very effective at preventing

enteropathic hyperoxaluria, but its use is limited by itsside-effect profile.

Dietary calcium should not be reduced. Thiazidediuretics are only useful in patients who have renal stonesdue to hypercalcuria.

Answers 249 and 250

Answer 249b. Calcium oxalate.

Answer 250c. High fluid intake.

Management of renal stones

General measures1. Increase fluid intake to enable a diuresis of at least 2 litres per day.2. Low-salt, low-oxalate, low-protein diet.3. Consider drug therapy if there is evidence of active stone disease or stone enlargement despite 3–6 months

of dietary modification. The specific drug therapies depend on the conditions predisposing to the stonesand the urine calcium and urate concentrations.

Specific measures for the five main types of renal stonesComposition of stone Predisposing condition Specific management1. Calcium phosphate Hypercalcuria Thiazide diuretics

Distal (type 1) RTA Potassium citrate/thiazide diuretics (if hypercalcuria)2. Calcium oxalate Hypercalcuric states Thiazide diuretic

Enteropathic hyperoxaluria Calcium carbonateHyperuricosuria Potassium citrate

3. Uric acid stones Gout/hyperuricosuria Allopurinol or potassium citrate4. Struvite stones Recurrent UTI* Surgical removal5. Cystine stones Cystinuria Penicillamine

CaptoprilTiopronin

*Due to Proteus or Klebsiella species

Deteriorating renal function (usually due to interstitialfibrosis with chronic therapy or arteriolar vasoconstrictionin the early treatment), headache, rising BP and anepileptic seizure are all in keeping with ciclosporin

toxicity. Acute ciclosporin toxicity is reversible bytransiently stopping the drug and re-introducing it atlower doses. Treatment of chronic nephrotoxicity is moredifficult. Options include reduction of the dose ofciclosporin or switching to an alternative immuno -suppression regime.

Answer 251

b. Ciclosporin toxicity.

Page 254: Rapid review of clinical medicine for mrcp part 2

Clinical Cases 253

This 40-year-old male had a long history of recurrentlower respiratory chest infection that required aggressivetreatment with intravenous antibiotics. He had a recentappendicectomy that proved to be technically difficult.He was a non-smoker and married with three children. Achest X-ray performed after surgery is shown.

Question 252

What is the diagnosis?a. Cystic fibrosis.b. Bronchiectasis.c. Kartagener’s syndrome.d. Immotile ciliary syndrome.e. Right lower lobe collapse.

A 54-year-old non-insulin-dependent diabetic is seen inthe diabetic control clinic. His blood pressure is130/80 mmHg. HbA1c measures 9.6%. The creatinine is160 �mol/l. Urinalysis reveals protein ++ and glucose ++.

Question 254

What is the best treatment for preventingdeterioration of renal function?

a. Insulin.b. Metformin.c. Irbesartan.d. Atenolol.e. Gliclazide.

A 32-year-old woman, who had recently seen her GP fora chest infection, visited her GP again complaining ofbeing clumsy and dropping things. Her husband had alsonoticed that her speech was slurred in the evening whenhe saw her after work. There was no previous medicalhistory or family history of note. She was on nomedication. She appeared well and there was nothing tofind on general examination. Her pupillary reflexes, visualfields and fundoscopy were normal. She had diplopia onlooking left and upwards to the right. All cranial nerveswere normal. Tone, power and reflexes were all normal aswere coordination and sensation.

Question 253

What is the most likely diagnosis?a. Guillain–Barré syndrome.b. Myasthenia gravis.c. Motor neurone disease.d. Miller–Fisher syndrome.e. Multiple sclerosis.

L 225522

Page 255: Rapid review of clinical medicine for mrcp part 2

254

The chest X-ray reveals thickened and dilated bibasalbronchioles, consistent with bronchiectasis, which aremost prominent in the right lower lobe. Lookingcarefully at the left/right annotation on the chest X-rayone notices that the X-ray has been switched the wrongway round (left side is on the right side). Therefore, thepatient also has dextrocardia. The difficulty with theappendicectomy suggests that the appendix must havebeen in the left iliac fossa. The combination of situsinversus and bronchiectasis is consistent with a diagnosisof Kartagener’s syndrome.

Cystic fibrosis is not associated with dextrocardia andthere is no mention of gastrointestinal complications –that would normally relate to cystic fibrosis – in thispatient. Furthermore, males with cystic fibrosis areinfertile as a rule.

Immotile ciliary syndrome is a genetic disorder that isinherited as an autosomal recessive trait. It ischaracterized by abnormal ciliary motility in the upperand lower respiratory tracts. Common manifestationsinclude sinusitis, nasal polyps and bronchiectasis.Headaches and hydrocephalus are recognized. Whiledextrocardia is present in 50% of cases, patients with thisdisorder are invariably infertile, unlike the patient inquestion. The diagnosis is made by demonstratingabnormal ciliary function. Treatment of lower respiratorytract infections requires aggressive antibiotic treatment.

Answer 252

c. Kartagener’s syndrome.

The patient has evidence of overt diabetic nephropathy,i.e. impaired renal function and frank proteinuria. Urinedipsticks only become positive for protein once urineprotein concentration is between 0.3 and 0.5 g/day.

The UKPDS study showed that aggressive glycaemiccontrol prevented microvascular complications. However,the vast majority of patients studied had either no

microvascular complications or mild microvascularcomplications. There is currently a lack of good evidenceregarding the effect of tight glycaemic control on theprogression of diabetic nephropathy in patients withadvanced microvascular complications. On the otherhand trials using ACE inhibitors and more recentlyangiotensin receptor blocking agents have shownretardation of diabetic nephropathy in patients with type1 and type 2 diabetes mellitus respectively who haveestablished renal disease. (See Question 154.)

Answer 254

c. Irbesartan.

Answer 253

The patient has evidence of fatiguability (slurred speechin the evenings and extraocular muscle weakness) whichhelps make the diagnosis. There are no neurological signson examining the limbs, therefore all the other optionsprovided are excluded. Weakness is exacerbated bypregnancy, hypokalaemia, infection, over-treatment ofmyasthenia gravis, a change in climate, emotion, exerciseand certain drugs, for example aminoglycosides.

The incidence of myasthenia gravis has two age peaks:the second and third decades (F > M) and the sixth andseventh decades (M > F). Acetylcholine receptorantibodies are detectable in 90% of patients. A thymoma

is associated in 10% of cases and thymus abnormalities in75%. There is a high incidence of myasthenia gravis inpatients with thyrotoxicosis, RA, SLE and DM. Thedisease may never progress beyond an ophthalmoplegia.Periods of remission up to three years occur.Thymectomy improves the outlook unless a thymoma ispresent and achieves remission in up to 30% of patients.The survival rate in the presence of a thymoma is 30%.

Miller–Fisher syndrome, which is a variant ofGuillain–Barré syndrome, is characterized by ophthal -moplegia, sensory ataxia and a descending (predomi -nantly motor) neuropathy.

(See Answers 65, 200.)

b. Myasthenia gravis.

Page 256: Rapid review of clinical medicine for mrcp part 2

Clinical Cases 255

Questions 255 and 256

A 60-year-old male presented with sudden onset ofreduced visual acuity in the right eye. Six months prior tothe presentation the patient had experienced a three-dayepisode of ataxia that had resolved spontaneously. Thepatient had a history of diabetes mellitus that was wellcontrolled on insulin. There was no family history of note.The patient consumed 12 units of alcohol per week.

On examination he was in sinus rhythm. The bloodpressure measured 130/80 mmHg. Neurologicalexamination was normal with the exception of markedlyreduced visual acuity in the right eye. Fundoscopy wasnormal.

Question 255

What is the most probable diagnosis?a. Central retinal artery occlusion.b. Leber’s hereditary optic neuropathy.c. Multiple sclerosis.d. B12 deficiency.e. Ischaemic optic neuropathy.

Question 256

What is the treatment of choice?a. B12 injections.b. Aspirin.c. High-dose folic acid.d. IV methylprednisolone.e. No treatment required.

A 14-year-old girl was investigated for abdominal pain.Abdominal examination was normal.

Investigations are shown.

(See Question 213.)

Question 257

Hb 10.8 g/dlWCC 8 � 109/l

(normal differential)Platelets 200 � 109/lMCV 75 flSodium 136 mmol/lPotassium 3.8 mmol/lUrea 6 mmol/lCreatinine 100 �mol/lAmylase 100 iu/l (NR <220 iu/l)Serum Iron 33 �mol/l (NR 13–32 μmol/l)TIBC 49 �mol/l (NR 42–80 μmol/l)Urinary 640 nmol/24h

coproporphyrins (NR 50–350 nmol/24h)

What is the diagnosis?a. Acute intermittent porphyria.b. Porphyria cutanea tarda.c. Haemochromatosis.d. Lead poisoning.e. Hereditary coproporphyria.

A 24-year-old woman presented with severe menorrhagia.There were several bruises on her arms and legs.

Investigations are shown.

Question 258

Hb 6.2 g/dl WCC 29 � 109/lPlatelets 32 � 109/lMarrow aspirate Metamyelocytes

Promyelocytes ++++Myelomonocytes and

myeloblastsD-dimer Massively elevated

What is the most likely chromosomal abnormalityaccounting for this disorder?

a. Monosomy 17.b. Translocation (9,22).c. Translocation (15,17).d. Translocation (4,11).e. Trisomy 12.

Page 257: Rapid review of clinical medicine for mrcp part 2

256

The patient presents with sudden onset of reduced visualacuity in one eye and fundoscopy suggests the possibilityof optic neuritis. The preceding history of transient ataxiasix months previously is highly suggestive of an under -lying diagnosis of multiple sclerosis.

The diagnosis of multiple sclerosis is usually clinicaland should be considered in any patient who hasneurological episodes that are scattered in time (two ormore separate episodes) and space (two or more separatelocations) (Table). There is no single test that canconfirm the diagnosis, although the use of MRI scanningin conjunction with clinical findings has improveddiagnostic accuracy. CSF analysis for oligoclonal bandscontinues to remain useful.

Optic neuritis and all other acute episodes in multiplesclerosis that are serious enough to cause distress orincrease limitation of activities should be treated with acourse of high-dose steroids as soon as possible after theonset of symptoms. The therapy of choice is intravenousor oral methyprednisolone for 3–5 days.

Answers 255 and 256

Answer 255c. Multiple sclerosis.

Answer 256d. IV methylprednisolone.

Diagnostic criteria for suspected multiplesclerosis (McDonald criteria)

• If the patient has two or more clinical attackswith two or more objective lesions then MS isdiagnosed clinically although many neurologistsstill request an MRI scan of the brain to helpconfirm MS or exclude other diagnoses

• If the patient has two or more attacks with oneobjective lesion then further information isrequired from MR brain or CSF analysis. Thereare two possible approaches:(1) The following findings on brain MRI

showing dissemination in space confirm the diagnosis:

One gadolinium-enhancing lesion orNine T2 hyperdense lesions if no gadolinium-enhancing lesion plus

(a) one or more infratentorial lesions plus(b) one or more juxtacortical lesions plus(c) three or more periventricular lesions

(2) Alternatively positive CSF for oligoclonal bands and brain MRI showing two or more lesions consistent with MS is sufficient to make the diagnosis

NB: One cord lesion can substitute one brain lesion.If a patient with two or more attacks but only oneobjective lesion goes on to have a further attackinvolving a different site while awaiting further investi -gation, the diagnosis of MS becomes more probable.

Abdominal pain and raised serum coproporphyrin levelsare indicative of either lead poisoning or hereditarycoproporphyria. The most probable answer is leadpoisoning since lead poisoning is much more commonthan hereditary coproporphyria and there is a history ofpatient exposure to lead through his occupation.

Hereditary coproporphyria is inherited as an auto somaldominant trait and is due to a partial deficiency in the

enzyme coproporphyrinogen oxidase. The condition is rareand has a prevalence of 2 per million. The symptoms areidentical to those of lead poisoning although photo -sensitivity may also be an additional feature. In patients withhereditary coproporphyria, faecal and urinary copropor -phyrin levels are hugely elevated. In contrast, copropor -phyrin levels are only modestly elevated in lead poisoning.

While acute intermittent porphyria also presents withabdominal pain, faecal and urinary coproporphyrin levelsin AIP are low. (See Question 213.)

Answer 257

d. Lead poisoning.

The blood film and coagulation profiles are consistent withAML type 3 (promyelocytic leukaemia). AML 3 isassociated with disseminated intravascular coagulation. Thegenetics of AML 3 are understood. Reciprocal translocationof the long arms of chromosomes 15 and 17 is the basicgenetic defect. The break point on chromosome 17 hasbeen mapped to the retinoic acid receptor alpha (RAR-

alpha). The translocation creates a fusion gene known asPML-RAR-alpha, which impairs apoptosis inpromyelocytes. This particular form of AML is amenable totreatment with all-trans retinoic acid. Karyotyping isimportant in AML type 3 as another rarer variant t (11-17)does not respond to all-trans retinoic acid.

More urgently, the patient requires fresh-frozen plasmaand a platelet transfusion to treat the DIC. In the absenceof active bleeding platelet transfusion is considered only ifthe platelet concentration falls below 20 � 109/l.

Answer 258

c. Translocation (15,17).

Page 258: Rapid review of clinical medicine for mrcp part 2

Clinical Cases 257

Question 259

These ECG traces are from 40-year-old men. One isbreathless on exertion (259a) and the other has a systolicmurmur (259b).

Question 260

A 17-year-old male was referred to the local paediatricianbecause his mother was concerned that he wasconsiderably shorter than his peers. He was well inhimself. He had never required to shave, and had justrecently noticed some pubic hair in his genital area. Onexamination, he measured 1.58 m. He had slight bilateralgynaecomastia. His voice had not broken. There was no

facial hair. There was scanty pubic hair in the genitalregion and his testes were small. His father and motherwere 1.83 m and 1.68 m tall, respectively.

Results of dynamic pituitary function testing with0.2 units/kg insulin, 100 �g of LHRH and 200 mg ofTRH were as follows:

Time Glucose GH Cortisol TSH LH FSH Testosterone(min) (mmol/l) mu/l (nmol/l) (mu/l) (iu/l) (iu/l) (nmol/l)

0 4.0 1.4 400 2.0 29 16 34

20 2.8 12.0 860 5.2 40 24 60

30 1.6 28 1270 - - - -

40 1.5 32 1450 - - - -

60 2.2 20 1120 8 34 20 60

What is the most probable diagnosis?a. Prader–Willi syndrome.b. Kleinfelter’s syndrome.

c. Recent onset of puberty.d. Kallman’s syndrome.e. Testicular feminization syndrome.

Question 259a

1. What information does the ECG give you withrespect to cardiac morphology?

2. Give four acquired causes and three inherited causesfor the ECG abnormalities.

Question 259b

1. List two abnormalities.2. Suggest two possible diagnoses.

225599aa

225599bb

Page 259: Rapid review of clinical medicine for mrcp part 2

258

Answer 259

Information regarding atrial enlargement is gained fromleads II and V1 (259c). Right atrial enlargement ischaracterized by a tall P-wave in both II and V1 thatexceeds 0.25 mV (or two-and-a-half small squares).Voltage criteria for left atrial enlargement are a little morecomplicated. In lead II, the P-wave has a bifid appearanceand the duration of the P-wave is 120 ms (three smallsquares). In V1, the P-wave is biphasic with a terminal(negative) portion of the P-wave exceeding or equal to40 ms in duration (one small square) and more than orequal to –0.1 mV (one small square) in depth. Thepresence of a biphasic P-wave in V1 which does not satisfythese criteria cannot be diagnosed as left atrialenlargement on the ECG.

Information regarding voltage criteria for ventricularhypertrophy is gained from leads V1 and V5, particularlyif the Sokolow’s criteria (most commonly used) are to be

applied. Right ventricular enlargement is suggested bythe sum of the R-wave in V1 and the S-wave in V5 or V6exceeding 1.25 mV (12.5 small squares). In addition,there may be ST-segment depression and T-waveinversion in leads V1–V3 and right axis deviation. Leftventricular hypertrophy is characterized by the sum of theS-wave in V1 and the R-wave in V5 or V6 exceeding40 mm. The additional presence of ST-segmentdepression and/or T-wave inversion in V5 and V6 ismore suggestive of the diagnosis.

Answer 260

The findings are consistent with recent onset of puberty.The basal gonadotrophin and testosterone levels aregenerally high during puberty. The patient has scantypubic hair and gynaecomastia, both markers of puberty.Prader–Willi and Kleinfelter’s syndrome are recognizedcauses of primary hypogonadism; however, the hightestosterone level is against this. Testicular feminizationsyndrome would be associated with a similar testosteroneand gonadotrophin level; however, patients affected withthe testicular feminization syndrome are phenotypicallyfemale (see Answer 397). Kallman’s syndrome is rare andoften inherited as an X-linked recessive trait. It ischaracterized by deficient gonadotrophin production bythe pituitary, anosmia, cleft palate and colour blindness.These patients may also have congenital renal

and cerebral defects. Other causes of primaryhypogonadism in males are given (Table).

Causes of primary hypogonadism in the male

• Testosterone biosynthetic enzyme deficiencies• Kleinfelter’s syndrome• Prader–Willi syndrome• Lawrence–Moon–Biedel syndrome• Noonan’s syndrome• Rubella• Mumps orchitis• Castration• Cystic fibrosis• Sickle cell anaemia• Cirrhosis• Chemotherapy• Alcohol

Answer 259a1. The ECG demonstrates voltage criteria for right

atrial enlargement and right ventricular hypertrophy.2. Acquired causes include:

i. Fibrotic lung disease.ii. Bronchiectasis.iii. Thromboembolic pulmonary disease.iv. Chronic obstructive airways disease.Inherited causes:i. Cystic fibrosis.ii. α-1 antitrypsin deficiency.iii. Ostium secundum atrial septal defect (may beinherited as an autosomal dominant trait).

c. Recent onset of puberty.

Answer 259b1. i. Voltage criteria for left atrial enlargement.

ii. Left ventricular hypertrophy (see explanation).2. i. Aortic stenosis.

ii. Hypertrophic cardiomyopathy.

Lead I

Lead VI

0.25 mV

0.25 mV120 ms

Right atrial enlargement Left atrial enlargement

0.1mV

40 ms↔

225599cc

Page 260: Rapid review of clinical medicine for mrcp part 2

Clinical Cases 259

Question 261

A 47-year-old female visited her GP to request hormonereplacement therapy. She had been amenorrhoeic for fourmonths, and had complained of sweating excessively andfatigue. She had also noticed an increase in the size of herhands such that her wedding ring was very tight, her shoesize had increased recently and her head appeared larger.Three years ago she had an episode of haemoptysis,following which she had a chest X-ray (261) thatrevealed a small opacity in the right lung. She wassubsequently lost to follow-up.

On examination, she had large, spade-shaped handsand broad feet. There was a palpable smooth thyroidgoitre. The heart rate was 96 beats/min and the bloodpressure was 160/90 mmHg. On examination of therespiratory system there was dullness and reduced airentry at the right lung base.

The patient was referred to an endocrinologist forfurther investigations.

Initial investigations are shown.

Hb 13 g/dlWCC 9 � 10 9/lPlatelets 210 � 10 9/lMCV 89 flSodium 135 mmol/lPotassium 4 mmol/lUrea 6 mmol/lCreatinine 98 μmol/l

Calcium 2.5 mmol/lPhosphate 1 mmol/lAlbumin 40 g/lAlkaline phosphatase 100 iu/lBilirubin 10 μmol/lAST 20 iu/lALT 25 iu/lChest X-ray (261)

CT scan of thorax Bulky mediastinum due to lymphadenopathy (mediastinal views)MRI of pituitary NormalTSH 3 mu/l (normal range 0.5–5.0 mu/l)Thyroxine 110 nmol/l FSH 60 iu/lLH 30 iu/l

Oral glucose tolerance test with 75 g glucose after an overnight fast:

Time (min) 0 30 60 120Blood glucose (mmol/l) 8 15 20 10.5GH (mu/l) 60 64 94 58

Insulin tolerance test: 0.3 units/kg:

Time (min) 0 30 45 60 90 120Blood glucose (mmol/l) 8 4 1.8 3 8 11Cortisol (nmol/l) 180 260 700 620 500 400

Serum calcitonin Normal Serum PTH NormalSerum glucagon Normal Serum noradrenaline NormalSerum VIP Normal Urinary VMA Normal

1. List two inferences you can make from the oral glucose tolerance test.2. Comment on the insulin tolerance test.3. Comment on the abnormal gonadotrophin levels, and give the most likely explanation.4. Comment on the chest X-ray.5. What is the connection between the chest X-ray and the endocrine abnormality?

226611

Page 261: Rapid review of clinical medicine for mrcp part 2

260

Answer 261

Question 262

A 29-year-old male living in a hostel presented with athree-week history of reduced visual acuity and a six-weekhistory of watery diarrhoea. There was no history ofheadaches or abdominal pain. On further questioning, hementioned he had a sore mouth and throat, and found itdifficult to swallow. He had not been on any medicationsin the past six months.

On examination, he was thin. Inspection of his tongueand oral cavity are shown (262a, b). Examination of hisfundi is shown (262c). Examination of the abdomen andcentral nervous system was normal.

Investigations are shown.

Hb 10 g/dlWCC 4 � 109/lPlatelets 170 � 109/lSodium 134 mmol/Potassium 4.1 mmol/lUrea 4 mmol/lCreatinine 67 μmol/lGlucose 4 mmol/lChest X-ray ClearUpper gastrointestinal Oesophagitis endoscopyOesophageal washings (262d)Stool microscopy (262e)Rectal biopsy (262f)

1. The patient has diabetes mellitus and acromegaly.The fasting blood glucose is over 6.7 mmol/l andthe 2-hour blood glucose level is 10.5 mmol/l.Impaired glucose tolerance is diagnosed whenfasting blood glucose is above 6.7 mmol/l and the2-hour blood glucose is under 10 mmol/l. Theresting growth hormone level of above 4 mu/l ishighly suggestive of acromegaly. In normal people,growth hormone secretion is suppressed to below2 mu/l with hypergly caemia; however, inacromegaly there is failure of growth hormonesecretion suppression and there may be aparadoxical rise in growth hormone levels withhyperglycaemia.

2. There is adequate hypoglycaemia (blood glucosefalls to under 2.2 mmol/l after 45 min) and there isan appropriate rise in serum cortisol (>550 nmol/l),suggesting adequate pituitary ACTH reserve.

3. The patient is aged 47 and has amenorrhoea. Thegrossly elevated gonadotrophin reflects menopause.The low circulating oestrogen level (lack of negativefeedback of oestrogen on the pituitary) is thestimulus for hypersecretion of the gonadotrophins.

4. The chest X-ray demonstrates a large, well-definedcircular mass situated in the lower lobe of the rightlung which may be consistent with a carcinoma,adenoma or a large secondary metastasis.

5. The endocrine tests are suggestive of acromegaly. Themost common cause of acromegaly is auto nomoussecretion of growth hormone by an acidophiladenoma in the pituitary. In this case, the pituitaryappears normal on the MRI scan and there is noother pituitary endocrine abnormality. It is highlypossible that the cause of acromegaly is due to ectopicgrowth hormone or GHRH secretion. Ectopicgrowth hormone or GHRH secretion is a very rarebut recognized charac teristic of certain tumours,particularly those affecting the lung, pancreas orovary. Small-cell lung carcinoma or bronchialcarcinoid, and ovarian teratomas are capable ofgrowth hormone or GHRH secretion. Morecommon examples of ectopic hormone productioninclude ACTH (small-cell carcinoma lung), ADH(small-cell carcinoma lung), PTH-related peptide(hypernephroma, squamous-cell lung carcinoma),erythropoietin (hepatocellular carcinoma, hyper -nephroma) and 5-HIAA (carcinoid tumours).

1. What is the cause of his reduced visual acuity?2. Why does he have dysphagia?3. What is the cause of the abnormality on his tongue?4. Give two possible explanations for his diarrhoea.5. What two drugs should the patient take for his

diarrhoea and reduced visual acuity?

Page 262: Rapid review of clinical medicine for mrcp part 2

Clinical Cases 261

226622aa 226622bb

226622cc

226622dd 226622ee

226622ff

Page 263: Rapid review of clinical medicine for mrcp part 2

262

Answer 262

The patient has evidence of several AIDS-definingconditions. The fundal appearance is consistent withCMV retinitis, which is the most common cause of visualimpairment in these patients. The diagnosis is clinical.Early retinal changes include narrowing of the lumen ofthe retinal vessels. This is followed by vascular occlusionand perivascular haemorrhages (red areas), resulting inretinal ischaemia (white areas). Without therapy, thecondition progresses to bilateral blindness. Treatment iswith intravenous ganciclovir, followed by oralmaintenance therapy. CMV may also cause encephalitis,colitis and cholangitis, all of which are best treated by thesame drug. In the gastrointestinal tract CMV isdiagnosed histologically by the demonstration of roundintranuclear inclusion bodies within epithelial cells of theintestinal mucosa, as shown in the rectal biopsy (262f).

Oral candidiasis is common in patients with HIV andis treated with nystatin or fluconazole. Gastric washingsin this patient reveal Candida hyphae. OesophagealCandida may cause severe dysphagia, but responds to thesame treatment. Hairy leukoplakia affects the lateralborders of the tongue. Unlike Candida, it cannot bescraped off the tongue with ease. The condition ispainless and is due to infection with EBV. Its presence ishighly suggestive of HIV infection. Acyclovir may be ofsome benefit in its management.

The stool reveals evidence of Cryptosporidium, whichis the most common cause of diarrhoea in patients withHIV infection. The diarrhoea may be transient orpersistent, and may be mild or profuse. The diagnosis ismade by staining stool with modified Ziehl–Neelsen stain(262e) to demonstrate red cysts of Cryptosporidium.Treatment may be difficult and usually involves anti-diarrhoeal agents and fluid replacement. Definitive drugsin the management of cryptosporidial infection includespiramycin and paromomycin.

The neurological complications of HIV are discussedin Answer 277. Other complications are shown (Table).1. CMV retinitis.

2. Oesophageal candidiasis.3. Oral hairy leukoplakia.4. i. Cryptosporidial infection.

ii. CMV colitis.5. i. Ganciclovir.

ii. Paromomycin.

Complications of HIV

Respiratory• Pneumocystis carinii pneumonia• CMV pneumonia• Cryptococcal pneumonia• TB (including Mycobacterium avium

intracellulare)• Histoplasmosis• Lymphoma• Kaposi’s sarcoma

Skin• Seborrhoeic dermatitis• Molluscum contagiosum• Folliculitis• TB• Candida infection• Tinea infection• Pityriasis versicolor• Kaposi’s sarcoma

Gastrointestinal• HIV enteritis• CMV colitis • Giardiasis• Cryptosporidium• Isospora belli• Salmonella enteritidis• Shigella dysentery• Candida infection

Mouth• Herpetic stomatitis• Aphthous ulcers• Kaposi’s sarcoma

Page 264: Rapid review of clinical medicine for mrcp part 2

Clinical Cases 263

Question 263

A 16-year-old Nigerian male presented to hospital withsevere pain affecting his left knee and right elbow. Herequired admission to another hospital with similar painthree years ago, and was treated with painkillers. He hadexperienced milder pains at home for which he hadrequired paracetamol. He was one of four siblings andhad an older sister who was known to have a form ofanaemia.

On examination, he was distressed with pain. Histemperature was 38°C (100.4°F). The left knee and rightelbow joints were extremely painful, but joint movementwas not restricted. The rest of the physical examinationwas normal.

Investigations are shown.

Hb 10.2 g/dlWCC 11 � 109/lPlatelets 190 � 109/lMCV 69 flMCHC 29 g/dlReticulocyte count 7.7%Blood film Numerous target cells

and polychromasiaSodium 135 mmol/lPotassium 3.7 mmol/lUrea 6 mmol/lAspartate aminotransferase 48 iu/lBilirubin 40 μmol/lAlkaline phosphatase 130 iu/l

1. What is the mechanism of joint pain in this patient?2. List two investigations you would perform to help

with the diagnosis.3. What is the underlying diagnosis?

Question 264

A 35-year-old Irish nurse returns from Baltimore and isadmitted with sharp right lower chest pain which is worseon inspiration. The following day she develops similarpain in the left lower chest. On examination, she has a

temperature of 39°C (102.2°F) and reduced air entry atthe base of the right lung. There are pleural rubs at thebase of both lungs.

Investigations are shown.

Hb 12 g/dlWCC 3.5 � 109/lPlatelets 300 � 109/lESR 33 mm/hChest X-ray Normal lung fields and

cardiac sizeArterial gases:

pH 7.4PaO2 11.5 kPaPaCO2 4.5 kPaBicarbonate 25 mmol/l

1. List two possible diagnoses which account for allher symptoms and signs.

2. List at least two tests you would perform to confirmeach diagnosis.

Page 265: Rapid review of clinical medicine for mrcp part 2

264

Answer 264

The patient has symptoms suggestive of polyserositis(pleurisy and pericarditis), a high temperature, a slightlyelevated ESR and a relative leucopenia. The most probablediagnosis is SLE, although epidemic myalgia or Bornholm’ssyndrome due to Coxsackie virus (A or B) infection maypresent in a similar fashion. The patient has returned fromBaltimore, where there may have been an epidemic ofCoxsackie virus infection. Familial Mediterranean fever is

also characterized by polyserositis and a fever; however, herplace of origin is against the diagnosis.

SLE is a multi-system disorder due to a vasculiticprocess. The disease is thought to have a genetic andauto-immune basis, although a viral aetiology has beenproposed by some. It usually affects patients below 40years, is more common in Negroes, and has a female-to-male ratio of 10:1. The disease often presents withfatigue, arthralgia and arthritis. Pyrexia is common. Inaddition, there may be renal, pulmonary, cardiac,neurological, haematological and cutaneous involvement.Renal disease is either due to acute diffuse, diffuseproliferative, membranous or focal segmentalglomerulonephritis. Pleurisy, pleural effusions andfibrotic lung disease are well recognized (30–40%).Pericarditis is the most common cardiologicalmanifestation of the condition, affecting 30% of patients.

1. i. Systemic lupus erythematosus.ii. Bornholm’s disease (Coxsackie virus).

2. i. Double-stranded DNA antibody.ii. Antinuclear antibody.iii. CRP.iv. Serum complement.

Answer 263

Sickle cell anaemia (HbSS) is the most common sicklingdisorder and it is inherited as an autosomal recessive trait. Itis due to a point mutation in both �-globin genes, whichresults in the substitution of the amino acid glutamine forvaline at position 6. Other sickling disorders include HbSCdisease (where one �-globin chain is similar to that seen inHbSS and the other �-globin chain is similar to that seen inHbC, another haemoglobin variant), and HbS/�-thalassaemia trait. Patients with HbSC make equal quantitiesof HbS and HbC, whereas those with HbS/�-thalassaemiatrait make predominantly HbS. The amino acid substitutionin HbS causes red cell sickling during deoxygenation,leading to increased rigidity and aggregation in the micro-circulation. This results in a haemolytic anaemia and episodictissue infarction – ‘the sickle cell crises’ (Table).

The sickle cell gene is spread widely throughout Africa,some Mediterranean countries, the Middle East and parts ofIndia. It confers protection to malaria.

Patients with sickle cell anaemia are anaemic (Hb6–10 g/dl) and have a high reticulocyte count (10–20%).The blood film demonstrates polychromasia, target cells andvariable numbers of sickle cells. In general, patients withHbSC disease have a less severe form of anaemia, moretarget cells and fewer sickle cells. They have fewer crises;however, retinal damage, aseptic necrosis of neck of femurand recurrent haematuria are important complications.Pulmonary emboli may occur, particularly during and after

pregnancy. Individuals who are carriers, i.e. those with justone abnormal �-globin gene (sickle cell trait) are notanaemic and do not have clinical abnormalities, but mayrarely develop a sickling crisis in conditions of extremehypoxia. Renal papillary necrosis and an inability toconcentrate urine is recognized in a very small proportion ofindividuals with sickle cell trait.

The diagnosis is made by Hb electrophoresis. A definitivediagnosis requires the demonstration of sickle cell trait inboth parents.

Complications of sickle cell anaemia

• Hand and foot syndrome. Usually seen in infancy• Painful bony crises. Seen later in life. Precipitated

by cold, dehydration, fever or hypoxia• Aplastic crises. May follow infection from

parvovirus B19• Splenic sequestration. Common in infancy. Severe

anaemia and splenic enlargement• Hepatic sequestration. As with splenic

sequestration• Pulmonary syndromes (thrombotic and

sequestration)• Cerebral syndromes (usually thrombotic)• Infections from capsulated organisms

(autosplenectomy)• Gall stones (pigment). Usually in adults• Progressive renal failure (papillary necrosis).

Usually in adults• Chronic leg ulcers. Usually in adults• Recurrent priapism• Avascular necrosis of femoral heads• Salmonella osteomyelitis

1. Bone infarction.2. i. Hb electrophoresis.

ii. Family screening.3. Sickle cell anaemia. It is also possible that he may

have another sickle syndrome such as HbSC diseaseor HbS/β-thalassaemia trait.

Page 266: Rapid review of clinical medicine for mrcp part 2

Clinical Cases 265

Question 265

An 8-year-old male with a known history of haemophiliaA presented with a ten-hour history of pain in the leftgroin, which radiated to the back. He described this as adull pain which was constant in nature. He was givenFactor VIII concentrate and admitted for observation;however, the pain persisted and a few hours later wasradiating into the anterior aspect of the left thigh.

On examination, he appeared unwell. He was afebrile.The heart rate was 90 beats/min and the blood pressurewas 100/55 mmHg. The patient’s left hip was held inflexion. Power testing revealed that he was unable toextend his left hip from the flexed position and kneeextension on the left side was weak. The left knee jerkwas absent, and sensory testing demonstrated reducedsensation in the anterior and medial aspect of the leftthigh. Examination of the abdomen was normal.

Pericardial constriction and tamponade are rarecomplications. Non-infective endocarditis (Libmann–Sacks endocarditis) is extremely rare and is rarely ofclinical significance. It is usually a post-mortem finding.Neurological manifestations include peripheral andcranial neuropathy, aseptic meningitis, seizures, chorea,neuropsychiatric disorders and a syndrome resemblingmultiple sclerosis. Haematological manifestations arecommon, especially in active disease, and include mildnormochromic, normocytic anaemia, leucopenia,thrombocytopenia and auto-immune haemolysis.Leucopenia occurs in 65% of patients, which is in contrastwith the other vasculitides where the WCC is elevated.Skin manifestations include the characteristic malarbutterfly rash on the face in acute SLE or chronic discoidlesions in areas exposed to sunlight. Livedo reticularis,vasculitic skin nodules, alopecia and Raynaud’sphenomenon may also occur.

The collection of symptoms affecting different systemsshould raise suspicion in a young woman, and thediagnosis is confirmed by detection of antibodies todouble-stranded DNA, which is relatively specific forSLE. However, the test is positive in only 60% ofpatients. Antinuclear antibody is present in 90% ofpatients. It is less specific for SLE and may be present in

other vasculitides. The ESR is usually high and mayexceed 100 mm/h. The CRP is characteristically normalunless there is underlying infection, a feature common topatients taking immunosuppressant medication formanagement of disease. Complement levels, particularlyC3, are depressed. Circulating lupus anticoagulant, anantiphospholipid antibody, is detected in 30–40% ofpatients.

Management includes NSAIDs for musculoskeletalproblems. Chloroquine is useful where cutaneousmanifestations predominate. Corticosteroids and otherimmunosuppressants (azathioprine, cyclophosphamideand chlorambucil) are used when disease is severe.

Epidemic myalgia due to Coxsackie A or B (Bornholm’sdisease) is also possible. Despite the name, sporadic casesdo exist. Children and young adults are affected mostcommonly. The symptoms are usually sudden andcomprise abdominal or thoracic pain. Myalgia affecting thechest muscles, fever and headaches are usually present.Pleurisy and pericarditis are recognized features, and bothpleural and pericardial rubs may be present. The diagnosisis clinical, but the virus may be isolated in faeces. Acuteand convalescent sera for Coxsackie virus antibodies willconfirm the diagnosis if there is a four-fold rise in antibodytitres between the two samples.

1. Which of the following cannot explain why he failedto respond?a. Inadequate dose of factor VIII.b. Antibodies to factor VIII.c. Factor VIII may have expired.d. Factor VIII stored at 4°C (39.2°F).e. Deficiency of other clotting factor owing tochronic active hepatitis.

2. What is the cause of the neurological signs?a. Occlusion of the femoral artery.b. Allergic reaction to Factor VIII.c. Femoral nerve compression.d. Prolapsed L3/L4.e. Spinal cord tumour.

3. What is the diagnosis?a. Psoas abscess.b. Psoas haematoma.c. Necrosis of the quadriceps muscle.d. Haemarthrosis affecting the left hip joint.e. Spinal cord arachnoiditis.

4. Which investigation would be least useful in themanagement of this patient?a. CT scan of the abdomen.b. Factor VIII antibody level.c. Factor VIII estimation.d. APTT.e. Bleeding time.

Page 267: Rapid review of clinical medicine for mrcp part 2

266

Answer 265

Haemophilia A is inherited as an X-linked recessivecondition. The incidence is between 30 and 120 permillion. It is due to the deficiency or low levels of FactorVIII, which results in abnormal clotting and a tendencyto spontaneous bleeding or bleeding after minor injury.Important sites of bleeding are the joints, the psoasmuscle, sublingual and pharyngeal areas, renal andgastrointestinal tract. Cerebral haemorrhage is animportant cause of death.

The question requires knowledge regarding importantbleeding sites in haemophilia and the management ofsuch an emergency. The presentation comprises pain inthe left groin, hip held in flexion, absent knee jerk(L3,L4) and reduced sensation in the anterior and medialaspect of the thigh (L2,L3). All these features are typicalof femoral nerve compression (L2–L4). A femoral nervelesion actually causes weakness of hip flexion, andtherefore in this case the flexed hip implies irritation ofthe nerve. In the context of the history of haemophiliathis almost certainly represents a psoas haematoma. (Thefemoral nerve crosses the iliac fossa between the psoasmuscle and the iliacus muscle before entering the thighdeep to the inguinal ligament.) A psoas abscess maypresent with similar neurological signs; however, theabsence of fever or rigors is against this. Haemarthrosis iscommon in haemophilia, and although it is possible for apatient to have haemorrhage into the hip bone, thiswould not explain the sensory signs or the absence of theknee reflex. Spinal cord arachnoiditis generally presents

with root pain and gradual paraplegia. Musclehaematomas may cause necrosis, but in this case themuscle at risk is the psoas, not the quadriceps muscle.Femoral artery thrombosis would be most unusual in ahaemophiliac, and the presentation would consist ofsudden onset of pain and paralysis of the left lower limb.

Failure to manage the bleed may be owing to a varietyof reasons, which have been listed in the items in the firstpart of the question. Patients with Factor VIII antibodiesare those who are completely deficient in Factor VIII, orhave very low (<5%) levels. In this case, possibletherapeutic remedies are very high doses of Factor VIII,purified porcine Factor VIII, Factor IX concentrates oractivated prothrombin complex (contains activated X).An inadequate dose of Factor VIII is a very importantcause of continued bleeding. Factor VIII which has beenheld at a temperature exceeding 4°C (39.2°F) for morethan a couple of hours is not very effective. In the past,up to 50% of patients had CAH, and it is possible thatunderlying chronic liver disease may prevent earlyhaemostasis owing to several other clotting factordeficiencies.

Further useful investigations should consist of a CTscan of the abdomen to confirm the psoas haematoma.The APTT is an indicator of the severity of the clottingabnormality and a good guide to adequate replacementof Factor VIII levels. Factor VIII levels are useful indetermining the amount of Factor VIII which should bereplaced to correct the APTT. Estimation of Factor VIIIantibodies is very useful because it calls for othertherapeutic applications which have been mentionedabove. The BT is of no use because it is unchanged inhaemophilia, but is elevated in von Willebrand’s disease(Table).

1. d. Factor VIII stored at 4°C (39.2°F).2. c. Femoral nerve compression.3. b. Psoas haematoma.4. e. Bleeding time.

Clotting abnormalities

PT APTT TT BT

Haemophilia A N ↑ N NHaemophilia B N ↑ N N*Von Willebrand’s disease N ↑ N ↑Vitamin K deficiency ↑ N N NHeparin therapy ↑ ↑ ↑ (↑)**DIC ↑ ↑ ↑ (↑)**

* Can be differentiated from haemophilia A, by measuring Factor VIII and IX levels. In haemophilia A, FactorVIII is low, and in haemophilia B, Factor IX is low.

** Increased if the platelet count is low.

Page 268: Rapid review of clinical medicine for mrcp part 2

Clinical Cases 267

Question 267

A 37-year-old man was admitted to hospital with a three-day history of a generalized headache and photophobia.He gave a five-day history of a dry cough before the onsetof headache. On examination, he was unwell. Thetemperature was 37.7°C (99.9°F). Kernig’s sign wasabsent. There was no evidence of papilloedema or anyneurological deficit. There was no history of travel abroad.He was married with two children. He was a non-smokerand consumed 4–8 units of alcohol per week. Thefollowing day his condition deteriorated. He becamedrowsy and developed weakness in his lower limbs.

On examination, he was drowsy and his temperaturewas 38°C (100.4°F). He had a left facial nerve palsy andhad failure of lateral gaze on the same side. The upperlimbs were normal. The lower limbs had diminishedpower. Lower-limb reflexes were brisk. The left plantarwas upgoing and the right plantar was equivocal.Abdominal reflexes were diminished.

Investigations are shown.

This is an ECG (266) from an 18-year-old male with asingle episode of syncope. He had no past medical historyof note. His mother died suddenly while swimming atthe age of 29, but no obvious cause was found at post-mortem examination.

CSF analysis:Cells White cells 80/mm3

(40% lymphocytes)Red cells Absent

Protein 0.8 g/lGlucose 3 mmol/l (blood

glucose 4.4 mmol/l)Gram stain NegativeEEG Slow waves

What is the ECG diagnosis?a. Wolff–Parkinson–White syndrome.b. Brugada’s syndrome.c. Long QT syndrome.d. Arrhythmogenic right ventricular

cardiomyopathy.e. Commotio cordis.

What is the diagnosis?a. Viral encephalomyelitis.b. Listeria meningoencephalitis.c. Tuberculous meningitis.d. Lyme disease.e. Cortical vein thrombosis.

Question 266

226666

Page 269: Rapid review of clinical medicine for mrcp part 2

268

Commotio cordis is a term used to describe the onset ofventricular fibril lation after being struck in theprecordium by a highly projectile object. It is arecognized cause of death in ice hockey. Post-mortemexamination reveals a normal heart.

Answer 267

The most likely diagnosis is viral encephalomyelitis. Thereis a prodrome of dry cough in an otherwise fit, relativelyyoung healthy male with no past history or history oftravel, followed by drowsiness and signs consistent withbrainstem involvement. Most cases of viral encephalitis areself-limiting and may present as a febrile illness associatedwith headache and photophobia. In a small proportion ofpatients the illness is more severe and is accompanied byprogressive drowsiness, coma, seizures and focalneurological signs, particularly affecting the brain andspinal cord. Demyelination of the spinal cord andbrainstem is characteristic. The patient has left-sided 6thand 7th cranial nerve palsies, which are suggestive ofpontine involvement. In addition, he has brisk lower-limbreflexes, upgoing plantars, and absent abdominal reflexessuggestive of spinal cord involvement. The CSF reveals amodestly elevated WCC with a significant number oflymphocytes and a modestly elevated protein count. TheCSF sugar content is normal, which excludes tuberculousand bacterial meningitis. Other causes of high lymphocytecount and elevated protein are shown in Table A. TheEEG demonstrates the characteristic slow waves seen inviral encephalitis, although this appearance may also beseen in metabolic encephalopathies and cortical veinthrombosis. The most commonly implicated viruses in viralencephalitis are measles, herpes, and influenza.

Lyme disease may also cause neurological involvement.It is caused by the spirochaete Borrelia burghdorferi,which is transmitted by tick bites. The initial illness (stage1) is characterized by a fever and a typical erythematousindurating rash at the site of the tick bite, known aserythema chronicum migrans. The second stage of thedisease occurs between two and three weeks afterwards,and is characterized by cardiac and neurologicalinvolvement. Cardiac manifestations include pericarditis,myocarditis and heart block of varying degrees.Neurological involvement is characterized by symptoms ofmeningitis, focal neurological signs including cranial nervepalsies and a radiculopathy. There is no history of tickbites or travel to North America; however, the history oftick bites is not always elicited. It is only remotely possiblethat the patient may have been in contact with ticks. CSFfindings include lymphocytosis and a modestly elevatedprotein count (Table B). The CSF sugar is normal. The

diagnosis is made by demonstrating elevated titres of IgGto the spirochaete.

The diagnosis of Listeria meningoencephalitis is lesslikely for several reasons. First, listerosis usually affectsneonates, elderly patients, pregnant women and theimmunosuppressed. Second, the CSF usuallydemonstrates features of a bacterial meningitis (highneutrophil count, high protein and low sugar), but Gramstain is positive in a very few cases. The condition ischaracterized by a febrile illness. Cerebral involvement ischaracterized by features of meningitis and drowsiness.Focal neurological signs and cranial nerve palsies arerecognized features.

Cortical vein thrombosis is characterized by rapid onsetof headache, drowsiness and focal neurological signs andepilepsy. The EEG reveals slow waves. The CSF ischaracterized by a high content of neutrophils andlymphocytes and a high protein content. It maycomplicate bacterial meningitis or tuberculous meningitis.It usually occurs in young children following seriousillness and dehydration. It is a recognized complication ofmyeloproliferative disorders, pregnancy and the

Table A Causes of lymphocytosis and elevatedCSF protein

Infections• Viral meningitis• Tuberculous meningitis• Listeria meningoencephalitis• Neurosyphilis• Leptospirosis• Lyme disease• Brucellosis• Toxoplasmosis• Cryptococcal meningitis• Partially treated bacterial meningitis• Cerebral abscess

Non-infectious• Cerebral lymphoma/leukaemia• Sarcoidosis• SLE• Behçet’s disease• Multiple sclerosis• Cortical vein thrombosis

Answer 266

c. Long QT syndrome (see Question 368). Brugada’ssyndrome is discussed in Answer 353 and ARVC inAnswer 362.

a. Viral encephalomyelitis.

Page 270: Rapid review of clinical medicine for mrcp part 2

Clinical Cases 269

contraceptive pill. There is a possibility that the suddendeterioration in the patient’s condition may be due to thiscomplication. However, CSF features are not in keepingwith bacterial or tuberculous meningitis, the prodromal

illness was short and did not seem severe, and his pastgood health is against a coexisting disorder that may havepredisposed to cortical vein thrombosis.

Table B CSF findings in infectious meningitis

Pressure WCC (mm3) Protein g/l SugarViral meningitis Normal 10–100 (lymphocytes) 0.4–0.8 >1/2 blood glucoseor encephalitis* Slightly raised

Bacterial meningitis** Raised 200–3,000 (neutrophils) 0.5–2.0 <1/3 blood glucose

TB meningitis Raised 50–500 (lymphocytes) 0.5–3.0 <1/3 blood glucose

Lyme disease Normal 50–500 (lymphocytes) 0.4–0.8 >1/2 blood glucoseSlightly raised

Cryptococcal Normal 50–300 (lymphocytes) 0.5–3.0 Usually lowmeningitis Slightly raised

* Similar findings may be seen in cerebral toxoplasmosis, leptospiral meningitis, brucellosis, Listeria meningitis.** In partially treated bacterial meningitis there may be evidence of a pleiocytosis (presence of both lymphocytesand neutrophils) where the predominant cells are lymphocytes. The sugar content of the CSF may be onlyslightly reduced.

Question 268

A 20-year-old female is admitted to hospital with a 12-hour history of weakness in her arms and legs. This wasaccompanied by paraesthesia in her hands and feet. Fivedays before admission she experienced severe painbetween her shoulder blades, and shortly afterwardsdeveloped blurred vision. The back pain had persistedbut was less severe on admission. There was no history ofheadaches or nausea, but she had been constipated fortwo days and had intermittent lower abdominal pain. Atthe age of eight years she suffered a bout of viralmeningitis for which she was in hospital for a week. Atthe age of 15 years she took an overdose of sleepingtablets while her parents were undergoing a divorce. Shehad enjoyed good health otherwise.

On examination, the patient was not distressed buthad an expressionless face and found it difficult to close

her eyes. She was afebrile. Examination of her fundidemonstrated blurred discs but no haemorrhages. Powerin all her limbs was markedly diminished. She found itdifficult to raise her arms above her shoulders, and couldnot move her legs. The tone in the legs was diminishedand the reflexes in all her limbs were absent. Abdominalreflexes were present and the plantar response wasnormal. On examination of the abdomen she had asmooth palpable mass just above the symphysis pubis.

Investigations were as follows:

Hb 13 g/dlWCC 7 � 109/lPlatelets 300 � 109/lCT scan of brain Normal

1. What two investigations (apart from CSFexamination) would affect your immediatemanagement?a. Forced vital capacity.b. Muscle biopsy.c. Serum creatinine phosphokinase.d. CSF analysis.e. Urinary porphobilinogen estimation.f. MRI scan brain.g. Tensilon test.

h. Arterial blood gases.i. Stool culture.

2. What is the diagnosis?a. Myasthenia gravis.b. Acute polymyositis.c. Acute intermittent porphyria.d. Multiple sclerosis.e. Guillain–Barré syndrome.

Page 271: Rapid review of clinical medicine for mrcp part 2

270

Answer 268

The patient presents with generalized muscle weaknessfollowing severe pain in the mid-thoracic area. She alsocomplained of diplopia before the weakness, and hasevidence of papilloedema and generalized motorneuropathy on examination. The patient has no sensorylevel to suggest spinal cord compression, therefore theconstipation and the palpable bladder indicate anunderlying autonomic neuropathy. The diagnosis which fitsbest with the symptoms and signs is Guillain–Barrésyndrome or post-infectious polyneuropathy.

Guillain–Barré syndrome is a polyneuropathy which maydevelop over a few days or even a few weeks after aninfectious illness, usually involving the respiratory tract.Enterovirus, EBV, Mycoplasma, Campylobacter jejuni andChlamydia have been implicated, but the condition mayoccur following surgical conditions. In the majority of casesthe underlying cause is never identified. The onset is oftenheralded by severe lumbar or interscapular pain, as in thiscase, and is followed by proximal, distal or generalizedmuscle weakness due to a motor neuropathy; however,some sensory loss is also present. Sensory loss commonlymanifests as distal paraesthesiae and loss of vibration andpositional sensation. The latter results in a sensory ataxia.

The cranial nerves may also be involved, particularly thoseinnervating the extraocular muscles of the eye. In thecontext of the history above, the diplopia is almost certainlydue to cranial nerves supplying the extraocular muscles.Bulbar involvement is also recognized. Autonomicneuropathy may complicate some cases, and may present asaccelerated hypertension, tachyarrhythmias, abdominalpain, and bladder atony. Death may occur from respiratoryfailure due to respiratory muscle paralysis, or fromarrhythmias and haemodynamic instability due toautonomic neuropathy. Aspiration may also cause death inpatients with bulbar muscle involvement. The differentialdiagnosis and prognostic markers of the disease are listedbelow (Tables A and B).

The disease is characterized by a significantly elevatedprotein content in the CSF in the presence of a normal CSFWCC. However, cases due to infectious mononucleosismay be associated with a mild pleiocytosis. The differentialdiagnosis of a high CSF WCC and a high protein content istabulated below (Table C). Papilloedema may develop dueeither to accelerated hypertension or to impaired CSFresorption because of the elevated CSF protein content.The presence of papilloedema in this case makes itmandatory to measure the blood pressure. Histologically,there is segmental demyelination – possibly due to a cell-mediated delayed hypersensitivity. In the form ofGuillain–Barré which accompanies Campylobacter jejuniinfection there is an acute, purely axonal motor neuropathywith rapid progression and a mortality rate as high as 5%.

Management of the disease is by intravenousimmunoglobulin or plasmapharesis. There is no role forcorticosteroids in the management of Guillain–Barrésyndrome.

A variant of Guillain–Barré is the Miller–Fishersyndrome, which comprises external ophthalmoplegia,ataxia and areflexia. The condition is highly suggested bythe presence of antibodies to the sphyngomyelincomponent GQ1b.

Table A Differential diagnoses ofGuillain–Barré syndrome

• Acute polymyositis• Myaesthenia gravis• Acute intermittent porphyria (motor and

autonomic neuropathy)• Lead poisoning• Hypokalaemic periodic paralysis

Table B Poor prognostic markers inGuillain–Barré syndrome

• FVC <1.1 litres• Arterial blood oxygen <8 kPa• Arterial blood carbon dioxide >6 kPa• Coexistent lower respiratory tract infection• Coexistent pulmonary oedema• Accelerated hypertension• Arrhythmias• Bulbar muscle involvement• Guillain–Barré syndrome following

Campylobacter infection

1. a. Forced vital capacity.h. Arterial blood gases.

2. e. Guillain–Barré syndrome.

Table C Causes of a high CSF protein in thepresence of a normal CSF cell count

• Guillain–Barré syndrome• Spinal cord tumour causing spinal block (Froin’s

syndrome)• Acoustic neuroma• Lead encephalopathy• Subacute sclerosing panencephalopathy

Page 272: Rapid review of clinical medicine for mrcp part 2

Clinical Cases 271

Question 269

This pedigree chart (269a) is from a family with ableeding disorder.

A B

C D

E F

Affected male Unaffected male*

Affected female Unaffected female*

* Does not exclude inheritance of an autosomalrecessive gene in either sex or an X-linked recessivedisease in a female.

Question 270The following are cardiac catheter data from an 18-year-old female with breath lessness on exertion.

Chamber Pressure Oxygen saturation (mmHg) (%)

Superior vena cava – 68Inferior vena cava – 67Right atrium a = 14

v = 20(high) 68(mid) 69(low) 83

Right ventricle 92/20 83Pulmonary artery 94/42 82PCWP a = 14

v = 22Left ventricle 106/20 96Aorta 100/70 96

1. List at least four physical signs that youwould elucidate on examination of thispatient.

2. List at least three abnormalities which maybe seen on the ECG.

3. What is the diagnosis?

1. What is the mode of inheritance?a. X-linked recessive. d. Autosomal recessive.b. Autosomal e. Autosomal dominant

dominant. with incompletec. X-linked dominant. penetrance.

2. What is the most probable diagnosis?a. von Willebrand’s d. Dysfibrinogenaemia.

disease. e. Factor VII b. Haemophilia A. deficiency.c. Antithrombin III

deficiency.

3. What are the chances of A being a carrier?a. 0%. b.25%. c. 50%. d. 75%. e. 100%

4. What are the chances of B being affected?a. 0%. b.25%. c. 50%. d. 75%. e. 100%

5. What are the chances of C being affected?a. 0%. b.25%. c. 50%. d. 75%. e. 100%

6. What are the chances of D being a carrier?a. 0%. b.25%. c. 50%. d. 75%. e. 100%

7. What are the chances of E being affected?a. 0%. b.25%. c. 50%. d. 75%. e. 100%

8. What are the chances of E being a carrier?a. 0%. b.25%. c. 50%. d. 75%. e. 100%

9. What are the chances of F being a carrier?a. 0%. b.25%. c. 50%. d. 75%. e. 100%

226699aa

Page 273: Rapid review of clinical medicine for mrcp part 2

272

Answer 269

The answer is illustrated (269b). In this pedigree,only males are affected. Therefore, the condition mustbe X-linked recessive. In X-linked recessive conditions,only males will be affected; females will be carriers. Inthis pedigree, the female in generation I is a definitecarrier because she has two affected sons (who haveinherited the X-chromosome with the abnormal genefrom their mother). Female A in generation II is alsoa definite carrier because she has an affected son.There is a 50% chance that female B is a carrierbecause a female carrier (her mother) will transmit theabnormal gene to 50% of her sons and 50% of herdaughters. A female harbouring an X-linked recessivegenetic abnormality will not be affected – assuming herother X-chromosome is free of the genetic mutation.Female C is a definite carrier because her father is affected.In X-linked conditions the male passes his X-chromosome(with the abnormal genetic mutation) to all his daughters.In contrast none of the sons receive the abnormal gene fromthe father because they inherit his Y chromosome. For thesame reasons, female D is also a definite carrier.

There is a 50% chance that female X in generation IV is acarrier, therefore half of her sons will be affected and half of

her daughters will be carriers, that is, there is a 25% chanceof male E being affected and a 25% chance of female Fbeing a carrier.

The underlying diagnosis is haemophilia A. The otherpossibility is haemophilia B, which is also inherited as anX-linked recessive trait and is due to abnormally lowFactor IX levels. Dysfibrinogenaemia is inherited as anautosomal dominant or recessive trait. Factor Vdeficiency is inherited as an autosomal recessive trait andmost cases of von Willebrand’s disease are inherited as anautosomal dominant trait.

1. a. X-linkedrecessive.

2. b. Haemophilia A.3. e. 100%.4. a. 0%.

5. a. 0%.6. e. 100%.7. b. 25%.8. a. 0%.9. b. 25%.

A B

C D

A 100% chance of being acarrier because she hastransmitted the abnormalgene to her son

B 50% chance of being acarrier

C 100% chance of being acarrier

D 100% chance of being acarrier but will not beaffected

E 25% chance of beingaffected

F 25% chance of being acarrier

X 50% chance of being acarrier

100% chance of being a carrier because her sons are affected

X

Answer 270A step up in oxygen saturation in the upper right atriumwould indicate a sinus venosus defect, and a step up in themid right atrium would indicate an ostium secundum ASD.(See Interpretation of Cardiac Catheter Data, page 418.)

1. i. A right ventricular heave. The right ventricle musthave hypertrophied to generate such a high systolic pressure.

ii. A pansystolic murmur of mitral regurgitation at the apex. The large ‘v’-wave in the PCWP suggests mitral regurgitation.

iii. Fixed splitting of second heart sound from ASD.iv. Pansystolic murmur of tricuspid regurgitation at

the left sternal edge. The large ‘v’-wave in the right atrial pressure suggests tricuspid regurgitation.

v. Systolic flow murmur in the pulmonary area.

I

II

III

IV

V

2. i. Partial RBBB.ii. Right ventricular hypertrophy.iii. Left atrial enlargement.iv. Right atrial enlargement.v. Left axis deviation. The ECG in ostium primum

defects characteristically reveals partial orcomplete RBBB and left axis deviation (270). In contrast, patients with ostium secundum defects have partial RBBB and right axis deviation.

3. There is a step up in the oxygen saturation in thelow right atrium, suggesting an ostium primumASD. The defect is associated with mitral andtricuspid ring defects, causing both mitral andtricuspid regurgitation.

E F

226699bb

227700

Page 274: Rapid review of clinical medicine for mrcp part 2

Clinical Cases 273

A 19-year-old dustman was admitted to hospital withsevere retrosternal chest pain associated with belchingand vomiting. He was a non-smoker. He had beencelebrating a friend’s birthday and had consumed aconsiderable amount of alcohol the night before. He wasdiagnosed as having epilepsy 12 years ago and was onregular anticonvulsant medication. There was no familyhistory of ischaemic heart disease or epilepsy. He had anolder brother who was fit and well. He had been inhospital eight months previously with an axillary veinthrombosis which had been treated with warfarin for sixmonths. He wore glasses for being short-sighted.

On examination, he was thin and tall, measuring 1.9 m. He had a kyphoscoliotic spine. Examination of hishands, oral cavity and left eye is shown (271a–c,respectively). The findings on fundoscopy are also shown(271d). A cardiovascular examination was normal. Thepatient had pectus excavatum; the chest was clear onauscultation. An abdominal examination revealed mildtenderness in the epigastrum.

Investigations are shown.

Hb 11 g/dlAST 60 iu/lWCC 5 � 109/lBilirubin 11 μmol/lPlatelets 160 � 109/lAlkaline phosphatase 112 iu/lMCV 105 flAlbumin 43 g/lU&E NormalChest X-ray Kyphoscoliosis

Clear lung fieldsNormal heart size

ECG NormalGastroscopy Gastro-oesophagitis

1. What is the diagnosis?2. Give two explanations for the raised MCV.3. How would you confirm the diagnosis?4. How is the condition inherited, and where is the

chromosomal defect?

Question 271

227711aa 227711bb

227711cc 227711dd

Page 275: Rapid review of clinical medicine for mrcp part 2

274

Answer 271

The chest pain due to oesophagitis has no connectionwith the underlying diagnosis. The examiner expects thereader to pick up on the skeletal, ocular and neurologicalabnormalities to come to a diagnosis of homocystinuria.

Homocystinuria is due to cystathionine deficiency. Asa result, there is accumulation of homocysteine which inturn is oxidized to homocystine. There is no productionof cystine. Plasma and urinary levels of homocystine areelevated. Homocystine is thought to affect cross-linkingof collagen, leading to skeletal, ocular, neurological andvascular abnormalities.

Skeletal: The patients have the same skeletalabnormalities as in Marfan’s syndrome. These includetallness, kyphoscoliosis, arachnodactyly, high-archedpalate, pectus excavatum and flat feet. The ligaments arelax, and muscular development weak. Osteoporosis is a

feature and classically affects the vertebrae. These patientsneed to be distinguished from Marfan patients, who havea strong family history, and often have evidence ofcardiovascular involvement in the form of mitral valveprolapse and aortic root dilatation, but do not haveassociated neurological abnormalities. Osteoporosis is nota feature of Marfan’s syndrome.

Ocular: The suspensory ligaments of the lens areweakened, causing lens dislocation which is usuallydownward. This is complicated by glaucoma,iridodonesis, myopia and cataracts. Optic atrophy andretinal detachment are also recognized ocular problems.

Neurological: Mental handicap occurs inapproximately 50% of patients and 10–20% of patientshave epilepsy. The degree of mental retardation can bemild or severe. There is a form of homocystinuria whichresponds to pyridoxine. Patients with this form are lessseverely affected.

Vascular: Both venous and arterial thromboses occurand are thought to be due to the effects of methioninecausing vascular damage, and increased plateletadherence. This has important therapeutic implications inpatients undergoing surgery for correction of skeletalabnormalities, because there is a risk of deep-veinthrombus, and prophylactic heparin should be givenfollowing surgery. The oestrogen-containing oralcontraceptive pill is contraindicated in women withhomocystinuria. Recent studies have shown thathomocystinuria is also associated with prematurecoronary artery disease.

Treatment is with dietary restriction of methionineand cystine supplements. Pyridoxine should be givenimmediately after the diagnosis has been made and isbeneficial in some cases.

1. Homocystinuria.2. i. Phenytoin therapy. Phenytoin is thought to

interfere with folate absorption. Other anticonvulsants causing folate deficiency include primidone and barbiturate therapy.

ii. The conversion of homocystine to methioninerequires large amounts of the enzyme 5-methyl tetrahydrofolate methyltransferase, which utilizes folate as a cofactor. During the enzyme reaction some of the folate is degraded, and folate deficiency can occur.

3. The urine gives a positive nitroprusside test. Raised levels of plasma methionine and homocystine are more reliable because the nitroprusside test is also positive in cystinuria.

4. Autosomal recessive inheritance. The gene is located on chromosome 21.

Page 276: Rapid review of clinical medicine for mrcp part 2

Clinical Cases 275

Question 273

A 78-year-old male presented with a several weeks’history of weakness, lethargy, weight loss and vomiting.

On examination, the blood pressure (standing) was80/50 mmHg.

Blood results are shown.

Sodium 124 mmol/lPotassium 4.9 mmol/lUrea 15 mmol/lBicarbonate 15 mmol/lBlood glucose 4.0 mmol/l

Question 272

A 59-year-old farmer was referred to a chest unit with an 18-month history of progressive breathlessness. Over the pastfour years, he had noticed a dry cough in the afternoons andevenings. His effort tolerance three years previously was verygood, allowing him to work on the farm for almost 12 hoursper day. Since then he had resolved to work only two days aweek owing to fatigue and breathlessness, and had askedboth his sons to take over his farm.

His GP diagnosed asthma, but his condition hadcontinued to deteriorate despite the use of steroid andsalbutamol inhalers. He was now breathless after walkingjust 200 metres. He was a non-smoker. There was nofamily history of lung disease. He had never worked withasbestos, and did not keep pets.

On examination, he was cyanosed and tachypnoeic.He had evidence of early clubbing. The JVP was notraised, and heart sounds were normal. On auscultation ofthe lung fields there were bilateral fine end-inspiratorycrackles in both lungs.

Investigations are shown.

Hb 12 g/dlWCC 5 � 109/lPlatelets 400 � 109/lESR 54 mm/hChest X-ray Diffuse reticulonodular

shadowing in both apices and midzones

FEV1 (%) 50 Blood gases on air:FVC (%) 52 pH 7.45FEV1/FVC (%) 89 PaCO2 3.5 kPaTLC (%) 56 pO2 6.4 kPaKCO (%) 60 Bicarbonate 22 mmol/l

1. What is the diagnosis?2. List at least two tests that would help to confirm

the diagnosis.

List three possible causes for this biochemical picture.

Question 274

A 51-year-old dentist presented to his GP because he haddifficulty lifting his right arm for over one week. For thepast few days he could not hold the razor to his face withhis right arm while shaving. About two weeks previouslyhe received a hepatitis B booster vaccine which wasassociated with pain over his deltoid and shoulder regionfor almost 48 hours.

On examination there was wasting over the deltoidregion. There was reduced power on abduction, flexionand internal rotation of the right arm and flexion of theright elbow. The right biceps and supinator reflexes wereabsent. All other examinations were normal.

Investigations are shown.

Hb 14.3 g/dlWCC 5 � 109/lPlatelets 199 � 109/lESR 11 mm/hBiochemistry NormalX-rays of chest and Normal

cervical spine

What is the diagnosis?a. Radiculopathy secondary to Lyme disease.b. Neuralgic amyotrophy.

c. Syringomyelia.d. Cervical myelopathy.e. Motor neurone disease.

Page 277: Rapid review of clinical medicine for mrcp part 2

276

Answer 273

The combination of lethargy, weight loss, vomiting,hypotension, hyponatraemia and hyperkalaemia issuggestive of Addison’s disease. Addison’s disease is causedby destruction of the adrenal glands, resulting in deficiencyof glucocorticoid and mineralocorticoid hormones. Themost common cause of Addison’s disease is auto-immuneadrenalitis. Other causes are listed (Table). The conditionhas an insidious presentation in the majority of cases, butmay present as a crisis. Lethargy, anorexia, nausea,vomiting, abdominal pain and symptoms of posturalhypotension are common. Some patients are misdiagnosedas having depression. In the elderly, the condition maypresent as a confusional state. On examination, there maybe evidence of pigmentation which is prominent in skincreases, scars and the buccal mucosa. The supine bloodpressure may be normal, but there is a marked drop insystolic pressure on standing (>25 mmHg).

Acute Addisonian crisis is characterized by coma,profound hypotension and hypoglycaemia. Theelectrolytes are usually abnormal (as above) but may alsobe normal. Acute adrenal failure or Addisonian crisis is amedical emergency which should be aggressively treatedwith intravenous hydrocortisone and saline. Dextrose isused in hypoglycaemic cases.

Long-term treatment is with oral hydrocortisone,prednisolone or dexamethasone (glucocorticoids) andfludrocortisone (mineralocorticoid).

Addison’s disease is diagnosed by performing thesynacthen test. Serum cortisol is measured before and 30min after intramuscular ACTH (synacthen). In normalindividuals, the serum cortisol rises to above 600 nmol/l,or is increased by 300 from the basal level. Patients withAddison’s disease will not demonstrate this level of rise.

Causes of Addison’s disease (primaryhypoadrenalism)

• Autoimmune adrenalitis• TB• Carcinomatous infiltration of adrenal glands

(bronchial carcinoma)• Adrenal haemorrhage or infarction

(Waterhouse–Friedrichson syndrome)• Infiltration disorders:

• haemochromatosis• amyloidosis• granulomatous disorders

• Metyrapone therapy• Schilder’s disease• Wolman’s disease (hepatosplenomegaly and

steatorrhoea)

Answer 272

Extrinsic allergic alveolitis is a cell-mediated allergicreaction that occurs in response to inhalation of a varietyof agents. It is characterized by interstitial inflammationand progressive lung fibrosis. Patients typically presentwith fever and dyspnoea several hours after inhaling theantigen. In this case, the farmer who has been working inthe morning will not develop symptoms until lateafternoon. During the reaction the patient will betachypnoeic, febrile and have widespread coarse cracklesthroughout the lung. Continuing exposure andinflammation results in interstitial fibrosis, leading toprogressive dyspnoea and functional limitation. The chestX-ray reveals nodular shadows in the upper zones thateventually involve the mid-zones. In severe cases the lung

will have a honeycomb appearance on the X-ray owing tosevere fibrosis. Several antigens are recognized andseveral occupations are at risk. In farmers, the culpritantigen is commonly the spores of the fungus M. faeniithat are inhaled while forking mouldy hay. Malt workersmay be affected by inhalation of products of the fungusAspergillus clavatus while turning germinating barley,and pigeon keepers may develop the condition afterinhaling avian proteins from feathers of the birds.

The diagnosis relies on an abnormal chest X-ray, andthe demonstration of antibodies to the culprit antigens.High-resolution CT scan of the thorax will reveal theextent of parenchymal involvement. Formal spirometry isuseful in assessing lung reserve in patients with fibrosis.The KCO is reduced owing to interstitial lung damage.Transbronchial lung biopsy is rarely required to make thediagnosis unless the cause of lung fibrosis is unclear.Bronchoalveolar lavage reveals increased cells that arepredominantly lymphocytes.

Management is to remove the patient from theantigen. In the early stages of the disease, high-doseprednisolone will cause regression of the condition.

1. Extrinsic allergic alveolitis.2. i. Precipitins from Micropolyspora faenii.

ii. High-resolution CT scan.iii. Bronchoalveolar lavage.iv. Bronchoscopy with transbronchial lung biopsy.

i. Addison’s disease/primary hypoadrenalism.ii. Diuretic therapy.iii. Salt-losing nephritis.

Page 278: Rapid review of clinical medicine for mrcp part 2

Clinical Cases 277

Adrenal antibodies are present in cases of auto-immuneadrenalitis. In cases of tuberculous adrenal glanddestruction there may be obvious calcification of theadrenal gland on the plain abdominal X-ray.Alternatively, the demonstration of an inappropriatelyraised serum ACTH and a low simultaneous serumcortisol level is diagnostic.

Diuretic therapy with spironolactone (aldosteroneantagonist) or combined loop diuretic and potassium-

sparing diuretic preparations will produce the samebiochemical picture. Dehydration from diuresis may leadto pre-renal failure, reflected by rising urea and a lowbicarbonate. The serum aldosterone is elevated indehydration due to diuretics. This is in direct contrast toAddison’s disease, where the aldosterone level will below. Salt-losing nephritis is characterized biochemicallyby hyponatraemia and impaired renal function.

Question 275

A 28-year-old obese female was admitted to the ward forfurther investigation after presenting with lethargy. Shedid not give any further history of significance. Her bloodpressure in clinic was 128/85 mmHg. Blood results areshown.

After these blood results, the patient was questionedfurther about her bowel habit and medication. She denied

having diarrhoea or taking any medication. She wasobserved on the ward for 48 hours, during which timeher blood pressure did not fluctuate.

A repeat serum renin level was 1,600 pg/ml/h.

Sodium 135 mmol/lPotassium 2.2 mmol/lUrea 7.3 mmol/lCreatinine 60 μmol/lBicarbonate 36 mmol/lRenin 1,500 pg/ml/h

(NR [lying supine] 300–1,300 pg/ml/h)

24-h urine output 500 ml24-h urine potassium 10 mmol/l

1. Comment on the initial results.2. Suggest a possible cause for the abnormalities, given

the results of the tests performed while she wasobserved in hospital.

Answer 274

The diagnosis is neuralgic amyotrophy. This is aninflammatory process affecting the roots of C5 and C6and occasionally C7. It usually follows an infective illnessor trauma to the deltoid region. It is preceded by painover the shoulder and arm followed by weakness andwasting of the muscles supplied by the affected roots.Spontaneous recovery is usual but may take severalmonths. The differential diagnosis is cervical cord

compression, but the absence of lower limb signs isagainst this. Lyme disease may produce a similarradiculopathy but in this case there is no history of a skinrash, tick bite, cardiac involvement or arthropathy.Furthermore, the history of immunization into thedeltoid is more in keeping with amyotrophy.Syringomyelia is unlikely in the absence of sensory signs.Although motor neurone disease may present with purelower motor neurone signs as in this case, the rapid onsetof neurological signs in just one limb does not favour thediagnosis.

b. Neuralgic amyotrophy.

Page 279: Rapid review of clinical medicine for mrcp part 2

278

Answer 275

Potassium may be lost via the GI tract (diarrhoea,vomiting, intestinal fistulae, villous adenoma, nasogastricaspiration) or the kidneys (primary and secondaryhyperaldosteronism, Cushing’s syndrome, Liddle’ssyndrome, renal tubular disorders or drugs). Loss ofpotassium from the GI tract is associated withconservation of potassium by the kidneys.

Hypokalaemia is generally associated with a metabolicalkalosis with the exception of RTA.

Many conditions associated with renal loss ofpotassium also cause hypertension. The most commoncause of hypertension, hypokalaemia and alkalosisencountered in the MRCP and similar examinations isprimary hyperaldosteronism (Conn’s syndrome).However, other recognized causes include Cushing’s,Liddle’s syndrome (pseudohyperaldosteronism),accelerated hypertension, renal artery stenosis orhypertensive therapy with diuretics (Table). Liddle’s

1. The patient has a hypokalaemic alkalosis and anappropriate hypokaluria. Patients with a serumpotassium of <3.5 mmol/l conserve potassium andexcrete <20 mmol/l of potassium in the urine. Theurea is modestly elevated, as is her serum renin,both of which suggest hypoperfusion of the afferentglomerular vessels. This may occur in dehydration,shock, glomerular and renal parenchymal disease.However, in primary renal disease the serumcreatinine is also elevated, and in this history theblood pressure is not suggestive of shock.

2. In the context of this history the most probablecause for the hypokalaemia is loss of potassium fromthe gastrointestinal tract. This is also causingdehydration and hence the elevated urea and renin.There is no history of diarrhoea or vomiting, butgiven her obesity there is a chance that she isabusing laxatives or has self-induced vomiting inorder to try and lose weight (275).

Serumpotassium<3.5mmol/l

Checkurinarypotassium

>20 mmol/l

?Hypertensive

Yes

?Renin concentration

Malignant hypertensionRenal artery stenosisDiuretic therapy

Conn’s syndromeCushing’s syndromeLiddle’s syndrome

No

Check bicarbonate

DiureticsCarbenoloxoneAminoglycosidesBarrter’s syndrome

<20 mmol/l

Gastrointestinallosses:• diarrhoea• vomiting• reduced intake (starvation)

Algorithm for the investigation of hypokalaemia

Increased

Decreased

RTA

227755

Page 280: Rapid review of clinical medicine for mrcp part 2

Clinical Cases 279

Question 276

A 17-year-old female was referred to the gynaecologistwith primary amenorrhoea. She had been relatively wellthroughout her life. Her mother was concerned that shewas not gaining any weight, despite eating well. She hadbeen investigated for diabetes by the GP and was foundto have a normal blood glucose. Apart from complaining

of intermittent headaches and feeling tired at the end ofthe day, there were no other symptoms. She was still atcollege. She had two brothers and a sister who were well.Her sister was aged 14 and had just started menstruating.

On examination, the patient measured 1.5 m andweighed 40 kg. She was pale, and had scanty axillary andpubic hair. Breast development had just begun.

Investigations are shown.

Causes of hypokalaemic alkalosis and hypertension

• Primary hyperaldosteronism (Conn’s syndrome)• Cushing’s syndrome• Liddle’s syndrome• Accelerated hypertension• Hypertension treated with diuretics• Renal artery stenosis• Carbenoxolone therapy• Liquorice abuse• Congenital adrenal hyperplasia (11-β-hydroxylase deficiency)*

*In 11-β-hydroxylase deficiency, the aldosterone levels are low; however, there is excessive production ofdeoxycorticosterone which has strong mineralocorticoid effects and causes both hypertension and hypokalaemia.

Hb 9.8 g/dlWCC 5 � 109/lPlatelets 190 � 109/lMCV 104 flBlood film Target cells, anisocytosis,

macrocytes, poikilocytosisSodium 137 mmol/lPotassium 4.1 mmol/lUrea 3 mmol/lCreatinine 78 μmol/lCalcium 2.0 mmol/lPhosphate 0.75 mmol/lAlbumin 37 g/lAST 20 iu/lAlkaline phosphatase 300 iu/lBilirubin 11 μmol/lThyroxine 100 nmol/lTSH 2.0 mu/l

1. What is the cause of the patient’s amenorrhoea?2. What is the most probable underlying diagnosis?3. Give two possible explanations for the alkaline

phosphatase level.4. Give two tests you would perform to confirm the

diagnosis.

syndrome is a tubular defect characterized by excessivesodium absorption and increased potassium excretion.The biochemistry is identical to that of Conn’s syndrome,but there is no abnormality of the renin–angiotensin–aldosterone axis.

In the absence of hypertension, other causes ofhypokalaemic alkalosis include carbenoxolone therapy,

liquorice abuse, aminoglycosides, and Bartter’s syndrome– which is due to hyperplasia of the juxtaglomerularapparatus. In Bartter’s syndrome, affected childrenpresent with nocturia, muscle weakness and failure tothrive. It is important to note that carbenoxolone therapyand liquorice abuse may cause hypertension.

Page 281: Rapid review of clinical medicine for mrcp part 2

280

Answer 276

The patient is short for her age. She has a macrocyticanaemia. The blood film reveals hypochromasia,macrocytosis and target cells, suggesting either isolatedfolate (or possibly B12) deficiency or combined iron andfolate deficiency. The bone biochemistry is in keepingwith vitamin D deficiency. The slightly low albumin levelis indicative of poor nutritional status in this context.Failure to grow, or weight loss despite a normal appetite,plus a combination of nutritional deficiencies shouldalways raise the question of a malabsorption syndrome.The lack of secondary sexual characteristics and delayedmenstruation in this patient may be explained by the factthat she has yet to reach puberty, either because of aconstitutional delay or because puberty has been delayedowing to a malabsorption syndrome.

Malabsorption syndromes are a recognized cause ofprimary and secondary amenorrhoea in girls and youngwomen, predominantly as a result of poor nutritioncausing profound weight loss. Readers may fall into thetrap of diagnosing hypopituitarism in this case, given theshort stature and absent secondary sexual characteristics.However, the diagnosis of hypopituitarism fails to explainthe normal thyroid function tests and the abnormal bonebiochemistry. Anaemia is a recognized feature ofhypopituitarism. It is usually normochromic normocytic,but may be macro- or microcytic.

A variety of conditions produce malabsorptionsyndromes (Table), but the most common cause by far inthe Western world is coeliac disease. This affects 1 in2,000 patients in the UK, and is even more common inIreland, affecting 1 in 300. The disorder is characterizedby total villous atrophy in the small intestine as a result ofingestion of gluten-containing food. The majority (80%)of patients have the haplotype HLA A1, B8, DR7 orDQW2, suggesting a genetic basis. There is 30%disconcordance among identical twins, which suggeststhat non-genetic factors may also be involved. Smallbowel injury occurs as a result of gluten ingestion. Glutenis a high-molecular weight compound which containsvarious gliadins. �-gliadin damages small bowel mucosa

in these patients, probably through an immune-mediatedmechanism. Antigliadin antibodies are strong evidencefor this postulation. Small bowel damage producesmalabsorption which manifests as weight loss, abdominalpain, diarrhoea and nutritional deficiencies. Diarrhoeamay be absent in up to 20% of patients. The disease canpresent at any age. In children, it occurs soon afterweaning and presents as failure to thrive. In adults, itpresents with abdominal symptoms, weight loss andfatigue. Ankle oedema may complicate albumindeficiency. Mouth ulcers may occur as a result of irondeficiency. Osteomalacia, tetany and muscle weaknessmay be present owing to vitamin D deficiency.

Anaemia is present in 50% of cases. Macrocytosis isinvariably due to folate deficiency. Iron deficiency iscommon, but B12 deficiency is rare. Hypochromia andtarget cells may be present on the blood film.

Howell–Jolly bodies may be seen on the blood film inpatients who develop splenic atrophy. The diagnosis ismade by sampling small bowel from the second portionof the duodenum (D2 biopsy) via endoscopy or a jejunalbiopsy (276). Antiendomyosium antibodies are detectedin almost 50% of all cases of coeliac disease, and arehighly specific for the disorder. Antigliadin antibodiesand antireticulin antibodies are found in the serum of themajority of patients.

Treatment is with a gluten-free diet. Gluten is presentin the cereals wheat, rye, barley and oats, all of whichshould be excluded from the diet.

Associations of coeliac disease include auto-immunethyroid disorders, chronic liver disease, fibrosingalveolitis, ulcerative colitis and insulin-dependent diabetesmellitus. Complications include small bowel lymphomaand adenocarcinoma and splenic atrophy.

1. Underweight owing to malabsorption(malabsorption syndrome).

2. Coeliac disease.3. i. Osteomalacia.

ii. Patient in a growth spurt.4. i. Duodenal biopsy or jejunal biopsy.

ii. Antiendomyosium, antigliadin or antireticulinantibodies.

227766

Page 282: Rapid review of clinical medicine for mrcp part 2

Clinical Cases 281

Small bowel malabsorption syndromes

Disorder Additional comments

Coeliac disease High antigliadin or antireticulin antibodiesAntiendomyosium antibodies highly specific

Dermatitis herpetiformis Gluten-sensitive enteropathy. Associated subepidermal blistering rash

Bacterial overgrowth Low B12 but normal or high serum folate. Previous gastric surgery (Billroth II), jejunal diverticuli

Giardiasis May be detected on stool microscopy. Travel abroad or immunosuppressed

Whipple’s disease Associated locomotor/neurological/cardiac features HLA B27PAS-positive macrophages which contain the bacillus Tropheryma whipelli onsmall bowel biopsy

Tropical sprue History of living in an endemic area for >3 months

Radiation enteritis Previous history of abdominal irradiation

Crohn’s disease Abdominal pain/diarrhoea predominate. May have isolated B12 deficiencywhich is not corrected with intrinsic factor

Hypogammaglobulinaemia Very similar to cystic fibrosis. Gut, respiratory and locomotor involvement.Low globulins. Giardia infections common

Zollinger–Ellison syndrome History of recurrent peptic ulceration predominates

Intestinal lymphangiectasia Hypoproteinaemia as a result of protein-losing enteropathy and steatorrhoea. Associated immunoglobulin deficiency. Congenital dilatation of small bowel lymphatics. May occur secondary to pericardial constriction or severe heart failure. Primary lymphatic abnormalities elsewhere may lead to chylous pleuraleffusions and chylous ascites

Question 277

A 30-year-old sailor was admitted to the navy hospitalwith a seven-day history of increasing confusion, malaise,fever and reduced visual acuity in his left eye. Two weekspreviously he had developed weakness affecting his leftarm and left lower limb. He had a past history ofHodgkin’s lymphoma which was successfully treated withchemotherapy five years ago.

On examination, he was confused. His temperaturewas 40°C (104°F). There was evidence of cervicallymphadenopathy on palpation of the neck. Onexamination of the fundi, there was widespread retinalscarring in both eyes, the right eye being more affectedthan the left. There was no obvious papilloedema. Theleft-sided limbs demonstrated marked weakness.

Investigations are shown.

Hb 11 g/dlWCC 6 � 109/lPlatelets 190 � 109/lESR 80 mm/hEEG Diffuse slow wave activity

1. Suggest at least two possible diagnoses.2. Give one investigation which would help make the

diagnosis in each case.3. What treatment would you institute for each of

your diagnoses?

Page 283: Rapid review of clinical medicine for mrcp part 2

282

Answer 277

The patient is a sailor. In MRCP and similarexaminations questions, sailors are associated with a highlevel of promiscuity and high risk for HIV infection. Thispatient is confused, has focal neurological signs, andevidence of bilateral scarring of the retina. The confusionmay be the manifestation of an encephalitis, meningitis,cerebral abscess or cerebral metastases in the context ofthe question.

Focal neurological signs may be caused by all of theabove. The presence of chorioretinitis and focalneurological signs favours the diagnosis of cerebraltoxoplasmosis, TB or cryptococcal meningitis. Cerebrallymphoma per se does not cause chorioretinitis; however,it is possible that the patient has both cerebral lymphomaand a CSF infection causing chorioretinitis. Otherrecognized causes of chorioretinitis in patients with HIVinclude CMV and syphilitic infection. The slow waves onthe EEG are suggestive of an encephalitic process, whichmay be part of the cerebral effect of the HIV itself, or as aresult of opportunistic infections such as CMV, herpesvirus, toxoplasmosis, atypical mycobacteria orlymphomatous infiltration.

The diagnosis and management for each condition areoutlined above. It may be difficult to differentiatebetween cerebral lymphoma and toxoplasmosis in this

particular case. The usual management policy would beto treat for cerebral toxoplasmosis in the first instanceand repeat the CT scan in two weeks, which shouldreveal resolution of the ring-enhancing lesions if they aredue to toxoplasmosis. If the lesions persist, then a tissuediagnosis using brain biopsy may become necessary.

The cerebral manifestations of HIV infection are listed(Table).

Cerebral manifestations of HIV

• HIV dementia• Progressive multifocal leucoencephalopathy• Tuberculous meningitis/tuberculoma• Toxoplasmosis• Lymphoma• Herpes encephalitis• Peripheral neuropathy

1. i. Cerebral toxoplasmosis.ii. Tuberculous meningitis.iii. Cryptococcal meningitis.iv. Cerebral lymphoma.

2. i. CT scan brain (277). Multiple ring-enhancinglesions may suggest cerebral toxoplasmosis, TBabscesses or lymphoma.

ii. CSF IgM levels or Sabin–Feldman dye test for Toxoplasma gondii.

iii. CSF staining with auramine or Ziehl–Neelsen stain may detect TB and staining with Indian ink will identify cryptococcal meningitis.

iv. Brain biopsy.3. i. Pyrimethamine and sulphadiazine for

toxoplasmosis.ii. Ansamycin and clofazimine for TB (particularly

effective for atypical TB).iii. Amphotericin for cryptococcal meningitis.iv. Radiotherapy to the skull for cerebral lymphoma.

227777

Page 284: Rapid review of clinical medicine for mrcp part 2

Clinical Cases 283

A 15-year-old girl presented with a 48-hour history of asore throat and dark urine. On examination she appearedrelatively well at rest. The blood pressure was100/60 mmHg.

Investigations are shown.

Question 278

HB 12.5 g/dlWCC 11 � 109/lPlatelets 240 � 109/lSodium 136 mmol/lPotassium 4.1 mmol/lUrea 4.2 mmol/lSerum creatinine 80 �mol/lUrinalysis Microscopic haematuria

Which of the following antibodies is likely to be raisedin the blood?

a. ASO. b. ANA.c. Anti-GBM.d. ANCA.e. None of the above.

Question 279

A 55-year-old man was seen by a respiratory physician fordaytime somnolence. According to his wife he snoredloudly in his sleep. He was a life-long heavy smoker. He

took salbutamol intermittently for breathlessness. Onexamination he weighed 88 kg and was 1.72 m tall.There was an expiratory wheeze heard throughout hislung fields.

Investigations are shown.

Arterial blood gases:pH 7.35PaCO2 7.0 kPaPaO2 8.2 kPaBicarbonate 35 mmol/lO2 saturation 85%

FEV1 45% predictedFVC 55% predictedFEV1/FVC 50%

What is the most appropriate therapy?a. Nocturnal oxygen therapy.b. Uveoplasty.c. Nocturnal continuous positive-pressure airway

ventilation.d. Steroid therapy.e. Protryptiline.

2 4 6 8Time (h)

100%

80%

60%

O2

sat

(%)

Oxygen saturation during sleep

Page 285: Rapid review of clinical medicine for mrcp part 2

284

Causes of haematuria after an upper respiratory tract infection include IgA nephropathy, post-streptococcal glomerulonephritis and non-specificmesangioproliferative glomerulonephritis. IgAnephropathy is probably the most common cause ofglomerulonephritis after a URTI. Patients present withrecurrent episodes of gross haematuria one to three daysafter a URTI. The haematuria resolves within a few days.Some patients develop acute renal failure. The serum IgAmay be raised in 50% of cases but there are no otherspecific auto-antibodies to help make the diagnosis.

Post-streptococcal glomerulonephritis is induced byinfection with specific strains of group A, ß-haemolyticstreptococci (such as type 12 and type 49). This canoccur in sporadic cases or during an epidemic. Theclinical presentation can vary from asymptomatic,microscopic haematuria to the full-blown acute nephritic

syndrome and renal failure. Although the presentationmay be similar to that seen with IgA nephropathy, thefollowing can be used to differentiate between thesedisorders (Table). Renal biopsy is not required inmost cases.

Answer 278

e. None of the above.

Post Strep IgA GMN nephropathy

Onset of 10 days after 5 days after sore haematuria sore throat throat

Recurrent Rare Commonhaematuria

Throat culture Positive Negative

ASO titre Positive Negative

Haematuria after Rare Common6 months

The patient has features of obstructive sleep apnoea. Hesnores and has daytime somnolence. He also appears tobe overweight. The sleep study shows marked oxygendesaturations throughout the night. These desaturationsrepresent apnoeic episodes due to airway obstruction.The fact that he also has respiratory failure suggests thathe must also have underlying parenchymal lung disease,since pure obstructive sleep apnoea syndrome does notusually cause respiratory failure unless the patient has lowtidal volumes due to gross obesity, as seen in thePickwickian syndrome.

Obstructive sleep apnoea is usually due to anabnormality in the size or collapsibility of the pharynx.During sleep there is a combination of factors (reducedtone input into the upper airways, diminished reflexes to

protect the pharynx from collapse) that cause upperairway obstruction. The resulting apnoea eventuallywakes the patient. Airway obstruction manifests as loudsnoring with apnoeic episodes.

Since sleep is disturbed the patients complain oftiredness and daytime somnolence. Early morningheadaches due to CO2 retention are a recognized feature.The sympathetic response to the apnoeic episodes resultsin systemic hypertension and left ventricular hypertrophy.There is some anecdotal evidence that OSA is associatedwith a higher cardiovascular mortality.

Treatments include weight reduction, avoidance ofalcohol or sedatives. Protryptil ine (a respiratorystimulant) is not particularly effective. The most effectivetherapy is nocturnal continuous positive-pressureventilation, which keeps the upper airways patent.Reduced tolerance and compliance with nocturnal CPAPare the main issues.

Answer 279

c. Nocturnal continuous positive-pressure airwayventilation.

Page 286: Rapid review of clinical medicine for mrcp part 2

Clinical Cases 285

A 34-year-old Nigerian male presented with a 48-hourhistory of fever and headache. According to his wife hebecame confused two hours prior to admission and wastalking nonsense. The patient underwent a CT scan ofthe brain (280).

Question 280

What is the management?a. IV cefotaxime.b. Neurosurgical referral.c. IV acyclovir.d. IV amphotericin.e. IV dexamethasone.

Question 281

A 62-year-old asymptomatic female had the followingthyroid function tests as part of a health screen. She wasnot taking any medication and had always enjoyed goodhealth:

Serum T4 20 iu/lSerum TSH 8 iu/lAntithyroid myoperoxidase antibodies: absent

What is the management of her abnormal thyroidfunction tests?

a. Start thyroxine.b. No treatment. Repeat thyroid function tests in

six months.c. Reassure. No further follow up required.d. Perform a radio-iodine scan and treat only if

there is reduced uptake in the gland.e. Start triiodothyronine.

A 64-year-old male presented with reduced visual acuity.He had a long history of hypertension.

Question 282

What is the ophthalmological diagnosis (252)?a. Central retinal vein occlusion.b. Central retinal artery occlusion.c. Optic atrophy.d. Myelinated nerve fibre.e. Papilloedema.

228800

228822

Page 287: Rapid review of clinical medicine for mrcp part 2

286

Herpes simplex encephalitis is most usually due to HSV 1and presents with headache, fever and drowsiness. Thevirus may complicate HSV infecting the oropharyngealarea or may be due to reactivation of latent virus in theCNS. It affects the inferior aspects of the frontal lobes andthe medial aspects of the temporal lobes. Olfactory andgustatory hallucinations and impairment of memory are

recognized. The patient may exhibit multiple cranial nervepalsies and ataxia. Some patients develop hypomania or theKluver–Bucy syndrome (‘psychic blindness’, loss of fearand anger response and increased sexual activity).Untreated patients develop seizure and lapse into acomatosed state. The mortality is high. CT scan may revealhypodense medial temporal lobes that enhance withcontrast. The diagnostic investigation of choice is CSFanalysis for PCR for HSV. Treatment comprises earlytreatment with IV acyclovir for 14–21 days in total.

Answer 280

c. IV acyclovir.

The patient has very mild subclinical hypothyroidism. Shedoes not have raised antithyroid microsomal peroxidaseantibiodies to indicate chronic auto-immune thyroiditis.Only a small proportion of such patients progress toovert hypothyroidism per year. Treatment of all patientsin this group has cost implications and commits patientsto lifelong therapy early; therefore, the currentrecommendation is to observe and perform thyroidfunction tests every six months.

In asymptomatic patients, the presence of antithyroidperoxidase antibodies is an indication to treat sinceapproximately 5% of patients with antithyroid peroxidaseantibodies progress to overt hypothyroidism each year. Inasymptomatic patients without circulating antithyroidperoxidase antibodies, treatment is recommended oncethe TSH reaches 15 iu/l (algorithm). Treatment ofhypothyroidism at this level improves lipid profile andblood pressure and may have an impact on reducingcardiovascular morbidity from hypothyroidism.

Patients with clinical (symptomatic) hypothyroidismshould be treated irrespective of the TSH concentration.

Answer 281

b. No treatment. Repeat thyroid function tests insix months.

Indications for treating hypothyroidism

Symptoms

Yes No

Treat

Yes No

Treat Thyroid MPOAntibodies

Yes No

Treat Observe

TSH >14

The patient had blurred disc margins with a fewhaemorrhages around the disc. The diagnosis ispapilloedema, which is a feature of raised intracranialpressure. Causes of papilloedema are tabulated.

Answer 282

e. Papilloedema. Causes of papilloedema

• Malignant or accelerated hypertension• Benign intracranial hypertension• Intracranial mass lesions• Intracerebral haemorrhage• Meningitis, encephalitis• Cavernous sinus thrombosis• Severe hypoxia• Severe hypercapnia• Severe anaemia• Lead poisoning

Page 288: Rapid review of clinical medicine for mrcp part 2

Clinical Cases 287

A 62-year-old female with a two-year history of non-insulin-dependent diabetes mellitus was admitted withnausea and vomiting. Her diabetes was usually wellcontrolled on a combination of chlorpropamide 500 mgand metformin 500 mg bd. One week previously she wasseen by her GP with general malaise and noted to have aBM stix reading of 19 mmol/l, and the dose ofmetformin was doubled.

On examination she was drowsy (Glasgow coma score10) but clinically well perfused. The heart rate was120 beats/min and regular. The pulse was bounding.The blood pressure was 100/60 mmHg. Thetemperature was 36.8°C (98.2°F). All other physicalexamination was normal.

Investigations are shown.

Question 283

Hb 15.2 g/dlWCC 16.4 � 109/lPlatelets 284 � 109/lSodium 134 mmol/lPotassium 5.0 mmol/lUrea 30.5 mmol/lCreatinine 160 �mol/lChloride 106 mmol/lBlood sugar 9 mmol/Arterial blood gases:

pH 7.3PaCO2 2.6 kPaPaO2 9.6 kPaHCO3 11 mmol/l

Urinalysis: Protein + 1 Blood – negativeGlucose + 1 Ketones – negative

What is the diagnosis?a. Renal tubular acidosis.b. Diabetic ketoacidosis.c. Lactic acidosis secondary to sepsis.d. Hyperosmolar non-ketotic diabetic

coma.e. Acute pancreatitis.

A 32-year-old woman had had persistently raised BPmeasurements over the past six months. Her systolic bloodpressure readings had ranged from 158 to 170 mmHg andthe diastolic blood pressure readings had ranged between90 and 98 mmHg. She was asympto matic. She was not onany medications (including the oral contraceptive pill).There was no past medical history of note or any familyhistory of hypertension. Physical examination did not revealany abnormalities other than a BP reading of160/96 mmHg. Blood urea and electrolytes were normal.

Question 285

What is the most probable cause for the raised bloodpressure?

a. Essential hypertension.b. Primary hyperaldosteronism.c. Cushing’s syndrome.d. Renal artery stenosis.e. Adult polycystic kidney disease.

A 39-year-old woman with depression presented withnausea, vomiting, tremor and convulsions.

Investigations are shown.

Question 284

Sodium 149 mmol/lPotassium 3.9 mmol/lUrea 18 mmol/lTSH 14 mu/l

What is the diagnosis?a. Lithium toxicity.b. Hypothyroidism.c. Chronic renal failure.d. Tricyclic antidepressant overdose.e. Hyperthyroidism.

Page 289: Rapid review of clinical medicine for mrcp part 2

The patient has features of septicaemia and a metabolicacidosis. Overproduction of lactic acid secondary todiminished organ perfusion as a result of hypotension isthe most probable cause of the acidosis. Lactic acidosisusually occurs in states where there is increased pyruvateproduction due to excessive anaerobic metabolism, forexample in shock states, excessive exercise, grand malseizures, cyanide toxicity, and carbon monoxidepoisoning. Renal failure and liver disease cause lacticacidosis primarily owing to decreased utilization of thesubstrate. Biguanide drugs and malignancy cause lacticacidosis but the mechanisms by which acidosis occurs areunclear (Table).

The diagnosis can be confirmed by measuring theplasma lactate concentration. Concurrent treatment withmetformin is probably contributing to the lactic acidosis.

Metformin causes lactic acidosis in conditions associatedwith shock or in the presence of coexisting renal or liverfailure.

The most important step is to stop metformin and torestore circulation with intravenous saline. The efficacy ofand indications for alkali administration in hypoperfusion-induced lactic acidosis remain unresolved.

288

Answer 283

c. Lactic acidosis secondary to sepsis.

Causes of lactic acidosis

• Shock (hypovolaemic or septic)• Severe hypoxia• Renal failure• Liver failure• Drugs – phenformin, metformin• Leukaemia• Total peripheral nutrition• Glucose-6-phosphate dehydrogenase deficiency

The patient is relatively young but does not havesymptoms to suggest a secondary cause such asheadaches, hyperhidrosis, palpitation or nocturia. She hasnot been described to be cushingoid and the absence of

hypokalaemia makes a primary hyperaldosteronism orCushing’s disease unlikely. Adult polycystic kidney rarelypresents with hypertension before the fifth decade, andrenal artery stenosis is uncommon in young females.Essential hypertension accounts for over 90% of all casesof hypertension even in this particular age group.

Answer 285

a. Essential hypertension.

Answer 284

The patient has a history of depression therefore it ispossible that her presentation is secondary to drugoverdose. Overdose with neuroleptic agents, tricyclicantidepressant drugs or lithium may present with areduced conscious level and convulsions. However, thebiochemistry in this case is highly suggestive of lithiumoverdose. The raised serum sodium and urea indicatesevere water loss, as may occur in diabetes insipidus.Lithium is a recognized cause of nephrogenic diabetesinsipidus. The patient also has biochemical evidence ofhypothyroidism. Chronic use also interferes with iodineuptake in the thyroid and may result in hypothyroidism.

Lithium has a narrow therapeutic–toxic range. While aplasma level of between 0.6 and 1.2 mmol/l istherapeutic, a plasma level of 2.5 mmol/l or more is toxic.Toxicity is precipitated by conditions causing sodiumdepletion or a reduction in glomerular filtration rate,therefore diarrhoea, vomiting, diuretics, ACE inhibitorsand NSAIDs may all cause Li toxicity. Features of toxicityinclude tremor, cerebellar signs, drowsiness, convulsions,hyperpyrexia, nephrogenic diabetes insipidus, longQT/ventricular arrhythmias and hyperpyrexia.

Treatment involves fluid repletion and forced diuresis.Haemodialysis is necessary when plasma Li levels exceed4 mmol/l in patients with normal renal function but theindications for dialysis is much lower (2.5 mmol/l) inpatients with known renal insufficiency.

a. Lithium toxicity.

Page 290: Rapid review of clinical medicine for mrcp part 2

Clinical Cases 289

Question 287

A 75-year-old woman was admitted with an anteriormyocardial infarction for which she receivedthrombolysis. Within 6 hours of admission she becameconfused. The blood pressure was 80/36 mmHg. TheJVP was raised. Both heart sounds were normal and therewas an added third heart sound; there were no audiblemurmurs. Auscultation of the chest revealed fine cracklesin the mid- and lower-zones in both lungs.

Subsequent investigations were as follows:

Hb 12 g/dlWCC 15 � 109/lPlatelets 200 � 109/lSodium 136 mmol/lPotassium 3.9 mmol/lUrea 17 mmol/lCreatinine 140 �mol/lGlucose 16 mmol/lChest X-ray Bilateral alveolar shadows and small bilateral pleural effusionsECG Sinus tachycardia; ST elevation and q waves in V1–V6Echocardiography Dilated left ventricle with akinetic anterior wall and apex. No evidence of VSD.

Functional mitral regurgitation. No pericardial effusion

Which of following therapeutic options is most likelyto improve outcome?

a. Further thrombolytic therapy.b. IV noradrenaline.c. NIPPV.d. Intra-aortic balloon counterpulsation.e. Early invasive coronary revascularization.

A 29-year-old male with Marfan’s syndrome had aprolonged stay in hospital following a subarachnoidhaemorrhage, which rendered him with a residual lefthemiparesis. A right posterior communicating arteryaneurysm was clipped. On discharge the patient hadnormal speech and was able to carry out basic domestictasks. During his hospital stay he had been maderedundant from his job as a postman. Followingdischarge he had been placed in a warden-controlled flat.His friends had seen very little of him for the past month.

His GP had treated him for reactive depression. He wasadmitted after being found collapsed by the warden oneevening.

On arrival at hospital he was deeply comatosed andhad a Glasgow coma scale of 5. The heart rate was 130beats/min and regular. The blood pressure was110/70 mmHg. The respiratory rate was 14/min andtemperature was 36.8°C (98.2°F).

Both pupils were 9 mm in diameter and reactedsluggishly to light. All reflexes were brisk and bothplantars were upgoing. Examination of his fundidemonstrated temporal pallor of his optic discs. Therewere no retinal haemorrhages. Heart sounds were normaland his chest was clear. Abdominal examination revealeddullness over the suprapubic area. During his admissionhe suffered two epileptic seizures, which were short lived.

Routine investigations were as follows:

Question 286

Hb 14 g/dlWCC 12 � 109/lPlatelets 175 � 109/lSodium 136 mmol/lPotassium 4.6 mmol/lUrea 6.0 mmol/lCreatinine 107 �mol/lHCO3 18 mmol/lGlucose 6.1 mmol/lECG Sinus tachycardia;

QRS duration 110 msecCXR Cardiac size normal.

Normal lung fieldsCT scan brain Old left-sided infarction

What is the most common cause of mortality in thissituation?a. Overwhelming septicaemia.b. Cardiac tamponade.c. Status epilepticus.d. Ventricular arrhythmias.e. Cerebral oedema.

Page 291: Rapid review of clinical medicine for mrcp part 2

290

The patient has hypotension, oliguria, raised JVP andpulmonary oedema following an extensive anteriormyocardial infarction. Clinically she has evidence ofcardiogenic shock, which is associated with a poorprognosis (50–80% mortality within 48 hours). In thecontext of the clinical circumstances, the differentialdiagnosis is between severe left ventricular dysfunctiondue to loss of a large amount of myocardium, acuteVSD, papillary muscle rupture causing severe mitralregurgitation or cardiac tamponade. The patient doesnot have any obvious murmurs but the echocardiogramproves useful in resolving the cause of cardiogenicshock. Echocardiography is an important investigativetool in a patient with cardiogenic shock and should beemployed whenever possible.

There is no evidence that further thrombolytictherapy would be useful. Insulin is recommended inpatients with a blood sugar >11 mmol/l aftermyocardial infarction, but insulin is unlikely to influencethe immediate prognosis in this particular situation,where cardiac embarrassment is the main problem.NIPPV is contraindicated in cardiogenic shock becauseit may cause a further drop in blood pressure.Noradrenaline and other inotropes are transiently usefulin improving cardiac contractility and blood pressure buthave not been shown to improve outcome. Similarlyballoon counterpulsation may be transiently effective atresting the left ventricle until invasive revascularization iscarried out, but has not been shown to reduce mortalityon its own. Indeed the only treatment that has beenshown to improve prognosis in randomized controlledtrials is early invasive revascularization. Earlyrevascularization is key to improving prognosis in apatient with cardiogenic shock.

Answer 287

e. Early invasive coronary revascularization.

The patient has features consistent with the diagnosis oftr icycl ic antidepressant overdose. Ventriculararrhythmias are the most common cause of death inTCAD overdose.

Intravenous sodium bicarbonate is the single mosteffective intervention for the management of TCADcardiovascular toxicity. This agent can reverse QRSprolongation, ventricular arrhythmias and hypotension.Because acidosis aggravates TCAD toxicity, thebeneficial action of sodium bicarbonate may be partlydue to correction of acidosis. It is clear, however, thatsodium bicarbonate administration is effective evenwhen the arterial pH is normal.

This beneficial effect appears to be mediated byincreases in both pH and the plasma sodiumconcentration. Alkalinization to an arterial pH of 7.5,for example, appears to reduce the incidence of cardiacarrhythmias, and intravenous sodium bicarbonate (in adose of 1–2 mEq/kg) is the treatment of choice forsudden-onset ventricular tachycardia, ventricularf ibri l lat ion or cardiac arrest. Class Ia and Icantiarrhythmic agents should be avoided as they blocksodium ion channels and may increase the prevalence ofventricular arrhythmias.

To maintain an arterial pH of 7.5, an intravenousinfusion of two 50 ml ampoules of sodium bicarbonate(containing approximately 90 mEq of sodium bicar b-onate) in 1 litre of 5% dextrose in water is started in allcomatosed patients fol lowing TCAD poisoning,particularly those with a QRS duration above 0.10 sec(100 msec).

(See Answers 28,209,366.)

Answer 286

d. Ventricular arrhythmias.

Page 292: Rapid review of clinical medicine for mrcp part 2

Clinical Cases 291

Question 289

A patient was brought into the Accident and EmergencyDepartment in ventricular fibrillation. After prolongedresuscitation he was admitted to the Intensive Care Unitand developed oliguric renal failure. Serum and urineelectrolytes were as follows:

Serum electrolytes Sodium 132 mmol/lPotassium 5 mmol/lUrea 30 mmol/lCreatinine 400 �mol/lOsmolality 320 mOsmol

Urine electrolytes Sodium 90 mmol/lPotassium 30 mmol/lUrea 120 mmol/lCreatinine 7 �mol/lOsmolality 300 mOsmol

Which two comments below suggest that the patienthas developed acute tubular necrosis?

a. Increased urine urea:serum urea.b. Increased urine creatinine:serum creatinine.c. Increased urine sodium:serum sodium.d. Post-cardiac arrest renal failure.e. Increased urinary sodium excretion.f. Relatively low urine osmolality.g. Urine osmolality:plasma osmolality 1:1.1.h. Increased urine potassium.i. Reduced urine potassium.j. Increased serum creatinine.

Question 288

A 17-year-old male presented with blurring of visionfollowed by total loss of vision in the left eye. A few dayslater he developed total blindness in the other eye. Therewere no other neurological symptoms. There was no pastmedical history of significance or any history of alcoholabuse. The patient was not taking any medications. Hehad a maternal uncle who had become blind before theage of 18 years. On examination there was evidence ofbilateral optic atrophy and a mild tremor.

Investigations are shown.

FBC NormalU&E NormalLiver function NormalVitamin B screen NormalSerum lead Not elevatedCT scan brain NormalCSF analysis Normal

What is the diagnosis?a. Multiple sclerosis.b. Ischaemic optic atrophy.c. Toxic optic neuritis.d. Leber’s hereditary optic neuropathy.e. Cerebellar syndrome.

A 70-year-old patient was admitted in a drowsy state afterbeing found collapsed by his neighbour. On admissionthe Glasgow coma score was 9. A CT scan (290) of thebrain was performed as an emergency.

Question 290

What is the diagnosis?a. Subarachnoid haemorrhage.b. Intracerebral haematoma.c. Epidural haematoma.d. Subdural haematoma.e. Herpes simplex encephalitis.

229900

Page 293: Rapid review of clinical medicine for mrcp part 2

Acute tubular necrosis is caused by renal ischaemia andmay be precipitated by a variety of insults includinghypovolaemia and septicaemia (causing reduced renal

perfusion), drugs and rhabdomyolysis. It is usuallycharacterized by oliguric renal failure that is oftenreversible. Ischaemia typically affects the medullary area,therefore (unlike pre-renal failure), the concentratingability of the urine is diminished and sodium excretion isinappropriately increased (Table).

292

Answer 289

e. Increased urinary sodium excretion.g. Urine osmolality:plasma osmolality 1:1.1.

Pre-renal failure versus ATN

Measure Pre-renal failure ATNUrine output Low LowUrine osmolality >500 <350U/P osmolality 1.5:1 1.1:1Ucreat/Pcreat >20:1 <20:1Urine sodium <20 mmol/l >50 mmol/l

This young male presents with bilateral blindness in arelatively short space of time. There are no obviousacquired precipitating factors such as drugs or toxins.The patient does not have diabetes and has normal B12levels. He does not have any symptoms or otherindications to suggest an ischaemic optic neuropathy.Optic neuropathy is a common feature of multiplesclerosis but it is usually unilateral. The presence ofbilateral optic neuropathy should raise the possibility of atoxic neuropathy or the rarer Leber’s hereditary opticneuropathy.

The family history of premature blindness in amaternal uncle indicates a hereditary aetiology andprovides a vital clue to the underlying diagnosis. Leber’shereditary optic neuropathy is an inherited disorder ofmitochondrial DNA encoding the electron transport

chain that is characterized by bilateral subacute opticneuropathy. Three specific mutations – at positions3460, 11778 and 14484 – account for over 90% of allrecognized mutations responsible for the disorder. Allmale offspring of affected females are affected(mitochondrial gene inheritance pattern).

LHON produces severe and permanent visual loss inyoung males. The time interval between affected eyesmay be very short. Tremor and a multiple sclerosis typeillness are recognized features. Young children may alsohave extrapyramidal symptoms, and epilepsy, ataxia,spasticity, mental retardation and peripheral neuropathymay also be present.

The diagnosis is made on muscle biopsy, which revealsabnormal mitochondria on electron microscopy withdefective respiratory enzymes within the mitochondria.The demonstration of point mutations on geneticanalysis of mitochondrial DNA using white blood cells isdiagnostic.

Answer 288

d. Leber’s hereditary optic neuropathy.

There is a collection of fresh blood in the right subduralspace (bright white) with significant mass effect. Thetreatment involves neurosurgical intervention.

Answer 290

d. Subdural haematoma.

Page 294: Rapid review of clinical medicine for mrcp part 2

Clinical Cases 293

A 68-year-old male was brought into the Accident andEmergency Department with severe chest pain and anECG consistent with a recent anterior myocardialinfarction. According to his wife he had beenexperiencing pain for three days but over the previousthree hours the pain had become very intense. He wasdiabetic. On arrival at hospital he received IVtenectoplase. Two hours after thrombolysis he droppedhis blood pressure and became very breathless. Onexamination the heart rate was 120 beats/min andregular. The blood pressure was 80 mmHg systolic.Auscultation over the precordium revealed a pansystolic

murmur. Auscultation of the lungs revealed widespreadinspiratory crackles.

Question 291

What is the immediate investigation of choice toascertain the cause of the deterioration?

a. Transoesophageal echocardiography.b. Transthoracic echocardiogram.c. Swann–Ganz right heart catheter.d. Chest X-ray.e. 12-lead ECG.

Question 292

A 44-year-old male presented to his GP with a two-dayhistory of severe boring intermittent pain just below thesupraorbital ridge of the right eye. The pain had awokenthe patient from his sleep in the early hours of themorning and lasted 3 hours before subsiding. The painrecurred again in the afternoon and persisted for 4 hoursdespite the patient having taken paracetamol. In 48hours he had experienced five episodes of similar pain.The patient had also noticed that he had a blocked noseand therefore made a self-diagnosis of sinusitis. Herecalled having two similar episodes of pain two and fourmonths previously that he attributed to sinusitis andtreated himself with steam inhalations for four days. Onexamination the patient was distressed with pain. Hisright eye was red and watering. He had a partial ptosisand miosis affecting the right eye. His blood pressure was160/90 mmHg. All other physical examination wasnormal.

1. What is the most probable diagnosis?a. Tension headache.b. Hemiplegic migraine.c. Migrainous neuralgia.d. Ophthalmoplegic migraine.e. Posterior communicating artery aneurysm.

2. Choose two therapeutic steps from the followinglist for treating his symptom rapidly:

a. IV verapamil.b. Ergotamine.c. Atenolol.d. IM sumatriptan.e. High-concentration inhaled oxygen.f. High-dose oral augmentin.g. Oral gabapentin.h. Carbamazepine.i. Lithium.j. Oral verapamil.

Question 293

A 17-year-old girl with insulin-dependent diabetesmellitus was investigated for weight loss andamenorrhoea.

Investigations are shown.

Hb 10 g/dlWCC 5 � 109/lPlatelets 200 � 109/lESR 12 mm/hSodium 135 mmol/lPotassium 3.6 mmol/lUrea 2 mmol/lAST 21 iu/lAlkaline phosphatase 160 iu/lAlbumin 37 g/lCalcium 2.0 mmo/lPhosphate 0.7 mmol/Glucose 8 mmol/lHbA1c 7.2%

What is the most probable cause for the weight loss?a. Poorly controlled diabetes mellitus.b. Anorexia nervosa.c. Coeliac disease.d. Crohn’s disease.e. Addison’s disease.

Page 295: Rapid review of clinical medicine for mrcp part 2

294

The patient has developed severe pulmonary oedema aftera myocardial infarction. He also has a pansystolic murmur,indicating that he may have mitral regurgitation due topapillary muscle rupture or a ventricular septal defect.

Echocardiography would be the most practicalmethod of differentiating between the possibilities. Asthe patient is so unstable, a transthoracic echocardiogramis preferable to transoesophageal echocardiogram. Ingeneral, transthoracic echo is better at visualizing a VSDthan transoesophageal echocardiogram; however, thelatter is superior in the diagnosis of papillary musclerupture. The indications for transoesophagel echo -cardiography in routine clinical practice are listed in thetable below. Swan–Ganz right heart catheterization may

also help but it is invasive and requires the patient to liesupine. Mitral regurgitation is diagnosed bydemonstrating a high PCWP with large v-waves. VSD is diagnosed by demonstrating a left-to-right shunt at the level of the ventricles (step up inoxygen saturation in the right ventricle).

Answer 291

b. Transthoracic echocardiogram.

Indications for transoesophagealechocardiography

• Patients who have poor transthoracic echowindows

• Identification of thrombi in the left atrialappendage in atrial fibrillation

• Assessment of prosthetic valve endocarditis• Assessment of complications of native and

prosthetic valve endocarditis• Diagnosis of aortic dissection• Evaluation of atrial septal defect

The features are typical of migrainous neuralgia (Table).High-concentration inhaled oxygen (10–12 l/min) andthe triptan drugs are very effective at aborting an acuteattack. Verapamil is the drug of choice at preventingfrequent recurrence, although lithium, methysergide,sodium valproate, ergotamine and prednisolone are alluseful therapies in this respect. Methysergide is no longerused owing to its association with retroperitonealfibrosis.

Answer 292

1. c. Migrainous neuralgia.2. d. IM sumatriptan.

e. High-concentration inhaled oxygen.

Characteristic features of migrainous neuralgia

• Frequency 0.1%; M:F ratio 3:1• Unilateral pain, often around the ocular orbit;

severe• Periodic, usually every 1–3 months; 1–2 attacks

daily for 8–10 weeks per year• Pain lasts between 15 min and 3 hours at a time• Patient well inbetween bouts of pain• Associated unilateral watering eye, miosis, ptosis,

nasal congestion or rhinorrhoea• Acute attacks are best treated by 100% oxygen

inhalation at 10–12 l/min for 15 min • Sumatriptan is very useful in some patients but

not effective above 6 mg• Preventative therapy includes verapamil in high

doses (up to 720 mg per day)

The commonest cause of weight loss in a patient withinsulin-dependent diabetes is poor glycaemic control.The HbA1c of 7.2% is not suggestive of recent poorcontrol. Furthermore the calcium level is at the lowerlimit of normal, the phosphate is low and the alkaline

phosphatase is high, indicating biochemical osteomalacia.The haemoglobin is also low. Low haemoglobin and lowserum calcium should raise the suspicion ofmalabsorption in the absence of renal failure. The mostlikely diagnosis is coeliac disease. Coeliac disease isassociated with type 1 diabetes mellitus, IgA deficiency,inflammatory bowel disease, hyperthyroidism, fibrosingalveolitis and primary biliary cirrhosis.

Answer 293

c. Coeliac disease.

Page 296: Rapid review of clinical medicine for mrcp part 2

Clinical Cases 295

A 62-year-old male presented with acute dyspnoea. Hehad clinical and radiological evidence of pulmonaryoedema that responded well to intravenous nitrates and

diuretics. He was discharged on furosemide 40 mg odand ramipril 5 mg od. Since discharge he had beenrelatively well. He could walk unlimited distances on theflat and could negotiate inclines without dyspnoea. Theheart rate was 80 beats/min. The pulse rate was regular.The blood pressure measured 105/70 mmHg. Bothheart sounds were normal and the chest was clear.

Investigations were as follows:

Question 296

Hb 13 g/dlWCC 8 � 109/lPlatelets 200 � 109/lSodium 138 mmol/lPotassium 4.2 mmol/lUrea 6 mmol/lCreatinine 100 �mol/lEchocardiography Dilated left ventricle

with poor left ventricular function

Coronary angiography Normal coronary arteries

Which drug would you initiate next?a. Digoxin.b. Spironolactone.c. Atenolol.d. Bisoprolol.e. Candesartan.

A 35-year-old patient with type 1 diabetes presented threemonths following a renal transplant with fever, nightsweats and malaise. He had experienced two episodes oftransplant rejection, which were successfully reversed withcorticosteriods. He was currently maintained onprednisolone and ciclosporin. On examination he had atemperature of 39°C (102.2°F). The heart rate was120 beats/min and blood pressure was 110/65 mmHg.The respiratory rate was 40/min. Precordial examinationrevealed a soft systolic murmur at the apex. The chest wasclear. The abdomen including the graft was non-tender.

Investigations are shown.

Question 294

Hb 10 g/dlWCC 3.5 � 109/lNeutrophils 1.1 � 109/lLymphocytes 2.4 � 109/lEosinophils 0.03 � 109/lBasophils 0 � 109/lMonocytes 0.1 � 109/lPlatelets 140 � 109/lESR 100 mm/hSodium 130 mmol/lPotassium 5.5 mmol/lChloride 87 mmol/lBicarbonate 24 mmol/lUrea 9 mmol/lCreatinine 133 �mol/lAST 60 iu/lALT 90 iu/lAlkaline phosphatase 110 iu/lGlucose 6 mmol/lUrine output 45 ml/hUrinary microscopy NADECG Sinus tachycardia

What is the most likely diagnosis?a. Acute renal transplant rejection.b. CMV infection.c. Endocarditis.d. Gram-negative sepsis.

A 36-year-old nurse with two children, aged 8 and 14 yearsof age, went to see her GP, concerned that her maternalaunt had recently had a pulmonary embolus and washeterozygous for the Factor V Leiden mutation. The patientwas taking the combined oral contraceptive pill and wasadamant that she did not want any more children. Her ownmother had never had thrombotic tendencies and hadnormal pregnancies except for varicose veins during hersecond pregnancy. The patient herself had never experiencedany thrombotic episodes and was a non-smoker.

Question 295

What is the best advice for the patient?a. Finish the current monthly packet and stop taking

the contraceptive pill.b. Referral to a gynaecologist to discuss sterilization.c. Suggest alternative methods of contraception.d. Risk of thrombotic tendencies low; therefore

reassure.e. Screen her blood for Factor V Leiden mutation.

Stop pill if positive.

Page 297: Rapid review of clinical medicine for mrcp part 2

296

The neutropenia and mild hepatitis in a patient a fewmonths after renal transplantation favour CMV infection.The raised creatinine level may reflect dehydration; ifacute, however, concomitant allograft rejection (a featureof CMV infection) and ciclosporin toxicity could alsoexplain the raised creatinine level.

Infections are the leading cause of morbidity andmortality in the early post-transplant period, as more than80% of recipients suffer at least one episode of infectionin the first year. Infection and allograft dysfunctioncaused by rejection are closely interrelated through theuse of immunosuppressive therapy (see below).

CMV is the most important infection in renaltransplant recipients. More than two-thirds of donors andrecipients have detectable IgG anti-CMV antibodies inthe plasma prior to transplantation. It is thereforecommon for the donor and/or recipient to be CMV-positive at the time of transplantation. The virus can betransmitted from the donor either by blood transfusionor by the transplanted kidney; the concurrentadministration of immunosuppressive drugs to preventrejection further increases the risk of clinically relevantCMV disease.

Symptomatic CMV infections typically occur one to fourmonths after transplantation, but can develop later.Typically, the onset of disease usually follows the period ofmaximal immunosuppression (associated with neutropenia)for the prevention and treatment of acute rejection. Clinicalfeatures include fever, deteriorating renal function and atender graft. However, the latter feature is rarely present asit is masked by immuno suppressive therapy. CMV infectionin transplant patients may affect other organs causinghepatitis, pneumonitis, pancreatitis, gastroenteritis,meningo-encephalitis and rarely myocarditis.

Patients with organ involvement benefit from a 2–3week course of ganciclovir. Since the immune system maybe directly suppressed because of CMV infection,modification of the immunosuppressive regimen is alsorequired.

Marked leucopenia is a cardinal feature of CMVdisease among patients treated with azathioprine ormonoclonal antilymphocyte preparations. Increasing useof mycophenolate instead of azathioprine and effectiveprophylactic antiviral therapy in most patients means thatfever and leucopenia may not be as prominent a featureof CMV disease as it used to be.

Infections in post-transplant patientsMonth 1The usual post-operative surgical infections, similar tothose seen in non-immunosuppressed patientsundergoing similar surgical procedures, are mostcommon, e.g. wound infections, line sepsis, urinary tractinfection and pneumonia.Months 2–6Infections with immunomodulating viruses, particularlyCMV, are most important. In addition to the clinicalsyndromes induced by these viruses, theirimmunomodulating properties predispose to opportunisticinfections with organisms such as Pneumocystis carinii,Listeria monocytogenes and Aspergillus fumigatus. Otherinfections commonly occurring during this period includehepatitis, herpes simplex, herpes zoster, Mycobacteriumtuberculosis and Epstein–Barr virus, which can becomplicated by the development of lymphoproliferativedisorders. Recurrence or relapse of UTI can also occur.Month 6 onwardsCommunity-acquired infections, for example, influenza,bacterial infections (pneumococcus) and recurrence ofinadequately treated opportunistic infections.

Answer 294

b. CMV infection.

The risk of venous thromboembolism is approximately3–4 times higher in patients who are heterozygous forthe Factor V Leiden mutation. The risk is increased to30–40 fold in women who are smokers or on thecontraceptive pill. Even though population screening forFactor V Leiden mutation is not cost effective, since onlya small proportion of patients with it will suffer from

systemic thromboembolism, the patient cannot beadvised accurately without screening for this mutation(screening for Factor V Leiden mutation is reserved onlyfor patients with multiple family members with a historyof venous thromboembolism). One could argue thateven if she was heterozygous for the Factor V Leidenmutation the risk of venous thromboembolism is lowerthan the risk of an unwanted pregnancy. However, thepatient is worried about her risk of venousthromboembolism, and estimation of that risk is notpossible without knowing her Factor V Leiden status.

Answer 295

e. Screen her blood for Factor V Leiden mutation. Stop pill if positive.

Page 298: Rapid review of clinical medicine for mrcp part 2

Clinical Cases 297

Table A Therapeutic modalities in the chronic management of dilated cardiomyopathy

Lifestyle modification Restrict salt consumptionRestrict fluid intake to 1.5 litres per day Aerobic exercise for 30 min three times per weekReduce alcohol consumption to a minimum

Pharmacological therapy Loop diuretics and digoxin to treat pulmonary and peripheral congestionACE inhibitors, beta-blockers, AIIRBs and spironolactone to improve

symptoms and prognosisAmiodarone to treat ventricular tachycardia

Device therapy ICD to prevent arrhythmogenic deathBiventricular pacing to improve symptoms

Surgery Coronary revascularization, valve surgery to improve cardiac functionCardiac transplantation

The patient has a dilated, poorly contracting left ventricleand therefore has dilated cardiomyopathy. There are noobvious reversible causes in this case such as B12deficiency, anaemia, haemochromatosis, thyroiddysfunction or coronary artery disease. The treatment ofdilated cardiomyopathy aims to improve symptoms andprognosis.

The low cardiac output resulting from dilatedcardiomyopathy offsets the renin–angiotensin–aldosterone system and the sympathetic nervous systemin an attempt to restore circulation. Unfortunatelyactivation of both systems has a detrimental effect on theheart, causing volume overload, progressive deteriorationof cardiac function owing to adverse remodelling, cardiacarrhythmias and sudden death.

The goals in the management of heart failure are toimprove symptoms and prognosis. Therapeuticmodalities in chronic heart failure include lifestylemodification, pharmacotherapy, device therapy andsurgery (Table A).

Loop diuretics are very effective at improving symp -toms of pulmonary and peripheral congestion rapidly butdo not have an impact on prognosis. Digoxin isparticularly effective at providing symptomatic relief inpatients with atrial fibrillation, but has also been shownto be effective in patients in sinus rhythm through itsweakly inotropic effects. Interestingly it is the onlyinotropic agent used in heart failure that has not beenshown to cause sudden death. Like loop diuretics,digoxin does not have an impact on prognosis.

The prognosis of dilated cardiomyopathy has beenimproved significantly by the use of pharmacologicalagents that inhibit the renin–angiotensin–aldosteronesystem and beta-blockers. ACE inhibitors improve

symptoms in patients with left ventricular systolicdysfunction, irrespective of their functional capacity, andshould be used in all patients with heart failure providedthere are no contraindications and they are tolerated.Impaired baseline renal function is not a contraindicationto initiating ACE inhibitor therapy although cautionneeds to be exercised in patients with a serum creatinineof >265 mmol/l. An increase in serum creatinine of morethan 25% after starting an ACE inhibitor may be anindication that the patient has underlying reno-vasculardisease, which should be excluded before continuingACE inhibitor therapy. ACE inhibitor therapy may beinitiated in patients with a systolic blood pressure readingas low as 90 mmHg.

AIIRBs have also been shown to provide symptomaticand prognostic benefit. While one major trial (theCHARM study) has shown that AIIRBs provide additiveprognostic benefit when they are used in patients alreadytaking ACE inhibitors, the use of AIIRBs is currentlyreserved for patients who develop side-effects on ACEinhibitors. The AIIRBs used in heart failure are losartan,candesartan and valsartan.

Spironolactone, used in relatively small doses, isprognostically beneficial when used in patients who are inNYHA functional class III or IV despite taking ACEinhibitors, AIIRBs or beta-blockers. In this particularquestion the patient is in NYHA functional class I, i.e. heis not functionally limited when performing usual dailytasks, therefore spironolactone is not indicated.

Certain β1 receptor specific beta-blockers have beenshown to improve symptoms and prognosis in patientswith systolic left ventricular dysfunction. Beta-blockerscurrently licensed for use in heart failure includecarvedilol, bisoprolol, metoprolol and more recentlynebivolol. Atenolol has not been evaluated andacebutalol has not been shown to improve symptoms orprognosis in heart failure patients. Beta-blockers improve

Answer 296

d. Bisoprolol.

(Continued over page)

Page 299: Rapid review of clinical medicine for mrcp part 2

298

Management algorithm for chronic heart failure

Loop diuretic to improvepulmonary congestion

plusACE inhibitor

NYHA class I–IIStart beta-blocker

NYHA class III–IVIncrease loop diuretic

and add digoxinor

Add AIIRB

NYHAclass?

NYHAclass?

NYHA I–IIStart beta-blocker

NYHA III–IVStart

spironolactone

Cautious initiation of beta-blocker. If patient does

not tolerate beta-blockeror is still very

symptomatic thenconsider for biventricular

pacing if appropriate

cardiac status via many mechanisms (Table B). Theoverall effect of beta-blocker therapy is to improve strokevolume and cardiac ejection fraction. Beta-blockertherapy in heart failure is associated with better functionalcapacity, lower rehospitalization rates and lowermortality. Initially beta-blockers were recommended onlyin patients in NYHA functional classes I and II.However, further studies showed that they can be usedcautiously in patients in NYHA classes III and IV, where

they also provided significant symptomatic andprognostic benefit. Beta-blockers are recommended in allpatients with systolic heart failure provided there are nocontraindications. Owing to the negatively inotropiceffect of beta-blockers they should be used at the lowestpossible doses (Table C) and the dose is graduallymaximized over a period of two months. Ideally beta-blockers should be commenced once symptoms and signsof pulmonary congestion have resolved on a loop diureticand ACE inhibitor. A practical algorithm for themanagement of chronic heart failure is provided below.

Table C Beta-blocker dose

Beta-blocker Initiation doseBisoprolol 1.25 mg odCarvedilol 3.125 mg odMetoprolol 6.25 mg tdsNebivolol 1.25 mg od

Table B Mechanisms of action of beta-blockers

• Reduce myocardial ischaemia• Increase diastolic filling time and improve

stroke volume• Inhibit cardiotoxic effects of catecholamines

such as apoptosis• Inhibit down-regulation of β1 receptors• Prevent adverse cardiac remodelling• Prevent arrhythmias

Page 300: Rapid review of clinical medicine for mrcp part 2

A 40-year-old male consulted his GP at the end of theyear for wheeze and dyspnoea. He was given abronchodilator with good effect. Two weeks later he hadan exacerbation of his symptoms, which responded wellto a one-week course of steroids. Follow ing this hissymptoms were stable and completely resolved when hewent to the Caribbean for a two-week holiday, only toreturn within a few days when he came back to England.He had no history of atopy and did not keep any pets.He did not smoke. He was married with two childrenand had been working as a solderer for a carmanufacturing company for the past 14 months. Apartfrom a bronchodilator inhaler, the patient was not takingany other medications.

Clinical Cases 299

Question 297

A 75-year-old woman with a long history of atrial fibrillationwas seen in the anticoagulant clinic and noted to have anINR of 8.2. She had been on warfarin for three yearswithout any complications. One week previously she hadreceived a course of erythromycin for a lower respiratorytract infection. There was no history of blood loss from therespiratory and gastrointestinal tracts or the genito-urinarysystem. There was no history of alcohol abuse. The INRtwo weeks previously was 2.3. On examination she appearedwell but had multiple bruises on her arms.

Investigations are shown.

Hb 12 g/dlWCC 5.2 � 109/lPlatelets 180 � 109/lUrea 6 mmol/lCreatinine 92 �mol/lAST 20 iu/lAlkaline phosphatase 30 iu/lAlbumin 39 g/l

What is the management of her anticoagulation?a. Stop warfarin and repeat the INR in three days.b. Stop warfarin, give intravenous fresh-frozen

plasma and 5 mg intravenous vitamin K.c. Stop warfarin and give 5 mg oral vitamin K.d. Stop warfarin and give 0.5 mg intravenous

vitamin K.e. Stop warfarin and give fresh-frozen plasma.

Question 298

1. What is the diagnosis?a. Adult-onset asthma.b. Occupational asthma.c. Allergic bronchopulmonary aspergillosis.d. Extrinsic allergic alveolitis.e. Chronic obstructive airways disease.

2. What is the most practical test for making thediagnosis?

a. Peak flow monitoring at work and home.b. Allergen skin testing.c. CT thorax.d. Aspergillus precipitins.e. Formal pulmonary function tests.

A patient on treatment with prednisolone andchlorambucil for chronic lymphatic leukaemia over a five-year period presented with a two-day history of jaundice.

Investigations are shown.

Question 299

Hb 11 g/dlWCC 30 � 109/lPlatelets 160 � 109/lPCV 44 l/lMCV 100 flBlood film Spherocytes, red cell

agglutinationBilirubin 78 μmol/lAST 40 iu/lAlkaline phosphatase 100 iu/lAlbumin 39 g/l

What is the cause of her jaundice? a. Folate deficiency.b. Auto-immune haemolytic anaemia.c. Acute hepatitis.d. Chlorambucil therapy.e. Cholestasis secondary to steroids.

Page 301: Rapid review of clinical medicine for mrcp part 2

300

The patient has been complaining of cough and wheezefor the past year that has responded to a bronchodilatorinhaler and then a steroid trial. The history would beconsistent with reversible bronchospasm. While adult-onset asthma can occur at this age, there appears to be arelationship between the onset of his symptoms and thetime he has worked as a car solderer. Further evidencefor this comes from the fact that his symptoms resolvedwhile he was away on holiday only to return a few daysafter he arrived back to the country (and presumablyreturned to work). These findings indicate that thepatient is exposed to an allergen at work that causesbronchospasm. Soldering material contains colophony,which is a large molecular weight allergen that inducesbronchospasm by hypersensitivity immune reactions.

The diagnosis of occupational asthma is made bydemonstrating improvement of peak expiratory flow rateor lung volumes when the patient is away from theallergen. Indicators of bronchospasm reversibility are thesame as those for non-occupational asthma and includean increase in PEFR of 20% (or at least 60 l/min ) or anincrease in FEV1 by 15% (or at least 200 ml) when thepatient is away from the allergen. A PEFR is the mostpractical method of making the diagnosis of occupationalasthma; however, there is the potential problem ofhonesty of the subject.

Allergen skin testing is not practical, as skin testreagents are not available for most allergens that causeoccupational asthma. The technique may be able toidentify asthma due to certain high molecular weightallergens such as animal or plant proteins.

Management is to remove the patient from or reduceexposure to the allergen.

The answer to this question is based upon guidelinesprovided by the British Society of Haematology foranticoagulation reversal with warfarin, which are asfollows:

• In the event of major bleeding stop warfarin and give5 mg IV vitamin K slowly and IV fresh-frozen plasmaor prothrombin concentrate.

• INR >8 with no major bleeding or minor bleedingthen stop warfarin and re-start when INR has fallenbelow 5. If the presence of risk factors for bleedingsuch as renal failure, hepatic dysfunction or lowplatelets, give either 0.5 mg of IV vitamin K slowly or5 mg oral vitamin K. The INR should be checkedafter 24 hours and the dose of vitamin K repeated if itis still too high.

• INR 6–8 with no major bleeding or minor bleeding,then stop warfarin and re-start when the INR hasfallen below 5.

• INR <6 but more than 0.5 units above target, thenreduce dose or stop warfarin and re-start once INR isbelow 5.

Answer 297

a. Stop warfarin and repeat the INR in three days.

Answer 298

1. b. Occupational asthma.2. a. Peak flow monitoring at work and at home.

Chronic lymphatic leukaemia is associated with a warmauto-immune haemolytic anaemia. In this case thepresence of spherocytes and red cell agglutination, raisedMCV and bilirubin suggests haemolysis. The other causeof raised macrocytosis in lymphoma is folate deficiency(see Question 204).

Answer 299

b. Auto-immune haemolytic anaemia.

Page 302: Rapid review of clinical medicine for mrcp part 2

Clinical Cases 301

Question 301

A 67-year-old Afro-Caribbean patient was being followedup in clinic for hypertension. He had been adhering to alow-salt diet. He was currently taking bendroflumetha -

zide 2.5 mg od. The blood pressure measured160/90 mmHg. Blood pressure prior to the initiation ofbendroflumethazide was 180/96 mmHg.

Investigations are shown.

Sodium 138 mmol/lPotassium 3.3 mmol/lUrea 6 mmol/lCreatinine 150 �mol/lGlucose (fasting) 13 mmol/l12-lead ECG Left ventricular hypertrophyUrinalysis No blood pre protein

Glucose +

Which one of the following is the best therapeuticoption in the management of his blood pressure?

a. Increase dose of thiazide diuretic.b. Add beta-blocker.c. Add losartan.d. Add calcium channel blocker.e. Add doxasosin.

A 30-year-old clerical worker presented with a cough,fever and rash. The chest X-ray is shown (302a).

Question 302

What is the radiological diagnosis?a. Right middle lobe consolidation.b. Right lower lobe consolidation.c. Right-sided empyema.d. Right-sided lung abscess.e. Right-sided pleural effusion.

This is a chest X-ray (300) from a 50-year-old patientwho was investigated for cough and dyspnoea.

Question 300

What is the radiological diagnosis?a. Hiatus hernia.b. Thoracic aortic aneurysm.c. Left lower lobe abscess.d. Pericardial calcification.e. Achalasia.

330000

330022aa

Page 303: Rapid review of clinical medicine for mrcp part 2

Hypertension is very common among black patients andaffects 1 in 2 individuals aged >40 years. Hypertension inblack patients usually occurs at a younger age, is moresevere and associated with a higher rate of stroke, heartfailure and renal failure than in white patients.Hypertension in black patients is salt-sensitive andassociated with an expanded vascular volume. It respondswell to low salt intake and to pharmacological agents thatpromote urinary sodium excretion and to vasodilators.Thiazide diuretics or calcium channel blockers are the first-line drugs of choice. Since plasma renin levels are generallylow in black patients, agents affecting therenin–angiotensin–aldosterone system (i.e. ACE inhibitors,angiotensin II receptor blockers and beta-blockers) are notparticularly effective when used as first-line agents (seeAnswer 123). However any of these three drugs is effective

when added to a thiazide or a calcium channel blocker.Adding a calcium channel blocker to a thiazide diuretic orvice versa is not particularly effective, as both drugs workpredominantly by increasing urinary sodium excretion.

The theory is to use a diuretic or calcium channelblocker to increase sodium loss. This should offset therenin–angiotensin–aldosterone system and make theaddition of an ACE inhibitor, AIIRB or beta-blockermore effective. In this particular case losartan is the drugof choice since the patient is diabetic and has evidence ofdiabetic nephropathy and LVH. Losartan has beenshown to specifically reduce mortality in hypertensivediabetics (LIFE study). It has also been shown to causeregression of LVH (LIFE study) and to retardnephropathy (RENAAL study). (See Answer 125).

Doxasosin should only be used when other (moreprognostically useful) options have been explored. Theonly condition where there is a compelling indication forthe use of doxasosin in hypertension.

302

Answer 301

c. Add losartan.

There is loss of the contour of the right heart border onthe PA film. There is also evidence of a slight increase inthe opacification of the right lower lung and a slightreduction in overall lung volume compared with the leftlung. These features are consistent with right middle lobecollapse/consolidation.

The right middle lobe accounts for just 10% of the totallung volume and is more prone to collapse than the otherlung lobes because of decreased collateral ventilation.Owing to its small size, collapse of the right middle lobehas little impact on the appearance of the surroundingstructures. In the PA film there may be a triangular opacitypointing laterally. In some patients an absent contour ofthe right heart border is the only indicator of right middlelobe collapse/consolidation. The diagnosis can beconfirmed on a lateral film, which demonstrates an obliquetriangular opacity with its apex pointing towards the hilum(302b, arrows). This appearance results from the antero-superior displacement of the major fissure and the postero-inferior displacement of the minor fissure.

Answer 302

a. Right middle lobe consolidation.

The patient has a localised air-filled cavity with a fluidlevel behind the heart which is highly characteristic of ahiatus hernia. In achalasia the whole oesophagus isdilated so there is wide mediastinum with an air fluidlevel in the upper chest.

Answer 300

a. Hiatus hernia.

330022bb

Page 304: Rapid review of clinical medicine for mrcp part 2

Clinical Cases 303

A 70-year-old male underwent coronary angiographyafter presenting with unstable angina. The angiogramrevealed that three-vessel coronary artery disease resultingin deployment of stents in the left anterior descendingand right coronary arteries. The following day hedeveloped general malaise and a fever. On examination hehad a vasculitic rash on his hands and feet.

The patient had a past history of asthma. His medi -cations included aspirin, clopidogrel, simvastatin andheparin.

Investigations are shown.

Question 303

Hb 15 g/dlWCC 11 � 109/lNeutrophils 8 � 109/lLymphocytes 1.5 � 109/lEosinophils 1.5 � 109/lPlatelets 200 � 109/lESR 80 mm/hCRP 100 g/lUrea 13 mmol/lCreatinine 178 �mol/lANA NegativeANCA NegativeC3 LowC4 LowBlood culture SterileUrinalysis White cells ++

Protein ++

What is the diagnosis?a. Infective endocarditis.b. Cholesterol atheroemboli.c. Churg–Strauss syndrome.d. Polyarteritis nodosa.e. Contrast nephropathy.

Question 304

A 30-year-old garage mechanic presented with a one-week history of generalized malaise, fever, cough andintermittent haemoptysis.

Investigations are shown.

Hb 7.5 g/dlMCV 76 flWCC 8 � 109/lMCH 28 pgUrea 15 mmol/lCreatinine 136 �mol/lChest X-ray Bilateral alveolar shadowingRenal ultrasound Normal-sized echogenic

kidneysUrinalysis >50 rbc, red cell casts

seenLung function: Actual PredictedFEV1 (l) 2.6 3.0FVC (l) 2.9 4.2TLC (l) 5.1 6.5KCO 2.8 2.2

What are the two best investigations that wouldestablish the diagnosis?

a. Serum IgE specific for Aspergillus.b. Serum angiotensin-converting enzyme level.c. Antistreptococcal antibodies.d. Lupus anticoagulant.e. Serum cryoglobulin.f. Antiglomerular basement membrane antibodies.g. Antineutrophil cytoplasmic antibodies.h. Serum IgA.i. Serum complement.j. Urinary Legionella antigen.

Question 305Which of the following drugs have an impact on theprognosis in heart failure?

a. Atenolol.b. Digoxin.c. Furosemide.d. Carvedilol.e. Bumetanide.

Page 305: Rapid review of clinical medicine for mrcp part 2

304

The history of renal failure and cutaneous manifestationsafter coronary angiography is most consistent withcholesterol atheroemboli.

Cholesterol atheroemboli may complicate manipu lationof atheromatous arteries, such as during angio graphy,angioplasty or vascular surgery. Embolization of cholesterolcrystals results in fever, raised inflammatory markers,eosinophilia, hypocomplementaemia, cutaneousmanifestations and renal failure. Cholesterol atheroembolilodge in small capillaries in the skin, producing livedoreticularis and occasionally a vasculitic rash. The distalextremities may appear blue. The emboli may also causeischaemia of the renal cortex leading to cortical atrophy anda progressive decline in renal function that persists for overtwo weeks. Urinalysis usually reveals white cells in the urineand mild proteinuria; however, proteinuria in the nephriticrange and haematuria have also been described. Baselinerenal function may never return to normal in severe cases.

In patients developing renal failure after coronaryangiography the differentiation between contrastnephropathy and cholesterol atheroemboli can be difficult.However, in contrast nephropathy cutaneous manifestationsare uncommon. Furthermore, renal function usually returnsto baseline after approximately one week whereas renalfailure secondary to cholesterol atheroemboli may persist forseveral weeks, and renal function may never normalize.

The diagnosis of cholesterol emboli is confirmed bythe demonstration of cholesterol crystals on skin or renalbiopsy. Treatment is supportive. Anticoagulation shouldbe stopped, since it is thought that it prevents healing ofthe atheromatous lesions.

Although Churg–Strauss syndrome may present withvasculitis and haematuria in an individual with a history ofasthma (asthma may predate the vasculitic manifestationsof Churg–Strauss syndrome by several years), the temporalrelationship between coronary angiography and the onsetof the illness favours cholesterol atheroemboli. Polyarteritisnodosa is also unlikely for similar reasons. Renalinvolvement in Churg–Strauss syndrome and polyarteritisnodosa would also be associated with microscopichaematuria or red cell casts.

Answer 303

b. Cholesterol atheroemboli.

The patient presents with an acute renal–pulmonarysyndrome characterized by pulmonary haemorrhage andan acute nephritis. The differential diagnosis (in decreasingfrequency) is between antiglomerular basement membrane(anti-GBM) disease (Goodpasteur’s syndrome), Wegener’sgranulomatosis, microscopic polyangiitis, polyarteritisnodosa, SLE and post-streptococcal glomerulonephritis.In individuals with a coexistent purpuric rash the diagnosisof Henoch–Schönlein purpura may also be considered,although pulmonary vasculitis is a very rare complicationof the disorder.

The most probable diagnosis in this particular case isanti-GBM disease followed by a vasculitis. The first twodiagnostic investigations should include anti-GBMantibodies and serum antineutrophil cytoplasmicantibodies. The presence of anti-GBM antibodies in thiscontext is virtually diagnostic of anti-GBM disease. Thepresence of ANCA is highly suggestive of Wegener’sgranulomatosis or a related disorder. SLE classicallypresents with constitutional symptoms such as lethargyand malaise together with arthralgia and mildhaematological features. Pulmonary haemorrhage is arecognized complication but is a relatively raremanifestation of the disorder.

Answer 304

f. Antiglomerular basement membrane antibodies.g. Antineutrophil cytoplasmic antibodies.

Prognostically useful drugs in heart failure are tabulatedbelow. In Afro-Caribbean patients the combination ofisosorbide dinitrate or mononitrate and hydralazine hasbeen shown to improve prognosis in symptomaticpatients when added to conventional heart failure therapythat includes ACE inhibitors, beta-blockers andspironolactone (see Answer 296).

Answer 305

d. Carvedilol. Prognostically useful drugs in heart failure

Class of drug ExamplesACE inhibitors Captopril, enalapril, lisinopril,

ramipril, trandolaprilBeta-blockers Carvedilol, bisoprolol,

metoprolol, nebivololARII receptor Losartan, valsartan,

candesartan blockersSpironolactone

Page 306: Rapid review of clinical medicine for mrcp part 2

Clinical Cases 305

Question 306

A 49-year-old male with alcohol-related liver diseasepresented with burning epigastric pain and loss ofappetite. An upper gastrointestinal endoscopy revealedgastritis and he had very small oesophageal varices thathad not bled. There was no previous history ofhaematemesis or malena. The patient was haemo -dynamically stable.

What is the most effective prophylactic therapy toprevent a variceal bleed in this particular situation?

a. Enalapril.b. Atenolol.c. Banding the varices.d. Propranolol.e. Octreotide.

Question 307

A 70-year-old female presented with sudden onset ofexpressive dysphasia and a right hemiparesis. She had along-standing history of hypertension and had beeninvestigated for an episode of amurosis fugax five yearsago. There was no history of diabetes mellitus and thepatient was a non-smoker. The patient was takingbendroflumethiazide 2.5 mg for hypertension. She wasnot on any other medication.

On examination she had a heart rate of100 beats/min, which was irregularly irregular. Theblood pressure measured 170/100 mmHg. There wasclear evidence of an expressive dysphasia and a right-sided hemiparesis. Auscultation of the heart revealednormal heart sounds without any murmurs. There wereno carotid bruits.

Investigationsare shown.

A 67-year-old male with a mechanical aortic valve isadmitted with a dense left hemiparesis. CT scan of thebrain reveals evidence of cerebral infarction but nohaemorrhage. The INR on admission is 2.1.

Question 309What is the best step in managing his anticoagulation?

a. Stop warfarin.b. Reverse anticoagulation with Vitamin K.c. Reverse anticoagulation with FFP.d. Increase dose of warfarin.e. Stop warfarin and switch to intravenous heparin.

This is a blood film (308) from a young male whopresented with severe lower back pain.

Question 308

What is the haematological diagnosis?a. Hereditary spherocytosis.b. Microangiopathic haemolytic anaemia.c. Hereditary elliptocytosis.d. Sickle cell anaemia.e. Leucoerythroblastic anaemia secondary to bone

marrow infiltration.

12-lead ECG Atrial fibrillation CT scan brain Cerebral infarction affecting

the territory supplied by the left middle cerebral artery

What is the immediate treatment of choice?a. Aspirin.b. Warfarin.c. Heparin.d. Add ACE inhibitor.e. DC cardioversion.

330088

Page 307: Rapid review of clinical medicine for mrcp part 2

306

All patients with cirrhosis of the liver should undergo uppergastrointestinal endoscopy to evaluate the presence ofvarices. Prevention of bleeding of varices reduces mortality.

There is good evidence from large trials that beta-blockers, particularly propranolol and nadolol, are highlyeffective agents at controlling portal hypertension andpreventing variceal bleeds. Beta-blockers are re commendedas prophylactic therapy in patients with mild to moderateoesophageal or gastric varices to prevent bleeding.Sclerotherapy is not recommended for primary prophylaxis,

as the risk of causing haemorrhage from the procedureoutweighs the benefits. While band endoscopic ligation isthe treatment of choice following acute or recurrent varicealhaemorrhage, the role of endosopic ligation as primaryprophylactic therapy is not entirely clear and should bereserved for patients with large varices at high risk ofbleeding who are intolerant of beta-blockers.

Octreotide, transjugular intrahepatic portosystemicshunt and surgery do not have a role in primaryprophylaxis for variceal bleeding. Drugs inhibitingangiotensin production have been evaluated in smalltrials, but they have not convincingly reduced the risk ofvariceal haemorrhage.

Answer 306

d. Propranolol.

Answer 307

The patient has suffered a recent cerebrovascular accident.In this particular case the CVA may be secondary tothrombosis of the middle cerebral artery due toatherosclerotic plaque rupture or due to an embolus arisingfrom the carotid arteries or the left atrium. The probabilityof embolism from left atrial thrombus is high since thepatient is in atrial fibrillation. A cerebral haemorrhage hasbeen excluded by CT scan; therefore, treatment with anantithrombotic agent such as aspirin or clopidogrel is

relatively safe. Both drugs will be effective in preventingfurther thrombotic episodes. In relation to AF, the mosteffective method for preventing systemic thrombo embolismis anticoagulation. However, anticoagulation early after athrombotic or embolic CVA increases the risk ofhaemorrhage into the infarcted area. Current guidelinesrecommend withholding anticoagulation for at least twoweeks following a CVA in the context of AF. Theanticoagulation agent of choice in chronic AF is warfarin,which may be safely started after two weeks. Persistentlyelevated blood pressure should be treated after 72 hours.(See explanation and algorithm in Answer 103.)

a. Aspirin.

It is very difficult to stop anticoagulation completely inthis situation and there is a high risk of prosthetic valveobstruction with thrombus. Furthermore, it is likely thatthe embolic cerebrovascular accident in this patient wasdue to under anticoagulation for the mechanical valve.Patients with mechanical valve prostheses are usuallymaintained on an INR of 3–4.

Although continued anticoagulation increases the risk ofa cerebral haemorrhage into the infarcted area, the aimhere would be to cautiously achieve therapeuticanticoagulation to prevent further embolic strokes. At thesame time the patient should be observed carefully forneurological deterioration in the event of a cerebralhaemorrhage. Switching to heparin enables more rapidanticoagulation and also more rapid reversal ofanticoagulation compared to using warfarin alone.

Answer 309

e. Stop warfarin and switch to intravenous heparin.

The blood film reveals multiple sickle cells and targetcells. Bone crisis is the most common type of sickle-

related crisis in patients with sickle cell anaemia. Themanagement includes pain relief with opiates,rehydration, oxygen and antibiotics if there is evidence ofinfection. (See Question 263.)

Answer 308

d. Sickle cell anaemia.

Page 308: Rapid review of clinical medicine for mrcp part 2

Clinical Cases 307

A 20-year-old male was investigated for a six-year historyof blistering of the forehead and the hands that wasworse during the summer. He was not on anymedications. He had been investigated for severalepisodes of severe central abdominal pain over the pasttwo years for which no obvious cause had beenidentified. He admitted to binge drinking and consumedup to 100 units of alcohol per week. Abdominalexamination was normal. Investigations are shown.

Question 310Hb 13.8 g/dlWCC 8 � 109/l

(normal differential)Platelets 200 � 109/lBlood film NormalAmylase 100 iu/l

(NR <220 iu/l)Serum iron 33 mmol/l

(NR 13–32 mmol/l)TIBC 49 mmol/l

(NR 42–80 mmol/l)Serum ferritin 300 iu/lUrinary coproporphyrins 1640 nmol/24h

(NR 50–350 nmol/24h)Faecal coproporphyrin 1900 nmol/24hUrinary protoporphyrin 400 nmol/24h

(slightly raised)CT scan abdomen Normal

What is the diagnosis?a. Acute intermittent porphyria.b. Porphyria cutanea tarda.c. Haemochromatosis.d. Variegate porphyria.e. Hereditary coproporphyria.

An alcohol-dependent patient was admitted with asterixisand jaundice. Blood results are shown.

The patient was given IV thiamine and prescribeddiazepam 10 mg six-hourly. Over the next 24 hours hiscondition deteriorated. He became drowsy andcollapsed. The BM stix reading was 6 mmol/l.

Question 312

Hb 10g/dlMCV 102 flWCC 14 � 109/lPlatelets 80 � 109/lSodium 131 mmol/lPotassium 3.2 mmo/lUrea 2 mmol/lGlucose 4.2 mmol/l

What is the definitive treatment?a. IV flumazenil.b. IV dextrose 50% (50 ml). c. IV hydrocortisone.d. IV naloxone.e. IV acyclovir.

A 65-year-old male presented with pain in his wrists(311a). There was no history of trauma or joint painselsewhere. He had a past history of hypertension andgout. He smoked 20 cigarettes per day and consumed 24units of alcohol per week.

Question 311

What investigation would you perform next?a. Chest X-ray.b. Serum parathyroid hormone level.c. Bone scan.d. Auto-antibody screen.e. Serum ACE.

331111aa

Page 309: Rapid review of clinical medicine for mrcp part 2

308

Hereditary coproporphyria is an autosomal dominantporphyria caused by deficiency of the enzymecoproporphyrinogen oxidase. The disorder results inincreased excretion of coproporphyrin in the urine andfaeces and is characterized by a combination of neurovisceralfeatures that are similar to but less severe than those seen inacute intermittent porphyria, and photosensitivity. Patientsexperience blistering, scarring and hypertrichosis of the skinexposed to sunlight that is identical to that seen in porphyriacutanea tarda. HC has been reported mainly in the UK,Europe and North America. HC is precipitated by the samefactors that precipitate acute intermittent porphyria.

The disorder can be differentiated clinically from acuteintermittent porphyria by the fact that photosensitivity isnot a feature of acute intermittent porphyria. In patientswho do not have photosensitivity, the differentiationfrom acute intermittent porphyria is based on the

identification of high concentrations of urinary and faecalporphyrins in HC.

Another porphyria that causes both neurovisceralsymptoms and photosensitivity is variegate porphyria. VPis also an autosomal dominant-type hepatic porphyriacaused by deficiency of the haem biosynthetic enzymeprotoporphyrinogen oxidase. It is common in the SouthAfrican white population. The disorder is differentiatedfrom HC by measuring urinary and faecalcoproporphyrins and protoporphyrins. Both arecharacterized by high concentrations of faecal and urinarycoproporphyrins; however, in VP the concentration ofprotoporphyrins is considerably higher than that seen inHC.

The management of neurovisceral symptoms in bothVP and HC is similar to that of acute intermittentporphyria (Answer 147). There is no effective therapy forphotosensitivity other than avoidance. Unlike porphyriacutanea tarda, the photosensitivity does not respond tochloroquine.

Answer 310

e. Hereditary coproporphyria.

Answer 312

The patient has become encephalopathic owing todiazepam. Patients with advanced cirrhosis may be

particularly sensitive to benzodiazepines because of anincreased concentration of benzodiazepine receptorligands in the brain. Flumazenil has been shown to behelpful for encephalopathic patients who have receivedbenzodiazepines.

a. IV flumazenil.

The patient has a symmetrical periosteal bone reaction(311b periostitis, arrows). The differential diagnosis ofsymmetrical periostitis is tabulated below. Hypertrophicpulmonary osteoarthropathy, a recognized cause of asymmetrical periostitis, may complicate bronchialcarcinoma therefore, the next investigation should be achest X-ray.

Answer 311

a. Chest X-ray.

Causes of a symmetrical periostitis

• Hypertrophic pulmonary osteoarthropathy• Vascular insufficiency• Thyroid acropathy• Fluorosis• Pachydermoperiostitis

331111bb

Page 310: Rapid review of clinical medicine for mrcp part 2

Clinical Cases 309

A 60-year-old female presented with a two-month historyof lethargy and dysphagia for solid food. Her appetitewas slightly reduced, but there was no history ofindigestion or weight loss. The examination of her handsand tongue are shown (313a , b). There was nolymphadenopathy, or any other palpable masses in theneck. Examination of all the major systems was normal.

Investigations are shown.

Question 314

A 14-year-old female was referred to a specialist becauseshe had difficulty in walking over the past eight months.She tripped easily. Her feet had adopted a strange shapeand she had problems fitting her shoes. Her father hadweakness affecting his legs and was under regular review bya neurologist.

On examination, she appeared well and a generalexamination was normal. There was wasting of the smallmuscles of the hand, but upper-limb tone was normal andthe reflexes were preserved. On examination of the lowerlimbs she had bilateral pes cavus. The calf muscles werewasted and exhibited fasciculation. There was mild

weakness of dorsiflexion, plantar flexion and eversion at theankle joint. Examination of the thighs was normal. Boththe ankle and knee reflexes were absent. Proprioceptionwas impaired at the level of the first metatarsal joint. Two-point discrimination sense was also impaired in both feetbut pain, temperature and pressure sense was normal.

Hb 8.8 g/dlWCC 6 � 109/lPlatelets 170 � 109/lBlood film (313c)Barium study (313d)

1. What is the probable cause of the appearance of thehands and the tongue?

2. Comment on the blood film.

3. What is the abnormality on the barium study?4. List two possible diagnoses.5. Which single investigation would you perform?

Question 313

1. What is the diagnosis?2. What three tests would you perform to confirm the

diagnosis?

331133aa 331133bb

331133cc 331133dd

Page 311: Rapid review of clinical medicine for mrcp part 2

310

Answer 314

The patient presents with difficulty in walking. Onphysical examination, she has pes cavus, weakness of thedistal aspects of the upper and lower limbs. The lower-limb reflexes are absent and proprioception and two-point discrimination sense are impaired. These are thefeatures of HSMN. A family history is not given, but thisshould be an important part of the investigation.

HSMN is divided into Types I, II and III. Type I isthe classic, familial type in which there is a diffusedemyelinating neuropathy which usually presents in thefirst two decades. Distal wasting of the lower limbsproduces peroneal muscular atrophy and thecharacteristic ‘inverted champagne bottle’ appearance.Deformities of the feet and scoliosis are common. Thesensory fibres of proprioception are affected, producing asensory ataxia. Linkage studies on families with thisneuropathy have localized the condition to a genetic

defect on the long arm of chromosome 1. Nerveconduction is very reduced.

Type II HSMN is similar, but much milder. In thiscondition, nerve conduction is only very slightly reducedor normal.

HSMN III – also known as Dejerine–Sottas disease –is inherited as an autosomal recessive trait. It is a slow,progressive mixed sensory and motor neuropathy thatstarts in the first decade and is typically associated withenlargement of the peripheral nerves, which are easilypalpable on examination. Differential diagnosis ofthickened or enlarged peripheral nerves is listed below.Recently, the classification of the HSMNs has beenextended to include types IV and V. Descriptions of thelatter are beyond the scope of this book.

Answer 313

This woman has the features of oesophageal web, whichis a rare condition characterized by submucosalinflammation, usually in the post-cricoid region,although any part of the oesophagus can be affected. Theinflammation produces a ‘web’ or a concentric stenosis.The condition affects women in their fourth to sixthdecade. The classic features are dysphagia, iron-deficiencyanaemia, koilonychia and other stigmata of irondeficiency, such as glossitis. It is a pre-malignantcondition, and can predispose to carcinoma of theoesophagus, which needs to be excluded by endoscopyand biopsy of the stricture.

Causes of thickened peripheral nerves

• HSMN type III• Acromegaly• Tuberculoid leprosy• Neurofibromatosis• HIV syndrome

1. The woman has koilonychia and glossitis, which arefeatures of iron-deficiency anaemia.

2. Anaemia with microcytosis, anisocytosis,poikilocytosis and hypochromia. The features areconsistent with iron-deficiency anaemia.

3. Stricture in the upper oesophagus consistent withan oesophageal web (Paterson–Brown–Kelly orPlummer–Vinson syndrome).

4. The differential diagnosis is between oesophagealweb and carcinoma of the upper oesophagus.

5. Oesophagoscopy and biopsy of stricture forhistological analysis.

1. Charcot–Marie–Tooth disease (hereditarysensorimotor neuropathy (HSMN)/peronealmuscular atrophy).

2. i. Nerve conduction studies.ii. Sural biopsy.iii. Family screening.

Page 312: Rapid review of clinical medicine for mrcp part 2

Clinical Cases 311

A 59-year-old female was referred to the out-patientclinic with lethargy and chest pain consistent with angina.She was on a gluten-free diet and on long-standingtreatment for a skin rash (316). On examination, she wasthin and had pale conjunctivae. She appeared cyanosed.The heart rate was 110 beats/min, with good volume;the blood pressure was 130/75 mmHg. The heartsounds were normal and the chest was clear.

Investigations are shown.

Hb 8 g/dlWCC 11 � 109/lPlatelets 240 � 109/lMCV 110 flPCV 0.34Reticulocytes 10%Blood film Polychromasia, macrocytosis,

target cells, a few fragmented red cells

Serum B12 470 ng/l Red cell folate 65 μg/lSerum folate 1.8 μg/l

A family pedigree suffering from a very rare condition isshown (315).

Question 315

1. What is the mode of inheritance?2. Give one example of a condition inherited in this

way.

1. Give two possible explanations for the folate levels.2. What is the most likely cause for the blood picture?3. What additional morphological feature in the blood

film would be diagnostically helpful?4. What test would you request to determine the cause

of the cyanosis?5. Give three reasons for the patient’s chest pain.

Affected male Unaffected male*

Affected female Unaffected female*

* Does not exclude inheritance of an autosomalrecessive gene in either sex or an X-linked recessivedisease in a female.

Question 316

331155

331166

Page 313: Rapid review of clinical medicine for mrcp part 2

312

Answer 315

During conception, all the mitochondria inherited by thezygote come from the ovum because the spermatozoon isrelatively depleted of mitochondria by the time it reachesthe ovum; therefore, abnormalities in mitochondrial DNAcan only be transmitted by a female. An affected female willtransfer the abnormal gene and the abnormal phenotype toall offspring; however, a male does not transfer the disease.

Mitochondrial DNA contains a limited number of genesthat may be subject to point mutations, deletions orduplications. Abnormalities in mitochondrial DNA result ina variety of abnormalities, including easy fatigue, myopathy,epilepsy, diabetes, optic atrophy, deafness, retinitispigmentosa and cardiomyopathy. Calcification of basalganglia is well recognized with mitochondrial disease. Themost common mutation is an A to G mutation atnucleotide 3243. Most recognized mutations occur intransfer RNA or genes encoding the proteins involved inthe oxidative phosphorylation pathway. The serumlactate/pyruvate ratio is elevated, indicating impairedaerobic glycolysis. CSF lactate may be elevated even when

serum lactate is normal. Muscle biopsy may reveal redragged fibres (increased fibre staining with stain forsuccinate dehydrogenase) representing abnormalmitochondria.

Answer 316

The patient is on a gluten-free diet, suggesting that shehas a gluten-sensitive enteropathy. She is on treatmentfor a long-standing rash (316) which is characteristic ofdermatitis herpetiformis. The association betweendermatitis herpetiformis and gluten-sensitive enteropathyis well recognized. Dermatitis herpetiformis is effectivelytreated with dapsone. An important side-effect ofdapsone is haemolytic anaemia because of its oxidanteffect on Hb. This effect is severe in patients withglucose-6-phosphate dehydrogenase deficiency, but mayalso occur in some individuals as an idiosyncratic

reaction, particularly with chronic use of the drug. Ironwithin the haem molecule of haemoglobin is oxidizedfrom the ferrous state (Fe2+) to the ferric state (Fe3+),resulting in methaemoglobin (Table A). This leads todepletion of the red cell NADPH (which is utilized to tryand maintain iron in the ferrous form) and consequentreduction in the integrity of the red cell membrane.Patients with glucose-6-phosphate dehydrogenasedeficiency have lower cellular levels of NADPH thannormal individuals, and are more severely affected.Methaemoglobin has a very high affinity for oxygenmolecules, resulting in a leftward shift in the oxygendissociation curve and impaired unloading of oxygen tothe tissues (Table B). Methaemoglobin has a bluishcolour, therefore patients with significantmethaemoglobinaemia appear cyanosed even when thearterial oxygen tension is normal. Pulse oximetry showslow oxygen saturations because it does not registeroxymethaemoglobin. A normal arterial oxygen tensionand a low oxygen saturation should raise the suspicion ofmethaemoglobinaemia in a patient who appearscyanosed. This contrasts with carbon monoxidepoisoning, in which oxygen saturation on the pulseoximeter is normal but arterial oxygen tension may be

Syndromes associated with mitochondrial gene mutations

• MERRF: myoclonic epilepsy, ragged red fibres(mutation in t-RNA Lys)

• MELAS: mitochondrial encephalomyopathy,lactic acidosis, stroke (mutation in t-RNA Leu)

• NARP: neuropathy, ataxia, retinitis pigmentosa(mutation at nucleotide 8893; coding forATPase 6)

• LHON: Leber’s hereditary, optic neuropathy(mutation in NADH dehydrogenase Subunit 4)

• Kearnes–Sayer syndrome: progressive externalophthalmoplegia, retinitis pigmentosa, heartblock, myopathy, diabetes

• Usher’s syndrome: deafness, retinitispigmentosa, myopathy

1. Inheritance of maternal mitochondrial geneticabnormality.

2. Conditions inherited in this way are listed (Table).

1. i. Malabsorption.ii. Dapsone-induced haemolysis.

2. Dapsone-induced haemolytic anaemia.3. The presence of Heinz bodies (oxidative

haemolysis).4. Serum methaemoglobin level.5. i. Anaemia.

ii. Reduced delivery of oxygen to the heart due to leftward shift of the oxygen dissociation curve resulting from methaemoglobinaemia.

iii. Tachycardia.

Page 314: Rapid review of clinical medicine for mrcp part 2

Clinical Cases 313

normal because the pulse oximeter cannot differentiatebetween oxyhaemoglobin and carboxyhaemoglobin.

Methaemoglobin levels exceeding 20% are associatedwith dyspnoea and headaches. Patients may developangina, despite normal coronary arteries, owing toreduced delivery of oxygen to myocardial cells. Collapseand sudden death may occur when levels exceed 60%.The blood film will demonstrate features of intravascular

haemolysis (polychromasia, macrocytes, red cellfragmentation) and Heinz bodies, which are small redcell inclusions of denatured Hb. These are scarce on theperipheral blood film unless the spleen has beenremoved. Other causes of methaemoglobinaemia areshown (Table A). Dapsone-induced chronic haemolysismay lead to depletion of folate.

Table A Causes of methaemoglobinaemia

GeneticNADPH diaphorase deficiencyHb M

AcquiredDrugs: Sulphonamides

PrimaquinPhenacetin

Toxins: Ferric cyanideChlorateAniline dyes

Table B Causes of left-shift of O2 dissociationcurve

• Methaemoglobinaemia• Sulphaemoglobinaemia• Fetal Hb• 2,3 diphosphoglycerate deficiency• Alkalosis• Hypocapnia• Hypothermia

A 76-year-old female was admitted to hospital withmalaise, weakness, nausea and anorexia. She had beenrelatively well until two weeks previously when shedeveloped a painful left calf for which she was prescribed‘some tablets’ by her GP. On examination, she appeareddehydrated and her blood pressure was 90/50 mmHg.She had clinical evidence of a deep-vein thrombosis inher left calf. All other physical examination was normal.

Investigations are shown.

Hb 13 g/dlWCC 8 � 109/lPlatelets 300 � 109/lMCV 88 flSodium 128 mmol/lPotassium 6.2 mmol/lChloride 88 mmol/lBicarbonate 13 mmol/lUrea 24 mmol/lCreatinine 297 μmol/lBlood glucose 10 mmol/lUrinalysis 24-hour sodium

68 mmol/l

What is the cause of this patient’s low sodium?a. Addison’s disease.b. Diuretic excess.c. Acute tubulointerstitial nephritis secondary to

NSAIDs.d. Diabetic ketoacidosis.e. Vomiting.

Question 317

Page 315: Rapid review of clinical medicine for mrcp part 2

314

All five suggested answers can be responsible forhyponatraemia, but in a woman who has recently beentreated for a painful calf it is most likely that she wasprescribed some form of pain killer. She has subsequentlypresented with anorexia, malaise and vomiting and hasderanged renal function. Urinalysis reveals a 24-hoururine of over 20 mmol/l sodium, which suggests a salt-losing state when one considers her low serum sodium.The most probable cause of her presentation is an acutenephritis secondary to prescription of an NSAID.

The biochemistry is partly consistent with Addison’sdisease; however, the diagnosis is less likely in the contextof the history and the presence of a raised blood sugar.Diuretic excess is also possible. One could assume that shewas prescribed a diuretic; this has resulted in dehydrationand pre-renal failure, but the serum bicarbonate and thepotassium levels suggest that this is unlikely. Diureticsused to treat ankle swelling alone comprise thiazides orloop diuretics, which result in a hypokalaemic alkalosis.Potassium-sparing drugs such as amiloride alone orspironolactone are not the drugs of choice to treat ankleswelling. Vomiting is also a recognized cause ofhyponatraemia, but there is no mention of it in thehistory, and the raised potassium is unlikely to explainprofuse vomiting, which usually results in low sodium,potassium and chloride levels because gastric fluidcontains high concentrations of all these ions.

Diabetic ketoacidosis would be lowest in thedifferential diagnosis because the blood sugar is relativelylow. The hyponatraemia resulting from ketoacidosis is aresult of osmotic diuresis from the very high blood sugar.A random blood sugar of 10 mmol/l could representimpaired glucose tolerance or mild non-insulin-dependent diabetes, but is against diabetic ketoacidosis.

NSAIDs may cause acute tubular necrosis or an acutetubulointerstitial nephritis (Table). Acute tubular necrosisis an increasing problem with NSAID prescription.NSAIDs inhibit cyclo-oxygenase activity and decrease

prostaglandin synthesis. Prostaglandins have a number offunctions in the kidney, one of which is to stimulate reninproduction and thus activate the angiotensin–aldosteronesystem. With NSAIDs, there is a state ofhyporeninanaemic hypoaldosteronism causinghyperkalaemia, hyponatraemia, increased urinary sodiumloss and dehydration. In patients with pre-existing renaldisease, or in diabetics, this may be significant enough toproduce acute renal failure. Such patients usually have asignificant hyperkalaemia. Acute tubulointerstitialnephritis is often due to a hypersensitivity reaction todrugs (Table). NSAIDs and the penicillins are mostcommonly implicated in the UK. Patients may presentwith malaise, arthralgia and/or acute renal failure. Manypatients have eosinophilia or eosinophiluria.

Treatment of both forms of renal failure involveswithdrawal of the offending drug. The treatment of acutetubular necrosis is supportive, and involves careful fluidbalance, correction of hyperkalaemia and hypovolaemia,and dialysis if necessary. Acute interstitial nephritis istreated in the same way, but in addition high-doseprednisolone is prescribed to reduce the acuteinflammatory response in the kidney.

c. Acute tubulointerstitial nephritis secondary toNSAIDs.

Causes of acute interstitial nephritis

• Penicillins• NSAIDs• Sulphonamides• Allopurinol• Phenindione• Acute pyelonephritis• Sickle cell crisis• Lead poisoning

Renal manifestations of NSAIDs

• Acute tubular necrosis• Acute interstitial nephritis• Minimal change glomerulonephritis

Answer 317

Page 316: Rapid review of clinical medicine for mrcp part 2

Clinical Cases 315

The above (318a, b) are two rhythm strips from a 24-hour ECG in a 70-year-old male with dizziness andpalpitation.

1. What is the diagnosis?2. What is the management?

Question 318

Question 319

A 40-year-old dentist presented with a five-month historyof weight loss. He had recently become jaundiced.

Blood results are shown.

AST 40 iu/lAlkaline phosphatase 165 iu/lBilirubin 80 μmol/lAlbumin 32 g/lα-fetoprotein 40 ng/l Serum ferritin 450 μg/lHepatitis serology Hep B sAg positive

Hep B eAg positiveHep B cAb positive

1. What is the diagnosis?2. Which four tests would you perform next?3. What is the management?

331188aa

331188bb

Page 317: Rapid review of clinical medicine for mrcp part 2

316

Answer 319

The differential diagnosis of weight loss and jaundice iswide, but the presence of Hep B eAg and Hep B sAgnarrows this down to active hepatitis B infection. (Note:dentistry is a high-risk occupation for hepatitis B.)

The symptoms have been present for five months,suggesting that there is a chronic active infection.Investigations should be based on confirming the extentof hepatic damage and determining the patient’sinfectivity and immune status. The �-fetoprotein ismodestly elevated in severe viral hepatitis and activecirrhosis. Levels exceeding 1,000 ng/l are stronglysuggestive of hepatocellular carcinoma. Liver biopsy willreveal a ground-glass appearance of infected hepatocytesand evidence of balloon degeneration. Serologicalmarkers such as Hep B eAb and Hep B sAb willdetermine infectivity and immune response. Liverultrasound may be useful in determining the extent ofliver damage when clotting and platelets are veryabnormal. Hepatitis B-positive patients in high-riskoccupations should also be offered HIV testing, becauseboth conditions are acquired in the same manner fromthe same type of patients. There is evidence thattreatment with �-interferon in patients with circulatingHep B sAg and Hep B eAg will reduce viral replicationand infectivity.

Some 2% of patients with hepatitis B infection developfulminant hepatitis, while 10% of patients become carriersafter the initial infection. Some carriers are positive for

Hep B sAb, but have normal liver histology, while othershave chronic active or chronic persistent hepatitis.Chronic active hepatitis may progress to cirrhosis of theliver. Patients with cirrhosis of the liver as a result ofhepatitis B infection are more prone to hepatocellularcarcinoma.

Hep B sAg appears in the circulation 1–5 weeks beforethe onset of symptoms. It is no longer detectable whenthe infection has resolved, but its presence six monthsafter infection indicates chronic infection.

Hep B eAg appears in the circulation shortly before or1–3 weeks after the onset of symptoms. Its presenceindicates active viral replication, either in acute or inchronic infection, and high infectivity. Its disappearanceis suggestive of resolution of infection. Hepatitis B virusDNA also appears at around the same time and itspresence also indicates active viral replication.Disappearance of Hep B virus DNA correlates withresolution of infection. Hep B cAg is only seen withinhepatocytes, and does not circulate in the serum.

IgM anti-Hep B cAb appears at the time of onset ofsymptoms. It is a marker of recent hepatitis B infectionand may be the only marker of hepatitis B infectionbefore the appearance of Hep B sAg. IgG anti-Hep BcAb is present lifelong after hepatitis B infection. It doesnot indicate immunity. Hep B eAb appears after the cAb.Its presence indicates reduced infectivity. Hep B sAb isthe last antibody to appear. Its presence indicatesimmunity.

Interferon therapy will reduce Hep B e and sAg levels(40% and 10%, respectively). All patients with circulatinge and sAg are offered treatment with interferon unlessthere is evidence of gross hepatic decompensation.Interferon has the effect of exacerbating coexisting auto-immune conditions, and is contraindicated in thesesituations.

Answer 318

The diagnosis is consistent with sick sinus syndrome,which is a degenerative condition affecting the sino-atrialnode and the atrial myocardium. It is characterized by

long sinus pauses and paroxysmal atrial arrhythmiascomprising atrial tachycardia, flutter and fibrillation.These patients are at increased risk of systemic emboli.Many go on to develop atrioventricular nodal disease.

Patients in sinus rhythm are implanted with an atrialpacemaker as they have effective atrial transport; however,those with atrial fibrillation are treated with ventricularpacing. Unlike atrioventricular block, pacing for sicksinus syndrome does not save lives.

1. Sick sinus syndrome. Rhythm strip 318a revealssinus tachycardia followed by sinus arrest or sinuspause of 2.5 seconds. Rhythm strip 318b revealsatrial fibrillation with a rapid ventricular rate.

2. Symptomatic patients should be paced.

1. Chronic active hepatitis B infection.2. i. Hepatitis B e antibody (Hep B eAb)/hepatitis s

antibody (Hep B sAb).ii. Liver ultrasound.iii. Liver biopsy (if platelets and clotting not

severely deranged).iv. HIV antigen.

3. �-interferon.

Page 318: Rapid review of clinical medicine for mrcp part 2

Clinical Cases 317

Question 320

A 62-year-old female with rheumatoid arthritis wasreferred to an ophthalmologist with a three-day history ofa painful left eye. The visual acuity was normal. She had asimilar problem three years ago which required therapywith oral steroids. She was diagnosed with rheumatoidarthritis 21 years ago after presenting with pain in herhands, feet and elbows. Initial treatment comprisedNSAIDs, then chloroquine, but over the past five yearsshe was successfully managed on a combination ofNSAIDs and penicillamine. On further questioning, shecomplained of a slightly reduced appetite, lethargy andbreathlessness on trying to negotiate inclines, and swollenankles. There was no other past medical history. She wasa non-smoker. She was married and her husband workedas an architect.

On examination, the appearance of the eye was asshown (320a). The anterior chamber was clear and thepupils reacted normally to light. Visual acuity was 6/6 inthe left eye, and 6/12 in the right eye. Examination ofthe fundi was normal. Examination of her hands and feetrevealed evidence of a generalized, symmetrical deform -ing arthropathy. The heart rate was 90 beats/min andregular. The blood pressure was 140/90 mmHg. Thejugular veins were not distended. The heart sounds werenormal. Auscultation of the lung fields revealed fineinspiratory crackles at both bases, which persisted aftercoughing. On examination of the abdomen, there was apalpable spleen 3 cm below the costal margin. The liverwas not palpable, and there was no ascites. Examinationof the lower limbs revealed multiple bruises and pittingankle oedema to the shins.

Investigations are shown.

Hb 7.4 g/dlWCC 3 � 109/lPlatelets 45 � 109/lMCV 78 flESR 56 mm/hSodium 134 mmol/lPotassium 4.4 mmol/lUrea 13 mmol/lCreatinine 168 μmol/lCalcium 2.0 mmol/lPhosphate 1.2 mmol/lAlkaline phosphatase 200 iu/lAlbumin 29 g/lIgM 2.0 g/l IgG 19 g/l IgA 1.9 g/l Immunoelectro- Increased gamma-

phoresis globulin bandNo paraproteinaemiaRheumatoid factor positive

1 in 460Chest X-ray NormalUrine dipstick Protein ++

Blood 0

1. What is the most probable cause of the painful eye?2. Give two possible explanations for the FBC.3. List two tests you would perform to ascertain the

cause of the abnormal FBC.4. List two possible causes for the ankle swelling?5. What is the most likely cause of the breathlessness?6. Which two investigations would you perform to

ascertain the cause of the breathlessness?

332200aa

Page 319: Rapid review of clinical medicine for mrcp part 2

318

Answer 320

This woman with long-standing rheumatoid arthritispresents with a second attack of unilateral painful red eye,which has previously required systemic steroids. Themost probable diagnosis is scleritis. Other causes of a redeye in rheumatoid arthritis include episcleritis andkeratitis secondary to Sjögren’s syndrome. The former ispainless, and the latter is bilateral. It is important to notethat the only definitive method of differentiating scleritisfrom episcleritis without a history is by slit-lampexamination of the eye. Recurrent episodes of scleritismay result in perforation of the sclera (scleromalaciaperforans) (320b).

In addition to the scleritis, she has other problemscommonly seen in patients with rheumatoid arthritis whichinclude anaemia, Felty’s syndrome (which itself may beresponsible for the anaemia), renal and respiratoryinvolvement. Drugs used in the treatment of rheumatoidarthritis may themselves cause many of the complicationsseen in rheumatoid arthritis, and a knowledge of the drugsused – and their complications – is essential in themanagement of a patient with rheumatoid arthritis.

The anaemia is part of pancytopenia which, in the contextof this question, is due to either penicillamine-inducedmarrow aplasia or hypersplenism resulting in Felty’ssyndrome. The former may be diagnosed by histologicalexamination of the marrow, and the latter by demonstratingincreased red cell uptake and destruction by the spleen.

The low albumin, swollen ankles and proteinuria areindicative of nephrotic syndrome, which may be due to apenicillamine-induced membranous glomerulonephritis orrenal amyloid complicating chronic inflammation. Othercauses of renal involvement in patients with rheumatoidarthritis are tabulated below. The fine inspiratory crackles atboth lung bases probably represent interstitial lung disease,and require investigation with formal spirometry and KCO.Early interstitial fibrosis may not be identified on a chest X-ray, but respiratory function tests will reveal reduced lungvolumes and reduced KCO. The haematological, ocular,renal, and respiratory complications of rheumatoid arthritisare tabulated (Tables A –D).

Table A Causes of anaemia in rheumatoidarthritis

• Iron deficiency secondary to NSAIDs• Marrow suppression from drugs such as

penicillamine and gold• Hypersplenism seen as part of Felty’s syndrome• Anaemia of chronic disease• Folate deficiency resulting from increased cell

turnover in chronic inflammation• Auto-immune haemolysis• Associated pernicious anaemia

Table B Causes of ocular involvement inrheumatoid arthritis

• Episcleritis/scleritis/scleromalacia• Sjögren’s syndrome• Cataracts secondary to steroid therapy• Retinopathy secondary to chloroquine• Extraocular muscle tendon synovitis• Extraocular muscle paralysis secondary to

penicillamine-induced myaesthenia• Extraocular muscle paralysis secondary to a

mononeuritis multiplex

1. Scleritis.2. i. Penicillamine-induced marrow suppression.

ii. Hypersplenism (Felty’s syndrome).3. i. Bone marrow aspirate/trephine.

ii. Red cell sequestration study.4. i. Penicillamine-induced membranous

glomerulonephritis and secondary nephroticsyndrome.

ii. Renal amyloidosis.5. Interstitial lung fibrosis or fibrotic lung disease.6. Formal spirometry and transfer factor.

332200bb

Page 320: Rapid review of clinical medicine for mrcp part 2

Clinical Cases 319

Question 321

An 11-year-old male was referred to the localpaediatrician after his mother complained that he wasunusually short for his age. His appetite was normal, andthere was no history of recent diarrhoea or weight loss.He was performing well at school and enjoyed a relativelyactive life. His mother was short, measuring 1.48 m, buthis father measured 1.81 m. There were no siblings.

On examination, the boy measured 0.97 m. There wasno evidence of pallor, cyanosis or oedema. The heartsounds were normal and the chest was clear. Theabdomen was soft, and there was no palpableorganomegaly. The lower limbs appeared bowed.Neurological examination was normal.

Investigations are shown.

Table C Causes of renal involvement in rheumatoid arthritis

• Acute tubulointerstitial nephritis secondary to NSAIDs• Renal papillary necrosis secondary to analgesic abuse• Renal amyloid• Membranous nephropathy secondary to gold or penicillamine• Minimal change glomerulonephritis complicating NSAIDs

Table D Causes of respiratory involvement in rheumatoid arthritis

• Cricoarytenitis • Pneumonitis• Pleural effusion • Pulmonary nodules• Pulmonary fibrosis • Caplan’s syndrome (nodules plus progressive• Bronchiolitis obliterans massive fibrosis in coal worker’s disease)

Hb 11 g/dlWCC 5 � 109/lPlatelets 180 � 109/lSodium 135 mmol/lPotassium 4 mmol/lUrea 4 mmol/lCalcium 2.2 mmol/lPhosphate 0.3 mmol/lAlbumin 39 g/lAlkaline phosphatase 500 iu/lX-ray of knees (321)

1. What is the abnormality on the X-ray?2. What is the diagnosis?3. What tests would you perform to confirm the

diagnosis?4. What is the management?

332211

Page 321: Rapid review of clinical medicine for mrcp part 2

320

Answer 321

Both the boy and his mother are short. There isradiological evidence of rickets and ahypophosphataemia, but the serum calcium level isnormal. The findings are consistent with the diagnosis ofhypophosphataemic rickets or vitamin D resistant rickets.The condition is inherited as an X-linked dominantcondition. The fundamental abnormality is failure of

phosphate reabsorption by the kidney. The diagnosis isconfirmed by the demonstration of a high urinaryphosphate excretion rate.

Unlike vitamin D dependent rickets, myopathy doesnot occur. The main features of the condition are shortstature and radiological evidence of rickets. Ossificationof the ligamentum flavum is recognized and may causeparaplegia. The serum calcium and active metabolites ofvitamin D are usually normal.

Correction of the disorder is by a combination ofhigh-dose vitamin D and oral phosphate supplements.Vitamin D alone aids resolution of biochemical rickets,but does not promote growth of height. Untreatedpatients are short and rarely measure more than 1.53 m.

Other causes of renal rickets are proximal RTA andFanconi’s syndrome (Table A). Vitamin D dependentrickets is very rare. Two forms are recognized, thecharacteristics of which are listed (Table B).

Table A Causes of rickets and osteomalacia

• Vitamin D deficiency due to poor nutrition, malabsorption• Chronic renal failure (failure to synthesize 1α-hydroxycholecalciferol)• Renal tubular disorders (see above)• Anticonvulsants (rapid metabolism of 25-hydroxycholecalciferol to inactive derivatives)• Tumours (certain mesenchymal tumours; cause unknown; associated profound myopathy)• Vitamin D dependent rickets (Table B)

1. Splaying of the metaphyses and widening of theepiphyseal plate, indicating rickets.

2. Hypophosphataemic rickets; X-linked dominanthypophophataemic rickets; vitamin D resistantrickets.

3. i. Urinary phosphate excretion (elevated).ii. Renal tubular reabsorption rate of phosphate.

4. High-dose vitamin D and oral phosphatesupplements.

Table B Characteristics of vitamin D dependent rickets

Type 1 Type 2

Mode of inheritance Autosomal recessive Autosomal recessive

Defect Reduced Resistance to 1α-hydroxycholecalciferol 1,25-dihydroxycholecalciferol

1,25-dihydroxycholecalciferol level Reduced Elevated

Myopathy Present Present

Associations Alopecia

Treatment Very high doses of Very high doses of 1,25-dihydroxycholecalciferol 1,25-dihydroxycholecalciferol

and i.v. calciumSpontaneous resolution with advancing age is recognized

Page 322: Rapid review of clinical medicine for mrcp part 2

Clinical Cases 321

Question 322

A 25-year-old homosexual male presented with anepileptic seizure.

Investigations are shown.

Question 323

A 57-year-old male was admitted to the coronary care unitwith a two-hour history of crushing central chest painradiating to his jaw. On examination, he was sweating.His heart rate was 90 beats/min, and regular, and bloodpressure was 160/90 mmHg. He had a 12-lead ECG(323), after which he was given recombinant tissueplasminogen activator. Two hours later he complained offeeling dizzy and unwell. On examination, he was pale,but not cyanosed. His heart rate was 110 beats/min, andregular. His systolic blood pressure was 80 mmHg. TheJVP was elevated 7 cm above the sternal angle. The heartsounds were normal and the chest was clear.

Investigations are shown.

Sodium 136 mmol/lPotassium 4.0 mmol/lUrea 8 mmol/lCreatinine 90 μmol/lGlucose 6 mmol/l

Hb 13 g/dlWCC 17 � 109/lPlatelets 159 � 109/lChest X-ray Normal

1. List at least three possible diagnoses.2. List three investigations you would perform on the

CSF to help make the diagnosis.

1. What is the ECG diagnosis?a. Acute anterior myocardial infarction.b. Acute inferior myocardial infarction.c. Acute inferior myocardial infarction complicated

by third degree atrioventricular block.d. Acute posterior myocardial infarction.e. Acute posterolateral myocardial infarction.

2. What is the cause of his symptoms two hours after thrombolysis?a. Complete heart block complicating myocardial

infarction.b. Acute pulmonary embolus.c. Acute mitral regurgitation following papillary

muscle rupture.d. Acute VSD complicating myocardial infarction.e. Right ventricular hypokinesia.

CD4 count 300 � 106/lViral load 1,000 copies/mlCT scan NormalCSF pressure 10 cmH2OCSF analysis:

White cells 580/mm3 (90% lymphocytes)Red cells 3/mm3

Protein 0.9 g/lGlucose 1.5 mmol/lGram stain NegativeZiehl–Neelsen Negative

stainBlood glucose 5 mmol/l

332233

Page 323: Rapid review of clinical medicine for mrcp part 2

322

Answer 322

This homosexual male with a low CD4 count presentswith an epileptic seizure. The CD4 count suggests thathe has advanced HIV infection. The CSF reveals alymphocytosis, high protein and a sugar concentrationless than half of the blood sugar. The differentialdiagnosis of these CSF findings is listed (Table). The

normal CT scan is against the diagnosis of a cerebralabscess, and metastatic brain disease; however, all theother diagnoses listed are possible.

Causes of CSF lymphocytosis, raised CSFprotein and CSF sugar less than half of bloodsugar

• Tuberculous meningitis• Brucellosis*• Fungal meningitis• Cerebral lymphoma• Leukaemic infiltration into the brain• Metastatic brain disease• Cerebral abscess

*This may be similar to that seen in viralmeningitis.

Answer 323

The ECG on admission reveals ST-segment elevation ofmore than 2 mm in leads I and aVl. This is consistentwith a limited lateral myocardial infarction. The J-pointin leads II and V3–V6 is elevated, but there is nopathological ST-segment elevation in these leads. Thereis also evidence of a dominant R-wave in leads V1–V3which, in the context of the history, represents posteriorextension of the infarct. Posterior infarction gives theopposite appearance of an acute anterior infarction;therefore instead of Q-waves (which would be seen in thecase of an anterior infarction) there are prominent R-waves. The full diagnosis is acute posterolateralmyocardial infarction.

There are several possible causes of hypotension in thisman; however the only real possibility in this clinicalscenario is right ventricular infarction and secondary rightventricular hypokinesia. This is a recognized complicationof inferior and posterior myocardial infarction, and ischaracterized by a reduced cardiac output from the rightheart causing hypotension and a raised JVP. Treatment is

with a careful fluid challenge of 100 ml at a time toincrease the PCWP to approximately 10 cmH2O, whichshould be monitored using a Swan–Ganz catheter.Haemorrhage is unlikely with a raised JVP unless thepatient has bled into his pericardial space. However, inthis rare event one would expect an increased heart sizeon the chest X-ray.

While allergic reactions are relatively common withnon-human thrombolytic agents such as streptokinase,they are rare with recombinant human tissueplasminogen activator. In any case, allergic shock occursduring infusion of the drug rather than two hoursafterwards. Although pulmonary embolism is arecognized complication of myocardial infarction andcould explain the raised JVP, low blood pressure andnormal chest X-ray, it does not occur for at least threedays after myocardial infarction. Both papillary musclerupture and ventricular septal perforation, which usuallyoccur 72 hours following myocardial infarction, areusually associated with an acute fall in cardiac output andpulmonary oedema. Complete heart block may causehypotension, but is most unlikely in a patient with a heartrate of 110 beats/min.

1. i. Tuberculous meningitis.ii. Cryptococcal meningitis.iii. Listeria meningitis.iv. Cerebral lymphoma or leukaemia.v. Brucellosis.

2. i. Ziehl–Neelsen stain or PCR on the CSF for tuberculous meningitis.

ii. Indian ink stain on CSF to identify Cryptococcus neoformans.

iii. CSF analysis by the cytology department to look for malignant cells.

1. e. Acute posterolateral myocardial infarction.2. e. Right ventricular hypokinesia.

Page 324: Rapid review of clinical medicine for mrcp part 2

Clinical Cases 323

A 5-year-old male was admitted with a one-week historyof fever, general malaise and pain in his hands and feet.Two weeks previously he had had a bout of tonsillitiswhich required a course of antibiotics. On the day ofadmission he complained of intermittent chest painradiating into his back and his arms. He was born at 41weeks’ gestation via a normal vaginal delivery. He hadachieved his milestones normally. He was the only child.His father was a businessman, and his mother a teacher.He had been seen by the GP two days earlier anddiagnosed as having a viral illness for which he wasprescribed paracetamol syrup.

On examination, he appeared unwell. He had a feverof 40°C (104°F). His heart rate was 140 beats/min andregular. His lips appeared oedematous, and examinationof his mouth revealed a white-coated tongue withprominent papillae. On examination of the neck therewas widespread, tender, cervical lymphadenopathy.Examination of the pharynx and tonsillar area wasnormal. The palms of his hands and feet were red, andthere was evidence of early desquamation of the skin onthe palmar surface of both hands. There was tendernessover the metacarpophalangeal joints of both hands. The

interphalangeal joints in both feet were tender.Cardiovascular and respiratory examination was normal.

Investigations are shown.

Hb 10.5 g/dlWCC 15 � 109/lPlatelets 490 � 109/lESR 90 mm/hCRP 80 g/lBiochemistry NormalBlood cultures NormalChest X-ray NormalECG Sinus tachycardia

Inverted T- waves V1–V6

Question 325

A 33-year-old single male was admitted with a one-weekhistory of fever, sore throat, a maculopapular rash on thetrunk and cervical lymphadenopathy. His only pastmedical history consisted of treatment for urethritis oneyear previously following a trip abroad. His most recent

sexual contact was eight weeks ago. Four monthspreviously, he expressed the wish for HIV testing, whichwas negative.

Investigations are shown.

Hb 11 g/dlWCC 6 � 109/lPlatelets 180 � 109/lBlood film (325)Paul Bunnell test NegativeCMV titre Increased IgG at a titre of 1/16Toxoplasma titres NegativeVDRL NegativeFTA IgG PositiveHep B sAg AbsentHCV Ab Absent

1. What is the diagnosis?2. What other important investigation would you

perform?3. List two important therapeutic steps in his

management.

1. What is the cause of this patient’s symptoms?2. What further definitive investigation is required?

Question 324

332255

Page 325: Rapid review of clinical medicine for mrcp part 2

324

The combination of mucocutaneous, rheumato -logical and dermatological involvement in a youngboy shortly after a sore throat could be due toscarlet fever, Stevens–Johnson syndromecomplicating streptococcal infection, anidiosyncratic reaction to antibiotics, or a systemicreaction to a general viral illness. The rash is nottypical of Stevens–Johnson syndrome. Thedescription of the tongue and the generalizederythema fit well with scarlet fever; however, theadditional chest pain and abnormal ECG should raise thepossibility of Kawasaki’s disease.

Kawasaki’s disease has been very topical, and it is onlya matter of time before it appears in generalexaminations. It is an acute multi-system vasculitis whichaffects medium-sized arteries. It appears to be endemic inJapan, where there have been more than 100,000 casessince 1991. In contrast, the incidence in Britain is 35 permillion. The disorder is thought to be due to a severeimmune reaction following streptococcal orstaphylococcal infections, although other organisms havealso been implicated. The condition is almost confined tochildren below the age of six years.

Characteristic features include:• Fever for more than five days.• Erythematous palms and soles followed by

desquamation of skin from these areas.• Conjunctival congestion.• Strawberry tongue and swollen lips.• Polymorphous rash.• Non-purulent cervical lymphadenopathy.

Vasculitis commonly affects the coronary arteries causingmicro-aneurysms, coronary thrombosis and myocardialinfarction. Death is usually from cardiac failure following

infarction. Features of cardiac involvement includetachycardia, prolonged PR interval, ST-segment elevationand T-wave inversion.

Other organs which may be affected include the lungs(pulmonary infiltrates), the brain (convulsions, cranial nervepalsies, deafness, CVA) and the gastrointestinal tract(abdominal pain from ischaemia). The small joints of thebody may be affected in approximately 30% of cases.

The diagnosis is clinical. There are no specific tests,but inflammatory markers such as the ESR, CRP andplatelets are elevated. The WCC is also elevated, as arethe immunoglobulins. The haemoglobin may fall rapidly.Coronary involvement may be detected non-invasively byviewing the origins of the coronaries by echo -cardiography (324). This is not difficult in youngchildren because they are thin.

The mainstay of treatment is oral aspirin andintravenous Sandoglobulin (c globulin). These reduce themortality of the condition from more than 30% to lessthan 1%.

Differential diagnosis of Kawasaki’s disease isstreptococcal infection (scarlet fever), staphylococcalinfection (scalded skin or toxic shock syndrome),Stevens–Johnson syndrome, juvenile chronic arthritis,leptospirosis, rickettsial infection and a generalized drugreaction.

Answer 325

HIV seroconversion occurs 2–6 weeks after HIVinfection. The illness resembles infectious mononucleosis,comprising fever, pharyngitis, lymphadenopathy and a

rash. The patient may have a mild anaemia, relativelymphopenia and thrombocytopenia. The blood film mayreveal atypical lymphocytes (325). Other differentialdiagnoses include CMV infection, Toxoplasma infectionand secondary syphilis.

Although secondary syphilis produces an identicalsyndrome to HIV seroconversion, the syphilis serology is

1. Kawasaki’s disease.2. Transthoracic echocardiography to visualize

the coronary arteries on the short-axis viewsof the aorta.

3. i. Oral aspirin.ii. Intravenous human immunoglobulin.

1. The diagnosis is HIV seroconversion.2. Repeat HIV antibody test.

2

3

1

Key: 1 right coronary artery 2 Aortic root 3 left coronary artery

Answer 324

332244

Page 326: Rapid review of clinical medicine for mrcp part 2

Clinical Cases 325

suggestive of previous syphilis infection rather thansecondary syphilis. In secondary syphilis, reaginantibodies become positive in almost 100% of cases, as dothe more specific antibodies such as the FTA and theTPHA. However, the VDRL test becomes negative inover one-third of patients following therapy or in tertiaryand quaternary syphilis, whereas the more specificantibodies persist in almost every patient, even aftersuccessful treatment (Table). In this case, the negativeVDRL in a patient with a pharyngitis and a rash is againstsecondary syphilis, the more likely diagnosis being HIV

seroconversion. Persistent FTA suggests previous syphilisinfection.

HIV IgG antibodies are detectable six weeks afterinfection with the virus. The detection of HIV antibodyusing a Western blot technique or ELISA is the mostwidely performed test if HIV infection is suspected. In thecase of a negative HIV antibody test (usually in very earlyinfection), the diagnosis may be confirmed by measuringHIV p24 antigen, a viral capsid protein which is detectablewithin 24 hours of the onset of illness. Alternatively, PCRmethods may be used to detect viral RNA.

Positive syphilis serology in relation to the various stages of syphilis

Test Primary syphilis (%) Secondary syphilis (%) Tertiary syphilis (%)VDRL 70 99 77FTA 55 100 96TPHA 80 99 95

Question 326

A 39-year-old male presents with a three-month historyof lower back pain. Over the past two years he hasexperienced three episodes of severe right lower quadrantpain.

Investigations are shown.

Sodium 138 mmol/lPotassium 3.5 mmol/lChloride 119 mmol/lBicarbonate 13 mmol/lUrea 7 mmol/lCreatinine 112 μmol/lCalcium 2.1 mmol/lPhosphate 0.7 mmol/l

1. What is the diagnosis?2. What single investigation would you perform to

confirm the diagnosis?3. What is the cause of his back pain?4. List two simple investigations that may help to

confirm the cause of his back pain.5. What is the cause of his abdominal pain?6. List two radiological tests that may help to confirm

the cause of his abdominal pain.

Page 327: Rapid review of clinical medicine for mrcp part 2

326

Answer 326

The biochemical results reveal a hypokalaemic,hyperchloraemic acidosis. In addition, there are lowcalcium and phosphate levels. These are features of RTA,which is characterized by abnormalities in urinaryacidification. RTA may be divided into type 1 (distalRTA) and type 2 (proximal RTA). The causes andassociations of renal tubular acidosis are tabulated below.Distal RTA is due to defective excretion of H+ in thedistal convoluted tubule, and proximal RTA is due tofailure of H+ secretion by the proximal convoluted tubule.In both cases there is hypokalaemia and hypercalcuria.Hypokalaemia can be explained as follows.

In distal RTA the defective H+ excretion means thatthe absorption of sodium requires increased potassiumexcretion. In proximal RTA there is excessive bicarbonateloss, which in turn results in excessive sodium loss. Theresulting hypotension activates the renin–angiotensin–aldosterone system to aid sodium and waterretention, but at the expense of potassium loss.Hypercalcuria is more common in distal RTA.Osteomalacia is common to both, but nephrocalcinosisand nephrolithiasis are confined to distal RTA.

Patients with distal RTA may present at any age. Inchildren, the condition presents with failure to thrive. Inadults, it presents with renal colic or muscle weaknessfrom hypokalaemia. Patients with proximal RTA presentwith symptomatic acidosis, polydipsia, polyuria,hypokalaemic myopathy and rickets or osteomalacia. Thediagnosis should be suspected if an individual has ahyperchloraemic acidosis, and can be confirmed bydemonstrating an early morning urinary pH of >5.5. Insome individuals in whom the diagnosis is suspected, butthere is no evidence of systemic acidosis, an acid load testusing ammonium chloride is required. This involvesmeasuring urinary pH on samples collected hourly for 6hours after the administration of intravenous ammoniumchloride. Failure to acidify the urinary pH below 5.5indicates RTA.

Treatment is with sodium bicarbonate and potassiumcitrate. The latter reduces calcium excretion and providessupplementary potassium.

Causes of RTA

Distal (Type 1) Proximal (Type 2)Inherited Inherited

Part of Fanconi’s syndrome

Acquired AcquiredPrimary biliary cirrhosis HyperparathyroidismCAH Heavy metal poisoning (lead, arsenic)Obstructive nephropathy MyelomaVitamin D intoxication AcetazolamideTetracycline

1. Distal (type I) RTA.2. Early morning urinary pH measurement or

measurement of urine pH after ammoniumchloride infusion.

3. Osteomalacia.4. i. X-ray of the lumbar spine may reveal osteopenia

or Looser’s zones in osteomalacia.ii. Serum alkaline phosphatase level, which is

characteristically raised in osteomalacia.5. Renal colic.6. i. Plain abdominal X-ray may identify a radio-

opaque renal stone.ii. Intravenous urography is the investigation of

choice to identify stones in the renal tract.

Page 328: Rapid review of clinical medicine for mrcp part 2

Clinical Cases 327

A 64-year-old female was admitted to hospital with athree-month history of increasing breathlessness andswollen ankles. She had been well until two-and-a-halfyears ago, when she experienced sudden onset ofbreathlessness while walking. She was observed in hospitalovernight but was much improved the following day andallowed home. She had a further similar episode fourmonths later, and then again a year later. On each occasionshe improved within 24–48 hours. In between the twomost recent episodes she had noticed that she was unableto exert herself as much without becoming breathless.There was no history of chest pain or a cough. Physicalexamination performed by the senior house officers duringeach hospital admission had been normal, with theexception of a sinus tachycardia of 110 beats/min. A chestX-ray performed on the admission one year ago is shown(327a). The patient had a past medical history ofhysterectomy aged 43 years for menorrhagia due to uterinefibroids. Three years ago she had surgery for varicose veinsof the lower limbs. She was a non-smoker. She was nottaking any medication. She was a receptionist in a doctor’ssurgery for 20 years before retiring 20 years ago. She wasmarried and lived with her husband. The couple had a dogand a parrot as pets.

On examination, she was breathless at rest andcyanosed. There was no clubbing. She had marked pittinglower-limb oedema up to her thighs. The heart rate was102 beats/min and blood pressure 105/60 mmHg. TheJVP was elevated to the level of the mastoid. Closerinspection revealed prominent c-v waves. On examinationof the precordium, the apex was displaced to the anterioraxillary line. There was a marked left parasternal heave. Onauscultation of the precordium there was a soft systolicmurmur and a loud fourth heart sound at the left lowersternal edge. The chest was clear. Abdominal examinationdemonstrated ascites and a pulsatile liver edge 5 cm belowthe costal margin. The ECG performed on admission isshown (327b).

Respiratory function tests performed on this admissionare shown.

Actual PredictedFEV1 (l/min) 0.94 1.26–2.94FVC (l/min) 1.12 1.82–3.81TLC (l) 2.1 4.38–6.8Transfer factor 3.4 4.2–8.8Corrected transfer factor 0.4 0.7–1.62

What investigation is most likely to provide thediagnosis?

a. Echocardiography.b. CT pulmonary angiography.c. High-resolution CT scan of the lungs.d. Arterial blood gases.e. Right heart catheter plus pulmonary

angiography.

Question 327

332277aa

332277bb

Page 329: Rapid review of clinical medicine for mrcp part 2

328

The patient has a history of progressive breathlessness,and now presents with ECG evidence of right ventricularhypertrophy and a clinical picture of severe right heartfailure. Lung function tests demonstrate a restrictivedefect and a low transfer factor. She has cor pulmonalecomplicating multiple pulmonary emboli, extrinsicallergic alveolitis, or CFA. The latter two diagnoses areunlikely in the absence of chest signs on three hospitaladmissions and the normal chest X-ray one yearpreviously. In contrast the chest X-ray in multiplepulmonary emboli may appear relatively normal, the onlyevidence of pulmonary emboli being oligaemic lungfields. Another possible diagnosis is right heart failure dueto chronic pulmonary hypertension from an ostiumsecundum ASD; however, the chest X-ray in this

condition would reveal cardiomegaly. Investigations toconfirm the diagnosis of multiple pulmonary embolishould include arterial blood gases (which will show lowPaO2 and PaCO2 due to severe ventilation/perfusionmismatch) and a ventilation–perfusion scan todemonstrate perfusion defects. A right heart cathetercombined with pulmonary angiography would providethe most diagnostic information and would be useful inassessing the severity of pulmonary hypertension. SpiralCT scan of the thorax is an increasingly popular methodof identifying peripheral pulmonary emboli, which maybe difficult to detect with angiography. Echo -cardiography is useful in excluding an ASD with shuntreversal or tight mitral stenosis. Echo-Doppler may beuseful in quantifying pulmonary artery pressure; however,it has no direct role in the diagnosis of multiplepulmonary emboli.

A 42-year-old, insulin-dependent diabetic wasinvestigated further after he was noted to have a positivedipstick for protein on urine analysis. He was diagnosedwith diabetes at the age of 23 years after presenting withketoacidosis. Initial management with porcine insulin wasgood; however, he was switched to human insulin 10years ago because of severe lipoatrophy at sites ofinjection. Over the past three years blood glucose controlhad been suboptimal, and the dose of insulin had beenincreased on five occasions. He was married with twochildren. He owned a grocer’s shop which was open

seven days a week and was staffed by the patient and hiswife at all times. He smoked five cigarettes per day. Hedid not consume alcohol.

On examination, he had bilateral corneal arcus. Hisheart rate was 80 beats/min and regular; blood pressurewas 150/90 mmHg. There was no evidence of cardiacenlargement or cardiac failure. The peripheral pulses wereall palpable, and there were no audible bruits over thecarotid arteries or in the abdomen. Examination of hisfundi is shown (328a).

Investigations are shown below.

Hb 12.8 g/dlWCC 6 � 109/lPlatelets 212 � 109/lMCV 89 flSodium 136 mmol/lPotassium 3.4 mmol/lUrea 6.8 mmol/lCreatinine 117 μmol/lGlucose 11 mmol/l

Glycosylated Hb 9% (normal range 3.8–6.4%)Cholesterol 8.2 mmol/lTriglycerides 8 mmol/l24-hour urinalysis Volume 2.7 l

Protein 0.7 g/lRenal US Two kidneys, 12 cm each

Normal parenchymal texture

Renal biopsy (328b [haematoxylin and eosin stain])

e. Right heart catheter plus pulmonary angiography.

Answer 327

Question 328

Page 330: Rapid review of clinical medicine for mrcp part 2

Clinical Cases 329

The patient was commenced on captopril, the dose ofwhich was gradually increased to 50 mg, three timesdaily. Three months later, the patient was admitted withvague abdominal discomfort, nausea and anorexia. Onexamination, he appeared euvolaemic. The bloodpressure was 120/65 mmHg.

Investigations are shown.Another investigation was performed, which is shown

(328c).

Question 329

A 16-year-old female was brought into the Accident andEmergency Department with shortness of breath.

Blood gases on inspiring room air are shown.

pH 7.49PaCO2 2.4 kPaPaO2 15.5 kPaBicarbonate 12 mmol/lO2 saturation 96%

Sodium 133 mmol/lPotassium 6.4 mmol/lUrea 32 mmol/lCreatinine 500 μmol/lGlucose 10 mmol/l

1. What is demonstrated on the examination of thefundi?

2. What is shown on the renal biopsy?3. What is the cause of his proteinuria?4. What was the cause of the marked deterioration in

renal function?5. List six management strategies to help ameliorate

the renal function and the rate of futuredeterioration in renal function.

1. What is the acid–base disturbance?2. What is the underlying cause?3. What is the management?

332288aa 332288bb

332288cc

Page 331: Rapid review of clinical medicine for mrcp part 2

330

Overt diabetic nephropathy is based on thedemonstration of over 0.5 mg of protein on a 24-hoururinary collection. A protein content of this level is easilydetected on a urine dipstick. More recently, it has beennoted that some patients pass small amounts of albuminin the urine (0.02–0.3 mg/24 h) before the developmentof a positive dipstick test. Such patients are said to have‘micro-albuminuria’ or incipient diabetic nephropathy.

The prevalence of diabetic nephropathy in the UK is4–8%. The pathological basis for this is diabeticglomerulosclerosis, as shown in the renal biopsy (328b).

Patients with diabetic nephropathy have an increasedmortality from cardiovascular and renal causes. Poorglycaemic control, raised blood pressure and smokingexpedite the development and progression ofnephropathy. In patients with early nephropathy the GFRis increased, and the associated hyperinfiltration isthought to damage glomeruli by increasing theintraglomerular capillary blood pressure. Patients withnephropathy very often also have retinopathy, and viceversa.

The management includes good glycaemic control,maintaining blood pressure below 135/85 mmHg (thisslows the rate of fall in the GFR rate by 60%). ACEinhibitors reduce albumin excretion in patients withmicro-albuminuria. In diabetic rats, ACE inhibitorsreduce intraglomerular capillary pressure and prevent adecline in GFR. Whether they have the same effect inhumans is currently under investigation. Renal functionshould be observed carefully after the initiation of anACE inhibitor because they may precipitate renal failurein patients with silent renal artery stenosis, as in this case.Renal failure is precipitated by reduction in the pressurewithin the afferent arteriole, which causes a significantdecrease in glomerular perfusion pressure in the presenceof renal artery stenosis (328c).

Answer 329There is a low PaCO2 and a high pH, suggesting arespiratory alkalosis. The serum bicarbonate is very low,indicating a coexistent metabolic acidosis. In chronicrespiratory alkalosis there would be increased excretion ofbicarbonate ions in an attempt to lower the arterial pH;however, the bicarbonate concentration is only modestlyreduced. (See Acid-base Disturbance, page 426.)

1. Small retinal haemorrhages, and micro-aneurysmsconsistent with background diabetic retinopathy.

2. There is evidence of eosin staining (pink) fibrotic(hyaline) nodules characteristic of diabeticglomerulosclerosis, sometimes known asKimmelsteil–Wilson nodules.

3. Diabetic nephropathy. Other causes of protein inthe urine in diabetics include a urinary tractinfection.

4. Angiotensin-converting enzyme inhibitor-inducedglomerular hypoperfusion in a patient with bilateralrenal artery stenosis.

5. i. Stop ACE inhibitor.ii. Bilateral renal angioplasty.iii. Improve diabetic control.iv. Control hyperlipidaemia.v. Ensure blood pressure below 140/85 mmHg.vi. Advise against smoking.

1. There is a respiratory alkalosis and a metabolicacidosis.

2. Aspirin toxicity (see also Answer 172).3. The management is dependent on the serum

salicylate level. Levels below 500 mg/l are treatedwith intravenous saline to promote renal clearance.Levels between 500 and 750 mg/l are treated withintravenous bicarbonate but may require forcedalkaline diuresis on a high-dependency unit. Levelsbetween 750 and 1,000 mg/l are best treated withhaemodialysis.

Answer 328

Page 332: Rapid review of clinical medicine for mrcp part 2

Clinical Cases 331

Question 330

A 46-year-old garage mechanic was investigated in amedical clinic for weight loss. Six months previously hehad commenced a voluntary diet to reduce weight. Helost 4 kg over two months, following which he stoppedhis diet. Despite a normal appetite he continued to loseanother 4 kg in weight over the next four months. Hehad also noticed that he tired easily, and was finding itdifficult to perform heavy manual work as part of hisprofession. His arms felt weak. He was breathless onstrenuous effort and found heavy lifting increasinglydifficult. More recently, he had developed lower backpain which was worse when he sat upright and wasassociated with pain and numbness of his left calf and leftfoot. The pain was worsened by movement and he wasforced to stop working. His appetite remained good. Hehad moved his bowels twice daily ever since he couldremember. The stool was of normal consistency and didnot contain blood. He was married with three children.He stopped smoking 15 years ago after an attack ofbronchitis. He consumed 2–3 units of alcohol per week.

On examination, he was of relative heavy build. Therewas no evidence of pallor or clubbing. The heart rate was102 beats/min and irregularly irregular. The bloodpressure was 130/80 mmHg. The JVP was not raised.On examination of the upper limbs there was no muscletenderness, but there was wasting of the proximalmuscles of the upper limb with weakness of abductionand adduction. The upper limb reflexes were brisk.Sensory testing was normal. On examination of the lowerlimbs, there was wasting of the muscles of the thighs.Power testing demonstrated weakness of plantar flexionon the left side. Reflexes were brisk with the exception ofthe left ankle jerk, which was absent. Plantar responses

were normal. Sensory testing demonstrated reduced pin-prick sensation affecting the anterolateral aspect of theleft foot.

Investigations are shown.

Hb 14 g/dlWCC 6 � 109/lPlatelets 210 � 109/lESR 10 mm/hChest X-ray NormalSodium 135 mmol/lPotassium 3.4 mmol/lUrea 5 mmol/lCalcium 2.4 mmol/lPhosphate 1 mmol/lAlbumin 40 g/l

Question 331

This ECG (331) is from a 35-year-old male withsudden onset of expressive dysphasia and rightupper-limb weakness. On examination, he wasapyrexial. The heart rate was 100 beats/min andregular; blood pressure was 110/80 mmHg. Hehad a soft systolic murmur. There were nocarotid bruits. He had been well andasymptomatic all his life, and was a non-smoker.

1. What is the cause of the absent ankle jerk?a. Paraneoplastic syndrome.b. Prolapsed intervertebral disc between L4/L5.c. Prolapsed intervertebral disc between L5/S1.d. Peripheral neuropathy.e. Mononeuritis multiplex.

2. What is the diagnosis?.a. Polymyositis.b. Polymyalgia rheumatica.c. Thyrotoxic myopathy.d. Eaton–Lambert syndrome.e. Paraneoplastic syndrome.

1. List three abnormalities.2. What is the most likely cause of his murmur?3. What is the most probable cause of his neurological signs?

333311

Page 333: Rapid review of clinical medicine for mrcp part 2

332

Answer 330

The patient has weight loss, despite a normal appetite.The differential diagnosis for this scenario ismalabsorption, diabetes mellitus, phaeochromocytomaand thyrotoxicosis. He has atrial fibrillation, which ismost consistent with thyrotoxicosis. He also has proximalmuscle wasting of his limbs, a well-known feature ofthyrotoxic myopathy. A paraneoplastic syndrome isunlikely because there is no immediate evidence of anunderlying neoplastic process. For similar reasons, he isunlikely to have Eaton–Lambert syndrome, which is amyasthenic syndrome associated with malignancy.Proximal myopathy is a feature, but reflexes are absent.In contrast to myasthenia, weakness improves withrepetitive muscular stimulation. Both polymyalgia

rheumatica and polymyositis are associated with muscletenderness and a raised ESR, which are absent in thispatient. Furthermore, polymyalgia rheumatica is adisorder that affects the elderly rather than youngpatients.

The chronic back pain worsened by movement andradiating to his left calf is consistent with a lumbarradiculopathy. A lateral disc protrusion is most likely. Anabsent ankle jerk (S1), weakness of dorsiflexion of the leftside, and impaired sensation of the anterolateral aspect ofthe foot (L5 and S1) is consistent with an L5/S1 discprolapse. In the case of an L4/L5 lesion the knee jerkwould be absent. A peripheral neuropathy ormononeuritis multiplex cannot explain the rootsymptoms (back pain radiating to calf and made worsewith movement) or the lack of neuropathy elsewhere (allother reflexes preserved).

Answer 331

The combination of right axis deviation and RBBB in ayoung, previously asymptomatic man with a soft systolicmurmur is suggestive of an underlying ostium secundumASD. Ostium primum defects are associated with left axisdeviation and RBBB. Although both pulmonary stenosisand VSDs can produce similar ECG changes, a softmurmur in a patient with a VSD is suggestive of a largedefect which would almost certainly cause symptoms ofcardiovascular dysfunction before the third decade. Mildpulmonary stenosis is characterized by lack of symptoms

and a soft systolic murmur, heard best in the left secondintercostal space, but this is not usually enough to causeright ventricular hypertrophy. Moreover, it does notexplain the neurological defect in this case, which hasresulted from a paradoxical embolus of thrombus withinthe systemic veins to the brain via a defect in theinteratrial septum. Paradoxical embolism is a rare butwell-recognized complication of all septal defects.Thrombus from the systemic veins may result in embolito the brain, limbs and gastrointestinal tract. Paradoxicalembolus may be the presenting feature in a patient with acardiac septal defect.

The differential diagnosis is a mycotic emboluscomplicating infective endocarditis involving an ASD;however, the patient is not febrile and infectiveendocarditis very rarely involves an ASD.

1. i. Right axis deviation.ii. RBBB.iii. Voltage criteria for right ventricular hypertrophy.

2. Ostium secundum ASD.3. Paradoxical embolus from the systemic veins.

1. c. Prolapsed intervertebral disc between L5/S1.2. c. Thyrotoxic myopathy.

Page 334: Rapid review of clinical medicine for mrcp part 2

Clinical Cases 333

A 22-year-old basketball player presented withsudden onset of severe left-sided chest painradiating into his neck four hours before hewas due to play in a large tournament. He hadnever experienced any previous chest pain,and was extremely fit, according to his teamcoach.

On examination, he was distressed withchest pain. His heart rate was 70 beats/minand blood pressure 190/110 mmHg.Examination of the precordium and the chestwas normal. An ECG was performedimmediately (332), and he was subsequently admitted tothe Coronary Care Unit. The following day he wasasymptomatic and his blood pressure was normal.Examination of his fundi did not demonstrate any fundalchanges of hypertension, and general examination didnot reveal any peripheral stigmata of hyperlipidaemia. Hedenied any previous history of hypertension, diabetesmellitus or a family history of coronary artery disease. Hehad never smoked previously.

Investigations following admission are shown.An echocardiogram performed three days later

demonstrated minor apical hypokinesia, but satisfactoryoverall systolic function. Coronary angiography beforedischarge was entirely normal.

Hb 13 g/dlWCC 12 � 109/lPlatelets 204 � 109/lSodium 139 mmol/lPotassium 3.9 mmol/lGlucose 4.2 mmol/lCreatinine kinase 560 iu/l

Question 333

A 66-year-old male with a long history of breathlessnesspresented with a three-day history of profoundbreathlessness and a cough productive of yellow sputum.

Arterial blood gases are shown.

pH 7.33PaCO2 8.8kPaPaO2 6.5kPaBicarbonate 36 mmol/lBase excess +11

Question 332

1. What important piece of information should beelicited in the history to help ascertain the cause ofhis chest pain?

2. Which drug would you use in the management ofhis hypertension?

What is the acid–base abnormality?a. Acute respiratory acidosis.b. Metabolic alkalosis.c. Partially compensated respiratory acidosis.d. Compensated respiratory acidosis.e. Compensated metabolic alkalosis.

333322

Page 335: Rapid review of clinical medicine for mrcp part 2

334

Answer 332

The clinical scenario is that of an elite athlete presentingwith an acute myocardial infarction but subsequently hehas normal coronary arteries, although theechocardiogram is indicative of myocardial damage. Whilethere are several causes of myocardial ischaemia despitenormal coronary arteries (Table A), the possibility ofcocaine (an amphetamine) abuse should be considered.

Cocaine abuse has reached epidemic proportions insome groups of elite athletes; however, the incidence ofcardiac complications remains low. Cocaine inhibits re-uptake of the catecholamine neurotransmitternoradrenaline, which is a potent vasoconstrictor. By thismechanism it may induce coronary artery spasm andmyocardial infarction. The drug has also been implicatedin causing myocarditis through direct effects on theheart. In addition, its local anaesthetic properties blockthe fast inward sodium and slow calcium currents withinmyocardial cells, thereby predisposing to lethalventricular arrhythmias. Cocaine also prolongs the QT-interval and increases the risk of torsades de pointes-typepolymorphic ventricular tachycardia. Arrhythmias are besttreated by antiarrhythmic agents which do not prolongthe QT-interval. Beta-blockers are contraindicated in themanagement of cocaine-induced cardiac problemsbecause they would result in unopposed alpha activity,causing profound vasoconstriction. Cocaine is a relativelyrare cause of sudden death in young elite athletes.Relatively more common causes are shown (Table B).

Non-cardiac causes of mortality from cocaine includeCVA, status epilepticus and rhabdomyolysis.

Table A Myocardial ischaemia/infarction withnormal coronary arteries

• HCM• Aortic stenosis• Hypertensive heart disease• Coronary arteritis, particularly polyarteritis

nodosa• Coronary artery embolus secondary to

intramural thrombus or vegetation• Severe anaemia• Thyrotoxicosis• Phaeochromocytoma• Cocaine• Amphetamines

Table B Causes of sudden cardiac death inyoung elite athletes

• HCM (almost 50% of all cases)• Coronary artery anomalies• Arrhythmogenic right ventricular

cardiomyopathy• Myocarditis• Mitral valve prolapse• Wolff–Parkinson–White syndrome• LQTS• Drug abuse• Sarcoidosis• Rhabdomyolysis• Heatstroke• Commotio cordis

Answer 333

The high PaCO2 and low pH is diagnostic of respiratoryacidosis. The arterial bicarbonate is high, indicating that

there is increased absorption of bicarbonate ions by thekidney in an attempt to normalize the pH. As the pH isstill slightly lower than normal, this is most accuratelytermed partial compensation. (See Acid–base Disturbance,page 426.)

1. A history of amphetamine abuse.2. A calcium channel blocker such as amlodipine or

verapamil.

c. Partially compensated respiratory acidosis.

Page 336: Rapid review of clinical medicine for mrcp part 2

Clinical Cases 335

A 16-year-old Turkish female was referred to thegynaecological outpatient clinic with a 14-month historyof amenorrhoea. She started menstruating at the age of 13years. She had almost regular periods until the age of 14and after this had three scanty periods and none for thepast year. She was born in Turkey and had only recentlycome to England. At the age of nine years she had beencommenced on hydrocortisone 15 mg in the morning and10 mg in the evening for an illness about which bothparents were very vague. Apart from occasional dizziness

on standing suddenly, and infrequent headaches, therewas no other past history of note. She had an 18-year-oldsister who was well. Her parents were first cousins. Herfather worked as a tailor and her mother was a housewife.None of them spoke much English.

Examination was declined by the patient, but theheight and weight were recorded (1.52 m (under 50thcentile) and 42 kg (less than 3rd centile)), and the bloodpressure measured (90/60 mmHg).

Investigations are shown.

Hb 13 g/dlWCC 7 � 109/lPlatelets 200 � 109/lSodium 138 mmol/lPotassium 4.0 mmol/lUrea 4 mmol/lCreatinine 60 μmol/lBlood sugar 5 mmol/lCalcium 2.4 mmol/lPhosphate 1.0 mmol/lAST 22 iu/lAlkaline phosphatase 179 iu/lThyroxine 18 pmol/lTSH 3.1 mu/lFSH 5 u/l (NR 1–7 iu/l)LH 8 u/l (NR 2–20 iu/l)Oestradiol 90 pmol/l

(NR 40–92 pmol/l)Testosterone 9 nmol/l (NR 1–3 nmol/l)

17-hydroxyprogesterone 23 nmol/l (NR <5 nmol/l)

Androstenedione 16 nmol/l (NR 5–12 nmol/l)

Dihydroepiandrostenedione 21 nmol/l(NR <12 nmol/l)

Recumbent plasma renin 9 pmol/l/ml/h (NR 0.2–2.2 pmol/l/ml/h)

Ambulant plasma renin 12 pmol/l (NR 1.2–4.4 pmol/l)

Prolactin 385 mu/l (NR <450 mu/l)

Urinary pregnantetriol Elevated +++US ovaries Normal-sized ovaries

containing multiple cysts

Small uterus

1. What is the diagnosis?2. Which other drug should this patient be given?3. Suggest at least three possible causes for the amenorrhoea.

A 75-year-old male was investigated for increasingdyspnoea.

Results of investigations are shown.

Chamber Pressure Oxygen (cmH2O) saturation (%)

Right atrium 6 65Right ventricle 25/6 67Pulmonary artery pressure 25/18 67PCWP (mean) 16 Left ventricle 210/18 98Aorta 210/55 98

What sign would you ascertain onauscultation of the precordium?

a. Ejection systolic murmur in the aorticarea.

b. Prominent right ventricular in pulse.c. Fixed splitting of the second heart

sound.d. Long early diastolic murmur at left

lower sternal edge and in the aorticarea.

e. Soft mid-diastolic rumbling murmurin the mitral area.

Question 334

Question 335

Page 337: Rapid review of clinical medicine for mrcp part 2

336

The presenting complaint of amenorrhoea has a wide dif -ferential diagnosis (Table A); however, the endocrine testsare most compatible with the diagnosis of congenitaladrenal hyperplasia. Congenital adrenal hyperplasia may becaused by a number of enzyme defects in the biosyntheticpathway for steroid hormone synthesis within the adrenalgland. The deficiencies are inherited as an autosomal reces -sive trait. The most common abnormality by far is 21-hydroxylase deficiency, the gene for which is on chromo -some 6 (adjacent to the HLA B locus). The enzyme isresponsible for the conversion of 17-hydroxyprogesteroneto cortisol (334).

21-hydroxylase deficiency leads to elevated 17-hydroxyprogesterone, which is metabolized to adrenalandrogens. These ketosteroids (particularly pregnanetetriol)are found in high concentrations in the urine. The patientsare cortisol-deficient, and some may also have aldosteronedeficiency. Patients with severe 21-hydroxylase deficiencypresent in infancy with vomiting and failure to thrive.Ambiguous genitalia are common. Males may have hypo -spadias and females may have fusion of the labia. Males withmild deficiency may present with precocious puberty andfemales with primary amenorrhoea, hirsutism and virilism.Hirsutism occurring before menarche is suggestive of con -genital adrenal hyperplasia. Lack of cortisol is responsiblefor high ACTH levels, which continue to drive thebiosynthetic pathway.

Laboratory investigations reveal an elevated plasma 17-hydroxyprogesterone and ACTH. Urinary pregnanetriol(ketosteroid) levels are also elevated. Treatment is withhydrocortisone replacement. The dose is similar to thatused in Addison’s disease. The therapeutic response is

judged by suppression of the urinary pregnanetriol levels tothe upper limit of normal. Considerable care should betaken not to give a very high dose of hydrocortisone, as thismay stunt growth. Patients with aldosterone deficiency asjudged by an elevated renin level and postural hypotensionshould receive fludrocortisone.

In this patient there are several potential causes ofamenorrhoea. The most obvious cause is her very lowweight (Table A). Another important cause is the raisedtestosterone levels caused by an inadequate dose ofhydrocortisone. The dose should be enough to inhibitACTH secretion and suppress androgen synthesis. In thiscase, the adrenal androgen levels (dihydroepiandro -stenedione) are very high. The patient also has evidence ofpolycystic ovaries, which are themselves a cause of primaryamenorrhoea but may coexist with hyper androgenic statessuch as congenital adrenal hyperplasia and Cushing’ssyndrome. The differential diagnoses of congenital adrenalhyperplasia are shown (Table B).

Differentiating congenital adrenal hyperplasia fromarrhenoblastoma is usually easy because in the formercortisol levels are low and dihydroepiandrosterone sulphate

Table B Differential diagnoses of mild21-hydroxylase deficiency

• Polycystic ovaries• Androgen-secreting adrenal tumour• Arrhenoblastoma• Cushing’s syndrome

Table A Causes of amenorrhoea

• Physiological • Primary ovarian failure (pregnancy) • Congenital adrenal

• Systemic disease hyperplasia(e.g. coeliac disease) • PCOS

• Weight loss • Cushing’s syndrome• Anorexia nervosa • Androgen-secreting • Excessive stress adrenal tumour• Hypopituitarism • Arrhenoblastoma• Prolactinoma

170H-pregnenolone3-βHSD

170H-progesterone21-hydroxylase

11-deoxycortisol11-β hydroxylase

Cortisol

Dehydroepiandrostenedione3-βHSD

AndrostenedionePeripheral tissues

Testosterone

1. Congenital adrenal hyperplasia (21-hydroxylasedeficiency).

2. Fludrocortisone.3. i. Underweight.

ii. Inadequate treatment for congenital adrenal hyperplasia.

iii. Elevated testosterone.iv. Coexistent polycystic ovary syndrome.

Answer 334

333344

Pregnenolone3-βHSD

Progesterone21-hydroxylase

Deoxycorticosterone11-β hydroxylase

Corticosterone18-hydroxylase

180H-corticosterone18-oxidase

Aldosterone

Cholesterol

Desmolase 17-α hydroxylase C-17 lyase

3-βHSD = 3-β hydroxysteroid dehydrogenase

Page 338: Rapid review of clinical medicine for mrcp part 2

Clinical Cases 337

Question 336

A 58-year-old male presented with nausea and pain in hisright upper quadrant. On examination he had tenderhepatomegaly with an irregular edge.

Investigations are shown.

Hb 8 g/dlWCC 18 � 109/lPlatelets 100 � 109/lNeutrophils 68%Normoblasts 8%Myeloblasts 8%Myelocytes 4%Metamyelocytes 6%Lymphocytes 15%

Question 337

A 13-year-old male was admitted to hospital withcomplete loss of vision in his right eye. He had beenplaced in a juvenile offenders’ residential home sixmonths ago for stealing on several occasions. His parentshad separated a few months previously, and before thathe had been living with his father, who was an alcoholabuser, and had three dogs. Two weeks before admissionhe had complained of reduced visual acuity in his righteye, which had progressively become worse.

On examination, he appeared well. He hadgeneralized lymphadenopathy, and was clinically blind inthe right eye. Examination of the fundus in the right eyerevealed two large, raised white lesions situated adjacentto the optic disc and in the macular region. He wasapyrexial and all other physical examinations werenormal.

Investigations are shown.

Hb 11.4 g/dlWCC 6.6 � 109/l (differential below)Platelets 180 � 109/lNeutrophils 4.1 � 109/lLymphocytes 1.2 � 109/lEosinophils 1.1 � 109/lMonocytes 0.11 � 109/lBasophils 0.09 � 109/lESR 25 mm/hBiochemistry Normal

1. What is the cause of his anaemia?a. Chronic myeloid leukaemia.b. Marrow infiltration by malignant cells.c. Aplastic anaemia.d. Myelofibrosis.e. Hydatid disease.

2. What investigation would you perform to confirmthe diagnosis?a. Bone marrow trephine.b. Bone scan.c. Ultrasound of the liver.d. Blood analysis for Philadelphia chromosome.e. Leucocyte alkaline phosphatase level.

1. What is the diagnosis?2. List two investigations useful in confirming the

diagnosis.

is high. Similarly, congenital adrenal hyperplasia can easilybe distinguished from Cushing’s syndrome by measuringserum cortisol and ACTH.

Differentiation from an adrenal androgen-secretingtumour is possible by performing a low-dosedexamethasone suppression test, which will reduce urinarypregnanetriol levels in congenital adrenal hyperplasia, but

not in an adrenal androgen-secreting tumour.The differentiation from PCOS is possible by measur ing

17-hydroxyprogesterone levels and dihydro epiandrosteronesulphate, both of which are high in congenital adrenalhyperplasia. Alternatively, a synacthen test may be per -formed which is associated with a normal cortisol responsein PCOS, but not in congenital adrenal hyperplasia.

Answer 335

The only abnormality is the very wide pulse pressure inthe ascending aorta, which would be consistent with thediagnosis of aortic regurgitation. (See Interpretation ofCardiac Catheter Data, page 418.)

d. Long early diastolic murmur at left lower sternaledge and in the aortic area.

Page 339: Rapid review of clinical medicine for mrcp part 2

338

Answer 336

The patient has a leucoerythroblastic anaemia (Table),which is characterized by immature granulocytes and redcells (normoblasts) in the peripheral blood film. Thischaracteristically occurs when the marrow is replaced bymalignant cells, cells containing metabolic products ofstorage disorders, fibrous tissue and bone. Leukaemias,lymphoma, adenocarcinoma, myeloma andmyeloproliferative disease are all capable of malignantinfiltration. In this case, the patient has a ‘knobbly’ liver,suggesting liver metastases most probably from a primarylung carcinoma. Carcinoma of the lung also metastasizesto the bone and may result in marrow infiltration by

malignant cells. Other causes of a leucoerythroblasticanaemia are tabulated above. In marble bone disease bonyreplacement of the marrow causes the leucoerythroblasticanaemia. In myelosclerosis the cause is extensive fibrosiswithin the marrow space, and in Gaucher’s disease thecause is large white cells laden with glucocerebrosides.

Causes of a leucoerythroblastic anaemia

• Malignant infiltration of any type• Storage disorders, e.g. Gaucher’s disease• Myelosclerosis• Marble bone disease (osteopetrosis)• Leukaemoid reaction secondary to severe

infections

Answer 337

The patient has contracted Toxocara infection from thedogs. Toxocariasis is due to migrating larvae of theparasite T. canis. The disease occurs where there is a largedog or cat population. It is well-recognized in theWestern world, and it is estimated that approximately2–3% of the population carry antibodies to the parasite.The parasite is carried in dogs (T. canis) and cats (T. cati). Embryonated eggs are passed into faeces andmay be ingested by humans, particularly children in closecontact with cats and dogs (ingestion of contaminated furor soil). The larvae from the eggs hatch out into thesmall intestine and migrate to liver, lungs and brain,where they stimulate local granuloma formation by theimmune system. They do not mature into adult worms.

Toxocariasis can produce two distinct clinical patterns;the classical visceral larva migrans syndrome, and oculartoxocariasis. Most children who develop the classical larvamigrans remain asymptomatic. The minority develop

myalgia, cough and anorexia. An urticarial rash is notuncommon. Physical examination may reveallymphadenopathy and hepatosplenomegaly. The bloodpicture comprises a neutrophil leucocytosis and asignificant eosinophilia. The acute illness is of 2–3 weeks’duration, following which complete recovery is usual. Insome individuals complete recovery may take as long astwo years. The diagnosis is made by demonstratingpositive antibodies to the secretory or excretory productsof Toxocara. Liver biopsy may demonstrate granulomaswith eosinophilic infiltration.

Ocular toxocariasis is characterized by granulomaformation in the eye. If the macular area is involved,blindness may occur. The general manifestations ofclassical visceral larva migrans and accompanyingeosinophilia may be absent. Physical examination revealsa round swelling near the optic disc. The diagnosis ismade by the demonstration of IgM antibodies toToxocara in the aqueous or vitreous humour.

Treatment of the classical visceral larva migrans is withthiabendazole or diethylcarbamazine. Ocular toxocariasisis treated with laser photocoagulation. Local orintraocular steroids may be used as adjunct therapy.

1. b. Marrow infiltration by malignant cells.2. a. Bone marrow trephine.

1. Ocular Toxocara infection or ocular larva migrans.

2. i. IgM antibodies to Toxocara in the aqueous or vitreous humour of the eye.

ii. IgM antibodies to Toxocara in the blood.

Page 340: Rapid review of clinical medicine for mrcp part 2

Clinical Cases 339

Question 339

A 72-year-old woman underwent upper gastrointestinalendoscopy for epigastric discomfort, which revealed mildantral gastritis. She had a history of transient ischaemicattacks for which she was taking aspirin 75 mg daily. Thepatient also took regular diclofenac for arthritis. Theurease breath test for H. pylori was negative.

What is the best management of her epigastricsymptoms?

a. Start omeprazole 20 mg daily.b. Switch patient from aspirin to clopidogrel.c. Switch patient from diclofenac to rofecoxib.d. Start ranitidine 300 mg daily.e. Start misoprostil.

Question 338

A 58-year-old man presented with recent onset ofepigastric burning that was precipitated by hunger orafter a heavy meal. His appetite was unchanged and hisweight was stable. He was a non-smoker and did notconsume alcohol. The patient took paracetamol only fora painful left hip.

Investigations are shown.

The patient was treated with H. pylori eradicationtherapy and was completely asymptomatic when reviewedsix weeks later. The Hb was 13 g/dl.

Hb 11 g/dlMCV 70 flFerritin 20 iu/l Colonoscopy NormalH. pylori urease test Positive Upper gastrointestinal endoscopy showed smallduodenal ulcer

What is the next management step?a. Reassure and discharge. Review again if he

becomes symptomatic.b. Continue with proton pump inhibitor

indefinitely.c. Repeat upper gastrointestinal endoscopy to

ensure the ulcer has healed.d. Repeat urease breath test to ensure eradication

therapy is successful.e. Cimetidine therapy indefinitely.

A 68-year-old woman presented with lethargy, mildweight loss, weakness, and progressive pain and stiffnessof her shoulders and thighs. On examination, she hadtender shoulder and thigh muscles and a fever of 38°C(100.4°F).

Investigations are shown.

Question 340

Hb 11 g/dlWCC 13 � 109/lPlatelets 160 � 109/lESR 85 mm/hRheumatoid factor 1/640AST 86 iu/lALT 296 iu/lAlkaline phosphatase 100 iu/lBilirubin 12 �mol/lLDH 1500 iu/l

What is the diagnosis?a. Polymyositis.b. Polymyalgia rheumatica.c. Rheumatoid arthritis.d. Primary biliary cirrhosis.e. SLE.

Page 341: Rapid review of clinical medicine for mrcp part 2

Gastroduodenal toxicity secondary to non-steroidal anti-inflammatory drugs (NSAIDs) is relatively common. Thepatient takes two NSAIDs, namely diclofenac and aspirin.The best treatment would be to stop both drugs, but sheneeds diclofenac for the treatment of her arthritis andaspirin to reduce the risk of transient ischaemic attacks.Until recently patients experiencing intolerablegastroduodenal side-effects from NSAIDs used forarthritis were switched from NSAIDs to COX IIinhibitors such as rofecoxib; however, the latter group ofdrugs have been associated with an increased frequencyof adverse cardiovascular events and have beenwithdrawn from the market.

The patient requires aspirin to prevent adversecardiovascular and cerebrovascular events. Althoughclopidogrel has fewer gastroduodenal side-effects thanaspirin, it is not free from gastroduodenal toxicity itself.Proven therapies that prevent gastroduodenal toxicityresulting from NSAIDs include treatment withmisoprostil, a prostaglandin E analogue, or lansoprazole,a proton pump inhibitor. None of the other protonpump inhibitors have been proven or tested (in largetrials) to be effective in this situation. H2 blockers arenot effective in preventing gastroduodenal toxicity.Instead of stopping both diclofenac and aspirin, the besttreatment would be to add misoprostil.

This patient with dyspepsia had iron deficiency anaemia,which is an indication for upper gastrointestinalendoscopy. He has a duodenal ulcer and has evidence of

H. pylori infection. His symptoms have been successfullytreated with eradication therapy. He does not warrantany further investigation unless he has a recurrence ofsymptoms. Over 90% of duodenal ulcers secondary to H.pylori infection heal following eradication therapy. (SeeAnswer 369.)

340

Answer 338

a. Reassure and discharge. Review again if hebecomes symptomatic.

Answer 339

e. Start misoprostil.

The differential diagnosis is between polymyalgiarheumatica and polymyositis (Table). Both present with

lethargy and weakness, and both may cause apolyarthropathy and tender muscles; however, elevatedmuscle enzymes and positive serology for rheumatoidarthritis are more characteristic of polymyositis. In 20%,anti-jo-1 antibodies may be present.

Answer 340

a. Polymyositis.

Polymyositis versus polymyalgia rheumatica

Polymyositis Polymyalgia rheumaticaMuscle tenderness Common CommonMuscle wasting Common May occurProximal myopathy Present RecognizedHeadaches Absent May be presentSkin involvement Present (30%) AbsentRaised enzymes Present (usually) AbsentANA Present (30%) AbsentRh factor Present AbsentAnti-jo-1 Abs Present (20%) Absent

Page 342: Rapid review of clinical medicine for mrcp part 2

Clinical Cases 341

A 74-year-old male presented with sudden onset ofdyspnoea. There was no preceding history of a cough orchest trauma. He was a smoker. Four weeks previously hehad had a right knee replacement. There was a pasthistory of prostatic carcinoma, which was being

controlled with antiandrogen therapy. On examinationhe was not cyanosed. The heart rate was 90 beats/minand blood pressure was 140/88 mmHg. The JVP wasnot raised. Both heart sounds were normal. Auscultationof the lungs revealed decreased air entry at the left lungbase. The lower limbs appeared normal.

Investigations are shown.

Question 341

Arterial blood gases (air):pH 7.45PaCO2 3.4 kPaPaO2 9.1 kPaHCO3 21 mmol/l

Chest X-ray Left lower lobe shadowingECG Non-specific T wave flattening in

leads V5 and V6Inverted T waves in III and aVf

What is the diagnostic investigation of choice?a. Ventilation–perfusion lung scan.b. Serum cardiac troponin level.c. Echocardiography.d. CT pulmonary angiography.e. Serum d-dimer assay.

A 70-year-old patient presented with dizziness (342).

Question 342

What is the ECG diagnosis?a. Third degree AV block.b. Mobitz type II second degree AV block.c. First degree AV block.d. Nodal bradycardia.e. Mobitz type I second degree AV block.

334422

Page 343: Rapid review of clinical medicine for mrcp part 2

342

The patient presents with sudden onset of dyspnoea in theabsence of a lower respiratory tract infection or pulmonaryoedema. He has a major risk factor for venousthromboembolism, having had a recent knee replacement.A coexisting history of prostatic carcinoma may alsocontribute to his risk of venous thrombo embolism.Therefore the clinical probability of the presentation beingsecondary to pulmonary embolism is high. The probabilityof PE is calculated by (a) demonstrating the absence ofanother reasonable clinical explanation of the presentationand (b) the presence of a major risk factor (Table). If both(a) and (b) are present the probability of PE is high; ifeither (a) or (b) is present the probability is intermediate;and if neither (a) nor (b) is present the probability is low.

In patients where the clinical probability of PE is high,the investigation of choice is CT pulmonary angiography,(341) which has superseded ventilation–perfusionscanning in the investigation of PE. It is superior toventilation–perfusion scanning in diagnosing andexcluding PE. It is quicker to perform, easier to arrangeurgently out of hours, and rarely needs to be followed upby further imaging. Furthermore, CT pulmonary

angiography may identify an alternative diagnosis whenPE has been excluded.

Ventilation–perfusion scan may only be considered asa first-line investigation if all of the following criteria aresatisfied:• Facilities to perform the test are available on site.• The chest X-ray is normal.• There is no significant cardio-pulmonary disease.• Standardized reporting criteria are used.• A non-diagnostic test is always followed by further

imaging.A normal ventilation–perfusion scan reliably excludes PE,but an intermediate scan should be followed up by CTpulmonary angiography.

In patients who have an intermediate probability of PEon clinical grounds alone, a d-dimer assay should beperformed. A negative d-dimer assay reliably excludes PEand the need for imaging to diagnose PE. There arecurrently three commercially available d-dimer assays(SimpliRED, Vidas and MDA). A SimpliRED is onlyeffective in excluding PE in patients with low probabilitywhereas the Vidas and MDA are useful in intermediate andlow probability cases (algorithm).

Echocardiography is only diagnostic in massive PE andmay reveal right heart dilatation and dysfunction as wellas thrombus in the inferior vena cava, right atrium orright ventricle. Our patient does not have any features tosuggest massive PE such as circulatory collapse, acidosisor clinical signs of right heart strain.

In patients with symptoms of PE and clinical featuresof a lower-limb DVT, the confirmation of deep-veinthrombosis on Doppler ultrasonography of the deep legveins obviates the need for pulmonary imaging.

Answer 341

d. CT pulmonary angiography.

Major risk factors for thromboembolism

Risk factor ExamplesSurgery Major abdominal,

pelvic, hip or kneeObstetrics Late pregnancy,

Caesarean section, puerperium

Lower limb problems Fracture, varicose veinsMalignancy Abdominal/pelvic,

advanced metastaticReduced mobilityPrevious DVT

Investigation of PE

Assess clinical probability

High Intermediate Low

D-dimer N/A SimpliRED Vidas/MDA Any D-dimer

PE No PE Another diagnosis

Warfarin

Start LMWHCT Pulmonary angiogram

D-dimer assayPositive Negative

334411

Page 344: Rapid review of clinical medicine for mrcp part 2

Clinical Cases 343

A 44-year-old male who was receiving haemodialysisthree times weekly complained of fatigue, night sweatsand a dry cough and was found to have a haemoglobin of

8.5 g/dl. His haemoglobin two months previously hadbeen 11.4 g/dl on regular erythropoietin andintermittent intravenous iron supplements. There was nohistory of haematemesis or malena.

Investigations are shown.

Question 344

Hb 8.5 g/lMCV 87 flWCC 11 � 109/lSerum ferritin 580 mg/lSerum B12 260 ng/lSerum folate 7 mg/lSerum aluminium 6 mg/lPlasma PTH 15 pmol/l (NR 0.9–5.4 pmol/l)C-reactive protein 110 mg/l (NR <10 mg/l )

What is the most likely cause for the resistance toerythropoietin therapy?

a. Aluminium toxicity.b. Hyperparathyroidism.c. Chronic sepsis.d. Red cell aplasia.e. Iron deficiency.

A 68-year-old Caucasian male complained of a four-yearhistory of increasing breathlessness on exertion. Therewere no other respiratory symptoms. There was nohistory of smoking. The patient had a history ofhypertension and had been taking an ACE inhibitor forover six years. He was a retired clerk.

On examination he appeared well and was not cyanosed.There was evidence of early clubbing. Auscultation of thelungs revealed fine end inspiratory crackles. All otheraspects of physical examination were normal.

Investigations are shown.

Question 343

What is the most probable diagnosis?a. ACE inhibitor related alveolitis.b. Bronchiectasis.c. Cryptogenic fibrosing alveolitis.d. Sarcoidosis.e. Asbestosis.

Hb 13 g/dlWCC 6 �109/lPlatelets 200 �109/lESR 28 mm/hCRP 32 g/lArterial blood gases:

pH 7.4PaCO2 4.3 kPaPaO2 8.3 kPaBicarbonate 26 mmol/l

Chest X-ray (343)

There is failure of some P-waves to conduct that is precededby progressive prolongation of the PR interval, which is

typical of Mobitz type I second degree AV block. Theconduction disturbance is due to an abnormality at the levelof the AV node. In the presence of symptoms thetreatment is implantation of a per manent pacemaker.

e. Mobitz type I second degree AV block.

Answer 342

334433

Page 345: Rapid review of clinical medicine for mrcp part 2

344

The patient has resistance to erythropoietin – despitesatisfactory iron stores – the causes of which are tabulatedbelow. Iron deficiency is by far the commonest cause ofanaemia despite erythropoietin. Erythropoeitin shouldnot be started until iron stores are replenished asevidenced by a serum ferritin ≥150 mg/l, or thetransferrin saturation is ≥20%.

The data provided in the question are against folate orB12 deficiency or aluminium toxicity. The patient has amildly raised serum PTH, as one would expect in apatient with chronic renal failure. In generalerythropoietin resistance is only seen in bone disease dueto severe hyperparathyroidism. The history of malaise andnight sweats together with the raised serum CRP,indicates that the most probable cause for failure torespond to erythropoietin is chronic sepsis. Theidentification and treatment of the source of sepsis shouldhelp resolve the anaemia.

Treatment with erythropoietin in patients with chronicrenal failure has significantly improved the quality of lifeand well-being of patients. Erythropoietin prevents thedevelopment of cardiac enlargement and high outputheart failure. For unexplained reasons treatment witherythropoietin has also been shown to improve sleeppattern, sexual function, and increase cognitive function.Erythropoietin is usually administered subcutaneouslytwo or three times per week. It is well tolerated butcomplications include hypertension, headache (15%) andflu-like symptoms (5%). A specific brand of recombinanterythropoietin alpha, termed Eprex, has been associatedwith pure red cell aplasia owing to the development oferythropoietin-specific antibodies.

Answer 344

c. Chronic sepsis.

Causes of resistance to erythropoietin inpatients with renal failure

• Inadequate iron stores• Blood loss• Folate or B12 deficiency• Inadequate dialysis• Chronic inflammation• Sepsis• Severe hyperparathyroidism• Aluminium toxicity

There is no obvious cause for the increasingbreathlessness in the history. Although the patient hasbeen taking an ACE inhibitor for six months, there is noassociation between ACE inhibitors and pulmonaryfibrosis. The physical findings and the chest X-ray areconsistent with cryptogenic fibrosing alveolitis. There isreticulonodular shadowing of both lung bases. Thedifferential diagnosis for bibasal lung shadowing is shown(Table). Although sarcoidosis is listed in the potentialchoices, there is no previous history of acute sarcoid norany evidence of multi-system involvement.

Answer 343

c. Cryptogenic fibrosing alveolitis. Causes of lower lung fibrosis

• Cryptogenic fibrosing alveolitis• Asbestosis• Aspiration pneumonia• Rheumatoid arthritis• Drug-related pulmonary fibrosis• Systemic sclerosis• Sarcoidosis

Page 346: Rapid review of clinical medicine for mrcp part 2

Clinical Cases 345

Question 346

A 72-year-old male presented with anorexia, nausea andintermittent falls secondary to dizziness. The BPmeasured 110/60 mmHg when lying and 70/30 mmHgon standing (accompanying dizziness). The JVP wasraised. Cardiac auscultation revealed a loud third heartsound. There was mild pitting ankle oedema.

Investigations were as follows:

Hb 8.9 g/dlWCC 9 � 109/lPlatelets 170 � 109/lESR 89 mm/hSodium 131 mmol/lPotassium 3.8 mmol/lUrea 14 mmol/lCalcium 2.8 mmol/lPhosphate 1.9 mmol/lAST 26 iu/lAlkaline phosphatase 140 iu/l Albumin 19 g/lTotal protein 70 g/l24-hour urinalysis:

Protein 5 g/lSodium 7 mmol/l

What is the cause of his dizziness?a. Hypoadrenalism.b. Postural hypotension secondary to

dehydration.c. Autonomic neuropathy secondary to

amyloidosis.d. Anaemia.e. Syndrome of inappropriate ADH secretion.

A 38-year-old male with HIV presented with confusionand ataxia and reduced visual acuity in the left eye. Abrain MRI scan is shown (345a).

Question 345

What is the diagnosis?a. Multiple sclerosis.b. Progressive multifocal leucoencephalopathy.c. Cerebral toxoplasmosis.d. Cerebral lymphoma.e. Cerebral tuberculosis.

334455aa

Page 347: Rapid review of clinical medicine for mrcp part 2

346

The patient has anaemia, a raised ESR, andhypercalcaemia, which are all consistent with thediagnosis of multiple myeloma. The heart failure andnephrotic syndrome in a patient with myeloma is highlysuggestive of primary amyloidosis complicating themultiple myeloma. Primary amyloidosis is a recognized

cause of autonomic neuropathy, which would explain thefalls and the postural hypotension. The low sodium inthe context of postural hypotension may lead somecandidates to opt for the diagnosis of Addison’s disease,but in this particular case it is most likely to representpseudohyponatraemia due to increased protein in theplasma rather than true hyponatraemia. (The causes ofautonomic neuropathy are discussed in the Answer 395).

Answer 346

c. Autonomic neuropathy secondary to amyloidosis.

The MRI scan reveals multiple bilateral, asymmetricalconfluent high intensity signals without any evidence ofraised intracranial pressure or mass effect. This is thecharacteristic finding in progressive multifocalleucoencephalopathy. The lesions usually involve theperiventricular areas and subcortical white matter. Thespinal cord is rarely affected in PMLE.

In contrast, most patients with HIV syndromedevelop cerebral disorders associated with cerebraloedema and mass effect on CT scan or MR scan of thebrain (345b).

Multiple sclerosis may have similar findings on theMRI scan; however, confusion and multifocalabnormalities occurring simultaneously in an immuno -compromised patient are more consistent with thediagnosis of PMLE.

PMLE is characterized by rapidly progressive focalneurological deficit without evidence of raisedintracranial pressure. The disorder affectsimmunocompromised patients who become infected with

the human polyoma virus, JC virus. Manifestations aredue to cerebral white matter involvement but cerebellarand brainstem involvement also occur. Clinical featuresinclude cognitive impairment, visual field defects, andhemiparesis. Aphasia, ataxia and cortical blindness arealso recognized. The severity of the clinical findings isdisproportionate to the abnormalities identified on brainscans. Patients with advanced disease go on to developsevere dementia and coma. In most patients death usuallyoccurs within a year of diagnosis. The diagnosis is usuallymade on clinical and radiological grounds. Definitivediagnosis requires brain biopsy. The identification of JCvirus on PCR of CSF specimens supports the diagnosis inpatients with clinical features to suggest PMLE; however,many immunocompromised patients without PMLE andsome healthy individuals have positive PCR results for JCvirus, therefore the investigation cannot be relied upon tomake the diagnosis in isolation.

Other infections caused by JC virus in immuno -compromised patients include haemorrhagic cystitis inbone marrow transplant patients and interstitial nephritisin renal transplant patients.

Answer 345

b. Progressive multifocal leucoencephalopathy.

Cerebral disorders in HIV

CT or MRI scan brain

Mass effect

Cerebral toxoplasmosisCerebral lymphomaCryptococcal meningitisTB meningitis or tuberculomaNocardia infectionBacterial abscessCMV encephalitis

No mass effect

PMLEHIV encephalitis

334455bb

Page 348: Rapid review of clinical medicine for mrcp part 2

Clinical Cases 347

A 78-year-old female patient presented with acuteabdominal pain and vomiting. A plain abdominal film isshown (347a).

Question 347

What is the diagnosis?a. Large bowel obstruction.b. Nephrolithiasis.c. Gallstone ileus.d. Acute chronic pancreatitis.e. Acute colitis.

A 30-year-old female presented with malaise, nausea,vomiting and right upper quadrant pain in the last weekof her pregnancy. She had tolerated the pregnancy verywell prior to the event. Her blood pressure controlthroughout the pregnancy had been good, with the last

reading measuring 96/60 mmHg two weeks previously.There was no history of headache or diarrhoea.

On examination she appeared pale. There was no obviousevidence of jaundice. There was marked right upper quadranttenderness. The blood pressure measured 130/86 mmHg.

Investigations are shown.

Question 348

Hb 10 g/dlWCC 12 � 109/lPlatelets 33 � 109/lPT 12 secAPTT 43 secSodium 137 mmol/lPotassium 3.4 mmol/lUrea 12 mmol/lCreatinine 223 �mol/lBilirubin 20 μmol/lAST 250 iu/lALT 298 iu/lAlk Phos 200 iu/lLDH 800 iu/lAlbumin 33 g/lAmylase 150 iu/l (NR <220 iu/l)Urinalysis Protein +Hepatobiliary ultrasound Normal

What is the definitive treatment?a. IV cefuroxime and metronidazole.b. Start methyldopa.c. Deliver the baby urgently.d. Exploration laparoscopy.e. IV hydrocortisone.

334477aa

Page 349: Rapid review of clinical medicine for mrcp part 2

348

There is evidence of small bowel obstruction and threevisible gallstones (in right upper quadrant, adjacent to thehead of the right femur and in the left upper quadrant,347b). The patient has gallstone ileus, which is a rarecomplication of gallstones that results from mechanicalsmall bowel obstruction following ulceration of a gallstonethrough the common bile duct directly into the smallbowel. Most patients who develop gallstone ileus havepreviously had cholecystitis. As the stone enters the smallbowel it becomes bigger owing to sedimentation of thegut contents. The vast majority of obstructing stones are>2 cm in diameter. Obstruction characteristically occurs inthe ileum, which is the narrowest part of the small bowel.

The diagnosis requires erect and supine plain abdominalfilms. The presence of small bowel obstruction, visiblegallstones in the small bowel and air in the biliary tree(arrows) is diagnostic. In many instances, however, thegallstones in the bowel are not visible on plain abdominalfilms, and abdominal ultrasound and CT scan are morespecific investigations. Initial treatment involves resting thebowel and restoring fluids. Definitive treatment is surgical.

Answer 347

c. Gallstone ileus.

The patient presents with nausea, vomiting and rightupper quadrant discomfort towards the end of herpregnancy. Her blood pressure is at the upper limit ofnormal and considerably higher than it was two weekspreviously. The patient also has proteinuria, a micro-angiopathy as characterized by the fragmented red cellson the blood film, abnormal liver enzymes and lowplatelets. The diagnosis is consistent with the HELLPsyndrome (haemolysis, elevated liver enzymes and lowplatelets. The condition affects approximately 1 per 1000pregnancies. It is more common in multiparous women.Some authorities believe that it is a manifestation ofsevere pre-eclampsia; however, approximately 15% ofaffected females do not have hypertension andproteinuria, which are the classical hallmarks of pre-eclampsia, causing other authorities to postulate that itmay be a completely separate disorder.

HELLP syndrome usually occurs in the last trimesterof pregnancy or within the first week of parturition. Afew cases may have been reported to occur in the secondtrimester. Patients often present with malaise, nausea,vomiting and right upper quadrant tenderness. Thecondition may mimic several other medical conditionssuch as viral hepatitis and cholecystitis. Hypertension andproteinuria are often present. The blood film shows

evidence of a micro-angiopathic haemolytic anaemiaassociated with a platelet count, and liver enzymes areraised. The hepatic transaminase concentrations areelevated but there is no recognized diagnostic limit. Ingeneral the presence of schistocytes on the blood film, aplatelet count <100 � 109/l, an AST >70 iu/l and anLDH >600 iu/l are highly suggestive of the HELLPsyndrome. Clotting is normal unless HELLP iscomplicated by disseminated intravascular coagulation.

The differential diagnosis includes TTP, HUS andfatty liver. In TTP and HUS a history of precedinghypertension or proteinuria is usually absent and the onlyhaematological abnormalities are micro-angiopathy andlow platelets. DIC does not complicate HUS and TTP.Fatty liver may be difficult to differentiate from HELLPas it presents in a similar fashion; however, the

Answer 348

c. Deliver the baby urgently.

Management of HELLP

1. Deliver the foetus as soon as possible.2. Intravenous corticosteroids in females <34

weeks’ gestation to defer early delivery.3. Treat hypertension.4. Platelet and blood transfusion if there is

bleeding.5. Magnesium sulphate for epileptic seizures.6. Dialysis for rapidly deteriorating renal function.

334477bb

Page 350: Rapid review of clinical medicine for mrcp part 2

transaminase levels are usually in the thousands and theclotting screen is frequently abnormal in fatty liver.

Patients may develop severe hypertension, epilepticseizures and bleeding complications. The mainstay oftreatment is to deliver the foetus as soon as practical. In

females >34 weeks’ gestation this is the definitivetreatment. In females <34 weeks’ gestation it may bepossible to defer delivery with intravenous corticosteroidswhile the foetal lungs mature.

Clinical Cases 349

Question 350

A 75-year-old male presented with exertional chest pain.On examination he had an ejection systolic murmurconsistent with aortic stenosis. The 12-lead ECGrevealed voltage criteria for left ventricular hypertrophy.Echocardiography confirmed calcific aortic stenosis witha pressure gradient of 68 mmHg across the aortic valve.

What is the most important prognostic factor in thehistory?

a. Presence of left ventricular hypertrophy on 12-lead ECG.

b. Presence of left ventricular hypertrophy on echocardiography.

c. Pressure gradient >60 mmHg across the aortic valve.

d. Presence of symptoms.e. Calcified aortic valve.

A 30-year-old male presented with sudden onset of left-sided weakness. The was no history of headache. He hadexperienced transient weakness of his right arm threeyears previously, which resolved spontaneously after 5 hours and did not receive medical attention. He had apast history of multiple episodes of epistaxis which hadnot been investigated. Eight months previously he hadexperienced a cough associated with fever andintermittent haemoptysis, which improved after a courseof amoxicillin, but he had been more breathless thanusual since the episode. There was no family history ofpremature atherosclerosis. His father was fit and well buthis mother was anaemic and had been investigated onnumerous occasions for gastrointestinal haemorrhage.The patient smoked 20 cigarettes per day.

On examination he had a left-sided hemiparesis. Thepulse was regular. The blood pressure measured

120/70 mmHg. Auscultation of the heart was normal.All peripheral pulses were easily palpable; there was noclinical evidence of bruits over the carotid arteries. Thechest was clear. Apart from small red cutaneous lesionsaround the inner aspect of the left nostril there were noother abnormalities on examination.

Investigations are shown.

Question 349

What is the diagnosis?a. Bronchial carcinoma.b. Polycythaemia rubra vera.c. Wegener’s granulomatosis.d. Hereditary haemorrhagic telangiectasia.e. Patent foramen ovale.

Hb 18 g/dlWCC 11 � 109/lPlatelets 206 � 109/lPCV 0.58CRP 9 g/lTotal cholesterol 5.2 mmol/lBlood glucose 4.1 mmol/l12-lead ECG NormalEchocardiogram NormalCT scan brain NormalCarotid Doppler studies NormalChest X-ray (349a)

334499aa

Page 351: Rapid review of clinical medicine for mrcp part 2

350

Indications for surgery in severe aortic stenosis include:

• Presence of symptoms. • Patients with severe AS undergoing coronary artery

bypass surgery.• Patients with severe AS undergoing surgery on the

aorta or other heart valve(s).

Severe left ventricular hypertrophy, a pressure gradient≥100 mmHg across the aortic valve and systolicdysfunction are indications for surgery in asymptomaticpatients. However, most patients in these categories arerarely asymptomatic. Patients with impaired leftventricular function in the context of aortic stenosis havea poor prognosis if untreated.

Answer 350

d. Presence of symptoms.

This young patient presents with a stroke in the absence ofany of the conventional risk factors for atherosclerosis. Hehas a long history of epistaxis. He also has haemoptysis,and his chest X-ray reveals a well-defined round opacityadjacent to the right heart border that has a linear shadowat the 1 o’clock position, consistent with a feeding vesselindicating a pulmonary arterio-venous mal formation.There is a family history of anaemia and gastrointestinalhaemorrhage and visible telangiectasia in the left nostril.The most likely underlying diagnosis is hereditaryhaemorrhagic telangiectasia. The increased haemoglobinconcentration in this patient reflects physiologicalpolycythaemia due to chronic hypoxaemia.

HHT, also known as Osler–Weber–Rendu syndrome,is a genetic disorder resulting in arterio-venousmalformations in various parts of the body including theskin, liver, respiratory system, GI tract and the brain.Abnormalities in the genes encoding the angiogenesisfactors endoglin and activin receptor-like kinase 1 havebeen identified on chromosome 9 and 12 respectively.

The commonest manifestation of the disorder isspontaneous epistaxis, which may be severe. Up to 70% ofpatients with HHT have pulmonary arterio-venousmalformations, which may be asymptomatic but can beassociated with haemoptysis, exercise intolerance,hypoxaemia, pulmonary hypertension and paradoxicalemboli. The latter may be complicated by migraines,transient ischaemic attacks, stroke and cerebral abscess.Telangiectasia may occur anywhere in the GI tract andgrow with age. GI haemorrhage occurs in older patients(aged 60 years or over).

Cutaneous telangiectasia are the commonest physicalsign. Bruits over organs with telangiectasias may be heardin a few patients.

Pulmonary arterio-venous malformations are bestvisualized with contrast CT scan of the thorax (349b).Blood gas analysis may demonstrate hypoxia. Pulmonaryfunction tests reveal a low transfer factor. Some patientswith diffuse arterio-venous malformations exhibitorthodeoxia (decrease in oxygen saturation on standingfrom a supine position), because lying down reducesblood flow through malformations in dependent parts ofthe lung.

Answer 349

d. Hereditary haemorrhagic telangiectasia.

Complications of HHT

• Migraines• Seizures• TIA/CVA• Cerebral abscess• Cerebral haemorrhage due to cerebral arterio-

venous malformation• Haemothorax• Massive haemoptysis• Pulmonary hypertension• Central cyanosis• GI haemorrhage• Anaemia due to blood loss• Polycythaemia due to chronic hypoxia

334499bb

Page 352: Rapid review of clinical medicine for mrcp part 2

Clinical Cases 351

A 36-year-old woman presented with fever and malaise.She had been commenced on carbimazole forsymptomatic thyrotoxicosis four weeks previously. Onexamination her temperature was 39.2°C (102.5°F) andher mouth was red. Examination of all the major systemswas normal.

Investigations are shown.

Question 351

Hb 12 g/dlWCC 0.8 � 109/l (neutrophils 0.3)Platelets 190 � 109/lCRP 240 mg/lChest X-ray NormalB cultures No growthMouth swabs No growthUrine culture No growthStool culture No pathological organisms

What is the best initial antibiotic combination in thispatient?

a. IV cefuroxime and IV clarithromycin.b. IV benzylpenicillin.c. IV vancomycin and gentamicin.d. IV Tazocin and gentamicin.e. IV acyclovir.

A 16-year-old Spanish boy was admitted to hospital witha three-day history of abdominal pain and jaundice. Oneweek before admission he had developed a sore throat,accompanied by fever and myalgia. He had experiencedthree similar episodes of jaundice and abdominal painpreceded by a sore throat and fever at the ages of 6, 12and 14, and on each occasion he made an uncomplicatedrecovery. There was no history of recent travel abroad.He was the only child in the family.

On examination, he had a temperature of 39°C(102.2°F). He was mildly jaundiced and had tendercervical lymphadenopathy. Examination of the throatrevealed a purulent exudate. Examination of the lungswas normal. On examination of the abdomen there was apalpable spleen 5 cm below the costal margin.

Investigations are shown.

Question 352

Hb 9.1 g/dlWCC 6 � 109/lPlatelets 100 � 109/lMonospot test NegativeBilirubin 33 mmol/lALT 23 iu/lAlkaline phosphatase 120 iu/lUrinalysis Urobilinogen ++++

Nil else

What is the diagnosis?a. Hereditary spherocytosis.b. Infectious mononuclueosis.c. Glucose-6-phosphate dehydrogenase deficiency.d. Sickle cell anaemia.e. Gilbert’s disease.

Page 353: Rapid review of clinical medicine for mrcp part 2

352

The patient has severe neutropenia, presumablysecondary to carbimazole therapy (see Table A for causesof neutropaenia). Neutropenia is defined as a neutrophilcount of < 1.5 × 109/l. Many patients with mildneutropenia may remain asymptomatic; however, patientswith neutrophil counts < 0.5 × 109/l are at risk ofoverwhelming sepsis with viral, bacterial and fungalinfections, common sites of infection being the skin, gutmucosa and the urinary tract.

All neutropenic patients should have a septic screen(blood cultures, urine culture, mouth nasopharyngealswabs, culture of any septic focus which is easily visibleon examining the patients and chest X-ray) beforecommencing antibiotic treatment. The antibiotics used totreat severe neutropenia should be broad spectrum andcover both Gram-positive and negative bacteria.

The usual regime is an antipseudomonal beta-lactampenicillin such as azlocillin, tazocillin or aztreonam, or a

carbapenem such as meropenem, or a third generationcephalosporin such as ceftazidime, together with an

Answer 351

d. IV Tazocin and gentamicin.

Table A Causes of neutropenia

Drugs: Antithyroid (carbimazole)SulphonamidesAnticonvulsantsNon-steroidal anti-inflammatory

drugs (phenylbutazone)Antibiotics (chloramphenicol)PhenothiazinesAny drugs used for chemotherapy

Malignancy: Lymphomas, leukaemia

RadiotherapyInfections: Tuberculosis

Viral infections

Megaloblastic anaemiaToxins: Alcohol

Hypersplenism

Table B Antibiotic use in neutropaenic sepsis

Initial antibiotic therapy• Monotherapy

Ceftazidime or imipenem or meropenem• Dual therapy

Aminoglycoside plus beta-lactam with anti-pseudomonal action, or a third-generationcephalosporin, or a carbapenemVancomycin should be added to mono or dual therapyif there are indwelling venous catheters, evidence ofcardiovascular compromise or severe mucositis

Afebrile within first 3–5 days• If aetiology identifiedAdjust to most appropriate therapy based uponantibiotic sensitivity of organism• If no aetiology identified Low risk: Switch to oral antibiotics (usuallyciprofloxacin and augmentin)High risk: Continue with same antibiotics

Persistent fever during the first 3–5 days• Reassess after 3 days• If no change: Continue antibiotics. Stop

vancomycin if cultures are negative• If deteriorating: Change antibiotics.• If febrile after day 5: Consider adding anti-fungal agent

Duration of antibioticsAfebrile on day 3

Yes No

Neutrophil count ≥0.5 after 48 h

Neutrophil count< 0.5 after 48 h

Neutrophil count≥0.5 after 48 h

Neutrophil count<0.5 after 48 h

Stop antibiotics if noobvious focus of

infection

Low risk High risk

Continue antibiotics

Stop when afebrile for 5–7 days Stop antibiotics 4–5 days after neutrophil count is ≥ 0.5

Continue for 2–3 weeks if neutrophil count is <0.5

335511

Page 354: Rapid review of clinical medicine for mrcp part 2

Clinical Cases 353

Question 353

A 16-year-old boy was found dead in bed by his mother.A detailed post-mortem failed to reveal any obviouscause of death. His father died suddenly aged 30. He hada paternal aunt who was being investigated for syncopalepisodes. His sister was diagnosed as having epilepsyaged 11.

What is the most probable cause of death?a. Hypertrophic cardiomyopathy.b. Ecstasy abuse.c. Anomalous coronary arteries.d. Commotio cordis.e. Long QT syndrome.

The history of recurrent episodes of abdominal pain andjaundice precipitated by a sore throat in a young boy withanaemia, splenomegaly and a raised urinary urobilinogen issuggestive of an inherited haemolytic anaemia. There is afamily history of splenectomy to treat anaemia in a first-degree relative to support a hereditary anaemia. The mostlikely diagnosis is HS, although other possible causes of thispicture include hereditary elliptocytosis, glucose-6-phosphate dehydrogenase deficiency, pyruvate kinasedeficiency. Sickle cell anaemia may be associated withhaemolytic crises during infection, however the spleen isimpalpable because it has atrophied owing to multipleinfarcts. The history of a purulent sore throat and haemolysiscould be explained entirely by infectious mononucleosis;however, it would be unusual for a young boy to have threeepisodes of EBV infection. In this case, the sore throat hasbeen the precipitant for increased haemolysis in an individualwith a red cells which are vulnerable to haemolysis. Gilbert’sdisease is associated with mild jaundice during illness butdoes not explain the anaemia or haemolysis.

Hereditary spherocytosis (352) is an autosomal dominantdisorder of red cells. It is due to a defect in the membraneprotein spectrin, which results in membrane loss as the cellpasses through the spleen. The surface-to-volume ratio isreduced and the cells become spherocytic. Spherocytes areeventually destroyed by the spleen. The disorder may presentas neonatal jaundice; however, most patients have a low-

grade haemolysis with an intermittent increase in the rate ofhaemolysis during infections. During these episodes thepatient may become very anaemic and appear jaundiced.Some individuals are asymptomatic and some may present inmiddle age with complications from gallstones. Aplasticcrises may occur following infection with parvovirus, andmegaloblastic anaemia may occur owing to folate deficiency.Leg ulcers are well recognized. The blood film willdemonstrate at least 50% of spherocytes. When incubated insolutions of increasing hypotonicity or in their own plasmafor over 24 hours, the cells demonstrate an increased rate ofhaemolysis (red cell fragility test). There is evidence of ahaemolytic anaemia (raised bilirubin, reduced haptoglobins,increased urinary urobilinogen). Treatment of anaemia iswith splenectomy.

Answer 352

a. Hereditary spherocytosis.

aminoglycoside (Table B). Patients may be treated withjust one antibiotic instead of dual therapy. Antibioticsused as monotherapy include ceftazidime, imipenem andmeropenem. While there is no convincing evidence thatmonotherapy is inferior to dual therapy (i.e. one of theseantibiotics and an aminoglycoside), most authoritieschoose to treat with dual therapy, as monotherapy maypromote antibiotic resistance.

Patients with venous lines or indwelling catheters,evidence of MRSA on skin or nasal swabs should alsoreceive vancomycin.

Fever persisting for over a week on the antibioticsabove necessitates the addition of intravenous ampho -tericin to cover systemic fungal infection.

The duration of antibiotic therapy is outlined in thealgorithm (351).

Patients with severe neutropenia should receivediligent mouth care and fluconazole lozenges to preventoral candidiasis.

335522

Page 355: Rapid review of clinical medicine for mrcp part 2

354

The patient has died suddenly in the absence of previous illhealth and the postmortem has failed to reveal anyabnormality. By definition this individual is a victim of thesudden adult death syndrome. There are approximately 500such cases in the UK each year. Most deaths are thought tobe secondary to fatal cardiac arrhythmias. Many conditionsmay be identifiable on 12-lead ECG during life and manymay be familial. It is now recommended that all first-degreerelatives of victims of SADS undergo cardiovascularevaluation to identify conditions that could potentially causesudden cardiac death.

In this particular case there is reason to believe thatthe patient may have died from a familial cardiac disordersince there is a family history of premature sudden deathand syncope. Long QT syndrome is a well recognizedcause of sudden death in young patients where the post-mortem is normal. The diagnosis can be made ante-mortem with 12-lead ECG. Patients with long QTsyndrome may be entirely asymptomatic or experiencesyncope/sudden death or epileptic seizures due tomalignant ventricular arrhythmias. Patients with long QTsyndrome are often misdiagnosed as having epilepsy.

Most sudden non-traumatic deaths in youngindividuals are due to cardiac disease. The causes ofsudden cardiac death with normal post-mortem findingsare shown below (Table).

Brugada’s syndrome is a genetic disorder caused bymutations within the sodium ion channel SCN5A. Thedisorder is inherited as an autosomal dominant trait andcharacterized by a propensity to fatal ventricular arrhythmiasand sudden death in patients with structurally normal hearts.The resting ECG shows a characteristic peculiar patternconsisting of a pseudo-right bundle branch block andpersistent ST elevation in leads V1 to V3 (353). Thewidened S-wave in the left lateral leads that usuallycharacterizes RBBB is absent in most patients with Brugada’ssyndrome, suggesting that there is a high take off of the STsegment in the right precordium (J-wave) rather than true

RBBB. The QT interval may be prolonged in leads V1–V3.The prevalence of the Brugada ECG is 0.7–1%. The disorderis much more common in males and in patients from the FarEast, e.g. from countries such as Nepal and Thailand. Allclinical manifestations of Brugada’s syndrome are related tolife-threatening ventricular arrhythmias. Sudden death maybe the first and only presentation, occurring in up to one-third of patients. Mortality is much more common in males.Arrhythmic events occur between the third and seventhdecades. Most arrhythmic events occur during sleep,suggesting that an imbalance between the sympathetic andparasympathetic tone may be important in the pathogenesisof Brugada’s syndrome. The diagnosis of Brugada’ssyndrome is often difficult since the disorder is not wellestablished. ECG and clinical features are important.Brugada’s syndrome should be considered in patients whomeet one of the following criteria:

Appearance of ECG pattern in more than one rightprecordial lead (V1–V3) under baseline conditions in thepresence or absence of a sodium ion channel blocker, andone of the following:• Documented ventricular fibrillation.• Self-terminating polymorphic ventricular tachycardia.• Family history of sudden cardiac death at <45 years of

age.• Type 1 ECG pattern in family members.• Electrophysiological inducibility.• Syncope.• Nocturnal agonal respiration.

The only effective treatment in the prevention of suddendeath in Brugada’s syndrome is an implantablecardioverter defibrillator, and it has been proposed thatpatients with aborted sudden cardiac death or sustainedVT should be treated with an ICD.

Commotio cordis is a term used to describe ventriculartachycardia or ventricular fibrillation occurring after beingstruck in the precordium by a high velocity projectile suchas a base ball or an ice hockey puck. The individual maydie suddenly on the sports field. The heart appears normalat post-mortem examination. (See Question 368.)

Answer 353

e. Long QT syndrome.

Causes of sudden cardiac death in patients withstructurally normal hearts

• Ion channel diseases of the heart such as long QTsyndrome or Brugada’s syndrome

• Accessory pathways such asWolff–Parkinson–White syndrome

• Commotio cordis• Focal myocarditis• Atypical right ventricular cardiomyopathy that

may not be recognized by the pathologist• Drugs such as amphetamines• Electrolyte disturbances

335533

Page 356: Rapid review of clinical medicine for mrcp part 2

Clinical Cases 355

A 60-year-old female presented with a dry cough andhaemoptysis. She had a past history of tuberculosis. Achest X-ray and CT scan of the thorax are shown infigures 355a and b, respectively.

Question 355

What is the diagnosis?a. Recurrence of tuberculosis.b. Bronchogenic carcinoma.c. Staphylococcus aureus pneumonia.d. Aspergilloma.e. Aspiration pneumonia.

Question 354

A 63-year-old male with end-stage renal failure and a12-month history of dialysis had been on an active renaltransplant list for over six months. Over the past fewmonths dialysis therapy had been incomplete owing topoor tolerance, and his general health had started todeteriorate. He had made enquiries about renaltransplants and had heard that he could travel to somethird-world countries and purchase a kidney at a cheapprice. He was married with one son aged 16 years old.He did not have any siblings and both parents had died.His wife was not keen to donate her kidney, however hisson was eager to donate to help his father.

What course of action would you advise?a. Persuade his wife to donate the kidney.b. Encourage him to travel abroad to purchase a

kidney.c. Advise him to stay patiently on the transplant list.d. Advise him to take his son’s kidney.e. Make an urgent request to the cadaveric

transplant co-ordinator to prioritize a transplant for him.

335555aa 335555bb

Page 357: Rapid review of clinical medicine for mrcp part 2

356

The plain X-ray reveals a scarring in the left apexconsistent with previous tuberculosis. In the right apexthere is an opacity surrounded by a crescent of air(Monod’s sign). This finding is typical of an aspergilloma(mycetomoa) in an old tuberculous cavity. The CT scanof the thorax demonstrates the mass lesion within thelung cavity (the air around the aspergilloma is black).

Aspergilloma is due to colonization by the speciesAspergillus fumigatus rather than direct tissue invasion,and is usually not an indicator of an immuno -compromised state. It arises most commonly in oldtuberculous cavities but may occur in cavities created byneoplasms, sarcoidosis and other fungal infections such ashistoplasmosis (causes of pulmonary cavities aretabulated). The aspergilloma comprises fungal hyphae,inflammatory cells and fibrin. Patients may be asympto -matic but the vast majority experience haemoptysis. Somepatients complain of chest pain, wheeze and a fever. Thediagnosis is made with radiological tests. The chest X-rayis usually diagnostic but confirmation of the aspergillomamay require a CT scan or MRI scan of the thorax. Theidentification of Aspergillus in the sputum in suchpatients is highly suggestive of the diagnosis.

Patients with recurrent haemoptysis are treated withsurgical resection of the cavity and removal of theaspergilloma.

Answer 355

d. Aspergilloma.

Causes of cavitating lung lesions

Infections• Staphylococcus aureus pneumonia• Klebsiella pneumonia• Pseudomonas colonization• Tuberculosis• Histoplasmosis• Hydatid disease

Neoplasia• Benign• Malignant Primary

Secondary

Vascular• Pulmonary emboli• Vasculitides

Granulomatous disease• Sarcoidosis

This question relates to ethical issues around organdonation for transplantation. Availability or renal donorsfor transplants is a major problem in all developedcountries. Most patients receive transplants fromcadavers; however, success from live donor transplants issuperior to cadaveric transplants even in non-relateddonors. A blood group match in essential. A HLA matchis desirable but not mandatory.

In this particular situation, a first-degree relative isprepared to donate his kidney. The individual in questionis above the age of 16 and in a situation where he maysign his own consent to donate his kidney. Absolutecontra-indications to organ donation for renaltransplantation are tabulated below. The son shouldundergo routine blood tests, urinalysis, 24-hour urineassessment of protein estimation and creatinine clearance,HLA typing, serology for viral antibodies including CMVand HIV, IVU and renal angiography or equivalent.

It is unethical and potentially dangerous to advise himto purchase a kidney from elsewhere. Attempting toexpedite his transplantation via the transplant co-ordinatormay be seen as discrimination as there may be other, morestoical individuals with an equal need for a new kidney.Forcing his wife to donate a kidney and asking him to waitpatiently for a transplant when there is an ideal optionavailable to him contravenes good medical practice.

Absolute contra-indications to organ donation forrenal transplant include:

Answer 354

d. Advise him to take his son’s kidney.

• Age < 16 years• Proteinuria or haematuria• Abnormal creatinine clearance• HIV infection• Malignancy• Psychosis• Pregnancy• Chronic illness

Page 358: Rapid review of clinical medicine for mrcp part 2

Clinical Cases 357

A 26-year-old female with cystic fibrosis wishes to becomepregnant. She has never had any previous pregnancies. Shewas menstruating once or twice every three months. Shehad been with her partner for six years. She was diagnosedwith cystic fibrosis as a neonate after presenting with rectalprolapse. Her condition was relatively satisfactory inchildhood. During adolescence she experienced three to

four respiratory tract infections per year that werecontrolled with intravenous antibiotics. Her sputum hadcultured Pseudomonas aeruginosa for over six years. Herlung function trend showed an FEV1 of 60–65% predictedover the past four years. Echocardiography revealed anormal right heart and an estimated pulmonary arterypressure of 24 mmHG. Her body mass index was 21.

Question 356

What advice would you give her?a. To avoid pregnancy and consider referral to a gynaecologist for tubal ligation.b. To eradicate Pseudomonas aeruginosa from the sputum with aggressive antibiotic therapy before conceiving.c. To increase her BMI to above 24 with adequate nutrition before conceiving.d. To go ahead and become pregnant but explain the risk of her offspring being affected. e. To have her partner screened for D508 mutation on chromosome 7 prior to conceiving.

A 67-year-old male presented with pain in both groinregions that was worse on walking. He had a long historyof diabetes mellitus that was complicated by nephropathy,for which he had a successful renal transplant three yearspreviously. A plain X-ray of the hips is shown (357).

Question 357

What is the diagnosis?a. Multiple myeloma.b. Avascular necrosis of the femora.c. Osteoarthritis.d. Osteomalacia.e. Paget’s disease.

A 78-year-old male was investigated for lower back painthat started suddenly on bending over to pick up a pair ofshoes. He had a past medical history of hypertension andhad recently been commenced on therapy for depressionafter presenting with low mood, lethargy and constipa -tion. He was taking bendroflumethiazide, senokot,fluoxetine and regular paracetamol

Investigations are shown.

Question 358

Hb 10 g/dlWCC 10�109/lPlatelets 180�109/lMCV 84 flESR 96 mm/hSodium 133 mmol/lPotassium 4.6 mmol/lUrea 11 mmol/lCreatinine 180 �mol/lCalcium 3.2 mmol/lPhosphate 1.4 mmol/lAlbumin 30 g/lTotal protein 74 g/lX-ray lumbar spine Wedge fracture 4th lumbar

vertebra

What investigation would you perform next toconfirm the diagnosis?

a. Skeletal survey.b. Serum PTH level.c. MRI scan lumbar spine.d. Bone marrow biopsy.e. Chest X-ray.

335577

Page 359: Rapid review of clinical medicine for mrcp part 2

The patient has stable pulmonary function and normalpulmonary artery pressures, which is the main medicalconcern with respect to the risk of pregnancy for thefemale. Females with poor lung reserve may developrespiratory failure once the diaphragm is splinted by thegravid uterus. Patients with pulmonary hypertension areat risk of cardiac failure.

Approximately 20% of females with cystic fibrosis areinfertile owing either to malnourishment or to viscidcervical secretions. However, most women are capable offalling pregnant, and advice regarding conception ismainly based on the clinical risk of cardiac or heart failureduring gestation. Patients with severe lung disease orsignificant pulmonary hypertension (≥50 mmHg) shouldbe discouraged from becoming pregnant owing to thehigh risk of cardio-respiratory failure.

Most women with stable pulmonary function and FEV1values ≥60% predicted usually tolerate pregnancy withoutcomplication. Patients with unstable respiratory function

should have treatment optimized to achieve persistentFEV1 of 60% or more prior to conception. While mostadult lungs are colonized with Pseudomonas species thatare difficult to eradicate completely, patients who harbourPseudomonas burkholderia are at particularly increased riskof pulmonary infections, and efforts should be made toeradicate the organism prior to conception.

All females and their partners should be offeredgenetic counselling to explain the risks of the offspringbeing affected. In this respect the partner should beoffered genetic screening for the commoner mutationscausing cystic fibrosis to predict more accurately the riskof the offspring being affected. However, this is not themain reason for advising against pregnancy.

Following pregnancy, patients should be screenedearly for gestational diabetes due to underlying insulindeficiency. It is important to ensure that the patient isadequately nourished and gaining weight appropriately.

The anaesthetist should be alerted to the patient priorto delivery. Epidural anaesthesia is advised to reduce theeffect of labour on minute ventilation. Supplementaloxygen may be required during the peri-partum period.

358

Answer 356

d. To go ahead and become pregnant but explain therisk of her offspring being affected.

The patient has increased lucency in both femoral headsbelow the articular surfaces. The femoral heads have losttheir smooth spherical shape, indicating advanced disease.These findings are consistent with the diagnosis ofavascular necrosis affecting both femoral heads. Avascularnecrosis is a frequent complication following renaltransplantation. The aetiology is multi-factorial butsteroid therapy for immunosuppression plays animportant role. The prevalence of avascular necrosis ofbone following transplantation has reduced significantlysince the use of ciclosporin. Other causes of avascularnecrosis of bone are tabulated below. Patients presentwith groin or buttock pain that is worse on movement.Treatment may involve conservative measures such asanalgesia and reduced movement of the joint concerned,but generally core decompression, osteotomy or jointreplacement is required to treat the condition.

Avascular necrosis of bone may complicate deep-seadiving (decompression sickness). It is better known asCaisson’s disease. Episodic decompressions can lead tothe formation of nitrogen bubbles that occlude arteriolesand cause avascular necrosis. The prevalence is related tothe frequency of exposure and the depth reached.

Answer 357

b. Avascular necrosis of the femora.

Causes of avascular necrosis of the head of femur

• Congenital dislocation of the hip• Perthes’ disease• Trauma (subcapital fractures of the femur)• Steroid therapy• SLE• Sickle cell anaemia• Diabetes mellitus• Alcohol abuse• Gaucher’s disease• HIV infection• Caisson’s disease

Back pain, anaemia, hypercalcaemia, renal impairmentand hypergammaglobulinaemia amount to a diagnosis ofmultiple myeloma. Although a skeletal survey is alsouseful in the diagnosis of multiple myeloma, bonemarrow analysis is the test of choice.

Answer 358

d. Bone marrow biopsy.

Page 360: Rapid review of clinical medicine for mrcp part 2

Clinical Cases 359

A 76-year-old man was seen in the Accident andEmergency Department after an episode of loss ofconsciousness. He had become increasingly confused andsuffered several falls over the past two months. Over thepast six months his family had noticed a gradualdeterioration in his health. He had a constant non-productive cough. He appeared apathetic anduninterested in his surroundings. His appetite wasreduced, but according to his wife he was constantlythirsty and had recently become incontinent of largevolumes of urine. His GP had commencedantidepressants three weeks previously. He smoked 40cigarettes per day until two weeks previously but did not

consume alcohol. On examination he was generallyconfused and dehydrated but the remainder of thephysical examination was normal. Blood glucose onadmission was 6 mmol/l.

Question 359

What investigation would you perform next to helpachieve a diagnosis?

a. CT scan brain.b. Serum calcium.c. Chest X-ray.d. Urine culture.e. Lumbar puncture.

An 18-year-old female suffers from epilepsy and mentalretardation. Her mother was similarly affected. A CT scanof the brain is shown (360).

Question 360

Which characteristic sign would you expect to find onphysical examination?

a. Short metacarpals.b. Adenoma sebaceum (facial angiofibromas).c. Retinitis pigmentosa.d. Bilateral ptosis.e. Deafness.

Question 361

What is the mode of inheritance in the family shown(361)?a. Autosomal recessive.b. Autosomal dominant.c. X-linked recessive.d. X-linked dominant.e. Autosomal dominant with incomplete penetrance.

Affected male Unaffected male*

Affected female Unaffected female*

* Does not exclude inheritance of an autosomal recessivegene in either sex or an X-linked recessive disease in a female

336600

336611

Page 361: Rapid review of clinical medicine for mrcp part 2

360

In a life-long heavy smoker, the symptoms of persistentcough, weight loss, polydipsia and polyuria in thepresence of a normal blood glucose are indicative ofbronchial carcinoma complicated by hypercalcaemia.Hypercalcaemia is associated with nephrogenic diabetesinspidus. The hypercalcaemia may be secondary to bony

metastases or PTH-related peptide secretion. The fallsand confusion may be secondary to hypercalcaemia orcerebral metastases. In any case, the only test of theoptions given that would strongly suggest the diagnosis isa chest X-ray. While many candidates would have optedfor serum calcium as the correct answer, it is important tonote that hypercalcaemia is a consequence of manyconditions but is not a diagnosis.

Answer 359

c. Chest X-ray.

The patient has a history of mental retardation andepilepsy. Her mother also has epilepsy, raising thepossibility of a familial condition. The CT scan of the brainshows subependymal calcified nodules which representcalcified hamartomas. This finding is almost pathog -nomonic of tuberous sclerosis in a patient who has epilepsy.

Tuberous sclerosis is a neurocutaneous disorder thatis inherited as an autosomal dominant trait with variablephenotypic expression. It is characterized by epilepsy,mental retardation and facial angiofibromas (Vogt’s triad)and benign tumour in the brain, heart and kidneys. Theprevalence ranges from 1/5000 to 1/10,000 births.Mutations in two genes are responsible for the diseaseprocess. The first gene, TSC1, is on chromosome 9 andencodes the protein hamartin. The second gene, TSC2, ison chromosome 16 and encodes the protein tuberin. Theprecise functions of hamartin and tuberin are unknown,but animal studies suggest that both play an importantrole in brain development.

Only 50% of patients present with the classic Vogt’striad, therefore the diagnosis relies on the demonstrationof two major and one minor criteria or one major andtwo or more minor criteria from Table A. Other featuresinclude shagreen patches, hypomelanotic lesions andperiungual fibromas.

Short metacarpals are a feature of pseudo-hypoparathyroidism (see Question 159) and are associatedwith calcification of the basal ganglia. Bilateral ptosis has alarge differential diagnosis, but mitochondrial disorders area recognized cause of bilateral facial weakness andcalcification of the basal ganglia. Retinitis pigmentosa is

not a feature of tuberous sclerosis. Causes of retinitispigmentosa are shown in Table B.

Answer 360

b. Adenoma sebaceum (facial angiofibromas).

Table A Criteria for the diagnosis of tuberoussclerosis

Major criteria • Facial angiofibromas• Shagreen patches• Three or more hypomelanotic macules• Periungual fibromas• Renal angiomyolipoma• Cardiac rhabdomyoma• Lymphangiomyomatosis• Subependymal nodules• Subependymal giant cell astrocytomas

Minor criteria• Confetti skin lesions• Gingival fibromas• Bone cysts• Multiple renal cysts• Hamartomatous rectal polyps• Non-renal hamartomas• Retinal achromic patch• Cerebral white matter radial migration lines

Table B Causes of retinitis pigmentosa

• Familial (autosomal dominant)• Friedreich’s ataxia• Refsum’s disease• Abetalipoproteinaemia• Laurence–Moon–Biedl syndrome• Usher’s syndrome• Kearne’s Sayer syndrome• Alport’s syndrome

Answer 361

e. Autosomal dominant with incomplete penetrance.In the second generation there is an unaffected couplewith an affected son. One of the two parents hasinherited the gene but not expressed the phenotype.

Page 362: Rapid review of clinical medicine for mrcp part 2

Clinical Cases 361

A 20-year-old cyclist was seen by a cardiologist afterexperiencing a sudden syncopal episode while training fora major race. For the past six months he had complainedof deteriorating fitness – despite strenuous training – andhigher heart rates during training sessions. There was nohistory of chest pain, palpitation or breathlessness duringnormal exertion. He was a non-smoker and drank amaximum of 6 units of alcohol per week. He denied allforms of drug abuse.

His father died suddenly at the age of 33 years. Thecause of death was not entirely clear but was attributed toa form of ‘heart disorder’. His sister had recently seen a

cardiologist following a 2-year history of troublesomepalpitation and was in the process of having furtherinvestigations. The patient had not recently experiencedany form of flu-like illness.

On examination he had an athletic build. His heartrate was 60 beats/min and regular and his blood pressurewas 110/70 mmHg. The JVP was not raised. Thecardiac apical impulse was forceful. On auscultation therewas a soft systolic murmur at the left lower sternal edge,which resolved on standing, and a third heart sound.

The patient had the following ECG (362a):

Question 362

Echocardiography revealed a slightly dilated left ventriclewith good systolic function and a dilated right ventriclethat was hypokinetic. There was no evidence of anyvalvular abnormality. He underwent an exercise testfollowing which he experienced dizziness.

An ECG performed while he was symptomatic isshown below (362b).

A subsequent 24-hour ECG revealed >5000ventricular ectopic beats.

What is the most probable diagnosis?a. Right ventricular outflow tract tachycardia. d. Dilated cardiomyopathy.b. Hypertrophic cardiomyopathy. e. Multiple pulmonary emboli.c. Arrhythmogenic right ventricular cardiomyopathy.

336622aa

336622bb

Page 363: Rapid review of clinical medicine for mrcp part 2

362

The combination of T wave inversion in the rightventricular leads (V1–V3), dilated and hypokinetic rightventricle, ventricular tachycardia with left bundle branchmorphology and more than 1000 ventricular ectopicswith left bundle branch morphology in the context of afamily history of premature sudden cardiac death isconsistent with the diagnosis of arrhythmogenic rightventricular cardiomyopathy (Table).

ARVC is characterized by fibrofatty replacement ofthe right ventricle, resulting in right ventriculardysfunction and arrhythmias of right ventricular origin(LBBB morphology). The disorder is due to mutations ingenes encoding cell adhesion proteins. The two mainproteins identified thus far are desmoplakin andplakoglobin. The disorder has a prevalence of 1 in 5000.Over one-third of cases are familial, with an autosomaldominant inheritance pattern. Patients often experiencepalpitation, presyncope or syncope. Sudden death fromventricular fibrillation may be the first presentation andoccurs most commonly during physical exercise.

The ECG is abnormal in 90% of cases with symptomsand usually shows T-wave inversion in the rightventricular leads and depolarization abnormalities afterthe QRS complex (often termed epsilon waves).

Echocardiography may be normal but in gross casesdemonstrates aneurysms of the right ventricular wall.

The MRI scan is more diagnostic, revealing fattyinfiltration of the right ventricle.

Right ventricular biopsy is not generally performed toconfirm the diagnosis because the free wall of the rightventricle (which is most commonly affected) is thethinnest portion of the right ventricle and is at risk ofperforation, the disease is segmental, therefore false-negatives may be generated, and it is impossible todemonstrate transmural fibrofatty infiltration with anendomyocardial biopsy.

The diagnosis relies on family history, electro -cardiography and abnormalities of the right ventricle onimaging (echo, MRI and ventriculography).

Management involves abstinence from strenuousphysical exertion and antiarrhythmic therapy in the formof either beta-blockers or amiodarone. Implantation ofan ICD is reserved for patients who continue to havesyncope or VT despite antiarrhythmic drug therapy.

Answer 362

c. Arrhythmogenic right ventricular cardiomyopathy.

Causes of VT with LBBB morphology

• ARVC• Right ventricular outflow tract ventricular

tachycardia*• Dilated cardiomyopathy• Congenital cyanotic heart disease

* This is an important differential diagnosis but inthis case the resting ECG is normal and there areno morphological right ventricular abnormalitieson imaging. The condition is benign as opposed toARVC.

Question 363

This is a chest X-ray from an asymptomatic 50-year-oldbanker (363).

What is the diagnosis?a. Cryptogenic fibrosing alveolitis.b. Miliary tuberculosis.c. Healed chicken pox pneumonia.d. Pulmonary metastases.e. Histoplasmosis.

336633

Page 364: Rapid review of clinical medicine for mrcp part 2

Clinical Cases 363

A 54-year-old male presented with an acute painfulswollen left knee. There was no history of trauma or anysystemic features. There was a past history of a bleeding

peptic ulcer that was treated 3 months ago. Onexamination the temperature was 37.8C (100.0F). Theleft knee was tender and erythematous.

Investigations are shown.

Question 364

Hb 16 g/dlWCC 13 � 109/lPlatelets 359 � 109/lESR 78 mm/hCRP 92 g/lBlood cultures NormalJoint aspirate Needle-shaped crystals with

strong negative birefringence under polarized light

What is the most appropriate management of thepatient?

a. Diclefenac alone.b. Allopurinol.c. Indomethacin plus proton pump inhibitor.d. Indomethacin plus misoprostol.e. Inject joint with steroid.

Question 365

A 77-year-old male was seen in the memory clinic owingto progressive deterioration of cognitive function.According to his daughter he had been diagnosed withParkinson’s disease two years previously, for which hewas commenced on L-dopa. Over the past few months

he had become intermittently confused and hadhallucinations of monkeys in his garden. He hadexperienced several falls in the house and had beeninvestigated by the cardiologists for recurrent episodes ofsyncope, but no cause was identified.

On examination he had evidence of bradykinesia, mildtremor and mild cog-wheel rigidity. The mini mental testscore was 9. Serial 3 counting was satisfactory. Short-term memory was impaired.

Investigations are shown.FBC NormalESR 33 mm/hSerum B12 NormalSerum folate NormalU&E NormalBlood sugar NormalThyroid function Normal12-lead ECG SR; left axis deviationChest X-ray NormalUrinalysis NormalCT scan brain Mild global cerebral

atrophy

What is the diagnosis?a. Alzheimer’s disease.b. Pick’s disease.c. Multi-infarct dementia.d. Lewy body dementia.e. Side-effect of L-dopa.

Page 365: Rapid review of clinical medicine for mrcp part 2

The presence of hallucinations, falls and syncope in apatient with Parkinson’s disease is suggestive of dementiawith Lewy bodies, which is the most common form ofdementia associated with Parkinson’s disease. It ischaracterized by the presence of Lewy bodies in thebrainstem and the cortex. Lewy bodies are intracytoplasmicinclusions composed of alpha-synuclein and ubiquitin. Thedementia is slowly progressive. There are fluctuations incognition in relation to alertness and attentiveness. Memoryimpairment is mild in the early stages. Patients experiencerecurrent visual hallucinations of people and animals.Features of parkinsonism are generally mild. Daytimedrowsiness and sleepiness, falls, syncope and delusions arewell recognized. Patients with Lewy body dementia areparticularly sensitive to neuroleptic agents. There isconsiderable overlap between Parkinson’s disease, Lewybody dementia and Alzheimer’s disease. Daytime sleepinessor drowsiness appears to be more common in Lewy bodydementia than the other two entities.

Picks’s disease is a subtype of fronto-temporal dementiathat is characterized by Pick bodies in the neocortex and

hippocampus. The disorder presents with language and/orbehavioural abnormalities. The mean age of onset is thesixth decade. The disorder may occur as early as 35 yearsbut is rare after the age of 75 years. The presence ofbehavioural or language deficits manifested by early andprogressive changes in personality or loss of languagefunction in the absence of another cause, is highlysuggestive of Pick’s disease. Neuroimaging with CT scanor MRI will reveal atrophy of the anterior temporal andfrontal lobes. Extrapyramidal motor symptoms, bulbar andspinal motor neurone disease are recognizedmanifestations of Pick’s disease. There is currently notreatment available for Pick’s disease.

Vascular dementias are abrupt in onset followed bystep-wise deterioration. Physical examination usuallyreveals focal abnormalities consistent with previousstroke, and neuroimaging reveals multiple infarcts.

Alzheimer’s disease is the commonest cause ofdementia and is characterized by gradual and progressiveglobal diminution of cognitive function. Loss of short-term memory is the most prominent feature in the earlystages. There is considerable overlap betweenAlzheimer’s disease and the conditions described above.The diagnosis of Alzheimer’s disease is clinical.

364

The treatment options for acute gout include the use ofnon-steroidal anti-inflammatory agents, colchicine orinjection of the joint with steroids. The most effectivemethod of treating monoarticular gout is aspiration of thejoint with intra-articular steroid injection. This methodprovides rapid relief of symptoms and produces no systemicside-effects. NSAIDs are relatively contraindicated in a

patient with a recent history of a bleeding gastric ulcer.Although NSAIDs may be prescribed cautiously with eithera proton pump inhibitor or misoprostol, the use of intra-articular steroid is safer and more effective. Colchicine is analternative to NSAIDs but is also associated with frequentgastrointestinal side-effects and dermatologicalcomplications. Allopurinol may exacerbate acute gout but isthe drug of choice in preventing further attacks once theacute attack has been treated.

Answer 364

e. Inject joint with steroid.

Answer 365

d. Lewy body dementia.

There are multiple calcified nodules throughout both lungfields in an otherwise asymptomatic patient. The differentialdiagnosis of multiple nodular opacities in the lungs istabulated below. Patients with miliary TB are systemicallyunwell and complain of fever, cough and night sweats.Although histoplasmosis may present with multiple calcifiednodules in the lung, the patient is completely well whereaspatients with miliary histo plasmosis usually experienceprofound respiratory symptoms. Cryptogenic fibrosingalveolitis is associated with diffuse reticulonodular

shadowing in both lung bases. Few pulmonary metastasesmay become calcified but a common example includesfollicular adenocarcinoma of the thyroid.

Answer 363

c. Healed chicken pox pneumonia. Causes of diffuse calcified nodules in the lungs

Infections TBHistoplasmosisCoccidiocomycosisChicken pox in adulthood

Occupational dusts SilicosisCaplan’s syndrome

Pulmonary metastasesAlvelolar microlithiasis

Page 366: Rapid review of clinical medicine for mrcp part 2

Clinical Cases 365

A 68-year-old male with disseminated colonic carcinomawas admitted to hospital having collapsed at home. He hadbeen complaining of severe back pain for 48 hours, forwhich he was taking two different analgesic agents. He wasalso taking bronchodilator inhalers for chronic asthma.

On examination he was drowsy and had a Glasgowcoma score of 8. The pupils were small and equal and

reacted slowly to light. Inspection of the fundi was notpossible. The heart rate was 120 beats/min. The bloodpressure measured 100/60 mm Hg. The respiratory ratewas reduced to 8/min. The tone in the limbs wasnormal. The lower limb reflexes were brisk and theplantar response was extensor bilaterally.

Investigations are shown.

Question 366

Sodium 133 mmol/lPotassium 3.7 mmol/lUrea 8 mmol/lCalcium 2.7 mmol/lAlbumin 29 g/lArterial blood gases on 60% oxygen:

pH 7.2PaCO2 9.6 kPaPaO2 28 kPaHCO3 28 mmol/l

CT scan brain Multiple cerebral metastases

What is the most immediate treatment that isrequired?

a. Intravenous bicarbonate. b. Intravenous naloxone.c. Intravenous doxopram.d. Intravenous dexamethasone.e. Intravenous flumazenil.

Question 367

A 64-year-old male presented with a three-month historyof copious diarrhoea associated with lower abdominalcramps and weight loss. The patient moved his bowel upto 20 times per day. There was no blood in the stools.He had a past history of gallstones, for which heunderwent a cholecystectomy five years ago. He had notexperienced any episodes of biliary colic since surgery.The patient was also in the process of being investigatedfor intermittent dizziness that usually occurred aftermeals and was preceded by a burning sensation over theface, neck and chest. According to his wife he developeda red complexion during these episodes, which lasted for30–60 seconds at a time. On two occasions he hadalmost lost consciousness when he stood suddenly duringthese episodes. The patient had not travelled abroad forover eight months. He was not taking any medication.His father had ulcerative colitis and his paternal nephewhad gluten sensitive enteropathy.

On examination he had several telangiectasiae on hisface, neck and upper trunk. The JVP was raised withprominent V waves. On auscultation of the precordium,there was a soft systolic murmur. The chest was clear.Abdominal examination revealed a palpable liver edge 5 cm below the costal margin that had a firm andirregular consistency. Rectal examination was normal.

Investigations are shown.

Hb 11 g/dlMCV 80 flWCC 12 � 109/lPlatelets 200 � 109/lESR 68 mm/hSodium 130 mmol/lPotassium 3.4 mmol/lUrea 9 mmol/lAST 40 iu/lAlkaline phosphatase 320 iu/lBilirubin 16 μmol/lAlbumin 29 g/l

What is the diagnosis?a. Coeliac disease.b. Crohn’s disease.c. Disseminated colonic carcinoma.d. Carcinoma of the head of pancreas.e. Carcinoid syndrome.

Page 367: Rapid review of clinical medicine for mrcp part 2

366

The patient takes analgesia for chronic pain, presumablyrelated to the disseminated carcinoma. He presents withdrowsiness, miosis, reduced respiratory rate and arespiratory acidosis. The patient has tachycardia, which isprobably due to vasodilatation from the acidosis. The mostprobable cause of the presentation is opiate overdose, whichmay be accidental or deliberate. The immediate treatmentto reverse the effects is intravenous naloxone. Althoughbenzodiazepine overdose can present in identical fashion,the patient is more likely to have taken excess opiate for hispain rather than a benzodiazepine. Flumazenil, the antidotefor benzodiazepine overdose, should be considered only ifrespiratory depression is not reversed with naloxone.

The patient also has cerebral metastases, which may becausing raised intracranial pressure and may be

responsible for the pyramidal tract signs; however, thepresentation is more consistent with opiate overdose.Dexamethasone should be instituted if the patient hassymptoms of raised intracranial pressure or focalneurology once he is more conscious.

Tricyclic antidepressant overdose also presents withdrowsiness, tachycardia, hypotension and upgoingplantars and brisk reflexes. However, acidosis is due toreduced bicarbonate levels rather than retention ofcarbon dioxide. Patients with tricyclic antidepressantoverdose also have antimuscarinic features. In this respectone would expect the patient to have dilated rather thanconstricted pupils. Intravenous bicarbonate is reserved formanaging tricyclic antidepressant overdose.

Doxopram is a respiratory stimulant that is rarelyeffective and very rarely used in clinical practice nowadays.

Answer 366

b. Intravenous naloxone.

The patient has copious diarrhoea and weight loss as wellas intermittent dizziness associated with a redcomplexion, which would be consistent with carcinoidsyndrome. The presence of a firm and irregular liver edgeis highly suggestive of metastases to the liver which isusually required for the clinical manifestation of carcinoidtumours arising from the gastrointestinal tract.

Crohn’s disease and coeliac disease rarely causecopious diarrhoea and would not explain the flushing. Inulcerative colitis loose stool is often blood-stained.

Carcinoid syndrome is caused by tumours that havethe ability to store and synthesize polypeptides andbiogenic amines and prostaglandins. The tumourscommonly arise from the gut, notably the small bowel,appendix and rectum, and may arise from the lungs andpancreas. The tumours commonly produce serotonin, ametabolite of the amino acid tryptophan, histamine andkallikrein. These biogenic compounds are inactivated bythe liver. After metastasis to the liver, the amines aresecreted into hepatic veins and enter the systemiccirculation to cause symptoms.

In the vast majority of cases, the symptoms dependlargely on the proportion of serotonin, histamine orkallikrein produced by the tumour. In general, hindguttumours such as those arising from the rectum lack thedecarboxylase enzyme that catalyses the conversion oftryptophan to 5-hydroxytryptamine, therefore thesetumours are unable to cause the carcinoid syndrome.

Symptoms include flushing, diarrhoea, andbronchospasm. Flushing is the hallmark of carcinoidsyndrome and occurs in almost 90% of patients. Flushingis intermittent and affects the upper trunk and face. Thepatient feels warm during these episodes and mayexperience palpitation and dizziness due to profoundvasodilatation causing a drop in blood pressure. Flushingmay be precipitated by eating, defecation, consumingalcohol, emotion, pressure on the liver or generalanaesthesia. Diarrhoea is probably the most disablingsymptom and may occur up to 30 times per day. It maybe explosive and associated with abdominal cramps.Bronchospasm is present in 20% of cases and isexacerbated by beta-agonists.

Cardiac involvement results in a fibrogenic reactionaffecting the pericardium and the right heart valves. Theleft-sided cardiac valves are not affected unless there isbronchial involvement, since the lungs inactivatebioactive amines produced by carcinoid tumours. Cardiacinvolvement is a very poor prognostic marker.

Patients with carcinoid syndrome are at risk ofdeveloping pellagra owing to the util ization oftryptophan to form serotonin rather than nicotinic acid(the active form of niacin).

Treatment is usually with octreotide or alpha-interferon. Resection of hepatic metastases may providegood palliation.

Some tumours may produce polypeptides such asinsulin, gastrin, growth hormone, ACTH, vasointestinalpeptide and somatostatin.

Answer 367

e. Carcinoid syndrome.

Page 368: Rapid review of clinical medicine for mrcp part 2

Clinical Cases 367

A 16-year-old girl collapsed at school while playinghockey. According to the physical training tutor, she wasunconscious for a few seconds and then was completelyrousable and did not realize what had happened. Theepisode occurred without any warning symptoms. Therewas no history of trauma or head injury during the game,or any ill health before the game. There was no history ofepilepsy. She had experienced two previous syncopalepisodes at the ages of 8 and 11 years. On the firstoccasion she had witnessed a road traffic accident, and onthe second occasion she collapsed after receiving a hoaxtelephone call. She had been completely well since thelast syncopal episode. She was not taking any medication.She had a younger brother, aged 14, who was well. Herparents were well. Her mother had a past history ofsyncopal episodes during her adolescence, for which nocause had been found. Her maternal aunt was founddead in her bed at the age of 15, for which no cause wasfound at post-mortem.

On examination, the patient was 1.55 m tall andweighed 48 kg. Her heart rate was 65 beats/min andirregular. The blood pressure was 110/65 mmHg.Examination of the precordium was completely normal.There were no carotid bruits, and all peripheral pulseswere palpable. The chest was clear and abdominalexamination was normal. Examination of the centralnervous system, including the fundi, was completelynormal

Investigations are shown.The following day, the patient was being seen by the

consultant cardiologist and his team on the ward whenshe had a syncopal episode. The tracing from the ECGmonitor is shown (368b).

Question 368

Hb 14 g/dlWCC 12 � 109/lPlatelets 300 � 109/lBlood glucose 5 mmol/lECG (368a)Sodium 136 mmol/lPotassium 3.5 mmol/lUrea 4 mmol/l

1. What is the diagnosis?a. Wolff–Parkinson–White syndrome.b. Lown–Ganong–Levine syndrome.c. Hypertrophic cardiomyopathy.d. Congenital LQTS.e. Drug overdose.

2. What is the underlying abnormality in this disorder? a. Myocardial disarray.b. Accessory conduction pathway.c. Abnormal sodium/potassium channels in cardiac

myocytes.d. Fibrofatty replacement of the myocardium.e. Myocardial fibrosis.

3. How would you manage this patient immediately?a. Intravenous isoprenaline.b. Intravenous atropine.c. Intravenous adenosine.d. A beta-blocker.e. Atrial pacing.

4. What other important investigation would you perform?

a. Cardiac enzymes.b. Echocardiogram.c. Electrophysiological studies.d. Family screening.e. Serum cholesterol.f. Serum potassium.g. Serum magnesium.

336688aa

336688bb

Page 369: Rapid review of clinical medicine for mrcp part 2

368

The combination of a family history of premature suddendeath, long QT and polymorphic ventricular tachycardiaassociated with syncope is diagnostic of idiopathic LQTS.The condition is familial in up to 90% of cases, and canbe inherited both as an autosomal dominant and recessivetrait. The rarer variant (autosomal recessive) is associatedwith congenital nerve deafness and known as theJarvell–Lange–Neilsen syndrome. The more commonautosomal dominant form is often known as theRomano–Ward syndrome.

In congenital LQTS defects within transmembraneion channels produce abnormalities in cardiacrepolarization, leading to an increased propensity forsyncope due to fatal ventricular arrhythmias in situationscausing increased sympathetic activity. Syncope may beprecipitated by physical exertion, intense emotion andsudden loud auditory stimuli. The classic arrhythmia is aform of polymorphic ventricular tachycardia termed‘torsades de pointes’, which may degenerate toventricular fibrillation, causing sudden death. Thus far,abnormalities have been demonstrated in genes encodingboth potassium and sodium ion transport channelproteins which are located on chromosomes 11, 3, 4 and7 (see below).

Myocardial disarray is a feature of HCM; an accessoryconduction pathway would indicate Wolff–Parkinson–White or Lown–Ganong–Levine syndrome;fibrofatty replacement of the myocardium would indicatearrhythmogenic right ventricular cardiomyopathy; andmyocardial fibrosis is a finding common to allcardiomyopathies and ischaemic heart disease.

Treatment is aimed at reducing the effects of thesympathetic nervous system on the conduction tissue.The immediate treatment of choice to prevent furtherventricular tachycardia is beta-blockers, which may begiven intravenously in the first instance. In situationswhere this produces a profound bradycardia a permanentpacemaker is also implanted. There is some evidence thatintravenous magnesium may arrest ventriculararrhythmias in the acute state, even when the serummagnesium level is normal. Patients who continue tohave ventricular arrhythmias despite heavy beta-blockadeare implanted with an automatic cardioverterdefibrillator. It is worth noting that therapy withisoprenaline, a sympathomimetic agent which is useful inacquired LQTS, is contraindicated in congenital LQTS.

Screening of first-degree relatives is very important inthis condition. In approximately 60% of cases thediagnosis of idiopathic LQTS is made during screening of

first-degree relatives of affected patients. Low serummagnesium and potassium are causes of acquired LQTSwhich should always be excluded. In this case, there is noobvious cause for an electrolyte deficiency. No marks areawarded for electrophysiological studies because thediagnosis and the potential for VT are obvious.

Acquired causes of LQTS are usually due to drugs andelectrolyte disturbances (Table). In such cases, treatmentinvolves correction of the electrolyte disturbance andwithdrawal of the offending drug. Rectifying serumelectrolytes and drug levels can take several days, andprevention of polymorphic VT relies on shortening theQT-interval by atrial pacing at a rate of 60–70beats/min.

Genetics of LQTSCongenital LQTS is a recognized but rare cause of suddendeath in young adults. Death is due to polymorphictachycardia (torsades de points) degenerating intoventricular fibrillation. Syncope is common in affectedpatients, and is due to transient cerebral hypoperfusionfrom VT. Common stimuli of polymorphic VT includephysical exertion, intense emotion and auditory stimuli.Genes encoding sodium and potassium ion channels causedisease. To date, there are four different loci onchromosomes 3, 4, 7 and 11. On chromosome 7, there is amutation in the HERG gene (potassium channelabnormality). Other mutations include the SCN5A(chromosome 3; sodium ion channels) and KVLQT1(chromosome 11; potassium ion channels).

(See Question 353.)

Answer 368

1. d. Congenital LQTS.2. c. Abnormal sodium/potassium channels in cardiac

myocytes.3. d. A beta-blocker.4. d. Family screening.

Acquired causes of long LQTS

• Drugs• Class Ia and Ic antiarrhythmic agents• Tricyclic antidepressants, neuroleptic agents• Erythromycin• Ketoconazole• Certain antihistamines• Cisapride• Electrolyte disturbances• Hypokalaemia • Hypomagnesaemia• Hypocalcaemia• Cardiovascular diseases• Myocardial infarction• Complete heart block• Sick sinus syndrome• Mitral valve prolapse• Others• Subarachnoid haemorrhage• Liquid protein diet

Page 370: Rapid review of clinical medicine for mrcp part 2

Clinical Cases 369

Question 369

A 44-year-old man presented with recent onset of epigastricburning that was precipitated by hunger or after a heavymeal. His appetite was unchanged and his weight wasstable. He consumed 21 units of alcohol per week but was

a non-smoker. The patient was not on any medication.There was no family history of carcinoma of the stomach.The urease breath test for Helicobacter pylori was positive.

What is the next management step?a. Reassure.b. Start omperazole, amoxycillin and clarithromycin for 7–10 days.c. Start omeprazole, amoxycillin and clarithromycin for one month.d. Request urgent upper gastrointestinal endoscopy.e. Cimetidine therapy.

Question 370

A 72-year-old patient with previous admissions forexacerbation of chronic obstructive airways disease wasadmitted to hospital with a six-day history of coughproductive of yellow sputum and dyspnoea. He smokedfive cigarettes per day but had previously been a heavysmoker. He had a past medical history of ischaemic heartdisease and had had a stent to the right coronary arteryfollowing a myocardial infarction two years ago. Hismedications included inhaled iprotropium bromide, inhaledsalbutamol and inhaled pulmicort. His GP had commencedamoxycillin five days previously.

On examination he was cyanosed. The respiratory ratewas 30/min. The heart rate was 120 beats/min andregular. The blood pressure measured 150/90 mmHg.Both heart sounds were soft. Auscultation of the lungfields revealed coarse crackles throughout the lungs.

Investigations are shown.

Hb 16 g/dlWCC 17 � 109/lPlatelets 400 � 109/lSodium 138 mmol/lPotassium 4.0 mmol/lUrea 7 mmol/lCreatinine 120 �mol/lChest X-ray Hyperinflated lung

markings with reduced vascular markings.

Arterial blood gases (2 l oxygen/min):pH 7.1PaCO2 8.2 kPaPaO2 6 kPaHCO3 30 mmol/l

12-lead ECG Atrial fibrillation/RBBB (ventricular rate 120)

Urinalysis Blood +

What is the most effective management of his presentation?a. IV aminophylline.b. Non-invasive positive pressure ventilation.c. IV doxapram.d. IV heparin.e. IV hydrocortisone.

Page 371: Rapid review of clinical medicine for mrcp part 2

370

The patient has dyspepsia and a positive urease breathtest suggestive of Helicobacter infection. Such patientsshould receive eradication therapy. Recommendederadication therapy for H. pylori infection includes a full-dose proton pump inhibitor twice daily, amoxycillin 1 gtwice daily and clarithromycin 500 mg twice daily for oneweek. An alternative regime is full-dose proton pumpinhibitor twice daily, metronidazole 400 mg twice dailyand clarithromycin 250 mg twice daily for 7–10 days.

The therapeutic recommendations in patients withuncomplicated dyspepsia are as follows:1. Ensure all ulcerogenic medications have been stopped.2. Treat dyspepsia with a proton pump inhibitor for a

month.3. If symptoms persist the patient should be tested for

H.pylori infection with the intent to treat. The mostuseful test for H. pylori is the urease breath test. First-line treatment regimes are described in the firstparagraph above. Most duodenal ulcers (more than90%) respond to eradication therapy.

4. Patients who develop recurrence of symptoms aftereradication therapy, particularly those aged over 55years of age, should be referred for upper gastro -intestinal endoscopy.

5. Upper gastrointestinal endoscopy should also beperformed on patients presenting with features of

chronic gastrointestinal bleeding, progressive uninten -tional weight loss, dysphagia, persistent vomiting, irondeficiency anaemia, epigastric mass or suspicious bariummeal to exclude a malignant peptic ulcer.

6. In patients noted to have a duodenal ulcer at endoscopy, aurease breath test is performed to check for persistence ofH. pylori infection (assuming that the patient has alreadyreceived eradication therapy at initial presentation). Inpatients with evidence of H. pylori infection, second-lineeradication therapy is offered (assuming the patient hasrelapsed on first-line eradication therapy). If the patientbecomes asymptomatic after this, no further investigationis necessary. Patients with evidence of persistent H. pyloriinfection on eradication therapy are treated with long-term, low-dose antisecretory therapy with either protonpump inhibitors or H2 receptor blockers. Some patients(5%) have duodenal ulcers in the absence of H. pyloriinfection. These patients are also managed with long-termantisecretory drugs.

7. Patients with gastric ulcers at endoscopy should bebiopsed to exclude malignancy. Gastric ulcer patients aretreated similarly to patients with duodenal ulcers.Treatment with eradication therapy leads to healing ofthe vast majority of gastric ulcers. Unlike patients withduodenal ulcers, patients with gastric ulcers should haverepeat endoscopy at six weeks, even in the absence ofsymptoms, to ensure healing of the ulcer, as gastric ulcersare associated with higher complication rates and areoften malignant. Failure of a gastric ulcer to heal may bean indication of malignancy and warrants repeat biopsies.

Answer 369

b. Start omeprazole, amoxycillin and clarithromycinfor 7–10 days.

The patient has a severe infective exacerbation of chronicobstructive pulmonary disease as evidenced by the markedhypoxia and respiratory acidosis. Early NIPPV is requiredand has been shown to be as effective as invasive ventilation(without the problems associated with invasive ventilation)in terms of expediting recovery and reducing mortality.Patients who should be considered for NIPPV include thosewho have a moderate to severe dyspnoea, a respiratory rateof >25/min, an arterial pH <7.35 and a PaCO2 >6 kPa.

There is no place for IV aminophylline in themanagement of acute exacerbations of COPD. It is noteffective and increases the risk of arrhythmias in thehypoxic patient. Similarly doxapram, a respiratorystimulant, is no longer used in the management of acuteexacerbations of COPD.

Short-term high-dose steroids (two-week course) havebeen shown to be effective in the management ofexacerbations of COPD (NEJM 1999; 340 (25);1941–47) (Table). Steroids cause a modest increase in

FEV1 and reduce in-hospital stay. Steroid usage for morethan two weeks does not have additional benefit. Thebenefit from systemic steroid therapy does not exceed sixmonths.

Answer 370

b. Non-invasive positive pressure ventilation.

Management of exacerbations of COPD

• Oxygen to maintain PaO2 ≥ 8 kPa (or O2 satura-tion ≥ 90%) is essential to prevent tissue hypoxia.

• Inhaled bronchodilators (beta-agonists andanticholinergic drugs) are the mainstay of treatment.Anticholinergic bronchodilators are most effective.Beta-agonist inhalers have an additive effect.

• The use of antibiotics remains controversial and isreserved for patients with copious sputumproduction. Amoxycillin, ampicillin, septrin andtetracyline have been shown to be effective. Newerantibiotics such as the quinolones and themacrolides such as clarithromycin have not beenshown to be superior to older antibiotics.

• Short-term high dose steroids for up to two weeks.• Early NIPPV in patients with severe exacerbation

associated with respiratory failure.• Aminophylline and doxapram are not recommended.

Page 372: Rapid review of clinical medicine for mrcp part 2

Clinical Cases 371

A 50-year-old man presented with a 24-hour history ofheamatemesis and malena. There was no precedinghistory of abdominal discomfort. He consumed80–100 units of alcohol per week.

On examination he was pale. His heart rate was110 beats/min and blood pressure measured90/60 mmHg.

Investigations are shown.

The patient was resuscitated with 2 units of blood and2 litres of dextrose 5% and underwent urgent endoscopy,which revealed a bleeding oesophageal varix.

Question 371

Hb 10 g/dlWCC 16 � 109/lPlatelets 80 � 109/lPT 22 sSodium 131 mmol/lPotassium 3.2 mmol/lUrea 14 mmol/lCreatinine 90 �mol/lAST 160 iu/lBilirubin 32 μmol/lAlbumin 28 g/l

What is the best treatment to prevent furtherbleeding?

a. Intravenous fresh frozen plasma.b. Sclerotherapy of the bleeding varix.c. Band ligation of the bleeding varix.d. Intravenous octreotide.e. Intravenous glypressin.

A 50-year-old male presented with collapse after a febrileillness. He had been generally unwell for almost twoyears, complaining of fatigue and listlessness. His GP hadcommenced him on antidepressant medication after hecomplained of reduced appetite, constipation and loss oflibido. The patient had recently given up his job as abank clerk owing to inability to cope with the pressure.He had been taking antihypertensive medication that had

recently been stopped after the development of posturalhypotension.

On examination he was pale. He had sparse axillary hair.The blood pressure on admission was 80/50 mmHg. Therewas an appendicectomy scar which appeared normal. Bothheart sounds were normal. Neurological examinationrevealed diplopia on lateral gaze on the left side.

Investigations are shown.

Questions 372 and 373

Hb 10 g/dl WCC 7 � 109/lPlatelets 200 � 109/lMCV 86 flSodium 130 mmol/lPotassium 4 mmol/lUrea 5 mmol/lGlucose 4 mmol/lTSH 0.3 mu/l

(NR 0.4–3.6 mu/l)Chlolesterol 6.9 mmol/lChest X-ray NormalECG Sinus bradycardia rate

60 beats/min

Question 372What is the full diagnosis?

a. Primary hypothyroidism.b. Addison’s disease.c. Secondary hypothyroidism.d. Herpes encephalitis.e. Panhypopituitarism.

Question 373What treatment is required immediately?

a. IV saline 0.9%.b. IV dextrose 50 ml (50%).c. IV tri-iodothyronine.d. IV hydrocortisone 100 mg.e. IV acyclovir.

Page 373: Rapid review of clinical medicine for mrcp part 2

Algorithm for the management of a variceal haemorrhage

372

The patient has a history of alcohol abuse and presents witha significant oesophageal variceal haemorrhage as defined bya systolic BP <100 mmHg or a pulse >100/min. Varices area common source of bleeding in patients with cirrhosis ofthe liver and account for almost one-third of all deaths fromthis condition. A variceal bleed is associated with a mortalityof approximately 50%. Aggressive resuscitation with bloodand colloids and achievement of haemostasis saves lives.Following resuscitation, the first method of choice is band

ligation of the bleeding varices (see algorithm [371] andTable for management).

Answer 371

c Band ligation of bleeding varix.

Upper gastrointestinal haemorrhage ? varices

Endoscopy available?

Yes No

Oesophageal varices Gastric varices

Band ligation orsclerotherapy

Gastro-oesphageal

varices

Isolatedgastricvarices

Uncontrolled Controlled

Treat as oesphagealvarices

Balloontamponade

Band eradicationprogramme

TIPSS/surgery

Injectionwithfibrin

glue orTIPSS

Recurrentbleed

Follow up at three and sixmonths and then annually

Refer for TIPSSor surgery

Goals in the management of bleeding varices

General1. Restore circulation to keep systolic blood pressure

≥ 90 mmHg and urine volume ≥ 50 ml/h2. Maintain Hb >9 g/dl3. Correct clotting with fresh-frozen plasma in

patients with raised prothrombin times4. Platelet transfusion in patients with platelet count

<50 � 109/l in the presence of active bleeding.Note large transfusions may produce dilutionalthrombocytopenia

5. Prevent aspiration of vomitus by gastricdecompression via a nasogastric tube

6. Reduce risk of encephalopathy with lactulose,ensuring the existing hypokalaemia is corrected

7. Antibiotic prophylaxis with cephalosporin orquinolones has been shown to reduce overallmortality from infective complications in patientspresenting with variceal haemorrhage

Specific (371)1. Following resuscitation the patient should undergo

endoscopy with band ligation of bleeding varices.Band ligation is superior to sclerotherapy and isassociated with a lower complication rate.Sclerotherapy should be performed if band ligationis not possible owing to continued bleeding

2. Glypressin and somatostatin (or octreotide) areeffective at controlling acute variceal bleeding. Bothdrugs act by causing mesenteric arteriolarvasoconstriction and reduction of portal pressures.Somatostatin is better tolerated than glypressin.These drugs are usually reserved for situationswhere endoscopy is not available

3. Gastric varices are not usually amenable to bandligation and are generally treated with injection offibrin glue or transjugular intrahepaticportosystemic shunt

4. When bleeding is difficult to control (i.e. there is aneed to transfuse more than 4 units of blood toachieve an increase in systolic BP of 20 mmHg orto 70 mmHg or more, and/or an inability toachieve a pulse rate reduction to less than 100beats/min or a reduction of 20/min frombaseline), a Sengstaken–Blakemore tube should beinserted until further endoscopic treatment, TIPSSor surgery is instituted

Eradication

The patient presents with collapse following a febrile illnessbut has been generally unwell with fatigue and depressionfor almost two years. He has hypotension, sparse body hair,a low sodium and low TSH. The symptoms of fatigue, lowaffect and loss of libido may be attributed to stress ordepression. However, in the context of the overall

Answers 372 and 373

Answer 372e. Panhypopituitarism.

Answer 373d. IV hydrocortisone 100 mg.

337711

Page 374: Rapid review of clinical medicine for mrcp part 2

presentation they are typical of hypopituitarism. Fatigue andloss of libido are early features of gonadotrophin deficiency.Gradual loss of secondary sexual characteristics and reducedmuscle bulk follow later. Depression could be explained bydeficiency of any of the anterior pituitary hormones. Thecollapse following a febrile illness and the low sodiumalmost certainly represent significant cortisol depletion dueto low ACTH reserve. The low TSH clinches the diagnosis,as secondary hypothyroidism can only occur as a result of apituitary disorder. The extra-ocular muscle dysfunctionsuggests that the most probable cause of thehypopituitarism is a pituitary tumour (see Table A for thecauses of hypopituitarism). Pituitary tumours causeheadache and localized pressure effects. Compression of theoptic chiasm causes bitemporal hemianopia and pressure onthe cavernous sinus may result in paralysis of the extra-ocular muscle function.

Although the presentation may also be explained byAddison’s disease (primary hypoadrenalism) there are twofeatures which favour a low ACTH reserve rather thanprimary adrenal gland failure. Firstly, the patient has anormal potassium level. Potassium concentrations aregoverned by aldosterone, which is not influenced byACTH. Aldosterone levels are normal or increased inpanhypopituitarism. The low sodium level inpanhypopituitarism is due to SIADH caused by cortisoldeficiency. Secondly, patients with low ACTH do notexhibit hyperpigmentation. Hyperpigmentation inAddison’s disease is explained by the fact that ACTH issecreted as part of a large hormone precursor pro-opiomelanocortin, which is cleaved to produce bothACTH and melanocyte stimulating hormone. The latteris responsible for hyperpigmentation that is mostnoticeable in skin creases and scars (Table B).

Manifestations of hypopituitary crisis include collapse orcoma. The management includes treatment ofhypoglycaemia if present followed by urgent administrationof intravenous hydrocortisone 100 mg, which should becontinued six hourly. Circulation is restored withintravenous saline. The patient should undergo an urgentMRI brain scan to exclude pituitary apoplexy as this is an

indication for urgent neurosurgical referral. Thyroidhormone replacement is in the form of intravenous tri-iodothryronine. However, thyroid replacement shouldonly be given after intravenous hydrocortisone has beenadministered, otherwise the effect of cortisol deficiencybecomes even more pronounced.

Clinical Cases 373

A 31-year-old female presented with acute onset ofswelling of the neck after attending the funeral of a closefriend. On examination she had well-demarcated oedemaof the lips, eyes and ear lobes. The heart rate was100 beats/min. The blood pressure was 90/50 mmHg.The respiratory rate was 24 /min. The chest was clear.

She had previously attended the Accident andEmergency Department with intermittent ankle swellingand abdominal pain. Her father had suffered with similarepisodes for many years but had never had to seek medicalattention.

Question 374

What is the most effective immediate management ofthis patient?

a. Emergency tracheostomy.b. Subcutaneous adrenaline.c. Intravenous corticosteroids.d. C1 esterase inhibitor concentrate.e. Fresh-frozen plasma.

Table A Causes of panhypopituitarism

Cause NotesPituitary masses Tumours, cystsExtra-pituitary tumours CraniopharyngiomaPituitary surgeryPituitary irradiationSheehan syndrome Infarction after massive

haemorrhage,e.g. during child birth

Pituitary apoplexyInfiltrative disorders Haemochromatosis,

lymphocytic hypophysitisGenetic disorders Mutation in PROP-1 or

Pit-1Hypothalamic disorders

Table B Differential diagnosis of Addison’sdisease and panhypopituitarism

Symptom Addison’s Panhypo-disease pituitarism

Lassitude, malaise, Common CommondepressionLoss of body hair Recognized CommonHyperpigmentation Common Not part of

disorderOcular signs Not present Dependent

on cause*Collapse Recognized Only occurs

in severe ACTH depletion

Low sodium Common CommonHigh potassium Characteristic Not part of

disorder*Present secondary to tumour

Page 375: Rapid review of clinical medicine for mrcp part 2

374

Hereditary angio-oedema is a genetic disorder inheritedas an autosomal dominant trait that results in a deficiencyof C1 esterase inhibitor. Deficiencies in this protein allowunchecked activation of the classic complement pathwayand other biochemical systems.

Patients can present with any combination of cutaneousangio-oedema, abdominal pain, or acute airwayobstruction. Prior to the development of effective therapy,the mortality rate was 20–30%. Typically, presentation is inlate childhood or early adolescence following trauma,infection, dental procedures, or emotional stress. There isan increasing frequency and severity of episodes duringpuberty, menstruation and ovulation. The acquired formtypically occurs at an older age and may not be associatedwith trauma. The symptoms of C1 esterase inhibitordeficiency range in severity from a minor inconvenience tolife-threatening oedema.

Al though preventab le and t rea tab le , thecomplications of this disease do not respond well tothe usual therapies for angio-oedema and are morerefractory to the use of intramuscular adrenaline,antihistamines and steroids. Therefore, establishmentof the correct diagnosis is critical. The managementof hereditary angio-oedema is outlined in Table.

Acquired C1 esterase inhibitor deficiency may occur withlymphoproliferative disorders and autoantibodies targetedagainst C1 esterase inhibitor. The presentation is similar tohereditary angio-oedema. Patients with acquired C1 esterasedeficiency also have deficiency in C2 and C4 levels.

Answer 374

d. C1 esterase inhibitor concentrate

Management of hereditary angio-oedema

• Ensure airway is unobstructed (tracheostomy incases of severe upper airway obstruction)

• Treat circulatory collapse in the event of acuteanaphylaxis

• Intramuscular adrenaline 1 in 1000• C1 esterase inhibitor concentrate is the most

effective therapy but it is not readily available. If C1esterase inhibitor concentrate is unavailableimmediately fresh-frozen plasma may be given inthe interim. FFP has been given to acutely repleteC1 esterase inhibitor and as prophylaxis pre-opera -tively and before labour. However, this preparationalso provides substrate for C1 Inh, which mayparadoxically worsen the clinical situation

• Stanozolol and danazol are useful at increasing C1esterase inhibitor levels acutely and prophylacticallyand have been shown to be effective in themanagement of hereditary angio-oedema

(See Question 155.)

A 17-year-old female was admitted with a six-week historyof malaise and joint pains. She had pain affecting her neck,left shoulder and the small joints of both her hands. Shehad intermittent fever associated with profuse sweating. Shewas born in Kenya and travelled to England for furtherstudies two years ago. She had not travelled abroad since.She was well while in Kenya, although she had had twomild attacks of malaria at the ages of 5 years and 12 years.She was a non-smoker and a teetotaller. Two days beforecoming into hospital she developed some sharp infra-mammary chest pain, which was worsened by movement,deep inspiration and on lying flat. There was no history ofcough or sputum production. Her appetite was slightlyreduced and she had lost 2 kg in weight.

On examination, she appeared unwell. She had atemperature of 39.2°C (102.6°F) and a widespread rash,

shown in (375a). The heart rate was 110 beats/min andblood pressure 110/65 mmHg. She had small, non-tender, palpable lymph nodes in the anterior cervicaltriangle. On examination of the cardiovascular system shehad a very soft systolic murmur at the left lower sternalangle, and a soft third heart sound. The chest was clear.Abdominal examination revealed a palpable spleen 3 cmbelow the costal margin. She had a few shotty lymphnodes in the inguinal region.

Cervical spine movements were slightly limited owingto pain. Her hands were swollen and tender. The leftshoulder was painful to abduct, but there was no obviousswelling of the joint. Examination of the central nervoussystem and fundoscopy were normal.

Investigations are shown – see opposite page.

Question 375

Page 376: Rapid review of clinical medicine for mrcp part 2

Clinical Cases 375

An obese woman was diagnosed as having pernicious anae -mia when she presented with lethargy and was found to havea raised MCV, low B12 level and intrinsic factor antibodies.

A blood count performed at the time was as follows:

She was treated with intramuscular B12 injections for ayear but continued to complain of lethargy.

Blood results were as follows:

Question 376

Hb 9.1 g/dl WCC 7 � 109/lPlatelets 250 � 109/lMCV 110 fl

Hb 11 g/dl WCC 7.4 � 109/lPlatelets 260 � 109/lMCV 105 fl

Which two other coexisting diagnoses should be considered?

a. Myelodysplasia. b. Chronic liver disease.c. Folate deficiency.

d. Orotic aciduria.e. Carcinoma of the stomach.f. Hypothyroidism.g. Chronic renal failure.

h. Myeloma.i. Crohn’s disease.j. Iron deficiency.

Hb 10 g/dlWCC 17 � 109/lPlatelets 470 � 109/lMCV 82 flMalaria parasite AbsentESR 86 mm/hCRP 54 g/lSodium 134 mmol/lPotassium 3.8 mmol/lUrea 5 mmol/lCreatinine 70 �mol/lBlood cultures × 3 NegativeThyroxine 98 nmol/lTSH 1.1 mu/lECG (375b)Chest X-ray NormalEchocardiogram (375c)Rhesus factor AbsentUrinalysis Trace of protein

No blood

What is the most probable diagnosis?a. Adult-onset Still’s disease.b. Systemic lupus erythematosus.c. Brucellosis.d. Infective endocarditis.e. Lymphoma.

337755aa 337755bb

337755cc

Page 377: Rapid review of clinical medicine for mrcp part 2

376

The patient was diagnosed as having pernicious anaemiaon the basis of a large MCV, low B12 levels and intrinsicfactor antibodies. Indeed, an MCV >110 fl is highlysuggestive of a megaloblastic marrow (i.e. B12, folatedeficiency or treatment with cytotoxic drugs). Thepatient has responded partly to B12 injections asevidenced by the increased haemoglobin and partialimprovement in the MCV. However, she continues toremain symptomatic and has subnormal Hb and MCV.

While there are several causes of raised MCV, the twomost probable diagnoses in this case include coexistingfolate deficiency or hypothyroidism. Coexistent alcoholabuse may also explain the failure to respond completely.

Folate deficiency may coexist with B12 deficiency inpatients with chronic malabsorption or in patientssuffering from alcohol abuse.

Although hypothyroidism is a recognized cause ofmacrocytosis, approximately 10% of patients with auto-immune hypothyroidism also have pernicious anaemia.Most physicians would normally exclude coexisting auto-immune hypothyroidism in patients with perniciousanaemia. Causes of macrocytosis are tabulated as shown.

Answer 376

c. Folate deficiency.f. Hypothyroidism.

The characteristics of the chest pain are consistent withacute pericarditis, which is confirmed on the ECG.The patient has malaise, fever, arthralgia, a rash,lymphadenopathy, arthritis and splenomegaly.

The differential diagnosis is between adult-onsetjuvenile chronic arthritis or Still’s disease, SLE, andlymphoma. The high CRP is against the diagnosis of SLE.While lymphoma is quite possible, the seronegativearthritis, neutrophil leucocytosis, high fever andmaculopapular rash are best explained by Still’s disease.Adult Still’s disease is rare. It affects young adults agedbetween 16 and 35 years. The characteristics are identicalto those of juvenile chronic arthritis in younger patients.The diagnosis is essentially clinical, and the clinical criteriaare outlined below (Table). Antinuclear factor is negativein Still’s disease. Generalized lymphadenopathy is relativelycommon, and lymph node biopsy shows only reactivechanges. If both these tests are negative, then adult Still’sdisease is the most likely diagnosis.

Infective endocarditis is unlikely with three negativeblood cultures. The echocardiogram (375c) reveals anormal aortic and mitral valve, and right-sided endocarditisis most unusual in young, previously healthy individualsunless there is a history of intravenous drug abuse.Rheumatic fever may be suspected in a woman living inEast Africa with a murmur, fever, arthritis and pericarditis;however, there is no history of streptococcal infection, andher symptoms are relatively long-standing for rheumaticfever. In this case, the murmur is likely to represent

hyperdynamic circulation due to the fever; soft systolicmurmurs and third heart sounds are relatively common inyoung individuals with fever. Malaria is unlikely after twoyears of being completely asymptomatic while in England.While Plasmodium malariae infections can be chronic, theydo not present with frequent high fever and sweats ofsudden onset. Brucellosis partly fits the clinical presentation;however, it is important to remember that it has beeneradicated in the UK through pasteurization of milk.Sarcoidosis may mimic Still’s disease, but the absence ofocular, respiratory and skin signs make it less probable.Familial Mediterranean fever is most unlikely for tworeasons. First, it has a predilection for Jews and Arabs;second, it usually causes a monoarticular arthropathy.

In this case, ANF and lymph node biopsy arenecessary to exclude SLE and lymphoma, respectively.

Answer 375

a. Adult-onset Still’s disease.

Diagnosis of adult-onset Still’s diseaseRequires each of:1. Quotidian fever >39°C (102.2°F)2. Leucocytosis >15 × 109/13. Arthralgia/arthritis4. Macular/maculopapular rash

Plus two of:(a) Serositis (pleuritic/pericarditic)(b) Splenomegaly(c) Generalized lymphadenopathy(d) Negative theumatoid factor(e) Negative anti-nuclear factor

Page 378: Rapid review of clinical medicine for mrcp part 2

Clinical Cases 377

Causes of macrocytosisCause NotesB12 or folate MCV >110 fl

deficiencyCytotoxic drugs MCV >110 flInherited disorders Orotic aciduria

of DNA synthesisHypothyroidismLiver diseaseAlcoholism Even in the absence of

established liver diseaseMyelodysplastic

syndromeHaemolysis Increased reticulocytes in

the circulationDrugs Zidovudine (increasingly

common cause of macro-cytosis), azathioprine, methotrexate, hyroxyurea, ddI, ddC

MyelomaHyperlipidaemia

A 42-year-old woman was rescued from a house firewhere she had become unconscious. On admission shewas drowsy and had a marked wheeze.

Blood results were as follows:

Question 378

Arterial blood gases on 28% FiO2:pH 7.2PO2 28 kPaPCO2 3.4 kPaBicarbonate 12 mmol/lO2 saturation 100%

Serum lactate 5.6 mmol/lCarboxyHb level 10%

List two immediate management steps from thefollowing list:a. 60% oxygen.b. 40% oxygen.c. 100% oxygen.d. Intravenous bicarbonate.e. Intravenous sodium thiosulphate.f. Intravenous hydrocortisone.g. Nebulized salbutamol.h. Nebulized ipratropium bromide.i. IV acetylcysteine.j. IV adrenaline.

A 58-year-old patient was noted to be in atrial fibrillationprior to ENT surgery. He was asymptomatic. An electivecardioversion was planned to try and restore sinusrhythm. There was no history of hypertension or valvedisease. The patient had never experienced a transientischaemic attack or stroke. On examination the heart ratewas 80 beats/min. Both heart sounds were normal andthere were no murmurs.

Question 377

What treatment is most effective at preventing strokeafter the restoration of sinus rhythm?

a. Aspirin.b. Warfarin for two weeks prior to cardioversion

and two weeks afterwards.c. Warfarin for 3–4 weeks prior to cardioversion

and four weeks afterwards.d. Warfarin for four weeks prior to cardioversion

only.e. Lifelong anticoagulation.

Chronic renal impairment causes a normochromic,normocytic anaemia. Carcinoma of the stomach isassociated with pernicious anaemia and would normallyresult in a normocytic or microcytic anaemia (blood lossanaemia). Iron deficiency causes a microcytic anaemia.

Page 379: Rapid review of clinical medicine for mrcp part 2

378

Smoke inhalation is associated with four majorconsequences (see Table).

This question tests the candidate’s knowledge of theconsequences of inhalation of systemic toxins, whichinclude carbon monoxide and hydrogen cyanide. Thesesubstances impair the delivery and/or utilization of oxygenand may result in systemic tissue hypoxia and rapid death.

Carbon monoxide is the main cause of smoke-relatedfatalities and causes almost 80% of all smoke inhalationdeaths. Symptoms and signs include headache, nausea,malaise, altered cognition, dyspnoea, angina, seizures,cardiac arrhythmias, congestive heart failure and/or coma.

Hydrogen cyanide is a highly toxic compound thatcan be formed in the high temperature combustion of anumber of common materials such as polyurethane,nylon, wool and cotton. Cyanide binds to a variety ofiron-containing enzymes, the most important of which isthe cytochrome a-a3 complex, which is critical forelectron transport during oxidative phosphorylation. Bybinding to this molecule, minute amounts of cyanide caninhibit aerobic metabolism and rapidly result in death.

The typical clinical syndrome due to cyanidepoisoning is one of rapidly developing coma, apnoea,cardiac dysfunction and severe lactic acidosis in

conjunction with a high mixed venous O2 and a lowarteriovenous O2 content difference.

The diagnosis of CO poisoning requires estimation ofcarboxyHb levels, because a pulse oximeter cannotdifferentiate between HbO2 and HbCO. The diagnosis ofcyanide poisoning is more difficult to make particularly asblood cyanide levels cannot be obtained in a timely fashionbut a very high lactate level is an important indicator.

Treatment of CO and hydrogen cyanide poisoninginvolves inhalation of 100% oxygen to displace bothmolecules from their protein binding sites. Hyperbaricoxygen at 2 atmospheres decreases the half-life ofcarboxyHb. It is recommended in unconscious patientsand in those with a carboxyHb level of >40%. Specifictreatment of hydrogen cyanide is sodium thiosulphate,which converts cyanide to the much less toxic thiocyanate.

Answer 378

c. 100% oxygen.e. Intravenous sodium thiosulphate.

The four major consequences of smoke inhalationConsequence Effect1. Thermal injury2. Hypoxic gas FiO2 may be as low as 11%

inhalation3. Exposure to direct Wheeze, bronchorrhoea

bronchopulmonarytoxins (e.g. SO2, NO,CCl4, chlorine)

4. Exposure to Severe hypoxia and acidosissystemic toxins (e.g. CO and hydrogen cyanide)

The three main goals in the management of atrialfibrillation include restoration to sinus rhythm (if possible),control of ventricular rate (if restoration of sinus rhythm isnot possible or desirable), and the prevention of systemicthromboembolism. Stagnation of blood in the atrialappendage promotes thrombus formation. Restoration tosinus rhythm from atrial fibrillation increases the probabilityof thromboembolism (dislodging of the thrombus) since

effective atrial contraction is restored. To minimize this riskit is recommended that the patient be anticoagulated for atleast three weeks prior to DC cardioversion. Although sinusrhythm is restored after successful DC cardioversion, theatria may remain stunned for a few weeks before theycontract properly again, therefore the patient should remainon warfarin for at least four weeks afterwards. NOTE: Patients with previous TIA or CVA, impaired leftventricular function and mitral valve disease should remainanticoagulated indefinitely irrespective of restoration tosinus rhythm. (See Answer 184.)

Answer 377

c. Warfarin for 3–4 weeks prior to cardioversion and 4 weeks afterwards.

A 58-year-old man presented with a three-month history oflower back pain. He had also noticed weakness in his legswhen climbing stairs, and had considerable difficulty inraising himself from a chair. More recently, he was excessivelythirsty and was waking frequently at night to pass urine. Hehad lost over 4 kg in weight since the onset of his illness.Before this he was well. He was married with two children.

He had been smoking over 20 cigarettes per day for morethan 30 years. He consumed 3 units of alcohol per day.

On examination, he was thin. The blood pressure was160/90 mmHg. He had wasting of his thigh muscles.Power in both the arms and legs was slightly diminished,but sensation and reflexes were preserved.

Investigations are shown.

Question 379

Page 380: Rapid review of clinical medicine for mrcp part 2

The patient was given 8 mg dexamethasone for threedays, and repeat 9.00 am and midnight serum cortisolwas over 1,000 nmol/l.

Clinical Cases 379

Hb 11 g/dlWCC 13 � 109/lPlatelets 480 � 109/lMCV 89 flBlood film NormalESR 44 mm/hSodium 140 mmol/lPotassium 3 mmol/lUrea 8 mmol/lBicarbonate 30 mmol/lGlucose 10 mmol/lCalcium 2.7 mmol/lPhosphate 1.5 mmol/lAlbumin 36 g/lTotal protein 60 g/l LFT NormalSerum cortisol at 9.00 am >1,000 nmol/lSerum cortisol at midnight >1,000 nmol/l

1. What is the cause of this patient’s lower limbweakness?a. Diabetic amyotrophy.b. Steroid-induced myopathy.c. Hypokalaemia.d. Eaton–Lambert syndrome.e. Metastatic spinal cord compression.

2. What is the most likely cause of the back pain?a. Bone metastases.b. Myeloma.c. Infiltration of the lower spinal nerves by carcinoma.d. Osteoporosis.

3. List two possible diagnoses.a. Cushing’s disease.b. Conn’s syndrome.c. Cortisol-secreting adenoma.d. Ectopic ACTH production from a bronchial

carcinoma.e. Pseudo Cushing’s syndrome secondary to alcohol

abuse.f. Cortisol secreting adenocarcinoma.g. Carcinoma of the pancreas.h. Multiple myeloma.i. Liddle’s syndrome.j. Osteoporosis.

4. Which two tests would be most useful in makingthe diagnosis?a. Serum ACTH.b. Chest X-ray.c. CT scan of the adrenals.d. Petrosal venous sampling.e. Lying and standing renin and aldosterone levels.f. Low-dose dexamethasone suppression test.g. High-dose dexamethasone suppression test.h. CT scan brain.i. Urinary ketosteroid estimation.j. Serum gamma GT concentration.

A 70-year-old female presented with sudden onset ofdyspnoea. There was no preceding history of a cough orchest trauma. She was a smoker. Four weeks previouslyshe had had a right hip replacement.

On examination she was cyanosed. The heart rate was140 beats/min and irregular. The blood pressure was

70 mmHg systolic. The JVP was raised. Both heartsounds were normal, and auscultation of the lungsrevealed decreased air entry at the left lung base. Thelower limbs appeared normal.

Investigations were as follows:

Question 380

Arterial blood gases (air):pH 7.21PaCO2 3.1 kPaPaO2 7.1 kPaHCO3 11 mmol/l

Chest X-ray Clear lung fieldsECG Sinus tachycardia; right ward

axis; ST segment depression in leads V2–V4

The patient was treated with 28% oxygen. What otherimmediate treatment is recommended?

a. Oral aspirin, oral clopidogrel, subcutaneous heparin, and intravenous tirofiban.

b. Intravenous alteplase 50 mg.c. Subcutaneous fractionated heparin.d. Intravenous noradrenaline.e. Intravenous unfractionated heparin.

Page 381: Rapid review of clinical medicine for mrcp part 2

380

The patient presented with lower back pain and clinicalsymptoms of a proximal lower limb myopathy. The latteris confirmed clinically. He is diabetic and hypertensive.Routine blood results demonstrate a low potassium andelevated bicarbonate. The differential diagnosis at thispoint is between Cushing’s disease (ACTH-secretingbasophil adenoma from the pituitary), cortisol-secretingadrenal adenoma or ectopic ACTH production. Thediagnosis is narrowed down to a cortisol-secretingadrenal adenoma and ectopic ACTH by the failure ofsuppression of cortisol following high-dosedexamethasone. (See Answer 173.)

The short history of the presenting complaint andhistory of smoking favour ectopic ACTH production bycarcinoma of the bronchus. These patients produce largeamounts of ACTH and present primarily with weaknessand myopathy associated with hypokalaemia, rather thana Cushingoid appearance.

The proximal myopathy is due to steroid excess, andthe accompanying muscle weakness is compounded bythe hypokalaemia. The hypokalaemia per se does notcause myopathy unless it is very long standing. Thediabetes would account for the polydipsia and polyuria,

but amyotrophy is unlikely to account for the proximalmyopathy in a patient with such a short history and theabsence of neuropathy. Eaton–Lambert syndrome maycomplicate oat cell bronchial carcinoma. It ischaracterized by proximal myopathy, fatiguability anddepressed reflexes or absent reflexes. It is due to adeficient release of acetylcholine from the nerveterminals. The diagnosis does not explain the normalreflexes or the diabetes and hypertension.

The patient has back pain and, in the context of anunderlying carcinoma, this may be suggestive of bonymetastases to the spine. Further support for this comes fromthe elevated serum calcium. Spinal cord compression,however, is not plausible in the absence of spasticity in thelower limbs, the absence of a sensory level, and the presenceof normal reflexes. In cord compression the reflexes arebrisk. Infiltration of the spinal nerves by carcinoma is alsounlikely because this would normally cause root pain andweakness of muscles supplied by the nerve affected, as wellas altered sensation in the affected dermatomes.

Osteoporosis is a recognized complication ofcorticosteroid excess, and fractured vertebrae are arecognized complication of osteoporosis; thus, it ispossible for either to be the cause of lower back pain inthis situation. The bone biochemistry in osteoporosis,however, is normal (with the exception of raised alkalinephosphatase in the presence of a fracture). In this case thecalcium and phosphate are elevated. Myeloma wouldexplain the raised ESR, raised calcium and the back pain;however, in the context of the abnormal dexamethasonesuppression test this diagnosis is unlikely.

Answer 379

1. b. Steroid-induced myopathy. 2. a. Bone metastases.3. d. Ectopic ACTH production from a bronchial

carcinoma.f. Cortisol secreting adenocarcinoma.

4. a. Serum ACTH.b. Chest X-ray.

The patient presents with acute dyspnoea associated withsevere circulatory collapse, hypoxia, hypocarbia andmetabolic acidosis. In addition her JVP is raised,indicating right heart strain. The ECG shows ST segmentdepression, which may be part of right heart strain ormyocardial ischaemia secondary to severe hypoxia.

Patients with massive pulmonary embolism judgedclinically by cardiac arrest, circulatory collapse, severehypoxia, acidosis or signs of right heart strain are treatedwith early thrombolysis. The usual treatment is a 50 mgbolus of alteplase followed by a heparin infusion. Thediagnosis of massive pulmonary embolism can beconfirmed by echocardiography or CT pulmonaryangiography; however, the seriousness of the circulatorycollapse may not allow time for investigation untilthrombolysis has been instituted.

Patients with intermediate or high probability of non-massive pulmonary embolus are treated with heparinuntil imaging. Low-molecular weight heparin ispreferable to unfractionated heparin, as it is equallyeffective and easier to use. Unfractionated heparin is onlyconsidered in situations where rapid reversal ofanticoagulation is required.

Once the diagnosis has been confirmed all patients arecommenced on warfarin and the heparin is stopped oncethe INR exceeds 2. Patients remain on warfarin for sixweeks if the pulmonary embolus was secondary to atemporary risk factor such as recent lower-limborthopaedic surgery. In patients with idiopathicpulmonary embolus warfarin is continued for threemonths. Patients with recurrent pulmonary emboli orpermanent risk factor for pulmonary emboli areanticoagulated indefinitely if the risks of pulmonaryembolism outweigh the dangers of haemorrhage.

Answer 380

b. Intravenous alteplase 50 mg.

Page 382: Rapid review of clinical medicine for mrcp part 2

Clinical Cases 381

A 21-year-old male presented with headaches,drowsiness, nausea, lethargy and general malaise. Hisappetite was reduced, but he had not lost any weight.The only other positive feature in the history was that oftroublesome back pain and cramps in the legs over thepast 18 months. The past medical history comprised along history of intermittent lower abdominal pains as achild which were associated with vomiting but hadsubsided by the age of 15 years. At the age of 17 he wasseen by a paediatrician for short stature and delayedpuberty, but was then lost to follow-up. He had workedas an assistant in a florist’s shop for eight years. He tookvery occasional analgesia for the back pain. He was single.He was a non-smoker, and did not consume alcohol.

On examination, he was 1.53 m tall and weighed 45 kg. Secondary sexual characteristics were present. Theheart rate was 98 beats/min, and regular, and bloodpressure was 210/110 mmHg. The jugular venouspulsation was just visible in the neck with the patientrecumbent at 45°. On examination of the precordium,the apex was not displaced, but was heaving in nature.The second heart sound was loud. Examination of therespiratory system and the abdomen was normal. Onexamination of the central nervous system, the patientwas slightly drowsy, and the Glasgow coma scale was13/15. The pupils were both equal in size and reacted tolight. The appearances on examination of a fundus isshown (381a).

Investigations are shown.

Hb 9 g/dlWCC 10 × 109/lPlatelets 180 × 109/lMCV 82 flSodium 136 mmol/lPotassium 6 mmol/lUrea 43 mmol/lCreatinine 500 μmol/lCalcium 2.0 mmol/lPhosphate 1.7 mmol/lAlkaline phosphatase 305 iu/lAlbumin 35 g/lChest X-ray Slight increase in the

cardiothoracic ratioUpper lobe venous

distensionX-ray of hand (381b)US kidneys Bilaterally shrunken kidneys

Question 381

1. List four abnormalities on the fundi.2. What is the cause of the hypertension?3. Explain the low calcium.4. Describe the abnormality on the hand X-ray, and

with what is it consistent?5. Explain the anaemia.6. What is the best explanation for the tachypnoea?7. What is the best management of the hypertension

in the short term?8. List five steps in the long-term management of this

patient.

338811aa 338811bb

Page 383: Rapid review of clinical medicine for mrcp part 2

382

Answer 381

The patient presents with hypertension, grossly abnormalrenal function, grade IV hypertensive retinopathy (TableC) and drowsiness. These features could be explained byprimary accelerated hypertension; however, the anaemia,bone biochemistry and X-ray appearance of the handindicate that the most likely cause of the hypertension ischronic renal failure. It is difficult to be certain about theaetiology of the renal impairment because the past historyis vague and the kidneys are shrunken. Renal biopsy inthese instances reveals extensive glomerular scarring and israrely helpful in the management of the patient. Thehistory of intermittent abdominal pain and vomiting andinvestigation for short stature suggest that the cause maybe chronic pyelonephritis. Other causes of chronic renalfailure and its complications are shown (Tables A and B).

The long- and short-term management of this patient isoutlined in 7 and 8 above. It is important to be aware thaterythropoietin itself may worsen hypertension; therefore,extra care is required in managing hypertension in anindividual taking erythropoietin. Good hypertensivecontrol will reduce the rate of decline in renal function;however, given his weight of just 45 kg, a serum creatinineof 500 �mol/l almost certainly represents end-stage renalfailure requiring dialysis. In some patients with chronicrenal failure and hypertension, reducing intravascularvolume with dialysis is all that is required to control bloodpressure.

Table A Causes of chronic renal failure

• Chronic glomerulonephritis• Chronic pyelonephritis• APCKD• Diabetes mellitus• Chronic hypertension• Renal artery stenosis• Renal papillary necrosis secondary to chronic

analgesia• Hyperuricaemic nephropathy• Chronic nephrolithiasis

Table B Complications of renal failure

• Anaemia secondary to erythropoietin deficiency• Osteomalacia/rickets secondary to deficiency of

active vitamin D metabolites• Hypertension (secondary to salt and water

retention) and its complications• Metabolic acidosis• Hyperkalaemia• Osteodystrophy secondary to

hyperparathyroidism resulting from chronichyperphosphataemia

• Fluid overload causing pulmonary and peripheraloedema

• Pruritus secondary to hyperphosphataemia• Skin pigmentation• Hyperlipidaemia• Premature coronary artery disease (combination

of hypertension and hyperlipidaemia)• Neuropathy

1. i. Arteriovenous nipping.ii. Flame-shaped haemorrhages.iii. Cotton wool spots or retinal infarcts.iv. Blurred disc or papilloedema.

2. Chronic renal failure (Table A).3. Failure of 1a-hydroxylation of vitamin D, causing

low levels of 1,25-dihydroxycholecalciferol, whichis an active vitamin D metabolite.

4. Subperiosteal bone resorption and severedemineralization of the third metacarpalsbilaterally. There is also an expansion in theproximal phalanx of the left third digit. Such acystic appearance is seen in severehyperparathyroid-related bone disease.

5. Deficiency of erythropoietin.6. Uraemic acidosis. Pulmonary oedema is less likely,

given the relatively clear chest and absence of interstitial shadowing on the chest X-ray (Table B).

7. Haemodialysis.8. i. Regular haemodialysis.

ii. Control hypertension.iii. Erythropoietin for anaemia.iv. a-1-calcidol for hypocalcaemia.v. Calcium carbonate to bind phosphate and prevent

osteodystrophy.

Table C Wegener’s classification of hypertensiveretinopathy

I Silver wiring of the arteriolesII Arteriovenous nipping

III Flame-shaped haemorrhages and cotton wool spots

IV Papilloedema

Page 384: Rapid review of clinical medicine for mrcp part 2

Clinical Cases 383

Question 382

A 42-year-old male was investigated for a three-yearhistory of intermittent abdominal pain. He had sufferedfour episodes of pyelonephritis in the past four yearswhich were treated by his GP. There was a long historyof migraines, for which he took mefenamic acid andparacetamol. Over the past few months he hadcomplained of increasing lethargy. There was no historyof diarrhoea, weight loss, urinary hesitancy, nocturia orpost-micturition dribble. Four months ago he was notedto have a blood pressure of 180/110 mmHg, which wasunder review. He was a florist by occupation. He was anon-smoker and consumed very occasional alcohol. Hisfather died suddenly from a subarachnoid haemorrhage atthe age of 40 years, but his mother was still alive andwell. He had three brothers aged 44, 45 and 46 yearsrespectively; the eldest brother had hypertension.

On examination, he appeared well at rest. There wasno clubbing or peripheral oedema. The blood pressurewas 180/105 mmHg. Examination of the abdomenrevealed tenderness in both loins. Deep palpation of theloins was difficult owing to tenderness. The percussionnote throughout the abdomen was resonant andauscultation demonstrated normal bowel sounds.Examination of the cardiovascular and respiratory systemwas normal. Fundoscopy demonstrated silver wiring andarteriovenous nipping.

Investigations are shown.

Hb 15 g/dlWCC 5 × 109/lPlatelets 240 × 109/lESR 15 mm/hSodium 128 mmol/lPotassium 3.8 mmol/lUrea 20 mmol/lCreatinine 400 μmol/lCalcium 2.1 mmol/lPhosphate 1.9 mmol/lAlbumin 39 g/lSerum renin ElevatedUrinalysis Protein ++

Blood ++24-hour urinary sodium

89 mmol/l

A 16-year-old East African female was admitted with atwo-week history of intermittent, sharp, right upperquadrant pain which was worse on inspiration. The painradiated to the umbilicus and right loin and was mostsevere on the night of admission. There were no othersymptoms. She had just finished menstruating. Her familyhad very recently immigrated to Britain from Kenya.

On examination, she had shallow respiration. Thetemperature was 37.5°C (99.5°F). There were tenderness

and guarding in the right hypochondrium. The liver waspalpable 6 cm below the costal margin, and there wastenderness in the 9th intercostal space in the mid axillaryline. The right lung base was dull to percussion withreduced air entry on auscultation.

Investigations are shown.

Hb 13 g/dlWCC 16 × 109/l (neutrophil leucocytosis)Platelets 590 × 109/lESR 70 mm/hU&E NormalChest X-ray Raised right hemidiaphragm

Clear lung fieldsUrinalysis Normal

What is the most likely diagnosis?a. Bilateral renal artery stenosis.b. Chronic pyelonephritis.c. Hypernephroma.d. Genetic polycystic kidney disease.e. Analgesic nephropathy.

1. What is the most likely diagnosis?2. List at least three investigations you would perform

to confirm the diagnosis.

Question 383

Page 385: Rapid review of clinical medicine for mrcp part 2

384

Answer 382

This middle-aged patient presents with abdominal painand advanced renal failure. All of the options given maycause abdominal pain and renal failure (Table). Theepisodes of pyelonephritis in a middle-aged male shouldraise suspicion of obstructive uropathy or an anatomicalabnormality in the renal tract. In the context of thepossibility of the latter, the family history of subarachnoidhaemorrhage and hypertension should arouse thesuspicion of APCKD. The normal Hb in the presence ofchronic renal failure should clinch the diagnosis becauseAPCKD is one of the few causes of chronic renal failurewhere the Hb is preserved. This is due to inappropriateerythropoietin secretion in some patients with thecondition. Hypernephroma may also cause polycythaemiafor the same reason but does not cause a salt-losingnephropathy as seen in this case.

APCKD is inherited as an autosomal dominant trait. Itis characterized by multiple thin-walled cysts in thekidney, which become progressively larger over the yearsand cause renal failure by compression of the nephrons.In 90% of patients, the abnormal gene is on the short armof chromosome 16 (PKD 1) which encodes atransmembrane protein termed polycystin. The exactfunction of it is unclear but it is thought to be involved inmediating cell-matrix interactions. Of the other 10% ofpatients, some have abnormalities linked to a locus on

chromosome 4 (PKD 2). Clinical symptoms compriseflank pain, haematuria, nocturia and recurrent urinarytract infections. Hypertension develops in 75% of cases.End-stage renal failure may occur at any age. Thepatients may be anaemic owing to haematuria anduraemia; however, polycythaemia secondary to increasederythropoietin is recognized. Malignant transformationwithin cysts is also recognized. Associations include berryaneurysms causing intracranial haemorrhage and mitralvalve prolapse. Management includes family screening offirst-degree relatives, early treatment of urinary tractinfections, and meticulous blood pressure control.

Abdominal pain and renal failure

• Henoch–Schönlein purpura• Haemolytic uraemic syndrome• Polyarteritis nodosa• APCKD• Nephrolithiasis• Analgesic nephropathy• Retroperitoneal fibrosis• Dissecting aortic aneurysm• Legionnaire’s disease• Acute pancreatitis• Ethylene glycol poisoning

The patient presents with hepatomegaly, a raised righthemidiaphragm, and localized tenderness in the 9thintercostal space. These features are consistent with ahepatic abscess. She has been in East Africa, which makesan amoebic abscess the most probable cause of her signsand symptoms. Some 50% of patients with amoebicabscess have no history of dysentery. Symptoms includeright hypochondrial pain, fever, sweating, intermittentfever and rigors. There may also be hepatic enlargement

and localized tenderness in the right hypochondrium orintercostal spaces. Enlargement of the liver abscess isusually upwards, and may cause compression of the righthemidiaphragm and atelectasis of the overlying lung.Jaundice and abnormality of LFT are rare. US or CT scanwill demonstrate the abscess or abscesses. Aspiration ofthe pus may demonstrate the organism. Sigmoidoscopyand biopsy for microscopy and stool microscopy have alow yield, but indirect haemagglutination tests have hightitres in the presence of a hepatic abscess. The lesionsmay be as large as 10 cm. Rupture of the abscess into thelung and development of a hepatobronchial fistula is arecognized complication (anchovy-sauce sputum).Peritonitis, pericarditis and cutaneous sinus formation arerare complications.

Treatment is with metronidazole or tinidazole.Diloxanide is also given to eliminate any on-goinggastrointestinal infection.

d. Genetic polycystic kidney disease.

1. Hepatic abscess secondary to Entamoeba histolytica(amoebic liver abscess).

2. i. Ultrasound of the liver.ii. Microscopy from liver aspirates.iii. Indirect haemagglutination tests (over 95% with

liver abscess are positive, but the test cannot distinguish between past and current infection).

Answer 383

Page 386: Rapid review of clinical medicine for mrcp part 2

Clinical Cases 385

A 14-year-old male was admitted to hospital with a 12-hour history of colicky right-sided abdominal pain andvomiting. He had a past history of recurrent chestinfections that had been treated by his GP. He was alsothought to be asthmatic, and took a regularbronchodilating inhaler. He had intermittent loose, bulkystool but had never experienced abdominal pain. He had a

younger brother and sister who were well. His father wasa shop assistant and his mother was a part time GP’sreceptionist.

On examination, he did not appear acutely unwell. Hehad mild clubbing. The abdomen was distended. Therewas generalized tenderness on palpation, as well as a firmmass in the right iliac fossa. Bowel sounds were tinkling.The rectum was empty. Auscultation of the lung fieldsrevealed coarse crackles throughout the lung fields.

Growth charts for height and weight are shown(384a, b). Investigations are shown.

Hb 10 g/dlWCC 14 × 109/lPlatelets 480 × 109/lMCV 80 flSodium 135 mmol/lPotassium 3.2 mmol/lUrea 9 mmol/lTotal protein 70 g/lAlbumin 34 g/lCalcium 2.2 mmol/lPhosphate 0.7 mmol/lChest X-ray (384c)Abdominal X-ray (384d)

1. What is the abnormality on the growth chart?2. Comment on the chest X-ray.3. Comment on the abdominal X-ray.4. What is the cause of his presentation problem?5. What is the management of his current problem?6. What is the full diagnosis?7. What test would you request to confirm the

diagnosis?

Question 384

40

30

20

10

6 7 8 9 10 11 12 13 14 15 16Age, years

97 90 75 50 25 10 3

Wei

ght

(kg)

6 7 8 9 10 11 12 13 14 15 16Age, years

97 90 75 50 25 10 3

Hei

ght

(cm

)

5050

Early (+2SD) maturersLate (-2SD) maturers

140

130

120

110

100

338844aa 338844bb

338844cc 338844dd

Page 387: Rapid review of clinical medicine for mrcp part 2

386

Answer 384

Cystic fibrosis is an autosomal recessive condition with aprevalence of 1/2,500 which is characterized by viscidrespiratory and gastrointestinal secretions. The

abnormality is in the gene encoding a chloride ionchannel in the nasal epithelium, lungs, salivary glands,pancreas, intestine and bile ducts. This chloride ion

Table A The clinical manifestations of cystic fibrosis

RespiratoryNasal polypsOtitis mediaAsthmaBronchiolitis Recurrent pneumonia Staphylococcus aureus, Haemophilus influenzae,

Pseudomonas aeuroginosaBronchiectasis Lung abscess/empyema

HaemoptysisAspergillus infectionLung fibrosisPneumothoraxCor pulmonale

GastrointestinalMeconium ileus (birth)Rectal prolapse (neonate)Steatorrhoea (pancreatic insufficiency)Meconium ileus equivalent (infancy onwards)Malabsorption (fat-soluble vitamins due to pancreatic insufficiency usually, or biliary obstruction)Gall stonesSecondary biliary cirrhosis

MiscellaneousNon-erosive arthropathyInfertility in males

1. This patient has fallen off the third centile for heightat the age of 10 years and for weight at the age of14 years.

2. The chest X-ray is grossly abnormal. There iswidespread fibrotic change, patchy consolidation,and bronchiectasis. In the context of this history thefindings are consistent with cystic fibrosis.

3. The erect abdominal X-ray reveals dilated loops ofsmall bowel and multiple fluid levels consistent withsmall bowel obstruction.

4. Meconium ileus equivalent. This is bowelobstruction resulting from a combination ofsteatorrhoea and viscid intestinal secretions causingfaecal impaction in the ascending colon or theileocaecal junction.

5. The management steps are as follows:• Rest the bowel; the patient should be completely

restricted from oral solids and fluids and shouldhave intravenous fluid supplements. A nasogastrictube should be placed in situ to prevent gastricdilatation and vomiting.

• Diagnostic and therapeutic manoeuvres: a

Gastrografin enema – which is both diagnostic andtherapeutic – should be performed. Patients withany form of obstruction should not have bariumstudies.

• Other therapeutic steps: acetylcysteine givenintravenously or through the nasogastric tube hasbeen shown to be very useful in resolving bowelobstruction.

• In the long term, the plan is to reduce steatorrhoea,which can be done by increasing pancreaticsupplements. There have been recent reports offibrous colonopathy in association with a high doseof Pancrease pancreatic supplements. The risk of thecondition with Creon pancreatic supplements is verysmall.

6. The full diagnosis is cystic fibrosis. The diagnosisshould be suspected in any young child whopresents with chronic gastrointestinal andrespiratory complaints.

7. The diagnosis of cystic fibrosis can be confirmed bydemonstrating a sweat sodium concentrationexceeding 60 mmol/l.

Page 388: Rapid review of clinical medicine for mrcp part 2

Clinical Cases 387

channel has been termed the CFTR. The gene is locatedon chromosome 7. Over 200 different mutations havebeen described within the gene, but by far the most

common is a point mutation which results in the deletionof the amino acid phenylalanine on position 508. Thismutation accounts for 70–80% of all cases. The clinicalmanifestations are listed (Table A). The differentialdiagnoses are other causes of recurrent respiratoryinfections and diarrhoea in young children, which includehypogammaglobulinaemia and intestinallymphangiectasia. The genetic heterogeneity of thecondition means that the diagnosis continues to rely onthe demonstration of a sweat sodium >60 mmol/l.However, there are conditions associated with a false-positive sweat sodium test (Table B). It is also worthnoting that a small proportion of patients with cysticfibrosis do not have a positive sweat sodium test.

Table B Causes of a false-positive sweat sodium test

• Hypothyroidism• Familial hypoparathyroidism• Mucopolysaccharidoses• α-1 antitrypsin deficiency • Addison’s disease

Question 385

A 6-year-old female was seen in the Accident andEmergency Department with a two-year history of apainful swollen right knee which developed after she fellat school. Two years previously she had a similar episodeaffecting the left knee, following which she had persistentswelling and discomfort for several weeks. At that stageshe had an FBC which was normal and an X-ray of theleft knee which revealed soft tissue swelling. In theinterim she had recurrent episodes of painful swelling ofthe left knee for no obvious reason, which would subsideafter a few days. At the age of two years she sustained alarge haematoma on her forehead after a fall at home.The only other past history of note was a recent reviewby the ophthalmologist for a left convergent squint.

She had achieved her milestones normally, but hadproblems with reading which were attributed to hersquint. She was one of three siblings. All three wereadopted and had come from different families. Herparents appeared very caring. The father was abusinessman who spent many days away from home andthe mother was a housewife.

On examination, she was below the third centile forheight and was on the 50th centile for weight. There wasa painful swelling over the left knee consistent with asynovial effusion. The right knee was not swollen, butmovement of the knee demonstrated crepitus. There wasan obvious convergent squint. The neck was short andthe hands, particularly the fourth and fifth digits,appeared small.

Investigations are shown.

Hb 11 g/dlWCC 6 × 109/lPlatelets 112 × 109/lBlood film NormalPT 12 sAPTT 94 s BT 8 min Sodium 136 mmol/lPotassium 3.8 mmol/lUrea 6 mmol/lCreatinine 80 μmol/lCalcium 2.3 mmol/lPhosphate 1.0 mmol/lAlbumin 40 g/lX-ray of left knee Soft-tissue swelling

No bony deformityX-ray of right knee Chondrocalcinosis

1. List two different diagnoses.2. What is the mechanism for the swollen knee?3. Give two possible underlying causes for the swollen

knee.4. What two investigations would you request to

confirm your answer to 1?

Page 389: Rapid review of clinical medicine for mrcp part 2

388

Answer 385

The patient has several morphological features of Turner’ssyndrome (Table C, Answer 397). All of thesemorphological features, particularly short metacarpals, mayalso occur in pseudohypoparathyroidism; however, thenormal bone biochemistry is against the diagnosis. Shortmetacarpals may occur in other conditions (Table A).

The patient also has an important clotting abnormality,suggested by the elevated APTT. The swollen knee is mostprobably due to haemorrhage into the joint. The right kneehas features of chondrocalcinosis possibly due to recurrenthaemarthroses. The PT is normal, suggesting that thedefect is in the intrinsic pathway of the clotting cascade.Causes of a raised APTT include deficiency in any one of

the clotting factors in the intrinsic pathway, DIC, orheparin therapy. The normal PT and fibrinogen level makeDIC very unlikely. The normal BT excludes vonWillebrand’s disease (Table B). The most likely clottingabnormality here is either Factor VIII (haemophilia A) orFactor IX (haemophilia B; Christmas disease) deficiency.Both are X-linked recessive; thus normal females with two Xchromosomes will not exhibit the disease phenotype, even ifthey have inherited the abnormal gene. However, patientswith Turner’s syndrome have just one X chromosome andeffectively have the same chance of being affected by an X-linked recessive genetic condition as males.

Table A Causes of short metacarpals

• Turner’s syndrome• Noonan’s syndrome• Pseudohypoparathyroidism• Sickle cell dactylitis• Juvenile chronic arthritis• Recurrent hand trauma

Question 386

A 48-year-old clerical worker in a large insurancecompany was admitted with malaise, increasing shortnessof breath and confusion. One week before admission hehad a dry cough, chills, headaches and generalized achesthroughout his body. He saw his GP, who prescribedamoxycillin 500 mg three times daily. Two days later, thepatient developed generalized abdominal pain and loosemotion. Over the next 24 hours, he had becomeincreasingly confused, and started to hallucinate. His wifebecame concerned, and brought him to the Accident andEmergency Department. He had been in Spain six weeksbefore his illness. He smoked 20 cigarettes a day, andconsumed 4 units of alcohol a night.

On examination, he appeared unwell and dehydrated.His temperature was 38.7°C (101.7°F). The heart ratewas 110 beats/min, and regular, and the blood pressurewas 120/70 mmHg. On examination of the respiratorysystem he was noted to have a respiratory rate of 35/min.On auscultation, there were coarse crackles at the rightlung base. Examination of the precordium was normal.Abdominal examination revealed generalized abdominaltenderness, but no guarding or rigidity. Central nervoussystem examination revealed a Glasgow coma score of9/15. The patient was disorientated in time and space,but there was no focal neurological abnormality.

Investigations are shown.

1. i. Turner’s syndrome.ii. Haemophilia A or haemophilia B.

2. Haemarthrosis; bleeding into the knee joint.3. i. Haemophilia A.

ii. Haemophilia B.4. i. Buccal smear for karyotyping to confirm Turner’s

syndrome.ii. Factor VIII assay or Factor IX assay to confirm

haemophilia A or B respectively.

Table B Differentiation between haemophilia A, B and von Willebrand’s disease

HA HB Von Willebrand’s

PT Normal Normal NormalAPTT ↑ ↑ ↑BT Normal Normal ↑Factor VIIIc ↓↓ Normal ↓Factor VIIIVWB Normal Normal ↓Ristocetin platelet aggregation Normal Normal ↓

Page 390: Rapid review of clinical medicine for mrcp part 2

Clinical Cases 389

Hb 13.5 g/dlWCC 12.1 × 109/l (neutrophils

10.2 × 109/l; lymphocytes 1.5 × 109/l)

Platelets 140 × 109/lSodium 128 mmol/lPotassium 4 mmol/lUrea 10 mmol/lCreatinine 160 μmol/lBilirubin 22 μmol/lALT 75 iu/lAlkaline phosphatase 102 iu/lGamma GT 102 iu/lAlbumin 29 g/l

Arterial blood gases:pH 7.37PaO2 7.3 kPaPaCO2 4 kPaBicarbonate 18 mmol/lOxygen saturation 83%

Chest X-ray Right lower lobe consolidation, and a small associated pleural effusion

ECG Sinus tachycardiaUrinalysis Trace of blood

Protein ++

Question 387

A 34-year-old West African female presented with athree-month history of increasing ankle oedema andabdominal distension. There was no history of cough,exertional dyspnoea or orthopnoea. She was married witha five-year-old daughter. She was a non-smoker, anddrank alcohol very occasionally. She had worked as a fish-farm administrator in Nigeria before moving to Englandthree years ago.

On examination, there was no evidence of pallor,jaundice or lymphadenopathy. She had considerable,pitting, lower-limb oedema. The heart rate was 100beats/min, and irregularly irregular. The blood pressurewas 95/75 mmHg. The JVP was elevated to the level ofthe ear lobes. On examination of the precordium theapex beat was not palpable. The heart sounds were soft.There were no cardiac murmurs. Auscultation of the lungfields was normal. Abdominal examination revealed firmhepatomegaly palpable 6 cm below the costal margin andshifting dullness. The spleen was not palpable.Neurological examination and examination of the skinwere normal.

Investigations are shown.

Hb 12 g/dlWCC 7 × 109/l

(normal differential)Platelets 300 × 109/lUrinalysis Protein +Chest X-ray Normal cardiac size and clear

lung fieldsECG Low QRS voltage and non-

specific lateral T-wave changes

1. What is the most probable diagnosis?2. List three possible causes for the low sodium.3. List three further investigations you would perform

to make a rapid diagnosis.4. What antibiotic therapy would you institute while

awaiting the definitive diagnosis?

5. List three therapeutic interventions you wouldinitiate to manage this condition assuming adefinitive diagnosis has already been made.

1. What investigations would be most useful inmaking the diagnosis?

2. What is the most probable diagnosis?

Page 391: Rapid review of clinical medicine for mrcp part 2

390

Answer 386

This previously fit middle-aged man presents withrespiratory problems, abdominal symptoms and confusion.Biochemical tests reveal a low sodium and evidence ofrenal dysfunction. The full blood count reveals a mildlyraised WCC with a relative lymphopenia. The mostprobable diagnosis which ties all of this together isLegionnaire’s disease. The condition is caused by theorganism Legionella pneumophilia. Epidemic outbreaks canoccur among previously fit individuals staying in hotels,institutions or hospitals where the cooling systems havebeen contaminated with the organism. It can also occur insporadic cases and in the immunocompromised where thesource of infection is unknown. It is spread via the aerosolroute. Incubation period is between 2 and 10 days. Malesare affected more than females.

Legionnaire’s disease is preceded by a prodromal illnesssimilar to a viral infection with headaches, myalgia,anorexia and pyrexia. Approximately 50% have abdominalsymptoms (nausea, vomiting and diarrhoea). Neurologicalsymptoms such as confusion, hallucinations, peripheralneuropathy, myelitis and, rarely, a cerebellar syndrome canoccur. Patients may develop renal failure, usually secondaryto an acute tubulointerstitial nephritis. Liver involvement isusually mild and characterized by abnormal LFT.

Hyponatraemia is common and is usually due toinappropriate ADH secretion; however, loss of sodiumfrom the kidneys (acute tubulointerstitial nephritis) andgastrointestinal tract (diarrhoea and vomiting) may alsocontribute. Other biochemical features includehypocalcaemia, hypophosphataemia, and raised creatininekinase (myositis). On the blood film there is a normal ormildly raised WCC with a moderate neutrophilia andrelative lymphopenia. A mild coagulopathy can occur.Chest X-ray usually shows a unilateral lobar and thenmultilobar shadowing, with or without an associatedpleural effusion.

A rapid diagnosis can be made using the testsmentioned above. Urinary Legionella antigen has a yield of90% within the first week of the illness. Serology relies onmeasuring antibody titres in acute and convalescent serumsamples separated by 10–14 days; therefore a reader wouldnot score marks if asked specifically for tests to make arapid diagnosis. Similarly, culturing this fastidious organism(using buffered charcoal yeast media) may take up to threeweeks and has a low yield, and therefore would not scoremarks.

Treatment involves high-concentration oxygen andantibiotic therapy with intravenous erythromycin orazithromycin plus oral rifampicin. Before the diagnosis ofLegionnaire’s disease is confirmed, the patient should alsoreceive a cephalosporin to cover the possibility ofpneumonia due to Streptococcus pneumoniae, which maypresent similarly. Rehydration would have to be performedwith CVP monitoring as the patient has evidence of renalinvolvement, and the plasma electrolytes should bemonitored daily.

Conditions causing lung and brain involvementsimultaneously are common in postgraduate examinationsin which Legionnaire’s disease is commonly tested. Othercauses of simultaneous lung and cerebral involvement aretabulated.

1. Legionnaire’s disease.2. i. Syndrome of inappropriate ADH secretion.

ii. Diarrhoea and vomiting.iii. Salt-losing nephritis/acute tubulointerstitial

nephritis.3. i. Urine for Legionella antigen.

ii. Direct immunofluorescent antibody test on sputum, bronchial washings, pleural effusion or blood.

iii. IgM RMAT for Legionella.4. Intravenous cefotaxime and oral erythromycin.5. i. Oxygen.

ii. Intravenous erythromycin or azithromycin and oral rifampicin.

iii. Intravenous rehydration with CVP monitoring.

Conditions simultaneously involving the lowerrespiratory tract and central nervous system

InfectionsHIVTBLegionnaire’s diseaseMycoplasma pneumoniaStreptococcal/staphylococcal pneumoniaMalariaCysticercosis

MalignancyPrimary or secondary bronchial carcinoma

Granulomatous diseaseSarcoidosisHistiocytosis X

VasculitidesPolyarteritis nodosaWegener’s granulomatosisSLE

Neuroectodermal syndromesNeurofibromatosisTuberose sclerosis

Sickle cell syndromes

Page 392: Rapid review of clinical medicine for mrcp part 2

Clinical Cases 391

Answer 387

The patient has signs of severe right-heart failure, but thechest X-ray reveals a normal-sized heart. The differentialdiagnosis of severe heart failure in the absence ofsignificant cardiac enlargement is between pericardialconstriction and a restrictive cardiomyopathy.

If we consider the possibility of restrictivecardiomyopathy (Table A), then there is little informationin the history to suggest systemic sclerosis,haemochromatosis, carcinoid syndrome or anthracyclinetherapy. We are left with the possibility of malignancy,amyloidosis, EMF or idiopathic restrictivecardiomyopathy; however, the latter is extremely rare andsome cardiologists are unsure if it even exists. There is noreal evidence of malignancy. Cardiac amyloidosis isusually associated with myeloma, of which there is no real

evidence in this case. Moreover, it is more common inthe 6th and 7th decades and more common in males. Itis often associated with macroglossia and neuropathy,which appear to be absent here. EMF has two variants:the tropical and temperate forms (Table B). The findingsin favour of the tropical form of EMF is its insidiouspresentation and normal eosinophil count. However,tropical EMF is confined to young malnourished,poverty-stricken individuals with a high parasite load, andit is highly unlikely that any of these factors would applyto a fish-farm administrator.

The most probable diagnosis is pericardialconstriction. This is usually insidious and presents withsigns of severe right-heart failure, but little in the way ofleft-heart failure. Contrary to popular belief, most casesof pericardial constriction are idiopathic. Other causes ofpericardial constriction are listed in Table C.

In a tropical country, tuberculous pericarditis wouldbe a possible aetiological candidate; however, there iscomplete absence of radiological evidence of previouspulmonary TB.

Continued overpage

Table A Causes of restrictive cardiomyopathy

• Idiopathic (extremely rare)• EMF: temperate

tropical• Systemic sclerosis• Amyloidosis• Haemochromatosis• Carcinoid syndrome• Malignancy• Anthracycline

Table B Differences between temperate and tropical EMF

Temperate TropicalPresentation Acute InsidiousFever Common UnusualMulti-system involvement Common UnusualPrevious ill health or malnourishment Uncommon Very commonEosinophil count High Normal

Table C Causes of pericardial constriction

Idiopathic Bacterial pericarditisTB Radiotherapy

1. i. Echocardiography.ii. Cardiac MRI.iii. Left and right cardiac catheter.

2. i. Pericardial constriction.ii. Restrictive cardiomyopathy.

Page 393: Rapid review of clinical medicine for mrcp part 2

392

A 57-year-old male was admitted with a seven-monthhistory of increasing breathlessness and swollen ankles.Just before admission he had developed paroxysmal

nocturnal dyspnoea. He had seen his GP, who hadprescribed 80 mg of furosemide, with very slight effect.He had no past medical history of note, and there was no

Table D Echocardiographic features of pericardial constriction and restrictive cardiomyopathy

Pericardial constriction Restrictive cardiomyopathyLeft and right atria Enlarged/normal EnlargedLeft and right ventricles Normal Usually normal/enlargedEndocardium Normal BrightMyocardial texture Normal Normal/brightValves Normal Affected (mitral and tricuspid

regurgitation)Doppler Low E/A inspiration Large E*, small A**

High E/A expirationPericardium May be thickened Normal

* Early or passive left ventricular filling** Late or active ventricular filling

Question 388

A 23-year-old homosexual requested an HIV test after hispartner had been diagnosed as having the condition. Apartfrom a febrile illness eight weeks previously, he had beenrelatively well. Physical examination was entirely normal.

Investigations performed in clinic are shown.

Hb 13 g/dlWCC 6 × 109/lPlatelets 310 × 109/lHIV serology IgG HIV antibody-positiveCD4 count 510 × 106/l

(NR 500–1,300 × 106/l)Viral RNA 20,000 copies/mlChest X-ray Normal

1. Does this patient require treatment?2. Which medication(s) should be considered?

Question 389

Pericardial constriction Restrictive cardiomyopathy

LV

LV

RV

RV

Echocardiography is useful in differentiatingpericardial constriction from a restrictive cardiomyopathy(Table D). Restrictive cardiomyopathy has classicechocardiographic features. In pericardial constriction,the pericardium may seem to be thickened. Cardiac MRImay be more useful than echocardiography indemonstrating pericardial thickness.

An unequivocal method of differentiating the twoconditions is by performing a left- and right-heart cardiaccatheterization. Pericardial constriction encompasses thewhole heart, therefore the right and left end-diastolicpressures are equal, as are the atrial pressures. In contrast,restrictive cardiomyopathy is patchy; thus LVEDP isslightly higher than right ventricular end-diastolic pressure(387). In addition, there is evidence of mitral and/ortricuspid regurgitation, which is generally absent inpericardial constriction.

338877

Page 394: Rapid review of clinical medicine for mrcp part 2

Clinical Cases 393

Hb 11 g/dlWCC 6 × 109/lPlatelets 100 × 109/lMCV 88 flESR 70 mm/hSodium 135 mmol/lPotassium 3.9 mmol/lUrea 15 mmol/lCreatinine 110 μmol/lBilirubin 33 μmol/lAST 64 iu/lAlkaline phosphatase 200 iu/lAlbumin 18 g/lECG (389a)Chest X-ray (389b)Renal US Renal size enlarged slightly

bilaterally, but no ureteric obstruction

Echocardiogram (389c)Renal biopsy (389d)

family history of cardiovascular disease. He was a non-smoker and consumed about 7 units of alcohol per week.He had been a patron for a large charity organization forseveral years.

On examination, he appeared pale and was breathlesson very slight exertion. The heart rate was 100beats/min, and regular; blood pressure was 90/60mmHg. The JVP was raised 6 cm above the sternal angleand revealed prominent a- and c–v waves. The apex wasnot palpable, but on auscultation of the precordium therewas a loud third heart sound. On examination of thelungs there was reduced air entry bilaterally at both lungbases. Abdominal examination revealed hepatomegalypalpable 7 cm below the costal margin, and some ascites.

Investigations are shown.

1. Comment on the chest X-ray.2. Comment on the ECG.3. What is the renal diagnosis?4. What is the cause of cardiac failure?

5. What is the overall diagnosis?6. List two important investigations that you would

perform.

338899aa 338899bb

338899cc

338899dd

Page 395: Rapid review of clinical medicine for mrcp part 2

394

Answer 388

Patients infected with HIV may present following theseroconversion syndrome, with generalizedlymphadenopathy or with AIDS-defining opportunisticillnesses (Table). The management of HIV infection isdependent on the CD4 count and the viral RNA load.Recommendations for initiating therapy in patientsinfected with HIV include all patients with a CD4 countof <500 � 106/l, as well as asymptomatic patients whohave a CD4 count >500 � 106/l but evidence of activeviral RNA (>10,000 viral copies per ml). Some expertswill initiate treatment in patients with a CD4 count >500� 106/l in the presence of any detectable level of virus.Patients with AIDS-defining illnesses usually have a CD4<200 � 106/l. Treatment is in the form of triple therapy,comprising combinations of nucleoside analogues that

inhibit the viral reverse transcriptase enzyme and aprotease inhibitor that inhibits viral protein synthesis.

Examples of nucleoside analogue combinationsinclude AZT and didanosine (AZT/ddI), AZT andzalcitabine (AZT/ddC) and AZT and lamivudine(AZT/3TC). Commonly used protease inhibitorsinclude ritonavir, indinavir and saquinavir.

Answer 389

The combination of nephrotic syndrome and cardiacfailure should always raise the suspicion of primaryamyloidosis or AL amyloid. Primary amyloid affectsapproximately 10–15% of all patients with myeloma. Inprimary amyloidosis, the amyloid is made up ofimmunoglobulin light chains and glycoprotein. Clinicalfeatures include restrictive cardiomyopathy, macroglossia,peripheral neuropathy, nephrotic syndrome, purpura, andisolated clotting Factor X deficiency. Cardiac amyloid has apoor prognosis as it results in a stiff ventricle with poorsystolic and diastolic function. It responds poorly to

diuretics. Arrhythmias and conduction tissue disease arecommon. The ECG commonly reveals small complexes,pathological Q-waves and non-specific T-wave changes.Echocardiography demonstrates thickened left and rightventricular walls, which have a ground-glass appearance.The ventricular cavities are small, but the atria are enlarged(as in this case). The interatrial septum is often bright. Thedefinitive diagnosis of amyloidosis rests on tissue diagnosis.Rectal or gingival biopsy is useful; however, the former hasa lower yield. Serum amyloid protein scans are useful inidentifying tissues infiltrated with amyloid; however, animportant exception is cardiac infiltration, which is bestidentified with cardiac biopsy.

The other important type of amyloidosis for thecandidate doing higher specialist examinations in internalmedicine includes systemic amyloidosis. In this case, theprotein comprising amyloid is protein AA. This occurs inchronic inflammatory conditions or infections. Examplesinclude rheumatoid arthritis, bronchiectasis,inflammatory bowel disease and familial Mediterraneanfever. This type of amyloid has a predilection for thekidneys, causing nephrotic syndrome, haematuria andrenal failure. Hepatosplenomegaly is present in over 20%of cases.

1. The patient is HIV-positive. His CD4 count iswithin normal limits, but he has a very heavy viralload, which is generally a poor prognostic markerof disease progression and an indicator fortreatment.

2. A combination of two nucleoside analogues and aprotease inhibitor.

AIDS-defining conditions

Pneumocystis carinii pneumonia• TB (Mycobacterium avium intracellulare)• Cryptococcus neoformans• Cryptosporidium

Isospora belli• Oesophageal or disseminated Candida (but not

oral)• Herpes infection• CMV retinitis, colitis, oesophagitis and encephalitis• Oral leukoplakia• Toxoplasmosis• Kaposi’s sarcoma

1. i. Bilateral pleural effusions, the right effusion being larger than the left.

ii. Prominent pulmonary arteries.2. Small voltage complexes with T-wave inversion in

the lateral leads.3. Nephrotic syndrome due to renal amyloidosis.4. Restrictive cardiomyopathy secondary to cardiac

amyloid.5. Primary amyloidosis.6. i. Serum immunoglobulins/

immunoelectrophoresis.ii. Urinalysis for Bence-Jones protein estimation.

Page 396: Rapid review of clinical medicine for mrcp part 2

Clinical Cases 395

Question 390

A 28-year-old female presented with a four-week historyof progressive difficulty with walking, clumsiness of herhands, and slurring of the speech. Before this she hadbeen relatively well, with the exception of occasionalepisodes of central chest pain and breathlessness whenshe was stressed, or on rushing suddenly. In the past fourweeks she had fallen twice because she was ‘not incontrol of her lower limbs’. Her hands were clumsy andshe was having difficulty washing dishes or holding cups.There was no history of headaches or any family historyof neurological disease. She was a non-smoker andconsumed 1–2 glasses of wine per week.

On examination, she appeared well at rest. Shemeasured 1.81 m. She had a high-arched palate,kyphoscoliosis and mild bilateral pes cavus. Her speechwas mildly dysarthric. On examination of her upper limbsthe power was normal. The tone was slightly increased inboth forearms. There was clinical evidence of anintention tremor, past pointing, finger nose ataxia anddysdiadochokinesia. The reflexes were brisk bilaterally.On examination of the lower limbs the power was slightly

reduced bilaterally. The tone was increased and heel–shinataxia was present. The knee reflexes were brisk, but theankle jerks were absent. The plantar response wasupgoing bilaterally.

On examination of the cardiovascular system the heartrate was 90 beats/min, and regular; blood pressure was100/70 mmHg. The apex was not displaced, but washeaving in nature. On auscultation there was a harshejection systolic murmur at the left lower sternal edge,which was louder on standing suddenly and softer onsquatting. The remainder of the physical examination wasnormal.

Question 391

A 44-year-old Nigerian male was investigated for rightupper quadrant pain, abdominal distension and ankleswelling:

A lateral view chest X-ray is shown (391a).

Chamber Pressure Oxygen (mmHg) saturation (%)

Right atrium 19 68Right ventricle 48/20 68Pulmonary artery 40/20 68PCWP 18Left ventricle 90/20 96Aorta 85/65 96

1. What is the diagnosis?2. What is the cause of the cardiac murmur?3. List two other associations of the condition.

1. What are the abnormalities in the cardiac catheterdata?

2. What is the abnormality on the chest X-ray?3. What is the diagnosis?

339911aa

Page 397: Rapid review of clinical medicine for mrcp part 2

396

Answer 390

The patient has pes cavus and signs of cerebellar andpyramidal tract disease. In addition there is a history ofchest pains and breathlessness, suggesting cardiovasculardisease, and a cardiac murmur that is compatible withdynamic left ventricular outflow tract obstructioncharacteristic of hypertrophic obstructive cardiomyopathy.The complete diagnosis is Friedreich’s ataxia.

Friedreich’s ataxia is an autosomal recessive conditiondue to a mutation in a gene encoding the protein frataxinon chromosome 9. The mutation comprises unstableexpansions of the GAA repeat of variable size(120–1,700) in the first intron. The exact function offrataxin is unknown, but early reports suggest that it is amitochondrial iron transport protein. The prevalence is1/50,000. It is the most common example of thehereditary ataxias. There is degeneration of the dorsalspinal column, the spinocerebellar tracts, the pyramidaltracts, and the dorsal root ganglion cells. The syndrome isdominated by progressive ataxia. The usual age of onset isthe second decade, but the disease may not appear until aslate as the fourth decade. The lower limbs are generally

affected before the upper limbs. Difficulty in walking andstumbling regularly are early signs. Cerebellar signs mayproceed rapidly and include a coarse intention tremor,ataxia affecting the limbs and the trunk, nystagmus (25%)and dysarthria that may become severe. Involvement ofthe pyramidal tracts causes weakness of the lower limbs.The tone may be only slightly increased because of theaccompanying disturbance affecting the afferent fibresfrom the muscle spindles. The plantar response isextensor. Dorsal column involvement causes loss of jointposition, vibration and two-point discrimination sense.There is a peripheral neuropathy associated with musclewasting, and depressed or absent reflexes. Optic atrophy ispresent in approximately 30% of patients. Associatedskeletal deformities include kyphoscoliosis, pes cavus, pesequinovarus and high arched palate. Kyphoscoliosis andhigh arched palates are also present in Marfan’s syndromeand homocystinuria, but neither of these conditions isassociated with spinocerebellar degeneration.

The condition is associated with HCM in between 50and 70% of patients. The frequency of HCM increaseswith the size of the GAA expansion in the first intron. TheECG is abnormal in over 70% of patients anddemonstrates widespread T-wave inversion. Diabetesmellitus is present in approximately 10%. A very smallproportion of patients may also have sensorineuronaldeafness. Death usually occurs in the fourth decade, and isdue to cardiac failure

Answer 391

(See Interpretation of Cardiac Catheter Data, page 418.)

1. Friedreich’s ataxia.2. Hypertrophic cardiomyopathy (hypertrophic

obstructive cardiomyopathy).3. i. Diabetes mellitus.

ii. Optic atrophy.

1. i. Elevated and equal right and left atrial pressures.ii. Elevated and equal left and right ventricular end-

diastolic pressures.2. Calcification of the pericardium (391b, arrowed).3. The findings are consistent with the diagnosis of

pericardial constriction.

339911bb

Page 398: Rapid review of clinical medicine for mrcp part 2

Clinical Cases 397

Question 393

A 59-year-old female presented to the Accident andEmergency Department with sudden onset of retrosternalpain on bending down to get into her car. On examination,she was jaundiced. Her blood pressure was 140/80mmHg. She was extremely tender over the mid-sternal area,particularly when she attempted deep inspiration. The heartsounds were normal and her chest was entirely clear.Abdominal examination revealed hepatomegaly, which waspalpable 4 cm below the costal margin, and a palpable tip ofthe spleen. Only three days earlier she had seen her GP witha six-week history of malaise and was noted to havejaundice, which was under preliminary investigation. Therewas no history of abdominal pain or weight loss. She alsocomplained of increasing weakness, with particular difficultyin rising from a sitting position. She had never had anyprevious hospital admissions, but had seen the GP fortroublesome pruritus over the past four years for which shetook chlorpheniramine. She was noted to have a serumcholesterol level of 10.5 mmol/l during a routine healthcheck one year ago, for which she had been takingsimvastatin. She was widowed two years ago and now livedalone. There was no recent history of travel abroad.

Investigations are shown.

Hb 11 g/dlWCC 10 × 109/lPlatelets 108 × 109/lMCV 90 flSodium 135 mmol/lPotassium 4.7 mmol/lUrea 9 mmol/lCreatinine 125 μmol/lBilirubin 90 μmol/lAST 70 iu/lAlkaline phosphatase 400 iu/lAlbumin 33 g/lProtein 80 g/lCalcium 2.0 mmol/lPhosphate 0.9 mmol/lCholesterol 7 mmol/lCreatinine kinase 900 u/lChest X-ray Normal-sized heart

(PA view)Clear lung fieldsLooser’s zone right

clavicleECG Sinus rhythm

Partial RBBB

Question 392

A 52-year-old male presented with night sweats andlethargy. On examination, he was pale and had a palpablespleen 7 cm below the costal margin.

Investigations are shown.

Hb 9 g/dlWCC 52 × 109/lPlatelets 680 × 109/lMCV 88 flNeutrophils 67%Myoblasts 8%Myelocytes 10%Metamyelocytes 8%Lymphocytes 2%Eosinophils 1%Basophils 2%Nucleated red cell 2%

1. Suggest two possible diagnoses.2. Apart from the bone marrow, which two

investigations would help differentiate between the two diagnoses?

1. What is the most probable explanation for the chestpain?

2. What investigation would you perform to confirmthe diagnosis?

3. What is the cause of the failure to rise from a sittingposition?

4. Why is the serum cholesterol elevated?

5. How would you account for the raised creatininekinase?

6. List two steps in the management of herhypercholesterolaemia.

7. What is the cause of her jaundice?8. List two investigations you would perform to

confirm the cause of her jaundice.

Page 399: Rapid review of clinical medicine for mrcp part 2

398

Answer 393

The patient has PBC. There is the classic history ofpruritus preceding the onset of jaundice by a few years.The raised serum cholesterol and biochemical andradiological evidence of osteomalacia are suggestive ofchronic cholestasis in a jaundiced patient.Hypercholesterolaemia is the result of reduced biliary

excretion because bile is the only means for the humanbody to dispose of cholesterol; thus, many patients withPBC have xanthelasma and palmar xanthomas, which arestigmata of hypercholesterolaemia. Osteomalacia is dueto vitamin D deficiency resulting from fat malabsorption.Bile excretion into the small intestine is necessary for fat

Answer 392

Both chronic myeloid leukaemia and myelofibrosis aremyeloproliferative disorders characterized by a high (usually>50 × 109/l) WCC consisting of the granulocyte series.Both may be associated with a large spleen, and thereforethe differentiation between the two on clinical groundsalone is difficult. Myelofibrosis is characterized by fibrousreplacement of the marrow, and although the white cell andplatelet count may be very high, a leucoerthyro blastic bloodpicture is more charac teristic. A list of features which mayhelp distinguish chronic myeloid leukaemia from myelo -fibrosis is shown (Table).

Features which may help distinguish chronic myeloid leukaemia from myelo fibrosis

Parameter Chronic myeloid leukaemia Myelofibrosis

WCC >50 × 109/l >50 × 109/l

Large spleen Present Present

Blood film Numerous granulocytes at varying Leucoerythroblasticstages of maturation; may be Tear drop cells leucoerythroblastic (392)

Neutrophil leucocyte Low Normal/highalkaline phosphatase

Philadelphia chromosome Present in 90% Absent

Marrow Numerous granulocytes at Myelofibrosis; stains black with varying stages of maturation reticulin

1. i. Chronic myeloid leukaemia.ii. Myelofibrosis.

2. i. Neutrophil leucocyte alkaline phosphatase.ii. Karyotype for Philadelphia chromosome.

1. Fractured sternum due to osteomalacia.2. Lateral chest X-ray.3. Proximal myopathy due to osteomalacia.4. Reduced excretion of cholesterol owing to biliary

stasis.5. Simvastatin-induced myositis/rhabdomyolysis.

6. i. Dietary measures, low-fat diet.ii. Cholestyramine.

7. PBC.8. i. Antimitochondrial antibodies (IgM2).

ii. Liver biopsy, provided that clotting was notabnormal.

339922

Page 400: Rapid review of clinical medicine for mrcp part 2

Clinical Cases 399

digestion and absorption. Failure to absorb fats results infailure to absorb fat-soluble vitamins, notably vitamins A,D, E and K. Vitamin D deficiency leads to osteomalaciaand pathological fractures. This patient’s presentation isalmost certainly due to a sternal fracture, which may bedetected on a lateral chest X-ray. The raised creatininekinase is unlikely to represent an acute myocardialinfarction, given the chest tenderness. The most likelyexplanation for the high creatinine kinase level issimvastatin-induced myositis. In liver and/or renalfailure, simvastatin may cause frank rhabdomyolysis.Patients also taking fibrates and ciclosporin with thestatins are at risk of this complication. In this case, thebest method of managing the hypercholesterolaemia is bya low-fat diet and by therapy with cholestyramine, a resinwhich binds bile and promotes its excretion.

The diagnosis of PBC is confirmed by thedemonstration of IgM antimitochondrial antibodies(subtype M2), which are present in over 90% of thepatients. A liver biopsy shows a dense infiltration of theportal tract with lymphocytes and plasma cells. There isperiportal fibrosis and cholestasis, and granulomas arepresent around the small bile ducts.

ANF antibodies and rheumatoid factor are present inup to one-third of cases. Associations include CREST(calcinosis, Raynaud’s syndrome, oesophageal problems,scleroderma and telangiectasia) syndrome, membranousglomerulonephritis, RTA and Sjögren’s syndrome.

There is no specific proven therapy, althoughursodeoxycholic acid does retard progression and time tohepatic transplantation. Liver transplantation should beconsidered when the serum bilirubin exceeds100 μmol/l.

A 35-year-old female with Crohn’s disease since the ageof 15 years presented with a six-month history of foul-smelling diarrhoea and weight loss.

Investigations are shown.

Hb 10 g/dlWCC 7 × 109/lPlatelets 199 × 109/lMCV 106 flSerum folate 22 μg/lSerum B12 65 ng/l Faecal fat 50 mmol/l

(NR <20 mmol/l)Schilling test Pre-intrinsic factor 2% B12

isotope excretedPost-intrinsic factor 2% B12

isotope excreted

1. List two possible causes for this patient’s symptoms.2. Which investigation would you perform next?

Question 394

Page 401: Rapid review of clinical medicine for mrcp part 2

400

The patient has a history of Crohn’s disease and nowpresents with steatorrhoea, B12 deficiency and anabnormal Schilling test (Table A), with and withoutintrinsic factor. The two most likely causes for the B12deficiency in Crohn’s disease are primary B12 mal-absorption due to terminal ileal disease, or intestinaldestruction of dietary B12 by bacterial overgrowth withinsmall bowel strictures (394). Both conditions also causesteatorrhoea by impairing fat absorption due todisturbances in bile salt absorption; the former bypreventing absorption of bile salts which are required formicelle formation, and the latter by deconjugation anddehydroxylation of bile salts. Repeating the Schilling testafter a two-week course of metronidazole and tetracyclinemay help to differentiate between terminal ileal diseaseand bacterial overgrowth (Table B). Antibiotics destroythe large concentrations of Escherichia coli and/orBacteroides which are responsible for bacterial overgrowth,and following this the Schilling test (without intrinsicfactor) becomes normal. In terminal ileal disease, the testwill remain abnormal. A small-bowel enema may also helpin the differentiation by demon strating a terminal strictureresponsible for primary B12 malabsorption, and anystructural defects which will predispose to bacterial

overgrowth. Another method of differentiation is byjejunal aspiration and culture. Although E. coli andBacteroides are found in the jejunum, it is unusual to findconcentrations exceeding 106/ml unless there is

Table A The Schilling test

Part 1• Give 1 mg 58Co-B12 orally to a patient who has fasted overnight, and also give 1 mg of non-radioactive B12 by

intramuscular injection to saturate B12 binding proteins. Collect urine for 24 hours• Normal subjects excrete >10% of the radioactive dose

If part 1 is abnormal:

Part 2• Repeat Schilling with oral intrinsic factor• If excretion becomes normal, the primary abnormality is intrinsic factor deficiency• If excretion remains abnormal, then the differential is between primary B12 malabsorption and bacterial

overgrowth

Table B Differentiation of primary B12 malabsorption from bacterial overgrowth

1° B12 malabsorption Bacterial overgrowth

1. Schilling test after antibiotics Abnormal Normal2. Small bowel meal Terminal ileal stricture Fistulae, jejunal strictures3. Jejunal aspirate Normal bacteria High concentration of

Escherichia coli/Bacteroides4. 14C-glycocholate breath test Positive Positive

Table C Causes of B12 deficiency

• Low dietary intake (vegans)

Intrinsic factor deficiency• Congenital• Pernicious anaemia• Gastrectomy• Chronic atrophic gastritis

Primary B12 malabsorption• Terminal ileal disease• Bacterial overgrowth• Coeliac and tropical sprue (longstanding)• Pancreatic insufficiency

Miscellaneous• Fish tapeworm (Diphyllobothrium latum)• Drug, e.g. colchicine, neomycin

1. i. Primary B12 malabsorption.ii. Bacterial overgrowth.

2. Repeat Schilling test after a two-week course of antibiotics.

Answer 394

Page 402: Rapid review of clinical medicine for mrcp part 2

Clinical Cases 401

overgrowth. A 14C-glycocholate breath test ischaracteristically positive in bacterial overgrowth, asbacteria deconjugate bile salts to release 14C-glycine,which is metabolized to 14CO2. The radioactivity in thebreath is easy to measure; however, it is important torealize that the test may also be positive in terminal ilealdisease because unabsorbed radiolabelled bile salts pass

into the colon where the concentration of E. coli is veryhigh, leading to deconjugation of bile salts and liberationof 14C-glycine. In terminal ileal disease, faecal radioactivityis also high, whereas in bacterial overgrowth the radio -activity level is relatively low. Causes of B12 deficiency aretabulated (Table C).

Question 395

A 65-year-old male was seen in the out-patient’s clinicwith a one-year history of dizziness which was worse ongetting out of bed, and occasionally associated withcollapse. The patient also complained of weaknessaffecting both legs. On further questioning, he admittedto urinary incontinence and his wife noted that he wasgenerally slow and had an intermittent tremor affectinghis right hand.

On examination, he appeared slightly withdrawn. Hispulse was 84 beats/min, and blood pressure155/85 mmHg. The heart sounds were normal.Examination of the respiratory and gastrointestinalsystems was normal. On examination of the centralnervous system he had sluggish pupillary reflexes and

nystagmus. Examination of the fundi was normal. Therewas a pill-rolling tremor affecting the right hand. Therewas increased tone in both legs, with the right leg beingaffected more than the left. Power was slightly reduced(grade 4/5) in all limbs. The reflexes were generally brisk.The plantar response in the left foot was extensor, but wasequivocal in the right foot. Sensation was normal. His gaitwas ataxic.

B12 <150 ng/lSchilling

test

part 1

Normal

Abnormal

VeganTranscobalamin II deficiencyPregnancyThyrotoxicosis

Repeat Schilling test

with intrinsic factor

part 2

Normal

Abnormal

Pernicious anaemiaGastrectomyChronic atrophic gastritisCongenital intrinsic factordeficiency

BacterialovergrowthJejunal diverticuliBlind loopsyndrome

Repeat part 1

after antibiotics*

Normal

Abnormal

*10 days of tetracycline and metronidazole

Terminal ileal diseaseFish tapewormDrugs:• colchicine• neomycin• para-amino-salicylic acid

1. What is the most likely diagnosis?2. List two further investigations you would pursue to

confirm your diagnosis.

Investigation of B12

deficiency339944

Page 403: Rapid review of clinical medicine for mrcp part 2

402

Answer 395

The patient has a pill-rolling tremor and increasedmuscular tone, indicating an extrapyramidal syndromesuch as Parkinson’s disease. The presence of additionalneurological findings suggests that he has a Parkinson-plus syndrome. There are several Parkinson-plussyndromes in the literature, but the ones which are mostcommonly tested include multi-system atrophy and theSROS. Multi-system atrophy is the name given to asporadically occurring condition which begins in adultlife, usually in the sixth decade. It is characterized byneuronal cell loss and gliosis changes in:

• The substantia niagra pathways that causeParkinsonism.

• The olivopontocerebellar tract causing ataxia,dysarthria, nystagmus and pyramidal signs.

• The intermediolateral cell column (pre-ganglionicsympathetic neurones in the lateral horns of the spinalcord) causing autonomic nervous system failure.Other causes of autonomic failure are shown (Table).

These changes often overlap and may present as amixture of neurological abnormalities. When features ofParkinson’s disease predominate, the disorder is calledstriatonigral degeneration. When features of theautonomic failure predominate, the disorder is commonlyknown as Shy–Drager syndrome, and when cerebellarfeatures predominate it is called olivopontocerebellaratrophy. When all three systems are involved the disorderis known as multi-system atrophy. Unlike Parkinson’sdisease, there is a shortened life expectancy, with amedian survival of 9.3 years.

In Shy–Drager syndrome impotence is often the firstsymptom. Incontinence is common in both sexes. Speechdiffers from the hypophonic monotony of Parkinson’sdisease, incorporating quivering, straining, slurring, anddysarthric components. Patients may develop aphonia,anarthria and dysphagia, which are the same as inParkinson’s disease. Respiratory stridor can occur inabout 30% of patients. There is no specific treatment formulti-system atrophy and the associated disorders. Theuse of compression stockings, fludrocortisone, andpressor agents such as ephedrine have been used for thepostural symptoms. The response to L-dopa is usuallyabsent or poor.

Steele–Richardson–Olszewski syndromeThis is another multisystem degeneration syndrome. It ischaracterized by cell loss and neurofibrillary tangles in thebrainstem, globus pallidus, subthalamic and dentatenuclei, and causes the syndrome of progressivesupranuclear palsy. It begins in the seventh decade.Patients present with imbalance, particularly fallingbackwards, a coarse dysarthria, involuntary groans anddysphagia. Perseverance may be obvious in speech withrepetition of syllables and words (palilalia) or even ofwhole phrases (palilogia). Involuntary eye closure is oftenseen. The brow may be furrowed and the eyebrows raisedfrom frontalis hyperactivity in an attempt to keep the eyesopen, resulting in a surprised expression. Thecharacteristic feature in SROS is a supranuclear palsy ofvoluntary gaze. Voluntary downgaze is slow andincomplete, but when the patient’s neck is passivelyextended while fixing his gaze on the examiner’s nose, adownward gaze is obtained. Patients may therefore haveproblems with reading and eating. Patients with SROScan masquerade as having Parkinson’s disease because ofaxial rigidity and gait disturbances, but lack features ofdistal Parkinsonism (distal limb akinesia or rigidity).Resting tremor is almost never seen. There is no effectivetreatment. Median survival is about 6–7 years.

Causes of autonomic failure

1. Central (primary)Progressive autonomic failure (no obvious cause)Progressive autonomic failure with multi-system

atrophy (Shy–Drager syndrome)Progressive autonomic failure with Parkinson’s

disease

2. Central (secondary)Central brain lesions: craniopharyn gioma, vascular

diseaseEncephalitis, tabes dorsalis, Chaga’s diseaseSpinal cord lesionsFamilial dysautonomia

3. Distal autonomic neuropathiesGuillain–Barré syndrome, diabetes mellitus,

amyloid, porphyria, rheumatoid arthritis,myaesthenia gravis, Fabry’s disease

4. Tricyclic antidepressants, MAOIs, ganglion-blocking drugs

1. Multi-system atrophy or Shy–Drager syndrome.2. i. Lying and standing blood pressure.

ii. Autonomic nervous system testing (loss of sinus arrhythmias on ECG, loss of reflex bradycardia on carotid sinus massage, absence of a reflex tachycardia after the Valsalva manoeuvre).

Page 404: Rapid review of clinical medicine for mrcp part 2

Clinical Cases 403

A 31-year-old male was admitted to hospital with a two-week history of progressive dyspnoea on exertion andwheeze. He complained of aches and pains in his armsand legs. He had suffered from ulcerative colitis for fouryears, and required several courses of high-doseprednisolone until the last two months, when his colitiswas relatively quiescent. He also had a past history ofhyperthyroidism which was medically treated withcarbimazole. He was currently maintained onsulphasalazine 1 g twice daily for the ulcerative colitis.

On examination, he was dyspnoeic, had mild cervicaland axillary lymphadenopathy, and a generalizedmaculopapular rash. He had a fever of 38.5°C (101.3°F).The heart rate was 110 beats/min and regular. Theblood pressure was 140/80 mmHg. Heart sounds werenormal. Auscultation of the lung fields demonstratedwidespread wheeze. His abdomen was very muscular anddifficult to examine. All other examinations wereessentially normal.

Investigations are shown.

Question 396

Hb 13 g/dlWCC 15.3 × 109/l

(neutrophils 10.1 × 109/l,lymphocytes 1.0 × 109/l,eosinophils 4.2 × 109/l)

Platelets 308 × 109/lSodium 126 mmol/lPotassium 4.3 mmol/lUrea 7 mmol/lCreatinine 100 μmol/lESR 98 mm/hBlood and sputum cultures: Negative

Arterial blood gases (on air):pH 7.4PaCO2 3.3 kPaPaO2 8.7 kPaBicarbonate 21 mmol/lO2 saturation 92%

Chest X-ray (396a)

What is the diagnosis?a. Churg–Strauss syndrome.b. Eosinophilic pneumonitis.c. Allergic bronchopulmonary aspergillosis.d. Pneumocystis jirovecii pneumonia.e. Hodgkin’s lymphoma.

339966aa

Page 405: Rapid review of clinical medicine for mrcp part 2

404

Answer 396

The patient has a generalized illness consisting of awidespread macular rash, generalized lymphadenopathy,and a very high eosinophil count. He has a wheeze, andhis chest X-ray reveals patchy consolidation, which isworse in the right lung. He had been on prednisolone forulcerative colitis but has recently commencedsulphasalazine. The symptoms and signs appeared afterthe drug was commenced, and the most likely diagnosisfor his respiratory problem is an eosinophilic pneumonia.This question tests the reader’s knowledge of causes ofeosinophilia (396b) of which there are many; however,the most common cause is allergy. An allergic reaction todrugs may precipitate a profound eosinophilia with multi-system involvement, including eosinophilic infiltrationinto the lungs. Sulphonamides are a well-recognizedcause of eosinophilia and subsequent eosinophilicconsolidation. Other drugs implicated includeerythromycin, nitrofurantoin and imipramine. The mostobvious management of a drug-related eosinophilia is tostop the culprit agent. Pulmonary involvement respondswell to high-dose steroids, which may be tailed off over afew weeks. However, relapse is not uncommon and some

patients require a maintenance dose of 5 mgprednisolone for several months to prevent recurrence.

The differential diagnosis here is the Churg–Strausssyndrome, which is a small-vessel granulomatousvasculitis characterized by skin rash, respiratoryinvolvement giving asthmatic symptoms, neuropathy andeosinophilia. It is a variant of Wegener’s granulomatosis,but is a much milder disease and does not usually involvethe kidneys. Lymphoma is also associated witheosinophilia; however, it would have to be fairly extensiveto involve the cervical and axillary lymph nodes as well asthe chest. The normal Hb and lymphocyte count are notconsistent with extensive lymphoma.

Although the patient had been on steroids and iscurrently taking carbimazole, his normal neutrophilcount is highly against the development of anopportunistic infection such as TB or Pneumocystisjirovecii pneumonia (previously known as Pneumocystiscarinii pneumonia).

Ulcerative colitis itself is a recognized cause of a mildeosinophilia; however, an eosinophilia as high as in thiscase and pulmonary involvement cannot be explained bythe disease.

b. Eosinophilic pneumonitis.

Eosinophilia

>0.5 × 109

Mild 0.5–2.0

Severe >5

Allergies

Addison’s disease

Lymphoma (Hodgkin’s)

Immunodeficiency

(Wiskott–Aldrich)

Vasculitidies

(Churg–Strauss syndrome)

Radiation

Ulcerative colitis

Drugs:*

SulphonamidesChlorpropamideErythromycinNitrofurantoin

Asthma

Allergic rhinitis

Allergic bronchopulmonary

aspergillosis

Parasites*

or

Blistering

skin disease

Chest X-rayNormal

Abnormal

Eosinophilic pneumonia

Tropical pulmonary

eosinophilia

Eosinophilic leukaemia

Hypereosinophilic syndrome

Wucheria bancrofti

Strongyloides

Other worms

PemphigusPemphigoidErythema multiforme

339966bb

*May be associated with severe eosinophilia and abnormal X-ray.

Page 406: Rapid review of clinical medicine for mrcp part 2

Clinical Cases 405

Question 397

A 16-year-old female is investigated for short stature. Theresults of a dynamic pituitary function tests following

0.15 iu/kg of insulin, 100 μg of LHRH, and 200 μg ofTRH are as follows:

Time Glucose GH Cortisol TSH LH FSH Oestradiol(min) (mmol/l) mu/l (nmol/l) mu/l iu/l iu/l pmol/l

0 4.0 1.4 400 2.0 29 16 420 2.8 12.0 860 5.2 40 24 530 1.6 28 1270 - - - –40 1.5 32 1450 - - - –60 2.2 20 1120 8 34 20 6

1. The following statements are true with respect tothe pituitary function tests above:a. There was an inadequate stimulus for GH

secretion.b. The GH response was abnormal.c. The findings are consistent with pituitary

Cushing’s syndrome.d. The patient is hypothyroid.e. There is evidence of primary hypogonadism.

2. The following investigations would be useful inconfirming the diagnosis:

a. Buccal smear for karyotyping.b. CT scan of the pituitary gland.c. Dexamethasone suppression test.d. Investigation of other family members.e. Thyroid antibodies.

3. The following diagnoses are possible in thispatient:a. Hypothyroidism.b. Turner’s syndrome.c. PCOS.d. Testicular feminization syndrome.e. Coeliac disease.

Question 398

A 70-year-old male on treatment for congestive cardiacfailure with an angiotensin-converting enzyme inhibitor,spironolactone, digoxin and furosemide was admittedwith nausea. An ECG taken on admission is shown(398).

Which investigation would you perform next?a. Serum potassium.b. Serum magnesium.c. Cardiac troponin I.d. Echocardiography.e. 24-hour ECG.

339988

Page 407: Rapid review of clinical medicine for mrcp part 2

406

Answer 397

In order to answer this question correctly the readerrequires knowledge of the normal results of dynamicpituitary function tests and gonadotrophin and sexhormone levels (Tables A and B).

In this case, there is an adequate stimulus to GH andcortisol secretion. The GH response is normal, and thereis no evidence of hypothyroidism. The basal oestradiol islow and the basal LH and FSH are high, suggestingprimary hypogonadism. The LHRH test is normal (as isthe case) in primary hypogonadism. The most probablediagnosis is Turner’s syndrome. Turner’s syndrome ischaracterized by a single X chromosome (karyotype 45XO). It has an incidence of approximately 1 in 2,500.The main abnormality is gonadal dysgenesis, theconsequences of which are primary amenorrhoea andinfertility. Oestradiol levels are low, resulting in elevatedgonadotrophin levels via the positive feedback on thepituitary. The patients are typically short and have anumber of other morphological abnormalities (Table C).The diagnosis is confirmed by a buccal smear whichreveals an absent Barr body. There are several othercauses of hypogonadism in females (Table D).

Polycystic ovary syndrome may also cause partialhypogonadism, but in contrast with the other causes ofhypogonadism given in Table D, the oestradiol levels areonly slightly reduced and the FSH level is low (Table E).Although coeliac disease is a recognized cause of shortstature (and also delayed puberty), the gonadotrophinresponse to LHRH is entirely normal. The commonly

examined causes of short stature in the MRCP andsimilar examinations are listed (Table F).

Testicular feminization syndrome is due to theabsence of testosterone receptors on end organs. Thepatient has male karyotype, but is phenotypically femaleowing to the absence of the effect of testosterone in thenormal development of male gonads. Testicles arepresent but are usually intra-abdominal or in the hernialorifices. Serum testosterone levels are very high. Lack oftestosterone receptors on the pituitary means that there isno negative feedback system to inhibit gonadotrophinsecretion, therefore both LH and FSH are very high.Excess stimulation of the testicular remnants leads toexcess oestrogen production by the testes, which iscomparable with levels found in females. The oestrogeninitiates breast development and maintains breasts. Lackof the testosterone effect means that pubic hair may beabsent or scanty. These patients are not particularly short.They present with amenorrhoea. The diagnosis can beconfirmed by karyotyping.

Pituitary Cushing’s is unusual in this age group. It isdue to a basophil adenoma within the pituitary whichsecretes ACTH, causing adrenal hyperplasia and excesscortisol production. The serum cortisol is >900 nmol/land there is loss of diurnal variation, i.e. the midnightcortisol is also >900 nmol/l (normally 150–750 nmol/l).In the majority of cases high-dose dexamethasone (8 mgper day) suppresses cortisol production (see Answer 173).In some cases the cortisol response to dexamethasone isunaffected and in these instances the main definitivemethod of diagnosing pituitary Cushing’s is to performinferior petrosal sinus sampling for ACTH.

Table A Normal responses to dynamic pituitary function tests

• Following an insulin tolerance test, the GH level rises to >20 mu/l and the serum cortisol level should rise to≥550 nmol/l and increase by 180 nmol/l from the basal level. The blood sugar should fall below 2.2 mmol/lto ensure adequate metabolic stress for GH and cortisol secretion. Note: An insulin tolerance test should notbe performed if the basal cortisol level is below 180 nmol/l

• Following a TRH test, the TSH should increase by >2 mu/l from the basal level. Basal TSH levels arebetween 0.5–5 mu/l

• Following an LHRH test, the LH and FSH should double from the basal level

Table B Normal gonadotrophin and sex hormone levels in males and females

Females MalesFollicular phase, FSH 1–10 iu/l and LH 2–20 iu/l FSH 1–7 iu/l and LH 1–10 iu/lOestradiol <110 pmol/l Testosterone 10–35 nmol/lTestosterone 0.5–3 nmol/l

1. e. There is evidence of primary hypogonadism.2. a. Buccal smear for karyotyping.3. b. Turner’s syndrome.

Page 408: Rapid review of clinical medicine for mrcp part 2

Clinical Cases 407

Table C Morphological features of Turner’s syndrome

Skeletal Renal anomalies• Short stature • Horse-shoe kidneys• Cubitus valgus• Short fourth metacarpals Miscellaneous abnormalities• High-arched palate* • Webbed neck (fused cervical vertebrae)• Shield chest with widely spaced nipples • Lymphoedema

• Low hair lineCardiac anomalies • Multiple naevi• Septal defects • Increased incidence of auto-immune thyroiditis • Coarctation of the aorta and diabetes

* Also present in Marfan’s syndrome, homocystinuria and Friedreich’s ataxia.

Table D Causes of primary hypogonadism in females

• Turner’s syndrome • Ovariectomy• Pure ovarian dysgenesis* • Chemotherapy• Swyer’s syndrome** • Steroid biosynthetic defect • Auto-immune oophoritis • (17-α-hydroxylase deficiency)

* The pathology is the same as Turner’s, but differs in that there is a normal complement of chromosomes and theother morphological features of Turner’s syndrome are absent. Some patients may have sensorineuronal deafness.** Swyer’s syndrome is characterized by the XY karyotype and gonadal dysgenesis. Affected individuals arephenotypically female. It is associated with a high incidence of gonadoblastomas.

Table E Characteristics of PCOS (Stein–Levental syndrome)

• Irregular menstruation • Testosterone level slightly higher than 3.0 nmol/l• Impaired fertility • High oestrone levels• Hirsutism • Slightly elevated LH level• Acne • Slightly depressed FSH level• Obesity • LH/FSH ratio generally exceeds 2• Low oestradiol levels

Table F Causes of short stature

• Constitutional delay* • Drugs, particularly steroids• Familial • Emotional deprivation• Systemic disorder, e.g. coeliac disease • Skeletal abnormalities (achondroplasia, • Endocrine causes, e.g. hypothyroidism or GH deficiency vitamin D-resistant rickets)

* Slow growth from the beginning, spontaneous puberty, slight delay in bone maturity, normal LHRH test.

Answer 398

Tall tented T-waves ( >1.25 mV; 12 small squares) is anECG feature of hyperkalaemia. In this case, the cause ofhyperkalaemia is a combination of potassium-sparingdrugs and obstructive uropathy. (See Table B, Answer220.)

a. Serum potassium.

Page 409: Rapid review of clinical medicine for mrcp part 2

408

Question 400

A young alcoholic who is well known to the local Accidentand Emergency Department was admitted to hospital afterbeing found collapsed. On examination, he was unrousable.His temperature was 36.5°C (97.7°F). His heart rate was

120 beats/min, and regular, with a weak pulse volume. Theblood pressure was 80/40 mmHg. The heart sounds werenormal. Auscultation of the lung fields revealed a fewcrackles at both lung bases. The abdomen was rigid andbowel sounds were absent. Rectal examinationdemonstrated soft brown stool. Examination of the fundiwas normal.

Investigations are shown.Hb 7 g/dlWCC 23 × 109/lPlatelets 50 × 109/lMCV 102 flINR 1.9Sodium 131 mmol/lPotassium 5.6 mmol/lUrea 17 mmol/lCreatinine 200 μmol/lBicarbonate 16 mmol/lCalcium 1.9 mmol/lPhosphate 1.0 mmol/lAlbumin 30 g/lAST 64 iu/lBilirubin 53 μmol/lAlkaline phosphatase 204 iu/lGamma GT 190 iu/lGlucose 21 mmol/lUrinalysis Protein +1

Blood 0Ketones +1

1. What is the diagnosis?a. Diabetic ketoacidosis.b. Perforated duodenal ulcer.c. Gastrointestinal haemorrhage secondary to

oesophageal varices.d. Acute pancreatitis.e. Methanol poisoning.

2. List five investigations that you would perform onthis patient immediately.

Question 399

A 65-year-old female was referred to a neurologist with asix-month history of progressive weakness and clumsinessof her left hand. She mentioned having particulardifficulty in washing the dishes and using the gear stickwhile driving her car. Her husband noticed that her gaithad been unsteady and she had suffered a few falls in thepast year. On systemic enquiry there was no history ofheadaches, visual disturbance or weight loss, but she hadexperienced dizziness on turning her head suddenly.

On examination she was thin and had markedkyphoscoliosis. There was evidence of wasting of thesmall muscles of the hands, which was more prominenton the left side than the right. There was reduced poweron flexion and abduction of the arms. The supinator andbiceps reflexes on the left side were absent but present onthe right. The triceps reflexes were brisk bilaterally. Thepower in both lower limbs was grade 4 out of 5 in allmuscle groups. The tone was increased. The ankle andknee reflexes were brisk bilaterally. The left plantarresponse was upgoing but the right one was equivocal.Sensation was normal with the exception of vibrationsense at both ankle joints.

1. What is the diagnosis?a. Motor neurone disease.b. Syringomyelia.c. Multiple sclerosis.d. Cervical myelopathy.e. Subacute combined degeneration of the spinal

cord.2. Which investigation would you perform to confirm

the diagnosis?a. Myelography.b. MRI scan cervical spine.c. Nerve conduction studies and electromyography.d. CSF analysis for oligoclonal bands.e. Serum B12 level.

Page 410: Rapid review of clinical medicine for mrcp part 2

Clinical Cases 409

A 44-year-old West Indian male presented with a three-weekhistory of polydipsia, polyuria and nocturia. He was passingcopious amounts of urine several times per day, and feltexcessively thirsty. According to his wife he was consumingup to five 2-litre bottles of cola per day, and several large (1-litre) flasks of water. His appetite and weight had beennormal. He was not taking any medication. For four years hehad been intermittently unwell, firstly with chest problems,and then with arthritis affecting his hands and feet. Therespiratory problems – which were characterized by a drycough and accompanied by night sweats, pains in his anklesand tender, raised lesions on his shins – were of relativelysudden onset. The joint pains responded to pain killersprescribed by a doctor whom he saw in the West Indies, butthe chest symptoms persisted for several months despite fourlong courses of antibiotics. The skin lesions on the shinsresolved about three months after the onset of his illness. Hewas eventually referred to a chest physician withbreathlessness on exertion, and after respiratory functiontests (top panel of results) and a chest X-ray was told he hadscarring on the lung, possibly due to several previous chestinfections, and was prescribed a steroid inhaler.

The patient’s breathing remained stable. He was able towalk for over two miles on the flat, but experiencedbreathlessness on negotiating inclines. After two years, hedeveloped pain and swelling in his hands and feet, whichwere controlled to some extent with co-proxamol. His onlyother complaint was of a gritty feeling in the eyes as the dayprogressed, and dryness of the mouth for several monthsbefore he developed polyuria. He worked as a busconductor. His brother, aged 58, had rheumatoid arthritiswhich was complicated with lung fibrosis.

On examination, the patient appeared well at rest, buthad mild bilateral parotid gland enlargement. Theappearance of the eyes is shown (401a). His eyes were red,but the visual acuity, visual fields, pupils and fundi werenormal. There was no focal neurological deficit. There wasno evidence of a previous BCG scar. There was boneswelling affecting several phalanges in the hands and feet.There was no joint swelling. Cardiovascular examination wasnormal. Auscultation of the lung fields revealed fineinspiratory crackles in the anterior chest, but auscultation ofthe posterior aspect of the chest was normal.

Investigations are shown (second panel of results).

Actual PredictedFEV1(l) 2.2 4.1FVC (l) 2.5 5.4TLC (l) 3.3 6.3KCO (mmol/min/kPa) 0.9 1.40

Sodium 143 mmol/lPotassium 4.5 mmol/lUrea 6 mmol/lCreatinine 90 μmolCalcium 2.3 mmol/lPhosphate 1.1 mmol/lAlkaline phosphatase 120 iu/lBlood glucose 4.2 mmol/lX-ray of hands (401b)

Fluid deprivation test:Time (min) PmOsm/l UmOsm/l0 303 12060 310 120120 318 120180 328 120240 335 1208 h after desmopressin 800

1. What were the painful lesions on the shins?2. What do the patient’s respiratory function tests

demonstrate?3. What does the X-ray of his hands reveal?4. Why does he have gritty eyes?5. What is the interpretation of the water deprivation

test?6. What is the overall or underlying diagnosis?7. What three investigations would you perform to

confirm your diagnosis?8. Give three therapeutic steps in his management.

Question 401

440011aa 440011bb

Page 411: Rapid review of clinical medicine for mrcp part 2

410

Answer 400

The clinical scenario is of an alcohol abuser who presentswith unconsciousness and a rigid abdomen. Thedifferential diagnosis is between a perforated duodenalulcer, acute pancreatitis, ethylene glycol poisoning andmethanol poisoning, as all four are recognized causes ofabdominal pain in alcoholism. The raised gamma GTsupports alcohol abuse. The low calcium favours eitheracute pancreatitis or ethylene glycol poisoning (oxidationof ethylene glycol to oxalic acid results in chelation of

calcium). However, the diabetic state is more in keepingwith acute pancreatitis, which results in damage to the

Diagnoses to consider in an unconsciousalcoholic

• Hepatic encephalopathy• Wernicke’s encephalopathy• Methanol poisoning• Ethylene glycol poisoning• Alcohol intoxication• Subdural haemorrhage• Haemorrhage from peptic ulcer or varices• Acute haemorrhagic pancreatitis• Tuberculous meningitis• Alcohol-induced hypoglycaemia

1. d. Acute pancreatitis.2. i. Serum amylase.

ii. Arterial blood gases.iii. Full clotting screen.iv. Chest X-ray.v. Ultrasound of the abdomen.

Answer 399

The patient presents with progressive weakness affectingher arms and hands. She has a combination of upper andlower motor neurone signs in the upper limbs and purelyupper motor neurone signs in the lower limbs. There isloss of vibration sense at the level of the ankles. This maybe a normal finding in patients in their seventh decadebut may indicate pathology in the dorsal columns.

The differential diagnosis is between motor neuronedisease, cervical cord tumour and cervical myelopathy,and subacute combined degeneration of the spinal cord.

Features against motor neurone disease include hergender (more common in males), the mid-cervical reflexpattern (absent supinator and biceps reflex butexaggerated triceps reflex), which is more suggestive ofcervical myelopathy, the association of her symptomswith rigidity in the neck and dizziness on turningsuddenly, which again suggests pathology in the cervicalcord, and the absence of weight loss, which is usuallymarked in motor neurone disease.

Subacute combined degeneration of the spinal cord isa remote possibility, but one would want more evidenceof a sensory neuropathy in this condition, for example,absence of light touch and proprioception and absentknee or ankle jerks.

Both cervical myelopathy and a cervical cord tumourmay present with predominantly upper limb signs, whichcharacteristically produce lower motor neuronedisturbances at the level of the lesion and upper motorsigns below it. The best investigation to differentiatebetween the two is an MRI scan of the spinal cord, which

has superseded myelography. In this case, the lesionappears to affect the C5 and C6 segments of the cervicalcord, hence the absent biceps and supinator reflexes onthe left side. This would not explain the weakness andwasting of the muscles of the hand as they are suppliedby C8 and T1; however, small muscle wasting in cervicalmyelopathy appears to be due to reduced blood flow tothe lower segments of the cord rather than to directcompression of the C8 and T1 spinal nerves. It is worthnoting that sensory disturbance in the upper limbs maybe absent or very mild in cervical myelopathy.

Cervical myelopathy is due to bulging or extrusion ofthe disc material into the cervical canal, which results inpressure atrophy and ischaemia. The main cause isosteoarthritis of the cervical spine. The posterior columns(dorsal tracts carrying proprioception, vibration sense andlight-touch fibres) and the lateral columns (pyramidaltracts carrying upper motor neurones) are affected. The5th–7th cervical segments are most commonly affected.The symptoms are those of lower motor neurone signs atthese levels and upper motor neurone signs below. Neckstiffness and upper limb pain may be present. Spasticity ofthe lower limbs is common and more pronounced thanphysical examination would suggest. Ataxia may occurowing to dorsal column involvement (sensory ataxia).Lhermitte’s sign (electrical shock feeling down the spinalcord and into the legs) is recognized, but may also occurwith cervical cord tumours, multiple sclerosis andsubacute combined degeneration of the spinal cord.

The presence of lower motor neurone signs and theabsence of cerebellar signs is against multiple sclerosis.

The absence of dissociated sensory loss is againstsyringomyelia.

1. d. Cervical myelopathy.2. b. MRI scan cervical spine.

Page 412: Rapid review of clinical medicine for mrcp part 2

Clinical Cases 411

islets of Langerhans, which produce insulin. Methanolpoisoning may also cause hyperglycaemia, buthypocalcaemia is unusual. Furthermore, the acidosis ismore severe in methanol poisoning and there is usuallyevidence of optic atrophy. The anaemia is quite severe,suggesting that there may be ongoing peritoneal bleedingfrom the pancreatitis. The MCV is high, indicating eitheralcohol-related marrow toxicity or alcohol-related liverdamage. The low platelets indicate either hypersplenismsecondary to portal hypertension or a DIC processcomplicating acute pancreatitis. The raised INR may bedue to hepatocellular dysfunction or it may be part of theDIC. Diabetic ketoacidosis would not explain the

unconsciousness in this patient given the fact that thebicarbonate is not very low and the ketonuria is mild. Themild ketonuria in this case is due to reduced dietary intake.Rhabdomyolysis is unlikely because the creatinine is notvery high, and there is no evidence of blood in the urineon dipstick testing (remember that myoglobin gives apositive dipstick test for blood).

A very high serum amylase would confirm acutepancreatitis in this case. The most common causes of deathin acute pancreatitis are haemorrhage, renal failure andrespiratory failure; therefore an FBC, blood gases, chest X-ray and clotting would give important prognosticinformation.

1. Erythema nodosum. Causes of erythema nodosumare shown (Table A).

2. A restrictive lung defect with a low KCO. Thedifferential diagnosis for conditions causing thispicture is discussed in Answer 217.

3. The X-ray of the hands demonstrates several lucentlesions at the ends of most phalanges.

4. Bilateral keratoconjunctivitis. 5. The fluid-deprivation test reveals an inability to

concentrate urine until the administration ofexogenous desmopressin (an ADH analogue), whichis indicative of cranial diabetes insipidus.Interpretation of a fluid deprivation test is discussedbelow. Conditions causing cranial diabetes insipidusand an abnormal chest X-ray include sarcoidosis, TBand histiocytosis X. In rare situations, bronchialcarcinoma may metastasize to the hypothalamus orpituitary and cause diabetes insipidus. Other causes ofdiabetes insipidus are shown (Table B).

6. The combination of erythema nodosum, ocular, lung,bone, parotid and lachrymal gland involvementstrongly suggests the diagnosis of sarcoidosis.

7. i. Transbronchial lung biopsy to demonstrate non-caseating granulomas.

ii. MRI scan of the pituitary to demonstrate high-intensity lesions in the mid-brain and hypothalamus.

iii.Serum ACE level. This is not specific for the diagnosis of sarcoidosis and may be elevated in TB, histoplasmosis, berylliosis, silicosis, lymphoma, diabetes mellitus, chronic liver disease, hyperthyroidism and Gaucher’s disease.

8. i. High-dose steroids. Indications for high-dose steroids in sarcoidosis include cerebral involvement, acute diffuse lung disease, uveitis and hypercalcaemia.

ii. Desmopressin.iii.Artificial tears.

Answer 401

Table B Causes of diabetes insipidusCranialFamilialAutosomal dominantDIDMOAD syndromeCerebral tumourSarcoidosisTBHistiocytosis XTraumaPituitary haemorrhage

NephrogenicX-linked (vasopressin

receptor-2 gene)Autosomal recessive

(aquaporin receptorgene)

HypokalaemiaHypercalcaemiaLithiumPost-obstructive

uropathySickle cell anaemiaGlibenclamideAminoglycosidesAmphotericin

Table A Causes of erythema nodosumCommon conditionsStreptococcal

infections, including rheumatic fever

SarcoidosisOral contraceptive pill

and pregnancyInflammatory bowel

diseaseSulphonamide drugs

Other conditionsBehçet’s syndromeTBLeprosyHistoplasmosisYersiniaNeisseria meningitidis

and gonococcal infections

Pasteurella pestis

Sarcoidosis is a multisystem granulomatous disease ofunknown aetiology that usually affects females morecommonly than males. It is much more common in blackpeople, and may present acutely with the Lofgren’ssyndrome, which is characterized by erythema nodosum,

Page 413: Rapid review of clinical medicine for mrcp part 2

412

arthralgia, dry cough, bilateral hilar lymphadenopathy onthe chest X-ray, and anterior uveitis. Patients withsarcoidosis may have lacrimal and parotid glandinvolvement, which is characterized by gland enlargementand sometimes referred to as Mikulicz syndrome (causes ofparotid gland enlargement are listed in Table C). Affectedpatients may complain of xerostomia and gritty eyesrespectively, the latter being due to keratoconjunctivitisresulting from impaired lacrimation. Mikulicz syndromemay also complicate TB, lymphoma and leukaemia.Mikulicz syndrome is relatively common and when it isassociated with acute uveitis and facial nerve palsy it isreferred to as Heerfordt’s syndrome. In most patients theacute illness will resolve, although some will continue tohave respiratory symptoms for up to two years, and a smallpercentage develop a chronic illness characterized byrelapses and remissions.

In chronic cases lung parenchyma becomes involved,leading to fibrosis commonly affecting the apices. Ocularinvolvement is well recognized. Manifestations includeanterior and posterior uveitis, which are characteristicallyassociated with visual symptoms and pupillaryabnormalities, and episcleritis, scleritis, choroiditis,cataracts and glaucoma. Corneal calcification complicatinghypercalcaemia may also occur in sarcoidosis.

Skin infiltration occurs in the chronic form of the illnessand comprises papules on the eyelids and around themouth. Papules affecting the nose may be disfiguring and

are referred to as lupus pernio. Arthralgia affecting thehands and feet is common; X-rays demonstrate smalllucent cysts in multiple phalanges. Cerebral involvement israre, but features include cranial nerve lesions, asepticmeningitis, psychosis and a multiple sclerosis-typesyndrome. Mononeuritis multiplex is a recognizedmanifestation of the peripheral nervous system.Hypercalcaemia occurs owing to the presence of excessiveproduction of 1,25-DHCC by granulomatous cells. Otherconditions where a similar phenomenon occurs include TBand Candida infection.

Water deprivation testIn patients with polyuria and polydipsia and a normalblood glucose, the differential diagnosis is betweencranial diabetes insipidus, nephrogenic diabetes insipidusand compulsive water deprivation. The water deprivationtest will help differentiate compulsive water drinkingfrom both types of diabetes insipidus (Table D).However, the differentiation of cranial diabetes insipidusfrom nephrogenic diabetes insipidus involvesmeasurement of plasma and urine osmolality followingadministration of desmopressin. Patients with cranialdiabetes insipidus are deficient in ADH and responddramatically to desmopressin with an increase in urineosmolality, whereas patients with nephrogenic diabetesinsipidus are resistant to the effects of ADH and thereforethe urine osmolality is not affected by desmopressin.

Table D After a water deprivation test

P mOsm/l U mOsm/lNormal 300 Over 720 or double the plasma osmolalityCompulsive water drinking 300 Incomplete response, i.e. the U mOsm/l is not

quite 720 and does not exceed twice the value of the P mOsm/l. Typical value 600–700 mOsm/l

Cranial diabetes inspidus >300 <150; over 720 mOsm/l after desmopressinNephrogenic diabetes insipidus >300 <150; remains <150 mOsm/l after desmopressin

Table C Causes of parotid gland enlargement

• Mumps • Haemochromatosis• Sarcoidosis • Amyloidosis• Lymphoma • Acromegaly• Leukaemia • Malabsorption syndromes• Sjögren’s syndrome • Drugs; propylthiouracil• Alcohol abuse • Toxins; lead• Bulimia • Hyperlipidaemia• Cystic fibrosis

Page 414: Rapid review of clinical medicine for mrcp part 2

Clinical Cases 413

Question 402

A 26-year-old female presented with a six-month historyof light-headedness on hanging the washing. She wasseen by her GP at the onset of her illness and he noted ablood pressure of 80/50 mmHg in both arms but noother abnormality. Over six months she had one episodeof weakness and numbness of the right arm and face foralmost an hour and two near syncopal episodes. Shedeveloped increasing lethargy, night sweats, and achesand pains in her upper limbs. Her appetite had reducedand she had lost almost 3 kg in weight. There was noother past medical history of note. She had smoked 10cigarettes per day for three years from the age of 17.

On examination she had flushed cheeks but there wasno skin rash. The heart rate was 90 beats/min andregular. The systolic blood pressure was 80 mmHg inboth upper limbs, but the diastolic blood pressure couldnot be ascertained. On examination of the precordium,the apex was heaving in nature but not displaced. Onauscultation there was an audible fourth heart sound.The radial pulses were absent bilaterally, the brachialpulses were weak bilaterally, the right carotid pulse wasabsent. The left carotid pulse was prominent. Bothfemoral pulses were easily palpable. Neurologicalexamination was normal.

Investigations are shown.

Hb 9.4 g/dlWCC 8 × 109/lPlatelets 430 × 109/lESR 112 mm/hSodium 135 mmol/lPotassium 4.5 mmol/lUrea 6 mmol/lCreatinine 84 μmol/lGlucose 4.5 mmol/lCholesterol 5.3 mmol/lAutoantibody screen Rheumatoid factor absent

Antinuclear antibodies present in a titre of 1 in 16

Syphilis serology VDRL and TPHA absentChest X-ray Normal-sized heart

Clear lung fieldsECG Voltage criteria for left

ventricular hypertrophyCT scan brain Normal

1. What is the diagnosis?a. Giant cell arteritis.b. Polyarteritis nodosa.c. Buerger’s disease.d. Takayasu’s arteritis.e. Relapsing polychondritis.

2. Which investigation would you request to confirmthe diagnosis?a. Aortography.b. Invasive blood pressure monitoring.c. Serum antineutrophil cytoplasmic antibody.d. Fluorescent treponemal antibody.e. Cardiac biopsy.

3. Which of the three below would best explain thereason for left ventricular hypertrophy on the ECG?a. Cardiac amyloid.b. Essential hypertension.c. Hypertension secondary to renal artery stenosis.d. Chronic renal failure.e. Fibromuscular dysplasia.f. Renal vasculitis.g. Acquired co-arctation of the aorta.h. Reduced distensibility of the ascending aorta.

Page 415: Rapid review of clinical medicine for mrcp part 2

414

Answer 402

The absence of pulses in the arms and neck and theassociated high ESR value in a young woman should raisethe diagnosis of Takayasu’s arteritis. Takayasu’s arteritis isa rare disorder with a prevalence of 1.5–2 per million. Ithas a strong female predisposition and is more commonin parts of Africa and Asia. The mean age of onset is 29years, but it can occur from any age between infancy andmiddle age. It is very uncommon after middle age. Thecondition is caused by granulomatous vasculitis of largevessels, particularly the aortic arch, which results inobliterative changes in the lumina of the vessels affected.Inflammation may be multi-segmental, interspersed withnormal segments. Symptoms comprise a systemic illness(sweats, weight loss, anorexia and myalgia) as well asvascular insufficiency in the limbs affected. Physical signs

include pulseless vessels. Hypertension is present in 50%of cases, but was not detected in this case because ofdisease in both subclavian arteries. The causes ofhypertension include renal artery stenosis, acquired co-arctation and reduced distensibility of the aortic arch.Complications are those of long-standing hypertension,aortic regurgitation and congestive cardiac failure.Inflammatory markers are usually raised and there may bean anaemia of chronic disease, but the diagnosis is aclinical one and can be confirmed with aortography. Inthis case, the patient had an occluded brachiocephalicand left subclavian artery. The differential diagnosis isbetween other conditions which may produce an aorticarch syndrome, such as syphilitic aortitis and relapsingpolychondritis; however, in this case syphilis serology isnegative and there are no clinical features of relapsingpolychondritis. Treatment is with steroids and vascularstents.

Aortography in this patient demonstrated occludedinnominate and left subclavian arteries (402). Only theleft common carotid artery is patent.

1. d. Takayasu’s arteritis.2. a. Aortography.3. c. Hypertension secondary to renal artery stenosis.

g. Acquired co-arctation of the aorta.h. Reduced distensibility of the ascending aorta.

440022

Page 416: Rapid review of clinical medicine for mrcp part 2

Data Interpretations Tutorials 415

Data Interpretations Tutorials

Condition Ca PO4 Alkaline Additionalphosphatase

1° and 3° hyperparathyroidism ↑ ↓ ↑

2° hyperparathyroidism/CRF ↓/N ↑ ↑

Hypoparathyroidism ↓ ↑ N PTH absent

Pseudohypoparathyroidism ↓ ↑ N PTH elevated

Magnesium deficiency ↓ ↑ ↑ PTH normal

Pseudopseudohypoparathyroidism N N N

Osteomalacia/rickets ↓ ↓ ↑

Osteoporosis N N N

Myeloma ↑ N N*

Paget’s disease N** N ↑ ↑ urinary hydroxyproline

Malignancy (bony metastases) ↑ N/↑ ↑

Sarcoidosis N/↑ N N

Vitamin D intoxication ↑ ↑ N

* Increased with pathological fractures. ** Increased after prolonged immobilization.

Calcium Biochemistry

Genetics

Patterns of inheritance

Autosomal dominant inheritance• Males and females are equally affected.• All individuals inheriting the abnormal gene are affected.• Offspring of affected parents (irrespective of the parental

sex) have a 50% chance of inheriting the disease.• This is in contrast to X-linked conditions where male-to-

male transmission of disease does not occur.

Autosomal recessive inheritance• Males and females are equally affected, but are fewer in

number than in autosomal dominant conditions.• Not all generations will be affected.• If both parents are carriers for the recessive gene 25% of

the offspring will be affected, 50% will become carriers,but will not have the disease and 25% will not have theabnormal gene.

• If an affected individual marries a carrier, then 50% off -spring will be affected and 50% offspring will be carriers.

• If an affected individual marries another affected, then alloffspring will be affected.

X-linked dominant inheritance• Males and females are both affected.• No male-to-male transmission.• Affected males transmit the disease to 100% of their

daughters.• An affected female will transmit the disease to 50% of all

offspring.

X-linked recessive inheritance• Males are affected.• Females are carriers.• No male-to-male transmission.• Daughters of affected males will be carriers.• Female carrier will have 50% affected sons and 50%

daughters who are carriers.

Inheritance of maternal mitochondrial genetic abnormalities• During conception, only mitochondria from the ovum

are passed on to the zygote.• Affected females will pass the disease to 100% off spring.• Affected males will not transmit disease to offspring.• Examples include Leber’s optic atrophy, Kearnes–Sayer

syndrome, MELAS (mitochondrial encephalopathy,lactic acidosis and stroke-like syndrome), MERF(mitochondrial encephalopathy and red ragged fibres).

Page 417: Rapid review of clinical medicine for mrcp part 2

Audiograms

An audiogram of both ears may be presented on the samegraph, or there may be a separate audiogram for each ear.A separate suffix will be given to differentiate between airconduction (AC) and bone conduction (BC).

In sensorineuronal deafness, both air and bone conduc -tion are diminished. The air–bone gap is usually narrow,i.e. there is not much difference between air and boneconduction. In conduction deafness, bone conduction issuperior to air conduction on the affected side.

Normal hearing can detect sounds of a frequencybetween 250–8,000 Hz at a sound intensity of 0–10 dB(1). The following audiograms are most commonlytested in higher post-graduate examinations.

Sensorineuronal deafness at progressively higher fre -quencies. In degenerative sensorineuronal loss (presby cu-sis), deafness is usually bilateral most profound at sounds

with high frequencies (2). Other causes of bi lateralsensorineuronal deafness include congenital rubella infec -tion, mumps infection and drugs such as high dose loopdiuretics and aminoglycosides.

Fluctuating low tone deafness (below 4,000 Hz) sug -gests Ménière’s disease. This is bilateral in 10% of cases(3) and may progress to sensorineuronal deafness in 25%of patients (2).

Sudden and profound hearing loss at a frequency of4,000 Hz is suggestive of noise-induced hearing loss (4).

Unilateral sensorineuronal hearing loss (5) may bedegenerative or due to unilateral auditory nerve damageas a result of a neuropathic process. It should also raisethe suspi cion of lesion in the cerebello-pontine angle onthe affected side, for example, an acoustic neuroma.

416

125 250 500 1000 2000 4000 8000Frequency HZ

125 250 500 1000 2000 4000 8000Frequency HZ

-10

0

10

20

30

40

50

60

70

80

90

100

110

120

130

140

Hea

ring

leve

l (dB

)

-10

0

10

20

30

40

50

60

70

80

90

100

110

120

130

140

Hea

ring

leve

l (dB

)

Right

Left

Right

Left

Unmasked

AC

BC

125 250 500 1000 2000 4000 8000Frequency HZ

125 250 500 1000 2000 4000 8000Frequency HZ

-10

0

10

20

30

40

50

60

70

80

90

100

110

120

130

140

Hea

ring

leve

l (dB

)

-10

0

10

20

30

40

50

60

70

80

90

100

110

120

130

140

Hea

ring

leve

l (dB

)

Right

Left

Right

Left

Unmasked

AC

BC

11

22

Page 418: Rapid review of clinical medicine for mrcp part 2

Data Interpretations Tutorials 417

125 250 500 1000 2000 4000 8000Frequency HZ

125 250 500 1000 2000 4000 8000Frequency HZ

-10

0

10

20

30

40

50

60

70

80

90

100

110

120

130

140

Hea

ring

leve

l (dB

)

-10

0

10

20

30

40

50

60

70

80

90

100

110

120

130

140

Hea

ring

leve

l (dB

)

Right

Left

Right

Left

Unmasked

AC

BC

125 250 500 1000 2000 4000 8000Frequency HZ

125 250 500 1000 2000 4000 8000Frequency HZ

-10

0

10

20

30

40

50

60

70

80

90

100

110

120

130

140

Hea

ring

leve

l (dB

)

-10

0

10

20

30

40

50

60

70

80

90

100

110

120

130

140

Hea

ring

leve

l (dB

)

Right

Left

Right

Left

Unmasked

AC

BC

125 250 500 1000 2000 4000 8000Frequency HZ

125 250 500 1000 2000 4000 8000Frequency HZ

-10

0

10

20

30

40

50

60

70

80

90

100

110

120

130

140

Hea

ring

leve

l (dB

)

-10

0

10

20

30

40

50

60

70

80

90

100

110

120

130

140

Hea

ring

leve

l (dB

)

Right

Left

Right

Left

Unmasked

AC

BC

33

44

55

Page 419: Rapid review of clinical medicine for mrcp part 2

In the example below, bone conduction is superior toair conduction sug gesting conduction deafness (6).Causes of conduction deaf ness are shown (Table).

418

Causes of conduction deafness

Ear waxChronic suppurative otitis mediaMiddle ear granulomaOtosclerosis

125 250 500 1000 2000 4000 8000Frequency HZ

125 250 500 1000 2000 4000 8000Frequency HZ

-10

0

10

20

30

40

50

60

70

80

90

100

110

120

130

140

Hea

ring

leve

l (dB

)

-10

0

10

20

30

40

50

60

70

80

90

100

110

120

130

140

Hea

ring

leve

l (dB

)

Right

Left

Right

Left

Unmasked

AC

BC

• Right-heart saturations do not exceed 75%. Saturations more than this are suggestive of a left-to-right shunt.

• Left-heart saturations vary from 96–98%. Saturations less than this are suggestive of a right-to-left shunt.

• In right-to-left shunts, the arterial saturations do not change with inspired high-concentration oxygen.

• A VSD with a right-to-left shunt and pulmonary stenosis can be differentiated from Fallot’s tetralogy byexamining the oxygen saturation in the left ventricle and the ascending aorta. In the case of a VSD, thesaturations in the left ventricle and the aorta will both be low and very similar. In the case of Fallot’s tetralogy,the aortic oxygen saturation will be much lower than the oxygen saturation in the left ventricle because theright ventricle pumps most of the deoxygenated blood into the overriding aorta.

• A pulmonary artery pressure exceeding 35 mmHg is suggestive of pulmonary hypertension.

• A pressure drop of more than 10 mmHg across the aortic or pulmonary valve is suggestive of aortic orpulmonary stenosis, respectively.

• The PCWP is equal to the LVEDP. When the PCWP exceeds the LVEDP, the diagnosis of mitral stenosisshould be considered.

• The diagnosis of mitral regurgitation cannot be made unless you are given the PCWP ‘v-wave’. A v-wavehigher than 20 mmHg is highly suggestive of mitral regurgitation.

• Aortic regurgitation is diagnosed by a wide pulse pressure in the aortic pressure.

• The right and LVEDP and the left and right atrial pressures are roughly equal in pericardial constriction.

Guidelines for the Interpretation of Cardiac Catheter Data

66

Page 420: Rapid review of clinical medicine for mrcp part 2

Data Interpretations Tutorials 419

Respiratory Function Tests

Obstructive lung defect

Examples:• Chronic bronchitis.• Emphysema.• Asthma.• Bronchiectasis.

Restrictive lung defect

Examples:• Interstitial lung disease:

sarcoidosis,fibrotic lung disease

• Pulmonary emboli.• Pulmonary oedema.• Neuromuscular disease affecting respiratory muscles.• Pulmonary haemorrhage.• Lymphangitis carcinomatosa.• Thoracic cage defects.

Spirometry

FEV1 (l) ↓FVC (l) ↓↓FEV1/FVC >80%TLC (l) ↓RV ↓TLCO (l) ↓KCO** (mmol/min/kPa) ↓

** KCO is charcteristically reduced in most cases ofrestrictive defect, with the exception of pulmonaryhaemorrhage, where it is increased, and thoraciccage deformities/neuromuscular defects andpneumonectomy where it is unchanged or increased.

Spirometry

FEV1 (l) ↓↓FVC (l) ↓FEV1/FVC <70% (normal

70–80%)TLC (l) ↑ (gas trapping)RV ↑TLCO (l) ↓KCO (mmol/min/kPa) ↓*

*The KCO is always low in obstructive airwaysdisease, with the very rare exception of a limitednumber of patients with asthma who may have ahigh KCO during an attack. The exact mechanismfor this is not understood. The KCO is significantlyreduced in emphysema, and if a young patient or anon-smoker has a low KCO in the context of anobstructive lung defect, always consider thepossibility of α-1 antitrypsin deficiency.

KCO

Increased KCO:• Polycythaemia.• Left-to-right shunts.• Pulmonary haemorrhage.• Asthma (exceptionally rare).• Thoracic cage deformities (may be normal).

Reduced KCO:• Interstitial lung disease.• Primary pulmonary hypertension.• Multiple pulmonary emboli.• Pulmonary oedema.• Lymphangitis carcinomatosa.• Arteriovenous malformation.• Anaemia (spirometry normal).• Obstructive airways disease (rare exception is asthma).

Page 421: Rapid review of clinical medicine for mrcp part 2

The normal flow loop curve has a triangular expiratoryflow limb and a semi-circular inspiratory flow limb (7).The explanation for this is as follows: following a full in -spiration, the lung’s recoil is maximal, as is pleural pres -sure. The onset of expiratory flow is therefore explosiveand reaches its peak within 0.01 s. The expiratory curvedecreases its flow gradually as the lung volume dropsfrom TLC to RV. This reflects the gradual drop in thelung’s elastic recoil as the lung gets smaller. In contrastto expiration, inspiration does not reach an instantmaximal flow. As the respiratory muscles contract, thepower increases progressively from the start of theinspiration to achieve a maximal flow. This takes arelatively long time, and the maximal inspiratory flow isonly achieved by the mid-point of the vital capacity. Theflow then slows again as the maximum inspired volume isreached, giving the inspiratory limb a semi-circularappearance.

Flow loops tested in the examination are those ofobstructive airways disease, extrathoracic airways obstruc -tion, intrathoracic airways obstruction, and a restrictivelung defect.

Obstructive airways disease In obstructive airways disease the amount of elastic recoilin the lung is reduced; therefore the onset of the expira -tory limb is not as explosive as in a normal individual andreaches its peak later than usual. As expiration continuesthe small airways collapse rapidly to produce a very earlydecline in flow which results in a ‘flat’ expiratory flowloop curve after the initial peak (8).

Extrathoracic airways disease obstructionExtrathoracic airways may be variable or fixed. Inextrathoracic obstruction, the inspiratory loop is reducedsignificantly compared with the expiratory flow. Normally,during inspiration, the negative intrathoracic pressurepulls the intrathoracic airways open and promotes inspira -tory flow. In extrathoracic obstruction this does nothappen; therefore the inspiratory flow rate is reduced. Invariable extrathoracic obstruction the expiratory flow limbis preserved (9). Examples of variable obstruction includetumour in the upper airways, fat, pharyngeal muscleweakness, vocal cord paralysis or enlarged lymph nodes. Infixed extrathoracic obstruction, such as tracheal stenosis,both expiratory flow and inspiratory flow limbs are‘blunted’; however, the inspira tory limb is affected muchmore than the expiratory limb (10).

Intrathoracic airways obstructionWhen obstruction is intrathoracic, inspiration is preservedbecause the large extrathoracic airways are patent; however,the expiratory flow is diminished because an in creasedintrathoracic pressure is required to overcome the obstruc -tion, which results in closure of the small airways (11).

Restrictive lung diseaseThe flow loop curve has the same shape as the normalcurve (7), but the lung volume is much smaller.

420

0 1 2 3 4 5Change in lung volume (l)

8

6

4

2

0

2

4

6

Flow (l/sec)

0 1 2 3 4 5 6 7 8Change in lung volume (l)

10

8

6

4

2

0

2

4

6

8

10

Flow (l/sec)

Expiration

Inspiration

Expiration

Inspiration

0 2 4 6Change in lung volume (l)

10

8

6

4

2

0

2

4

6

8

10

Flow (l/sec)

Expiration

Inspiration

Interpretation of Respiratory Flow Loop Curves

77

88 99

Page 422: Rapid review of clinical medicine for mrcp part 2

Data Interpretations Tutorials 421

0 1 2 3Change in lung volume (l)

4

2

0

2

4

Flow (l/sec)

Expiration

Inspiration

0 2 4 6 8Change in lung volume (l)

10

8

6

4

2

0

2

4

6

8

10

Flow (l/sec)

Expiration

Inspiration

Echocardiography

Echocardiography has featured, and will continue tofeature, in postgraduate examinations because it is widelyrequested, and pictures from ‘still frames’ are relativelyeasy to interpret. Both M-mode and two-dimensional(2-D) echocardiography are tested, although the latter isbeing shown with an increasing frequency because itallows the reader to have a better appreciation of cardiacchambers.

Normal values for cardiac dimensions are shown in thetable.

2-D echocardiographyThe view which is shown most often is the parasternallong-axis view (12). In this view it is not always easy tostudy the right ventricle in detail, and therefore it isunusual to be shown right-sided pathology in the

examination. Four-chamber views resemble what thereader envisages the heart to look like, except that theventricles are at the top and the atria are at the bottom(13). Most echocardiograms are accompanied by a scalewhere each square represents 1 cm.

Normal values for cardiac dimensions

Chamber/structure Measurement

Left atrium <4.5 cmLeft ventricular diastolic size <5.5 cmInterventricular septum <1.2 cmPosterior wall <1.2 cm

Key:1 Interventricular septum2 Left ventricle

3 Mitral valve4 Left atrium5 Left ventricular posterior wall

6 Aorta7 Right ventricle8 Right atrium

1

2

3

4

5

67

8

2

4

1100 1111

1122 1133

Page 423: Rapid review of clinical medicine for mrcp part 2

Figures 14 and 15 are diagrammatic representationsof a common parasternal long axis and an apical four-chamber view respectively.

Common examples of conditions shown in examinationsusing 2-D echocardiography:• Pericardial effusion (16, effusion is arrowed).• Left atrial myxoma (17, myxoma is arrowed).• Mitral stenosis (18, stenosed mitral valve is arrowed).• Vegetation on mitral valve (19, vegetation on the

anterior mitral valve leaflet is arrowed).

422

Right ventricularoutflow tract

Left ventricle

Mitral valve

Left ventricular posteror wall

Pericardium

Leftatrium

Aorta

Aortic valve

Rightventricle

Leftventricle

Interventricularseptum

Tricuspidvalve

Mitral valve

Leftatrium

Rightatrium

Interatrialseptum

Interventricularseptum

1144 1155

1166 1177

1188 1199

Page 424: Rapid review of clinical medicine for mrcp part 2

Data Interpretations Tutorials 423

• HCM (20, gross hypertrophy of the interventricularseptum is arrowed).

• Left ventricular thrombus (21 thrombus at the apex isarrowed; 22 thrombus is arrowed).

• Aortic stenosis (23, calcified aortic valve arrowed).• Vegetation on aortic valve (24, echogenic mass on the

aortic valve is arrowed).• Aortic dissection (25, aortic dissection flap is arrowed).

2200 2211

2222 2233

2244 2255

Page 425: Rapid review of clinical medicine for mrcp part 2

Figure 26 is a diagrammatic representation of com -mon examples shown on parasternal long-axis view.Figure 27 is a diagrammatic representation of commonexamples shown in the four-chamber view.

M-mode echocardiographyThe M-mode picture is derived by taking a cross-sectionalview from the 2-D parasternal long-axis view. The threemain views include sections at the level of the aortic valve,at the level of the tips of the mitral valve, and at the levelof the papillary muscle. The cross-sectional view at thelevel of the mitral valve tips is by far the most commonview which is shown in examinations because it allows theexaminers to test the readers on abnormalities on themitral valve and the left ventricle (28).

Examples commonly shown in M-mode echocardiography:• Pericardial effusion.• Left atrial myxoma (29, echogenic mass prolapsing

through the mitral valve orifice in diastole is arrowed).• Mitral stenosis (30, thickened anterior mitral valve

leaflet is arrowed; reduced excursion of the mitralvalve in diastole is shown [1]).

• Vegetation on mitral valve (31, echogenic mass on theposterior mitral valve leaflet is arrowed).

• Mitral valve prolapse (32, hammocking of the posteriormitral valve leaflet in systole is arrowed).

• HCM (33, systolic anterior motion of the mitral valveis shown [1]).

424

Hypertrophiccardiomyopathy Vegetation on

aortic valve

Calcific aorticstenosis

Atrialmyxoma

Mitral stenosis

Vegetationon mitralvalve

Pericardial effusionLeft ventricularthrombus

Left ventricularthrombus

Vegetationon mitralvalve

Atrialmyxoma

Hypertrophiccardiomyopathy(asymmetricseptalhypertrophy)

Mitral stenosis

Pericardium

Left ventricularposterior wallPosterior mitral valve leaflet

Anterior mitral valve leaflet

Right ventricular cavity

Chest wall Right ventricular wall

Leftventricle

Interventricular septum

1

22772266

2288 2299

33113300

Page 426: Rapid review of clinical medicine for mrcp part 2

Data Interpretations Tutorials 425

Importance of carefully inspecting mitral valveleaflets to diagnose cardiac abnormalities on the M-mode echocardiogramOn an M-mode echocardiogram, the normal mitral valvehas an anterior leaflet which has a characteristic ‘M’-shape and a posterior leaflet which has a semi ‘U’-shape.In diastole, the anterior leaflet moves forward and theposterior leaflet moves backward, and in systole both

leaflets appose. With the exception of pericardial effusion,careful inspection of the mitral valve will enable thereader to diagnose all the abnormalities which are com -monly tested (Table).

Figure 34 is a diagrammatic representation of themitral valve in relation to cardiac lesions on the M-modeechocardiogram.

The mitral valve in relation to cardiac lesions on the M-mode echocardiogram

Abnormality Mitral valve abnormalityMitral valve prolapse Normal appearance in diastole; however, in mid systole there is prolapse of

one or both leaflets

Mitral stenosis There is loss of the M-shape of the anterior leafletMitral valve excursion is reduced and there is anterior movement of the posterior leaflet in diastole

Atrial myxoma There is obliteration of the mitral valve cavity by an echogenic mass in diastole

Vegetation on mitral valve Echogenic mass on one or both leaflets

HCM The mitral valve moves forward towards the septum in systole (a phenomenon termed systolic anterior motion of the mitral valve (SAM). In addition, the leftventricular cavity is small and there is septal hypertrophy

Aortic regurgitation As the regurgitant jet flows back into the left ventricle it ‘tickles’ the anterior leaflet, causing a fluttering appearance

AMVL

PMVL

Normal systolicappearance

Echo on AMVL

Vegetation onmitral valve

AMVL: Anterior mitral valve leafletPMVL: Posterior mitral valve leafletMV: Mitral valve

Mitral valve prolapseHammocking of the leaflets in mid systole

Vegetationon PMVL

Mitral stenosis

Loss of M-shapeReduced excursionForward movement of PMVL

MyxomaEchogenic mass inmitral valve orifcie

Hypertrophiccardiomyopathy

Systolic anteriormotion of MV

Flutteringof AMVL

Aorticregurgitation

1

Normaldiastolicappearance

3322 3333

3344

Page 427: Rapid review of clinical medicine for mrcp part 2

Acid–base Disturbance

AcidosisThe main source of acid (H+ ions) in the body is tissuerespiration. Carbon dioxide produced by cellular respira -tion is converted to carbonic acid, which dissociates togenerate H+ (acid) and HCO3

– ions (buffering base). Theretention of CO2, increase in H+ or reduction in HCO3

may result in an acidosis.

CO2 + H2O ↔ H2CO3 ↔ H+ + HCO3–

Three types of acidosis are recognized:• Respiratory acidosis.• Metabolic acidosis with a high anion gap.• Metabolic acidosis with a normal anion gap.

Respiratory acidosis is characterized by a fall in arterialpH (<7.35) due to CO2 retention. Metabolic acidosismay result from the retention of fixed or organic acidscausing a reduction in the bicarbonate level (which is themain buffering agent for H+ in the blood) without anychange in the chloride situation. In these situations, it istermed metabolic acidosis with a high anion gap. Alterna -tively, metabolic acidosis may occur as a conse quence ofbicarbonate loss from the gastrointestinal tract or thekidneys. In these situations, chloride is retained, resultingin a hyper chloraemic acidosis or a meta bolic acidosis witha normal anion gap. The anion gap is calculated by sub -tracting the sum of the sodium and potassium concen -trations from the sum of the chloride and bicarbonateconcentrations. The normal anion gap is between10–18 mmol/l. The hallmark of metabolic acidosis ofeither type is a low arterial pH and a low bicarbonatelevel.

Regulation of arterial pH is controlled by the kidneysand the lungs. Acidosis can be compensated for either byremoving CO2 from the body (lungs) or by retainingbicarbonate ions (kidneys). Respiratory acidosis iscompensated for by the kidneys, which retain bicarbonateions. In compensated respiratory acidosis the pH isnormal, or almost normal, and the bicarbonate level ishigh. In compensated metabolic acidosis the arterial pHis normal, or almost normal, and the pCO2 is low.Respiratory acidosis is compensated by the respiratorysystem through a centrally mediated mechanism whichresults in hyperventilation and a consequent reduction inthe pCO2. Causes of respiratory and metabolic acidosisare given (Table).

AlkalosisAlkalosis may be respiratory or metabolic in origin. Inrespiratory alkalosis, there is a high pH due to hyper -ventilation, causing a low pCO2. Metabolic alkalosis isusually due to increased loss of H+ from the kidney orgastrointestinal tract, or to increased ingestion of alkaline

agents. It is characterized by a high bicarbonate and ahigh pH. Respiratory alkalosis is rare and is usually acute.Chronic cases, usually due to chronic hyperventilation,are compensated for by increasing bicarbonate excretionby the kidneys. Metabolic alkalosis – which is much morecommon than respiratory alkalosis – is compensated forby respiratory hypoventilation, which results in anincrease in pCO2 and hence an increase in H+.

426

Causes of respiratory and metabolic acidosis

Respiratory acidosis• Hypoventilation from any cause, e.g. obesity,

thoracic cage deformities and neuromuscular disorders

• Obstructive airways disease• Acute asthma

Metabolic acidosis (high anion gap)• Diabetic ketoacidosis• Uraemic acidosis• Salicylate poisoning• Lactic acidosis:

shockliver failuremetformin therapyglucose-6-phosphate dehydrogenase deficiencyleukaemia

• Methylene poisoning• Ethylene glycol poisoning

Metabolic acidosis (normal anion gap)• Severe diarrhoea• Pancreatic fistula• Ureterosigmoidostomy• RTA• Acetazolamide therapy

Causes of respiratory and metabolic alkalosis

Respiratory alkalosis:• Hyperventilation• Hysteria• Encephalitis• Brainstem lesions• Aspirin toxicity

Metabolic alkalosis:• Vomiting• Diuretics• Antacids• Hypokalaemic states (these increase renal loss of

H+ by the distal convoluted tubule)

Page 428: Rapid review of clinical medicine for mrcp part 2

Normal Ranges 427

Normal Ranges

SI units Traditional unitsHaematologyESR 0–10 mm in 1st h 0–10 mm in 1st hHb male 130–180 g/l 13–18 g/dl

female 115–150 g/l 11.5–15 g/dlMCH 27–32 pg –MCHC 32–36 g/dl 32–36%MCV 76–95 fl 76–95 x 106/mm3

PCV (haematocrit) male 0.40–0.54 l/l 40–54%female 0.35–0.47 l/l 35–47%

Platelet count 150–400 x 109/l 150–400 x 103/mm3

RBCs male 4.5–6.5 x 1012/l 4.5–6.5 x 106/mm3

female 3.9–5.6 x 1012/l 3.9–5.6 x 106/mm3

Reticulocyte count 0.2–2% 0.2–2%WCC total 4–11 x 109/l 4–11 x 103/mm3

neutrophils 2–7.5 x 109/l 2–7.5 x 103/mm3

lymphocytes 1.5–4.0 x 109/l 1.5–4.0 x 103/mm3

monocytes 0.2–0.8 x 109/l 0.2–0.8 x 103/mm3

eosinophils 0.04–0.4 x 109/l 0.04–0.4 x 103/mm3

basophils <0.1 x 109/l <0.1 x 103/mm3

ClottingAPTT 30–46 s 30–46 sBT 2–8 min 2–8 minFibrinogen 2–4 g/l 0.2–0.4 g/dlPT 12–14 s 12–14 sTT 15–19 s 15–19 s

EndocrinologyCortisol 09.00 h 170–700 nmol/l 6.1–25.2 �g/dl

24.00 h <140 nmol/l 5.0 �g/dlGH <10 �g/l <10 ng/mlOestradiol mid-cycle 500–1,100 pmol/l 500–1,100 pmol/lProlactin <360 mu/l <360 �u/mlRenin lying 2–4 �g/l/h 2–4 ng/ml/h

standing 5–14 �g/l/h 5–14 ng/ml/hTestosterone male 10–35 nmol/l 3–10 ng/ml

female 0.9–3.1 nmol/l 0.3–1.0 ng/mlThyroxine 68–174 nmol/l 4.5–13.6 �g/dlTSH 0.4–3.6 mu/l 0.4–3.6 �u/ml

Biochemistryα-fetoprotein <10 ku/l <10 u/mlALT 5–30 iu/l 5–30 mu/mlAlbumin 34–48 g/l 3.4–4.8 g/dlAlkaline phosphatase 25–100 iu/l 25–100 mu/mlAST 10–40 iu/l 10–40 mu/mlB12 (serum) 160–900 ng/l 160–900 pg/mlBicarbonate 22–30 mmol/l 22–30 mEq/lBilirubin 2–17 �mol/l 0.12–1.0 mg/dl

Page 429: Rapid review of clinical medicine for mrcp part 2

428

SI units Traditional unitsBiochemistry (continued)Calcium 2.2–2.7 mmol/l 8.8–10.8 mg/dlCholesterol 3.6–7.8 mmol/l 139–302 mg/dlChloride 100–106 mmol/l 100–106 mEq/lC-reactive protein 0–10 mg/l 0–1.0 mg/dlCreatinine 50–120 �mol/l 0.57–1.36 mg/dlCreatinine kinase males 17–148 iu/l 17–148 mu/ml

females 10–79 iu/l 10–79 mu/mlFerritin 15–250 �g/l 1.5–25.0 �g/dlFolate serum 3–20 �g/l 3–20 ng/ml

red cell 160–460 �g/l 160–460 ng/mlGamma GT 5–30 u/l 5–30 u/lGlucose (blood) 2.5–7.5 mmol/l 45–135 mg/dlIron (serum) males 14–32 �mol/l 78–180 �g/dl

females 10–30 �mol/l 56–168 �g/dlOsmolality 280–296 mOsm/kg 280–296 mOsm/kgPhosphate 0.8–1.5 mmol/l 2.5–4.65 mg/dlPotassium 3.5–5.0 mmol/l 3.5–5.0 mEq/lProtein 62–80 g/l 6.2–8.0 g/dlSodium 135–146 mmol/l 135–146 mEq/lTIBC 40–80 �mol/l 224–448 �g/dlUrate 0.12–0.42 2.0–7.0 mg/dlUrea 2.5–6.7 mmol/l 15.0–40.2 mg/dl (BUN)

ImmunologyIgA 0.8–4.0 g/l 80–400 mg/dlIgG 7.0–18.0 g/l 700–1,800 mg/dlIgM 0.4–2.5 g/l 40–250 mg/dl

Arterial Blood GasesPaCO2 4.7–6.0 kPa 35–45 mmHgPaO2 11.2–14.0 kPa 84–105 mmHgpH 7.35–7.45 7.35–7.45

CardiologymmHg

Venae cavae mean 2–8 Right atrium a-wave 3–6

v-wave 1–4mean 1–5

Right ventricle systolic 20–30end diastolic 2–7

Pulmonary artery systolic 16–30diastolic 4–13mean 9–18

PCWP 4.5–12Left atrium a-wave 4–14

v-wave 6–16mean 6–11

Left ventricle systolic 90–140end diastolic 6–12

Aorta systolic 90–140diastolic 70–90mean 70–110

Page 430: Rapid review of clinical medicine for mrcp part 2

Index 429

References are to case numbers.

acanthosis nigricans 173ACE inhibitors

contraindications/side-effects 10,132

coronary artery disease 11, 237diabetic nephropathy 92heart failure 10, 296hypertension 25, 103, 123, 198myocardial infarction 121

achalasia 231acidosis

combined respiratory/metabolic 34lactic 283metabolic 48, 82, 329renal tubular 326respiratory 333

acromegaly 261acute tubular necrosis (ATN) 60, 289Addison’s disease 218, 273, 372, 373adrenal hyperplasia, congenital 238,

334adrenocorticotrophic hormone

(ACTH) ectopic 379AIDS-defining conditions 262, 388alcohol-related disease 81, 90, 206,

224, 312, 371, 400alkalosis

hypokalaemic 220, 275metabolic 181respiratory 214, 329

alkaptonuria 71α-1 antitrypsin deficiency 72Alport’s syndrome 85alveolitis

cryptogenic fibrosing (CFA) 217,343

extrinsic allergic 45, 61, 272amaurosis fugax 211amenorrhoea 238, 334aminophylline 20amiodarone 22, 68

toxicity 36, 54amylase, serum 201, 400amyloidosis 124, 346, 389amyotrophy, neuralgic 274anaemia

auto-immune haemolytic 204, 299hypothyroidism 199iron deficiency 126, 313, 338leucoerythroblastic 336microangiopathic haemolytic

(MAHA) 152pernicious 102, 199, 376renal failure 126rheumatoid arthritis 320sickle cell 69, 263, 308sulphasalazine-induced 175

anaesthesia 186

ANCA 109, 141, 304aneurysms, intracranial 228angina 237angio-oedema 155, 374angiotensin II receptor blockers 10,

125, 296, 301ankylosing spondylitis 111, 177antiarrhythmic drugs 22, 36, 54, 68,

184, 222anticoagulation

atrial fibrillation 184, 377mechanical heart valves 309pregnancy 122raised INR 297

anticonvulsant drugs 190, 224, 312anti-GBM disease 171, 210, 304antiphospholipid syndrome 49, 70antithrombotic drugs 94, 103, 121,

237aortic dissection 160aortic regurgitation 335aortic stenosis 207, 350arterio-venous malformation,

pulmonary 349asbestosis 61aspergilloma 355aspergillosis, allergic

bronchopulmonary 21aspirin 11, 94, 103, 121, 307

toxicity 172, 329, 339asthma

management 91occupational 298

ataxia 105, 390atherosclerosis 75atrial enlargement 259atrial fibrillation (AF)

ECG trace 80, 138following MI 223management 22, 94, 184, 307,

377atrial flutter 138atrial myxoma 40, 178atrial septal defects 270, 331atrioventricular (AV) block 95, 220,

232, 235, 342atrophy, multi-system 395autonomic failure, causes 395autosomal dominant inheritance 361autosomal recessive inheritance 151

back pain 71, 88, 330, 379Barrter’s syndrome 181, 275Behçet’s syndrome 31benzodiazepines 224, 312Berger’s (IgA) nephritis 171, 185�2-agonists 91beta-blockers 25, 123, 132, 184,

237, 296�-thalassaemia trait 163

biliary colic 191blood grouping 86bone disease

metastatic 18, 97, 187myeloma 38, 124

bone marrow infiltration, malignant38, 233, 336

Bornholm’s disease 264bradycardia 54breast carcinoma 97bronchial carcinoma 37, 58, 61, 99,

359, 379brucellosis 212, 221Brugada’s syndrome 353bundle branch block 52

left 74, 113, 362right 113, 331

C1 esterase inhibitor 155calcitonin levels 39calcium channel blockers 25, 123,

237, 301carbamazepine 190carbimazole 351carbon monoxide poisoning 205, 378carcinoid syndrome 367cardiac catheter data 78, 144, 167,

207, 270, 335, 391cardiomyopathy

arrhythmogenic right ventricular(ARVC) 362

dilated 100, 296hypertrophic (HCM) 203, 390peri-partum 193restrictive 387

cardioversion 202carotid artery disease 112carpal tunnel syndrome 247cerebral calcification 30cerebral emboli, multiple 112cervical myelopathy 399cervical ribs 2chlamydial infections 41cholangiocarcinoma 33cholangiogram, percutaneous trans-

hepatic 148cholangitis

primary sclerosing 6, 33, 64, 104recurrent 33

cholesterol atheroemboli 303chondrocalcinosis 74choriocarcinoma 106, 107, 248chronic obstructive pulmonary

disease (COPD) 43, 370Churg–Strauss syndrome 4, 109,

303, 396ciclosporin toxicity 251cirrhosis 306, 312, 371

primary biliary (PBC) 33, 188,393

Index

Page 431: Rapid review of clinical medicine for mrcp part 2

Clinistix test 71clotting disorders 265, 385cocaine abuse 332coeliac disease 3, 276, 293Cogan’s syndrome 128colitis, ulcerative 6, 104common peroneal nerve palsy 108commotio cordis 266, 353complement deficiency 234Conn’s syndrome 220, 275copper metabolism 6coproporphyria, hereditary 257, 310corneal calcification 168coronary angiography 121coronary artery disease 1, 11, 121,

179, 197, 237corticosteroid excess 379Coxsackie virus 264creatinine kinase, raised 199Crohn’s disease 19, 31, 104,

249–50, 394cryoglobulinaemia 15, 109CSF abnormalities 267, 268, 322Cushing’s syndrome 173cyclophosphamide 8cystic fibrosis 47, 252, 356, 384cytomegalovirus infection 294

dantrolene 186dapsone 316deafness, sensorineuronal 145delirium tremens 224dementia, Lewy body 365dermatitis herpetiformis 316dextrocardia 66diabetes insipidus 230, 401diabetes mellitus 261, 293

HONK 23, 84ketoacidosis 82management 125, 242nephropathy 89, 92, 254, 328

digoxin 223, 296toxicity 68, 226

diuretics 296abuse 181thiazide 25, 123, 125, 132, 181,

301drug reactions, allergic 396dural sinus thrombosis, septic 128dystonia 57

ECG, in metabolic abnormalities 220echocardiography, transoesophageal

(TOE) 62, 291, 324E. coli 0157:H7 152Ecstasy abuse 172Eisenmenger’s phenomenon 167, 246emphysema 222empyema 56encephalitis, herpes simplex 280encephalomyelitis, viral 267endocarditis

infective 32, 44prosthetic valve 62, 63

enteritis, radiation 162eosinophilia 4, 96, 396

tropical pulmonary 21epilepsy 20, 28, 87, 147, 190erythema chronicum migrans 149erythema multiforme 154erythema nodosum 401erythropoietin 126, 381

resistance 344ESR, causes of very high 174ethylene glycol toxicity 48, 90

facial palsy 149Factor V Leiden mutation 295Fallot’s tetralogy 144fat embolism 113, 189Felty’s syndrome 77, 96, 320femoral head, avascular necrosis 357ferritin, serum 5, 6, 81, 102, 163,

206, 344fibrosis, endomyocardial (EMF) 387folate deficiency 376fracture, pathological 38, 124Friedreich’s ataxia 105, 390

gallstones 148, 191ileus 347

genetic testing 39, 47glomerulonephritis

acute 210minimal change 59, 137post strep 278

glycoprotein IIb/IIIa inhibitors 121gonadotrophin levels 397gout, acute 364growth hormone, ectopic secretion

261Guillain–Barré syndrome 268

haematemesis 133haematoma, subdural 290haematuria 29, 278haem biosynthesis 81, 147haemochromatosis 5, 74, 206haemoglobinuria, paroxysmal

nocturnal (PNH) 102, 156haemolysis, dapsone-induced 316haemolytic uraemic syndrome

(HUS) 87, 152haemophilia 265, 269, 385haemoptysis 61, 171, 210, 355haemorrhagic telangiectasia,

hereditary (HHT) 349haloperidol 30Hamman’s sign 157Hashimoto’s thyroiditis 101head injury 9heart block, complete 229heart failure 10, 124

cardiac pacing 100chronic management 296drugs 305simultaneous liver disease 74simultaneous renal failure 53

Heinz bodies 175, 316Helicobacter pylori infection 338, 369HELLP syndrome 348Henoch–Schönlein purpura 24heparin 115, 380heparin-induced thrombocytopenia

(HIT) type II 114, 115hepatic encephalopathy 169hepatic vein thrombosis 104hepatitis

chronic active 143, 169, 319hepatitis A 206

hiatus hernia 300histiocytosis X 35HIV infection 93, 165, 322, 388

acute 16cerebral disorders 277, 345complications 262

HIV seroconversion 110, 325Hodgkin’s lymphoma 96, 233homocystinuria 271hormone replacement therapy 134Howell–Jolly bodies 175, 276human chorionic gonadotrophin

(HCG) 106, 107, 248hydatidiform mole 248hydrogen cyanide poisoning 37821-hydroxylase deficiency 238, 334hyperaldosteronism, primary 220,

275hypercalcaemia 37, 39, 76, 220,

358, 359causes 168familial benign hypocalcuric 179management of symptomatic 50

hypercholesterolaemia, secondary199

hyperkalaemia 82, 136, 220hypernephroma 29hyperosmolar non-ketotic diabetic

coma (HONK) 23, 84hyperparathyroidism 39, 168hyperprolactinaemia 76hyperpyrexia 172hypertension

in black patients 301diabetes 125endocrine causes 220essential 285hypokalaemia 275malignant 198management 25, 103, 123polycystic kidney disease 135pregnancy 130–2retinopathy 381

hyperthermia, malignant 186hyperthyroidism 248hypocalcaemia 153, 159, 220hypogammaglobulinaemia 86hypogonadism, primary 146, 260,

397hypokalaemia 181, 220, 379

alkalosis 275periodic paralysis 215

430

Page 432: Rapid review of clinical medicine for mrcp part 2

hyponatraemia 9, 172, 230, 386hypopituitarism 372, 373hypotension 323

postural 218hypothermia 140hypothyroidism 199, 281, 376

immotile ciliary syndrome 252immunosuppression 44, 77, 251intervertebral disc prolapse 330intracranial hypertension, benign

239–41iron deficiency 126, 162, 211, 313

Kartagener’s syndrome 252Kawasaki’s disease 324ketoacidosis

diabetic 82euglycaemic 90

Kleinfelter’s syndrome 146

lead poisoning 213, 257Legionnaire’s disease 171, 225, 386leptospirosis 221leucoencephalopathy, progressive

multifocal (PMLE) 345leukaemia

acute myeloid 216chronic lymphatic 204, 299chronic myeloid 120, 392protomyelocytic 258

leukaemoid reaction 142Liddle’s syndrome 275Listeria meningoencephalitis 267lithium toxicity 28, 116, 284liver abscess, amoebic 383liver disease

methotrexate-induced 77simultaneous heart disease 74

liver function tests 6, 182Lofgren’s syndrome 401long QT syndrome 266, 353, 368lung defect

obstructive/restrictive 35restrictive 111, 217

lung fibrosisapical 177causes 35, 36, 217, 343cyclophosphamide-induced 8

lung lesionscalcified nodules 363cavitating 355

lung lobe collapse 227, 302lupus anticoagulant 49, 70Lyme disease 149, 267lymphocytosis 267

macrocytosis 165, 376malabsorption syndromes 162, 276,

394malaria 158Mallory–Weiss tear 133Marfan’s syndrome 79MCV, raised 102, 161, 163, 271, 375

meningitisbacterial 83viral 51, 267

meningococcal septicaemia 26, 234mesenteric ischaemia 75mesothelioma 61metabolic syndrome 182methaemoglobinaemia 316methanol toxicity 48, 90methotrexate 77microalbuminuria 89, 92Mikulicz syndrome 401mitochondrial gene mutations 315mitral valve prolapse 40mitral valve stenosis 80, 178monoclonal gammopathy of

undetermined significance(MGUS) 219

mononucleosis, infectious 16motor neurone disease 139, 200MRI scans, interpretation 18multiple endocrine neoplasia 1

(MEN 1) 76multiple endocrine neoplasia 2

(MEN 2) 39multiple sclerosis 105, 255–6myasthenia gravis 65, 200, 253myelofibrosis 120, 392myeloma, multiple 38, 117, 124,

346, 358myocardial infarction

aortic dissection 160atrial fibrillation 223cardiogenic shock 287coronary reperfusion 1non-ST elevation 121, 199, 243normal coronary arteries 332posterior 323ST-segment elevation 197, 323thrombolysis 180, 197

myopathysteroid-induced 379thyrotoxic 330

myotonic dystrophy 192

narcolepsy 67Neisseria meningitidis 26nephritis

acute interstitial 118, 119, 317IgA (Berger’s nephritis) 171, 185,

278nephropathy

diabetic 89, 92, 254, 328hyperuricaemic 216penicillamine-induced 17

nephrotic syndrome 59, 137, 389neuralgia, migrainous 292neurofibromatosis 145neuroleptic agents 30, 57neuroleptic malignant syndrome 209neutropenia, severe septic 351nifedipine 237NSAIDs 118, 339

renal manifestations 317

nutritional deficiency 161

obesity 182obstructive sleep apnoea (OSA) 279oesophageal varices 306, 371oesophageal web 313opiate overdose 366optic neuropathy, Leber’s hereditary

288Osler–Weber–Rendu syndrome 349osteomalacia 188, 219, 321, 393osteoporosis 134

pacemakers 95, 100, 232Pancoast tumour 99pancreatitis, acute 201, 400pancytopenia 102, 165panhypopituitarism 372, 373papilloedema 239–41, 268, 282paracetamol overdose 7, 27, 206paradoxical embolism 98, 331paralysis, hypokalaemic periodic 215parapneumonic effusions 56parathyroid hormone (PTH) 159,

168Parkinson’s disease 365, 395parotid gland enlargement 401paroxysmal nocturnal

haemoglobinuria (PNH) 102,156

patent ductus arteriosus 78, 246patent foramen ovale 98, 112penicillamine-induced membranous

nephropathy 17pericardial constriction 387, 391pericardial effusion 166periostitis 311phaeochromocytoma 39pituitary function tests, dynamic 397plasma osmolality 28, 90, 230plasmapheresis 65pleural effusion 58, 183pneumonia

aspiration 225chicken pox (healed) 363community acquired (CAP) 55,

56Mycoplasma 154

pneumonitis, cyclophosphamide-induced 8

pneumothorax 14, 79, 157polyangitis, microscopic 109polyarteritis nodosa (PAN) 109, 174polycystic kidney disease 135, 228,

382polycythaemia, secondary 73polycythaemia rubra vera 12, 73, 211polydipsia 35, 230, 401polyendocrine deficiency syndromes

218polymyalgia rheumatica (PMR) 174,

340polymyositis 200, 340polyserositis 264

Index 431

Page 433: Rapid review of clinical medicine for mrcp part 2

polyuria 35, 230, 401porphyria

acute intermittent 147variegate 310

porphyria cutanea tarda (PCT) 81pre-eclampsia 130–2pregnancy 248

anticoagulation 122cardiomyopathy 193cystic fibrosis 356epilepsy 87, 190HELLP syndrome 348hypertension 130–2

proteinuria 17, 59, 185pseudohypoparathyroidism 46, 159pseudopolycythaemia 73pseuodotumour cerebri 239–41psittacosis 41psoriasis 116psychiatric ward, patient collapse

209puberty onset 260pulmonary emboli 113, 156, 196,

341, 380multiple 327

pulmonary haemorrhage 171, 210pulmonary oedema 53, 244–5, 291pulmonary stenosis 13

radiation enteritis 162Raynaud’s phenomenon 15, 236Reiter’s syndrome 31renal artery stenosis 53, 130–2,

244–5renal carcinoma 29renal failure

with abdominal pain 382acute 60, 289, 317Alport’s syndrome 85anaemia 126chronic 126, 171, 344, 381and eosinophilia 4hyperuricaemic nephropathy 216myeloma 38, 117

renal stones 249–50renal transplantation 44, 251, 354,

357renal tubular acidosis (RTA) 326retinitis pigmentosa 360retinopathy, hypertensive 381rhabdomyolysis 153rheumatoid arthritis 17, 77, 96, 196,

222, 320rickets 321

rifampicin 118, 119Rinne’s test 141Roth spots 32

sacroiliitis 177salicylate poisoning 214, 329sarcoidosis 401Schilling test 394scleroderma 198, 236sensorimotor neuropathy, hereditary

(HSMN) 314sex hormone levels 397Shy–Drager syndrome 395sick euthyroid syndrome 9sickle cell anaemia 69, 263, 308sick sinus syndrome 318small bowel meal 162smoke inhalation 378spherocytosis, hereditary 191, 352spinal cord compression 18splenomegaly 12, 73, 96, 211statins 103, 121, 237steatohepatitis, non-alcoholic

(NASH) 182steatorrhoea 75Steele–Richardson–Olszewski

syndrome 395Still’s disease, adult-onset 375streptokinase 180stroke 112, 307, 309

management of first presentation103

young patients 98, 349sudden death 40, 203, 266, 332,

353, 368sulphasalazine 175sweat sodium test 384syndrome of inappropriate ADH

secretion 9, 129, 147, 154, 230syphilis 325systemic lupus erythematosus (SLE)

42, 105, 264

tachycardiaatrioventricular 208ventricular 28, 54, 150

Takayasu’s arteritis 402teratogens 190testicular tumours 106, 107tetanus 176theophylline toxicity 20thromboembolism

prevention in AF 94, 184, 307, 377risk factors 341

thrombolysis 1, 180, 197, 380thrombosis 70, 114, 115, 122thrombotic thrombocytopenic

purpura (TTP) 87thyroid carcinoma 39, 101, 164thyrotoxicosis 215, 330TLCO, raised 171, 210toxocariasis 337toxoplasmosis, cerebral 93transient ischaemic attacks (TIAs)

103, 112tricyclic antidepressant (TCAD)

overdose 28, 209, 286, 366troponins, cardiac 243tuberculosis

genitourinary 118, 119primary (pulmonary) 166spinal 88therapy toxicity 118, 119

tuberous sclerosis 360tumour lysis syndrome 216Turner’s syndrome 385, 397

ulcers, peptic 338, 369urate levels 216

ventricular arrhythmias 28, 54, 150,202, 286, 362

ventricular hypertrophy 235, 259ventricular septal defect 291vitamin B12 deficiency 394vitamin C deficiency 161vitamin D deficiency 188, 321, 393von Willebrand factor-cleaving

protease (ADAMTS13) 87von Willebrand’s disease 264, 385

warfarin 94, 122water deprivation test 401Weber’s test 141Wegener’s granulomatosis 109, 141,

171Wolff–Parkinson–White syndrome

208

X-linked inheritance 170, 194, 195,269

zidovudine (AZT) 165Zollinger–Ellison syndrome 75

432